Você está na página 1de 805

Um Curso de Cálculo e Equações

Diferenciais com Aplicações 1

Luı́s Gustavo Doninelli Mendes 23

1
Continuarei acrescentando material, além de corrigir possı́veis erros ou imperfeições. Por isso
sugiro que o improvável leitor não imprima o texto. Quando for estudá-lo dê uma olhada no
meu site se já há uma versão mais atualizada. Sugestões ou correções, por favor as envie para
mendes.lg@gmail.com
2
Professor Adjunto do Departamento de Matemática da UFRGS
3
Última atualização: 09/05/2012
Índice

Parte 1. Cálculo Diferencial e Integral e primeiras Aplicações 13


Capı́tulo 1. Introdução 15
1. O que é o Cálculo 15
2. Sobre o Curso 16
3. Sobre os Gráficos e Figuras 16
4. Alerta aos estudantes 16
5. Livros-texto e Referências 17
6. Programas úteis 18
Capı́tulo 2. Alguns dos objetivos do Cálculo 21
1. Funções e seus domı́nios 21
2. Função 23
3. Funções definidas a partir de outras funções 23
4. Diferentes domı́nios de funções 24
5. Gráfico descontı́nuo, mas que mesmo assim é gráfico 25
6. Função positiva, negativa e zeros ou raı́zes 25
7. Função crescente ou decrescente 26
8. Máximos e mı́nimos 28
9. Exercı́cios 29
Capı́tulo 3. Propriedade básicas dos números Reais 31
1. Os Reais como sistema de números: não dividirás por zero ! 31
2. Ordem nos Reais: não tirarás a raı́z quadrada de números negativos ! 32
3. Propriedades gerais das desigualdades 33
4. Intervalos e suas utilidades 36
5. Metamorfoses de cúbicas 39
6. Exercı́cios 46
Capı́tulo 4. Sequências e seus limites 47
1. Sequências 47
2. Limites de sequências 48
3. Definição e Propriedades fundamentais 49
4. Exercı́cios 53
Capı́tulo 5. Limites de funções definidas em intervalos 57
1. Operações elementares com limites de funções 58
2. A definição usual com ǫ e δ 59
3. Limites quando x tende ao infinito 61
3
4 ÍNDICE

4. Quando a parte é do mesmo tamanho do todo 66


5. Exercı́cios 68

Capı́tulo 6. A noção de Continuidade 71


1. Operações com funções contı́nuas 72
2. Polinômios, funções racionais e trigonométricas 74
3. Continuidade da função inversa 78
4. Dois teoremas fundamentais sobre funções contı́nuas 79
5. Primeiras aplicações do T.V.I 79
6. Raı́zes de polinômios cujo grau é ı́mpar 79
7. Raı́zes simples e fatoração de polinômios 81
8. Possı́veis raı́zes Racionais de polinômios a coeficientes inteiros 83
9. Exercı́cios 84

Capı́tulo 7. Geometria Analı́tica Plana 87


1. Equações de retas, coeficientes angular e linear 87
2. Ortogonalidade 89
3. Teorema de Tales no cı́rculo 90
4. A equação da reta de Euler 91
5. A inversa como reflexão de gráfico na diagonal 99
6. O método de Descartes para as tangentes a um gráfico 100
7. Um problema da Putnam Competition, n. 2, 1939 104
8. Exercı́cios 104

Capı́tulo 8. A Tangente ao gráfico, segundo o Cálculo 107


1. Retas secantes a um gráfico 107
2. A reta tangente a um gráfico 107
3. A reta tangente ao seno em (0, 0) é a diagonal 109
4. Interpretação Fı́sica da reta tangente 113
5. Exercı́cios 113

Capı́tulo 9. A derivada 115


1. Definição, primeiras propriedades e exemplos simples 115
2. Um Árbitro que só avalia as inclinações 117
3. Derivadas da soma e da diferença 119
4. Problema da Putnam Competition, n. 68, 1993 120
5. A segunda derivada 123
6. Exercı́cios 124

Capı́tulo 10. Sinal da derivada e crescimento 127


1. Teoremas de Rolle, Lagrange e Cauchy 127
2. O Teorema 0 das Equações Diferenciais 131
3. Critérios de crescimento e de decrescimento 133
4. Uma confusão frequente sobre o significado do sinal da derivada 134
5. Descontinuidade da função derivada 135
6. Exercı́cios 136
ÍNDICE 5

Capı́tulo 11. Aplicações da primeira e segunda derivadas 139


1. Primeiro critério de máximos e mı́nimos 139
2. Critério da segunda derivada 139
3. Um problema tı́pico para os engenheiros 140
4. Mı́nimos de distâncias e ortogonalidade 142
5. Concavidades dos gráficos 146
6. Mı́nimos quadrados e a média aritmética 149
7. Pontos de inflexões dos gráficos 151
8. Critério da derivada de ordem n 152
9. Confecção de gráficos de polinômios 154
10. Exercı́cios 155
Capı́tulo 12. Derivadas de seno e cosseno e as leis de Hooke 161
1. O cosseno como derivada do seno 161
2. Leis de Hooke com e sem atrito 163
3. Exercı́cios 166
Capı́tulo 13. Derivada do produto, indução e a derivada de xn , n ∈ Z. 167
1. Princı́pio de indução matemática 167
2. Derivada do Produto 169
3. Derivadas de x−n , ∀n ∈ N 170
4. Raı́zes múltiplas e fatoração de polinômios 171
5. A Regra de Sinais de Descartes para as raı́zes de um polinômio 173
6. Exercı́cios 177
Capı́tulo 14. Derivada da composição de funções 179
1. Regra da composta ou da cadeia 179
2. A derivada do quociente 183
3. Uma função que tende a zero oscilando 185
4. Confecção de gráficos de funções racionais 186
5. Involuções fracionais lineares 189
6. Um problema da Putnam Competition, n. 1, 1938 190
7. Uma função com derivada, mas sem a segunda derivada 192
8. Máximos e mı́nimos: o problema do freteiro 193
9. Exercı́cios 205
Capı́tulo 15. Derivadas de funções Implı́citas 207
1. Curvas versus gráficos 207
2. Teorema da função implı́cita 209
3. Reta tangente de curva e plano tangente de superfı́cie 212
4. Tangentes, pontos racionais de cúbicas e códigos secretos 213
5. Derivação implı́cita de segunda ordem 218
6. Exercı́cios 220
Capı́tulo 16. Funções inversas
√ e suas derivadas 221
1. Derivada de y = x 222
2. Distância versus quadrado da distância 223
6 ÍNDICE
1 m −m
3. Derivada da “função”x n , de x n e de x n 223
4. Derivadas do arcoseno e do arcocosseno 225
5. Derivada do arcotangente 228
6. Exercı́cios 231
Capı́tulo 17. Taxas relacionadas 235
1. Como varia um ângulo 235
2. Como varia uma distância 236
3. Lei dos cossenos e produto escalar de vetores 238
4. Exercı́cios 241
Capı́tulo 18. O Método de aproximação de Newton 243
Capı́tulo 19. O Princı́pio de Fermat e a refração da luz 247
1. Princı́pio de Fermat 247
2. Refração, distâncias ponderadas e Lei de Snell 249
3. Exercı́cios 253
Capı́tulo 20. As Cônicas e suas propriedades refletivas 255
1. Distância até uma parábola 255
2. Definição unificada das cônicas 257
3. A Parábola e sua propriedade refletiva 265
4. Prova analı́tica da propriedade do foco 269
5. A Elipse e sua propriedade refletiva 271
6. A Hipérbole e o análogo da propriedade refletiva 275
7. Famı́lia de cônicas co-focais ortogonais 281
8. Exercı́cios 284
Capı́tulo 21. Integração e o Primeiro Teorema Fundamental 285
1. Área sob um gráfico positivo 285
2. Qual função descreve as Áreas sob gráficos? 286
3. Primeira Versão do Primeiro Teorema fundamental do Cálculo 289
4. A Integral e suas propriedades 291
5. Teorema do valor médio de integrais 294
6. A integral indefinida e o Primeiro Teorema fundamental 295
7. Existem funções com primeira derivada, mas sem segunda derivada 297
8. Exercı́cios 298
Capı́tulo 22. Logaritmo natural e sua inversa, a exponencial 301
1. Existe uma função f 6≡ 0 que seja imune à derivação ? 301
2. Propriedades fundamentais do logaritmo e da exponencial 304
3. loga x , ∀a > 0 e ln | x | 306
4. As funções ex e ax , para a > 0 308
5. xa e sua derivada, a ∈ R. 309
6. Crescimento lento do logaritmo e rápido da exponencial 310
7. Uma observação sobre o termo geral de uma série infinita 313
8. Um problema da Putnam Competiton, n. 11, 1951 314
ÍNDICE 7

9. A regra de L’Hôpital 315


10. A função xx 319
11. Um problema da Putnam Competition, n. 22, 1961 321
12. Um modo de aproximar e por números Racionais 322
13. Funções f (x)g(x) em geral e suas indeterminações 323
14. Derivada logarı́tmica 324
15. Uma função extremamente achatada 326
16. Exercı́cios 329

Capı́tulo 23. Segundo Teorema Fundamental e Áreas 335


1. A descoberta de Gregory e Sarasa sobre área 335
2. Segundo Teorema Fundamental do Cálculo 336
3. Regiões entre dois gráficos 337
4. Um problema da Putnam Competition, n. 54, 1993. 340
5. Integral e centro de gravidade 343
6. Arquimedes e a parábola: prova versus heurı́stica 345
7. Exercı́cios 348

Capı́tulo 24. Integração por partes 353


1. Exercı́cios 356

Capı́tulo 25. Integração por substituição 359


1. A substituição trigonométrica x = sin(θ) 362
2. RÁreas
√ do Cı́rculo e Elipse 363
3. r 2 − x2 dx 365
4. Mais exemplos da substituição x = sin(θ) 365
5. Substituição trigonométrica x = tan(θ) 367
6. RMais
√ exemplos da substituição x = tan(θ) 367
7. r 2 + x2 dx 369
8. Substituição trigonométrica x = sec(θ) 369
9. MaisR √exemplos para a substituição x = sec(θ). 370
10. x2 − r 2 dxR 371
11. E as da forma √Ax3 +Bx12 +Cx+D dx ? 371
12. Exercı́cios 371

Capı́tulo
R 26.2 Integração de funções racionais 373
−1
1. R (ax + bx + c) dx 373
αx+β
2. dx 375
R ax2 +bx+c1
3. Ax3 +Bx2 +Cx+D
dx 377
4. Frações
R parciais em geral 380
1
5. (1+x2 )n
dx, n ≥ 2 383
6. Exemplos 384
7. Exercı́cios 387

Capı́tulo 27. Integrais impróprias 389


1. Um problema da Putnam Competition, n. 2, 1939 391
8 ÍNDICE

2. As primeiras Transformadas de Laplace, a função Gama e o fatorial 392


3. Fórmula de Euler para o fatorial 396
4. Exercı́cios 396
Capı́tulo 28. A curvatura dos gráficos 397
1. O comprimento de um gráfico 397
2. Um problema da Putnam Competition, n.2, 1939 399
3. Curvas parametrizadas e seu vetor velocidade 399
4. Integrais que ninguém pode integrar 401
5. Velocidade de um gráfico ou de uma curva 402
6. Definição de curvatura e sua fórmula 403
7. Qual a curvatura de uma quina ? 405
Capı́tulo 29. Séries convergentes 409
1. Séries k-harmônicas, k > 1. 409
2. A série geométrica 411
3. O teste da razão (quociente) 412
4. Um argumento geométrico para a série geométrica 414
Capı́tulo 30. Aproximação de Números e Funções importantes 415
1. Aproximações de raı́zes quadradas por números racionais 415
2. Raı́zes quadradas que são irracionais 415
3. Como tirar raı́z quadrada só com +, −, ×, / 416
4. Os Reais através de sequências de números Racionais 418
5. Aproximações de e por números Racionais 419
6. Arcotangente e cartografia 421
7. A aproximação de π dada por Leibniz 423
8. Aproximações de logaritmos 425
9. Aproximação de logaritmos de números quaisquer 426
10. Aproximação de ln(2) 428
11. Exercı́cios 428
Capı́tulo 31. Séries numéricas e de funções 429
1. Séries numéricas 429
2. Séries de potências 431
3. Séries de Taylor e os Restos de Lagrange, Cauchy e Integral 434
4. A série binomial e sua série de Taylor 439
5. Um devaneio sobre os números Complexos 442
6. Exercı́cios 443
Capı́tulo 32. O discriminante de polinômios de grau 3 445
1. Preparação para a fórmula de Cardano 445
2. A fórmula de Cardano para as três raı́zes Reais: viagem nos Complexos 449
3. O discriminante como curva 452
4. A curva discriminante entre as cúbicas singulares 454
5. Parametrização dos pontos racionais de cúbicas singulares 458
6. Cúbicas singulares aparecem como seções com o plano tangente 459
ÍNDICE 9

Capı́tulo 33. Discriminante dos polinômios de grau 4 463


1. A andorinha: o discriminante como superfı́cie 463
2. Discriminante como envelope de famı́lias de retas ou planos 465

Capı́tulo 34. Apêndice: O expoente 43 comanda a vida ! 467


1. Metabolismo versus massa corporal 467
2. Escalas log/log para um experimento 468
3. Reta de ajuste - método de mı́nimos quadrados 468
4. A Lei experimental de Kleiber 470
5. Justificação racional da Lei de Kleiber 471
6. O argumento 472

Parte 2. Equações diferenciais ordinárias e Aplicações 479

Capı́tulo 35. As primeiras equações diferenciais 481


1. A exponencial e as equações diferenciais 481
2. A definição original de Napier para o logaritmo 482
3. Decaimento radioativo e datação 484
4. Equações diferenciais lineares com coeficientes constantes 486
5. Objetos em queda-livre vertical 489
6. Queda ao longo de um gráfico 493
7. A curva que minimiza o tempo 496
8. Balı́stica e o Super Mário 500
9. Equações diferenciais lineares em geral 504
10. Um problema da Putnam Competition, n.14, 1954 504
11. Soluções das equações lineares gerais 506
12. Um problema da Putnam Competition, n. 49, 1958. 510
13. As equações de Bernoulli e sua redução a equações lineares 511
14. Exercı́cios 512

Capı́tulo 36. Aspectos gerais das equações de primeira ordem 515


1. Equações diferenciais e metamorfoses de curvas 515
2. Equações diferenciais em forma normal e as curvas Isóclinas 517
3. Existência e unicidade para y ′(x) = F (x, y) - Método de Picard 520
4. Equações separáveis 525
5. A clepsidra 527
6. Equações homogêneas 528
7. Equações exatas 530
8. Integral ao longo de um caminho 534
9. Derivada da integral em relação ao parâmetro - Fórmulas de Leibniz 536
10. Fatores integrantes 539
11. Equações implı́citas, discriminantes e envelopes 542
12. Um problema da Putnam Competition, n. 5, 1942 548
13. Equações de Clairaut e de Lagrange: isóclinas retas 550
14. Transformação de Legendre, dualidade e resolução de equações diferenciais 553
15. Apêndice: Funções contı́nuas de duas variáveis e continuidade uniforme 556
10 ÍNDICE

16. Exercı́cios 558


Capı́tulo 37. Curvas de Perseguição 559
1. O problema 559
2. As elipses isócronas, segundo A. Lotka 566
3. Um envelope que é uma curva de perseguição 568
4. Exercı́cios 570
Capı́tulo 38. Cinética quı́mica e crescimento bacteriano 571
1. Cinética quı́mica 571
2. Equação diferencial de uma reação de primeira ordem 573
3. Equação diferencial de uma reação de segunda ordem 574
4. Crescimento bacteriano 576
5. Ponto de inflexão da função logı́stica 580
6. Equação de Bernoulli e reações quı́micas de ordem fracionária 581
Capı́tulo 39. Newton e a gravitação 583
1. Atração segundo o inverso do quadrado da distância 583
2. Tempo de colisão e velocidade de escape 584
3. Nı́veis de energia 587
4. Órbitas planetárias 589
5. Velocidade e aceleração expressas em coordenadas polares 589
6. Grandezas constantes ao longo das trajetórias 592
7. As órbitas como cônicas em coordenadas polares 597
8. Oscilador harmônico 599
9. Área em coordenadas polares e a lei de Kepler sobre as áreas 601
10. Em torno da proposição XXX do Principia 602
11. A Equação de Kepler para o movimento planetário elı́ptico 606
Capı́tulo 40. Equações diferenciais de segunda ordem 609
1. Redução de ordem 609
2. Homogêneas, a coeficientes constantes 610
3. Não-Homogêneas, lineares de segunda ordem 614
4. Não homogênas: Método de Lagrange de variação de parâmetros 616
5. Um problema da Putnam Competition, n.58, 1987 617
6. Equação diferencial de um circuito elétrico simples 619
7. Não-homogêneas: Método de coeficientes a determinar 620
8. Sistemas de equações diferenciais 624
9. Um problema da Putnam Competition, n.2, 1939 626
10. Homogêneas, não-singulares, coeficientes variáveis: redução a constantes 627
11. Homogêneas, não-singulares, coeficientes variáveis: Método de D’Alembert 629
12. Existência de soluções de equações homogêneas e não-singulares 630
13. Propriedades das soluções de equações lineares de segunda ordem 632
14. Um problema da Putnam Competition, n. 15, 1955 635
15. O Teorema de Comparação de Sturm 638
16. Um problema da Putnam Competition, n. 22, 1961 639
17. Exercı́cios 641
ÍNDICE 11

Capı́tulo 41. Equações com pontos não-singulares: Airy, Hermite e Legendre 643
1. Solução explı́cita da Airy 643
2. Solução explı́cita da Hermite 645
3. Solução explı́cita da Legendre em torno de x = 0 647
4. Polinômios de Legendre e expansão em série do potencial gravitacional 649
5. Ortogonalidade dos polinômios de Legendre 650
Capı́tulo 42. Equação com ponto singular: Hipergeométrica de Gauss 653
1. Integral elı́ptica como série hipergeométrica 656
Capı́tulo 43. Equação com ponto singular: a Equação de Bessel 659
1. A definição original de Bessel 659
2. Zeros de funções de Bessel 661
3. Ortogonalidade das funções de Bessel 664
Capı́tulo 44. Equações com pontos singulares do tipo regular 667
1. A Equação de Euler e sua redução a coeficientes constantes 667
2. Solução direta da equação de Euler 670
3. Definições gerais e exemplos de pontos singulares regulares 672
4. Inı́cio do Método de Frobenius 673
5. Soluções explı́citas de algumas equações Bessel 676
6. A Equação de Bessel com ν = 13 e a solução da equação de Airy 679
7. Equação hipergeométrica com c 6∈ Z 680
Capı́tulo 45. Equações de Riccati 681
1. Soluções de Riccati segundo Daniel Bernoulli 682
2. Assı́ntotas verticais de soluções de equações de Riccati 687
3. Soluções das Riccati segundo Euler 688
4. A Equação de Bessel com ν = 41 e a solução da Riccati y ′ = x2 + y 2 691
5. Exercı́cios 691

Parte 3. Séries de Fourier e Equações diferenciais parciais 693

Capı́tulo 46. Séries de Fourier 695


1. Séries de Fourier e seus coeficientes 696
2. Séries de Fourier só de senos ou só de cossenos 699
3. Convergência pontual da Série de Fourier 699
4. Séries de Fourier de cos(r · sin(x)) e de sin(r · sin(x)), r ∈ R 706
5. Convergência absoluta da Série de Fourier 707
6. A solução da equação de Kepler via série de Fourier e funções de Bessel 710
7. Exercı́cios 713
Capı́tulo 47. Equações Diferenciais Parciais 715
1. Observações gerais, tipos, separação de variáveis, soluções clássicas 715
2. Equações parciais de primeira ordem e o método das caracterı́sticas 717
3. A Equação da difusão do Calor 717
4. Problemas de esfriamento unidimensionais 720
12 ÍNDICE

Capı́tulo 48. O operador de Laplace e as equações do calor e da onda 725


1. Laplaciano em coordenadas polares e esféricas 725
2. Estado estacionário do calor num disco e expansão em séries de Fourier 727
3. A fórmula integral de Poisson 729
4. Estado estacionário do calor na esfera e série de polinômios de Legendre 731
5. Exercı́cios 736
Capı́tulo 49. Equação da onda e as vibrações de cordas e membranas 737
1. Vibração de uma corda com extremos fixos, sem atrito 737
2. Vibração de uma corda infinita: Fórmula de D’Alembert 739
3. Modos normais de vibração de um tambor circular e as funções de Bessel 741

Parte 4. Cálculo diferencial e integral sobre os números Complexos 747


Capı́tulo 50. Um portal para o Cálculo Complexo 749
1. O Teorema de Green e as Relações de Cauchy-Riemann 759
2. A integral complexa e a idéia da primitiva Complexa 761
3. Curvas integrais como parte imaginária das primitivas Complexas 764
4. A exponencial Complexa e os ramos do logaritmo Complexo 766
5. O Teorema fundamental do Cálculo sobre os Complexos 768
6. Exercı́cios 769
Capı́tulo 51. Os Teoremas Fundamentais 771
1. A primitiva Complexa 771
Capı́tulo 52. Soluções detalhadas de alguns Exercı́cios 773
Parte 1

Cálculo Diferencial e Integral e primeiras


Aplicações
CAPı́TULO 1

Introdução

1. O que é o Cálculo
O Cálculo Diferencial e Integral ou, simplesmente o Cálculo, é a matemática que
está na base da ciência de hoje.
As ciências mais desenvolvidas como Fı́sica e Quı́mica não podem expressar seus
conceitos sem fazerem uso do Cálculo. Também a Economia e a Biologia cada vez
mais são matematizadas através do Cálculo.
O Cálculo foi fundamental na revolução cientı́fica dos séculos XVII e XVIII e de
lá para cá não cessou de produzir resultados e aplicações.
O Cálculo é uma teoria matemática, ou seja, um modo unificado de se ver uma
série de fatos matemáticos.
Na matemática, quando surge uma nova teoria, ao invés de se eliminar os resul-
tados das teorias anteriores, o que a nova teoria faz é:

• reobter os teoremas até então conhecidos,


• dar generalizações deles,
• produzir resultados completamente novos.

Isso só ocorre em matemática: em outras ciências uma nova teoria pode tornar
obsoleta e errada a teoria anterior.
Por exemplo, a determinação exata da Área de certas regiões, que com métodos
elementares exigiu o gênio de Arquimedes, com o Cálculo vira uma continha de rotina.
Mas através do Cálculo aparecem fatos novos e intrigantes sobre Áreas, como o fato
de regiões ilimitadas poderem ter Área finita.
Além de nos permitir provar tudo que já ouvimos falar de matemática no colégio,
o Cálculo vai nos transformar em verdadeiros McGivers, ou seja, aquele personagem
que com quase nada de recursos faz horrores de coisas, como aparelhos, armas, etc, e
suas missões. Através do Cálculo , só com as quatro operações +, −, x vamos poder
no Capı́tulo 30 aproximar com a precisão que quisermos:

• funções fundamentais como arctan(x), ln(x), etc



• números como p (p primo), π, e = exp(1).

Uma das inspirações fundamentais para o Cálculo foi a Fı́sica, ou Fı́sica-matemática


com a qual Isaac Newton revolucionou a ciência da época. Vários fenômenos fı́sicos
tiveram então uma explicação completa e unificada, através das técnicas do Cálculo.
Essas técnicas só ficarão aparentes à medida que o leitor entre na Segunda Parte
do Curso, que é a parte de Equações Diferenciais.
15
4. ALERTA AOS ESTUDANTES 16

2. Sobre o Curso
Um alerta: este curso trata de matemática superior. Em várias universidades,
inclusive a nossa, há uma a tentativa de se ensinar o Cálculo como se fosse uma
continuação do Ensino Médio, seu ensino sendo feito através de tabelas, regrinhas,
macetes.
Se refletimos um pouco, vemos que em alguns cursos como Farmácia, Economia,
Biologia, o Cálculo é uma das poucas disciplinas de matemática que terão na univer-
sidade. Desse modo, imitando o Ensino Médio, se cursaria um Curso Superior sem
ter contato com a Matemática Superior. A formação cientı́fica desses cursos ficaria
prejudicada e de fato não poderiam chamar-se cursos universitários.
Por isso neste Curso sempre que for possı́vel (exceto quando a explicação for
técnica demais) vamos tentar dar justificações matemáticas corretas, sem apelar para
a credulidade do estudante e argumentos de autoridade, do tipo acreditem em mim.
Os argumentos que damos são concatenações de idéias simples, mas às vezes ex-
igem um certo fôlego do leitor para acompanhá-lo do começo ao fim. Esse treino de
concentração certamente irá colaborar na formação técnico-cientı́fica do estudante.

3. Sobre os Gráficos e Figuras


Tentei fazer o máximo possı́vel de gráficos para ilustrar o conteúdo, usando o pro-
grama Maple 9 para fazê-lo numericamente, ou seja, realisticamente. Este programa é
pago, mas o estudante pode usar o XMaxima ou o Gnuplot que são programas livres,
do Linux, como auxiliar no estudo. Sempre que possı́vel usei a mesma escala nos dois
eixos, pois isso determina inclinações das retas e essas inclinações são importantes no
Cálculo1.
Mas nem sempre isso foi possı́vel, por exemplo quando as funções crescem muito
rápido, onde não dá para manter as mesmas escalas nos eixos x e y.
A teoria tem que ser sempre nossa guia na confecção de gráficos, pois os computa-
dores erram ao representar funções descontı́nuas ou funções que estão muito próximas
de um certo valor sem alcançar esse valor.
Também fiz figuras qualitativas e diagramas usando o programa Winfig, que é
pago, e o Xfig, do Linux, que é grátis.

4. Alerta aos estudantes


Por ser matemática superior, o Curso exige do aluno um empenho e atenção muito
diferente daquele exigido nos seus contatos anteriores com a matemática.
Principalmente o aluno deve usar de modo preciso os conceitos que vão sendo
apresentados (por ex. limites, continuidade, derivada). Se não os entender, per-
gunte ao professor até ter esclarecido o conceito. Pois embora às vezes pareçam ape-
nas conceitos qualitativos, são de fato bastante precisos e mais tarde dão resultados
quantitativos de absoluta precisão.
1Veja, por exemplo, que o gráfico do seno está errado em várias edições do livro do Anton,
pois ele não usou as mesmas escalas nos eixos x e y, portanto a inclinação na origem não fica bem
representada
CAPÍTULO 1. INTRODUÇÃO 17

Numa primeira leitura, o estudante pode ler o enunciado dos Teoremas e Afirmações,
sem ler todas as demonstrações. Mas de fato, só se entende completamente um fato
matemático quando se entende a sua demonstração.
Por último, é muito importante que o estudante pense nos exercı́cios propostos em
cada Capı́tulo. Mesmo que não responda todos, ao tentar fazer exercı́cios o conteúdo
vai sendo assimilado concretamente. E se o aluno não consegue fazer quase que
nenhum exercı́cio, então precisa voltar a refletir no conteúdo dado.
Alguns têm solução bastante detalhada, apresentada no Capı́tulo 52. Mas que só
devem ser lidas após muito trabalho pessoal do aluno.
Ao longo do livro aparecem problemas da prestigiada W. L. Putnam Mathematical
Competition, que ocorre anualmente desde sua Primeira Edição em 1938. Vão apare-
cendo à medida que desenvolvemos material suficiente para poder resolvê-los. Nessa
competição aparecem problemas difı́ceis, mas tratei de selecionar alguns simples e
acessı́veis.
Minhas fontes foram o site:
http://amc.maa.org/a-activities/a7-problems/putnamindex.shtml
(onde estão as Competições de 1985-2009) e o livro The W. L. Putnam Mathemat-
ical Competition, Problems and solutions, 1938-1964., Math. Association of America.
Esses problemas devem ser pensados pelo leitor e só depois do leitor apresentar a
sua resposta, do seu jeito de ver o problema, é que pode ler as respostas. Foi assim
que eu fiz: eu resolvi sozinho cada um dos que apresento, e minhas respostas não têm
a pretensão de serem as mais elegantes possı́veis.
Lembro o que um professor muito bom me disse: Só se aprende matemática re-
solvendo problemas !

5. Livros-texto e Referências
Livros ruins de Cálculo há vários, de cuyos nombres no quiero acordarme.
Bastante razoável o livro do G. Thomas, disponı́vel na biblioteca em várias edições.
Curto, direto e bom preço: R. Silverman, Essential Calculus with applications,
Dover.
Para mim um dos melhores livros de Cálculo é o de Michael Spivak, Calculus
(edições em espanhol e ingles na biblioteca da UFRGS). Aprende-se muito nesse livro
e me foi úil em alguns momentos na hora em que se fez necessário a precisão que falta
em outros livros. Claro que é bastante difı́cil como primeiro livro de Cálculo, mas o
esforço de ler qualquer seção dele é sempre recompensado.
Na Primeira Parte usei coisas que aprendi:
• no enciclopédico livro de R. Courant e F. John, Introduction to Calculus and
Analysis, Interscience, 1965.
• no curso de Elon Lima Curso de Análise, Projeto Euclides, SBM.
• no clássico E. T. Whittaker e G. Watson, A course of modern Analysis,
Cambridge, reimpressão de 1996.
• no belo livro de C.H. Edwards, The historical development of the Calculus,
Springer, 1979.
• no livro de S. Chandrasekhar, Newton’s Principia for the common reader,
Oxford University Press , 1995.
6. PROGRAMAS ÚTEIS 18

As referências usadas no Apêndice sobre a Lei de Kleiber, Capı́tulo 34, estão dadas
lá.

Na Parte 2, sobre Equações diferenciais, usei material do Courant-John, bem como


• o excepcional livro de M. Hirsch e S. Smale Differential equations, dynamical
systems and linear algebra, Academic Press, 1974,
• o muito bem escrito e motivante livro de G. Simmons Differential equations
with applications and historical notes, McGraw-Hill, 1972. Alguns Exercı́cios
propostos neste livro me serviram de guia para diversas Seções. Usei bastante
esse livro.
• o livro de H. S. Bear, Differential Equations, a Concise Course, Dover, 1962
é pequeno mas muito informativo. Nele se encontra uma prova perfeitamente
legı́vel do Teorema de existência de soluções de Picard, por exemplo.
• o de J. W. Bruce e P. j. Giblin, Curves and singularities, Cambrige U. Press,
1984.
• o clássico G. N. Watson A treatise on the theory of Bessel functions , Cam-
brige, 1958.
• o livro de A. Gray e G. B. Mathews, A treatise on Bessel functions and their
applications to Physics, McMillan and co, 1895.
• ademais usei no Capı́tulo 37 artigos de A. Bernhardt e de A. Lotka, bem
como
• o clássico livro de F. Gomes Teixeira, Traité des courbes speciales remar-
quables, planes et gauches, reimpressão de 1971, Chelsea Publishing Com-
pany.
• last but not least, E. Kamke, Differentialgleichungen- Losungsmethoden und
losungen, T. I, Chelsea Publisinhg Company, 1948.

6. Programas úteis
Programas como o Maple podem ser um grande auxiliar para o estudo: para
conferir contas, plotar curvas, etc, mas só serão úteis se o estudante tentar fazer
sozinho e depois usar os programas para checar seus resultados.
Para usuários do Windows existe o programa grátis WXMaxima, que você baixa
em instantes no site:
http://sourceforge.net/projects/maxima/files/Maxima-Windows/
5.21.1-Windows/maxima-5.21.1.exe/download
Esse programa faz tudo: resolve equações algébricas e diferenciais, deriva, integra,
faz gráficos, etc.
O Maple é programa análogo pago.
Também existe um site, http://www.wolframalpha.com, onde se pode fazer online
gráficos, integrais, limites e derivadas, o que é útil quando se está estudando fora de
casa.
Agradecimentos:

Agradeço ao Professor Mark Thompson, da Matemática da UFRGS, por ter


me disponibilizado Notas que serviram para a elaboração da Seção sobre Cinética
CAPÍTULO 1. INTRODUÇÃO 19

quı́mica. E também pelo livro de G. Gibson, An elementary treatise on the Calculus,


with illustrations from Geometry, Mechanics and Physics, reimpressão de 1956 da
edição de 1901, que me foi útil.
Agradeço ao Professor Vı́tor Pereira, da Geologia da UFRGS, que me explicou o
belo fenômeno da meia-vida da luz das super-novas.
As notas de Aula do Professor Eduardo Brietzke, da Matemática da UFRGS, para
a disciplina de Equações Diferenciais II, me serviram de fio-condutor entre os diversos
temas possı́veis. Abordei alguns dos exemplos que lá aparecem de um ponto vista um
pouco diferente. Lhe sou grato.
Agradeço às estudantes que fizeram Cálculo comigo em 2008: Pâmela Lukasewicz
Ferreira, por ter tomado notas do curso que dei e que me serviram de roteiro para
este texto e Mônica Hoeveler, por participações em aula e por sugestões de temas.
Agradeço aos estudantes Luciano Bracht Barros e Magno V. F. Teixeira da
Silva por conversas no fim da aula que me motivaram a escrever a Seção 6 do Capı́tulo
32.
O estudante Walter Ferreira Diniz Júnior resolveu vários problemas de modo
original, produziu exemplos, e até me indicou como escrever melhor a Seção 5 do
Capı́tulo 26 !
CAPı́TULO 2

Alguns dos objetivos do Cálculo

A descrição matemática dos fenômenos se faz principalmente a partir da noção de


função y = f (x) e de seu gráfico.
Se pudermos entender:

• se f (x) assume somente valores Reais, onde f (x) se anula, onde é positiva
ou negativa,
• se e onde f (x) cresce ou decresce à medida que x cresce,
• se f (x) se aproxima de um certo valor quando x cresce muito,
• se e onde f (x) tem valor máximo ou mı́nimo,
• no caso de y = f (x) ≥ 0, qual a área sob seu gráfico e acima do eixo dos x,
• se dado y pudermos descobrir qual x gerou y = f (x),

então podemos dizer que entendemos o comportamento da f (x).


Estaremos capacitados a fazer previsões sobre o fenômeno modelado por essa
função.
Esses são alguns dos objetivos do Cálculo.
Nas próximas Seções passamos lembrar / definir essas noções.

1. Funções e seus domı́nios


Os filósofos sempre se espantaram com o fato de que as coisas mudam, e se ques-
tionaram tanto sobre o que muda como sobre o que permanece nessas mudanças.
Os matemáticos também compartilham desse espanto e sempre se perguntaram,
ao ver que há mudanças, como as coisas mudam.
A resposta a essa pergunta pode ser tanto qualitativa como quantitativa, as duas
são interessantes. Por exemplo é qualitativa quando um astrônomo afirma que certo
cometa voltará a passar algum dia. É quantitativa no caso de Halley, que previu o
ano em que certo cometa voltaria, usando as ferramentas do Cálculo.
Se um fenômeno (a temperatura de um sistema, por exemplo) depende de um só
parâmetro (o tempo, por exemplo) é natural descrever sua evolução num gráfico da
função que associa a cada momento x a temperatura T (x). Esse gráfico formará uma
21
1. FUNÇÕES E SEUS DOMÍNIOS 22

curva no plano.

1
0,8
0,6
0,4
0,2
0
-2 -1 0 1 2
x

Figura: O gráfico de y = T (x) forma uma curva no plano.

Mas é claro que conhecemos fenômenos z = F (x, y) que dependem de dois fatores
e para descrever esse fenômeno precisariamos de gráficos que formam superfı́cies no
espaço, ao invés de curvas no plano. E em geral os fenômenos dependem de vários
parâmetros (em quı́mica, por exemplo, quantidades de reagentes, pressão, ph, etc).

Figura: O gráfico de z = F (x, y) forma uma superfı́cie no espaço

Os conceitos que aprenderemos neste curso se adaptam facilmente para superfı́cies,


mas vamos nos restringir a gráficos que são curvas. Ou como se diz, faremos o Cálculo
de 1 variável.
A seguir vamos começar a estabelecer conceitos qualitativos sobre gráficos que
são importantes no Curso. O manejo correto desses conceitos é fundamental para a
compreensão do resto do curso.
CAPÍTULO 2. ALGUNS DOS OBJETIVOS DO CÁLCULO 23

2. Função
Uma função é uma regra que associa a cada ponto1 de um conjunto (o domı́nio
da função) um ponto de um outro conjunto fixado (o contra-domı́nio). Dito de outro
modo, uma reta vertical traçada passando por um ponto do domı́nio de uma função
y = f (x) corta seu gráfico exatamente em 1 ponto. Por isso, por exemplo, um cı́rculo
não é gráfico de uma função y = f (x).
O subconjunto do contradomı́nio formado por pontos que são efetivamente valores
da função formam a imagem da função. Por exemplo,
f : R → R, f (x) = x2
tem como domı́nio e contradomı́nio os números Reais, mas sua imagem são apenas
os Reais não-negativos2.
Quando dizemos que f : I → J é sobrejetiva isto quer dizer que não somente
a imagem f (I) verifica f (I) ⊂ J, mas que de fato verifica f (I) = J. Ou seja, que
efetivamente todo ponto de J foi atingido pela f . Por exemplo, f (x) = x2 só é
sobrejetiva vista como função f : R → R≥0 .
É importante notar na definição de função que só há um valor associado a cada
ponto do domı́nio. Se houver ambiguidade na atribuição do valor então dizemos que a
função não está bem-definida naquele ponto. Por exemplo, quando perguntamos qual
é a raı́z quadrada de 9 há uma ambiguidade: pode ser que tomemos a raı́z positiva 3
ou a raı́z negativa −3.
Não confunda a definição de função com outra, a de função injetiva: uma função
é injetiva quando não associa o mesmo valor a dois pontos distintos de seu domı́nio.
Por exemplo, f : [0, 3] → R, f (x) = x2 é injetiva mas f : [−3, 3] → R, f (x) = x2 não
é injetiva.

3. Funções definidas a partir de outras funções


3.1. Função inversa. Imagine uma função que desfaz o efeito de outra função.
Por exemplo, uma dá a a velocidade de um carro em função do tempo trascorrido
v = v(t). Sua inversa diria para cada velocidade v qual o tempo necessário para
atingir essa velocidade t = t(v) (o que dá uma medida da potência do motor do carro,
por ex.)
Ou por exemplo, a temperatura de um objeto vai caindo com o tempo. Sabendo
quanto caiu a temperatura T (t) como determinar o tempo t transcorrido ?
Para se ter uma função inversa f −1 , a função f necessariamente tem que ser
injetiva !
Se não, vejamos: se y = f (x1 ) = f (x2 ) com x1 6= x2 , o que deve fazer f −1 com y
? Enviá-lo em x1 = f −1 (y) ou em x2 = f −1 (y) ? Isso é uma ambiguidade inaceitável
para f −1 .
Vamos mais tarde falar do sentido geométrico da função inversa.

1Para mim os números Reais formam um reta, portanto uso número ou ponto indistintamente.
2Várias vezes no curso usaremos isso: o quadrado de um número Real nunca é negativo
4. DIFERENTES DOMÍNIOS DE FUNÇÕES 24

3.2. Composição de funções. Dentre os modos mais úteis de se produzir um


função interessante a partir de funções simples está a composição de funções.
A idéia é simples e fundamental: o resultado de uma função g(x) vira entrada de
uma segunda função f .
A notação usual é: se f : I → J e g : J → K então (f ◦ g) : I → K faz
(f ◦ g)(x) := f ( g(x) ).
É claro que se pode compor um número qualquer de funções.
Pense em quantos exemplos encontramos disso na natureza, nas reações quı́micas,
nas indústrias, em que um processo complicado é dividido em várias etapas simples
concatenadas.
Neste Curso procedermos assim também: vamos primeiro entender os casos mais
simples e depois, via composição de funções, entender os mais complicados.

3.3. O que é a Área sob um gráfico ? Podemos usar o gráfico de uma função
para definir outra. Por exemplo, tomo a diagonal y = x como gráfico e me pergunto
pela Área do triângulo determinado pela origem, o eixo horizontal e um segmento
vertical de (x, 0) até (x, x). À medida que x avança no eixo dos x, a Área do triângulo
obtido aumenta e poderı́amos tentar descrever como essa Área depende de x isso num
outro gráfico.
Na definição do Logaritmo Natural, faremos exatamente isso, mas a área em
questão será delimitada sob o gráfico de 1/x e não sob y = x.

x=1 x
Figura: Área sob um o gráfico, de x = 1 até x.

Precisaremos saber primeiro, o que é a Área sob um gráfico curvado como 1/x.
Isso que foge do que sabemos do Ensino Médio, que são áreas de regiões elementares
como triângulos, quadrados, trapézios, setores circulares, etc. Só entenderemos isso
plenamente na Parte 2 do curso, com o conceito de Integral.

4. Diferentes domı́nios de funções


A princı́pio o domı́nio de uma função pode ser qualquer conjunto, mas neste Curso
usaremos como domı́nios quase sempre:
• todos os Reais R, ou
• intervalos de números reais, incluindo semi-retas ou
• apenas os Naturais N ⊂ R.
CAPÍTULO 2. ALGUNS DOS OBJETIVOS DO CÁLCULO 25

Mas é claro que em certas situações os domı́nios também podem ser a união de
vários intervalos (como se verá por exemplo na Seção 2.3 do Capı́tulo 6), somente os
números Racionais Q ⊂ R, etc.

5. Gráfico descontı́nuo, mas que mesmo assim é gráfico


Há gráficos que sofrem um salto abrupto, mas que mesmo assim são gráficos.
Por exemplo, o gráfico da função f : R → R, definida condicionalmente por
f (x) = x − 2, se x < 2 e f (x) = x2 se x ≥ 2.
O ponto 2 de seu domı́nio é um ponto catastrófico: se estamos em pontos que são um
pouquinho menores que 2 a função tem valores próxima do zero. Mas se mexemos
um pouco a coordenada x, chegando em x = 2 ou acrescentando algo positivo muito
pequeno ao 2, o valor da função já pula para ≥ 22 = 4.

y=4

x=2

Figura: O gráfico de função descontı́nua no ponto x = 2

Outro modo de ver o que acontece é que, enquanto seu domı́nio R é feito de um
só pedaço, sua imagem f (R) = R≤0 ∪ R≥4 é feito de dois pedaços: a função rasga seu
domı́nio em dois pedaços.
Esses gráficos são úteis para modelar matematicamente comportamentos explo-
sivos: uma explosão quı́mica, o comportamento de um animal à medida que aumenta
o stress, etc. Mas em cursos de Cálculo veremos gráficos que não tem essas variações
dramáticas de valores.

6. Função positiva, negativa e zeros ou raı́zes


Uma função f : I → R é positiva (negativa)3 se sua imagem está contida nos
Reais positivos (negativos).
Muito importante para um técnico ou cientista é determinar os pontos do domı́nio
onde a função se anula (ou, como se diz, onde corta o eixo dos x, que é dado por
y = 0). Ou seja, é importante resolver uma equação f (x) = 0.
No caso de polinômios esses pontos são as chamadas raı́zes. Aconselho o leitor a ler
o Teorema 7.1 no Capı́tulo 6, que prova a relação entre raı́zes e fatores de polinômios.
3Para evitar escrever duas frases onde só trocaria uma palavra, ponho em parênteses a modi-
ficação a ser feita na frase
7. FUNÇÃO CRESCENTE OU DECRESCENTE 26

Mais adiante, no Teorema 4.1 do Capı́tulo 6.1 explicaremos em termos do Cálculo


qual o significado das raı́zes múltiplas.

0
-2 -1 0 1 2
x
-2

-4

-6

Figura: Um gráfico de polinômio com 3 raı́zes

7. Função crescente ou decrescente


Definição 7.1. Uma função f : I → R é estritamente crescente exatamente quando

∀ x1 , x2 ∈ I, x1 < x2 ⇒ f (x1 ) < f (x2 ).

E dizemos que é apenas crescente exatamente quando

∀ x1 , x2 ∈ I, x1 < x2 ⇒ f (x1 ) ≤ f (x2 ).

Analogamente se define estritamente decrescente, trocando f (x1 ) < f (x2 ) por


f (x1 ) > f (x2 ).

0,8

0,6

0,4

0,2

0
1 1,5 2 2,5 3
x
CAPÍTULO 2. ALGUNS DOS OBJETIVOS DO CÁLCULO 27

Figura: Exemplo de gráfico de y = f (x) crescente.

1
0,8
0,6
0,4
0,2

0 0,5 1 1,5 2 2,5 3


x

Figura: Exemplo de gráfico de y = f (x) decrescente.

Claro que há funções que não são nem crescentes nem decrescentes, ou sejam, que
oscilam.

0,8

0,6

0,4

0,2

0
-0,6 -0,4 -0,2 0 0,2 0,4 0,6
x

Figura: Exemplo de gráfico de y = f (x) que oscila.

Uma observação simples mas útil:


Se uma função f é estritamente crescente (ou estritamente decrescente) então f
é injetiva.
De fato, se tomo quaisquer x1 , x2 diferentes de seu domı́nio, posso sempre me
perguntar qual deles é menor, por exemplo, x1 < x2 . Como a f é estritamente
crescente (ou estritamente decrescente), temos f (x1 ) < f (x2 ) (ou f (x1 ) > f (x2 )),
mas de qualquer forma f (x1 ) 6= f (x2 ). Logo é injetiva.

Um exemplo importante é o que já demos de uma função f que mede a Área
sob um gráfico de uma outra função positiva. É natural que f seja uma função
estritamente crescente, pois à medida que vamos para a direita no eixo x há mais
área sob o gráfico. Logo é natural que seja injetiva e tenha então uma inversa f −1 .
Volto nesse ponto, com f o Logaritmo Natural e f −1 a Exponencial.
8. MÁXIMOS E MÍNIMOS 28

Saber que uma função é crescente pode ser um fato extremamente relevante do
ponto de vista cientı́fico: por exemplo, um dos princı́pios fı́sicos mais fundamentais
é que a função Entropia é uma função crescente, ou seja, que as coisas têm uma
tendência a se desorganizar. É essa Entropia crecente que está na base da nossa
distinção entre passado, presente e futuro.

Por outro lado um exemplo marcante de função decrescente é a função y = f (x)


que dáa quantidade de uma substância radioativa no tempo x. Uma descoberta
cientı́fica fundamental foi a de descrever de modo quantitativamente preciso como é
essa função para cada substância radioativa.

É fundamental neste curso estabelecermos um critério para determinar se uma


função é crescente (ou é decrescente).
De preferência um critério que consista em entender uma função que seja mais
simples que a função f ela mesma ! Se não não adiantaria muito. Isso veremos no
Capı́tulo 10, que é muito importante.

8. Máximos e mı́nimos
Uma das grandes utilidades do Cálculo é encontrar pontos onde uma função atinge
seu máximo ou mı́nimo. Ou seja, o Cálculo serve para minimar ou maximizar: rendi-
mento de um processo, custos, gastos, etc, desde que o problema seja formulado
matematicamente.
Vamos definir um máximo local (analogamente um mı́nimo local).
Definição 8.1. Seja f : I → R e x ∈ I. Dizemos que x é máximo local se existe
algum intervalo
(−ǫ + x, x + ǫ)
centrado em x, tal que
∀x ∈ I ∩ (−ǫ + x, x + ǫ), f (x) ≤ f (x).
Já x é dito ser um máximo global de f : I → R se
∀x ∈ I, f (x) ≤ f (x).

É a mesma diferença que há entre ser o cara que corre mais rápido no clube do
bairro e ser o cara que corre mais rápido no mundo !

4,2

3,8

3,6

3,4

3,2

-0,6 -0,4 -0,2 0 0,2 0,4 0,6


x
CAPÍTULO 2. ALGUNS DOS OBJETIVOS DO CÁLCULO 29

Figura: Função com um mı́nimo global, um máximo local e um mı́nimo local.

Chamo a atenção de que há funções que simplesmente não tem máximo, como já
vimos no caso de f : (0, 5] → R, f (x) = x1 .
E existem as que não tem mı́nimo: por ex. f : R≥1 → R, f (x) = x1 .
De fato, se tomo n ∈ R≥1 , temos f (n) = n1 , que já sabemos fica tão próximo
quanto quisermos de 0, sem nunca atingir zero. Isso diz que f vai sempre diminuindo
um valor, não tendo portanto um ponto de seu domı́nio onde um valor mı́nimo fosse
atingido.
Dá vontade de dizer algo sobre o papel do 0 neste exemplo f : R≥1 → R, f (x) = x1 .
O 0 realmente nunca é atingido pela função mas de certo modo demarca, delimita o
conjunto imagem
f (R≥1 ) = (0, 1].
0 é o que se costuma chamar uma cota inferior do conjunto imagem f (R≥1 ), isto é,
∀y ∈ f (R≥1 ), 0 ≤ y.
E mais ainda, qualquer número maior que zero não é cota inferior de f (R≥1 ), pois
1
n
∈ f (R≥1 ) se aproxima o que quisermos de zero. Portanto 0 é a maior cota inferior
de f (R≥1 ), que se chama o Ínfimo desse conjunto.

9. Exercı́cios
Exercı́cio 9.1. Determine em que intervalos as funções a seguir são negativas ou
positivas e onde estão seus zeros:
vi) x2 − x
vii) x2 − 5x + 6
viii) x3 − x2
Exercı́cio 9.2. Dê exemplos de frases do dia a dia que são verdade, mas cujas
recı́procas não são verdade.
Exercı́cio 9.3. Negue as seguintes frases:
i) dado qualquer polı́tico, existe um valor de suborno tal que por esse valor ele se
corrompe.
ii) dada uma distância qualquer, existe um tempo tal que a partir daquele tempo
o asteróide dista da terra menos que a distância dada.
Exercı́cio 9.4. Imagine alguns exemplos, qualitativamente, sem precisar dar explici-
tamente a regra f (x), de funções:
i) positivas e crescentes,
ii) negativas e crescentes,
iii) negativas e decrescentes,
iv) negativas e decrescentes,
v) com mı́nimo local, mas sem mı́nimo global
vi) com máximo local e máximo global diferentes.
9. EXERCÍCIOS 30

Exercı́cio 9.5. Faça as composições f ◦ g ◦ h e h ◦ g ◦ f , onde:


i) f = x13 , g = sin(x) h = x + 5
ii) f = x2 , g = x1 , h = sin(x).
iv) Imagine algum exemplo onde aconteça f ◦ g ◦ h = h ◦ g ◦ f (o que é raro !).
Exercı́cio 9.6. (resolvido)
Determine explicitamente as funções inversas f −1 das funções f (x) a seguir. Teste
sua resposta verificando que x = f −1 (f (x)).
i) f : R → R, f (x) = x3
ii) f : R → R, f (x) = x3 + 1
iii) f : R → R, f (x) = (x − 1)3
iv): f : R → R, f (x) = −5 · x3 + 10.
x
v): f : (0, 1) → R, f (x) = 1−x 2 . Dica: o mais difı́cil neste item é não se equivocar

com os sinais.
CAPı́TULO 3

Propriedade básicas dos números Reais

As funções definidas nos Reais e tomando valores Reais são importantes pelas
aplicações ao mundo fı́sico. Por exemplo, se um Engenheiro me diz que a laje da peça
onde estou vai cair em 5 minutos eu certamente saio correndo √ da sala. Mas se um
Matemático me disser que a laje vai cair no tempo 5 · I := 5 −1, que fazer ?
Essa utilidade dos Reais, por corresponder à linha do tempo (passado = número
negativo, presente = 0, futuro = número positvo), tem como ônus o fato que as
funções Reais nem sempre estão definidas.
Veremos duas restrições, uma sobre quocientes e outra sobre a raı́z quadrada.
A primeira afeta não só os Reais, mas qualquer sistema de números. A segunda,
da Raı́z, é tı́pica dos números que podem ser ordenados.
1. Os Reais como sistema de números: não dividirás por zero !

Todo professor passa aulas e aulas repetindo que não se pode dividir por zero.
E infelizmente muitos alunos de Cálculo dividem por zero, pois confundem o fato
de um número ser pequeno com um número ser zero !
Mas a final, por quê não se pode dividir por zero ? No que podemos nos apoiar
para provar que não existe o número 10 ?
Nos bastará algumas das propriedades mais gerais dos R (por sinal compartilhadas
com outros sistemas de númros, como Q ou C), que são:
• existe um elemento neutro aditivo, 0, tal que 0 + x = x, ∀x ∈ R.
• ∀x ∈ R existe o inverso aditivo −x tal que x + (−x) = 0.
• existe um elemento neutro multiplicativo, 1, tal que 1 · x = x, ∀x ∈ R.
• ∀x ∈ R, x 6= 0, existe o inverso multiplicativo x1 tal que x · x1 = 1.
• 1 6= 0
• as operações de soma e produto são distributivas, associativas e comutativas.
De posse dessas propriedades, que são assumidas como verdades, posso provar :
Afirmação 1.1.
i) −x = −1 · x, ∀x ∈ R,

ii) 0 · x = 0, ∀x ∈ R.

iii) não existe 01 .


Demonstração.
De i):
0 = (1 − 1) · x ⇔ x − x = (1 − 1) · x ⇔
31
2. ORDEM NOS REAIS: NÃO TIRARÁS A RAÍZ QUADRADA DE NÚMEROS
NEGATIVOS ! 32

⇔ x − x = 1 · x − 1 · x ⇔ x − x = x − 1 · x ⇔ −x = −1 · x.
De ii):
0·x=0 ⇔ (1 − 1) · x = 0 ⇔
⇔ x−1·x=0 ⇔ x − x = 0,
e este último fato é verdade: x = x.
De iii):
Suponhamos por absurdo que exista o número 01 .
Então 0 · 10 = 1, pois o sentido de x1 é ser o inverso multiplicativo de x.
Mas o item ii) dá que:
1
0 · = 0.
0
Logo 0 = 1: contradição.


2. Ordem nos Reais: não tirarás a raı́z quadrada de números negativos !

Um aspecto bonito da matemática é que, após assumir a verdade de certos fatos


simples, podemos deduzir fatos novos, às vezes não tão simples.
Vamos assumir a validade dos seguinte Princı́pios (Axiomas):
• Princı́pio 0: Existe um subconjunto P dos Reais chamado de conjunto dos
números positivos. Vale para todo x ∈ R apenas uma das 3 possibilidades:
ou x ∈ P ou x = 0 ou −x ∈ P . O elemento neutro multiplicativo 1 é positivo.
• Princı́pio 1: A soma de quaisquer dois números positivos é um número
positivo.
• Princı́pio 2: o produto de um número positivo por um número positivo é
positivo.

Um número é chamado não-negativo se x ∈ P ∪ {0}. Denotamos os positivos


usualmente com x > 0 e os não-negativos com x ≥ 0. Os negativos, por x < 0.

Podemos agora provar :


Afirmação 2.1.
i) (Regra de multiplicação de sinais) (−x) · (−x) = x · x, ∀x ∈ R.
2
ii) x√ := x · x ≥ 0 ∀x ∈ R.
iii) x não é um número Real, se x < 0.
Demonstração.
De i):
De fato, pelo item i) da Afirmação 1.1 (−1) · x = −x.
Pela comutatividade e associatividade do produto:
(−x) · (−x) = (−1) · x · (−1) · x = (−1) · (−1) · x · x.
CAPÍTULO 3. PROPRIEDADE BÁSICAS DOS NÚMEROS REAIS 33

Só resta provar que


−1 · (−1) = 1,
ou seja, nos reduzimos a provar apenas a Regra dos Sinais para o −1. Ora,
−1 · (−1 + 1) = 0 ⇔ −1 · (−1) − 1 · 1 = 0 ⇔
⇔ −1 · (−1) − 1 = 0 ⇔ −1 · (−1) = 1,
como querı́amos.

De ii):
Se x = 0 então x · x = 0, pelo item ii) da Afirmação 1.1.
Se x > 0 então x · x > 0 (Pr. 2).
Se, por outro lado, x < 0 então −x > 0 (Pr. 0).
E então x · x = (−x) · (−x) > 0 (Pr. 3 e 2).

De iii): √
Suponha agora por absurdo que y := x ∈ R para x < 0.
Então y 2 ≥ 0 pelo item ii).
Mas então chegamos em

0 ≤ y 2 = ( x)2 = x < 0,
em contradição com o Princı́pio 0.


3. Propriedades gerais das desigualdades


Usando os Princı́pios 0 , 1, 2 e a Regra de Multiplicação de Sinais podemos provar
as propriedades a seguir, que são fundamentais.
Alerta: se o estudante não manejar bem essas propriedades terá problemas no
Curso.
Afirmação 3.1.
i) Se x ≥ y e z ≥ w então x + z ≥ y + w, ∀x, y, z, w ∈ R.
ii) Se x > 0 e y ≥ z então x · y ≥ x · z.
iii) Se x < 0 e y ≥ z então x · y ≤ x · z.
iv) se x > 0 então x1 > 0
v) se x > 1 então x1 < 1.
vi) 0 < x1 < x2 ⇒ 0 < x12 < x11 .
vii) 0 < x < 1 ⇒ 0 < x2 < x < 1.
viii) 1 < x ⇒ 1 < x < x2
ix) 0 < x1 < x2 < 1 ⇒ 1 < x12 < x11 .
x) 1 < x1 < x2 ⇒ x12 < x11 < 1.
xi): 0 < x < 1 ⇒ 1 < x1 < x12 .
xii): 1 < x ⇒ x12 < x1 < 1.
xiii): 0 ≤ x ≤ y e 0 ≤ z ≤ w então 0 ≤ x · z ≤ y · w.
3. PROPRIEDADES GERAIS DAS DESIGUALDADES 34

Demonstração.
i) Dados x, y, z, w ∈ R com
x≥y e z ≥ w,
podemos traduzir isso em:
(x − y) ≥ 0 e (z − w) ≥ 0.
Queremos provar que
x + z ≥ y + w,
que se traduz em
(x + z) − (y + w) ≥ 0,
ou, o que diz o mesmo:
(x − y) + (z − w) ≥ 0.
Isso é o que queremos. Para termos isso, podemos usar o Princı́pio 1, pois então com
esse princı́pio:
(x − y) ≥ 0 e (z − w) ≥ 0 ⇒ (x − y) + (z − w) ≥ 0.
ii) Temos que x > 0. Caso y = z então x · y = x · z. Por isso supomos que y > z,
ou seja, y − z > 0.
Queremos provar que x · y > x · z, ou seja, que
x · y − x · z > 0,
o que é o mesmo que dizer que
x · (y − z) > 0.
Isso é o que queremos. Então podemos usar o Princı́pio 2, que dá:
x>0 e y−z >0 ⇒ x · (y − z) > 0.
iii) Temos agora −x > 0 pelo Princı́pio 0. Caso y = z então x · y = x · z.
Por isso supomos y > z, ou seja, y − z > 0. Então o Princı́pio 2 dá:
(−x) · (y − z) > 0,
ou seja
−x · y + x · z > 0,
ou seja,
x · y − x · z < 0,
que é o que buscávamos provar:
x · y < x · z.
iv) Temos x > 0 e suponhamos por absurdo que x1 < 0.
Então − x1 > 0 e pelo Princı́pio 2:
1
x · (− ) > 0.
x
1
Mas x · (− x ) = −1. Logo obtemos −1 > 0 ou seja 1 < 0, que contradiz o Princı́pio 0.
v) Seja x > 1. Suponhamos por absurdo que x1 ≥ 1.
Se x1 = 1 então chegamos na contradição: 1 = x.
CAPÍTULO 3. PROPRIEDADE BÁSICAS DOS NÚMEROS REAIS 35
1
Se x
> 1 então multiplicando esta desigualdade por x > 1 > 0, temos
1
x· > x·1
x
(pelo item ii) já provado).
Como x · x1 = 1 pela própria definição de x1 e como x · 1 pela definição do neutro
1, obtemos
1 > x,
que contradiz x > 1.
Deixo para o leitor a prova das propriedades vi-xii, onde pode usar as propriedades
i) - v) que já foram provadas.

Faço a prova de xiii):


Como 0 ≤ x ≤ y e 0 ≤ z ≤ w então sai primeiro que 0 ≤ x · z.
Agora, para ver que x · z ≤ y · w, note que
x · z ≤ y · z,
pois 0 ≤ (y − x) · z.
Do mesmo jeito sai que:
y · z ≤ y · w,
e portanto
x · z ≤ y · w.


Proponho agora ao leitor o seguinte Exercı́cio: explicar com itens da Afirmação


3.1 algumas propriedades dos Gráficos das funções a seguir, a saber:
• por quê em determinado intervalo um está acima ou abaixo do outro,
• por quê isso se inverte ao passar de x = 1,

1,5

0,5

0
0 0,2 0,4 0,6 0,8 1 1,2
x
4. INTERVALOS E SUAS UTILIDADES 36

y = x em vermelho, y = x2 em verde, y = x3 em amarelo


e y = x4 em azul, para x ∈ [0, 1.2]

1,5

0,5

0,8 1 1,2 1,4 1,6 1,8


x

1 1
y= x
em vermelho, y = x2
em verde, para x ∈ [ 32 , 2]

4. Intervalos e suas utilidades


Um intervalo I ⊂ R é definido como o conjunto de todos os números Reais maiores
(ou iguais) a um certo número a e menores (ou iguais) que um certo b.1
Se impomos que sejam estritamente maiores que a e estritamente menores que b
temos um intervalo aberto
I = {x ∈ R; a < x < b}
denotado I = (a, b). Caso contrário surgem os intervalos semi-abertos, fechados, etc.
Um tı́pico intervalo que vamos usar no Curso será o intervalo aberto de raio ǫ > 0
centrado num ponto x:
(−ǫ + x, x + ǫ)
onde x é um ponto da reta dos Reais e ǫ > 0 é um número positivo fixado por nós.
O modo como vamos usar esses intervalos centrados é o seguinte: (−ǫ + x, x + ǫ)
será uma espécie de gaiola ou cercado em torno de x, delimitando pontos próximos
dele (à medida que ǫ > 0 é tomado pequeno).
Explico isso em mais detalhe:
Definição 4.1. A distância entre dois pontos x, x da reta dos Reais é definida pelo
módulo2 da diferença entre eles:
|x − x| = |x − x|.
1Podemos considerar a reta R toda ou uma semi-reta também como intervalos: veremos isso em
detalhe na Seção 4. Ao invés de usarmos o sı́mbolo (2, +∞) para denotar a semi-reta dos números
maiores que 2, prefiro usar o sı́mbolo R>2 : o motivo é evitar o mal uso do sı́mbolo +∞.
2para um número Real △, |△| := △, se △ ≥ 0 ou |△| := −△, se △ < 0
CAPÍTULO 3. PROPRIEDADE BÁSICAS DOS NÚMEROS REAIS 37

Pela definição de módulo, |x − x| < ǫ significa que


x − x < ǫ, se x − x ≥ 0 ou − (x − x) < ǫ, se x − x < 0.
É importante entender que:
Afirmação 4.1. (−ǫ + x, x + ǫ) é exatamente3 o conjunto dos pontos que distam de
x menos que ǫ > 0.
Demonstração.
Vamos mostrar primeiro que
(−ǫ + x, x + ǫ) ⊂ {x ∈ R; |x − x| < ǫ}.
Tome
x ∈ (−ǫ + x, x + ǫ),
com x 6= x (caso x = x não há nada a provar, pois ǫ > 0).
Ou seja x verifica:
−ǫ + x < x < x ou x < x < x + ǫ.
Que equivale (subtraindo x) a:
−ǫ < x − x < 0 ou 0 < x − x < ǫ.
Que equivale4 a:
0 < −(x − x) < ǫ ou 0 < x − x < ǫ,
ou seja, 0 < |x − x| < ǫ, como querı́amos.

Agora vamos mostrar que:


{x ∈ R; |x − x| < ǫ} ⊂ (−ǫ + x, x + ǫ).
.
Tome x ∈ {x ∈ R; |x − x| < ǫ}.
Se 0 ≤ x − x então temos
x−x< ǫ ⇔ x < x + ǫ,
e portanto x ∈ [x , x + ǫ).
Se x − x < 0 então
−(x − x) < ǫ ⇔ −x + x < ǫ ⇔ −ǫ + x < x,
ou seja, x ∈ (−ǫ + x , x).5.


3Dois conjuntos X e Y são iguais se X ⊂ Y e Y ⊂ X


4Atenção:
as desigualdade se invertem quando multiplicadas por um número negativo, por ex.,
1 < 2 < 3 mas −3 < −2 < −1
5O quadrado à direita significa que a demonstração terminou
4. INTERVALOS E SUAS UTILIDADES 38

4.1. O que é útil num intervalo aberto.


Os intervalos abertos são importante no Cálculo, e o ponto importante é que um
intervalo aberto tem uma certa tolerância com cada um de seus elementos. Podemos
mexer um pouquinho em cada um de seus elementos sem sair do intervalo aberto.
Mais especificamente:
Afirmação 4.2. Dado qualquer x ∈ (a, b) existe um pequeno intervalo aberto centrado
em x denotado Ix tal que Ix ⊆ (a, b).
Demonstração.
Considere as distâncias de x ∈ (a, b) até o extremo a e até o extremo b:
|x − a| := x − a > 0, |x − b| := b − x > 0
(são dois números positivos pois (a, b) é intervalo aberto).
Dentre os dois agora escolho o menor, chamando-o de δ0 > 0:
δ0 := mı́nimo{ x − a, b − x }.
Faça
Ix := (−δ0 + x, x + δ0 ),
e vamos verificar que
(−δ0 + x, x + δ0 ) ⊂ (a, b).
Para isso vamos supor que é o caso que δ0 = x − a, ou seja, que x está ou no centro
do intervalo (a, b) ou um pouco mais próximo de a que de b (analogamente no outro
caso). Então
(−δ0 + x, x + δ0 ) = ( −(x − a) + x, x + (x − a) ) =

= ( a, x + (x − a) ).
Ora supusemos estar na situação em que x − a ≤ b − x, logo:
(a, x + (x − a)) ⊆ (a, x + (b − x)) = (a, b),
portanto:
(−δ0 + x, x + δ0 ) ⊆ (a, b)
como querı́amos.


Observe nessa Prova que à medida que x se aproxima de a ou de b a tolerância


(medida pelo δ0 ) fica menor, mas sempre existe.
Já no intervalo semi-aberto I = (0, 5] não há tolerância nenhuma com seu elemento
5: ou seja, qualquer número δ > 0 que for somada a 5, já faz que 5 + δ não pertença
a (0, 5].
CAPÍTULO 3. PROPRIEDADE BÁSICAS DOS NÚMEROS REAIS 39

4.2. O que é útil num intervalo fechado.


Num intervalo aberto acontece de seus elementos estarem se aproximando cada
vez mais de um ponto que ele mesmo não está no intervalo, por assim dizer de um
fantasma. Por exemplo, os pontos 12 , 13 , . . . , n1 de (0, 5) estão cada vez mais próximos
de 0, mas mesmo assim 0 6∈ (0, 5). Isso não acontece no intervalo fechado [0, 5].
Dito de outro modo, no Curso não estamos apenas interessados em saber se um
certo número z pertence ou não pertence a um conjunto X ⊂ R, como se fazia no
ensino Médio. Também vamos querer saber se desse ponto z podemos achar elementos
x ∈ X tão próximos quanto quisermos.
• Se I é um intervalo aberto, pode acontecer que z ∈ / I e mesmo assim hajam
elementos de I tão próximos quanto quisermos.
• Se I é intervalo fechado, e há elementos de I tão próximos quanto quisermos
de z, então de fato z ∈ I.
Uma informação extremamente importante para um cientista é saber se uma
função que lhe interessa assume máximo ou mı́nimo em seu domı́nio e principal-
mente, saber onde o faz.
Somente os intervalos fechados I = [a, b] garantirão sempre máximos e mı́nimos
globais de funções, senão pode acontecer algo como segue.
Pense em f : (0, 5] → R, f (x) = x1 . À medida que vamos tomando os pontos
1/n ∈ (0, 5] a função vale
1
f ( ) = n,
n
que fica tão grande quanto quisermos. Note que (0, 5] não é um intervalo fechado.

5. Metamorfoses de cúbicas
Nesta Seção resolvi descrever curvas interessantes usando apenas propriedades
básicas do Reais, como regra dos sinais, desigualdades, módulo, etc. que já justifi-
camos acima neste mesmo Capı́tulo.
Tudo o que vem a seguir nesta Seção é baseado em que não há raı́z quadrada Real
de um número Real negativo.
Começemos com o conhecido cı́rculo y 2 + x2 = r 2 de raio r > 0. Observe que:

• podemos tomar o gráfico √ de y = r 2 − x2 para descrever o semicı́rculo su-
2 2
perior (ou tomar y = − r − x para o inferior).
• se r 2 − x2 > 0 há duas escolhas de raı́zes, positiva e negativa, e quando x = r
ou x = −r essas duas escolhas colapsam numa só, que é y = 0.
• Onde r 2 − x2√< 0 deixamos de trabalhar sobre os Reais, pois os valores asso-
ciados a y = r 2 − x2 passam para o terreno dos números Complexos.6Como
só tratamos neste Curso de funções a valores Reais, não existem pontos do
cı́rculo cuja coordenada x verifique r 2 − x2 < 0.
Por último, observe que mudando o valor de r muda o raio do cı́rculo, portanto
podemos pensar em y 2 + x2 = r 2 como sendo uma famı́lia de cı́rculos em que cada
elemento fica determinando pelo r. Veja a Figura:
6Há uma versão magnı́fica do Cálculo sobre os números complexos !
5. METAMORFOSES DE CÚBICAS 40

0,5

y 0
-1 -0,5 0 0,5 1
x

-0,5

-1

Bom, mas tratar de cı́rculos é covardia, pois temos sua imagem impressa na nossa
mente desde a infância.
Que tal tratarmos de alguma curva que não tenha sua imagem impressa na nossa
mente ? E ademaias, que tal tratarmos logo de uma famı́lia delas ?
Considere a familia de curvas dada por:
y 2 − x3 − r · x = 0, r 6= 0.
Vamos analisar separadamente o que acontece quando r > 0 e quando r < 0.

Caso r > 0:
Temos
y 2 = x3 + r x ⇔ y 2 = x · (x2 + r).
Como x2 + r ≥ r > 0, o sinal de x · (x2 + r) só depende do de x. Logo
• se x > 0 temos duas opções
p p
y = x · (x2 + r) ou y = − x · (x2 + r).
Ou seja, a curva não é um gráfico, ela tem uma parte no eixo y > 0 e uma
parte no eixo −y. Há√uma simetria relativa ao eixo dos x.
• ainda se x > 0, |y| = x3 + rx observo que fica tão grande quanto quisermos.
De fato, se dou o valor 7 K >> 1:
√3
x ≥ K 2 ⇒ x3 ≥ K 2 ⇒

⇒ x3 + rx ≥ K 2 ⇒ |y| = x3 + rx ≥ K.
p p
• essas duas escolhas y = x · (x2 + r) ou y = − x · (x2 + r) colapsam numa
só se x = 0, pois então y = 0.
• se x < 0 a(s) coordenada(s) y deixa de ser um número Real, ou seja, para
nós deixa de existir.
7O sinal >> 1 quer dizer bem maior que 1
CAPÍTULO 3. PROPRIEDADE BÁSICAS DOS NÚMEROS REAIS 41

Uma Figura compatı́vel8 com essa descrição é:

y 0
0 0,4 0,8 1,2 1,6
x
-1

-2

-3

Caso r < 0
Agora
y 2 = x · (x2 + r),
e (x2 + r) pode ser positivo, negativo ou positivo. Por isso o estudo do sinal de
x · (x2 + r)
é mais delicado.
Note que
√ √
x2 + r > 0 ⇔ x2 > −r > 0 ⇔ x2 > −r.
Só que √
x2 = |x|
e portanto temos √
x2 + r > 0 ⇔ |x| > −r.
√ √ √
Se x > 0, |x| >√ −r quer dizer x > −r mas se x < 0 isso quer dizer −x > −r,
ou seja x < − −r.
Em suma: √ √
x2 + r > 0 ⇔ x < − −r ou x > −r.
Então
• se x > 0 √
x · (x2 + r) ≥ 0 ⇔ x ≥ −r,
e teremos
√ duas opções de raı́zes para determinar y. Que colapsam para y = 0
se x = −r.
• se x ≤ 0, só teremos x · (x2 + r) ≥ 0 se (x2 + r) ≤ 0. Ou seja,

− −r ≤ x ≤ 0.
Nessa faixa de valores
√ de x teremos duas opções de y, que colapsam em y = 0
se x = 0 ou x = − −r.
8Na Figura traçada há mais informação do que a que justificamos. Somente na Seção 5 do
Capı́tulo 15 é que teremos esses dados.
5. METAMORFOSES DE CÚBICAS 42

Uma Figura compatı́vel com essa descrição é (r = −1).

y 0
-1 -0,5 0 0,5 1 1,5 2
x

-1

-2

Por último, note que se |r| vai ficando pequeno, então os pontos

√ √
(− −r, 0), (0, 0) e ( −r, 0)

vão se aproximando. Note que as ovais da parte negativa vão diminuindo de tamanho
quando |r| vai diminuindo.
Imagine r vindo de valores positivos, que vão ficando bem próximos de zero, pulam
o valor zero, e passam a assumir então valores negativos.
É como se de um continente fosse expelida uma ilhota, que vai ficando maior e
mais distante do continente: as quatro figuras a seguir tentam mostrar isso.

y 0
0 0,4 0,8 1,2 1,6
x
-1

-2

-3
CAPÍTULO 3. PROPRIEDADE BÁSICAS DOS NÚMEROS REAIS 43

Figura: A curva y 2 − x3 − x = 0.

y 0
0 0,5 1 1,5 2
x
-1

-2

-3

Figura: A curva y 2 − x3 − 0.4 x = 0.

y 0
-0,5 0 0,5 1 1,5 2
x
-1

-2

Figura: A curva y 2 − x3 + 0.3 x = 0.

y 0
-1 -0,5 0 0,5 1 1,5 2
x

-1

-2

Figura: A curva y 2 − x3 + x = 0.
5. METAMORFOSES DE CÚBICAS 44

5.1. Suavização do caso r = 0.


Há uma pergunta natural: o que acontece na curva y 2 − x3 − 0 x = y 2 − x3 = 0 ?
Já aviso: os programas gráficos ficam bem perdidos para traçar essa curva, se a
coordenada x fica próxima de 0.
Por isso vou proceder como em muitos ramos da ciência, vou tentar inferir qual
o formato dessa curva tomando curvas que entendamos e que estejam cada vez mais
próximas dela.
Num sentido que ficará claro mais tarde, essas curvas próximas são suaves ou
não-singulares (ver Definição 4.1 na Seção 4 do Capı́tulo 32).
Na Figura a seguir traço a curva y 2 − x3 = 0 só que estabeleço x ≥ 0.4, deixando
a região em torno de x = 0 como um mistério.

y 0
0 0,4 0,8 1,2 1,6
x
-1

-2

-3

A curva y 2 − x3 = 0, só que x ≥ 0.4.


Como quero ter mais luz sobre esse objeto y 2 − x3 = 0 não vou deformá-lo de novo
na famı́lia y 2 − x3 − r x = 0, mas sim noutra famı́lia:
y 2 − x3 + s = 0, s ∈ R>0 .
Observo que a relação
y 2 = x3 − s
permite
√ tirar raı́zes quadradas desde
√ que x3 − s ≥ 0. Portanto há duas opções de
x > 3 s ou apenas y = 0 se x = 3 s.
Ou seja:

• a curva y 2 = x3 −√ s só tem traço no plano Real se x ≥ 3
se
• a partir de x > s a curva √
3
é simétrica em relação
√ ao eixo x, já que temos
3
duas opções diferentes: y = x − s e y = − x − s. 3

Ademais note que se x > 3 s, então
√ √
y = x3 − s < x3
e √ √
y = − x3 − s > x3 .
ou seja:
CAPÍTULO 3. PROPRIEDADE BÁSICAS DOS NÚMEROS REAIS 45
2 3
• dado x > 0,
√ o traço da curva y = x + s que tem y > 0 fica sempre abaixo
do de y = x3 .
o traço da curva y 2 = x3 + s que tem y < 0 fica sempre acima
• dado x > 0, √
do de y = − x3 .

A Figura a seguir ilustra isso para y 2 − x3 + 8 = 0:

y 0
0,5 1 1,5 2 2,5
x

-2

-4

A curva y 2 − x3 = 0, só que x ≥ 0.4, e a curva y 2 − x3 − 8 = 0.


As Figuras a seguir ilustram curvas cada vez mais próximas:

y 0
0,5 1 1,5 2 2,5
x

-2

-4

A curvas y 2 − x3 = 0, y 2 − x3 + 8 = 0 e y 2 − x3 + 1 = 0.
6. EXERCÍCIOS 46

y 0
0,5 1 1,5 2 2,5
x

-2

-4

A curvas y 2 − x3 = 0, y 2 − x3 + 8 = 0, y 2 − x3 + 1 = 0 e y 2 − x3 + 0.5 = 0.
Será que agora o leitor consegue inferir a forma de y 2 − x3 = 0 ?
6. Exercı́cios
Exercı́cio 6.1. (resolvido)
Prove, ao invés de apenas assumir, que vale:
x · x = (−x) · (−x), ∀x ∈ R.
Exercı́cio 6.2. (resolvido)
Para quais valores de x:
i) −3x + 2 > 0 ?
ii) x2 − x > 0 ?
iii) 3x2 − 2x − 1 > 0 ?
iii) 3x + 2 > 2x − 8 ?
iv) |x − 6| < 2 ?
v) |x + 7| < 1 ?
Exercı́cio 6.3. (resolvido)
Prove que para quaisquer números Reais  e △:
| + △| ≤ || + |△|.
Exercı́cio 6.4. Como são os gráfico das funções (com domı́nio ∀x ∈ R):
i) y = |x|,
ii) y = −| x|,
iii) y = |x − 5|,
iv) y = |x| + |x − 1| + |x − 2| ?
CAPı́TULO 4

Sequências e seus limites

1. Sequências
Neste Curso será importante a situação em que o domı́nio de uma função será o
conjunto dos números Naturais N = {1, 2, 3, ...}. Nesse caso

f :N→R

é chamada de sequência.
A imagem de uma tal f é uma lista de números Reais. Como cada ponto de sua
imagem é do tipo f (n) é comum denotá-lo por xn e a sequência toda por (xn )n .

Exemplo 0: f : N → R dada por f (n) = K é a sequência mais boba de todas,


pois sua imagem é somente o conjunto {K} - chama-se sequência constante.

Exemplo 1: Uma sequência não tão boba é f : N → R dada por f (n) = 2n, cuja
imagem são os números Pares.

Exemplo 2:
Uma sequência fundamental para todo o Curso é
1
f : N → R, f (n) = .
n
No que segue, dizer que N é um conjunto ilimitado em R é dizer que sempre há
um número Natural maior que qualquer número Real que for dado.

Afirmação 1.1. O fato de que os números naturais N formam um conjunto ilimitado


nos R é equivalente ao fato de que os valores de f : N → R, f (n) = 1/n ficam tão
próximos quanto quisermos de 0, desde que n seja suficientemente grande.

Demonstração.
Uma equivalência é uma implicação em dois sentidos: ⇔.
Prova do sentido ⇒: Obviamente 1/n nunca é igual a 0: caso pensássemos o
contrário para algum n0 , obterı́amos de n10 = 0 e multiplicando por n0 obtemos que
0 = 1: absurdo.
A distância entre f (n) = 1/n e 0 é dada por |1/n − 0| = 1/n. Suponha que nos
foi dado um número positivo muito pequeno ǫ0 > 0. Queremos confirmar que

1/n < ǫ0
47
2. LIMITES DE SEQUÊNCIAS 48

a partir de um certo n, ou seja se n ≥ nǫ (onde uso a notação nǫ para destacar que


esse n depende do ǫ, quanto menor o ǫ maior o nǫ ). Mas negar o anterior seria dizer:
1
∀n ∈ N, ǫ0 ≤ .
n
n
Mas isso equivale (multiplicando por ǫ0 > 0):
1
∀n ∈ N, n≤
ǫ0
1
Concluirı́amos então que o número ǫ0
é maior que todos os números naturais, con-
tradizendo a hipótese.
Prova do sentido ⇐:
Se existe um número K ∈ R tal que ∀n ∈ N tenhamos n ≤ K então ∀n ∈ N
terı́amos K1 ≤ n1 . Logo a sequência n1 não se aproxima de 0 mais que K1 . Contradição.


Observação: É possı́vel se colocar um Axioma sobre os números Reais - chamado


Axioma de Completamento - que implica a propriedade de N ser ilimitado em R.
Para nós, neste Curso, o fato dos Naturais serem ilimitados é tomado como um
Axioma.
Podemos também dizer o conteúdo da Afirmação anterior de outro modo: dada
uma cerca (−ǫ + 0, 0 + ǫ), se tomamos um nǫ suficientemente grande, então ∀n ≥ nǫ
teremos 1/n ∈ (−ǫ + 0, 0 + ǫ). Ou seja, esperando o tempo suficiente nǫ , a partir dali
a sequência 1/n não sai mais da gaiola (−ǫ + 0, 0 + ǫ). Simbolicamente escreveremos
1
lim = 0,
n→+∞ n

que lê-se assim: zero é o limite da sequência 1/n ou a sequência tende a zero
Veremos adiante que há sequências que tendem de diversas maneiras diferentes
a pontos, algumas vão decrescendo em valores como a (xn )n = 1/n, outras vão
crescendo como −1/n, outras vão oscilando e assim por diante, mas o que é importante
é que:
• elas entram em qualquer cerca estabelecida em torno de seu limite, desde
que se espere o tempo nǫ suficiente e
• depois de lá entrarem não mais saem.
Veremos também que podemos combinar sequências simples (cujo limite podemos
intuir facilmente) para criar sequências complicadas, das quais não é possı́vel ter uma
intuição de seu limite (exceto alguém com poderes para-normais ...). Mesmo assim
poderemos matematicamente determinar esses limites.

2. Limites de sequências
O conceito de limite é o conceito fundamental do Cálculo, de onde surgem out-
ras noções importantes como continuidade, derivada e integral. Por isso este é um
Capı́tulo um pouco mais extenso.
CAPÍTULO 4. SEQUÊNCIAS E SEUS LIMITES 49

Imagine uma máquina, um sistema ou um processo tal que para um certo input
x dá um certo output f (x). Agora imagine que para um input parecido x + h (com
h pequeno) dá um output parecido: f (x + h) = f (x) + δ, com δ pequeno.
Apesar de ser uma situação plausı́vel, da qual temos muitos exemplos no dia a dia,
também sabemos que há exemplos da situação oposta, em que, apesar de x + h ∼ x
temos f (x + h) muito diferente de f (x). Essas duas possibilidades são tı́picas de
processos contı́nuos e descontı́nuos, respectivamente.
O objetivo deste capı́tulo é definir essas noções precisamente, pois nelas se apoiam
os dois conceitos centrais do Curso: Derivada e Integral.

3. Definição e Propriedades fundamentais


Vamos começar com a Definição 3.1, que é mais precisa e importante do que
parece.
Nela destaco que há:
• uma enorme exigência: onde dizemos ∀ǫ >, e
• uma imposição: a de que a partir de um certo nǫ a sequência não mais saia
de uma região onde entrou.
Definição 3.1. Um sequência (xn )n tende a um ponto L se ∀ǫ existe nǫ ∈ N tal que
se n ≥ nǫ então xn ∈ (−ǫ + L, L + ǫ).
Há diferentes formas pelas quais uma sequência pode tender a um limite; em
particular, com diferentes velocidades.
Por exemplo, Afirmo que xn = n12 tende a 0 mais rapidamente do que zn = n1 o
faz. Ou seja, Afirmo que o tempo nǫ (zn ) de espera para ter zn < ǫ é menor que o
tempo nǫ (xn ) que tenho de esperar para ter xn < ǫ. De fato,1:
r
1 1
nǫ (zn ) = ⌈ ⌉, nǫ (xn ) = ⌈ ⌉,
ǫ ǫ
q
e é claro que 1ǫ ≤ 1ǫ para ǫ pequeno.
Nos argumentos discutidos abaixo teremos às vezes que esperar o tempo n su-
ficiente para que duas ou mais sequências se aproximem de onde queremos. Como
podem ser diferentes, por precaução tomamos o maior dentre eles, para que as duas
ou mais sequências estejam onde queremos.
Teorema 3.1. (Propriedades fundamentais de sequências)
Sejam (xn )n e (zn )n duas sequências, com
lim xn = L1 e lim zn = L2 .
n→+∞ n→+∞

Então:
1) A sequência soma (xn + zn )n tem
lim (xn + zn ) = L1 + L2 .
n→+∞

1onde ⌈△⌉ significa o primeiro número Natural maior ou igual que △ ∈ R.


3. DEFINIÇÃO E PROPRIEDADES FUNDAMENTAIS 50

2) A sequência diferença (xn − zn )n tem


lim (xn − zn ) = L1 − L2 .
n→+∞

3) Se C ∈ R é uma constante, então a sequência (C · xn ) tem


lim (C · xn ) = C · L1 .
n→+∞

4) Seja (qn )n uma sequência qualquer tal que


∀n, |qn | ≤ K,
para algum K. Se L1 = 0 então limn→+∞ (qn · xn ) = 0
5) A sequência produto (xn · zn )n tem
lim (xn · zn ) = L1 · L2 .
n→+∞

6) Se L2 6= 0, então:
• i) a partir de um certo n, zn 6= 0 e
• ii) limn→+∞ xznn = LL21 .
7) Suponha adicionalmente que a partir de um certo n, xn ≤ L1 e que, para uma
sequência qualquer qn , a partir de um certo n temos
xn ≤ qn ≤ L1 .
Então
lim qn = lim xn = L1 .
n→+∞ n→+∞

Demonstração. (de alguns itens do Teorema 3.1)


Prova de 1) Nesse primeiro item, o ponto a lembrar é que xn e zn se aproximam
cada uma de um número a princı́pio distinto e que cada uma delas o faz possivelmente
com velocidade diferente.
O que queremos provar? Queremos saber se, esperando um tempo nǫ suficiente,
conseguimos que:
xn + zn ∈ (−ǫ + L1 + L2 , L1 + L2 + ǫ),
ou seja, como já explicamos, se |xn + yn − (L1 + L2 )| < ǫ. Vamos traduzir esta última
condição de outro modo, que leva em conta as duas hipóteses sobre xn e zn 2:
|xn + yn − (L1 + L2 )| = |xn − L1 + yn − L2 | ≤
≤ |xn − L1 | + |yn − L2 |.
Agora fazemos o seguinte: esperamos tempo suficiente nǫ para que tenhamos
ǫ ǫ
∀n ≥ nǫ , |xn − L1 | < e |zn − L2 | < .
2 2
2No último passo uso uma desigualdade (chamada desigualdade triangular, ver Exercı́cio 6.3)
que vale para quaisquer números Reais  e △:
| + △| ≤ || + |△|
, no nosso caso aplicadoa para  = xn − L1 e △ = yn − L2
CAPÍTULO 4. SEQUÊNCIAS E SEUS LIMITES 51

Então obtemos de acima:


ǫ ǫ
|xn + yn − (L1 + L2 )| ≤ |xn − L1 | + |yn − L2 | < + = ǫ,
2 2
exatamente o que querı́amos provar.
Prova de 2): Análoga à do 1), apenas fazendo agora:
|(xn − yn ) − (L1 − L2 )| = |xn − L1 + L2 − zn | ≤ |xn − L1 | + |L2 − zn |.
Prova de 3): agora queremos que a partir de um certo nǫ :
| C · xn − C · L1 | < ǫ.
É claro que posso supor C 6= 0, senão tudo é óbvio.
Ora então o que queremos é provar que:
| C · (xn − L1 ) | < ǫ,
3
ou seja queremos que
|C| · |xn − L1 | < ǫ.
Noto agora que, se espero tempo nǫ suficiente, tenho:
ǫ
|xn − L1 | < , onde C 6= 0
C
pois xn se aproxima tanto quanto quisermos de L1 . Então juntando as informações:
ǫ
|C · xn − C · L1 | = |C| · |xn − L1 | < C · = ǫ,
C
exatamente o que querı́amos.
Prova de 4): Aqui o que fazemos é esperar o tempo nǫ suficiente para que |xn | < Kǫ
(estou supondo que K 6= 0, pois se K = 0, então a hı́pótese |qn | ≤ 0 diz que qn = 0
∀n e tudo é óbvio, pois a sequência 0 · xn é a sequência constante, igual a 0). Então
para n ≥ nǫ :
ǫ
|qn · xn | = |qn | · |xn | < K · = ǫ,
K
como querı́amos.
Prova de 5): Queremos fazer
| xn · zn − L1 · L2 | < ǫ.
dese que n cresça o suficiente.
Mas posso escrever:
| xn · zn − L1 · L2 | =
= | xn · zn −xn · L2 + xn · L2 −L1 · L2 | =
| {z }
0
= | xn · (zn − L2 ) + L2 · (xn − L1 ) | ≤
≤ | xn · (zn − L2 ) | + | L2 · (xn − L1 ) | =
= | xn | · | (zn − L2 ) | + | L2 | · | (xn − L1 ) |
3Para quaiquer números Reais  e △ sempre vale:
| · △| = || · |△|;
no nosso caso, uso para  = C e △ = xn − L1
3. DEFINIÇÃO E PROPRIEDADES FUNDAMENTAIS 52

E agora noto que |xn | ≤ K para alguma K , pois xn tende ao L1 ∈ R. E tanto


| (xn − L1 ) | quanto | (zn − L2 ) | se faz tão pequeno quanto quisermos, pois zn tende a
L2 e xn tende a L1 .
Logo | xn · zn − L1 · L2 | fica tão pequeno quanto quisermos.

Prova de 6): Primeiro afirmo que a partir de um certo n temos


L2
| < |zn |.
|
2
Se L2 > 0, a partir de um certo n temos
L2
0< < zn
2
L2
pois 2
< L2 = lim zn . E se L2 < 0, a partir de um certo n
L2
zn < <0
2
pois lim zn = L2 < L22 .
Ou seja, a partir de um certo n:
L2
| < |zn |
|
2
e em particular a partir desse n, temos zn 6= 0.
No que segue já suponho que tomei esse n para que a partir dele:
L2
|
| < |zn |.
2
Então além de podermos dividir pelos zn , podemos afirmar que
|L2 |2
< |zn | · |L2 |
2
e portanto
1 2
< .
|zn · L2 | |L2 |2
Portanto
1 1 L2 − zn
| − |=| |=
zn L2 zn · L2
1
=| | · |L2 − zn | ≤
zn · L2
2
≤ · |L2 − zn |.
|L2 |2
Mas |L2 −zn | se faz tão pequeno quanto quisermos, desde que esperemos possivelmente
um tempo n ainda maior, já que lim zn = L2 .
Por exemplo, podemos esperar um n a partir do qual valha | L22 | < |zn | e também
ǫ · L22
|L2 − zn | < ,
2
CAPÍTULO 4. SEQUÊNCIAS E SEUS LIMITES 53

o que dá
1 1 2 ǫ · L22
| − |< · = ǫ.
zn L2 |L2 |2 2
Sobre 7): de fato, após esquecermos um certo número de termos das sequências,
temos
| qn − L1 | ≤ |xn − L1 |
e |xn − L1 | se faz tão pequeno quanto quisermos.


Chamo a atenção para uma propriedade, que provamos como parte do item 6), e
que será bastante útil:

Afirmação 3.1. Se limn→+∞ xn = L e L 6= 0 então a partir de um certo tempo n,


xn 6= 0. Em particular, se L > 0 (ou L < 0) então a partir de um certo tempo n,
xn > 0 (ou xn < 0).

Por último, será útil mais tarde se introduzimos dois sı́mbolos:


Definição 3.2. Dizemos que
lim xn = +∞
n→+∞

se ∀K > 0 existe um tempo nK tal que se n ≥ nK temos xn > K. Dizemos que


lim xn = −∞
n→+∞

se ∀K < 0 existe um tempo nK tal que se n ≥ nK temos xn < K.


Ou seja, sequências que ficam tão positivas quanto quisermos, ou sequências que
ficam tão negativas quanto quisermos, esperando o tempo n suficiente. Exemplos:
xn = n2 e xn = −n2 , respectivamente.

4. Exercı́cios
Exercı́cio 4.1. Exemplifique com sequências (xn )n bem simples a diferença entre as
seguintes frases:
i) a partir de um certo tempo n a sequência xn dista de L menos que um ǫ > 0 e
ii) existem tempos n arbitrariamente grandes tais que xn dista de L menos que
um ǫ > 0.
1
Exercı́cio 4.2. Para as sequências (xn )n abaixo e para a função y = f (x) = x2
, diga
o formato da sequência ( f (xn ) )n :
i) xn = √1n ,
ii) xn = n1 ,
iii) xn = n2 .
4. EXERCÍCIOS 54

Exercı́cio 4.3.
Explique se existem ou não os limites das seguintes sequências:
i) xn := 5 n,
ii) xn := (−1)n 5,
iii) xn := (−1)n (5 + n1 ),
iv) xn := (−1)n n5
v) xn := (−1)n n1 .
vi) xn = n1 + n2 + n3 ,
vii) xn = n1 · n2 · n3 .
Exercı́cio 4.4.
No dia-a-dia sabemos que todo gremista gosta de azul, mas nem todos que gostam
de azul são gremistas.
Tratando-se agora de sequências xn e zn , dê exemplos onde não existem
lim xn ou lim zn
n→+∞ n→+∞

mas que no entanto existam:


lim (xn + zn ) ou lim (xn · zn ).
n→+∞ n→+∞

Exercı́cio 4.5. (resolvido)


Prove duas propriedades fundamentais de limites:

i) se xn < 0 ∀n e se limxn = L então L ≤ 0. Dê exemplo onde todo xn < 0 mas


onde L = 0.

ii) se limxn = L e se ∀n xn ≤ zn ≤ L, então limzn = L.


Exercı́cio 4.6. Usando algumas sequências já estudadas em aula e propriedades de
+, −, ·, / de sequências, calcule:
1 1 300n2 + 35n + 1000
lim 3 · (2 − + 2 ), lim ,
n→+∞ n n n→+∞ n3 + n
300n2 + 35n + 1000 10123456789
lim , lim ,
n→+∞ 150n2 + n + 10000 n→+∞ n
30000000n + 1200000 2n7 + 35n + 1000
lim , lim .
n→+∞ n2 n→+∞ 3n7 + n + 10000
Dica: fatore n à força no numerador e no denominador as potências mais altas e
simplifique, antes de passar ao limite.
Exercı́cio 4.7. As sequências a seguir tendem a zero. Dado ǫ > 0 determine qual
n (em função de ǫ) é suficiente para termos |xn | < ǫ nas seguintes sequências: a):
xn = n14 , b): xn = √1n , c): xn = 4√1 n
1
Exercı́cio 4.8. A sequência xn = n
fica dentro do intervalo [0, 1] e é decrescente, ou
seja
xn+1 ≤ xn , ∀n.
CAPÍTULO 4. SEQUÊNCIAS E SEUS LIMITES 55

Já a sequência xn = 1 − n1 fica também dentro do intervalo [0, 1] mas é crescente, ou


seja xn+1 ≥ xn , ∀n. É verdade o seguinte Teorema: sequências que ficam dentro
de algum intervalo e que são ou bem crescentes ou bem decrescentes convergem para
algum limite.
Veja em quais sequências a seguir pode-se aplicar esse Teorema: a): xn = 5n1 2 , b):
n 2n 2n+1
xn = 5n1
, c): xn = (−2)
n
, d): xn = (−1)
n
, e): xn = (−1)n .
CAPı́TULO 5

Limites de funções definidas em intervalos

Neste Curso usaremos a noção de continuidade fortemente quando calcularmos


algumas Derivadas e mais adiante na teoria de Integração do Capı́tulo 21.
Daremos sua definição precisa no próximo Capı́tulo.
Mas para isso, antes precisamos entender a noção de limite de funções definidas
em intervalos. Até agora só vimos limites de um tipo de função, cujo domı́nio são os
Naturais, as chamadas sequências.
Agora vamos definir:
Definição 0.1. Seja uma função f : I → R, y = f (x) definida num intervalo I. Seja
x tal que exista alguma sequência xn ∈ I \ {x} com limn→+∞ xn = x.
Dizemos que função f tem limite L quando x tende a x, denotado por
lim f (x) = L, L ∈ R,
x→x

se para toda sequência xn contida em I \ {x}


lim xn = x
n→+∞

temos
lim f (xn ) = L.
n→+∞

Observações importantes sobre a Definição 0.1:


• O ponto importante nesta definição é que, não importa quantas sequências
tomemos com limn→+∞ xn = x, sempre as sequências f (xn ) tendem para o
mesmo número L.
• O fato de que não seja relevante como xn se aproxima de x, mas apenas que
xn se aproxima x, fica visı́vel no sı́mbolo que usamos:
lim f (x).
x→x

• O leitor verá mais tarde que às vezes x não está no domı́nio das funções, ou
seja, que não faz sentido perguntar por quanto a função vale nele, mas que,
como x está arbitrariamente próximo do domı́nio dessas funções, podemos
perguntar quanto a função vale em pontos do domı́nio cada vez mais próximos
dele.
• o valor f (x) pode ser bem diferente de limx→x f (x). Por isso tomamos
sequências xn contidas em I \ {x} (ou seja, que não valem nunca x).
57
1. OPERAÇÕES ELEMENTARES COM LIMITES DE FUNÇÕES 58

1. Operações elementares com limites de funções

A noção de limite de funções foi construı́da a partir da de limite de sequências;


assim que é natural que as propriedades de limites de sequências repercutam nas dos
limites de funções definidas em intervalos.
Teorema 1.1. (Propriedades fundamentais de limites de funções)
Sejam f e g cujos domı́nios são intervalos e seja x tal que existam sequências nos
domı́nios dessas funções que tendam a ele.
Suponha que existam:
lim f (x) = L1 e lim g(x) = L2 .
x→x x→x

Então:
1) A função soma f + g tem
lim (f + g)(x) = L1 + L2 .
x→x

2) A função diferença f − g tem


lim (f − g)(x) = L1 − L2 .
x→x

3) Se C ∈ R é uma constante, então a função (C · f )(x) := C · f (x) tem


lim (C · f )(x) = C · L1
x→x

4) Suponha uma função q(x) com o mesmo domı́nio da f (x) tal que |q(x)| ≤ K,
∀x. Suponha adicionalmente que L1 = 0. Então
lim ( f (x) · q(x) ) = 0.
x→x

5) A função produto (f · g)(x) tem


lim (f · g)(x) = L1 · L2 .
x→x

6) Se L2 6= 0, então: i) se x é suficientemente próximo de x então g(x) 6= 0 e ii)


limx→x fg(x)
(x)
= LL21 .
7) Suponha uma outra função q(x) definida no mesmo domı́nio e que adicional-
mente f (x) ≤ q(x) ≤ L1 . Então
lim q(x) = lim f (x) = L1 .
x→x x→x

Demonstração.
Prova do Item 1): Queremos saber se
lim ( f (xn ) + g(xn ) ) = L1 + L2 ,
n→+∞

quando tomamos qualquer sequência xn com


lim xn = x.
n→+∞

Mas por hipótese, limn→+∞ f (xn ) = L1 e limn→+∞ g(xn ) = L2 , quando tomamos


qualquer sequência xn com limn→+∞ xn = x.
CAPÍTULO 5. LIMITES DE FUNÇÕES DEFINIDAS EM INTERVALOS 59

Ora, pelo item 1) do Teorema 3.1, aplicado às sequências f (xn ) e g(xn ), concluimos
que limn→+∞ ( f (xn ) + g(xn ) ) = L1 + L2 .

A prova de outros itens fica para o leitor, bastando combinar a Definição 0.1 com
alguns itens do Teorema 3.1, bem como com a Afirmacao 3.1. 

2. A definição usual com ǫ e δ


Na maioria dos livros texto de Cálculo, o limite de uma função definida em um
intervalo é definido assim:
Definição 2.1. Dizemos que f tende a L quando x tende ao x, ou em sı́mbolos:
lim f (x) = L
x→x

se ∀ǫ > existe δ > 0 tal que se 0 < |x − x| < δ então |f (x) − L| < ǫ.

Observações:
• pense em ǫ > 0 como um número pequeno, que impõe o desafio de se encon-
trar o δ > 0 suficiente para termos |f (x) − L| < ǫ, desde que 0 < |x − x| < δ.
• o sı́mbolo ∀ǫ > 0 (para todo ǫ > 0) diz que ǫ será feito tão pequeno quanto
quisermos,
• veremos logo abaixo que o δ depende do ǫ, da natureza da f e também, em
geral, de cada ponto x.
• a cláusula 0 < |x − x| existe para que possamos ter funções com f (x) 6= L =
limx→x f (x).
Um pouco mais sobre o último item: suponha que temos uma f com f (x) bem
diferente dos valores f (x), para x próximos de x porém diferentes de x. Por exemplo
suponha que |f (x) − L| ≥ 1 , embora |f (x) − L| < ǫ é pequeno se x 6= x, mas x
próximo de x. Então |x − x| = 0 < δ, ∀δ > 0 e no entanto |f (x) − L| ≥ 1. Por isso na
Definição 2.1 estamos interessados apenas em controlar os valores f (x) para x 6= x.

Vejamos agora que essa nova Definição 2.1 tem o mesmo conteúdo da Definição
0.1 do Capı́tulo 4, mesmo que a princı́pio não pareçam o mesmo.
Afirmação 2.1. A Definição 2.1 é equivalente à Definição 0.1 do Capı́tulo 4.
Demonstração. (da Afirmação 2.1)
Provar a equivalência de duas definições é mostrar que uma implica a outra e
vice-versa.
Suponha por um momento a Definição 0.1 e por absurdo negue a Definição 2.1.
Então existe um ǫ0 > 0 especial tal que ∀δ > 0 existe um xδ com
0 < |xδ − x| < δ, mas |f (xδ ) − L| ≥ ǫ0 .
2. A DEFINIÇÃO USUAL COM ǫ E δ 60

Já que vale para todo δ > tomo-os da forma δ(n) := n1 . Então concluo que os
xδ(n) formam uma sequência de I \ {x} que tende a x, pois
1
0 < |xδ(n) − x| <
n
e já sabemos que os n1 ficam tão pequenos quanto quisermos. Com essa sequência
(xδ(n) )n no domı́nio da f , formo outra sequência f (xδ(n) ) na imagem da f , que não
tende a L já que
|f (xδ(n) ) − L| ≥ ǫ0 , ∀n,
ou seja, não se aproxima do número L mais que ǫ0 . Isso contradiz a Definição 0.1.
Agora suponha Definição 2.1 e vamos obter a informação dada pela Definição 0.1.
Considere qualquer sequência xn de I \ {x} que tenda a x: queremos saber então
se é verdade que f (xn ) tende a L. Ou seja, se dado ǫ > 0 existe nǫ ∈ N tal que
∀n ≥ nǫ temos |f (xn ) − L| < ǫ.
O que sei pela Definição 2.1 é que existe um δ > 0 tal que:
0 < |x − x| < δ ⇒ |f (x) − L| < ǫ.
Então tomo esse δ > 0 e, para ele, tomo um nδ ∈ N tal que:
∀n ≥ nδ ⇒ 0 < |xn − x| < δ
(o que funciona pois xn tende a x).
Logo |f (xn ) − L| < ǫ pois os xn entraram na região adequada em torno de x, que
é (−δ + x, x + δ).
A Figura ilustra:

L+ ε

f (x_n)
L− ε

x_n

x −δ x x +δ

Lembrando que o δ = δ(ǫ), pois depende de ǫ, obtivemos o que querı́amos, já que
|f (xn ) − L| < ǫ a partir de um certo tempo nδ(ǫ) .


Exemplos:
CAPÍTULO 5. LIMITES DE FUNÇÕES DEFINIDAS EM INTERVALOS 61

1)- f (x) = ax + b, polinômio de grau ≤ 1, tem limx→x f (x) = ax + b. De fato, se


a = 0 é claro que a f ≡ b constante tende a b. Caso a 6= 0, quando for dado ǫ > 0
ǫ ǫ
tome por exemplo δ(ǫ) := |a| . Então se |x − x| < |a| temos:
ǫ
|f (x) − L| = |ax + b − (ax + b)| = |a||x − x| < |a| · = ǫ,
|a|
como querı́amos.
2)- No exemplo 1) o δ só dependeu do ǫ. Agora dou um exemplo em que o δ
depende também do x, ficando cada vez menor à medida que o x vai sendo escolhido
mais perto de um extremo do domı́nio da f .
Seja f : R>0 → R, f (x) = x1 . Veremos na próxima Seção que limx→x f (x) = x1 .
Mas a Figura a seguir ilustra como vai ficando mais difı́cl encontrar o δ adequado à
medida que x > 0 se aproxima do 0.

Figura: Para um mesmo ǫ, preciso cada vez menores valores de δ

3. Limites quando x tende ao infinito


Quando um cientista quer entender um fenômeno, ele pode querer entender não
apenas o comportamento agora, mas sim a longo prazo. Por exemplo, pode se per-
guntar se a longo prazo a Lua permanecerá girando em torno da Terra.
Na linguagem do Cálculo isso se expressa numa pergunta assim: a que tende o
fenômeno quando o tempo x fica arbitrariamente grande ? O que se põe em sı́mbolos:
lim f (x) = L ∈ R, ou lim f (x) = L ∈ R.
x→+∞ x→−∞

Ambos sı́mbolos admitem dois tipos de definições (equivalentes)


Definição 3.1. Dizemos que
lim f (x) = L ∈ R
x→+∞

se ∀ǫ > 0 existe K > 0 tal que |f (x) − L| < ǫ, se x > K.


Ou
3. LIMITES QUANDO X TENDE AO INFINITO 62

Definição 3.2. Dizemos que


lim f (x) = L ∈ R
x→+∞

se ∀(xn )n contida no domı́nio de f com limn→+∞ xn = +∞ temos limn→+∞ f (xn ) =


L.
(onde limn→+∞ xn = +∞ foi apresentado na Definição 3.2).

Deixo para o leitor verificar a equivalência dessas duas Definições 3.1 e 3.2.
Analogamente se define limx→−∞ f (x) = L ∈ R.
Geometricamente, as Definições 3.1 ou 3.2 se ilustram na Figura a seguir, em que
o gráfico se aproxima da altura L cada vez mais:

0,98

0,96

0,94

0,92

50 100 150 200 250 300


x

Figura: Quando x aumenta o gráfico se aproxima de uma altura definida.

As propriedades básicas dessas noções são análogas àquelas do Teorema 1.1:


Teorema 3.1. Sejam f e g funções definidas em um intervalo ilimitado à direita.1
Suponha2
lim f (x) = L1 ∈ R e lim g(x) = L2 ∈ R.
x→+∞ x→+∞
Então:
1) A função soma f + g tem
lim (f + g)(x) = L1 + L2 .
x→+∞

2) A função diferença f − g tem


lim (f − g)(x) = L1 − L2 .
x→+∞

3) Se C ∈ R é uma constante, então a função (C · f )(x) := C · f (x) tem


lim (C · f )(x) = C · L1
x→+∞

4 ) Suponha uma função q(x) com o mesmo domı́nio da f (x) tal que |q(x)| ≤ K,
∀x. Suponha adicionalmente que L1 = 0. Então
lim ( f (x) · q(x) ) = 0.
x→+∞

1Enuncio apenas para x → +∞, pois é análogo se x → −∞


2
Atenção que L1 , L2 têm que ser números, não podem ser substituı́dos pelos sı́mbolos +∞ ou
−∞
CAPÍTULO 5. LIMITES DE FUNÇÕES DEFINIDAS EM INTERVALOS 63

5) A função produto (f · g)(x) tem


lim (f · g)(x) = L1 · L2 .
x→+∞

6) Se L2 = 6 0, então:
i) se x é suficientemente grande então g(x) 6= 0 e

ii) limx→+∞ fg(x)


(x)
= LL21 .
7) Suponha uma outra função q(x) definida no mesmo domı́nio e que adicional-
mente f (x) ≤ q(x) ≤ L1 . Então
lim q(x) = lim f (x) = L1 .
x→+∞ x→+∞

Demonstração.
Prova do item 1): Quero saber se a sequência soma f (xn ) + g(xn ) tende a L1 + L2 ,
se a sequência xn tem limn→+∞ xn = +∞. Mas por hipótese f (xn ) tende a L1 e
g(xn ) tende a L2 . Logo pelo item 1) do Teorema 3.1 aplicado às sequências f (xn ) e
g(xn ) obtemos que f (xn ) + g(xn ) tende a L1 + L2 .
Os outros itens se demonstram da mesma maneira. 

Exemplos:

1) Obviamente a função constante f ≡ C tem limx→+∞ C = C.


1
2) A função f : R<0 ∪ R>0 → R, f (x) = x
tem
1 1
lim = lim = 0.
x→+∞ x x→−∞ x
De fato, | x1 | < ǫ se |x| > K := 1ǫ , o que está de acordo com a Definição 3.1.

3)
C 1
lim = C · lim =C ·0=0
x→+∞ x x→+∞ x

usando o Teorema 3.1.


4) Também
1 1 1
lim 2
= lim ( · ) = 0 · 0,
x→+∞ x x→+∞ x x

pelo Teorema 3.1.

5)
1 1
lim (C + ) = C + lim =C +0=C
x→+∞ x x→+∞ x

usando o Teorema 3.1.


3. LIMITES QUANDO X TENDE AO INFINITO 64

6)
C1 x C1
lim = ,
x→+∞ C2 x + C3 C2
onde C1 , C2 , C3 são constantes não nulas. De fato, primeiro observe que se x se faz
tão grande quanto quisermos, em particular x > 0. Logo posso escrever:
C1 x x C1 C1
lim = lim C
= lim
x→+∞ C2 x + C3 x→+∞ x (C2 +
x
3
) x→+∞ (C2 + Cx3 )
e agora uso o Teorema 3.1 e os Exemplos anteriores , concluindo que
C1 C1
lim C
= .
x→+∞ (C2 + 3 )
x
C 2

7) O mesmo tipo de argumento do Exemplo 6) dá que:


an xn + an−1 xn−1 + . . . + a0 an
lim n n−1
= ,
x→+∞ bn x + bn−1 x + . . . + b0 bn
onde ai , bi são constantes, an 6= 0, bn 6= 0.
De fato, como posso supor x > 0:
an xn + an−1 xn−1 + . . . + a0
lim =
x→+∞ bn xn + bn−1 xn−1 + . . . + b0
an−1 a0
xn · (an + x
+ ...+ xn
)
= lim bn−1 b0
=
x→+∞ xn · (bn + + ...+
x xn
)
an−1
(an + x + . . . + xan0 ) an
= lim = ,
(bn + bn−1
x→+∞
x
+ . . . + xb0n ) bn
usando novamente o Teorema 3.1 e Exemplos prévios.
Ilustro o Exemplo 7) nas Figura que segue, onde an = a2 = 2 e bn = b2 = 1:

1,8

1,6

1,4

1,2

0,8

0,6

0 50 100 150 200


x

2x2 +x+4
Figura: Gráfico de x2 +3x+7
com x ∈ [0, 200].

8)
Se m < n, am 6= 0, bn 6= 0:
am xm + am−1 xm−1 + . . . + a0
lim = 0.
x→+∞ bn xn + bn−1 xn−1 + . . . + b0
CAPÍTULO 5. LIMITES DE FUNÇÕES DEFINIDAS EM INTERVALOS 65

De fato,
am−1
xm · (am + x
+ . . . + xam0 )
lim =
x→+∞ xm · xn−m · (bn + bn−1
x
+ . . . + xb0n )
am−1
1 (am + x
+ . . . + xam0 ) am
= lim bn−1
=0· = 0,
x→+∞ xn−m (bn + + . . . + xb0n ) bn
x
usando o Teorema 3.1.
Ilustro este Exemplo 8) na Figura a seguir, com am = a2 = 20 e bn = b3 = 0.01.
Escolhi o coeficiente b3 = 0.01 bem pequeno em relação ao a2 = 20 de propósito,
para indicar que não adianta, pois a longo prazo o grau 3 do denominador é mais
importante.

8000

6000

4000

2000

0
5 10 15 20 25 30
x

20x2 +30x+40
Figura: Gráfico de (0.01)x3
, para x ∈ [1, 30]

Estes dois Exemplos 7) e 8) ilustram o seguinte princı́pio: a longo prazo o que im-
porta são os graus mais altos dos polinômios envolvidos num quociente de polinômios.

9) Lembrando apenas que a função seno tem | sin(x)| ≤ 1, então


sin(x)
lim =0
x→+∞ x
1
pois limx→+∞ x
= 0 (use o Teorema 3.1).

0,4

0,3

0,2

0,1

0
20 40 60 80 100 120
x
-0,1

-0,2

sin(x)
Figura: O gráfico de x
para x ∈ [2, 130]
4. QUANDO A PARTE É DO MESMO TAMANHO DO TODO 66

4. Quando a parte é do mesmo tamanho do todo

Nesta Seção proponho explicar o seguinte Teorema, que parece um total absurdo:
Afirmação 4.1. A reta inteira de números Reais tem tantos pontos quanto o intervalo
aberto (−1, 1).

Em primeiro lugar preciso lembrar o que significa dois conjuntos terem o mesmo
número de elementos. O exemplo que mais gosto, para explicar essa noção, li num
um livro de Tarski.
Imagine num garçom colocando, para cada cliente, um garfo e uma faca ao lado
do prato. Ao final da tarefa, ele têm a seguinte conversa com o cozinheiro:
• cozinheiro: para preparar a refeição, gostaria de saber quantos clientes temos
hoje.
• garçom: não contei, não sei.
• cozinheiro: mas você não estava pondo os garfos e facas para cada um deles
?
• garçom: sim, mas só o que tenho certeza é que há tantos garfos quanto facas
à mesa.
• cozinheiro: mas como você pode ter certeza disso, sem saber quantos garfos
e facas você pôs, já que não contou ?
• garçom: ora, é fácil, sei que há tantos garfos quanto facas porque para cada
faca colocada, coloquei um garfo, e não mais de um garfo.
A moral dessa história é a seguinte: dois conjuntos têm o mesmo número de
elementos quando há uma função f sobrejetora (nenhuma faca sem garfo) e injetora
(não mais de um garfo) entre eles. Apesar de que não saibamos exatamente quantos
elementos os conjuntos têm.

Um exemplo conhecido já por Galileu é que há tantos números Naturais N quanto
números Pares 2N: de fato, existe a bijeção
f : N → 2N, f (n) = 2n,
cuja inversa dá f −1 (2n) = n. Apesar disso 2N ⊂ N, por isso se diz que, nesse caso, a
parte é do tamanho do todo !

Para provar a Afirmação 4.1, considero a seguinte função:


x
f : R → R, f (x) := .
|x|+ 1
Primeiro noto que está bem definida em todos os Reais, pois seu denominador nunca
se anula. Agora afirmo que f (R) ⊂ (−1, 1), ou seja, que
x
∀x ∈ R, −1 < < 1.
|x|+ 1
CAPÍTULO 5. LIMITES DE FUNÇÕES DEFINIDAS EM INTERVALOS 67

De fato, primeiro f (0) = 0 e se x > 0 então |x| = x e portanto:


x
0< < 1,
x+1
pois 0 < x < x + 1. E se x < 0, então |x| = −x e portanto:
x
−1 < < 0,
−x + 1
pois −1 · (−x + 1) = x − 1 < x.
O que não está ainda nada claro é se f é sobrejetora, ou seja, se
(−1, 1) ⊂ f (R), ou seja f (R) = (−1, 1).
Estou assumindo neste momento, sem demonstrar, que a imagem de f é algum
intervalo f (R) = (a, b) ⊂ (−1, 1).
O que quero mostrar agora é que não acontece que −1 < a nem que b < 1. Para
isso meu argumento é o seguinte: vou mostrar que
x x
lim =1 e lim = −1,
x→+∞ | x | + 1 x→−∞ | x | + 1

ou seja, pela Definição de limite, que f atinge valores tão próximos de 1 e de −1


quanto quisermos. Isso impedirá que −1 < a e que b < 1.
Mas se x → +∞ então em particular x > 0 e
x x x·1
lim = lim = lim = 1,
x→+∞ | x | + 1 x→+∞ x + 1 x→+∞ x · (1 + x1 )
pelo Teorema 3.1 e Exemplos que o seguem.
E se x → −∞ então em particular x < 0 e
x x x·1
lim = lim = lim = −1,
x→−∞ | x | + 1 x→−∞ −x + 1 x→−∞ x · (−1 + x1 )
pelo Teorema 3.1 e Exemplos que o seguem.
x
Agora só falta ver que f é injetiva: mas note que se x > 0, de y = x+1
obtenho
y = x − xy e daı́:
y
x= ,
1−y
x
que é bem definido pois y < 1. E se x < 0 então de y = −x+1
obtenho y = x + xy e
daı́:
y
x= ,
1+y
que é bem definido pois −1 < y.
Isso mostra que y = f (x) é injetiva, já que tenho explicitamente sua função inversa
x = f −1 (y).

As Figuras a seguir mostram parte dos gráficos de f e de f −1 , respectivamente:


5. EXERCÍCIOS 68

0,8
0,4
0
-4 -2 0 2 4
-0,4
-0,8x

0
-0,8
-0,40 0,4
0,8
x

-2

-4

Para terminar, chamo a atenção do leitor que f −1 : (−1, 1) → R faz uma espantosa
expansão do intervalo (−1, 1). A expansão feita por f −1 (y) depende sensivelmente
de y e aumenta cada vez mais à medida que y vai para os extremos do intervalo. Na
Parte 2 do Curso poderemos justificar e explicar melhor a seguinte Afirmação sobre
f −1 :

1
Afirmação 4.2. Se y ∈ [0, 1) então a taxa de expansão de f −1 é de (1−y)2
e a taxa
1
de expansão de f −1 (y) para y ∈ (−1, 0] é de (1+y)2.

Uma comparação é natural: um dos fenômenos mais bizarros do Universo é que


não apenas ele se expande, e que quanto mais longe mais ele se expande, mas também,
como se descobriu faz pouco tempo, que essa expansão está aumentando...

5. Exercı́cios
Exercı́cio 5.1. A seguir dado ǫ > 0 determine δ > 0 (em função de ǫ) tal que
|x − x0 | < δ implique |f (x) − L| < ǫ:

a): x0 = 1, f (x) = 555x, L = 555,


CAPÍTULO 5. LIMITES DE FUNÇÕES DEFINIDAS EM INTERVALOS 69

b): x0 = 0, f (x) = x2 , L = 0,

c): x0 = 0, f (x) = 555x2 , L = 0.


Exercı́cio 5.2.
1

0,5

x
0 10 20 30 40 50
0

-0,5

-1

A figura mostra o gráfico da função f : R>0 → (−1, 1) dada por


x−1
f (x) = .
x+1
Prove aquilo que é sugerido pelo gráfico, ou seja, que
lim f (x) = −1 e lim f (x) = 1.
xց0 x→+∞

Exercı́cio 5.3. Determine:


2
a): limx→2 x +5x+6
x+2
,
1
b): limx→2 (x−2)2
,
−1
c): limx→−6 (x+6)2
,
−1
d): limxր−6 x+6
,
−1
e): limxց−6 x+6
.
Exercı́cio 5.4. Considere os seguintes limites
x3 − 3x + 2 x3 − 3x + 2
lim e lim .
x→1 x−1 x→1 (x − 1)2
i) Antes de fazer contas, diga qual a diferença qualitativa que há entre os dois
casos.
ii) Calcule os limites.
iii) será que existe o
x3 − 3x + 2
lim ?
x→1 (x − 1)3
5. EXERCÍCIOS 70

Exercı́cio 5.5. Calcule


x3 − 2x2 − 4x + 8 x3 − 2x2 − 4x + 8
lim e lim .
x→1 x−2 x→1 (x − 2)2
Exercı́cio 5.6. i) Considere a função f : R → R definida por partes:
f (x) = −x, se x < −1,
f (x) = x2 + x + 1, se − 1 ≤ x ≤ 1,
f (x) = 2 · x, se 1 < x.
Existem os limites lim f (x) ou lim f (x)?
x→−1 x→1

ii) Ajuste os parâmetros b, c para que g : R → R definida por partes:


g(x) = −x, se x < −1,
2
g(x) = x + b · x + c, se − 1 ≤ x ≤ 1,
g(x) = 2 · x, se 1 < x.
tenha ambos os limites lim g(x) e lim g(x)
x→−1 x→1
CAPı́TULO 6

A noção de Continuidade

Na Definição a seguir pediremos um pouco mais que o que foi exigido na Definição
0.1, pois vamos pedir que:
• x ∈ I (domı́nio da função) e que
• limx→x f (x) = f (x)
ou seja que o limite L da função coincida com f (x):
Definição 0.1. Uma função f : I → R é contı́nua em x ∈ I se toda sequência xn de
pontos de seu domı́nio com
lim xn = x
n→+∞
tenha também
lim f (xn ) = f (x).
n→+∞
Quando dissermos apenas que f é contı́nua estamos querendo dizer f que é contı́nua
em cada ponto de seu Domı́nio.

Observações:
• Quer dizer então que, se uma função é contı́nua em x, é porque ela manda
todas sequências contidas no Domı́nio I de f que se aproximam de x em
sequências no Contra-Domı́nio que se aproximam de f (x).
• Concluı́mos que, para não termos a continuidade de f em x ∈ I, tem
que haver pelo menos uma sequência xn de pontos de seu domı́nio com
limn→+∞ xn = x, mas para as qual limn→+∞ f (xn ) 6= f (x) .
Isso pode acontece ou porque simplesmente não existe esse limite ou,
mesmo existindo, pode ser que seja diferente de valor esperado f (x).
• Só faz sentido dizer que f é descontı́nua (não-contı́nua) em pontos x de seu
Domı́nio1
Exemplos de descontinuidades:
1- f : R → R definida condicionalmente por: f (x) = x se x ≤ 0 e por x + 4 se
x > 0. Nesse exemplo, sequências xn < 0 que tendem a zero tem f (xn ) tendendo a
0; mas sequências xn > 0 que tendem a zero tem f (xn ) tendendo a 4.
2- f : [0, 5] → R, definida condicionalmente por f (0) = 3 e f (x) = 1/x, se
x ∈ (0, 5]. Aqui, sequências de números positivos xn que tendam a 0 tem f (xn )
ficando tão grande quanto quisermos, ou seja se afastando de f (0) := 3.
1Ao contrário do que faz o Anton em seu livro de Cálculo, para quem f : R \ {0} → R é
descontı́nua em x = 0 !!!
71
1. OPERAÇÕES COM FUNÇÕES CONTÍNUAS 72

3- f : [0, π1 ] → R, f (0) = 0 e f (x) = sen(1/x), se x ∈ (0, π1 ] (aqui apelo apenas


para o conhecimento de base, de que seno é uma função periódica, que tem valores
em [−1, 1] e que se anula em π). Aqui se tomamos xn > 0 conveniente tendendo a 0,
podemos conseguir f (xn ) tendendo para qualquer Lxn ∈ [−1, 1].

0,5

x
0,05 0,1 0,15 0,2 0,25 0,3
0

-0,5

-1

Figura: O gráfico de f (0) = 0 e f (x) = sin( x1 ) se x ∈ (0, π1 ].

1. Operações com funções contı́nuas


O próximo Teorema simplesmente re-escreve alguns itens do Teorema 1.1, no caso
em em x está no domı́nio de ambas as funções e em que L1 = f (x) e L2 = g(x).
Teorema 1.1. (Propriedades das funções contı́nuas) Suponha que f e g ambas são
contı́nuas em x, ou seja:
lim f (x) = f (x) e lim g(x) = g(x).
x→x x→x

Então:
1) A função soma f + g é também contı́nua em X ou seja
lim (f + g)(x) = (f + g)(x).
x→x

2) A função diferença f − g é também contı́nua em X ou seja


lim (f − g)(x) = (f − g)(x).
x→x

3) Se C ∈ R é uma constante, então a função (C · f )(c) := C · f (x) é contı́nua,


ou seja:
lim (C · f )(x) = C · f (x)
x→x
4) A função produto (f · g)(x) tem
lim (f · g)(x) = (f · g)(x).
x→x

5) Se g(x) 6= 0:
• i) se x é suficientemente próximo de x, então g(x) 6= 0 e
• ii) lim fg(x)
(x)
= fg(x)
(x)
.

A Afirmação 3.1 e a definição de função contı́nua implicam:


CAPÍTULO 6. A NOÇÃO DE CONTINUIDADE 73

Afirmação 1.1. (Princı́pio de Inércia das funções contı́nuas) Seja f : I → R


contı́nua em x, definida num intervalo aberto I.
• se f (x) > 0 então f (x) > 0 num intervalo aberto centrado em x.
• se f (x) > 0 então f (x) > 0 num intervalo aberto centrado em x.
Deixo a prova como um exercı́cio para o leitor, se bem que a figura a seguir diz
quase tudo:

L+ ε

L>0

L−ε

x
x −δ x +δ

Figura: f é contı́nua e positiva m x.

O Teorema a seguir é enunciado para a composição de 2 funções, mas pode ser


adaptado facilmente para qualquer número (finito) de composições de funções.
Afirmação 1.2. Seja g : I → J e f : J → K funções de intervalos em intervalos.
Suponha que g é contı́nua em x e que f é contı́nua em g(x). Então a função
composta
(f ◦ g)(x) := f (g(x))
é contı́nua em x.

Se g e f são contı́nuas, então f ◦ g é contı́nua.


Demonstração.
Queremos saber se para qualquer sequência (xn )n que tende a x, com xn ∈ I,
temos que a sequência f (g(xn )) ∈ K tende para f (g(x)).
O que sabemos pelas hipóteses sobre f e sobre g é, primeiro, que se xn ∈ I tende
a x então g(xn ) ∈ J tende a g(x).
Mas agora consideramos
z := g(x), e zn := g(xn ).
Essa sequência zn é uma sequência que tende a z. Pela hipótese de continuidade da
f , temos que f manda a sequência zn em uma sequência f (zn ) = f ( g(xn ) ) que tende
a f (z) = f (g(x)): exatamente o que querı́amos.


Na prática a Afirmação 1.2 permite-nos fazer a seguinte troca:


lim f ( g(xx ) ) = f ( lim g(xx ) ),
x→x x→x
2. POLINÔMIOS, FUNÇÕES RACIONAIS E TRIGONOMÉTRICAS 74

o que é muito útil para calcular limites.

2. Polinômios, funções racionais e trigonométricas


2.1. Polinômios.
Não imagino um exemplo mais simples de função contı́nua que a função constante
: f (x) ≡ C, C ∈ R. É claro que limx→x f (x) = C, pois f (x) = C simplesmente não
depende de x ou de x particulares.
Outro exemplo que é contı́nua é a função identidade f (x) = x, pois obviamente
lim f (x) = lim x = x.
x→x x→x

Uma consequência do Teorema 1.1 é que os polinômios:


f (x) := an · xn + an−1 · xn−1 + . . . + a1 · x + a0 , onde ai ∈ R
são funções contı́nuas. De fato, para um polinômio usamos um número finito de vezes
os itens 1), 2) , 3) e 4).

2.2. Funções racionais.


O item 5) do Teorema 1.1 diz então que a função F : R \ {0} :→ R, F (x) = x1 é
contı́nua, pois numerador e denominador são contı́nuos.
Isso é um pouco chocante, pelo aspecto do gráfico dessa, formado de duas partes.
Se lê em alguns livros que uma função contı́nua não tem rasgos no seu gráfico, mas
o correto é dizer que uma função contı́nua não introduz rasgos. Se o próprio domı́nio
dela já é formado como neste exemplo de dois pedaços como o de x1 ,
R \ {0} = R>0 ∪ R<0
então o gráfico pode ter dois pedaços, só não poder ter mais de dois pedaços.
O que sempre ficaria descontı́nua é qualquer tentativa de estender f (x) = x1 ao
ponto x = 0, pois se aproximando x pela direita 1/x > 0 fica tão positivo quisermos
e aproximando x pela esquerda 1/x < 0 fica tão negativo quanto quisermos.

Generalizando o exemplo x1 , defino uma função racional como o quociente PP12 (x) (x)
de dois polinômios. Resta saber, se adotamos esta definição, onde a função racional
está bem definida como função.
Vale o seguinte: se P1 (x) e P2 (x) não têm raı́zes comuns, então PP12 (x)
(x)
tem como
Domı́nio exatamente o conjunto
{ x ; P2 (x) 6= 0 }.
P1 (x)
E é uma função contı́nua.
P2 (x)
Porém, suponha que P1 (x) e P2 (x) têm alguma raı́z comum x, que é de ordem
m1 ≥ 1 para P1 (x) e de ordem m2 ≥ 1 para P2 (x). Então PP12 (x)
(x)
estará definida em x
se e somente se
m1 ≥ m2 .
Relembro essas noção de ordem ou multiplicidade de uma raı́z:
CAPÍTULO 6. A NOÇÃO DE CONTINUIDADE 75

Definição 2.1. Seja f (x) polinômio a coeficientes Reais.


Dizemos que x é raı́z de ordem exatamente m, se

f (x) = (x − x)m · g(x), m ∈ N,

para um g(x) polinômio a coeficientes Reais que não se anula em x.

2.3. Trigonométricas.
Considere agora um cı́rculo de raio 1.
Podemos usar o comprimento do arco do cı́rculo (medido no sentido antihorário
desde o eixo x > 0) como uma medida do ângulo central.
Assim um ângulo de 360 graus (antihorário, desde o eixo x > 0)) mede +2π (onde
π é tomado no sentido elementar de quociente entre o perı́metro e diâmetro de um
cı́rculo). Um ângulo de 90 graus antihorário mede +π/2, o de 180 antihorário mede
+π. É claro que há sempre uma ambiguidade de k · 2π nesse modo como medimos o
ângulo central.
A medida da projeção no eixo y (orientada como o eixo y) do arco de comprimento
θ é o seno do ângulo θ. Assim como a medida da projeção no eixo x (orientada como
o eixo x) do arco de comprimento θ é o cosseno do ângulo θ.

tan θ
senθ
θ

1 cos θ

Figura: Definição elementar de seno e cosseno

Seno e cosseno naturalmente são periódicos de perı́odo 2π, devido à ambiguidade


na medida do ângulo.
Agora vamos usar a intuição que temos de que, se variamos um pouquinho o arco
θ para θ + h, então as duas projeções vertical e horizontal mudam pouco (as projeções
são funções contı́nuas).
Ou seja, Afirmamos que seno e cosseno são funções contı́nuas por serem definidas
a partir de projeções.
Lembro que seno retrito a [ −π , π ] é uma função estritamente crescente; sua função
2 2
inversa chamada de arcoseno (pois diz de que arco o número dado é um seno) também
é estritamente crescente.
Isso vale em geral:

Se uma função y = f (x) é estritamente crescente, sua inversa x = f −1 (y) também


é.
2. POLINÔMIOS, FUNÇÕES RACIONAIS E TRIGONOMÉTRICAS 76

De fato, se por absurdo ocorresse que y 1 < y 2 mas f −1 (y 1 ) ≥ f −1 (y 2 ) então


terı́amos x1 = f −1 (f (x1 )) ≥ f −1 (f (x2 )) = x2 contradizendo que y = f (x) é estrita-
mente crescente.
sin(x)
Pelo item 5) do Teorema 1.1, a função cos(x) é contı́nua nos pontos onde cos(x) 6= 0,
ou seja para x 6= π/2 + k · π, k ∈ Z. Essa função é por definição a função tangente
sin(x)
tan(x) := .
cos(x)
Será importante mais adiante, quando falarmos dos coeficientes angulares de retas.
A periodicidade do seno do cosseno repercute na função tangente, que é periódica
de perı́odo π. Seu domı́nio é uma união de infinitos intervalos de comprimento π:
−π π −π π −π π
...∪( − π, − π) ∪ ( , )∪( + π, + π) ∪ . . .
2 2 2 2 2 2
e não é difı́cil de ver que quando restrita a cada intervalo ela é uma função:
• i) estritamente crescente e
• ii) que fica em módulo tão grande quanto quisermos se nos aproximamos
suficentemente dos extremos
sin(θ)
pois o denominador cos(θ) de cos(θ) se aproxima de zero enquanto o numerador sin(θ)
se aproxima de 1 ou de −1.

0
-1-0,5
0 0,51
x

-2

-4

Figura: Gráfico feito no computador de y = tan(x) em ( −π


2
+ 0.2, π2 − 0.2)

Nessa Figura, feita numericamente no computador, não pude pedir para o com-
putador trabalhar no intervalo ( −π , π ), pois os valores de tan explodem em módulo.
2 2
A restrição
−π π
tan : ( , )→R
2 2
tem uma inversa arctan : R → ( −π
2
, π2 ). Também é uma função estritamente crescente,
como já explicamos acima, mas seus valores não sobrepassam em módulo a π2 .
CAPÍTULO 6. A NOÇÃO DE CONTINUIDADE 77

1
0,5
0
-4 -2 -0,5 0 2 4
-1x

Figura: Gráfico de arctan(x)

Podemos expressar o comportamento de arctan(x) usando a notação da Seção 3:



π
lim arctan(x) =
x→+∞ 2
π
para dizer que arctan(x) fica tão próximo quanto quisermos de 2
se deixarmos
x crescer o suficiente;

π
lim arctan(x) = −
x→−∞ 2
para dizer que arctan(x) fica tão próximo quanto quisermos de − π2 se deixar-
mos x decrescer o suficiente;
E podemos introduzir novos sı́mbolos para comparar com o comportamento de
tan(x):

lim tan(θ) = −∞
θց− π2

significa que tan(θ) fica tão negativo quanto quisermos desde que θ > − π2
decresça e se aproxime o suficiente de − π2 .

lim tan(θ) = ∞
θր π2

π
significa que tan(θ) fica tão positivo quanto quisermos desde que θ < 2
cresça
e se aproxime o suficiente de π2 .
3. CONTINUIDADE DA FUNÇÃO INVERSA 78

3. Continuidade da função inversa


É possı́vel provar (mas a prova é um pouco técnica demais) que:
Afirmação 3.1. Se f : I → R, y = f (x) definida num intervalo I é contı́nua e
tem inversa, então f −1 : f (I) → I também está definida num intervalo f (I) e f −1
também é contı́nua.
Chamo a atenção que essa Afirmação pode ser falsa se o domı́nio da f não é um
intervalo2
Para ver um exemplo disso, considere uma f definida numa união de intervalos:
[0, a] ∪ (a + 1, b], que seja contı́nua e que tenha inversa. Note que a continuidade em
x = a só se refere ao comportamento a f em relação a sequências xn ∈ [0, a] que
tendam a x = a. As sequências xn ∈ (a + 1, b] do domı́nio da f não tendem ao ponto
a, pois distam dele pelo menos 1, então não interessam na análise da continuidade da
f em a. O gráfico que segue é um exemplo de uma tal f :

y = f(x)

0 a a+1 b

Figura: f : [0, a] ∪ (a + 1, b] → R contı́nua,


com x = f −1 (y) descontı́nua em f (a)

Agora Afirmo que a função inversa x = f −1 (y) é descontı́nua em y = f (a). De


fato, se yn < f (a) é uma sequência de pontos da imagem da f que tende a f (a) vemos
na Figura que limn→+∞ f −1 (yn ) = a. Mas se tomamos yn > f (a) uma sequência de
pontos da imagem da f que tende a f (a), vemos que limn→+∞ f −1 (yn ) = a + 1.
A Figura a seguir ilustra:

y = f^{−1} (x)

y = f(x)

0 a a+1 b

Figura: Aqui y = f (x) e y = f −1 (x) estão no mesmo sistema cartesiano

2Como esqueceu o Anton, na pag. 156, Teorema 2.6.2, da Oitava Edição do seu livro de Cálculo.
CAPÍTULO 6. A NOÇÃO DE CONTINUIDADE 79

4. Dois teoremas fundamentais sobre funções contı́nuas


A demonstração dos dois Teorema a seguir foge do conteúdo usual do Cálculo,
é visto em disciplinas mais avançadas de Análise Matemática.
É importante que o estudante medite sobre seus enunciados.

Teorema 4.1. (Teorema do Valor Intermediário - abrev.: T.V.I.)


Seja f : [a, b] → R função contı́nua com A = f (a) e B = f (b), com A 6= B, por
exemplo A < B.
Seja C qualquer número C ∈ (A, B). Então existe algum x ∈ (a, b) tal que
f (x) = C (pode haver mais de um x desse tipo)
Teorema 4.2. (Teorema de Bolzano-Weierstrass)
Seja f [a, b] → R contı́nua, onde [a, b] é intervalo fechado e limitado. Então f tem
mı́nimo e máximo globais assumidos em pontos de [a, b]

5. Primeiras aplicações do T.V.I

Vamos dar agora algumas aplicações iniciais do T.V.I. Mais tarde ele será impor-
tante na prova do Teorema Fundamental do Cálculo, na Parte 2 do Curso.
Primeiro um tı́pico teorema bem geral, mas que não diz nada sobre a solução em
cada caso especı́fico:
Proposição 5.1. Dado qualquer f : [0, 1] → [0, 1] contı́nua, existe x ∈ [0, 1] tal que
f (x) = x.
Demonstração.
Observe que geometricamente o que queremos é saber se o gráfico de y = f (x)
corta o gráfico da diagonal y = x.
Se f (0) = 0 ou se f (1) = 1 então corta e acabou, não há nada mais a provar.
Portanto vamos supor que f (0) ∈ (0, 1] e que f (1) ∈ [0, 1), para termos algo a provar.
É razoável olhar a função diferença entre elas: f (x) − x. Por ser uma diferença de
duas funções contı́nuas, f (x) − x também é função contı́nua. Ademais, f (0) ∈ (0, 1]
e f (1) ∈ [0, 1) dizem que:
f (0) − 0 > 0 e f (1) − 1 < 0.
Pelo T.V.I. existe algum x ∈ (0, 1) tal que:
f (x) − x = 0,
como querı́amos. 

6. Raı́zes de polinômios cujo grau é ı́mpar


A segunda aplicação do T.V.I.:
Proposição 6.1. Todo polinômio de coeficientes Reais e de grau ı́mpar tem algum
zero Real: f (x) = 0.
6. RAÍZES DE POLINÔMIOS CUJO GRAU É ÍMPAR 80

Observe que há polinômios de grau par sem zeros Reais, como f (x) = x2 + 1.
Demonstração. Seja f o polinômio de grau 2n − 1:
f (x) := a2n−1 · x2n−1 + a2n−2 · x2n−2 + . . . + a1 · x + a0 , ai ∈ R, n∈N
Caso a2n+1 > 0:
Escrevo para x > 0:
a2n−2 a0
a2n−1 · x2n−1 + a2n−2 · x2n−2 + . . . + a1 · x + a0 = a2n−1 x2n−1 · (1 + + . . . 2n−1 ).
x x
Pelo Teorema 3.1 e pelos Exemplos que o seguem, temos que
a2n−2 a0
lim ( + . . . 2n−1 ) = 0.
x→+∞ x x
Portanto para x > 0 suficientemente grande temos que
a2n−2 a0
1+ + . . . 2n−1 > 0.
x x
Logo, para x > 0 suficientemente grande, o sinal de
a2n−2 a0
a2n−1 x2n−1 · (1 + + . . . 2n−1 )
x x
2n−1 2n−1
é o mesmo sinal de a2n−1 x , que é a2n−1 x > 0.
Argumentando do mesmo jeito para x → −∞, concluimos que o sinal de
a2n−2 a0
a2n−1 x2n−1 · (1 + + . . . 2n−1 )
x x
para x < 0 suficientemente grande é o mesmo sinal de a2n−1 x2n−1 , que nesses pontos
é a2n−1 x2n−1 < 0.
Então
f (x) = a2n−1 · x2n−1 + a2n−2 · x2n−2 + . . . + a1 · x + a0
assumiu valores negativos e positivos.
Pelo T.V.I. e pela continuidade do polinômio f (x), tem que haver um ponto onde
f (x) = 0.
Caso a2n+1 < 0: completamente análogo.


Esse teorema (e sua prova) não dão nenhuma pista de como achar concretamente
algum ponto x onde f (x) = 0.
Em dois trabalhos, de 1690 e 1691, Michel Rolle tentou estabelecer um método
para determinar concretamente esses zeros.
Ele o fez de um modo bem confuso, pois não tinha uma boa definição de Derivada,
mas seu nome ficou associado ao teorema que estabeleceremos mais adiante no Capı́tulo
10 e que nos permitirá criar métodos para encontrar raı́zes de polinômios (e de funções
mais gerais).
Um aplicação interessante do Teorema de Rolle e do T.V.I. será dada na Seção 5
do Capı́tulo 13, para provar a Regra de sinais de Descartes, que dá uma estimativa
do número de raı́zes Reais de um polinômio.
CAPÍTULO 6. A NOÇÃO DE CONTINUIDADE 81

7. Raı́zes simples e fatoração de polinômios


Acho que pode ser útil na formção dos estudantes, ter uma prova do seguinte fato
fundamental:
Teorema 7.1. Seja f (x) = an xn + an−1 xn−1 + . . . + a0 um polinômio de grau n, com
coeficientes ai ∈ R.
São equivalentes:
• i) f (x) = 0 para alguma raı́z x ∈ R e

• ii) f (x) = (x − x) · g(x) onde g(x) é um polinômio de grau n − 1 com


coeficientes Reais.

Demonstração.
ii) obviamente implica i), pois:
f (x) = (x − x) · g(x) = 0.
A prova de que i) implica ii) será dividida em duas etapas.
A parte interessante é construir o g(x) que queremos em:
f (x) = (x − x) · g(x) + r,
onde r é uma constante.
Se tivermos feito isso, avaliaremos tudo em x:
0 = f (x) = (x − x) · g(x) + r = r,
para concluir que r = 0.
Para chegarmos na desejada expressão f (x) = (x−x)·g(x)+r, temos um algoritmo
a executar.
Para f (x) = an xn + an−1 xn−1 + . . . + a0 , faço
g1 (x) := an · xn−1
e subtraio
r1 (x) := f (x) − (x − x) · g1 (x).
O g1 (x) foi escolhido para que r1 (x) não tenha termo de grau n. Ou seja que esse
novo polinômio r1 (x) tem grau ≤ n − 1. Se por acaso r1 (x) ≡ 0 então
f (x) = (x − x) · g1 (x)
e já temos o que queremos, com r = 0 e g(x) := g1 (x).
Caso contrário r1 (x) = bk xk + bk−1 xk−1 + . . ., onde k ≤ n − 1; defino
xk−1
g2 (x) := ,
bk
e subtraio
r2 (x) := r1 (x) − (x − x) · g2 (x).
7. RAÍZES SIMPLES E FATORAÇÃO DE POLINÔMIOS 82

Pela definição do g2 (x) esse novo polinômio r2 (x) tem grau ≤ n − 2. Se dermos sorte
e r2 (x) ≡ 0 então
f (x) = (x − x) · [g1 (x) + g2 (x)],
e já temos o que queremos com r = 0 e g(x) = g1 (x) + g2 (x).
Caso contrário continuamos, considerando agora r2 (x) = cj xj + cj−1xj−1 + . . .,
onde j ≤ n − 2 e definindo g3 (x) e r3 (x) como fizemos antes.
O que importa é que o grau desse novo r3 (x) será ≤ n − 3. Ou seja, como vão
caindo os graus dos rk (x) a cada etapa, após no máximo n etapas chegaremos a um
rk (x) (k ≤ n) que ou bem é ≡ 0 ou bem tem grau zero, uma constante. Esse será o
r. E g(x) := g1 (x) + . . . + gk (x), k ≤ n. 

Digressão sobre o Teorema 7.1:


Se observarmos a prova desse Teorema vemos que, na fatoração
f (x) = (x − x) · g(x)
os coeficientes do polinômio g(x) são soma, subtrações, produtos, quocientes da raı́z
x e dos coeficientes ai de f (x).
Por isso, se a raı́z x fossse um número Complexo e a1 são Reais ou Complexos, de-
veria haver uma fatoração de f onde o polinômio g(x) tivesse coeficientes Complexos.
Por exemplo, temos
x3 − 1 = (x − 1) · (x2 + x + 1)
e isso é tudo que podemos fazer se estamos limitados a trabalhar com coeficientes
Reais.
Mas x2 + x + 1 tem raı́zes Complexas:
√ √ √ √
−1 − −1 3 −1 + −1 3
x1 := e x2 := ,
2 2
ous seja, as raı́zes Reais ou Complexas de x3 − 1 = 0 são 1, x1 , x2 . Portanto deveria
haver uma fatoração:
x3 − 1 = (x − x1 ) · g(x),
com os coeficientes desse novo g(x) nos Complexos.
Seguindo os passos do algoritmo dado na prova do Teorema 7.1 (com a mesma
notação), faço:
g1 (x) := x2
r1 := x3 − 1 − x2 · (x − x1 ) =
= x1 x2 − 1.
Agora
g2 (x) := x1 x,
r2 := r1 − x1 x · (x − x1 ) =
= x21 x − 1.
E também
g3 (x) := x21 ,
CAPÍTULO 6. A NOÇÃO DE CONTINUIDADE 83

r3 := r2 − x21 · (x − x1 ) =
= −1 + x31 = 0.
Portanto
g(x) := g1 (x) + g2 (x) + g3 (x) =
= x2 + x1 x + x21 ,
e a fatoração é
√ √
3 2 −1 − −1 3
x − 1 = (x − x1 ) · ( x + x1 x + x21 ), onde x1 := .
2
Note que:
(x − 1) · (x − x2 ) = x2 − (x2 + 1) x + x2 =
= x2 + x1 x + x21 ,
pois claramente
x2 + 1 = −x1 ,
e
x21 = x2 .
8. Possı́veis raı́zes Racionais de polinômios a coeficientes inteiros
Aproveito o tema das raı́zes de polinômios para lembrar o seguinte Teste, que
permite saber se pode haver raı́z Racional de um polinômio a coeficientes Inteiros:
Afirmação 8.1. Seja p(x) = ak · xk + ak−1 · xk−1 + . . . + a1 · x + a0 polinômio de grau
k ≥ 1 com coeficientes Inteiros:
ak , ak−1, . . . , a1 , a0 ∈ Z.
Suponha que p(x) tem alguma raı́z Racional, ou seja, da forma
m
x= ∈ Q, com m e n primos entre si.
n
Então m é divisor de a0 e n é divisor de ak .
Demonstração.
Suponho que:
m mk mk−1 m
p( ) = ak · k + ak−1 · k−1 + . . . + a1 · + a0 = 0.
n n n n
Então
mk mk−1 m
ak · k
+ ak−1 · k−1
+ . . . + a1 · = −a0
n n n
e multiplicando por nk :
ak · mk + n · ak−1 · mk−1 + . . . + a1 · nk−1 · m = −nk · a0
e daı́:
m · [ak · mk−1 + n · ak−1 · mk−2 + . . . + a1 · nk−1 ] = nk · (−a0 ).
Como
ak · mk−1 + n · ak−1 · mk−2 + . . . + a1 · nk−1 ∈ Z
temos que m é um divisor de nk · (−a0 ).
9. EXERCÍCIOS 84

Como m e n são primos entre si isso implica que m é divisor de a0 .


Também temos:
mk mk−1 m
−ak · k = ak−1 · k−1 + . . . + a1 · + a0
n n n
k
e portanto, multiplicando por n :
−ak · mk = n · ak−1 · mk−1 + . . . + nk−1 · a1 m + nk · a0
e daı́:
−ak · mk = n · [ak−1 · mk−1 + . . . + nk−2 · a1 · m + nk−1 · a0 ].
Como
ak−1 · mk−1 + . . . + nk−2 · a1 · m + nk−1 · a0 ∈ Z
isso diz que n é divisor de −ak · mk . Como m e n são primos entre si, isso implica
que n é divisor de ak .


Na Seção 5 do Capı́tulo 13 daremos uma prova da Regra de Sinais de Descartes,


que estima quantos zeros pode ter um polinômio a coeficientes Reais.

9. Exercı́cios
Exercı́cio 9.1. Considere a função definida assim: f (x) = 0 se x é um número
racional e f (x) = 1 se x é um número irracional.

i): Como é seu gráfico ?


ii): em que pontos ela é contı́nua ou é descontı́nua?
Exercı́cio 9.2. A soma, o produto e a composição de funções contı́nuas produz
funções contı́nuas. Usando isso calcule:
i) lim (3x − 4x) · (x5 − 2x)4 ,
x→1

ii) lim 4x − 3x · (x5 − 2x)4 .
x→1

Exercı́cio 9.3. Dê um exemplo de f (x) descontı́nua em algum ponto mas tal que
f 2 (x) é contı́nua em todos os pontos.
Exercı́cio 9.4. (resolvido)
Prove que a função definida por f (x) = x · sin( x1 ), se x > 0 e f (0) = 0 é contı́nua.
Exercı́cio 9.5. Prove a Afirmação 1.1, que chamei de princı́pio de inércia das funções
contı́nuas.
Exercı́cio 9.6. Um aluno me disse que, para descobrir em quais intervalos um
polinômio y = f (x) de grau n é positivo ou negativo, ele faz o seguinte.
Ele primeiro descobre todas as raı́zes Reais x1 , x2 , . . . , xk , onde k ≤ n.
Depois considera os intervalos (−∞, x1 ), (x1 , x2 ), etc , (xk−1 , xk ), (xk , +∞). Então
para saber o sinal de f em cada intervalo desses, ele examina o sinal de f (x) em um
único x de cada intervalo.
CAPÍTULO 6. A NOÇÃO DE CONTINUIDADE 85

O método dele está correto ? Se está, justifique-o com conceitos/ teoremas do


Cálculo.

Exercı́cio 9.7. Dê um exemplo de uma função f positiva em um ponto x, mas tal
que f (xn ) = 0 em pontos xn que formam um sequência com limn→+∞ xn = x.

Exercı́cio 9.8. Encontre o domı́nio da função racional f (x) = x21−1 . Descreva o que
acontece com o módulo e o sinal de f quando x se aproxima pela esquerda e pela
direita dos pontos onde ela não está definida.

Exercı́cio 9.9. (resolvido)


i) Prove que

5 · x2 + x √
lim = 5
x→+∞ x+2

2,2

1,8

1,6

1,4

1,2

0,8
20 40 60 80 100
x


5·x2 +x

Figura: Gráfico de y = x+2
, x ∈ [1, 100], 5 ≈ 2.23.
ii) Prove que

5 · x2 + 2 √
lim =− 5
x→−∞ x+2

Exercı́cio 9.10. (resolvido) Um exemplo que não parece estar ligado a quocientes,
mas que se calcula introduzindo quocientes:

√ 1
lim ( x2 + x − x ) = .
x→+∞ 2
9. EXERCÍCIOS 86

0,5

0,48

0,46

0,44

0,42

20 40 60 80 100
x

Figura: Gráfico de y = x2 + x − x, x ∈ [1, 100].

Exercı́cio 9.11. É um fato que o polinômio


y = x5 − 2x4 + x3 + x2 + 1
só tem uma raı́z Real. Não é fácil achá-la explicitamente. Mas com o Teorema do
Valor Intermediário você pode concluir que a raı́z Real é um ponto do intervalo [−1, 1].
Por quê ?
No Capı́tulo 18 daremos um método para determinar essa raı́z, que foi descoberto
por Newton (para variar ...)
Exercı́cio 9.12. (resolvido)
A equação x3 + 1 = 0 e, em geral, as as equações de grau ı́mpar
x2n+1 + 1 = 0, n∈N
tem obviamente como única raı́z Real o x = −1.
Não é fácil resolver explicitamente a equação x3 + ǫ · x + 1 = 0, com ǫ ≥ 0 fixado,
a menos que se conheça a fórmula de Cardano; com ela se obtém a raı́z Real
s r s r
3 1 1 ǫ 3 3 1 1 ǫ3
x= − + + − + + .
2 4 27 2 4 27
Torna-se intratável tentar resolver explicitamente o seguinte tipo de equação de
grau ı́mpar:
x2n+1 + ǫ1 · x2n−1 + ǫ2 · x2n−3 + . . . + ǫn−1 · x3 + ǫn · x + 1 = 0,
com
ǫi ≥ 0, i = 1, . . . n − 1 e ǫn > 0
fixados.
i) Prove que cada uma dessas equações têm um única raı́z Real.
ii) Prove que a raı́z de cada uma delas está em [−1, 0).
iii) Para cada número em [−1, 0) encontre alguma dessas equações que o tenha
como única raı́z.
CAPı́TULO 7

Geometria Analı́tica Plana

1. Equações de retas, coeficientes angular e linear


A equação de uma reta vertical por dois pontos (x, y1 ) e (x, y 2 ) é
x − x = 0.
Mas a equação de uma reta não-vertical por (x1 , y 1 ) e (x2 , y 2 ) é do tipo:
y = a1 · x + a0 , a1 , a0 ∈ R.
Ou seja, sua equação é um tipo bem simples de polinômio, cujo grau em x é ≤ 1.
Vamos usar uma notação mais habitual:
y = a · x + b, a, b ∈ R.
Afirmação 1.1. Os coeficientes a, b da equação y = ax + b da reta passando pelos
dois pontos (x1 , y 1 ) e (x2 , y 2 ) com x1 6= x2 são dados por:
y2 − y1
a= ,
x2 − x1
e
b = y 1 − a · x1 = y 2 − a · x2 .

Demonstração. De
y 1 = a · x1 + b e y 2 = a · x2 + b,
subtraindo-as, obtemos:
y 2 − y 1 = a · (x2 − x1 ),
de onde
y2 − y1
a= ,
x2 − x1
6 x1 ). E daı́ sai que:
(onde é crucial que x2 =
y − y1
b = y1 − ( 2 ) · x1 ,
x2 − x1
ou o que dá no mesmo:
y2 − y1
b = y2 − ( ) · x2 .
x2 − x1


87
1. EQUAÇÕES DE RETAS, COEFICIENTES ANGULAR E LINEAR 88

Note que esse número b é a altura em que a reta y = ax + b intersecta o eixo dos
y, que é dado por x = 0: de fato,
y = a · 0 + b = b.
Definição 1.1. Dados dois pontos distintos do plano (x1 , y 1 ) e (x2 , y 2 ) com coor-
denadas x1 6= x2 , definimos o coeficiente angular da reta ligando esses dois pontos
por:
y2 − y1 y − y2
= 1 .
x2 − x1 x1 − x2
Afirmação 1.2. O coeficiente angular é uma informação da reta, não dependendo
dos pontos particulares que usamos para calculá-lo.
Demonstração.
De fato, se tomo qualquer ponto (x3 , y 3 ) da reta y = a · x + b determinada por
(x1 , y 1 ) e (x2 , y 2 ), como y 3 = ax3 + b, então:
y3 − y1 (a · x3 + b) − (ax1 + b)
= = a,
x3 − x1 x3 − x1
e já vimos na Afirmação 1.1 que
y2 − y1
a= ,
x2 − x1
ou seja,
y3 − y1 y2 − y1
= .
x3 − x1 x2 − x1


Como consequência temos a seguinte observação útil para o Curso:


Afirmação 1.3. Dado um ponto (x1 , y 1 ) e um coeficiente angular pré-estabelecido
valendo a, então a única reta que passa por (x1 , y 1 ) e tem esse coeficiente angular é
dada por
y = a · x + (y 1 − a · x1 ).
Demonstração. De fato, tomando um ponto (x, y) genérico dessa reta, então
pela Afirmação 1.2
y − y1
= a,
x − x1
o que dá, isolando-se y:
y = a · x + (y 1 − a · x1 ).


Exemplos:
1)- a diagonal y = x tem coeficente angular 1 e a anti-diagonal y = −x tem
coeficiente angular −1.
2)- A reta horizontal y = b tem coeficiente angular 0, pois y = b = 0 · x + b.
CAPÍTULO 7. GEOMETRIA ANALÍTICA PLANA 89

Observações:

• Se x1 = x2 então a reta que liga (x1 , y 1 ) e (x2 , y2 ) é vertical e não tem um


coeficiente angular definido.
Temos a tentação de dizer que o coeficiente angular da reta vertical é
+∞. Mas se começamos com a anti-diagonal e a vamos levantando, os co-
eficientes angulares ficam cada vez mais negativos e ao atingir a posição
vertical ficariam −∞: essa ambiguidade entre +∞ e −∞ para o candidato
a coeficiente angular da reta vertical é que faz que seja melhor desistirmos
de atribuir um coeficiente angular à reta vertical.
• Geometricamente o coeficiente angular a representa o quociente entre o
cateto oposto y 2 − y 1 e o cateto adjacente x2 − x1 do triângulo retângulo
formado pelos pontos (x1 , y 1 ), (x2 , y 1 ) e (x2 , y 2 ): logo a = tan(α) ( tangente
do ângulo (anti-horário) α formado pela reta e o eixo horizontal). Vimos
na Seção 2.3 que se um ângulo que tende a +π 2
sua tangente tende a +∞,
enquanto que, se o angulo tende a −π 2
, sua tangente tende a −∞.
• Se fixamos a e variamos b em y = a · x + b estamos descrevendo uma famı́lia
de retas paralelas com a mesma inclinação.

2. Ortogonalidade

Deve estar claro pelo que já explicamos que duas retas y = ax + b1 e y = ax + b2 ,
com b2 6= b1 , são de fato paralelas.
Agora gostaria de explicar que uma par de retas y = ax + b1 e y = − a1 x + b2 , com
a 6= 0, são ortogonais.
Posso me restringir a considerar retas pela origem: y = ax e y = − a1 x, pois
estas são translações verticais das retas anteriores, e portanto têm entre elas o mesmo
ângulo que as anteriores. Posso supor também que a > 0 (caso a < 0 então − a1 > 0
e poderia trabalhar com este coeficiente angular).
Se escrevo a = B A
, com A, B > 0, então − a1 = − BA
.
Agora considero 3 triângulos (ilustrados na Figura a seguir):

• ∆1 dados pelos pontos (0, 0), (A, 0) e (A, B) e


• ∆2 dado pelos pontos (0, 0), (−B, 0) e (−B, A).
• ∆3 dado pelos pontos (0, 0), (A, B) e (−B, A).
3. TEOREMA DE TALES NO CÍRCULO 90

( A,B )

(−B , A )
∆3
∆1
∆2

(−B , 0) (0, 0) ( A, 0 ) x

Observe que ∆1 e ∆2 são triângulos retângulos e que a reta que contém a hipotenusa
de ∆1 é y = ax , enquanto que a reta que contém a hipotenusa de ∆2 é a reta y = − a1 x.

Então por Pitágoras as hipotenusas de ∆1 e de ∆2 valem o mesmo: A2 + B 2 .
Por outro lado o comprimento do segmento de reta ligando (−B, A) a (A, B) vale,
por definição:
p √
(B − A)2 + (A − (−B))2 = 2A2 + 2B 2 .

Portanto o triângulo ∆3 é isósceles, pois tem dois lados de mesmo tamanho λ :=



A2 + B 2 . Esses lados formam um ângulo em (0, 0) que denoto por α. E o terceiro
lado de ∆3 , oposto a α, mede
√ √
2A2 + 2B 2 = λ2 + λ2 .

Lembro agora que é válida a recı́proca do Teorema de Pitágoras (coisa pouco lembrada
no Ensino Médio), ou seja, se um lado maior de um triângulo é soma de quadrados de
outros dois lados menores, então o triângulo é retângulo no ângulo oposto ao maior
lado. Logo o triângulo ∆3 tem que ter ângulo reto em α, por ter um lado cuja medida
é λ2 + λ2 .
Logo y = ax e y = −1 a
x são de fato ortogonais, pois α é reto.

Apenas com as noções de coeficiente angular e de ortogonalidade é possı́vel provar


fatos bonitos e fundamentais da Geometria Euclidiana.
É o que faremos nas duas Seções seguintes.

3. Teorema de Tales no cı́rculo


Um dos mais bonitos teoremas da geometria Euclidiana é o Teorema de Tales no
Cı́rculo, que diz:

Afirmação 3.1. (Teorema de Tales)


Todos os ângulos inscritos no cı́rculo determinados pelo diâmetro são ângulos retos
(= π2 radianos).
CAPÍTULO 7. GEOMETRIA ANALÍTICA PLANA 91

Figura: O Teorema de Tales no Cı́rculo

Demonstração.
Vamos provar para pontos do Cı́rculo com coordenada y > 0 (para os outros é
análogo). √
Tome um ponto no do Cı́rculo de raio r > 0, de coordenadas (x, + r 2 − x2 ), onde
x ∈ [−r, r]. √
Queremos ver se os coeficiente angular a√da reta ligando (x, + r 2 − x2 ) a (r, 0) e
o coeficiente angular a′ da reta ligando (x, + r 2 − x2 ) a (−r, 0) satisfazem a condição
que expressa a ortognalidade:
a′ · a = −1.
Mas √ √
′ r 2 − x2 − 0 r 2 − x2
a = = ,
x − (−r) x+r

r 2 −x2
enquanto que a = x−r
e portanto:
√ √
′ r 2 − x2 r 2 − x2 r 2 − x2
a ·a= · = 2 = −1.
(x + r) (x − r) x − r2


4. A equação da reta de Euler


Um Teorema muito geral, que escapou de Euclides, mas não de Euler, é o seguinte:
Afirmação 4.1. (Reta de Euler)
Considere qualquer triângulo.
Se o triângulo não é equilátero, o Baricentro B, o Circuncentro C e o Ortocentro
H são pontos distintos mas são colineares. Ademais as distâncias entre eles verificam:
HB = 2 · BC.
Se o triângulo é equilátero, os três pontos coincidem num mesmo ponto.
Essa reta que contém esse três pontos é a reta de Euler.
4. A EQUAÇÃO DA RETA DE EULER 92

1,5

0,5

0
0 0,2 0,4 0,6 0,8 1

Figura: A reta de Euler representada por segmento intersectando


uma mediana, uma altura e uma mediatriz, para P = ( 32 , 2)

1,5

0,5

0
0 0,2 0,4 0,6 0,8 1

Figura: A reta de Euler representada por segmento intersectando


uma mediana, uma altura e uma mediatriz, para P = ( 51 , 2)

À medida que formos demonstrando esse fato iremos relembrando os conceitos


envolvidos. A demosntração dará as coordenadas explı́citas dos pontos e a equação
explı́cita da reta de Euler.

Demonstração.
Não perdemos muita generalidade se supusermos que o triângulo tem vértices:
(0, 0), (1, 0) e (A, B), B 6= 0,
pois isso se obtém escolhendo um sistema de coordenadas cartesiano adequado.
Os lados do triângulo fazem parte de três retas, das quais obviamente a primeira

l1 : y = 0.
CAPÍTULO 7. GEOMETRIA ANALÍTICA PLANA 93

A reta l2 é a que contém (0, 0) e (A, B), cuja equação é:


B
l2 : y = · x, se A 6= 0,
A
ou a reta vertical:
l2 : x = 0, se A = 0.
E a terceira é a que contem (1, 0) e (A, B), cuja equação é:
B B
l3 : y = ·x− , se A 6= 1
A−1 A−1
ou a reta vertical
l3 : x = 1, se A = 1.
Os pontos médios de cada lado do triângulo são:
1 A+1 B A B
( , 0), ( , ) e ( , ).
2 2 2 2 2
Considero agora as três medianas : retas ligando vértices a pontos médios dos
lados opostos.
A reta que liga (0, 0) a ( A+1
2
, B2 ) é
B
2 B
m1 : y= A+1
·x= · x, se A 6= −1,
2
A+1
ou a reta vertical
m1 : x = 0, se A = −1.
A reta que liga (1, 0) a ( A2 , B2 )

B B
m2 : y= ·x− , se A 6= 2,
A−2 A−2
ou a reta vertical
m2 : x = 1, se A = 2.
1
A reta que liga (A, B) a ( 2 , 0) é:
2B B 1
m3 : y= x− , se A 6=
2A − 1 2A − 1 2
ou a reta vertical:
1 1
x = , se A = .
m3 :
2 2
Supondo por um instante que estamos no caso geral, em que A 6= −1, 2, a intersecção
m1 ∩ m2 se obtem facilmente, resolvendo:
B B B
x= ·x−
A+1 A−2 A−2
que dá (usando B 6= 0):
A+1
x=
3
e portanto é
A+1 B
B := ( , ).
3 3
4. A EQUAÇÃO DA RETA DE EULER 94

Agora tratemos dos casos particulares que faltaram.


Se A = −1, então m1 ∩ m2 consiste na intersecção de x = 0 e y = − B3 x + B3 . Ou
seja é o ponto
B
(0, ),
3
que coincide com o B.
Se A = 2, então m1 ∩ m2 é dada por y = B3 x intersectada com x = 1, que dá o
ponto:
B
(1, ),
3
que coincide também com o B.
Agora Afirmo que
B ∈ m3 .
1
Se A 6= 2 então o fato ques eja verdade
2B A+1 B B
( )·( )− =
2A − 1 3 2A − 1 3
diz que B ∈ m3 .
Se A = 21 , então m3 é dada por x = 12 , que obviamente passa por
1
+1 B 2 1 B
B=( , ) = ( , ).
3 3 2 3
Esse ponto B, que em todos os casos possı́veis é
B = m1 ∩ m2 ∩ m3
é chamado Baricentro.
Considero agora as três mediatrizes: retas saindo de cada ponto médio em ângulo
reto com o lado.
A mediatriz pelo ponto médio ( 21 , 0) é fácil, é a reta:
1
md1 : x = .
2
A B
O lado que contém o ponto médio ( 2 , 2 ) está na reta l2 e essa reta ou é y = B
A
x,
se A 6= 0, ou a reta vertical x = 0 se A = 0.
Portanto mediatriz md2 pelo ponto médio ( A2 , B2 ) ou é horizontal
B
md2 : y= , se A = 0,
2
ou a reta:
A B A2
md2 : y=− ·x+( + ), se A 6= 0,
B 2 2B
(lembre que nunca B = 0).
Então md1 ∩ md2 é o ponto:
1 B
C: ( , ), se A = 0
2 2
ou
1 A · (A − 1) B
C: ( , + ), se A 6= 0.
2 2B 2
CAPÍTULO 7. GEOMETRIA ANALÍTICA PLANA 95

Afirmo agora que em qualquer caso:


C ∈ md3
onde md3 é a mediatriz do lado contendo om ponto médio ( A+1 2
, B2 ).
De fato, o lado está contido em l3 , cujas equações são:
B B
l3 : y = ·x− , se A 6= 1
A−1 A−1
ou a reta vertical
l3 : x = 1, se A = 1.
B
Portanto ou md3 é y = 2 no caso A = 1 e claramente passa por
1 B
C: ( , ),
2 2
ou
A−1 B A2 − 1
md3 : y=− ·x+ + , se A 6= 1,
B 2 2B
que passa também por
1 A · (A − 1) B
C=( , + ),
2 2B 2
como se vê em seguida.
Esse ponto C que verifica:
C = md1 ∩ md2 ∩ md3
é chamado Circuncentro (o Exercı́cio 8.7 ajudará a justificar essa nomenclatura).
Já podemos nos perguntar o que acontece se
B = C.
Isso ocorre quando:
A+1 1 B A · (A − 1) B
= e = + .
3 2 3 2B 2
1
A primneira dá A = 2 , que posta na segunda dá:
3
B2 = ,
4
√ √
ou seja B = 23 ou B = − 23 . √ √
Esse triângulo com (A, B) = ( 12 , 23 ) ou (A, B) = ( 12 , − 23 ) e com os outros vértices
em (0, 0) e (1, 0) é equilátero.
Agora consideremos as três alturas: retas que saem de vértices e são ortogonais
ao lado oposto.
Como veremos no Exercı́cio 8.6, se
P = (x, y) 6∈ r,
a reta P Q intersecta ortogonalmente r : y = ax + b em Q ∈ r com coordenadas
Q = (x, b) se a = 0
4. A EQUAÇÃO DA RETA DE EULER 96

ou coordenadas
x − a(b − y) x − a(b − y)
Q=( 2
, a·( ) + b ), se a 6= 0.
a +1 a2 + 1
A altura que sai de (A, B) e vai ortogonal até o lado l1 : y = 0 é portanto:
h1 : x = A.
A altura que sai de (0, 0) é:
h3 : y = 0, se A = 1,
pois nesse caso l3 : x = 1. Ou
A−1
h3 = − · x, se A 6= 1,
B
pois no caso geral
B B
l3 : y= ·x− .
A−1 A−1
A intersecção h1 ∩ h3 é portanto:
(1, 0), se A = 1
ou
A · (A − 1)
(A, − ), se A 6= 1.
B
Em qualquer caso,
A · (A − 1)
H = ( A, − ) = h1 ∩ h2 .
B
Afirmo que

H ∈ h2 ,
onde h2 é a altura que sai de (1, 0) e chega ortogonal a l2 .
Se l2 : x = 0 (quando A = 0) então
h2 : y=0
B
obviamente passa por H. E se l2 : y = A
· x (no caso A 6= 0) então:
A A
h2 : y = − ·x+ .
B B
Nesse caso também H ∈ h2 .
Esse ponto de encontro das três alturas é o Ortocentro.
Quando H = B ?
Quando
A+1 B A(A − 1)
A= e =− .
3 3 B
Que é exatamente quando:
1 3
A= e B2 = ,
2 4
que diz que se trata de triângulo equilátero, como já vimos.
CAPÍTULO 7. GEOMETRIA ANALÍTICA PLANA 97

Falta vermos também quando o Ortocentro coincide com o circuncentro. Isso se


dá quando
1 A(A − 1) A · (A − 1) B
A= e − = + ,
2 B 2B 2
que também dão
1 3
A= e B2 = ,
2 4
formando triângulos equiláteros.
Agora, supondo que nosso triângulo não seja equilátero, só nos resta encontrar a
equação da reta ligando B a C e conferir que ela passa pelo H.
A reta por B e C é ou bem a reta vertical
1 1
x= , se A = ,
2 2
quando o triângulo é isósceles, ou bem se A 6= 12 :
B 2 + 3A2 − 3A A(B 2 + A2 − 1)
y=− ·x+ .
B(2A − 1) B(2A − 1)
Esta é a reta de Euler !
Só falta agora verificarmos as distâncias.
Os quadrados das distâncias são:
2 2 1 A(A − 1) 1 2
HB := ( A − )2 + ( + B) =
3 3 B 3
10A2 B 2 − 10AB 2 + B 2 + 9A4 − 18A3 + 9A2 + B 4
= .
9B 2
Enquanto que

2 1 1 A(A − 1) 1 2
BC := ( A − )2 + ( + B) =
3 6 2B 6
10A2 B 2 − 10AB 2 + B 2 + 9A4 − 18A3 + 9A2 + B 4
= .
36B 2
ou seja
2 2
HB = 4 · BC ,
como querı́amos.


Observação 1:
Observe que temos a equação explı́cita e portanto podemos determinar casos onde
a reta de Euler é horizontal. Que ocorrem para pontos da forma
p
P = ( A, ± 3A(1 − A) ).
4. A EQUAÇÃO DA RETA DE EULER 98

0,8

0,6

0,4

0,2

0
0 0,2 0,4 0,6 0,8 1


Figura: A reta de Euler é horizontal para pontos da forma P = ( 32 , 3
6
).

Observação 2:
É natural termos curiosidade por qual seria o gráfico da função z = z(A, B), B 6= 0
dada por
z = 10A2 B 2 − 10AB 2 + B 2 + 9A4 − 18A3 + 9A2 + B 4 ,
pois vimos z = 0 está associado a um ponto muito especial no plano formado pelos
parâmetros (A, B): o ponto

1 3
( , ) ∼ (0.5, 0.8).
2 2
A Figura a seguir mostra uma parte dessa superfı́cie, com A ∈ [0, 1] e B ∈ [0.1, 1.3]
(na figura o eixo x é o dos A e o eixo y é o dos B).

0 1
1,2 0,8
1 0,6
0,8
y 0,6 0,4 x
0,4 0,2
0,2 0
CAPÍTULO 7. GEOMETRIA ANALÍTICA PLANA 99

Mas não se vê muita coisa. Já as próximas duas Figuras são perfis da superfı́cie,
e elas sim ilustram bem que um ponto próximo de (0.5, 0.8) é o mı́nimo dessa função
z = z(A, B) (na figura o eixo x é o dos A e o eixo y é o dos B).

0
1 0,8 0,6 0,4 0,2 1 ,2
0,8
00,2
0,4
0,6
x
y

0 1
0
0,8 x
0,6
0,4
0,2
1,2 1 0,8 0,6 0,4 0,2
y

5. A inversa como reflexão de gráfico na diagonal

Imagine uma função f : I → J, y = f (x) que admita uma função inversa f −1 :


J → I, x = f −1 (y).
Vamos supor agora que temos ambos os gráficos, de f e de f −1 , no mesmo sistema
de coordenadas (x, y), ou seja, por um momento pensemos em g = f −1 tomada com as
6. O MÉTODO DE DESCARTES PARA AS TANGENTES A UM GRÁFICO 100

mesmas abcissas e oordenadas que a f , ou seja, vamos ver ao mesmo tempo y = f (x)
e y = g(x).
Agora ligamos com uma reta r o ponto (A, B) := (x, f (x)) do gráfico de y = f (x)
com o ponto (B, A) do gráfico de y = g(x). Então o coeficiente angular dessa reta é:
A−B
a := = −1.
B−A
Ou seja que a reta r que os liga tem a mesma inclinação da anti-diagonal, a = −1,
ou seja, r é ortogonal à diagonal y = x. A equação dessa r é pelo que vimos na
Afirmação 1.3:
r : y = −x + (A + B).
E r corta a diagonal y = x no ponto cuja abcissa satisfaz:
x = −x + (A + B),
A+B
ou seja x = 2
, ou seja, no ponto com coordenadas ( A+B
2
, A+B
2
). E (A, B) e (B, A)
A+B A+B
são equidistantes de ( 2 , 2 ).
Concluı́mos que a diagonal y = x funciona como um espelho para os gráficos de
y = f (x) e y = g(x):
O gráfico da f −1 referido ao mesmo sistema (x, y) é um reflexão na diagonal do
gráfico da y = f (x)
y=x
(B,A)
r
y= f^{−1}(x)

(A,B)

y= f(x)

Figura: Os gráficos de f e f −1 no mesmo sistema cartesiano

6. O método de Descartes para as tangentes a um gráfico


Como a Geometria analı́tica foi um criação de René Descartes, nada mais justo
que indicarmos um bonito método criado por ele1
Pelo menos no meu caso, durante meu tempo de ensino Médio, só me lembro da
palavra reta tangente ser usada para referir a reta tangente de um cı́rculo.
Nesse caso, para um cı́rculo C de raio r e centro O, pode ser definida como a reta
t pelo ponto P que é ortogonal ao raio do Cı́rculo.
Em geral uma reta por um ponto P de C o intersecta noutro ponto, mas a reta
tangente t a P não pode intersectar C noutro ponto P ′: se por absurdo t∩C = {P, P ′}
1Me baseei mais no livro de Edwards, mas o leitor pode comparar com o que está nas páginas
95-113 de The geometry of René Descartes, Dover.
CAPÍTULO 7. GEOMETRIA ANALÍTICA PLANA 101

então no triângulo ∆ OP P ′ a hipotenusa OP ′ mediria o mesmo que o cateto OP ,


absurdo.
Descartes se perguntou pelo significado da reta ortogonal a um gráfico qualquer,
pois isso está ligado a questões de Óptica, de reflexão da luz em lentes, que lhe
interessavam.
Responder a essa questão dá a chave também para o significado da reta tangente
a um gráfico qualquer (pois uma é ortogonal à outra).
De fato não vamos lidar coma questão assim tão geral: suponhamos gráficos de
polinômios y = f (x).
Ele pensou em usar o que sabia de cı́rculos para atacar o caso geral de gráficos.
Para isso, considerou um ponto P = (x, f (x)) do gráfico e considerou Cı́rculo com
centro (c, 0) no eixo dos x, de raios r que passem por P = (x, f (x)).
Ou seja, escolhidos c, r teremos que x é raı́z de:
(f (x) − 0)2 + (x − c)2 − r 2 = 0.
Em geral, se c é escolhido de qualquer jeito, pode haver outra raı́z x′ dessa equação,
pois o cı́rculo
y 2 + (x − c)2 − r 2 = 0
pode cortar o gráfico de y = f (x) em mais de um ponto.

problema: Como escolher c para que x seja raı́z dupla de:


(f (x) − 0)2 + (x − c)2 − r 2 = 0,
ou seja, para que uma segunda raı́z x′ colida com x ?

Se conseguı́ssemos resolver esse Problema estarı́amos colocando o Cı́rculo de modo


a tocar, tangenciar o gráfico em P .
Ora, como sabemos qual a tangente ao Cı́rculo usarı́amos essa reta como tangente
ao gráfico !
Melhor do que explicar o método em abstrato será fazermos dois Exemplos.

Exemplo 6.1. Consider y = Cx2 uma parábola e tome P = (x, Cx2 ), com x > 0.
Comos os Cı́rculos com centro (c, 0) tem equação:
y 2 + (x − c)2 = r 2 ,
queremos encontrar uma raı́z dupla x de:
(Cx2 )2 + (x − c)2 − r 2 = 0,
ou seja queremos encontrar uma fatoração:
(Cx2 )2 + (x − c)2 − r 2 = (x − x)2 q(x)
onde q(x) é um polinômio de grau 2.
Ou seja queremos encontrar uma fatoração do tipo:
(Cx2 )2 + (x − c)2 − r 2 = (x − x)2 · (a2 x2 + a1 x + a0 ).
6. O MÉTODO DE DESCARTES PARA AS TANGENTES A UM GRÁFICO 102

Expandindo ambos os lados, formam-se dois polinômios de grau 4 em x, à esquerda e


à direita. Igualando os coeficientes do monômios x4 à esquerda e à direita faz aparecer
C 2 − a2 = 0 ⇔ a2 = C 2 .
Igualando os coeficientes de x3 à esquerda e à direita faz aparecer:
−a1 + 2xa2 = 0
ou seja
−a1 + 2x(C 2 ) = 0 ⇔ a1 = 2xC 2 .
Igualando os coeficientes de x2 à esquerda e à direita faz aparecer:
1 + 2xa1 − a0 − x2 a2 = 0,
ou seja
1 + 2x(2xC 2 ) − a0 − x2 C 2 = 0 ⇔ a0 = 1 + 3x2 C 2 .
Por último, igualando os coeficientes de x à esquerda e à direita faz aparecer:
−2c + 2xa0 − x2 a1 = 0
ou seja,
−2c + 2x(1 + 3x2 C 2 ) − x2 (2xC 2 ) = 0 ⇔ c = x + 2x3 C 2 .
Logo o Cı́rculo cujo centro é o ponto
O = (c, 0) = (x + 2x3 C 2 , 0)
e que passa por P = (x, Cx2 ) tangencia o gráfico de y = Cx2 nesse ponto P .

y 1

0
0 1 2 3 4 5
x
-1

-2

Figura: O gráfico de y = x2 e o cı́rculo tangente em P = (1, 1), de centro (3, 0).

O coeficiente angular da reta ligando O a P é:


f (x) Cx2 1
− =− 3 2
=− .
c−x x + 2x C − x 2xC
CAPÍTULO 7. GEOMETRIA ANALÍTICA PLANA 103

Ora, para passarmos ro raio do cı́rculo para a tangente basta tomar a reta ortog-
1
onal. E o coeficiente angular ortogonal ao anterior − 2xC é:
2Cx.
Logo a reta tangente ao gráfico em P vem dada por:
y − Cx2
= 2Cx ⇔ y = (2Cx) x + (Cx2 − 2Cx2 ).
x−x
Exemplo 6.2. Considere y = Cx3 e tome P = (x, Cx2 ), com x > 0. Queremos uma
raı́z dupla de:
(Cx3 )2 + (x − c)2 − r 2 = 0,
ou seja queremos encontrar uma fatoração:
(Cx3 )2 + (x − c)2 − r 2 = (x − x)2 q(x)
onde q(x) agora é um polinômio de grau 4.
Ou seja queremos encontrar uma fatoração do tipo:
(Cx3 )2 + (x − c)2 − r 2 = (x − x)2 · (a4 x4 + a3 x3 + a2 x2 + a1 x + a0 ).
Expandindo ambos os lados, formam-se dois polinômios de grau 6, à esquerda e à
direita. Comparando como fizemos antes os coeficientes de cada monômio, fazemos
surgir equações, que vão sendo resolvidas uma a uma, produzindo nesta ordem:
a4 = C 2 , a3 = 2xC 2 , a2 = 3x2 C 2 ,
a1 = 4x3 C 2 , a0 = 1 + 5x4 C 2 , c = x + 3x5 C 2 .
Logo o Cı́rculo cujo centro é o ponto
O = (c, 0) = (x + 3x5 C 2 , 0)
e que passa por P = (x, Cx3 ) tangencia o gráfico de y = Cx3 nesse ponto P .

1
y
0
0 1 2 3 4 5 6 7
x
-1

-2

-3

Figura: O gráfico de y = x3 e o cı́rculo tangente em P = (1, 1), de centro (4, 0).


8. EXERCÍCIOS 104

O coeficiente angular da reta ligando O a P é:


f (x) Cx3 1
− =− 5 2
=− 2 ,
c−x x + 3x C − x 3x C
O coeficiente angular da reta ortogonal a esta é
3x2 C
e daı́ se obtém em seguida a equação toda da reta tangente ao gráfico.

7. Um problema da Putnam Competition, n. 2, 1939


Só com o material desenvolvido até este Capı́tulo já se pode resolver o seguinte
problema:

Problema: Seja P ponto da curva y = x3 tal que a reta tangente ao gráfico em P


intersecta de novo o gráfico num ponto Q 6= P .
Mostre que a reta tangente ao gráfico em Q tem inclinação igual a 4 vezes a
inclinação em P .

Solução:
Seja P = (a, a3 ). Então a 6= 0 pois de P = (0, 0) a reta tangente é horizontal e
não intersecta o gráfico noutro ponto Q 6= P .
A reta tangente em P tem equação:
y = 3a2 · x − 2a2
e Q = (x, x3 ) verifica a equação:
x3 = 3a2 · x − 2a2 ⇔ x3 − 3a2 · x + 2a2 = 0.
Ora, a é raı́z dupla essa equação, já que em P há tangência, logo:
x3 − 3a2 · x + 2a2 = (x − a)2 · p(x)
onde p(x) é de grau 1 e facilmente se vê, por divisão, que:
p(x) = x + 2a.
Ou seja, o ponto Q tem coordenadas Q = (−2a, −8a3 ).
A inclinação da reta tangente por Q é:
3 · (−2a)2 = 3 · (4a2 ) = 4 · (3a2 ),
ou seja, 4 vezes a inclinação em P .

8. Exercı́cios
Exercı́cio 8.1. Qual é o coeficiente angular da reta y = y(x) determinada pela
equação 3y + 4x − 27 = 0 ?
CAPÍTULO 7. GEOMETRIA ANALÍTICA PLANA 105

Exercı́cio 8.2. i) determine a reta, na forma y = a · x + b, que passa por (1, 2) e


(4, 13).

ii) determine a reta, na forma y = a · x + b, que passa por (1, 2) com coeficiente
angular 5.
Exercı́cio 8.3. (resolvido)
Tentei resolver o sistema de equações:
y − 5x − 2 = 0 e 2y − 10x − 1 = 0,
e fiz o seguinte: da primeira equação obtive y = 5x + 2 e substitui esse y na segunda,
obtendo:
2(5x + 2) − 10x − 1 = 3 = 0,
o que é um absurdo, pois 3 6= 0.
Você poderia explicar, com os conceitos deste Capı́tulo por quê chego nesse ab-
surdo?
Exercı́cio 8.4. Agora tentei resolver os sistemas de duas equações:
y − ax + 1 = 0 e y − x + 2 = 0
(sim são vários sistemas de duas equações pois a ∈ R pode ser mudado).
Da primeira obtive: y = ax − 1 e substituindo na segunda obtive:
(ax − 1) − x + 2 = x(a − 1) + 1 = 0.
i) Supondo a − 1 6= 0 continue a resolução dos sistemas.
ii) explique geometricamente qual o significado da condição a − 1 6= 0.
Exercı́cio 8.5. Um outro modo se pensar a questão de como determinar a reta
y = a · x + b passando por dois pontos P1 = (x1 , y1 ) e P2 = (x2 , y2 ) é resolver o
sistema:
y1 = a · x1 + b e y2 = a · x2 + b,
cujas incógnitas são a, b.
i) qual a condição sobre P1 = (x1 , y1 ) e P2 = (x2 , y2) para que o sistema tenha
solução única ? O que diz a chamada Regra de Cramer neste caso ?
Agora considere o problema de determinar qual a curva da forma
y 2 = x3 + b · x + a
passa pelos pontos P1 = (−3, 0) e P2 = (4, 0).
ii) qual o sistema de equações a ser resolvido ? É muito diferente do anterior ?
iii) qual a solução (a, b) ?
Exercı́cio 8.6. (resolvido)
Seja y = ax + b a equação de uma reta r e seja P = (A, B) 6∈ r.
i) Encontre o ponto Q na reta r tal que o segmento P Q é ortogonal a r em Q.
ii) pode acontecer que a coordenada x de Q seja A ? Exatamente em que situações
?
8. EXERCÍCIOS 106

Exercı́cio 8.7. Prove que o circuncentro


1 A(A − 1) B
C=( , + ),
2 2B 2
equidista dos três vértices (0, 0), (1, 0) e (A, B) do triângulo (B 6= 0).
Conclua que há um cı́rculo centrado em C que passa pelos vértices do triângulo.
Dica: expanda os quadrados e simplifique.
Exercı́cio 8.8. (resolvido)
Veremos en detalhe no Capı́tulo 20 que as equações:
y2
x2 + 2 = 1
b
definem elipses com centro na origem.
Determine b2 para que a elipse correspondente seja tangente à reta y = −x + 5
em algum ponto dessa reta. (Dica: dá para fazer isso no estilo de Descartes).
Exercı́cio 8.9. (resolvido)
Dê a função inversa de f : R \ {0} → R, f (x) = x1 .
Conclua que essa função tem gráfico simétrico em relação à diagonal.
CAPı́TULO 8

A Tangente ao gráfico, segundo o Cálculo

No final do Capı́tulo anterior vimos que Descartes desenvolveu um engenhoso


método algébrico para definir e calcular retas tangentes a gráficos de polinômios.
Mas precisamos de um método mais geral. Para isso, estudaremos primeiro as
secantes a gráficos e depois, via o conceito de limite, definiremos as tangentes a
gráficos.

1. Retas secantes a um gráfico


Será interessante para nós pegarmos dois pontos de um mesmo gráfico e calcular-
mos a equação da reta que os liga, chamada secante ao gráficos pelos dois pontos.
Estaremos interessados pricipalmente em seu coeficiente angular.
Por exemplo, (x1 , f (x1 ) e (x2 , f (x2 ) definem uma reta y = ax + b com coeficiente
angular
f (x2 ) − f (x1 )
a= ,
x2 − x1
e coeficiente linear
f (x2 ) − f (x1 )
b = f (x1 ) − ( ) · x1 .
x2 − x1
Exemplos:
1)- Tome um x1 > 0 e fixe no gráfico da função f (x) = |x| o ponto (x1 , x1 ). Note
que os x2 próximos de x1 também são positivos e portanto as secantes determinadas
por (x1 , x1 ) e (x2 , x2 ) são sempre as mesmas, de fato, são todas iguais à diagonal
y = x. Analogamente, se x1 < 0 as secantes que envolvem o ponto (x1 , −x1 ) e outro
do gráfico bem próximo coincidem com a antidiagonal y = −x.
2) - Certamente nenhuma secante ao gráfico de y = x2 coincide com o gráfico;
vemos que aqui as secantes mudam de inclinação.

2. A reta tangente a um gráfico


Olhe agora somente o coeficiente angular da secante ao gráfico de y = f (x) por
dois de seus pontos :
f (x2 ) − f (x1 )
.
x2 − x1
Imagine que (x1 , f (x1 )) fica parado mas que (x2 , f (x2 )) está se movendo, no gráfico
de f , indo cada vez mais próximo de (x1 , f (x1 )). Se f é contı́nua, basta supor que a
coordenada x2 fica próxima de x1 para necessariamente f (x2 ) ficar mais próxima de
f (x1 ).
107
2. A RETA TANGENTE A UM GRÁFICO 108

Como x2 fica próximo de x1 sua diferença


h := x2 − x1
tem módulo pequeno. Para deixarmos o ponto (x1 , f (x1 )) em destaque, vamos escr-
ever o coeficiente angular acima como:
f (x1 + h) − f (x1 )
ax1 ,h := , onde x1 + h = x2 .
h

0
0 0,5 1 1,5 2
x
-1

-2

Figura: Duas secantes pelo ponto (1, 1) do gráfico de y = x2

A grande questão é:


Será que esses coeficientes angulares ax1 ,h tendem a um valor especı́fico bem de-
terminado ax1 1, quando h → 0 (independentemente do modo como h se faz pequeno)
?

É nesse ponto que se vê importância de podermos falar de algo como o h tender a
zero, sem precisar nunca ser zero: pois simplesmente não podemos dividir por h = 0
e precisamos calcular limh→0 ax1 ,h .
Atenção ! pois em geral pode não existir esse limite, como algo bem definido.
O exemplo mais simples é (que é uma função contı́nua !):
y = f (x) = |x| e x = 0.
De fato, se h > 0 e tende a zero, obtenho:
|0 + h| − |0| h
lim = lim =
h→0
h>0
h h→0
h>0
h

= lim 1 = 1,
h→0
h>0

1Claro que em geral ax1 depende do x1 escolhido


CAPÍTULO 8. A TANGENTE AO GRÁFICO, SEGUNDO O CÁLCULO 109

e no entanto:
|0 + h| − |0| −h
lim = lim =
h→0
h<0
h h→0
h<0
h
= lim −1 = −1,
h→0
h<0

0,8

0,6

0,4

0,2

0
-1 -0,5 0 0,5 1
x

Figura: Gráfico de y = | x |, para x ∈ [−1, 1].

Definição 2.1. Quando há uma posição limite de secantes, ou seja, quando existe
f (x1 + h) − f (x1 )
a := lim ax1 ,h , onde ax1 ,h := ,
h→0 h
dizemos que existe a Reta Tangente ao gráfico de f em (x1 , f (x1 )). É a reta dada
por:
y = a · x + b, pondo a := lim ax1 ,h
h→0
e onde b fica determinado pela imposição de que essa reta passe por (x1 , f (x1 ).

De f (x1 ) = a · x1 + b, obtenho o coeficiente linear:


b = f (x1 ) − (lim ax1 ,h ) · x1 .
h→0

É interessante que, embora as secantes não tenham muito a ver com o gráfico:
a tangente ao gráfico em um de seus ponto dá informação relevante sobre ele, ela
dá informação do formato do gráfico naquele ponto.
Dentre todas a retas passando por aquele ponto, a tangente ao gráfico é a mais
informativa do formato do gráfico.

3. A reta tangente ao seno em (0, 0) é a diagonal

Vamos dar uma justificação bem geométrica para o fato de que no gráfico do seno
existe uma reta tangente bem definida no ponto (0, 0): de fato sua equação é a mesma
da diagonal y = x.
Para isso começamos observando que:
3. A RETA TANGENTE AO SENO EM (0, 0) É A DIAGONAL 110

Afirmação 3.1. Valem:


sin(θ) < θ e θ < tan(θ), para 0 < θ < π/4,
e
tan(θ) < θ e θ < sin(θ), para − π/4 < θ < 0.
Demonstração.
Seja 0 < θ < π/4.
Considere três Áreas envolvidas:
• do triângulo △ com vértices em (0, 0), (1, 0) e em (cos(θ), sin(θ)). Note que
a base dele mede 1 e que sua altura é o sin(θ). Logo A△ (θ) = sin(θ)2
.
• do Setor circular (fatia do disco) de abertura θ do disco de raio 1, s(θ). Sua
área2 é denotada As (θ). Temos As (2π) = π e As (θ) = θ2 .
• do triângulo ∆ com vértices em (0, 0), (1, 0) e no ponto (1, tan(θ)), que é um
triângulo retângulo em (1, 0) Denote sua área por A∆ (θ). A base dele mede
1 e que sua altura é tan(θ). Logo A∆ (θ) = tan(θ)
2
.

(1, tan θ )

( cos θ, sen θ)

θ
(1,0)
(0,0)

Figura: Observe que △ ⊂ s(θ) ⊂ ∆

Das inclusões:
△ ⊂ s(θ) ⊂ ∆
obtemos:
A△ (θ) < As (θ) < A∆ (θ)
ou seja para 0 < θ < π/4:
sin(θ) θ tan(θ)
< < ,
2 2 2
que é o que queremos (se eliminamos o 1/2).
Por outro lado, se −π/4 < θ < 0 (isto é, θ é ângulo no sentido horário),
A△ (θ) < As (θ) < A∆ (θ)
2O Cálculo pode provar que a área de um disco de raio r é π · r2 , como o faremos nos Capı́tulos
sobre Integração. A Área de um setor de abertura θ (em radianos) no disco de raio r é
θ θ·r
· πr2 =
2π 2
.
CAPÍTULO 8. A TANGENTE AO GRÁFICO, SEGUNDO O CÁLCULO 111

agora significa (já que para cálculo de áreas tomo os módulos de números negativos):
− sin(θ) −θ − tan(θ)
< < ,
2 2 2
ou seja (multiplicando por −1):
tan(θ) θ sin(θ)
< <
2 2 2
o que queremos (eliminando o 1/2).


Afirmação 3.2. (Um Limite fundamental)


sin(θ)
lim =1
θ→0 θ
Demonstração.
Para 0 < θ < π/4, da Afirmação 3.1 temos
sin(θ)
θ< ,
cos(θ)
cos(θ)
e obtenho (multiplicando por θ
> 0):
sin(θ)
cos(θ) < .
θ
Ainda da Afirmação 3.1, para 0 < θ < π/4,:
sin(θ) < θ
e obtenho:
sin(θ)
< 1.
θ
Ou seja,
sin(θ)
cos(θ) < < 1, se 0 < θ < π/4.
θ
Uso agora o item 6) do Teorema 1.1, combinado com continuidade do cosseno, ob-
tendo:
sin(θ)
lim = lim cos(θ) = cos(0) = 1.
θց0 θ θ→0
Por outro lado, quando −π/4 < θ < 0 ainda temos cos(θ) > 0 e pela Afirmação 3.1
tı́nhamos:
sin(θ)
< θ,
cos(θ)
cos(θ)
de onde obtenho (multiplicando por θ
< 0):
sin(θ)
> cos(θ).
θ
De novo da Afirmação 3.1 para −π
2
< θ < 0:
θ < sin(θ)
3. A RETA TANGENTE AO SENO EM (0, 0) É A DIAGONAL 112

e obtenho (já que θ < 0):


sin(θ)
< 1.
θ
Então como antes obtenho:
sin(θ)
lim = lim cos(θ) = cos(0) = 1,
θր0 θ θ→0

o que é suficiente para sabermos que


sin(θ)
lim = 1.
θ→0 θ


1
0,8
0,6
0,4
0,2
0
-3 -2 -1 0 1 2 3
x

sin(θ)
Figura: Gráfico de y = f (x) = θ
para 0 6= θ ∈ [−π, π] e f (0) = 0.

Como consequência da Afirmação 3.2 e da definição de Reta Tangente ao gráfico


do seno em (0, 0), a tangente ao gráfico do seno em (0, 0) é exatamente a diagonal,
pois os coeficientes angulares de secantes por (0, 0) são:
sin(θ) − sin(0)
θ−0
e
sin(θ) − sin(0) sin(θ)
lim = lim = 1.
θ→0 θ−0 θ→0 θ

1,5

0,5

0
-1,5 -1 -0,5 0 0,5 1 1,5
x
-0,5

-1

-1,5
CAPÍTULO 8. A TANGENTE AO GRÁFICO, SEGUNDO O CÁLCULO 113

Figura: A diagonal é tangente ao seno em (0, 0)

4. Interpretação Fı́sica da reta tangente


Uma das fontes do Cálculo é a Fı́sica. Os conceitos de secantes e tangente a um
gráfico têm uma interpretação fı́sica natural.
Se x é pensado como sendo o tempo, podemos pensar em f (x) como a posição
de um objeto, determinada em relação a um ponto de origem, do qual nos afastamos
para a direita (valores positivos de f ) ou para a esquerda (valores negativos de f ).
Então
f (x2 ) − f (x1 )
é a distância percorrida no tempo transcorrido x2 − x1 e
f (x2 ) − f (x1 )
x2 − x1
é o que se costuma chamar a velocidade média.
É o que no dia-a-dia nos perguntam: você vai de casa até a faculdade em quanto
tempo ? E daı́ se deduz a velocidade média do seu trajeto.
Mas também poderia haver interesse de alguém nas velocidades marcadas no ve-
locimetro do seu carro a cada instante, para saber onde pegou engarrafamento, se teve
excesso de velocidade em alguns trechos, etc. O que é essa velocidade instantânea
no instante x1 ? Ora, é o limite:
f (x1 + h) − f (x1 )
lim .
h→0 h
Ou seja, o coeficiente angular da tangente ao gráfico da função posição f no
instante x1 dá a velocidades instantânea no momento x1 . Isso é o que marca o
velocı́metro do carro.
Essa interpretação que estamos dando dos conceitos que vimos ao caso do movi-
mento de um objeto, nos motiva a falar da aceleração, um conceito que usamos muito
no dia a dia. Falaremos disso na Seção 5 do Capı́tulo 9.

5. Exercı́cios
Exercı́cio 5.1. i) Determine os intervalos em que coeficientes angulares das secantes
da função f (−∞, 0) ∪ (0, +∞) → R, f (x) = 1/x são positivos ou negativos.
ii) Diga (ainda de modo bem intuitivo) o que acontece com esses coeficientes
angulares de secantes quando o ponto fixado x fica próximo de zero (separadamente
se x < 0 ou se x > 0) ou com módulo de x muito grande (x > 0 ou x < 0).
Exercı́cio 5.2. Calcule as equações y = ax + b das retas tangentes no ponto (1, 1)
dos gráficos de:
i): y = x2
ii): y = x3
iii): y = x4
5. EXERCÍCIOS 114

sin(x) sin2 (x)


Exercı́cio 5.3. Pedi para o programa Maple plotar y = x
e y = x
para
x ∈ [−3, 3] e ele repondeu:

0,8

0,4

0
-3 -2 -1 0 1 2 3
x

-0,4

Mas essas funções a princı́pio não estão sequer definidas em x = 0 ! Explique com os
conceitos de limite e continuidade o que o programa fez.
Exercı́cio 5.4. (resolvido)
Usando que limx→0 sin(x)
x
= 1 e composições prove que:
sin(k · x)
lim = k, ∀k ∈ R \ {0}.
x→0 x
e
tan(j · x) j
lim = , ∀k, j ∈ R \ {0}.
x→0 sin(k · x) k
CAPı́TULO 9

A derivada

1. Definição, primeiras propriedades e exemplos simples


A grandeza
f (x + h) − f (x)
, h 6= 0
h
é conhecida como quociente incremental. Ela compara, através do quociente, o in-
cremento (aumento, variação) dos valores da função com o incremento (aumento,
variação) na entrada da função.
E é assim que pensamos no dia-a-dia: não é muito informativo se dissermos quanto
aumentou o salário de alguém, de f (x) para f (x + h), se não dissermos quanto tempo
h foi necessário para o reajuste.
Também se dissermos que um carro passa de f (x) km/h para f (x+ h) km/h e não
dissermos em quanto tempo h o faz, não teremos uma idéia da potência do motor. E
assim por diante, há inúmeros exemplos de processos só são descritos corretamente
se usarmos quocientes incrementais.
Definição 1.1. A Derivada da função y = f (x) num ponto x de seu domı́nio é o
limite:
f (x + h) − f (x)
lim .
h→0 h
Denotamos1 esse limite por f ′ (x).

Observações:
• Não estamos dizendo que sempre exista f ′ (x), ao contrário, é uma bela pro-
priedade para uma f ter derivada f ′ (x). Quando dissermos apenas que f tem
Derivada (ou também, é Derivável ), estamos dizendo que ela tem Derivada
em cada ponto de seu domı́nio.
• após a definição de derivada, podemos redefinir a reta tangente ao gráfico
de y = f (x) no ponto (x, f (x)) como a reta que passa por esse ponto e tem
coeficiente angular f ′ (x). Essa reta se determina assim: pondo
y − f (x)
= f ′ (x)
x−x
obtenho:
y = f ′ (x) · x + (f (x) − f ′ (x)x).
1Essa notação lembra ade I. Newton, mas o outro criador do Cálculo, G. Leibniz usava a notação
df
dx (x), muito usada nos livros de Cálculo.
115
1. DEFINIÇÃO, PRIMEIRAS PROPRIEDADES E EXEMPLOS SIMPLES 116

Note o milagre que há numa derivada: o denominador da fração tende a zero e
mesmo assim a fração tende a um número definido. Isso certamente está ligado ao
fato de que o numerador tende a zero também, como vemos agora:
Teorema 1.1. Se existe o limite
f (x + h) − f (x)
lim ,
h→0 h
então:
• limh→0 ( f (x + h) − f (x) ) = 0
• limh→0 f (x + h) = f (x).
• f é contı́nua em x.
Demonstração.
Prova de i):
Fixe um ponto x qualquer do domı́nio da f . Parto de que existe
f (x + h) − f (x)
lim .
h→0 h
Então adaptando a nossa notação2 àquela do item 4) do Teorema 1.1, obtenho:
f (x + h) − f (x)
lim ( h · ) = 0.
h→0 h
Ou seja,
lim ( (f (x + h) − f (x)) = 0.
h→0

Prova de ii):
Dizer que limh→0 ( (f (x + h) − f (x)) = 0 é exatamente o mesmo que dizer
limh→0 f (x + h) = f (x).
Prova de iii): O iem ii) é a definição de continuidade da f em x. 

A recı́proca desse Teorema é falsa, como o mostra f (x) = |x| que, apesar de
contı́nua em todo seu domı́nio, não tem derivada no x = 0. De fato, já vimos que:
|0 + h| − |0| |0 + h| − |0|
lim = −1, mas lim = 1.
hր0 h hց0 h
Existem funções contı́nuas bastante bizarras, sem derivada em nenhum ponto.
Tente imaginar (sem conseguir, é claro !) uma espécie de serrote com uma infinidade
de dentes, que entre dois dentes tem mais outro e assim por diante. Um exemplo é
construı́do no livro Calculus, de M. Spivak.

2Na f (x+h)−f (x)


notação do Teorema 1.1, x = 0, x = h, uma das funções de h é h e a outra é a
identidade g(h) = h
CAPÍTULO 9. A DERIVADA 117

2. Um Árbitro que só avalia as inclinações


Comparando com a Seção 2 do Capı́tulo 8, concluı́mos que a Derivada f ′ (x) na
Definição 1.1 é o coeficiente angular da Tangente ao gráfico de y = f (x) em (x, f (x)).
Se o valor da Derivada f ′ (x) muda quando mede x isso significa que as inclinações
das tangentes variam ao longo do gráfico.
Vamos dar 4 Exemplos dos mais simples.
Imagine uma competição de surf em que 4 participantes realizam manobras de-
scritas por quatro gráficos diferentes: y = f1 (x) ≡ 1 (constante), y = f2 (x) = x,
y = f3 (x) = x2 e y = f4 (x) = x3 . Imagine também que um certo Árbitro da com-
petição tem a tarefa exclusiva de só medir e avaliar as inclinações das pranchas em
cada instante x, sem se interessar em medir as alturas atingidas pelos participantes.
Quem controla as alturas quem controla é outro Árbitro (e por sinal, nesses exemplos
tão simples é fácil saber onde cada função tem valores positivos, zero ou negativos).
Ou seja, que o Árbitro que só mede as inclinações calcula as Derivadas e apresenta
o gráfico de cada Derivada. A seguir, o resultado para cada um dos 4 concorrentes:

1): f1 (x) = 1:
1−1
f1′ (x) = lim = lim 0 = 0.
h→0 h h→0

0,8

0,6

0,4

0,2

0
-1 -0,5 0 0,5 1
x

Figura: y = f1 (x) ≡ 1 em vermelho e f1′ (x) ≡ 0 em verde.


2): f2 (x) = x:
(x + h) − x
f2′ (x) = lim = lim 1 = 1.
h→0 h h→0

0,5

0
-1 -0,5 0 0,5 1
x

-0,5

-1

Figura: y = f2 (x) = x em vermelho e f2′ (x) ≡ 1 em verde.


2. UM ÁRBITRO QUE SÓ AVALIA AS INCLINAÇÕES 118

3): Para f3 (x) = x2 , f3′ (x) = 2x: já fizemos essa conta na Seção 3 do Capı́tulo 8,
onde vimos a equação da tangente a esse gráfico.

0
-1 -0,5 0 0,5 1
x

-1

-2

Figura: y = f3 (x) = x2 em vermelho e f3′ (x) = 2x em verde.


4): f4 (x) = x3 :

(x + h)3 − x3 x3 + 3x2 h + 3x h2 + h3 − x3
f4′ (x) = lim = lim =
h→0 h h→0 h

h · (3x2 + 3x h + h2 )
= lim == lim (3x2 + 3x h + h2 ) = 3x2 ,
h→0 h h→0

pois o polinômio em h de grau ≤ 2 dado por 3x2 + 3xh + h2 é uma função contı́nua !

0
-1 -0,5 0 0,5 1
x

-1

Figura: y = f4 (x) = x3 em vermelho e f4′ (x) = 3x2 em verde.

Para confeccionarmos um gráfico interessante mais adiante, será útil se calculamos


à mão a derivada de:
5) f5 (x) = x4 :

(x + h)4 − x3 x4 + 4x3 h + 6x2 h2 + 4x h3 + h4 − x4


f4′ (x) = lim = lim =
h→0 h h→0 h

h · (4x3 + 6x2 h + 4x h2 + h3 )
= lim
h→0 h

= lim (4x3 + 6x2 h + 4x h2 + h3 ) = 4x3 ,


h→0
CAPÍTULO 9. A DERIVADA 119

pois o polinômio em h de grau ≤ 3 dado por 4x3 + 6x2 h + 4x h2 + h3 é uma função


contı́nua !
4

0
-1-0,50 0,5 1
x

-2

-4

Figura: y = f5 (x) = x4 em vermelho e f5′ (x) = 4x3 em verde.

3. Derivadas da soma e da diferença


A Afirmação a seguir torna bem mais rápido a determinação da derivada :
Afirmação 3.1. Sejam f (x) e g(x) funções deriváveis em x. Sejam a, b ∈ R. Então
a função a · f (x) + b · g(x) é derivável em x e sua derivada é:
( a · f (x) + b · g(x) )′ = a · f ′ (x) + b · g ′ (x).
Demonstração.
Temos pelas definições de derivadas e propriedades de limites (Teorema 1.1 do
Capı́tulo 5 ):

a · f ′ (x) + b · g ′ (x) :=

f (x + h) − f (x) g(x + h) − g(x)


= a · lim + b · lim =
h→0 h h→0 h
f (x + h) − f (x) g(x + h) − g(x)
= lim a · + lim b · =
h→0 h h→0 h
f (x + h) − f (x) g(x + h) − g(x)
= lim [a · +b· ]=
h→0 h h
a · (f (x + h) − f (x)) + b · (g(x + h) − g(x))
= lim =:
h→0 h
=: ( a · f (x) + b · g(x) )′ .

4. PROBLEMA DA PUTNAM COMPETITION, N. 68, 1993 120

4. Problema da Putnam Competition, n. 68, 1993


Convido o leitor a tentar resolver o problema a seguir sozinho e só depois de
bastante trabalho individual ler a resposta que eu apresento.
Problema:
Encontre todos os valores de α ∈ R para os quais as curvas
1 1
Cα : y = α · x2 + α · x + e Dα : x = α · y 2 + α · y +
24 24
tem algum ponto de tangência.
Solução:
Primeiro noto que as possı́veis intersecções Cα ∩ Dα são pontos cujas coordenadas
x satisfazem a equação:
1 1 1
E : x = α · (α · x2 + α · x + ) + α · (α · x2 + α · x + ) + ,
24 24 24
que é uma equação de grau 4 em x.
Portanto não podemos esperar mais de 4 raı́zes (contando alguma com multipli-
cidade).
Também noto que se um ponto P1 := (a, b) ∈ Cα ∩ Dα e tem
a 6= b
então também o outro ponto P2 := (b, a) ∈ Cα ∩ Dα .
Esses pontos P1 6= P2 estão em lados opostos da diagonal y = x. Por exemplo, se
b > a então é P1 = (a, b) que está acima da diagonal enquanto que P2 = (b, a) está
abaixo da diagonal.
Nesse caso
1
b = α · a2 + α · a + >a
24
e
1
a = α · b2 + α · b + < b.
24
Ou seja que a função contı́nua
1
φ(x) := α · x2 + α · x + −x
24
definida em [a, b] tem φ(a) > 0 e φ(b) < 0. Logo pelo Teorema do Valor Intermediário,
existe um ponto ξ ∈ (a, b) com
ψ(ξ) = 0,
ou seja, existe um ponto do plano
1
P3 := (ξ, α · ξ 2 + α · ξ + )
24
que pertence à diadonal, pois tem
1
ξ = α · ξ2 + α · ξ +
24
e ademais P3 ∈ Cα ∩ Dα . Ora então ξ é raı́z de E e ξ 6= a, b: há raı́zes demais dessa
equação de grau 4, contradição.
CAPÍTULO 9. A DERIVADA 121

Concluo então que só pode haver tangência dessas parábolas em algum ponto que
esteja na diagonal y = x.
Então esse ponto P := (x, x) verifica:
1
x = α · x2 + α · x +
24
de onde ponho α em evidência como:
1
x − 24
α= 2 .
x +x
Mas nesse P = (x, x), onde as curvas são tangentes, qual a inclinação possı́vel ?
Como Cα e Dα são simétricas em relação à diagonal, se a inclinação da reta
tangente à Cα em P é τ então a inclinação da reta tangente à Dα em P é τ1 . Como
há tangência das curvas, τ = τ1 o que dá τ = ±1.
Para Cα :
y ′(x) = 2 · α · x + α
logo
±1 = 2 · α · x + α
de onde
1 −1
α= ou α = .
2·x+1 2·x+1
Portanto temos duas possı́veis equações para x:
1
x − 24 1
=
x2 + x 2·x+1
ou
1
x − 24 −1
2
= .
x +x 2·x+1
Elas produzem duas equações quadráticas em x, que resolvo por Báskara. Uma tem
as soluções
1 −1
x= ou x =
4 6
e a outra √ √
−23 601 −23 601
x= + ou x = − .
72 72 72 72
Usando
1 −1
α= ou α =
2·x+1 2·x+1
em cada caso obtemos 4 valores possı́veis para α:
2 3
α1 := , α2 =
3 2
ou
−36 −36
α3 = √ , α4 = √ .
13 + 601 13 − 601
As Figuras a seguir ilustram as posições das parábolas Cα e Dα para esses 4 valores
α1 , α2 , α3 , α4 , bem como a reta diagonal:
4. PROBLEMA DA PUTNAM COMPETITION, N. 68, 1993 122

y 0
-2 -1 0 1 2
x

-1

-2

y 0
-2 -1 0 1 2
x

-1

-2

y 0
-2 -1 0 1 2
x

-1

-2
CAPÍTULO 9. A DERIVADA 123

0,5
x
-2 -1,5 -1 -0,5 0 0,5 1
0

-0,5y

-1

-1,5

-2

5. A segunda derivada
Um exemplo do dia-a-dia: pisando no acelerador do carro vemos o ponteiro do
velocı́metro mudar de posição, pois aumentamos a velocidade instantânea. Enquanto
que, pisando no freio do carro, desaceleramos o carro, diminuimos sua velocidade
instantânea.
Vamos usar o sı́mbolo da derivada
f ′ (x)
para denotar a velocidade instantânea em cada tempo x. O velocı́metro dá uma idéia
de quanto vale f ′ (x).
Note que antes tı́nhamos uma função f (x) que dava a posição em cada instante.
Agora estamos interessados em variar não a posição f (x) em cada instante, mas sim
a velocidade f ′ (x) em cada instante.
Então podemos perguntar agora quanto f ′ (x) variou num tempo determinado, ou
seja podemos falar da aceleração média:
f ′ (x2 ) − f ′ (x1 )
.
x2 − x1
Exemplo dessa grandeza no dia-a-dia: nas revistas especializadas em carros sempre
falam do carro que passa de zero a 100 km/h em tantos segundos.
Agora passando ao limite:
f ′ (x1 + h) − f ′ (x1 )
lim .
h→0 h
obtemos a aceleração instantânea no instante x1 . Um sı́mbolo para ela é:
f ′′ (x1 ) := (f ′ )′ (x1 )
e em geral, em cada instante x:
f ′′ (x) := (f ′ )′ (x)
Infelizmente nos carros de passeio normais não temos uma aparelho que meça isso,
um acelerômetro, para nos dizer qual a aceleração instantânea. Porém num escândalo
recente na Fórmula 1 se soube que se registra também os valores de aceleração em
6. EXERCÍCIOS 124

cada instante dos carros de corrida. Na Seção 2 do Capı́tulo 10 daremos um Exemplo


em que a aceleração/velocidade/posição de um carro contradiz o senso comum.
Na Fı́sica de Newton a aceleração instantânea f ′′ (x) := (f ′ )′ (x) joga um papel
primordial, pois ela (multiplicada pela massa) é a resultante de todas as forças que
agem sobre um corpo.
O que ele descobriu foi como, matematicamente, passar da aceleração instantânea
′ ′
(f ) (x) para a velocidade instantânea f ′ (x) e dai finalmente para a posição f (x) do
objeto em cada instante de tempo.
Começou postulando um formato para a aceleração resultante da força de atração
gravitacional do sol sobre os planetas, e chegou, matematicamente, no formato exato
das órbitas dos planetas (elipses,cônicas) (ou seja na f (x) ) e em suas velocidades
f ′ (x) (a lei de Kepler). Com isso transformou a astronomia em ciência.
No Capı́tulo 39 entenderemos o método que ele usou.
6. Exercı́cios
Exercı́cio 6.1. Qual o gráfico de f (x) = |x + 1|?
Onde é contı́nua e onde não tem derivada ?
Exercı́cio 6.2. Consider as funções definidas por:
f (x) = x2 + x + 2, se x < 1,
f (x) = −x2 + b · x + c, se x ≥ 1.
Ajuste os parâmetros b, c para que f seja contı́nua e derivável em x = 1.
Dica: impondo a continuidade se produz uma relação entre c = c(b). E o valor de
b sai de impôr-se a derivabilidade.
Exercı́cio 6.3. Usando apenas a definição, derive (onde C é uma constante ):
i) y ≡ C
ii) y = C · x,
iii) y = C · x2
iv) y = C · x3 ,
v) y = ( x − C )2
vi) y = ( x − C )3
Interprete geometricamente seus resultados, ou seja, explique que relações os
gráficos têm entre si.
Exercı́cio 6.4. A Figura a seguir mostra uma parte do gráfico de y = f (x) = | x x|+1
(vermelho) (estudada na Seção 4 do Capı́tulo 5) e parte do gráfico de y = x (verde).
1

0,5

0
-1 -0,5 0 0,5 1
x

-0,5

-1
CAPÍTULO 9. A DERIVADA 125

Ela sugere que f ′ (0) = 1. Prove isso mostrando separadamente que:


h
( h+1 )
lim =1
hց0 h
e
h
( −h+1 )
lim =1
hր0 h

Exercı́cio 6.5. Para fazer este Exercı́cio, lembre que x = y é inversa de f : R>0 →

R>0 , y = f (x) = x2 e que, pela Afirmação 3.1, x = y é uma função contı́nua.

i) Sem calcular a derivada de√f : R>0 → R>0 , f (x) = x, o que podemos prever
que aconteça com a derivada de x quando x > 0 tende a zero?
ii) Usando√apenas a definição de derivada, calcule a derivada da função f : R>0 →
R>0 , f (x) = x (Dica: quando ficar complicado lidar com a raı́z quadrada, lembre
que (a − b)(a + b) = a2 − b2 .)
iii) compare a fórmula obtida em ii) com o que previu em i).
Exercı́cio 6.6. (resolvido)
Seja f : R<0 ∪ R>0 → R, f (x) = x1 .
i) Sem calcular a derivada de f o que se pode pre-dizer do sinal dessa derivada ?
Em que intervalos é positiva ou negativa ? Pode se anular ?
ii) para calcular a derivada de f via a definição, só é preciso sabe somar e subtrair
duas frações e saber que as funções racionais são contı́nuas. Calcule-a via definição.
Exercı́cio 6.7. Defino uma função f : R → R condicionalmente por:
f (x) = 3x2 + 2, se x < 1, e f (x) = 3x + b, se x ≥ 1.
i) Escolha o coeficiente linear b para que f : R → R seja uma função contı́nua em
todos os pontos.
ii) Dá para escolher b de modo que f : R → R além de contı́nua também fique
derivável em todos os pontos ? Ou há algum ponto onde não haverá derivada ? Por
quê ?
iii) com b escolhidos para f ser contı́nua, qual o gráfico de f ′ (x) ?
Exercı́cio 6.8. (resolvido)
Se existe f ′ (x) então:
f (x + h) − f (x − h)
f ′ (x) = lim .
h→0 2h
Dê um exemplo simples onde existe limh→0 f (x+h)−f
2h
(x−h)
porém onde f ′ (x) não é
sequer contı́nua em x.
CAPı́TULO 10

Sinal da derivada e crescimento

1. Teoremas de Rolle, Lagrange e Cauchy


Tudo que precisamos sobre zeros, crescimento e decrescimento de funções sai de
dois Teoremas: de Rolle e de Lagrange (que de fato são equivalentes entre si).
Teorema 1.1. (Teorema de Rolle) Seja f : [a, b] → R contı́nua em [a, b] e derivável
em (a, b). Se f (a) = f (b) então existe algum ponto x ∈ (a, b) tal que f ′ (x) = 0.
Demonstração.
Considere o mı́nimo global mf e o máximo global Mf de f em [a, b].
Se mf = Mf isso quer dizer que f é constante: então para qualquer ponto de
(a, b) temos f ′ (x) = 0 e acabou.
Supomos então que mf < Mf .
Vamos nos convencer agora que não é possı́vel que ambos os valores mf e Mf sejam
valores de f nos pontos extremo a, b de [a, b]. De fato, se por exemplo f (a) = mf ,
como por hipótese f (a) = f (b), então f (b) = mf ; como Mf > mf então Mf será
atingido por x ∈ (a, b). Vice versa se supomos que f (a) = Mf , concluimos que mf é
atingido em x ∈ (a, b).
Agora vamos mostrar que num x ∈ (a, b) onde f (x) = mf ou onde f (x) = Mf
temos que ter f ′ (x) = 0.
Por exemplo, suponha x ∈ (a, b) onde f (x) = mf e por absurdo, suponha que

f (x) 6= 0:
Há dois Casos a considerar:
Caso 1): f ′ (x) < 0.
Já que x vive num intervalo aberto (a, b) existe pela Afirmação 4.2 um intervalo
centrado em x,
(−δ0 + x, x + δ0 ) ⊂ (a, b)
e por isso podemos tomar 0 < h < δ0 suficientemente pequeno para que x + h ∈ (a, b).
Então pela definição de derivada, temos:
f (x + h) − f (x)
lim <0
h→0 h
e nesse limite h pode ser tomado positivo ou negativo: tomando h positivo e pequeno
temos:
f (x + h) − f (x)
lim < 0,
hց0 h
f (x+h)−f (x)
o que implica que os quocientes incrementais h
são negativos para h positivo
suficientemente pequeno.
127
1. TEOREMAS DE ROLLE, LAGRANGE E CAUCHY 128

Mas o denominador é h > 0: logo os numeradores são negativos:


f (x + h) − f (x) < 0,
para 0 < h suficientemente pequeno. Portanto, f (x + h) < f (x) para 0 < h suficien-
temente pequeno. Ora, isso contradiz a hipótese de que f (x) = mf é mı́nimo global.
Essa contradição veio de supor f ′ (x) < 0 nesse x.
A Figura a seguir apenas serve para ilustrar a situação absurda obtida, onde a reta
em vermelho simboliza a tangente ao gráfico em (x, f (x)) = (x, mf ) (em vermelho).

m_f

x x+h ( h >0 )

Figura: Chegamos num absurdo deste tipo supondo f ′ (x) < 0 em x.

Caso 2): f ′ (x) > 0:


Novamente, já que existe um intervalo centrado em x,
(−δ0 + x, x + δ0 ) ⊂ (a, b),
podemos tomar h < 0 de módulo suficientemente pequeno (|h| < δ0 ) para que x + h ∈
(a, b). Então pela definição de derivada, temos:
f (x + h) − f (x)
lim >0
h→0 h
e tomando h < 0 temos
f (x + h) − f (x)
lim > 0,
hր0 h
o que implica que os quocientes incrementais f (x+h)−f
h
(x)
são positivos para h < 0 de
módulo suficientemente pequeno.
Mas o denominador é h < 0: logo os numeradores são negativos, ou seja,
f (x + h) < f (x)
para h < 0 de módulo suficientemente pequeno. Contradizendo a hipótese de que
f (x) = mf é mı́nimo global. Essa contradição veio de supor f ′ (x) > 0 nesse x. Como
antes, ilustramos a situação na Figura que segue1:

1Af não precisa ser crescente nessa região, como parece sugerir a Figura; f precisa apenas valer
menos que f (x). Voltaremos nisso na Seção 4 deste Capı́tulo
CAPÍTULO 10. SINAL DA DERIVADA E CRESCIMENTO 129

m_f

x+h x ( h<0 )

Figura: Chegamos nesse tipo de absurdo supondo f ′ (x) > 0 em x.

Logo concluimos que f ′ (x) = 0.


A prova análoga se f (x) = Mf .


O uso que Rolle fazia desse fato era para localizar zeros (raı́zes) de polinômios
apenas.
Ele pensava assim, sempre que houver duas raı́zes a e b sucessivas de um polinômio
p(x) de grau n tem que haver uma raı́z do polinômio p′ (x) situada no intervalo [a, b]
(veremos na Parte 2 que sempre a função Derivada de um polinômio é também um
polinômio). Mais ainda, como vimos já em alguns exemplos simples, o grau de p′ (x)
é n − 1. Logo pode ser mais fácil achar as raı́zes de p′ (x) que as do polinômio original
p(x). E aı́ teremos alguma informação sobre a possı́vel localização das raı́zes a e b de
p(x).
(obs.: Na Figura a seguir os eixos horizontal e vertical não estão na mesma escala)

10

0
-2 -1 0 1 2
x

-5

-10

Figura: Polinômio p(x) com 5 raı́zes Reais e p′ (x) com 4 raı́zes Reais.

Um aplicação interessante do Teorema de Rolle e do T.V.I. será dada na Seção 5


do Capı́tulo 13, para provar a Regra de sinais de Descartes, que dá uma estimativa
do número de raı́zes Reais de um polinômio.
1. TEOREMAS DE ROLLE, LAGRANGE E CAUCHY 130

O Teorema de Rolle pode ser generalizado:

Teorema 1.2. (Teorema do Valor Médio de Lagrange)2


Seja f : [a, b] → R contı́nua e derivável em (a, b). Então existe algum x ∈ (a, b)
tal que
f (b) − f (a)
f ′ (x) =
b−a

0,5

0
-1 -0,5 0 0,5 1
x
-0,5

-1

Figura: O gráfico em vermelho ilustra o Teo. de Lagrange em dois pontos.

Demonstração.
Seja p(x) a equação da reta passando por (a, f (a)) e (b, f (b)). Considere uma
nova função, a função diferença f − p dada por (f − p)(x) := f (x) − p(x).
Então f − p é contı́nua, pelo item 1) do Teorema 1.1. Pela derivada da soma
(Afirmação 3.1 Capı́tulo 9):
(f − p)′ (x) = f ′ (x) − p′ (x).
Agora noto que
(f − p)(a) = f (a) − p(a) = 0, e (f − p)(b) = f (b) − p(b) = 0,
e portanto estamos em condições de aplicar em (f − p) o Teorema de Rolle: portanto
existe algum x ∈ (a, b) onde
(f − p)′ (x) = 0,
ou seja onde
f ′ (x) = p′ (x).

2Atenção: muitos estudantes confundem o que diz o Teorema de Lagrange com o que diz a
definição da Derivada.
CAPÍTULO 10. SINAL DA DERIVADA E CRESCIMENTO 131

Por outro lado p(x) = a1 · x + a0 já que é um polinômio de grau ≤ 1 e sua derivada é
o coeficiente angular da reta: p′ (x) ≡ a1 e sabemos que
f (b) − f (a)
a1 = .
b−a
f (b)−f (a)
Portanto f ′ (x) = b−a
como querı́amos.


Mais geral ainda que o T.V. Médio de Lagrange é o seguinte:


Teorema 1.3. (Teorema do Valor Médio de Cauchy)3
Sejam f : [a, b] → R e g : [a, b] → R contı́nuas e deriváveis em (a, b). Então existe
algum x ∈ (a, b) tal que
f ′ (x) · (g(b) − g(a)) = g ′(x) · (f (b) − f (a)).
Demonstração.
Se definimos:
φ(x) := f (x) · (g(b) − g(a)) − g(x) · (f (b) − f (a)),
então φ(x) é contı́nua em [a, b], derivável em (a, b) e tem
φ(a) = f (a) · g(b) − g(a) · f (b) = φ(b).
Por Rolle existe x ∈ (a, b) com:
φ′ (x) = 0,
ou seja,
f ′ (x) · (g(b) − g(a)) − g ′ (x) · (f (b) − f (a)) = 0,
como querı́amos. 

2. O Teorema 0 das Equações Diferenciais


Para motivar o importante Teorema 2.1, começo descrevendo um exemplo.
Imagine um motorista que está dirigindo seu carro do Sul para o Norte numa
rodovia e que vê uma placa indicando que dali a alguns kilômetros há um posto da
polı́cia rodoviária. Como é usual, ele começa a freiar o carro mas o faz assim: começa
pisando no freio assim que vê a placa e vai gradualmente tirando o pé do freio de
modo bem cuidadoso, para que bem em frente do posto da polı́cia esteja acabando
de tirar o pé do freio e passe então para o acelerador, começando a acelerar bem
suavemente e depois aumentando a aceleração.
Freiar e acelerar são tipos de acelerações. Aceleração negativa ao freiar e positiva
quando pisamos no acelerador. Como explicamos na Seção 4 do Capı́tulo 8, podemos
representar matematicamente o que o motorista fez com as acelerações através da
função segunda derivada f ′′ (x) (Seção 5 do Capı́tulo 9), onde f ′ (x) é a função que
dá a velocidade a cada instante e f (x) a posição do carro a cada instante. A função
3Note que se g(x) := x, recaı́mos no Teorema de Lagrange
2. O TEOREMA 0 DAS EQUAÇÕES DIFERENCIAIS 132

posição será f (x) < 0 ao Sul do posto policial e f (x) > 0 ao Norte do posto e seu
aumento significa ir mais para o Norte.
Quando ele estava pisando no freio, f ′′ (x) < 0, quando pisa no acelerador, f ′′ (x) >
0. Onde f ′′ (x) < 0, a velocidade f ′ (x) estava decrescendo, e quando f ′′ (x) > 0 a
função velocidade f ′ (x) deve voltar a crescer.
Um exemplo disso seria:
f (x) = x3 , f ′ (x) = 3x2 , f ′′ (x) = 6x.

10

0
-2 -1 0 1 2
x

-5

-10

Figura: f vermelho, f ′ verde, f ′′ amarelo, escalas diferentes nos eixos.

O que é interessante neste exemplo é que em frente ao posto da polı́cia, quando


x = 0, a velocidade que aparece no velocı́metro é f ′ (0) = 0 e mesmo assim, em
nenhum instante o carro parou, já que f (x) = x3 é estritamente crecente.
Mas isso contradiz o nosso senso-comum, já que algo que se move a 0 km/h deveria
estar parado, pelo menos por algum tempo !
Para fazermos as pazes com o senso-comum, temos o seguinte Teorema, onde
a condição f ′ (x) = 0 se supõe que vale para x em todo um intervalo, mesmo que
pequeno:
Teorema 2.1. Seja f : I → R definida em um intervalo I não-degenerado.4
Suponha f ′ (x) ≡ 0. Então f (x) ≡ C (ou seja, f é constante).
Demonstração.
Não temos a capacidade de predizer qual a constante que iremos encontrar. O
que podemos apenas é raciocinar por absurdo: suponha que f não é constante.
Então existem x1 , x2 ∈ I tais que f (x1 ) 6= f (x2 ). Restrinja f ao domı́nio [x1 , x2 ].
Então pelo Teorema do Valor Médio de Lagrange aplicado à restrição f : [x1 , x2 ] → R
tem que haver um x ∈ (x1 , x2 ) tal que:
f (x1 ) − f (x2 )
f ′ (x) = .
x1 − x2
4Não-degenerado significa não se reduzindo a um ponto. Claro que I pode ser todo R. Mas
atenção que pode a conclusão pode ser falsa, se a f tem o domı́nio composto de mais de um intervalo
(disjuntos).
CAPÍTULO 10. SINAL DA DERIVADA E CRESCIMENTO 133

f (x1 )−f (x2 )


Mas x1 −x2
6= 0 e isso contradiz a hipótese de que f ′ (x) ≡ 0.


E dele decorre o Teorema a seguir (que chamo de 0 por um dos mais básicos):
Teorema 2.2. (O Teorema 0 das Equações Diferenciais) Sejam f : I → R e g :
I → R deriváveis, com f ′ (x) = g ′(x), ∀x ∈ I, onde I é um intervalo. Então f (x) ≡
g(x) + C.
Ilustro esse Teorema através da seguinte Figura:

12

0
-1 -0,5 0 0,5 1
x

Figura: Translações verticais de um gráfico e o gráfico da função derivada.

Demonstração.
Como já observamos, ∀x ∈ I, (f − g)′ = f ′ (x) − g ′(x). A hipótese dá então
que (f − g)′ (x) ≡ 0. Logo pelo Teorema 2.1, (f − g)(x) ≡ C (é constante) ; logo
f (x) ≡ g(x) + C.


3. Critérios de crescimento e de decrescimento


Decorrem facilmente de Rolle e Lagrange os desejados critérios:
Teorema 3.1. (Critérios de crescimento e de decrescimento)
Seja f : I = (a, b) → R derivável.
• i) se ∀x ∈ I, f ′ (x) ≥ 0 então f é crescente em I;
• ii) se ∀x ∈ I, f ′ (x) > 0 então5 f é estritamente crescente em I.
• iii) se ∀x ∈ I, f ′ (x) ≤ 0 então f é decrescente em I;
• iv) se ∀x ∈ I, f ′ (x) < 0 então f é estritamente decrescente em I.

5A recı́proca é falsa, como mostra f (x) = x3


4. UMA CONFUSÃO FREQUENTE SOBRE O SIGNIFICADO DO SINAL DA
DERIVADA 134

Demonstração.
De i): por absurdo suponha que f não é crescente. Significa que existem x1 , x2 ∈ I
com x1 < x2 para os quais:
f (x1 ) > f (x2 ).
Mas então o Teorema do Valor Médio de Lagrange aplicado à restrição f : [x1 , x2 ] → R
dá que existe algum x ∈ (x1 , x2 ) com:
f (x2 ) − f (x1 )
f ′ (x) = < 0,
x2 − x1
contradizendo a hipótese de que f ′ (x) ≥ 0 ∀x ∈ I.
De ii): Se supomos por absurdo que f não é estritamente crescente, significa que
existem x1 , x2 ∈ I com x1 < x2 para os quais:
f (x1 ) ≥ f (x2 ).
Novamente o Teorema do Valor Médio de Lagrange aplicado a f : [x1 , x2 ] → R dá
que existe algum x ∈ (x1 , x2 ) com:
f (x2 ) − f (x1 )
f ′ (x) = ≤ 0,
x2 − x1
contradizendo a hipótese de que f ′ (x) > 0 ∀x ∈ I.
De iii) e iv): são completamente análogas, mutatis mutandis 6


4. Uma confusão frequente sobre o significado do sinal da derivada

Peço atenção agora, para que se evite uma confusão que aparece em algumas
exposições.
As hipóteses dos itens ii) e iv) do Teorema 3.1 pedem que o sinal da função
derivada seja positivo (ou negativo) em todo um intervalo aberto I.
Seria falso um enunciado assim:

(falso !) Seja f : (a, b) → R derivável com algum x ∈ (a, b) onde f ′ (x) > 0
(f ′ (x) < 0). Então existe um intervalo centrado em x onde a restrição da f é cres-
cente (decrescente).

Claro que isso pode até funcionar em alguns exemplos, mas um teorema tem que
funcionar sempre !
A Figura a seguir ilustra uma função f que existe, que é derivável com f ′ (0) > 0,
e que no entanto não é nem crescente nem decrescente em nenhum intervalo centrado
em x (a Figura não mostra isso muito bem, mas as oscilações continuam a existir até
a origem).
6Essa expressão latina quer dizer, desde que adaptando, mudando, o que for conveniente; no
nosso caso, sinais, desigualdades.
CAPÍTULO 10. SINAL DA DERIVADA E CRESCIMENTO 135

Deduzimos então, após o Teorema 3.1, que a derivada f ′ (x) muda de sinal tão
perto de x = 0 quanto quisermos.

0,08

0,04

0
-0,2 -0,1 0 0,1 0,2
x

-0,04

-0,08

Figura: A função f oscila à esquerda e à direita de x = 0, embora f ′ (0) > 0.

A única propriedade que a f da Figura tem é que:

f vale mais que f (0) em pontos x um pouco maiores que x = 0 e f vale menos
que f (0) em pontos x um pouco menores que x = 0

(é isso nós aprendemos na prova do Teorema de Rolle 1.1). Vamos destacar isso
como uma afirmação:

Afirmação 4.1. Seja uma f derivável e x um ponto do intervalo aberto I onde f


está definida.
Se f ′ (x) > 0 então existe um intervalo J centrado em x, onde f (x) < f (x) se
x < x, x ∈ J e f (x) < f (x) se x < x, x ∈ J.
Se f ′ (x) < 0 então existe um intervalo J centrado em x, onde f (x) > f (x) se
x < x, x ∈ J e f (x) > f (x) se x < x, x ∈ J.

Demonstração.
Contida na demonstração do Teorema de Rolle.


5. Descontinuidade da função derivada


Voltando à f da Seção anterior 4, cuja derivada f muda de sinal tão perto de
x = 0 quanto quisermos, somos obrigados a concluir que sua função derivada f ′ (x)
não é uma função contı́nua em x = 0.
6. EXERCÍCIOS 136

De fato, se f ′ (x) fosse uma função contı́nua em x, então o princı́pio de inércia das
funções contı́nuas (Afirm. 1.1 do Capı́tulo 6) diria que f ′ (x) teria que ser positiva em
todo um intervalo centrado em x = 0.7
Conclusão: nem sempre vale f ′ (x) = limx→x f ′ (x). De fato nesse exemplo tratado
se pode mostrar que a igualdade f ′ (x) = limx→x f ′ (x) não vale porque o lado direito
limx→x f ′ (x) simplesmente não existe.
Mas temos:
Afirmação 5.1. Seja f : I → R onde I = (−δ + x, x + δ) é intervalo aberto centrado
em x.
Suponha que existe f ′ (x) ∀x ∈ I \ {x} e que existe:
lim f ′ (x) = L ∈ R.
x→x

Então f ′ (x) existe também e seu valor é f ′ (x) = L


Demonstração.
Considere a restrição de f (x) a [x, x + h] para h > 0 e aplique o T.V. Médio de
Lagrange:
f (x + h) − f (x)
= f ′ (ξh ), onde ξh ∈ (x, x + h).
h
Quando dizemos na hipótese:
lim f ′ (x) = L
x→x

dizemos que não importa como x tenda a x, necessariamente f ′ (x) tende a L. Ou


seja, não depende da cara do x que tende a x.
Ora, quando h ց 0 temos que ξh ∈ (x, x + h) tende a x e portanto
f (x + h) − f (x)
L = lim f ′ (ξh ) = lim =: f+′ (x),
hց0 hց0 h
a derivada à direita. Analogamente se obtém:
f (x + h) − f (x)
L = lim f ′ (ξh ) = lim =: f−′ (x)
hր0 hր0 h
para a derivada à esquerda e, portanto, f ′ (x) = L.


6. Exercı́cios
Exercı́cio 6.1. A figura que exemplifica o T.V.M de Lagrange no texto é o gráfico de
y = x3 . Quando x ∈ [−1, 1] em quais pontos do gráfico a inclinação da reta tangente
é 1 ?
7Se costuma chamar uma função f de classe C 1 se f é derivável e se f ′ (x) ela mesma é uma
função contı́nua.
CAPÍTULO 10. SINAL DA DERIVADA E CRESCIMENTO 137

Exercı́cio 6.2. 2) Explique (com os conceitos do Cálculo) o que se modifica e o que


não se modifica nos gráficos a seguir quando variamos o parâmetro b 6= 0 em:
i): y = fb (x) = bx2
ii) y = fb (x) = x2 + b
iii) y = fb (x) = x2 + bx − 1.
(Obs.: nos itens i) e iii) há certos pontos em que se vê bem as diferenças entre os
gráficos).

Exercı́cio 6.3. Encontre o ponto (ou os pontos) do gráfico de y = (x − 1)3 em que


sua(s) reta(s) tangente(s) é (são) paralela(s) à reta y = 3x.
Encontre o ponto (ou os pontos) do gráfico de y = x3 em que sua(s) reta(s)
tangente(s) é (são) ortogonal (s) à reta y = − 61 x.
Obs. Não precisa desenhar nada.

Exercı́cio 6.4. (resolvido)


Considere a famı́lia de gráficos

y = fb (x) := (−b + 4/3) · x2 + b · x + (2b − 7/3), b ∈ R,

dos quais plotei apenas 7 representantes (b = 1, 1.2, 1.3, 4/3, 1.6, 1.8, 2):

x
-3 -2 -1 0 1 2 3 4
0

-5

-10

Como se vê são gráficos bem diferentes, à medida que mudamos o parâmetro b.
6. EXERCÍCIOS 138

Mas quando se faz um zoom na região x ∈ [0.3, 0.7] do domı́nio, os pedaços dos 7
gráficos de y = fb (x) se parecem muito:

2,5

1,5

0,5

0
0,4
0,5
0,6
0,7
x

Explique o que aconteceu quando fizemos o zoom, após confirmar que que os pontos
(−1, −1) e (2, 3) pertencem a esses gráficos todos, ∀b ∈ R).
Dica: Teorema Valor Médio de Lagrange.
CAPı́TULO 11

Aplicações da primeira e segunda derivadas

1. Primeiro critério de máximos e mı́nimos


Se olharmos bem a demonstração que demos do Teorema de Rolle, veremos que
de fato já provamos o seguinte:
Afirmação 1.1. Seja f : (a, b) → R derivável. Se1 x ∈ (a, b) é ponto de Mı́nimo
Local ou de Máximo Local, então f ′ (x) = 0.

A recı́proca dessa Afirmação é em geral falsa: f (x) = x3 tem f ′ (0) = 0 e x = 0


não é nem Mı́nimo nem Máximo local.
No entanto temos o seguinte:
Afirmação 1.2. Seja f : (a, b) → R derivável, com x ∈ (a, b) onde f ′ (x) = 0.
• i) Suponha que existe um intervalo J centrado em x onde a função derivada
vale f ′ ≤ 0, se x < x, e f ′ ≥ 0, se x < x. Então x é Mı́nimo Local da f .
• ii) Suponha que que existe um intervalo centrado em x onde a função derivada
vale f ′ ≥ 0, se x < x, e f ′ ≤ 0, se x < x. . Então x é Máximo Local da f .
Demonstração.
De i): Temos que f ′ (x) ≤ 0 se x ∈ (−δ + x, x) e f ′ (x) ≥ 0 se x ∈ (x, x + δ).
Mas então pelo item iii) do Teorema 3.1, a função original f (x) é decrescente em
(−δ + x, x). E pelo item i) do Teorema 3.1 a função original f (x) é crescente em
(x, x + δ).
A conclusão é que x é ponto de Mı́nimo da f restrita a (−δ + x, x+ δ), um Mı́nimo
local portanto.
De ii): completamente análoga, mutatis mutandis.


2. Critério da segunda derivada


Primeiro vamos relembrar e reforçar o tema da segunda derivada ou aceleração
instantânea em termos fı́sicos.
Para definir uma aceleração instantânea usamos um limite do tipo:
f ′ (x + h) − f ′ (x)
lim ,
h→0 h
1É muito importante que (a, b) seja aberto, pois f : [0, 1] → R, f (x) = x tem pontos de máximo
e mı́nimo e no entanto f ′ (0) = f ′ (1) = 1, onde essas derivadas devem ser entendidas como derivadas
′ ′
à direita f+ (0) e à esquerda f− (1).
139
3. UM PROBLEMA TÍPICO PARA OS ENGENHEIROS 140

onde f ′ (x) é a função velocidade instantânea (e onde a f (x) de partida era a função
posição em cada instante).
Segundo a definição de derivada, o que fizemos lá foi derivar a função f ′ (x), ela
mesma já uma derivada da função f (x). Fizemos então uma segunda derivada:
f ′′ (x) := ( f ′ (x) )′ .
Sua definição então é essencialmente a mesma que demos para a derivada (que pas-
samos agora a chamar de primeira derivada), só que a matéria-prima para compôr os
quocientes incrementais não é uma função f (x) mas sim uma função f ′ (x).
Desse modo, posso enunciar:
Afirmação 2.1. Seja f : (a, b) → R derivável, tal que f ′ (x) também seja derivável.
• i): se f ′ (x) = 0 e f ′′ (x) > 0 então2 x é Mı́nimo local da f original.
• ii): se f ′ (x) = 0 e f ′′ (x) < 0 então x é Máximo local da f original.
Este teorema será generalizado na Afirmação 8.1, um critério da derivada n-ésima.
Demonstração. (da Afirmação 2.1)
De i): Pela Afirmação 4.1 do Capı́tulo 10, aplicada agora à função derivada f ′ (x),
temos que para x ∈ J centrado em x, f ′ (x) < 0 = f ′ (0) se x < x e 0 = f ′ (x) < f ′ (x)
se x < x.
Então recaı́mos exatamente no item i) da Afirmação 1.2. A conclusão portanto é
que x é Mı́nimo local.

De ii): completamente análoga, mutatis mutandis.




Com o material deste Capı́tulo 11 e do Capı́tulo anterior 10 estamos em condições


de confeccionar gráficos qualitativamente corretos de polinômios simples, de grau
baixo, e é o que faremos como Exercı́cio.
3. Um problema tı́pico para os engenheiros

Suponha que você tem o seguinte problema prático:

Construir um objeto retangular, onde a construção de cada x metros da largura


custa a metade da construção de cada z metros de comprimento. Gastando 10 reais
na fabricação de cada unidade, quais as medidas de x e z que maximizam a área do
objeto?
Traduzimos o problema assim: queremos maximizar a área
A(x, z) := z · x
com uma função custo 3 c(x, z) := x + 2z fixada em c(x, z) = 10:
x + 2z = 10.
2Recı́procafalsa: f (x) = x4 tem Mı́nimo local em x = 0 e se pode provar que f ′ (0) = f ′′ (0) = 0
3Também poderia dizer que a função custo é 2x + 4z, já que há dois lados que são largura e dois
que são comprimento. Mas a solução seria completamente análoga.
CAPÍTULO 11. APLICAÇÕES DA PRIMEIRA E SEGUNDA DERIVADAS 141

Note que a princı́pio a função área depende tanto de x como de z. Mas a condição
c(x, z) = 10 me permite escrever z = 10−x 2
e a função área como dependendo só de
uma variável:
10 − x x2
A(x) = ( ) · x = 5x − .
2 2
O domı́nio natural de A(x) é I = (0, 10), pois a largura x tem que ser positiva, e ao
mesmo tempo a condição c(x, z) = 10 diz que, quando z se aproxima de zero, x se
aproxima de 10.
Mas considerar A(x) definida num domı́nio um pouco maior, o intervalo [0, 10],
que tem a vantagem de ser um intervalo limitado e fechado, onde podemos usar o
Teorema 4.2 de Bolzano-Weiersstras, já que A(x) claramente é contı́nua.
Esse Teorema garante que existe um ponto de Máximo global de A : [0, 10] → R.
Mas onde ? Não adianta só sabermos que há uma solução, queremos achá-la !
Certamente não será em x = 0 ou em x = 10, pois nesses pontos a Área fica zero,
já que não largura ou comprimento. Então esse ponto x buscado está em (0, 10), o
que é promissor, pois poderemos tentar usar a Afirmação 1.2.
Para isso precisamos examinar alguns candidatos.
Conforme a Afirmação 1.1, eles terão que ser pontos onde
A′ (x) = 0.
x2
Ora, isso significa para A(x) = 5x − 2
que:
5 − x = 0,
pelo que já sabemos das derivadas, ou seja, o ponto é x = 5.
Mas claramente A′ (x) = 5 − x > 0 se x < 5 e A′ (x) = 5 − x < 0 se 5 < x. Logo
o item ii) da Afirmação 1.2 diz que realmente x é um Máximo local e portanto o
Máximo global, já que não há outro candidato. A área máxima desses objetos então
será
25
A(5) = .
2

12

10

0
0 2 4 6 8 10
x

x2
Figura: O gráfico de A : [0, 10] → R, A(x) = 5x − 2
.

Em geral, nos problemas desse tipo, aparecem diferentes candidados a Máximos


global, que foram aprovados no teste para Máximos locais dado pelo item ii) da
Afirmação 1.2, e então se faz necessário comparar os valores da função em questão
em cada um deles.
4. MÍNIMOS DE DISTÂNCIAS E ORTOGONALIDADE 142

4. Mı́nimos de distâncias e ortogonalidade

Suponha que P = (2, 1) e queremos descobrir qual o menor segmento de reta de


P até uma reta de equação y = ax + 1 (com algum a 6= 0 fixado) que não passe por
P.
Vamos fazê-o de dois modos distintos, que esperamos que dêem os mesmos resul-
tados.
Primeiro vamos usar nossa intuição, que diz que deve se tratar do segmento saindo
de P que é ortogonal à reta y = ax + 1. Ou seja, pelo que aprendemos na Seção 2 do
Capı́tulo 8, deve ser um ponto (x, ax + 1) tal que:
(ax + 1) − 1 −1
= ,
x−2 a
pois o lado esquerdo é o ceoeficiente angular da reta contendo o segmento que sai de
(2, 1). Então disso obtemos:
2
x= 2
a +1
e daı́ facilmente descobrimos o tamanho do segmento.
Por outro lado podemos, via as técnicas de Cálculo, tentar descobrir o mı́nimo da
função que mede a distância de P aos pontos da reta dada.
Para não cairmos numa derivada mais complicada, vamos modificar um pouco o
problema, tentando minimizar a função que é o quadrado da distância de P à reta,
dará também o ponto que minimiza a própria distância4
Essa função quadrado da distância é dada por:
(x − 2)2 + (y − 1)2 = (x − 2)2 + (ax + 1 − 1)2 =

= (a2 + 1)x2 − 4x + 5.
Então essa f (x) = (a2 + 1)x2 − 4x + 5 tem derivada f ′ (x) = 2(a2 + 1)x − 4 e f ′ (x) = 0
exatamente em x = a22+1 , o mesmo ponto encontrado acima.
É claro que f ′ (x) < 0 para x < x = a22+1 e f ′ (x) > 0 para x > x = a22+1 . Portanto
pelo item i) da Afirmação 1.2 f tem mı́nimo local, que de fato é o global nesse ponto
x.
Agora vejamos um Exemplo mais interessante. Quero minimizar a distância entre
2
P = (0, 7) e os pontos da parábola y = x2 .
Usando a intuição geométrica vou buscar esse ponto Q de mı́nima distância entre
aqueles em que o segmento desde P é ortogonal à tangente da parábola em Q.
Então, já que conheço as inclinações das tangentes à parabola em (x, ax2 ) como
sendo 2( x2 ) = x, a ortogonalidade que busco é dada por:
x2
2
−7 −1
= ,
x−0 x
4A Afirmação 2.1 do Capı́tulo 16 justificará rigorosamente o uso do quadrado da distância, ao
invés da própria distância, nos problemas de máximos/mı́nimos.
CAPÍTULO 11. APLICAÇÕES DA PRIMEIRA E SEGUNDA DERIVADAS 143

ou seja,
x2
x·( − 6) = 0.
2
A solução x = 0, onde claramente há ortogonalidade, é nitidamente um ponto de
máximo local da distância
√ entre P = (0,
√ 7) e a parábola.
Mas as soluções x = 12 e x = − 12 corresponderão, como veremos a seguir, a
dois pontos de mı́nimos. A Figura a seguir mostra esses pontos de ortogonalidade.

5
x
-4 -2 0 2 4
0

-5

-10

-15

-20

Figura: No gráfico aparecem dois pontos onde há ortogonalidade.

Visto de outro modo, via a técnica do Cálculo, considero a função que é o quadrado
da distância entre P = (0, 7) e a parábola:
x2
(x − 0)2 + (y − 7)2 = x2 + ( − 7)2 =
2
x4
= − 6x2 + 49.
4
x4
A derivada de f (x) = 4
− 6x2 + 49 é
f ′ (x) = x3 − 12x = x(x2 − 12).
O zero da derivada em x = 0 corresponde a√um máximo local.

Verificamos agora que os pontos x = 12 e x = − 12 são mı́nimos locais (e
globais). √ √
Observe que se 0 < x < 12 temos x(x2 − 12) < 0, enquanto √que se x > 12
temos x(x2 − 12) > 0. Logo o item i) da Afirmação 1.2 diz que x = 12 é mı́nimo de
f. √ √
Agora se x < − 12 temos x(x2 − 12) > 0, enquanto que se − √ 12 < x < 0 temos
x(x2 − 12) > 0. Logo o item i) da Afirmação 1.2 diz que x = − 12 é mı́nimo de f .

A Afirmação 4.1 a seguir justifica o uso da noção de ortogonalidade nos problemas


de máximos/mı́nimos:
4. MÍNIMOS DE DISTÂNCIAS E ORTOGONALIDADE 144

Afirmação 4.1.
i) Se a distância entre um ponto P e o gráfico de y = f (x) tem valor mı́nimo
ou máximo local P F > 0, onde F = (x, f (x)), então a reta tangente ao gráfico de
y = f (x) em F é ortogonal à reta P F .

ii) Sejam um gráfico y = f (x) de uma f derivável e uma reta r que não intersecta
esse gráfico.
Seja F ponto do gráfico de y = f (x) tal que P F > 0 realiza um valor mı́nimo ou
máximo local da distância entre pontos do gráfico e a reta r. Então a reta tangente
ao gráfico de y = f (x) em F é paralela à reta r.
Demonstração.
De i):

Considere F = (x, f (x)) ponto que realiza valor minimo local ou valor máximo
local da distância até um certo P = (x0 , y0 ) que foi dado.
Considere o cı́rculo C de raio P F centrado em P (lembro que P F > 0):
2
C = { (x, y); (x − x0 )2 + (y − y0 )2 = P F }.
Vou fazer aqui a suposição5 de que, perto de F , também C seja gráfico de uma função
y = g(x); que de fato é:
q
2
y = g(x) = y0 + P F − (x − x0 )2 , ∀x ∈ (−δ + x, x + δ).
Veja a Figura:

Considere a função
φ(x) := f (x) − g(x), ∀x ∈ (−δ + x, x + δ).
Suponha por absurdo que a reta tangente ao gráfico de y = f (x) em F não seja
igual à reta tangente a C em F (esta sim sabemos que é ortogonal à reta P F ).
Por exemplo, suponha por absurdo que f ′ (x) > g ′ (x) (o caso < é completamente
análogo).
Então φ′ (x) = f ′ (x) − g ′(x) > 0.
5que exigiria mais justificação
CAPÍTULO 11. APLICAÇÕES DA PRIMEIRA E SEGUNDA DERIVADAS 145

Como φ(x) = 0, a Afirmação 4.1 do Capı́tulo 10 dá que, para um certo ǫ > 0:

φ(x) > 0, ∀x ∈ (x, x + ǫ) e φ(x) < 0, ∀x ∈ (x − ǫ, x).

Ora, mas então

f (x) > g(x) ∀x ∈ (x, x + ǫ) e f (x) < g(x), ∀x ∈ (x − ǫ, x).

Então

f (x) − y0 > g(x) − y0 , ∀x ∈ (x, x + ǫ),

e portanto ∀x ∈ (x, x + ǫ):


p p 2
(f (x) − y0 )2 + (x − x0 )2 > (g(x) − y0 )2 + (x − x0 )2 = P F ,

o que diz que F não é ponto de máximo local da distância de P = (x0 , y0) até o
gráfico de y = f (x).
E do mesmo modo, obteremos ∀x ∈ (x − ǫ, x):
p p 2
(f (x) − y0 )2 + (x − x0 )2 < (g(x) − y0 )2 + (x − x0 )2 = P F ,

o que diz que F não é ponto de mı́nimo local da distância até P = (xo , y0 ).
Essa contradição com a escolha de F termina a prova do item i).

Item ii):
Sejam R ∈ r e F = (x, f (x)) tais que RF realizam valor mı́nimo local ou valor
máximo local da distância até o gráfico de y = f (x) e r.
O raciocı́nio da prova do item i) aplicado a um cı́rculo centrado em R de raio
RF > 0 dirá que a reta tangente ao gráfico de y = f (x) em F é ortogonal à reta RF .
Veja a Figura:

Mas, por outro lado, o mesmo raciocı́nio agora aplicado a um cı́rculo agora cen-
trado em F de raio RF > 0 dirá que a reta r (que é sua própria reta tangente) é
ortogonal à reta RF . Veja a Ffigura:
5. CONCAVIDADES DOS GRÁFICOS 146

Um fato básico da geometria euclidiana diz que, se uma reta r1 é ortogonal a uma
reta r2 e r2 é ortogonal a uma reta r3 , então r1 e r3 são paralelas.
Portanto a reta tangente ao gráfico de y = f (x) em F é paralela a r. 

Para concluir esta Seção, pensemos no caso da reta horizontal y = 0 e no gráfico


de y = x1 , ∀x > 0.
Como poderı́amos definir a distância entre essas duas curvas ?
Note que se dermos qualquer tamanho ǫ > 0 existem pontos xǫ ∈ (y = 0) e
zǫ ∈ (y = x1 ) tais que
xǫ zǫ = ǫ.
Basta tomarmos por exemplo xǫ := ( 1ǫ , 0) e zǫ := ( 1ǫ , ǫ).
Então seria natural dizer que a distância entre a reta horizontal y = 0 e o gráfico
de y = x1 é zero !
Mas note que essa distância zero entre curvas nunca é realizada por pontos de
y = 0 e de y = x1 , já que distância zero entre dois pontos significa que são o mesmo
ponto e no entanto
1
(y = 0) ∩ (y = ) = ∅.
x
Outra maneira de ver que a distância zero entre essas curvas nunca é realizada por
pontos de y = 0 e de y = x1 é o item ii) da Afirmação 4.1, pois y ′ = −1
x2
6= 0, ∀x > 0.

5. Concavidades dos gráficos


Na Definição 5.1 a seguir só me interesso no comportamento da função próxima
a cada um dos pontos de seu gráfico.
Definição 5.1. Diremos que uma função é localmente côncava para cima num ponto
(x, f (x)) de seu gráfico se existe um intervalo Ix centrado em x em que
f (x) > ax + b, ∀x ∈ Ix \ {x},
onde y = ax + b é a reta tangente ao gráfico em (x, f (x)).
Para definir localmente côncava para baixo num ponto (x, f (x)) basta trocar >
por <.
CAPÍTULO 11. APLICAÇÕES DA PRIMEIRA E SEGUNDA DERIVADAS 147

2
x
-2 -1 0 1 2
0

-2

-4

-6

Figura: Um função localmente côncava para cima em cada ponto do domı́nio


Afirmação 5.1. Suponha uma função f : I → R duas vezes derivável.
• i) Se ∀x ∈ I, f ′′ (x) > 0 então, f é localmente côncava para cima em cada
um dos pontos de seu gráfico.
• ii) Se ∀x ∈ I, f ′′ (x) < 0 então f tem localmente côncava para baixo em
cada um dos pontos de seu gráfico.
Demonstração.
De i):
Tome um ponto (x, f (x)) do gráfico. Seja y = ax + b a equação da reta tangente
ao gráfico nesse ponto.
Note que a função
φ(x) := f (x) − (ax + b)
tem
φ(x) = 0 e φ′ (x) = f ′ (x) − a = 0.
Ademais
φ′′ (x) = f ′′ (x) > 0.,
já que supomos que sempre f ′′ (x) > 0.
Então o Critério da Segunda Derivada (Afirmação 2.1, Capı́tulo 11) quando apli-
cado a φ diz que φ tem um mı́nimo local em x (local pois φ tem que ser restrita a um
intervalo Ix centrado em x para ter aı́ um ponto de mı́nimo).
Ou seja,
φ(x) > φ(x), ∀x ∈ Ix \ {x},
que significa
f (x) > ax + b, ∀x ∈ Ix \ {x},
como querı́amos provar.

De ii): Análogo, bastando usar o Critério da Segunda Derivada para ter um


máximo local.

5. CONCAVIDADES DOS GRÁFICOS 148

Na Definição 5.2 a seguir impomos um comportamento global sobre a função: ela


terá que ficar por cima (ou por baixo) de todas as retas tangentes a seu gráfico.
Definição 5.2. Direi que uma função f : I → R é côncava para cima se para todo
ponto x ∈ I,
f (x) > ax + b, ∀x ∈ I \ {x}
onde y = ax + b é a reta tangente ao gráfico em (x, f (x)).

25

20

15

10

0
-3 -2 -1 0 1
x
-5

Figura: Um função que não é côncava para cima, mas que


é localmente localmente côncava para cima se x < 0.
Afirmação 5.2. Suponha uma função f : I → R duas vezes derivável.
• i) Se ∀x ∈ I f ′′ (x) > 0 então f é côncava para cima.
• ii) Se ∀x ∈ I f ′′ (x) < 0 então f é côncava para baixo.
Demonstração.
De i):
Vamos fazer a prova por absurdo.
Pela Afirmação 5.1 sabemos f é localmente concava para cima em cada ponto de
seu domı́nio. Ou seja, dado qualquer x ∈ I existe um intervalo Ix centrado nele onde
f (x) > ax + b, ∀x ∈ Ix \ {x},
para y = ax + b reta tangente em (x, f (x)).
Portanto, se pensamos esta demonstração por absurdo, tem que existir6 algum
ponto (x, f (x)) para o qual existe um x0 ∈
/ Ix tal que
f (x0 ) ≤ ax0 + b,
para y = ax + b reta tangente em (x, f (x)).
Sem perda de generalidade suponhamos x0 > x.
Faço agora uma alteração na f , para que a reta tangente a (x, f (x)) seja horizontal.
Defino
φ(x) := f (x) − (ax + b).
Note que φ(x) = φ (x) = 0, mas φ′′ (x) = f ′′ (x) > 0, ∀x ∈ I. Agora temos

φ(x0 ) ≤ 0.
6Confira um exemplo disso na Figura anterior, com x ∼ −0.5 e x0 ∼ 1
CAPÍTULO 11. APLICAÇÕES DA PRIMEIRA E SEGUNDA DERIVADAS 149

Caso φ(x0 ) = 0:
Nesse caso, aplico o Teorema de Rolle a
φ : [x, x0 ] → R
e obtenho um ponto ξ ∈ (x, x0 ) onde φ′ (ξ) = 0.
Mas ξ > x e isso contradiz o fato que φ′ (x) é uma função estritamente crescente
(já que φ′′ (x) > 0), que partiu do valor φ′ (x) = 0.

Caso φ(x0 ) < 0:


Pelo que vimos na Afirmação 5.1, perto de x temos φ(x) > 0.
Como φ(x) é contı́nua e φ(x0 ) < 0 então o T.V.I. diz que há um ponto x̂0 ∈ [x, x0 ]
onde φ(x̂0 ) = 0. Portanto com esse novo x̂0 recaio na situação do Caso φ(x̂0 ) = 0 já
tratado.

De ii): completamente análoga. 

6. Mı́nimos quadrados e a média aritmética


Dados x1 , . . . , xk pontos na Reta dos Reais, que ponto x minimiza a soma dos
quadrados das distâncias a todos eles ?
O interesse prático desta questão é que os valores x1 , . . . , xk podem ter sido obtidos
após k aferições de um certo dado relevante (o comprimento de um objeto, uma
temperatura, um peso, etc) e o ponto x servirá para corrigir os prováveis erros nas
aferições.
Afirmação 6.1. Sejam dados x1 , . . . , xk ∈ R pontos. Então
• i) o ponto de mı́nimo global da função
f (x) := (x − x1 )2 + . . . + (x − xk )2
é o ponto
x1 + . . . + xk
x= ,
k
chamado de média arimética dos valores x1 , . . . xk .
• ii) sempre vale a desigualdade
k · (x21 + . . . + x2k ) > (x1 + . . . + xk )2
exceto se x1 = . . . = xk , quando vale então:
k · (x21 + . . . + x2k ) = (x1 + . . . + xk )2 .
Demonstração.
Item i)
Trata-se então de minimizar a função:
y = f (x) := (x − x1 )2 + . . . + (x − xk )2 .
que é uma parábola com concavidade para cima, já que:
f (x) = k · x2 − 2 · (x1 + . . . xk ) · x + (x21 + . . . + x2k ).
6. MÍNIMOS QUADRADOS E A MÉDIA ARITMÉTICA 150

Portanto seu mı́nimo está onde f ′ (x) = 0, ou seja, na raı́z de:


2k · x − 2 · (x1 + . . . xk ) = 0,
ou seja, em
x1 + . . . + xk
x=
k
que é chamada de média aritmética dos valores x1 , . . . xk .

Item ii)
Note que, por ser uma soma de quadrados,
y = f (x) = (x − x1 )2 + . . . + (x − xk )2 ≥ 0
e se para algum x0 ∈ R temos f (x0 ) = 0 então
(x0 − x1 )2 + . . . + (x0 − xk )2 = 0 ⇔ x0 = x1 = . . . = xk .
Portanto, se algum xi é diferente de algum outro xj , na lista que demos de x1 , . . . , xk ,
a equação quadrática em x:
y = f (x) = k · x2 − 2 · (x1 + . . . xk ) · x + (x21 + . . . + x2k ) = 0
não tem solução Real. Ou seja, se seu discriminante é negativo. Mas esse discrimi-
nante é:
(2 · (x1 + . . . xk ))2 − 4 · k · (x21 + . . . + x2k ) < 0,
ou seja,
(x1 + . . . xk )2 < k · (x21 + . . . + x2k ),
como querı́amos.


6.1. Retas de ajuste.


Agora trato de um problema parecido, mas diferente. Que só será considerado no
caso geral na Seção 3 do Capı́tulo 34.
Considere o quadrado da distância vertical de um ponto (x1 , y1) a uma reta y =
ax + b, ou seja:
(ax1 + b − y1 )2 ≥ 0
e = 0 exatamente quando (x1 , y1 ) está na reta.
Suponhamos que queremos encontrar a reta pela origem y = ax (não vertical) que
minimiza a soma dos quadrados das distâncias verticais até k pontos (x1 , y1 ), . . . (xk , yk )
(não todos os xi iguais a zero).
Denote as retas pela origem por y = ξx para deixar claro que a incógnita agora é
o coeficiente angular ξ.
E faça a função que dá a soma de quadrados de distâncias verticais:
f (ξ) := (ξx1 − y1 )2 + . . . + (ξxk − yk )2 .
Note que
f (ξ) = (x21 + . . . + x2k ) · ξ 2 − 2(x1 y1 + . . . + xk yk )ξ + y12 + . . . + yk2 .
CAPÍTULO 11. APLICAÇÕES DA PRIMEIRA E SEGUNDA DERIVADAS 151

Então f (ξ) é uma parábola com concavidade para cima, já que
x21 + . . . + xk2 > 0
(se esse número fosse zero todos os pontos tem coordenada x igual a zero).
Portanto se procuramos por um mı́nimo de f basta procurarmos onde f ′ (ξ) = 0.
Mas:
f ′ (ξ) = 2(x21 + . . . + x2k ) · ξ − 2(x1 y1 + . . . + xk yk ),
e portanto f ′ (ξ) = 0 se dá em:
x1 y1 + · · · + xk yk
ξ= .
x21 + . . . + x2k
Ou seja a reta a ser escolhida é:
x1 y1 + · · · + xk yk
y=( ) · x.
x21 + . . . + x2k
O problema interessante em geral é quando a reta buscada forma y = ξx + τ não
precisa passsar pela origem.
Essa reta aproximará simultâneamente vários pontos, que podem ser resultado de
aferições de dados relevantes.
O Capı́tulo 34 tratará de uma reta que minimiza soma de quadrados de distâncias
verticais de pontos xi , yi de interesse na Biologia, e cujo coeficiente angular ξ é uni-
versal.

7. Pontos de inflexões dos gráficos


Definição 7.1. Seja f contı́nua em I, intervalo aberto, e duas vezes derivável ao
menos em I \ {x}.
Chamamos x de ponto de inflexão da f se o sinal da f ′′ (x) muda em torno de x.
Ou seja, um ponto de inflexão marca a mudança de concavidade de uma função
(se era para cima, vira para baixo e vice-versa).

Exemplos:
• y = f (x) = x3 , que tem f ′′ (x) = 6x e ponto de inflexão em x = 0.
• em geral, y = f (x) = x2n+1 , ∀n ∈ N, têm inflexão em x = 0, já que
f ′′ (x) = 2n · (2n + 1) · x2n−1 .
1 4
• a função y = 4x 3 −x 3 é contı́nua em torno da origem, mas tem reta tangente
vertical na origem, ou seja não existe f ′ (0). Como
4(2 + x)
f ′′ (x) = − 5
x3
isso diz que f ′′ (x) > 0 para −2 < x < 0 e f ′′ (x) < 0 para x > 0, ou seja,
x = 0 é ponto de inflexão. Também f ′′ (x) < 0 para x < −2 e portanto
x = −2 é outro ponto de inflexão.
8. CRITÉRIO DA DERIVADA DE ORDEM N 152

• o gráfico de y = f (x) (em vermelho) na Figura a seguir representa a pop-


ulação de bactérias colocada num meio favorável, no tempo x.
A taxa de crescimento f ′ (x) (em verde) vai aumentando até atingir um
valor máximo (no ponto de inflexão x ≈ 1.1.), a partir do qual fatores como
escassez de nutrientes, aumento de detritos, começam a diminuir essa taxa
de crescimento.
No ponto de inflexão a aceleração f ′′ (x) do processo (em amarelo) é nula.

2
x
0 0,5 1 1,5 2 2,5 3
0

-2

-4

-6

A função f (x) será dada explicitamente nas Seções 4 e 5 do Capı́tulo 38.

8. Critério da derivada de ordem n


Uma função como y = f (x) = sin4 (x) claramente tem um ponto de mı́nimo local
em x = 0, já que se anula em zero e é positiva por perto. No entanto
f ′′ (x) = 4 sin(x)2 · (4 cos(x)2 − 1) e f ′′ (0) = 0,
por isso não está ao alcance do critério da segunda derivada (Afirmação 2.1). Também
f ′′′ (x) = 8 sin(x) cos(x) · (8 cos(x)2 − 5)
se anula em x = 0, porém:
f (iv) (x) = 256 cos(x)4 − 272 cos(x)2 + 40
tem valor f (iv) (0) = 24.

A Afirmação 2.1 se generaliza assim:


Afirmação 8.1. Suponha f : (a, b) → R com derivadas de todas as ordens7. Seja
n ∈ N.
7Não confunda a derivada de ordem n, f (n) , com a potência n-ésima f n .
CAPÍTULO 11. APLICAÇÕES DA PRIMEIRA E SEGUNDA DERIVADAS 153

i) se f ′ (x) = f ′′ (x) = . . . = f (2n−1) (x) = 0 mas f (2n) (x) > 0 então x é ponto de
mı́nimo local.
ii) se f ′ (x) = f ′′ (x) = . . . = f (2n−1) (x) = 0 mas f (2n) (x) < 0 então x é ponto de
máximo local.
ii) se f ′ (x) = . . . = f (2n) (x) = 0 mas f (2n+1) (x) 6= 0 então x é ponto de inflexão.
Demonstração.
Item i):
A prova completa seria ∀n ∈ N e aı́ então a indução matemática seria exigida.
Por isso, para simplificar mas mesmo assim dar uma ı́déia da prova, me atenho ao
primeiro caso relevante, ou seja quando
n = 2.
Temos por hipótese:
f ′ (x) = f ′′ (x) = f ′′′ (x) = 0 mas f (iv) (x) > 0.
Como há derivadas de todas as ordens, a função f (iv) (x) é contı́nua em x, pois é até
mesmo derivável. Logo pelo princı́pio de inércia das funções contı́nuas, existe um
intervalo Ix = (−δ + x, x + +δ) centrado em x tal que
f (iv) (x) > 0, ∀x ∈ Ix .
Então no intervalo Ix a função f ′′′ (x) é uma função estritamente crescente. Como por
hipótese f ′′′ (x) = 0, concluimos que:
f ′′′ (x) < 0 em (−δ + x, x) e f ′′′ (x) > 0 em (x, x + δ).
Ou seja que a função f ′′ (x) é estritamente decrescente em (−δ + x, x) e f ′′ (x) é
estritamente crescente em (x, x + δ). Como f ′′ (x) = 0 isso diz que:
f ′′ (x) > 0 em (−δ + x, x) ∪ (x, x + δ).
Agora então f ′ (x) é estritamente crescente em (−δ +x, x)∪(x, x+δ). Como f ′ (x) = 0
temos que
f ′ (x) < 0 em (−δ + x, x) e f ′ (x) > 0 em (x, x + δ).
Por último isso diz que f é estritamente decrescente em (−δ + x, x) e f é estritamente
crescente em ((x, x + δ). Logo x é ponto de mı́nimo.

Iem ii): Análogo, mutatis mutandis.

Item iii):
Temos por hipótese:
f ′ (x) = f ′′ (x) = f ′′′ (x) = f (iv) (x) = 0
mas f (v) (x) 6= 0. Por exemplo suponhamos
f (v) (x) > 0.
o caso negativo é análogo.
9. CONFECÇÃO DE GRÁFICOS DE POLINÔMIOS 154

Como há derivadas de todas as ordens, a função f (v) (x) é contı́nua em x, pois é
até mesmo derivável. Logo pelo princı́pio de inércia das funções contı́nuas, existe um
intervalo Ix = (−δ + x, x + +δ) centrado em x tal que
f (v) (x) > 0, ∀x ∈ Ix .
Então no intervalo Ix a função f (iv) (x) é uma função estritamente crescente. Como
por hipótese f (iv) (x) = 0, concluimos que:
f (iv) (x) < 0 em (−δ + x, x) e f (iv) (x) > 0 em (x, x + δ).
Ou seja que a função f ′′′ (x) é estritamente decrescente em (−δ + x, x) e f ′′′ (x) é
estritamente crescente em (x, x + δ). Como f ′′′ (x) = 0 isso diz que:
f ′′′ (x) > 0 em (−δ + x, x) ∪ (x, x + δ).
Agora então f ′′ (x) é estritamente crescente em (−δ+x, x)∪(x, x+δ). Como f ′′ (x) = 0
temos que
f ′′ (x) < 0 em (−δ + x, x) e f ′′ (x) > 0 em (x, x + δ).
Por definição, x é um ponto de inflexão.


9. Confecção de gráficos de polinômios


Considere a função polinomial y = f (x) = x3 − x.
O objetivo é fazer seu gráfico, de modo qualitativamente correto, sem qualquer
calculadora.
Primeiro noto onde f = 0, onde f > 0 ou f < 0 (pois essas informações não serão
fornecidas pela f ′ (x)).
Ora f (x) = x · (x2 − 1) e daı́ sai que
• f (x) = 0 exatamente para x = 0, −1, 1;
• f (x) > 0 para −1 < x < 0 ou x > 1;
• f (x) < 0 para x < −1 ou 0 < x < 1.
A derivada é f ′ (x) = 3x2 − 1 e portanto
q q
• f (x) = 0 em x = 3 , − 13 .
′ 1
q q
• f ′ (x) > 0 se x > 13 ou x < − 13 .
q q
• f ′ (x) < 0 se − 13 < x < 13 .
• f ′ (0) = −1
q
Essas informações sobre f (x) já dizem que x = 13 é ponto de mı́nimo local de

q
f (x) e que x = − 13 é ponto de máximo local de f (x). E também que f é crescente
q q q q
se x > 13 ou x < − 13 e que f (x) é decrescente se − 13 < x < 13 . Por último,
f ′ (0) = −1 diz que o gráfico perto da origem se parece com y = −x.
CAPÍTULO 11. APLICAÇÕES DA PRIMEIRA E SEGUNDA DERIVADAS 155

Agora f ′′ (x) = 6x, ou seja f ′′ (0) = 0, e em x = 0 há mudança de sinal da f ′′ (x).


Logo x = 0 é ponto de inflexão. Para x < 0 a concavidade de f é para baixo e para
x > 0 a concavidade de f é para cima.
A Figura a seguir recolhe essas informações, mas como as escalas são diferentes
nos dois eixos a informação f ′ (0) = −1 não é respeitada:

0
-1,5 -1 -0,5 0 0,5 1 1,5
x

-4

-8

Figura: y = f (x) = x3 − x (verm.), f ′ (x) (verde), f ′′ (x) (amar.)

Os Exercı́cios 10.5 e 10.6 desafiarão o leitor a fazer gráficos qualitativamente cor-


retos de polinômios, sem usar nenhuma calculadora.
Para compreender mais unificadamente a variedade de gráficos de funções cúbicas
do tipo y = ax3 + bx2 + cx + d, o leitor pode ler o Capı́tulo 32.
Na Seção 4 do Capı́tulo 14 faremos gráficos de funções racionais, quocientes de
polinômios.

10. Exercı́cios
2
Exercı́cio 10.1. 3) Encontre o ponto do gráfico de y = x2 que minimiza a distância
até P = (2, 1) pelos metodos i): de buscar pontos de ortogonalidade com o gráfico e
ii): via mı́nimo da função quadrado da distância.
Exercı́cio 10.2. 4) As Figuras i) e ii) abaixo dão dois exemplos de funções derivadas
f ′ (x), apenas dadas qualitativamente. Encontre f (x) (qualitativamente) que sejam
compatı́veis com cada f ′ dada.

0
-3 -2 -1 0 1 2 3
x
-2

-4

-6
10. EXERCÍCIOS 156

Figura i): Gráfico de uma função derivada f ′ .

15

10

5
x
-2 -1 0 1 2 3 4
0

-5

-10

-15

-20

Figura ii): Gráfico de uma função derivada f ′ .


Exercı́cio 10.3. A Figura mostra o gráfico de uma função e o de sua derivada. Qual
é qual e por quê ? (Justifique analisando a relação entre zero/sinal da f ′ e a f ter
máximo/mı́nimo ou ser crescente/decrescente).

80

40

0
-2 -1 0 1 2 3 4
x

-40

-80

Exercı́cio 10.4. Veja o gráfico a seguir como o gráfico de uma função derivada
y = f ′ (x).
i) Sobreponha a ele o gráfico de uma y = f (x) qualitativamente compatı́vel
(Atenção à relação entre zero/sinal de f ′ (x) e máximo, mı́nimo, crecimento, decresci-
mento da f ).
ii) faça com detalhe a região da f que corresponde ao máximo da f ′ (x).

1
x
-2 -1 0 1 2 3
0

-1

-2

-3

-4

Exercı́cio 10.5. (resolvido)


O objetivo deste Exercı́cio é confeccionar gráficos apenas qualitativamente corre-
tos, sem qualquer tipo de calculadora, de polinômios relativamente simples como:
i) y = f1 (x) = x3 − x2
ii) y = f2 (x) = x2 − x3 .
CAPÍTULO 11. APLICAÇÕES DA PRIMEIRA E SEGUNDA DERIVADAS 157

iii) y = f3 (x) = −2x2 + x3


iv): y = f4 (x) = x4 − 2x2 .
v): y = f5 (x) = 3x4 − 4x3 .
Faça-o seguindo o seguinte roteiro:
a) determine os zeros de f , e em quais intervalos a função f é positiva ou negativa.
b) calcule a derivada f ′ .
c) determine os zeros da função derivada f ′ , e em quais intervalos a função derivada
é positiva ou negativa.
d) calcule a segunda derivada e determine onde ela é zero, positiva e negativa.
e) com as informações de a), b), c) e d) esboce o gráfico de f ′′ (x); com base nesse,
o de f ′ (x) e com base nesse o de f (x).
Dica: em cada item fatore a maior potência possı́vel de x e então, para examinar
onde cada função é positiva e negativa basta usar a regra de multiplicação dos sinais:
+ · + = +, + · − = − e − · − = +.
Depois de pensar bastante, pois cada item pode exigir tempo, confira seus resul-
tados com as Soluções no Capı́tulo 52.

Exercı́cio 10.6. (resolvido)


Suponhamos que, seguindo o roteiro do Exercı́cio anterior, você entendeu o gráfico
de y = x3 − C · x2 , onde C ≥ 1 é uma constante.
E que chegou em algo do seguinte tipo:

x
-4 -2 0 2 4
0

-20

-40

-60

-80

-100

Sem fazer nenhuma conta mais, apenas raciocinando geometricamente, como deve
ser o gráfico de y = x3 + C · x2 ? (para C ≥ 1).

Exercı́cio 10.7. Dê um exemplo bem simples de uma f : [a, b] → R contı́nua tal
que f ′ (x) 6= 0 ∀x ∈ (a, b). Localize em seu exemplo onde estão o(s) máximo(s) e
mı́nimo(s).

Exercı́cio 10.8. Considere o ângulo formado no primeiro quadrante pelo eixo dos
y > 0 e a reta y = a · x, onde a > 0 será fixado.
Considere um ponto (A, B) nessa região (ou seja suponho B > a · A > 0).
10. EXERCÍCIOS 158

Qual a reta passando por (A, B) forma (no primeiro quadrante) um triângulo com
o eixo dos y > 0 e a reta y = ax de menor Área ?
Prove que a menor área é 2A · (B − Aa).
A figura ilustra três candidatas:
pz

tz
rz

Dica: lembre como calcular a área de um triângulo via determinante.


Exercı́cio 10.9. Encontre dois números x, y pertencentes ao intervalo [0, 1] cuja soma
é x + y = 1 e tais que
i) x2 + y 2 é máximo (justifique)
ii) x2 + y 2 é mı́nimo (justifique).
iii): para responder ao i) e ii) você estudou máximo e mı́nimo de uma função f (x).
Esboce seu gráfico, indicando onde sua derivada f ′ (x) é negativa, zero ou positiva.
Exercı́cio 10.10. Uma fábrica de azulejos fabrica pequenos revestimentos cerâmicos
(pastilhas) retangulares, que têm x cm de largura e y cm de comprimento.
O perı́metro de cada pastilha será fixado em 2 · (x + y) = 2.
i) descreva a função que dá a Área de cada pastilha como uma função A(x) só de
x.
ii) em qual domı́nio A(x) não é negativa ? Onde A(x) se anula ? Onde A(x) é
positiva ?
iii) Esboce o gráfico de A(x) (apenas qualitativamente). Como determinar x para
que o valor de A(x) seja máximo ?
iv) qual o formato e medidas da pastilha de maior Área ?
Exercı́cio 10.11. O custo de fabricação um objeto Retangular é dado por C(x, y) =
x3
6
+ y, pois o material usado na fabricação da lateral x é muitı́ssimo mais caro que o
da frente y. Supondo que sempre 1 ≤ x e que a Área tem que ser igual a 8, quais as
medidas x, y que minimizam o custo de fabricação ?
Exercı́cio 10.12. O custo de fabricação um objeto Retangular é dado por C(x, y) =
x2 + y, pois o material usado na fabricação da lateral x é muito mais caro que o da
frente y. Supondo que sempre 1 ≤ x e que a Área tem que ser igual a 16, quais as
medidas x, y que minimizam o custo de fabricação ?
CAPÍTULO 11. APLICAÇÕES DA PRIMEIRA E SEGUNDA DERIVADAS 159

Um aluno pensou assim sobre esse problema: já que o custo em função de x é
muito maior que em função de y, por que não usar o mı́nimo de x, ou seja, x = 1 e
y = 16, obtendo área de 16 e custo de 12 + 16 = 17 ?
Será que ele está certo ? Esse é mesmo o mı́nimo de custo ?

Exercı́cio 10.13. A área de um objeto retangular é A(x, y) = xy. O custo da


construção depende das dimensões x e y segundo a fórmula C(x, y) = 5x2 + y.
Maxime a área supondo fixado o custo em C(x, y) = 30.

Exercı́cio 10.14. Explique com os conceitos do Cálculo que relação pode haver entre
os dois gráficos apresentados em cada uma das três Figuras que seguem.
ii) Que muda de uma Figura para a outra ? O que não muda ?
iii) destaque propriedades geométricas relevantes de cada Figura (mı́nimos/máximos,
inflexões, raı́zes, etc).

10

0
-2 -1 0 1 2
x
-5

-10

10

0
-2 -1 0 1 2
x

-5

10

0
-2 -1 0 1 2
x
-2

-4

Exercı́cio 10.15. Entendendo zeros e sinais de , de sua derivada f ′ e da segunda


derivada f ′′ , confeccione o gráfico de f ′′ , o de f ′ e o de f , qualitativamente.
Apresente um gráfico acima do outro, identificando pontos importantes.

Exercı́cio 10.16. Entendendo zeros e sinais de f (x) = x2 − x3 , de sua derivada f ′ e


da segunda derivada f ′′ , confeccione o gráfico de f ′′ , o de f ′ e o de f , qualitativamente.
Apresente um gráfico acima do outro, identificando pontos importantes.

Exercı́cio 10.17. (resolvido)


Considere a Figura a seguir, que dá em vermelho o gráfico de y = x3 restrito a
x ∈ (−2, 1) e, em verde, o gráfico de x3 − 3x2 + 3x − 2 também para x ∈ (−2, 1).
10. EXERCÍCIOS 160

Prove que existe uma reta que apenas tangencia o gráfico verde e que consegue
passar entre os dois gráficos sem intersectar o gráfico vermelho.
Dica: a Figura sugere uma reta, prove que ela satisfaz o que se pede.
Exercı́cio 10.18. (resolvido)
Seja f derivável (tantas vezes quanto quiser).
Suponha que y = f (x) está definida na semireta [0, +∞) e tem sempre f ′′ (x) < 0
(concavidade para baixo em todo seu domı́nio).
Suponha que em um certo x valem f (x) > 0 e f ′ (x) < 0.
Determine um K para o qual se pode garantir que f (x) = 0 em algum ponto
x ∈ [x, K].
CAPı́TULO 12

Derivadas de seno e cosseno e as leis de Hooke

Hooke é sempre associado aos temas expostos na próxima Seção. Mas sua im-
portância cientı́fica vai muito além disso, como mostra o trecho da carta de Hooke
a Newton, de 1689, citado por James Gleick em Isaac Newton, uma biografia, Com-
panhia das Letras, p.132:

Resta agora conhecer as propriedades de uma linha curva [...] feita por uma
força atrativa central [...] em uma uma proporção duplicada em relação às distâncias
tomadas reciprocamente. Não duvido que por seu excelente método o senhor desco-
brirá [...]

1. O cosseno como derivada do seno


No final de Star Wars descobrimos queo mocinho é filho do grande vilão. Pois
nesta Seção vamos descobrir que o cosseno é a derivada do seno !
A derivada do seno em θ = 0 foi vista: sin′ (0) = 1 (Seção 5 do Capı́tulo 5 da
Parte 1).
Ou seja, sin′ (0) = cos(0). Será que isso é uma coincidência apenas? Ou será que
sin′ (θ) = cos(θ), ∀θ ∈ R ?
Vamos pôr um gráfico abaixo do outro e ver se são os gráficos são coerentes com
o que aprendemos no Capı́tulo 7 da Parte 1, sobre como a derivada determina o
comportamento de uma função.

1
0,5
0
0 1 2 3 4 5 6
-0,5 x
-1

Figura: O gráfico de y = sin(θ) (vermelho) e y = cos(θ)


(verde), para θ ∈ [0, 2π].

Observe que:
161
1. O COSSENO COMO DERIVADA DO SENO 162

• em θ = π2 ≈ 1.6 o seno tem seu máximo e nesse ponto θ = π2 o cosseno se


anula, passando de positivo para negativo.
• em θ = π ≈ 3.1 o cosseno tem seu mı́nimo −1 e nesse ponto θ = π a inclinação
do gráfico do seno parece ser −1. Ademais, as inclinações do gráfico do seno
vinham ficando mais negativas desde π2 e a partir de θ = π vão ficando menos
negativas.
• em θ = 3π 2
≈ 4.7 o cosseno se anula, passando de negativo a positivo e em
θ = 3π
2
o seno tem seu mı́nimo.
• por último, onde o cosseno é positivo (negativo) o seno é crescente (decres-
cente).
Todas essas observações são coerentes com o que aprendemos no final da Parte 1
e de fato:
Afirmação 1.1.
sin′ (θ) = cos(θ), ∀θ ∈ R.
Demonstração.
Começo com a definição de derivada em algum θ0 fixado e uso depois a formula
de seno de uma soma:

sin(θ0 + θ) − sin(θ0 )
sin′ (θ0 ) = lim =
θ→0 θ
sin(θ0 ) cos(θ) + cos(θ0 ) sin(θ) − sin(θ0 )
= lim .
θ→0 θ
Para poder continuar, agora vou usar o limite provado na Seção 3 do Capı́tulo 8:
sin(θ)
lim =1
θ→0 θ
e, ademais, um outro limite fundamental:
cos(θ) − 1
lim = 0,
θ→0 θ
cuja prova omito, mas que é no mesmo estilo.
Então as propriedades de limites de somas e produtos permitem que re-escreva o
de acima como:
(cos(θ) − 1) sin(θ)
sin′ (θ0 ) = lim [sin(θ0 ) · + cos(θ0 ) · ]=
θ→0 θ θ
(cos(θ) − 1) sin(θ)
= sin(θ0 ) · lim + cos(θ0 ) · lim =
θ→0 θ θ→0 θ
= sin(θ0 ) · 0 + cos(θ0 ) · 1 = cos(θ0 ),
como querı́amos. 

Um complemento:
A Figura a seguir exibe os gráficos de
sin(θ)
f1 (θ) = , para θ 6= 0 e f1 (0) := 1
θ
CAPÍTULO 12. DERIVADAS DE SENO E COSSENO E AS LEIS DE HOOKE163

e de
cos(θ) − 1
f2 (θ) = , para θ 6= 0 e f2 (0) := 0
θ
(note que defino separadamente os valores para θ = 0, para que as funções resultantes
sejam contı́nuas).

0,8
0,4
0
-3 -2 -1 0 1 2 3
-0,4
x

Figura: O gráficos de y = f1 (θ) (vermelho) e y = f2 (θ)


(verde) para θ ∈ [−π, π].

A vingança do cosseno ! Seu filho (sua derivada) é o oposto do malvado avô, o


seno:

Afirmação 1.2.
cos′ (θ) = − sin(θ), ∀θ ∈ R.
Demonstração. Seguindo as mesmas etapas da prova anterior, obtemos:
cos(θ0 + θ) − cos(θ0 )
cos′ (θ0 ) = lim =
θ→0 θ
cos(θ0 ) cos(θ) − sin(θ0 ) sin(θ) − cos(θ0 )
= lim =
θ→0 θ
(cos(θ) − 1) sin(θ)
= cos(θ0 ) · lim − sin(θ0 ) · lim =
θ→0 θ θ→0 θ
= cos(θ0 ) · 0 − sin(θ0 ) · 1 = − sin(θ0 ).
como querı́amos. 

2. Leis de Hooke com e sem atrito


A lei de Hooke diz que a força que um objeto1 sofre quando se estica uma mola
presa a ele é do tipo
F = −kf (x)
1Os objetos inicialmente serão tratados como pontos, o que é uma enorme simplificação da
realidade. Na Seção 5 do Capı́tulo 23 falaremos de centro de gravidade de objetos que não são
pontos
2. LEIS DE HOOKE COM E SEM ATRITO 164

onde k > 0 é uma constante e f (x) é a posição do objeto (veja a Figura a seguir). O
sinal negativo significa que a força é no sentido oposto do deslocamento. Se ignora o
atrito entre o objeto e a superfı́cie nessa formulação da lei.

Se tomamos a força F como sendo o produto de massa m pela aceleração f ′′ (x)


então a lei de Hooke é da forma
mf ′′ (x) = −k · f (x).
A seguir, na Afirmação 2.1, para simplificar e dispensar a derivada da composta
(que não vimos ainda), ponho k = 1.
Afirmação 2.1.
i): As funções f (x) = a · cos(x) + b sin(x) são periódicas de perı́odo 2π, têm
f (0) = a e f ′ (0) = b e satifazem
f ′′ (x) = −f (x), ∀x ∈ R.
ii): Ademais a · cos(x) + b sin(x) ≡ A · cos(x − q), onde
√ a
A = a2 + b2 e cos(q) = √ .
a2 + b2

A Afirmação 2.1 será reforçada na Seção 8 do Capı́tulo 39, onde se mostrará, entre
outras coisas, que as funções f (x) = a·cos(k ·x)+b sin(k ·x) são as únicas a satisfazer:
f ′′ (x) = −k · f (x), k ∈ R.

Demonstração. (da Afirmação 2.1)


De i):
Como o seno e o cosseno têm perı́odo 2π essas funções também têm esse perı́odo.
Pela derivada da soma e de seno e cosseno, obtemos
f ′′ (x) = (f ′ (x))′ = (a(− sin(x)) + b cos(x))′ =
= −a cos(x) − b sin(x) = −f (x).
Ademais, f (0) = acos(0) = a e f ′ (0) = b cos(0) = b.
De ii):
Note para o que segue que, se cos(q) = √a2a+b2 , então
b
sin(q) = √ .
a2 + b2
Temos então
A · cos(x − q) = A · [cos(x) · cos(−q) − sin(x) · sin(−q) =
CAPÍTULO 12. DERIVADAS DE SENO E COSSENO E AS LEIS DE HOOKE165

= A · [cos(x) · cos(q) + sin(x) · sin(q)] =


√ a √ b
= a2 + b2 · √ · cos(x) + a2 + b2 · √ · sin(x) =
2
a +b 2 a + b2
2

= a · cos(x) + b · sin(x),


Na figura a seguir note que não só a posição f (0) é relevante, mas que também a
inclinação f ′ (0) determina o tipo de oscilação que haverá.

0
0 1 2 3 4 5 6
x
-1

-2

Figura: Gráficos de y = a sin(θ) + b cos(θ) para alguns a, b e θ ∈ [0, 2π].

Claro que na realidade fı́sica sempre há algum atrito entre o objeto e a superfı́cie
e sabemos que com o tempo o objeto pára. Uma lei de Hooke mais realista levaria
em conta o atrito que surge com o deslocamento do objeto, ou seja, dependente da
velocidade f ′ (x) do objeto e seria do tipo
f ′′ (x) = −f (x) − kf ′ (x).
Na Figura a seguir ponho uma função satisfazendo f ′′ (x) = −f (x) ao lado de uma
função satisfazendo f ′′ (x) = −f (x)−0.1·f ′ (x). Uma função deste último tipo envolve
senos e cossenos e a função exponencial, que veremos mais adiante.

0,5

0
0 5 10 15 20 25 30 35
x
-0,5

-1

Figura: Funções satisfazendo a lei de Hooke


sem atrito (vermelho) e com atrito (verde).
3. EXERCÍCIOS 166

E se o atrito for maior, por exemplo, em f ′′ (x) = −f (x) − 0.3 · f ′ (x), então nesse
caso o objeto vai parar bem mais rápido, como na Figura a seguir:

0,5

0
0 5 10 15 20 25 30 35
x
-0,5

-1

Figura: Funções satisfazendo a lei de Hooke


sem atrito (vermelho) e com muito atrito (verde).

Resolveremos explicitamente a equação diferencial:


f ′′ (x) − f (x) − kf ′ (x)
na Seção 2 do Capı́tulo 40.
3. Exercı́cios
Exercı́cio 3.1. Determine se o ponto (0, 0) é máximo/mı́nimo ou inflexão de f,
sabendo que f ′ (x) = sen5 (x) · cos(x).
CAPı́TULO 13

Derivada do produto, indução e a derivada de xn, n ∈ Z.

Já vimos que a derivada de f (x) = 1 = x0 é f ′ (x) = 0, que a de f (x) = x = x1 é


f ′ (x) = 1 = 1x0 , que a de f (x) = x2 é f ′ (x) = 2x1 e até mesmo que a de f (x) = x4 é
f ′ (x) = 4x3 .
Ou seja, nos sentimos motivados a conjecturar que ∀n ∈ N, f (x) = xn tem
f (x) = nxn−1 .

Como podemos provar isso, se não podemos percorrer todos os Naturais ? Isso se
faz através do princı́pio de indução matemática.

1. Princı́pio de indução matemática


Em geral a palavra indução é usada nas ciências experimentais para referir ao
processo pelo qual alguém tenta concluir após um certo número de evidências que
certo fenômeno valerá sempre (ou qual a probabilidade disso ocorrer).
Já em matemática o significado é o seguinte: quando queremos provar uma certa
propriedade para todo n ∈ N, o que fazemos é:
• prová-la para n = 1,
• supô-la válida até n − 1 e
• prová-la para o próximo natural, ou seja, para n.
(A etapa em que supomos a propriedade válida até n − 1 é chamada de hipótese de
indução).
Se conseguimos fazer essa última etapa, a propriedade vale para todo n ∈ N.
A validade deste princı́pio está ligada à própria natureza (axiomas) dos números
Naturais.
Vejamos três exemplos, que além de bonitos em si mesmos, serão úteis mais adiante
no Capı́tulo 21:

Afirmação 1.1. ∀n ∈ N:
i) 1 + 2 + . . . + (n − 1) + n = (n+1)·n
2
.
ii) (1 + 2 + . . . + (n − 1) + n) = 1 + 23 + . . . + (n − 1)3 + n3 .
2 3

iii) 12 + 22 + . . . + n2 = n(n+1)(2n+1)
6

Demonstração.
2·1
Prova de i): Para n = 1 a fórmula diz simplesmente 1 = 2
o que é óbvio.
A hipótese de indução é
((n − 1) + 1) · (n − 1) n(n − 1)
1 + 2 + . . . + (n − 1) = = .
2 2
167
1. PRINCÍPIO DE INDUÇÃO MATEMÁTICA 168

De agora em diante temos que fazer algo para mostrar quanto vale 1 + 2 + . . . + (n −
1) + n. Ora
1 + 2 + . . . + (n − 1) + n = (1 + 2 + . . . + (n − 1)) + n =
n(n − 1) n(n − 1) + 2n
= +n= =
2 2
(n + 1) · n
= ,
2
como querı́amos.
Prova de ii): Para n = 1 a fórmula diz simplesmente que 12 = 13 o que é óbvio.
Faço a hipótese de indução:
(1 + 2 + . . . + (n − 2) + (n − 1))2 = 13 + 23 + . . . + (n − 2)3 + (n − 1)3 ,
e quero saber se vale também:
(1 + 2 + . . . + (n − 1) + n)2 = 13 + 23 + . . . + (n − 1)3 + n3 .
Agora vamos ter que fazer algo, trabalhar um pouco. Escrevo pelo binômio:
(1 + 2 + . . . + (n − 1) + n)2 = (1 + 2 + . . . + (n − 1))2 + 2 · (1 + 2 + . . . + (n − 1)) · n + n2
e para continuar uso a hipótese de indução:
(1 + 2 + . . . + (n − 1) + n)2 = 13 + 23 + . . . + (n − 1)3 + 2 · (1 + 2 + . . . + (n − 1)) · n + n2 .
Para terminar onde gostaria, preciso ver que
2 · (1 + 2 + . . . + (n − 1)) · n + n2 = n3 .
Mas posso usar a parte i) já provada para qualquer n, mesmo que da forma n − 1,
obtendo:
n · (n − 1)
(1 + 2 + . . . + (n − 1)) = ,
2
e portanto:
2 · (1 + 2 + . . . + (n − 1)) · n + n2 = (n · (n − 1)) · n + n2 =
= n3 ,
como precisávamos.
1(1+1)(2+1)
Prova de iii): para n = 1 a fórmula está correta 1 = 6
.
suponha válida até n − 1 e faço:
(n − 1)(n − 1 + 1)(2n − 2 + 1)
12 + 22 + . . . (n − 1)2 + n2 = + n2 =
6
3 2
2n − 3n + n
= + n2 =
6
2n3 − 3n2 + n + 6n2
= =
6
2n3 + 3n2 + n n(n + 1)(2n + 1)
= ,
6 6
como querı́amos.
CAPÍTULO 13. DERIVADA DO PRODUTO, INDUÇÃO E A DERIVADA DE
X N , N ∈ Z. 169

2. Derivada do Produto
Voltemos ao problema original: como derivar f (x) = xn ? Para n = 1 já sabemos
que a fórmula x′ = 1x0 está ok.
Gostariamos de supor a fórmula até n − 1 e prová-la então para n, de acordo com
o princı́pio de indução.
Mas quando escrevo xn e tento relacioná-lo com xn−1 só consigo imaginar a
seguinte relação:
xn = x · xn−1 .
Quando for derivar o lado esquerdo dessa expressão terei que derivar, no lado
direito, um produto de funções.
Como fazê-lo ? Certamente a derivada do produto não é o produto das derivadas,
pois (x2 )′ 6= x′ · x′ = 1 · 1.
Por isso precisamos de:
Teorema 2.1. Sejam f (x) e g(x) duas funções deriváveis com mesmo domı́nio de
definição. Então a função produto (f · g)(x) := f (x) · g(x) também é derivável e
(f · g)′ (x) := f ′ (x) · g(x) + f (x) · g ′ (x).
Demonstração.
Seja x e considere a definição de derivada:
f (x + h)g(x + h) − f (x)g(x)
(f · g)′ (x) = lim .
h→0 h
Agora vou fazer um truque, para fazer aparecer f ′ (x) e g ′ (x) nessa estória. Escrevo
f (x + h)g(x + h) − f (x)g(x) =
= f (x + h)g(x + h) −f (x)g(x + h) + f (x)g(x + h) −f (x)g(x) =
| {z }
0
= (f (x + h) − f (x)) · g(x + h) + f (x) · (g(x + h) − g(x)).
Portanto através deste truque obtemos que
(f (x + h) − f (x)) (g(x + h) − g(x))
(f · g)′ (x) = lim [ · g(x + h) + f (x) ].
h→0 h h
Mas limh→0 g(x + h) = g(x) pela continuidade de g e
f (x + h) − f (x) g(x + h) − g(x)
lim = f ′ (x) e lim = g ′ (x),
h→0 h h→0 h
portanto juntando isso (e lembrando que o produto de limites é o limite do produto):
(f · g)′ (x) = f ′ (x)g(x) + f (x)g ′ (x)

3. DERIVADAS DE X −N , ∀N ∈ N 170

Agora estamos em condições de terminar a prova de que


(xn )′ = nxn−1 .
Pra n = 1 vale, suponho válida até n − 1.
Escrevo xn = x · xn−1 e aplico o teorema da derivada do produto:

(x · xn−1 )′ = 1 · xn−1 + x · (xn−1 ) =

= xn−1 + x · (n − 1) · xn−1−1 =

= xn−1 + (n − 1) · xn−1 =

= n · xn−1 .

3. Derivadas de x−n , ∀n ∈ N
Se define x−n := x1n , ∀n ∈ N, onde claramente x 6= 0.
Com essa definição se obtem:
1
x−n · xn = ·n=1
n
e portanto x−n · xn = xn−n .
Queremos derivar essas funções x−n , e novamente o faremos via a indução matemática.
Vimos a derivada de f (x) = x−1 = x1 , x 6= 0 diretamente pela definição, na Parte 1
deste Curso. Como um Exercı́cio, vejamos agora como re-obter a derivada de x−1 = x1
usando a regra da derivada do produto.
Escrevo a identidade para x 6= 0:
1 = x−1 · x
e derivo. Á esquerda na identidade obtenho 0 e à direita a regra do produto dá:
0 = (x−1 )′ · x + x−1 · 1,
ou seja (x−1 )′ = − x12 = −x−2 .
Ou seja, que vale (x−1 )′ = −1 · x−1−1 .
Suponha provada a fórmula até n − 1 > 1: ou seja, que a derivada de x−(n−1) é
−(n − 1) · x−(n−1)−1 = −(n − 1) · x−n .
Então escrevo x−n = x−(n−1) · x−1 e pela derivada do produto:
(x−n )′ = (x−(n−1) )′ · x−1 + x−(n−1) · (−x−2 ) =

= −(n − 1) · x−n · x−1 − x−(n−1)−2 =

= −(n − 1) · x−n−1 − x−n−1 = −n · x−n−1 ,


como querı́amos.
CAPÍTULO 13. DERIVADA DO PRODUTO, INDUÇÃO E A DERIVADA DE
X N , N ∈ Z. 171

4. Raı́zes múltiplas e fatoração de polinômios


Agora que sabemos derivar xn , para qualquer n ∈ N, também saberemos derivar
qualquer polinômio de grau n:
f (x) = an xn + an−1 xn−1 + . . . + a0 , an 6= 0,
bastando para isso usar (n vezes) a regra da derivada da soma/subtração:
f ′ (x) = ( an xn + an−1 xn−1 + . . . + a0 )′ =
= (an xn )′ + (an−1 xn−1 )′ + . . . + a′0 =
= nan xn−1 + (n − 1)an−1 xn−2 + . . . + a1 .
Será conveniente chamar de derivada de ordem zero de uma f (x) a própria
função, em sı́mbolos: f (0) (x) := f (x).
Também chamar de derivada de ordem 1 a derivada usual: f (1) (x) := f ′ (x), bem
como f (2) (x) := f ′′ (x) e assim por diante.
É fundamental o fato seguinte:
Teorema 4.1. Seja f (x) um polinômio de grau n a coeficientes Reais.
São equivalentes as seguintes afirmações:
• i) f (x) = (x − x)k+1 · g(x), onde g(x) é um polinômio de grau n − (k + 1) a
coeficientes Reais.

• ii) f (0) (x) = f (1) (x) = . . . = f (k) (x) = 0 , onde 0 ≤ k ≤ n − 1.


Demonstração.
i) implica ii) :
Suponho f (x) = (x − x)k+1 · g(x), onde g(x) é um polinômio de grau n − (k + 1).
Note que f ′ (x) = (k + 1)(x − x)k g(x) + (x − x)k+1 g ′(x) é uma soma e cada parcela
dessa soma tem um fator (x−x)k ou (x−x)k+1. Asssim também ocorre com qualquer
das derivadas f (i) (x), com 0 ≤ i ≤ k ≤ n − 1: são somas onde cada parcela da soma
tem algum fator dentre:
(x − x)k+1 , (x − x)k , . . . , (x − x)2 , (x − x).
Logo f (i) (x) = 0, se 0 ≤ i ≤ k.

ii) implica i) :
Procederemos por indução em k.
Se k = 0, ou seja, k + 1 = 1, já vimos no Teorema 7.1 do Capı́tulo 6 que
f (0) (x) := f (x) = 0 ⇒ f (x) = (x − x) · g(x),
onde o grau de g é n − 1.
Tentemos provar para k = m ≤ n − 1, supondo válido o resultado para todo
k ≤ m − 1.
Nossa hipótese será que
f (0) (x) = f (1) (x) = . . . = f (m) (x) = 0.
4. RAÍZES MÚLTIPLAS E FATORAÇÃO DE POLINÔMIOS 172

Em particular:
f (0) (x) = f (1) (x) = . . . = f (m−1) (x) = 0
e a hipótese de indução dá:
f (x) = (x − x)m · g(x)
para um polinômio g(x) de grau n − m. Precisamos ver que
g(x) = (x − x) · g(x)
para termos o resultado desejado:
f (x) = (x − x)m · [(x − x) · g(x)] = (x − x)m+1 · g(x).
Pensemos por absurdo, que
g(x) 6= (x − x) · g(x)
para todo g(x) de grau n − m − 1.
Pelo Teorema 7.1 do Capı́tulo 6 aplicado ao g(x):
g(x) 6= 0.
Mas como
f (x) = (x − x)m · g(x) = (x − x)k · g(x)
então a derivada f (m) (x) = f (k) (x) é uma soma onde cada parcela tem algum fator
dentre
(x − x)k , . . . , (x − x)2 , (x − x)
exceto uma última parcela que é do tipo C · g(x), C ∈ R \ {0}.
As parcelas todas que formam f (m) (x) = f (k) (x) se anulam x, exceto a parcela
que contém o fator C · g(x). Logo f (m) (x) 6= 0: contradição.
Portanto, como querı́amos:
g(x) = (x − x) · g(x).


Para entender o que acontece num entorno de uma raı́z múltipla x de um polinômio
y = p(x) temos:
Afirmação 4.1. Se x é uma raı́z de ordem exatamente 2n, n ∈ N, então (x, 0) é
ponto de máximo ou de mı́nimo local de y = p(x).
Se x é uma raı́z de ordem exatamente 2n + 1, n ∈ N, então (x, 0) é ponto de
inflexão de y = p(x).
Demonstração.
A suposição de que x é uma raı́z de ordem exatamente 2n, n ∈ N significa que:
f (x) = (x − x)2n · g(x),
onde g(x) é um polinômio a coeficientes Reais tal que
g(x) 6= 0.
Então, como vimos na Afirmação anterior,
p(x) = p′ (x) = p′′ (x) = . . . = p(2n−1) (x) = 0
CAPÍTULO 13. DERIVADA DO PRODUTO, INDUÇÃO E A DERIVADA DE
X N , N ∈ Z. 173

mas se fizermos a derivada de ordem 2n temos algo do tipo:


p(2n) (x) = (2n)! · g(x) + (x − x) · h(x)
e portanto
p(2n) (x) 6= 0.
A Afirmação 8.1 do Capı́tulo 11 diz que há máximo ou mı́nimo local.
Já a suposição de que x é uma raı́z de ordem exatamente 2n + 1, n ∈ N significa
que:
f (x) = (x − x)2n+1 · g(x),
onde g(x) é um polinômio a coeficientes Reais tal que
g(x) 6= 0.
Então
p(x) = p′ (x) = p′′ (x) = . . . = p(2n) (x) = 0
mas se fizermos a derivada de ordem 2n + 1 temos algo do tipo:
p(2n+1) (x) = (2n + 1)! · g(x) + (x − x) · h(x)
e portanto
p(2n+1) (x) 6= 0.
A Afirmação 8.1 do Capı́tulo 11 diz que há uma inflexão.


5. A Regra de Sinais de Descartes para as raı́zes de um polinômio


Neste Capı́tulo, que trata da indução matemática poderemos provar uma regra
clássica, que possivelmente remonta a Harriot (1631) e que teria chegado a Descartes
via a obra de Cardano.
Trata-se de uma estimativa dos número de raı́zes Reais de um polinômio. Inicial-
mente se estima as raı́zes positivas, mas facilmente se adapta para as negativas.
Precisaremos da indução matemática sobre o grau n do polinômio. O procedi-
mento para recair em grau n − 1 será derivar o polinômio dado.
Começemos introduzindo algumas convenções e notações.
Quando x é uma raı́z de p(x) de ordem exatamente n diremos que, contada com
multiplicidade, ela vale por n raı́zes. O número de raı́zes positivas de um polinômio
p(x) contadas com multiplicidade será denotado a seguir ZP(p).
Ordenados pelo grau crescente de cada monômio, considere o número de vezes
que muda o sinal dos coeficientes sucessivos de um polinômio p(x). Esse número será
denotado por MS(p). Por exemplo,
MS(−1 + 3x − 3x2 + x3 ) = 3 e ZP(p) = 3, 0<x=1
MS(−1 − 3x − 3x2 + x3 ) = 1 e ZP(p) = 1, 0 < x = 22/3 + 21/3 + 1
MS(1 + x2 ) = 0 e ZP(p) = 0,
MS(−1 + x) = 1 e ZP(p) = 1, 0 < x = 1.
5. A REGRA DE SINAIS DE DESCARTES PARA AS RAÍZES DE UM
POLINÔMIO 174

Em seu livro Geometria, Descartes dá como exemplo:


p(x) = −120 + 106 · x − 19 · x2 − 4 · x3 + x4
para o qual
MS = 3 e ZP(p) = 3, 0 < x = 2, 3, 4.
Posso dar mais dois exemplos:
p(x) = 2 − 3 · x + 3 · x2 − 3 · x3 + x4
tem
MS = 4 e ZP(p) = 2, 0 < x = 1, 2;
p(x) = 8 − 12 · x + 14 · x2 − 15 · x3 + 7 · x4 − 3 · x5 + x6
tem
MS = 6 e ZP(p) = 2, 0 < x = 1, 2.
Afirmação 5.1. (parte da Regra de sinais de Descartes)
Seja p(x) = a0 + ak1 · xk1 + ak2 · xk2 + . . . + an · xn , polinômio a coeficientes Reais
de grau n ≥ 1 com
a0 · aki 6= 0 e 1 ≤ k1 ≤ k2 ≤ . . . ≤ n.
Então:

i) Se a0 · an > 0 então ZP(p) é um número par1. Se a0 · an < 0 então ZP(p) é


um número ı́mpar.

ii) ZP(p) = MS(p) ou ZP(p) = MS(p) − 2 · j para algum j ∈ N.

Claro que o número de raı́zes negativas de p(x) pode também ser estimado,
considerando-se a mesma Afirmação 5.1, mas aplicada agora para o novo polinômio:
q(x) := p(−x).

Demonstração. (da Afirmação2 5.1)

Prova do item i):

Caso a0 · an > 0:

Após possı́vel multiplicação por −1, posso supôr que


a0 > 0 e an > 0.
Ou bem o gráfico de y(x) não intersecta o eixo dos x > 0 - e nesse caso ZP(p) = 0
- ou bem o faz de dois modos possı́veis:
1Adoto a convenção de considerar 0 como número par.
2A prova que dou desta Afirmação expõe o que se aprende no artigo de Xiaoshen Wang, A
simple proof of Descartes’s rule of signs, The American Mathematical Monthly, Vol. 111, No. 6, p.
525-526. 2004
CAPÍTULO 13. DERIVADA DO PRODUTO, INDUÇÃO E A DERIVADA DE
X N , N ∈ Z. 175

• i): tangenciando o eixo. Formando portanto máximos ou mı́nimos locais de


y = p(x): nesse caso a raı́z tem multiplicidade par (compare com a Afirmação
4.1). A contribução a ZP(p) dessas tangências é par.
• ii): atravessando o eixo x > 0. O que pode ser feito transversalmente ou
formando inflexões. Neste caso cada raı́z tem multiplicidade ı́mpar (compare
com a Afirmação 4.1). Mas como
p(0) = a0 > 0 e lim p(x) = +∞,
x→+∞

pois an > 0, concuimos que cada vez que o eixo x > 0 é atravessado pelo
gráfico no ponto x1 no sentido do semi-plano y > 0 ao semiplano y < 0
deverá haver uma outra raı́z x2 em que o gráfico atravessa o eixo x > 0 no
sentido do semi-plano y < 0 ao semiplano y > 0. Então as raı́zes x1 e x2
contribuem juntas para ZP(p) com um número par, soma de dois ı́mpares.
Logo ZP(p) é par (incluindo o 0).

Caso a0 · an < 0:

Após possı́vel multiplicação por −1, posso supôr que


a0 > 0 e an < 0.
Como
p(0) = a0 > 0 e lim p(x) = −∞,
x→+∞

pois an < 0, o T.V.I. nos garante que há alguma raı́z e portanto ZP(p) ≥ 1. O
mesmo tipo de argumento do Caso anterior agora dá que ZP(p) é ı́mpar.

Prova do item ii):


Será feita por indução no grau n.
Para n = 1 temos p(x) = a0 + a1 · x.
A condição MS(p) = 0 equivale a a0 · a1 > 0. E nesta situação a raı́z
a0
x=− <0
a1
dá que ZP(p) = 0.
A condição MS(p) = 1 equivale a a0 · a1 < 0. E nesta situação a raı́z
a0
x=− >0
a1
dá que ZP(p) = 1.
Portanto ZP(p) = MS(p) e o item ii) vale para n = 1.
Suponhamos como hipótese de indução que a afirmação do item ii)
ZP(p) = MS(p) ou ZP(p) = MS(p) − 2 · j, j∈N
valha para quaisquer polinômios de grau ≤ n − 1.
Será útil re-enunciar esta hipótese da seguinte maneira equivalente:
5. A REGRA DE SINAIS DE DESCARTES PARA AS RAÍZES DE UM
POLINÔMIO 176

Hipótese: para quaisquer polinômios de grau ≤ n − 1 vale ZP(p) ≤ MS(p) e, ou


bem ZP(p) e MS(p) são pares ou bem ZP(p) e MS(p) são ı́mpares.
Seja agora o polinômio a coeficientes Reais de grau n ≥ 2:
p(x) = a0 + ak1 · xk1 + ak2 · xk2 + . . . + an · xn ,

a0 · aki 6= 0 e 1 ≤ k1 ≤ k2 ≤ . . . ≤ n.
Se divide o resto da prova em dois casos:

Caso 1) a0 · ak1 > 0:


Considero a derivada de p(x)

p′ (x) = (k1 · ak1 · xk1 −1 + k2 · ak2 · xk2 −1 + . . . + n · an · xn ,


Note que a0 · ak1 > 0 garante que
MS(p) = MS(p′ ).
Ademais, como a0 e ak1 têm o mesmo sinal e como o sinal do coeficiente do termo
de ordem mais alta de p e de p′ é o mesmo, a aplicação do Item i) já provado a p(x)
e depois a p′ (x) dirá que ou bem ZP(p) e ZP(p′ ) são números pares ou bem ZP(p)
e ZP(p′ ) são números ı́mpares.
Aplico a hipótese de indução a p′ (x), cujo grau é n − 1: ZP(p′ ) ≤ MS(p′ ) e, ou
bem ZP(p′ ) e MS(p′ ) são pares ou bem ZP(p′ ) e MS(p′ ) são ı́mpares.
Concluo por enquanto que ou bem ZP(p) e MS(p) são pares ou bem ZP(p) e
MS(p) são ı́mpares. Isso já prova parte do Item ii).
Agora, pelo Teorema de Rolle:
ZP(p′ ) ≥ ZP(p) − 1
pois não podem haver duas raı́zes sucessivas de p(x) sem que entre elas haja uma raı́z
de p′ (x).
Então:
MS(p) = MS(p′ ) ≥ ZP(p′ ) ≥ ZP(p) − 1,
ou seja,
MS(p) + 1 ≥ ZP(p).
Como sabemos que ou bem ZP(p) e MS(p) são pares ou bem ZP(p) e MS(p) são
ı́mpares isso força que:
MS(p) ≥ ZP(p),
como querı́amos para completar o Item ii).

Caso 2) a0 · a1 < 0: a prova é bem parecida.



CAPÍTULO 13. DERIVADA DO PRODUTO, INDUÇÃO E A DERIVADA DE
X N , N ∈ Z. 177

6. Exercı́cios
Exercı́cio 6.1. (resolvido)
Prove por indução: n! ≥ 2n−1 , ∀ n ≥ 2.
Exercı́cio 6.2. Derive o produto de três funções (deriváveis):
( f (x) · g(x) · h(x) )′
Exercı́cio 6.3. Produza 4 exemplos de polinômios p de grau 6 em que, no item ii)
da Afirmação 5:
ZP(p) = MS(p) − 2 · j,
o número j ∈ N vale j = 0, 1, 2, 3.
CAPı́TULO 14

Derivada da composição de funções

A composição de funções simples produzindo funções complicadas é o análogo


matemático da composição de processos simples que produzem efeitos complicados
na natureza, nas reações quı́micas, nos processos biológicos, etc.
Daı́ a importância de sabermos derivar composições.

1. Regra da composta ou da cadeia


A palavra que costuma se usar regra cadeia poderia ser substituı́da pelo sinônimo
regra da corrente, pois uma corrente é algo feito de elos simples.
A regra de derivação da função composta combina as derivadas de cada constitu-
inte da corrente de um modo bem determinado, como veremos.
Antes de enunciá-la em geral, considero algumas composições especı́ficas, que nos
ajudarão a entender a regra geral.
Considere as funções fn (x) := n·x, com n ∈ N fixado, g(x) = sin(x) e as compostas
(g ◦ fn )(x) = sin( n · x ). Suponha que fazemos a restrição g : [0, 2π] → R. Então
quando x percorre [0, 2π] o parâmetro z := n · x percorre n vezes esse intervalo. Ou
seja que o gráfico da a função sin( n · x ) é formado por n cópias do gráfico do seno,
claro que mais comprimidas. Abaixo pot o seno e sin(3x):

0,5
0
0 1 2 3 4 5 6
-0,5 x
-1

Figura: Gráfico de y = sin(x) (vermelho) e de y = sin(3x)


(verde) para x ∈ [0, 2pi].
Como vimos no Capı́tulo 12, o cosseno é a derivada do seno: onde o cosseno é
positivo (negativo) o seno é crescente (decrescente), onde o cosseno se anula o seno
tem seus máximos ou mı́nimos, etc. Ora, a função cos(nx) satisfaz qualitativamente
todas essas exigências, ou seja, se comporta qualitativamente como se fosse a derivada
de sin(nx). Ou seja, como fizemos na Parte 1 deste curso, onde os gráficos de f ′ e f
eram corretos apenas qualitativamente.
179
1. REGRA DA COMPOSTA OU DA CADEIA 180

Veja isso na próxima Figura, com n = 3:

0,5

0
0 0,5 1 1,5 2
x

-0,5

-1

Figura: Gráfico de y = sin(3x) (vermelho) e de y = cos(3x)


(verde) para x ∈ [0, 2π].

Mas o que esta Figura não tem de quantitativamente correto é o fato de que para
que sin(3x) faça 3 vezes o que o seno usual faz quando x percorre [0, 2π], sin(3x) tem
que ser mais rápido que o seno usual. Ou seja, em cada ponto as inclinações das
tangentes de sin(3x) são maiores que as do seno usual. Quanto maiores? Exatamente
3 vezes maiores.
Por isso a derivada de sin(3x) quantitativamente correta não é cos(3x) mas sim:

sin(3x)′ = 3 cos(3x)

e mais em geral:
sin(nx)′ = n cos(nx)

Mostro isso na Figura a seguir:

0
0 0,5 1 1,5 2
x
-1

-2

-3

Figura: Gráfico de y = sin(3x) (vermelho) e de sua


derivada (verde) para x ∈ [0, 2π].

Agora consider uma outra composição: f (x) = x2 e g(x) = sin(x), ou seja (g ◦


f )(x) = sin(x2 ). A diferença para o exemplo anterior, sin(3x) é que à medida que x
se aproxima de 2π x2 cresce cada vez mais rápido e a função sin(x2 ) faz aquilo que o
seno faz em cada vez menores intervalos, como mostra a figura a seguir:
CAPÍTULO 14. DERIVADA DA COMPOSIÇÃO DE FUNÇÕES 181

0,5
0
0 1 2 3 4 5 6
-0,5 x
-1

Figura: Gráfico de y = sin(x) (vermelho) e


de y = sin(x2 ) (verde) para x ∈ [0, 2π].

Qualitativamente falando, cos(x2 ) se comporta como esperamos da derivada de


sin(x2 ):

0,5
0
0 1 2 3 4 5 6
-0,5 x
-1

Figura: Gráfico de y = sin(x2 ) (vermelho) e


de y = cos(x2 ) (verde) para x ∈ [0, 2π].

De novo, o que está quantitativamente errado: as inclinações do gráfico de y =


sin(x2 ) estão ficando cada vez maiores quando x se aproxima de 2π. De quanto pre-
cisamos multiplicar a função qualitativamente correta da derivada para termos uma
função quntitativamente exata da derivada ? A resposta como vermos é: precisamos
multiplicar pela função 2x ! Ou seja, para cada x > 0 a correção muda neste exemplo:
A Figura a seguir superpõe os gráficos y = sin(x2 ) e de sua derivada, que veremos
é cos(x2 ) · 2x, e, ademais dá os gráficos de y = 2x e y = −2x. Essas retas passam
pelos pontos de máximo e mı́nimo locais da derivada.
1. REGRA DA COMPOSTA OU DA CADEIA 182

10

0
0123456
x

-5

-10

Figura: y = sin(x2 ) (vermelho), sua derivada (verde), y = 2x e


y = −2x, para x ∈ [0, 2π].

Por último, volto num limite calculado como Exercı́cio 5.4 do Capı́tulo 8:
sin(k · x)
lim = k.
x→0 x
Podemos olhá-lo do seguinte modo:
sin(k · x) − sin(k · 0)
lim =k
x→0 x
e reconhecemos então a definição da derivada da composta sin(k · x) em x = 0.
O Teorema a seguir generaliza essas observações:
Teorema 1.1. Sejam f : I → J e g : K → L funções definidas em intervalos, com
a imagem J de f contida no domı́nio K de g, J ⊂ K. Se f e g são seriváveis então
a função composta (g ◦ f ) : I → L, definida por (g ◦ f )(x) := g(f (x)) também é
derivável e ademais:
(g ◦ f )′ (x) = g ′ (f (x)) · f ′ (x).

A notação de Leibniz:
dy
A notação de G. Leibniz para a derivada de y = f (x) é dx . O valor de sua notação
fica claro quando escrevemos a regra da derivada da composta. Para y = f (x),
u = g(y) e u = g(f (x)):
du du dy
= · .
dx dy dx
O leitor verá, por exemplo no Capı́tulo 37, como é útil e confortável a notação de
Leibniz.

A prova da Afirmação 1.1 é técnica, prefiro tirar consequências.

A primeira consequência é que se pode derivar um número qualquer de com-


posições. Por exemplo, para tres funções podemos afirmar:
CAPÍTULO 14. DERIVADA DA COMPOSIÇÃO DE FUNÇÕES 183

Afirmação 1.1. Sejam f : I → J, g : K → L e h : M → N, com J ⊂ K e L ⊂ M.


Se f, g, h são deriváveis, então a função composta (h ◦ g ◦ f ) : I → L, definida por
(h ◦ g ◦ f )(x) := h(g(f (x))) é derivável e ademais:
(h ◦ g ◦ f )′ (x) = h′ (g(f (x))) · g ′ (f (x)) · f ′ (x).
Demonstração. De fato, associo h ◦ g ◦ f = h ◦ (g ◦ f ) e uso o Teorema 1.1 duas
vezes:
(h ◦ (g ◦ f ))′ (x) = h′ (g(f (x))) · (g ◦ f )′ (x) =
= h′ (g(f (x))) · g ′ (f (x)) · f ′ (x).


No Capı́tulo 16 sobre funções inversas vamos dar aplicações importantes da derivada


da composta.
Vejamos agora alguns exemplos simples:
• f = sin(x), g = x2 , então (g ◦ f )′ = 2 · (sin(x)) · cos(x)
• f = cos(x), g = x2 , (g ◦ f )′ = 2 · (cos(x)) · (− sin(x)) = −2 · cos(x) · sin(x).
• como consequência desse dois itens e da derivada da soma:
(sin(x)2 + cos(x)2 )′ = 2 · sin(x) · cos(x) − 2 · cos(x) · sin(x) ≡ 0,
o que é natural já que sin(x)2 + cos(x)2 ≡ 1.
• f (x) = x2 e g(x) = sin(x), então (g ◦ f )′ (x) = cos(x2 ) · 2 · x.

2. A derivada do quociente
Agora uma aplicação da regra da composta aos quocientes de funções:
Afirmação 2.1. Sejam f e g funções deriváveis com g nunca nula. Então
f (x) ′ f ′ (x) · g(x) − f (x) · g ′ (x)
( ) (x) = .
g(x) g 2(x)
Em particular:
1 g ′(x)
( )′ (x) = − 2 .
g g (x)
Demonstração.
Vou escrever primeiro
f (x) 1
= f (x) ·
g(x) g(x)
e derivar esse produto:
f (x) ′ 1 1 ′
( ) (x) = f ′ (x) · + f (x) · ( ) (x),
g(x) g(x) g(x)
1 1
Agora olho g(x)
como a composição de duas funções f1 (x) = g(x) e f2 (x) = x
= x−1 :
1
= (f2 ◦ f1 )(x).
g(x)
2. A DERIVADA DO QUOCIENTE 184
1
Já sabemos derivar f2 (x) = x
= x−1 , de fato: f2′ (x) = − x12 = −x−2 . Então a regra
da composta dá:
1 ′
( ) (x) = (f2 ◦ f1 )′ (x) =
g(x)

= f2′ (f1 (x)) · f1′ (x) =

1
=− · g ′(x).
g 2 (x)
Junto tudo:
f (x) ′ 1 1 ′
( ) (x) = f ′ (x) · + f (x) · ( ) (x) =
g(x) g(x) g(x)

1 1
= f ′ (x) · + f (x) · (− 2 · g ′ (x)) =
g(x) g (x)

f ′ (x) · g(x) − f (x) · g ′ (x)


= ,
g 2(x)
como querı́amos. 

Exemplos:
• Funções racionais são quocientes de polinômios fg . Onde g não se anula, a
fórmula da Afirmação 2.1 nos diz como derivá-las.
sin(x)
• A tangente é um quociente de funções deriváveis tan(x) = cos(x) . Onde o
cosseno não se anula podemos derivá-la obtendo:
cos(x) · cos(x) − sin(x) · (− sin(x))
tan′ (x) = =
cos2 (x)

1
=
cos2 (x)
1
e com a nomenclatura conhecida sec(x) := cos(x)
o que temos é

tan′ (x) = sec2 (x).


1
Então claramente tan′ (0) = cos2 (0)
=1e

lim tan′ (x) = lim tan′ (x) = +∞.


xր π2 −π
xւ 2

A seguir plotei os gráficos da tangente e de sua derivada restritas ao


intervalo (−1, 1). Não pude usar um intervalo mais parecido com o domı́nio
(− π2 , π2 ) porque os valores da tangente ficam muito grande em módulo.
CAPÍTULO 14. DERIVADA DA COMPOSIÇÃO DE FUNÇÕES 185

0
-1 -0,5 0 0,5 1
x

-1

Figura: A função tangente (vermelho) e sua derivada (verde) restritas a (−1, 1).

3. Uma função que tende a zero oscilando


sin(x2 )
Afirmação 3.1. A função f : [1, +∞) → R dada por f (x) = x
tem limx→+∞ f (x) =
0 mas não existe limx→+∞ f ′ (x).
Demonstração.
sin(x2 )
Como | sin(x2 )| ≤ 1 e limx→+∞ x1 = 0 então limx→+∞ x
= 0.
Para x > 0, a derivada do quociente dá:
cos(x2 ) · 2x − sin(x2 ) · 1 sin(x2 )
f ′ (x) = = 2 cos(x2
) −
x2 x2
e portanto quando x é muito grande f ′ (x) ≈ 2 cos(x2 ), ou seja, f ′ (x) percorre muitos
valores no intervalo [−1, 1], portanto f ′ (x) não tende a nenhum valor especı́fico.


A Figura a seguir ilustra em vermelho a f e em verde f ′ , com x ∈ [1, 10]:

x
2 4 6 8 10
0

-1

-2
4. CONFECÇÃO DE GRÁFICOS DE FUNÇÕES RACIONAIS 186

sin(x2 )
Já o comportamento de f (x) = x
quando x → 0 será tema do Exercı́cio 16.10
no Capı́tulo 22.

4. Confecção de gráficos de funções racionais

Exemplo: Considere y = f (x) = 21 − x24+4 .


Talvez a primeira coisa a se observar é que f (x) é uma função par, f (x) = f (−x),
pois essa simetria em relação ao eixo dos y ajuda muito para confeccionar o gráfico.
x2 −4
Como f (x) = 2(x 2 +4) , essa função se anula quando x = ±2 e é positiva exatamente

quando |x| > 2.


Ademais, uma bonita simplificação dá f ′ (x) = (x28x +4)2
. Ou seja que, x = 0 é ponto

crı́tico e, ademais, é mı́nimo local pois nele a f (x) passa de negativa para positiva.
Também é fácil ver que:
1
lim f (x) = lim f (x) = ,
x→+∞ x→−∞ 2
embora sempre f (x) < 21 ; ou seja, y = 21 é assı́ntota horizontal.
Para ver se há inflexões faço uma conta um pouco maior e obtenho:
8(3x2 − 4)
f ′′ (x) = −
(x2 + 4)3

que se anula em x = ± 23 3. Ou seja, a concavidade de y = f (x) é para baixo
√ √ √
em (−∞, − 23 3), muda para cima em (− 23 3, 23 3) e volta a ser para baixo em

( 32 3, +∞).
A figura a seguir ilustra tudo isso (apenas qualitativamente, já que as escalas nos
eixos são diferentes):

0,4

0,2

x
-10 -5 0 5 10
0

-0,2

-0,4

Exemplo:
CAPÍTULO 14. DERIVADA DA COMPOSIÇÃO DE FUNÇÕES 187

Agora vamos fazer o gráfico da função racional


x3 + 8x
f : R \ {−1, 1} → R, f (x) =
.
x2 − 1
Novamente queremos estar corretos apenas qualitativamente.
Como o numerador de f (x) é x· (x2 + 8), temos que f (x) = 0 exatamente se x = 0.
O numerador de f é negativo se x < 0 e positivo se x > 0. Já o denominador de f (x)
é negativo se −1 < x < 1 e positivo no resto do domı́nio.
Ou seja,
• f (x) = 0 exatamente se x = 0;
• f (x) > 0 se −1 < x < 0 ou x > 1.
• f (x) < 0 se x < −1 ou se 0 < x < 1.
Não é difı́cil ver que:
lim f (x) = −∞ lim f (x) = +∞,
xր−1 xց−1

lim f (x) = −∞ lim f (x) = +∞.


xր1 xց1

Agora examino (derivando pela regra do quociente):


x4 − 11x2 − 8
f ′ (x) = .
(x2 − 1)2
O numerador é do tipo z 2 − 11z − 8, com z = x2 .
Então f ′ (z) = 0 exatamente se
p √ √
11 ± (11)2 + 4 · 8 11 ± 153 11 ± 3 · 17
z= = = .
2 2 2

Mas 11−3·2 17
< 0, portanto, se queremos determinar x ∈ R onde f ′ (x) = 0, devemos
tomar: s

11 + 3 · 17
x=± .
2
√ q √ √
Podemos aproximar grosseiramente 17 ≈ 4 e 11+3·2 17 ≈ 15 ≈ 3.
Ou seja que a derivada f ′ (x) se anula num ponto x1 ≈ 3 e noutro x2 ≈ −3.
Antes de examinar f ′′ (x), note que não é difı́cil se convencer de que:
lim f (x) = +∞,
x→+∞

Como limxց1 f (x) = +∞ isso indica que x1 ≈ 3 é ponto de mı́nimo local da f (sem
usar qualquer teste).
Por outro lado como
lim f (x) = −∞
x→−∞

e limxր−1 f (x) = −∞, isso indica que x2 ≈ −3 é máximo local da f (sem usar
qualquer teste).
4. CONFECÇÃO DE GRÁFICOS DE FUNÇÕES RACIONAIS 188

Agora, com a regra da derivada do quociente, da composta e após simplificações,


obtemos:

18x(x2 + 3)
f ′′ (x) = .
(x2 − 1)3

Claramente f ′′ (x) se anula apenas em x = 0 e nesse ponto muda de sinal. Logo


x = 0 é um ponto de inflexão.
Para −1 < x < 0 ou para x > 1 temos f ′′ (x) > 0 e concavidade para cima.
Mas para x < −1 ou 0 < x < 1 temos concavidade para baixo.
Em particular, f ′′ (x1 ) > 0 e f ′′ (x2 ) < 0 o que comprova que são mı́nimo e máximo
locais respectivamente.
As três Figuras a seguir resumem essas observações: a primeira pega parte da
região x < −1, a segunda, parte da região −1 < x < 1 e a terceira, parte da região
x > 1.

x
-5 -4,5 -4 -3,5 -3 -2,5 -2 -1,5

-7

-8

-9

-10

-11

-12

x3 +8x
Figura: O gráfico de y = x2 −1
, x ∈ [−5, −1.5].

15

10

0
-0,8 -0,4 0 0,4 0,8
-5x

-10

-15
CAPÍTULO 14. DERIVADA DA COMPOSIÇÃO DE FUNÇÕES 189

x3 +8x
Figura: O gráfico de y = x2 −1
, x ∈ [−0.8, 0.8].

12

11

10

2 3 4 5 6 7
x

x3 +8x
Figura: O gráfico de y = x2 −1
, x ∈ [1.5, 5].

5. Involuções fracionais lineares


Vimos nos Exercı́cios do Capı́tulo 7 que f (x) = x1 tem f = f −1 , ou seja, é uma
involução.
Agora que sabemos derivar as funções racionais, vamos poder mostrar que há
involuções que são quocientes de funções lineares:
Afirmação 5.1. As funções racionais f : R \ { αγ } → R dadas por
α·x+β
f (x) = , com α2 + β · γ 6= 0
γ·x−α
(onde α, β, γ ∈ R) são inversı́veis, são involuções e portanto têm gráficos simétricos
relativos à diagonal.
Ademais, funções racionais do tipo
α·x+β
f (x) = , com α · δ − β · γ 6= 0
γ·x+δ
(onde α, β, γ, δ ∈ R) são inversı́veis e são involuções somente se δ = −α.
Demonstração.
Note que as funções
α·x+β
f (x) =
γ ·x−α
não estão definidas em αγ . De fato só estariam definidas aı́ se αx + β se anulasse
também em αγ . Mas então −β α
= αγ , ou seja, α2 + β · γ = 0 contrariando a hipótese.
Agora calculo a derivada, pela regra do quociente e obtenho após simplificação:
α2 + β · γ
f ′ (x) = − < 0,
(γ · x − α)2
portanto f (x) é estritamente decrescente, logo invertı́vel.
6. UM PROBLEMA DA PUTNAM COMPETITION, N. 1, 1938 190

Sua inversa é obtida:


α·x+β
y= ⇔y·γ ·x−y·α = α·x+β ⇔
γ·x−α
α·y+β
⇔y·γ·x−α·x= y·α+β ⇔x= ,
γ·y−α
ou seja, x = x(y) tem exatamente a mesma expressão de y = y(x).
Por isso são involuções e por isso são simétricas em relação à diagonal.
Ademais, se
α·x+β
f (x) =
γ·x+δ
então
α·δ−β ·γ
f ′ (x) = 6= 0.
(γ · x + β)2
Se obtém, como antes, de y = y(x):
−δ · y + β
x = x(y) = .
γ·y−α
Portanto se queremos um involução precisamos que δ = −α.


A Figura a seguir dá três exemplos:

1 2 3 4 5
x

1
Figura: Em vermelho a diagonal, em verde y = x
0.1·x+2
amarelo y = 3·x−0.1 e em azul y = 0.1·x+4
9·x−0.1
.

6. Um problema da Putnam Competition, n. 1, 1938


1
Dada a parábola y = 2m
· x2 , determine a menor corda ortogonal ao gráfico em
um dos extremos.

Solução:
Minha solução não é das mais elegantes, pois é na força bruta. Farei o seguinte:
CAPÍTULO 14. DERIVADA DA COMPOSIÇÃO DE FUNÇÕES 191

x2 x2
• determinarei os pontos que são os extremos (x0 , 2m0 ) e (x1 , 2m1 ) de uma corda
x2
ortogonal ao gráfico em (x0 , 2m0 ),
• pensarei no quadrado do comprimento1 da corda:
x21 x2
(x1 − x0 )2 + ( − 0 )2
2m 2m
como uma função f (x0 ) de x0 .
• procurarei f ′ (x0 ) = 0 e depois verei se f ′′ (x0 ) > 0.
x2
A reta que passa por (x0 , 2m0 ) e é ortogonal ao gráfico da parábola dada tem
equação:
−m 2m2 + x20
y= ·x+ .
x0 2m
(posso supor x0 6= 0 pois a reta ortogonal ao gráfico pela origem é vertical e não
intersecta o gráfico da parábola em nenhum outro ponto).
Essa reta intersecta de novo a parábola em
2 · m2
x1 = −x0 − ,
x0
como se descobre resolvendo uma equação quadrática.
A expressão do quadrado da distância entre esses dois pontos admite um boa
simplificação:
x2 x2
φ(x0 ) := (x1 − x0 )2 + ( 1 − 0 )2 =
2m 2m
2
2m2 2 (x0 + 2m x0
)2 x2
= (2x0 + ) +( − 0 )2 =
x0 2m 2m
2 2 3
4(x0 + m )
= .
x40
Agora derivo φ(x0 ) como função de x0 , obtendo:
−8 · (x20 + m2 )2 · (−x20 + 2m2 )
φ′ (x0 ) = .
x50
Portanto φ′ (x0 ) = 0 para dois valores:

x = ± 2 · m.
Para ver que esses pontos são mı́nimos locais de φ(x0 ) (e portanto globais, por falta
de outros candidatos)
√ podemos analisar o sinal de φ′ (x0 ) à esquerda e à direita deles.
Para x = 2 · m: note que para x0 < x e próximo dele, temos
−x20 + m2 > 0
e portanto φ′ (x0 ) < 0; para x0√> x e próximo dele, temos φ′ (x0 ) > 0.
Analogamente para x = − 2m.
1 A Afirmação 2.1 do Capı́tulo 16 justificará essa troca do comprimento pelo quadrado do
comprimento. O que ganhamos nessa troca é não precisar derivar a raı́z quadrada
7. UMA FUNÇÃO COM DERIVADA, MAS SEM A SEGUNDA DERIVADA 192

7. Uma função com derivada, mas sem a segunda derivada


Agora que já sabemos derivar quocientes, podemos considerar novamente a função
x
f : R → ( −1, 1 ), f (x) = ,
|x| + 1
estudada na Seção 4 do Capı́tulo 5.
x
Afirmação 7.1. Seja f : R → ( −1, 1 ) dada por f (x) = |x|+1
.
1 1
• f ′ (x) = (x+1) ′ ′
2 se x > 0; f (x) = (−x+1)2 se x < 0 e f (0) = 1.
−2 −2
• f ′′ (x) = (x+1) ′′ ′′
3 se x > 0; f (x) = (−x+1)3 se x < 0; mas não existe f (0).

Demonstração.
No Exercı́cio 6.4 do Capı́tulo 9 já vimos que f ′ (0) = 1.
Se x > 0 podemos usar a regra da derivada do quociente:
x ′ x · (x + 1)′ − x′ · (x + 1) 1
f (x)′ = [ ] = 2
=
x+1 (x + 1) (x + 1)2
e analogamente, se x < 0:
x 1
f (x)′ = [ ]′ = .
−x + 1 (−x + 1)2
Agora sobre f ′′ (x). Se existisse
f ′ (h) − f ′ (0)
f ′′ (0) := lim .
h→0 h
teriam que exister ambos lmites laterais
f ′ (h) − f ′ (0) f ′ (h) − f ′ (0)
lim e lim
hց0 h hր0 h
e ademais serem iguais !
Porém, já que f ′ (0) = 1:
1
f ′ (h) − f ′ (0) (h+1)2
−1
lim = lim =
hց0 h hց0 h

= lim (−h − 2) = −2,


hց0

enquanto que
1
f ′ (h) − f ′ (0) (−h+1)2
−1
lim = lim =
hր0 h hր0 h
= lim (2 − h) = 2.
hր0


CAPÍTULO 14. DERIVADA DA COMPOSIÇÃO DE FUNÇÕES 193

Os gráficos de f ′ e de f ′′ são mostrados a seguir:

x
-3 -2 -1 0 1 2 3
0

-1

-2

Figura: Note que f ′ (x) (vermelho) tem um bico em (0, 1).


Em verde está f ′′ (x). Note que f ′′ (0) não está definido.

8. Máximos e mı́nimos: o problema do freteiro


Agora que já sabemos derivar um conjunto grande de funções, podemos nos colocar
problemas de máximos e mı́nimos mais interessantes.
Imagine que você está transportando, numa mudança, um objeto retangular de
largura L dada. Durante o transporte ele não poderá ser deformado, nem vergado.
Você vem com ele por um corredor que mede l1 de largura e que dobra em ângulo
reto, chegando numa sala de largura l2 = k · l1 ≥ l1 , como mostra a Figura a seguir:

Pensando o problema como um problema no plano, não espacial, trata-se de de-


terminar o comprimento máximo do objeto retangular para que você consiga passá-lo
para a sala.
8.1. Caso L ≈ 0. Vamos primeiro considerar o caso em que a largura L do
objeto retangular é muito pequena (por exemplo, uma vara de alumı́nio de diâmetro
muito pequeno mas bem comprida). Vamos pensar então que L = 0 e o objeto é
uni-dimensional.
8. MÁXIMOS E MÍNIMOS: O PROBLEMA DO FRETEIRO 194

Primeiro noto que, se consigo passar uma vara de um certo tamanho para a sala
sem ter tocado o ponto C da Figura, então certamente passaria uma vara um pouco
maior, apoiando-me e pivotando em C.
Por isso, de agora em diante, posso pensar que me apoiarei em C, pivotando nesse
ponto.
A chave da resolução do problema é a seguinte: é notar que a restrição, o im-
pedimento, para se passar a vara está no mı́nimo da distância do segmento P1 P2 , à
medida que muda θ ∈ [0, π2 ]. Veja a Figura que segue:
P 2

l 2
d 2

θ C

d 1

P 1
l 1

Portanto trata-se de descobrir qual o mı́nimo de P1 P2 . Para isso, penso em


P1 P2 = P1 C + CP2
e ademais noto (identificando ângulos opostos pelo vértice) que:
l1 l2
cos(θ) = e sin(θ) = .
P1 C CP2
Ou seja:
P1 P2 (θ) = P1 C(θ) + CP2 (θ) =
l1 l2
= + .
cos(θ) sin(θ)
Repare que é natural que quando θ ≈ π2 (antes de começar a esquina) tenhamos
CP2 (θ) ≈ l2 mas P1 C(θ) fique arbitrariamente grande, ou seja não há retrições sobre
ele. Porém se θ ≈ 0 (após vencer a esquina) aı́ P1 C(θ) ≈ l1 enquanto CP2 (θ) fica
arbitrariamente grande.
Agora:
′ l1 · sin(θ) −l2 · cos(θ)
P1 P2 (θ) = + =
cos2 (θ) sin2 (θ)
l1 · sin3 (θ) − l2 · cos3 (θ)
= ,
sin2 (θ) cos2 (θ)
e portanto
′ l2 1 1
P1 P2 (θ) = 0 ⇔ tan(θ) = ( ) 3 = k 3 .
l1
CAPÍTULO 14. DERIVADA DA COMPOSIÇÃO DE FUNÇÕES 195
1
Ou seja, a derivada se anula em um único ponto: θ0 = arctan(k 3 ).
Para concluir que θ0 é o ponto de mı́nimo, basta conferir que
l1 l2
lim + = +∞
θց0 cos(θ) sin(θ)
e
l1 l2
limπ + = +∞.
θր 2 cos(θ) sin(θ)
Assim o valor máximo do comprimento da vara que poderemos passar é
l1 l2
P1 P2 (θ0 ) = + .
cos(θ0 ) sin(θ0 )
Vejamos Exemplos:

A Figura a seguir mostra a função P1 P2 (θ), para l1 = 1.2 e l2 = 2.4, quando
1
θ0 = arctan(2 3 ) ≈ 0.8999083481 e o valor máximo de comprimento é 4.99432582244
(plotado como reta horizontal em verde)

5,06

5,04

5,02

0,8 0,84 0,88 0,92 0,96


x


Já a próxima figura dá a função P1 P2 (θ) no caso l1 = l2 = 1.2, em que θ0 =
arctan(1) = π4 ≈ e o valor máximo da vara é 3.394112550 (horizontal em verde).

3,56

3,52

3,48

3,44

3,4

0,65 0,7 0,75 0,8 0,85 0,9


x
8. MÁXIMOS E MÍNIMOS: O PROBLEMA DO FRETEIRO 196

8.2. Para um objeto retangular. Agora vamos para o caso em que a largura
não pode ser considerada zero, ou seja L > 0, quando o objeto é bi-dimensional.
A Figura a seguir dá a geometria da situação (note que paralelismo/ortogonalidade
de retas transportam o ângulo θ para dois triângulos retângulos):
P 2
θ
D2 − d2

d 2 l 2

d 1 C
P 1

D1− d1
θ
l 1

Note que
l1 l2
cos(θ) = e sin(θ) = ,
D1 D2
de onde:
l1 l2
D1 = (D1 − d1 ) + d1 = e D2 = (D2 − d2 ) + d2 = ,
cos(θ) sin(θ)
e portanto:
l1 L l2
L · tan(θ) + d1 = e + d2 = ,
cos(θ) tan(θ) sin(θ)
o que dá:
l1 l2 1
(d1 + d2 )(θ) = + − L · (tan(θ) + )=
cos(θ) sin(θ) tan(θ)
l1 l2 L
= + − .
cos(θ) sin(θ) sin(θ) · cos(θ)
Essa é a função que quero minimizar, pois seu mı́nimo é o impedimento, a obstrução
para que continue se movendo a face externa (relativa a C) do objeto retangular.
A sua derivada é:
l1 · sin3 (θ) − l2 · cos3 (θ) − L · (2 · cos2 (θ) − 1)
(d1 + d2 ) ′ (θ) = .
sin2 (θ) cos2 (θ)
Queremos saber onde (d1 + d2 ) ′ (θ) = 0, e no caso L > 0 devemos usar métodos
numéricos (aproximações). Os programas como Maple/ Xmaxima , etc a resolvem
numericamente.
Aparecem algumas soluções complexas e uma solução Real positiva.
Para concluir que θ0 é o ponto de mı́nimo, basta conferir que
lim (d1 + d2 )(θ) = +∞
θց0
CAPÍTULO 14. DERIVADA DA COMPOSIÇÃO DE FUNÇÕES 197

lim (d1 + d2 )(θ) = +∞.


θր π2

Como
l1
lim = l1
θ→0 cos(θ)

basta analisar
l2 L
lim − =
θ→0 sin(θ) sin(θ) · cos(θ)

1 L
= lim · (l2 − ).
θ→0 sin(θ) cos(θ)

Mas
L
lim =L
θ→0 cos(θ)

e como l2 ≥ l1 > L, então

1 L 1
lim · (l2 − ) = lim = +∞.
θ→0 sin(θ) cos(θ) θ→0 sin(θ)

Quando θ se aproxima de π2 pela direita então é o sin(θ) que se aproxima de 1 e o


cos(θ) se aproxima de 0. Analogamente com o caso anterior, se obtém:

1
limπ (d1 + d2 )(θ) = limπ = +∞.
θր 2 θր 2 cos(θ)

Também se pode avaliar (d1 + d2 )′′ (θ0 ) e o valor dá positivo.

Uma questão aparece naturalmente:

Questão 1: haverá outro modo de resolver o problema com L > 0 em que a solução
(θ0 ) seja dada por um expressão exata ?

Um Exemplo: a figura a seguir dá a função P1 P2 (θ), para um objeto de largura



L = 1, quando l1 = 1.2, l2 = 2.4. Nesse caso o ponto θ0 onde P1 P2 (θ0 ) = 0 é
θ0 ≈ 1.065134018 e o valor máximo de comprimento do objeto é 2.860890636 (plotado
como reta horizontal em verde).
8. MÁXIMOS E MÍNIMOS: O PROBLEMA DO FRETEIRO 198

2,94

2,92

2,9

2,88

2,86
0,9 0,95 1 1,05 1,1 1,15 1,2
x

Outra questão é natural:

Questão 2: Qual a modelagem matemática do problema em dimensão 3 ? Ou seja,


quando damos largura e espessura fixadas, mas podemos girar o objeto no espaço ?
Dito de outro modo, o que fazer quando queremos passar um objeto como uma escada
bem comprida numa esquina ?

8.3. Área máxima do retângulo que dobra a esquina? Qual a área máxima
de uma figura retangular que consiga dobrar a esquina, no caso l1 = l2 = 1 ?
Se a figura é um quadrado de lado l é fácil de ver que l = 1 é o máximo, como na
Figura a seguir.

Portanto a área máxima de um quadrado que dobra essa esquina é 1. Mas, e se


fosse um retângulo não-quadrado ?
Como antes vou imaginar os retângulos se apoiando em C.
Pela simetria (l1 = l2 = 1 e o ângulo reto na esquina), posso pensar que a figura
retangular que se apoia em C é formada de duas partes de mesma área e formato,
uma para a direita de C e outra para a esquerda de C.
CAPÍTULO 14. DERIVADA DA COMPOSIÇÃO DE FUNÇÕES 199

Ademais, para um mesmo perı́metro, o quadrado é o retângulo de maior área (ver


Exercı́cio 10.10). Por isso, imagino à esquerda de C um quadrado de lado l e à es-
querda de C, outro, também de lado l, formando então um retangulo de comprimento
2l e largura l. Veja a Figura:
P 2

l
l
l
P 1
l
C

Agora continuo o lado da figura, de modo a obter triângulos como na figura que
segue:
P 2
θ
l r
l
1
l
P 1
l θ
C

Dos triângulos formados obtemos:


1 l
= sin(θ) e = tan(θ).
l+r r
Logo
l 1
r= e l+r = ,
tan(θ) sin(θ)
ou seja:
1 1
l · (1 + )=
tan(θ) sin(θ)
de onde:
tan(θ)
l(θ) = ,
sin(θ) · (1 + tan(θ))
8. MÁXIMOS E MÍNIMOS: O PROBLEMA DO FRETEIRO 200

Se encontramos um mı́nimo dessa função l(θ), para 0 < θ < π2 , esse será o imped-
imento a passar a figura retangular pela esquina, ou seja, dará o máximo da medida
l do retângulo (e com esse valor saberemos a área máxima da figura retangular).
Mas
sin(θ) − cos(θ)
l′ (θ) = .
1 + 2 · sin(θ) cos(θ)
Claramente, para 0 < θ < π2 :
π
l′ (θ) = 0 ⇔ sin(θ) = cos(θ) ⇔ θ = .
4
1
Como limθ→0 1+tan(θ) = 1, então
tan(θ) 1
lim l(θ) = lim = lim = 1,
θց0 θց0 sin(θ) θց0 cos(θ)

1
e como limθ→ π2 sin(θ)
= 1, então
tan θ
limπ l(θ) = limπ = 1.
θր 2 θր 2 1 + tan(θ)
Então
π 1
l( ) = √
4 2
é o mı́nimo global de l(θ). Veja a Figura:

0,9

0,85

0,8

0,75

0,2 0,4 0,6 0,8 1 1,2 1,4


theta

Figura: Gráfico de y = l(θ), θ ∈ (0.1, π2 − 0.1), onde π


4
≈ 0.78

Portanto a área máxima da figura retangular que dobra a esquina é:


1
2 · ( √ )2 = 1,
2
a mesma que encontramos para o quadrado de área máxima que dobra essa esquina.
Está ainda um problema em aberto determinar a área máxima da figura capaz de
dobrar a esquina, mesmo no caso l1 = l2 = 1, se deixamos livre o formato da figura.
Ou seja, valem figuras feitas de pedaços distintos, alguns curvados , etc.
CAPÍTULO 14. DERIVADA DA COMPOSIÇÃO DE FUNÇÕES 201

Há cotas máximas para a área, mas não se obteve ainda explicitamente uma figura
da qual se possa dizer: é esta ! É conhecido na literatura como o problema do sofá.
8.4. O caso L ≈ 0, mas com uma parede suave. Retomo o caso em que
L ≈ 0 e ainda na situação bem simples em que l1 = l2 = 1.
Coloque a Figura de um corredor que dobra em ângulo reto num sistema de
coordenadas cartesianas (x, y) de modo que:
• o ponto C seja C = (1, 1),
• a parede vertical externa faça parte da reta x = 0,
• a vertical interna, de x = 1,
• a parede horizontal externa faça parte de y = 2 e
• a vertical interna, de y = 1.
Imagine agora que as paredes internas (vertical e horizontal) da Figura sejam
derrubadas e substituı́das por uma parede suave, curvada, que faça parte do gráfico
de:
ǫ
y = fǫ (x) := 1 − , x > 1,
1−x
onde sempre ǫ > 0.
A figura a seguir mostra o que acontece para três escolhas de ǫ:

ǫ
Gráficos de y = 1 − 1−x com ǫ = 1 (vermelho)
ǫ = 0.5 (verde), ǫ = 0.2 (amarelo), y = 1 em azul
ǫ
Diminuindo ǫ o gráfico de y = 1 − 1−x vai se apertando sobre a parede horizontal
interna (em azul y = 1): de fato, cada x > 1 fixado,
fǫ (x) > fǫ′ (x), se ǫ < ǫ′ .
E também é claro que, fixado qualquer ǫ > 0,
lim fǫ (x) = 1
x→+∞

Note que se ǫ 6= 0, ainda que pequeno, a função é derivável e


ǫ
fǫ′ (x) = .
(x − 1)2
8. MÁXIMOS E MÍNIMOS: O PROBLEMA DO FRETEIRO 202

Então
lim fǫ′ (x) = +∞,
xց1

o que mostra que os gráficos de fǫ vão ficando cada vez mais verticais próximos de
x = 1.
Você também pode escrever a partir de fǫ (x):
(y − 1) · (x − 1) = −ǫ,
o que mostra que quando ǫ → 0 obtemos2:
(y − 1) · (x − 1) = 0
que é a união de retas x = 1 e y = 1.
Ou seja que as paredes internas foram substituı́das por um curvada como na
Figura a seguir (fixado um ǫ) e que a medida que o ǫ fica pequeno mais vai ficando
próxima da parede interna original em formato de letra L.

O Problema agora para o freteiro:


Problema: passar a maior vara possı́vel, sem entortá-la, possivelmente apoiando
a vara em algum ponto da parede interna suavizada.

A solução que proponho é a seguinte:


Estratégia: usar a resposta do caso original, com parede em forma de letra L,
para solucionar o caso em que a parede é suave

Comecemos com l1 = l2 = 1 (depois passo ao geral, l1 , l2 quaisquer).


Quero encontrar o ponto Cǫ = (x, fǫ (x)) e a inclinação da vara V em Cǫ tais que
seja minimizada a distância P1 P2 onde
P1 := V ∩ (x = 0) e P2 := V ∩ (y = 2).

2A curvatura κǫ desses gráficos e seu limite quando ǫ → 0 serão estudados na Seção 7 do Capı́tulo
28
CAPÍTULO 14. DERIVADA DA COMPOSIÇÃO DE FUNÇÕES 203

Meu candidato a ponto Cǫ será o ponto (xǫ , fǫ (xǫ )) do gráfico de y = fǫ (x) que
tem
l2 1
fǫ′ (xǫ ) = ( ) 3 = 1
l1
já que a solução do caso original era em
l2 1 π
θ0 = arctan(( ) 3 ) = arctan(1) = .
l1 4
E as retas que se apoiam na parede curvada serão as suas retas tangentes.
As soluções de fǫ′ (x) = 1 são

1 + ǫ1/2 e 1 − ǫ.
Fico apenas com √
xǫ := 1 + ǫ,
pois a outra solução está à esquerda da reta x = 1.
As retas tangentes de y = fǫ (x) num ponto geral (x, fǫ (x)) são:
ǫ x2 − 2(1 + ǫ) · x + 1 + ǫ
y= · x + .
(x − 1)2 (x − 1)2
e em particular em (xǫ , fǫ (xǫ )) a reta tangente é:
y = x − 2ǫ1/2 .

A intersecção de y = x − 2 ǫ com y = 2 é o ponto:

P2 := (2 + 2 ǫ, 2)
enquanto que a intersecção dela com x = 0 é:

P1 := (0, −2 ǫ).
A distância P1 P2 é (para l1 = l2 = 1):
q √ 2 √ 2 √ q √
mǫ := (2 + 2 ǫ) + (2 + 2 ǫ) = 2 · (2 + 2 ǫ)2 ,
e note que √
lim mǫ = 2 2 ≈ 2.828427124,
ǫ→0
o comprimento da diagonal do quadrado de lado 2, solução do caso original na figura
em forma de L.
Queremos ver se mǫ é o mı́nimo das distâncias P1 P2 onde P2 é a intersecção de
uma reta tangente genérica de y = fǫ (x) com y = 1 + l2 = 2 e P1 a intersecção da
reta tangente genérica com x = 0.
Ora,
2ǫx − ǫ − x2 + 2x − 1
P1 = (0, − ),
(x − 1)2
2ǫx − ǫ + x2 − 2x + 1
P2 = ( , 2),
ǫ
e s
(2ǫx − ǫ + x2 − 2x + 1)2 2ǫx − ǫ − x2 + 2x − 1 2
P1 P2 (x) = + (2 + ).
ǫ2 (x − 1)2
8. MÁXIMOS E MÍNIMOS: O PROBLEMA DO FRETEIRO 204

O numerador da fração3 que é P1 P2 (x) é dado pelo polinômio de grau 8 em x:
(ǫx5 − 5ǫx4 + 10ǫx3 − 10ǫx2 + 5ǫx − ǫ + x6 − 6x5 + 15x4 − 20x3 + 15x2 − 6x + 1 − ǫ3 x)·
·2 · (2ǫx − ǫ + x2 − 2x + 1),

e verifica-se que em x0 = 1 + ǫ:
′ √
P1 P2 (1 + ǫ) = 0

pois x0 = 1 + ǫ é raiz do fator de grau 5 em x:
ǫx5 − 5ǫx4 + 10ǫx3 − 10ǫx2 + 5ǫx − ǫ + x6 − 6x5 + 15x4 − 20x3 + 15x2 − 6x + 1 − ǫ3 x.
′′ √
Já a enorme fração que é P1 P2 (x) avaliada em x0 = 1 + ǫ vale:
√ √
2 2(2ǫ2 + 3 + 15ǫ + 11 ǫ + 9ǫ3/2 )
√ > 0.
ǫ(1 + ǫ)3

Logo x0 = 1 + ǫ é minimo local de P1 P2 (x).
Mas é bem claro que, para cada ǫ fixado:
lim P1 P2 (x) =
xց1
s
(2ǫx − ǫ + x2 − 2x + 1)2 2ǫx − ǫ − x2 + 2x − 1 2
= lim + (2 + ) = +∞
xց1 ǫ2 (x − 1)2
assim como
lim P1 P2 (x) =
x→+∞
s
(2ǫx − ǫ + x2 − 2x + 1)2 2ǫx − ǫ − x2 + 2x − 1 2
= lim + (2 + ) = +∞.
x→+∞ ǫ2 (x − 1)2

400

300

200

100

0
1,5 2 2,5 3 3,5 4
x

As funções P1 P2 (x) para ǫ = 1 (vermelho) e ǫ = 0.1 (verde)


x0 = 2 e 1.316227766 resp., m1 = 5.656854249 e m0.1 = 3.722854312.

3Conferi as contas que seguem no Maple, pois ficam grandes.


CAPÍTULO 14. DERIVADA DA COMPOSIÇÃO DE FUNÇÕES 205

9. Exercı́cios
Exercı́cio 9.1. Usando a regra do quociente e definições/relações trigonométricas,
prove que
cot′ (x) = − csc2 (x),
1 1
onde cot(x) = tan(x) e csc(x) := sin(x) .
Também mostre que:
sec′ (x) = tan(x) sec(x),
1
onde sec(x) := cos(x)
.

Exercı́cio 9.2. Considere f (x) = x2x+1 .


i) note que ela está definida em todos os reais.
ii) mostre que limx→+∞ f (x) = limx→−∞ f (x) = 0.
iii) determine seus pontos de máximo e mı́nimo locais (usando f ′ (x) e/ou f ′′ (x)).
iv) com o item ii) e iii) conclua que os máximos e mı́nimos locais são globais.
v) determine seus dois pontos de inflexão. (Dica: se você fizer cuidadosamente o
cálculo de f ′′ (x) verá que há simplificações no numerador e que fica fácil determinar
onde f ′′ (x) = 0.)
Exercı́cio 9.3. Considere o gráfico da função y = Ax , onde A > 0 fixado, para x > 0.
Considere retângulos formados pelos pontos (0, 0), P1.P 2, P3, onde P1 = (x, 0),
P2 = (x, Ax ) e P3 = (0, Ax ).
i) Note que todos eles têm a mesma área = A.
ii) Qual deles tem o menor perı́metro ? (Dica: determine um mı́nimo local e prove
que ele é de fato mı́nimo global)
Exercı́cio 9.4. Considere as funções y = fn (x) := x2n + x12n , onde n ∈ N.
i) Determine limx→0 fn (x), limx→+∞ fn (x) e limx→−∞ fn (x).
ii) Determine seus pontos de mı́nimos locais / globais.
iii) Prove que a concavidade desses gráficos é sempre para cima.
Exercı́cio 9.5. Calcule a segunda derivada da função
sin(x)
tan(x) := .
cos(x)
Exercı́cio 9.6. (resolvido)
Imagine que voce se lembra de cor da fórmula do seno da soma:
sin(x + y) = sin(x) · cos(y) + cos(x) · sin(y),
mas que se esqueceu completamente da fórmula do cosseno da soma.
i) Como o Cálculo pode obter a formula para o cosseno? Ou seja, como saber
derivar pode ajudar ?
ii) E se sei a do cosseno da soma, como obter a do seno da soma via Cálculo ?
Exercı́cio 9.7. Um ponto P move-se sobre a curva de equação y 3 − x2 = 0.
Determine a taxa de variação da coordenada y no instante em que P = (8, 4), se
a taxa de variação da coordenada x no mesmo instante é 1cm/s.
9. EXERCÍCIOS 206

Em outras palavras, a coordenada y ao longo dessa curva aumenta ou diminui, no


ponto P , quando aumentamos a coordenada x.
Obs. você não precisa esboçar a curva.
CAPı́TULO 15

Derivadas de funções Implı́citas

1. Curvas versus gráficos


Comecemos com a equação do cı́rculo de raio r:
x2 + y 2 = r 2 .
É importante nos darmos conta de que o cı́rculo como um todo não é gráfico de
nenhuma função f : R → R1.
Mas, dado um ponto P (x, y) do cı́rculo, uma porção do cı́rculo perto de P pode
ser descrita:
• como gráfico de y = y(x), para x num intervalo centrado em x, ou
• como gráfico de x = x(y), para y num intervalo centrado em y.

De fato, há dois casos a considerar:


Caso 1: se P = (x, y) no cı́rculo tem coordenada
x 6= −r, r,
√ √
então perto de P o cı́rculo é gráfico de y = 1 − x2 ou de y = − 1 − x2 .

Caso
p 2: se P é (−r, 0)
p ou P = (r, 0), então perto de P o cı́rculo é gráfico de x =
1 − y ou de x = − 1 − y 2 .
2

No Caso 1 podemos calcular a derivada da função y = y(x), para x num intervalo,


do seguinte modo: derivo a expressão x2 + y(x)2 = r 2 pela regra da composta:
(x2 + y(x)2 )′ = (r 2 )′ ⇔ 2x + 2y(x)y ′(x) = 0 ⇔
−2x
⇔ y ′(x) = .
2y(x)
√ √
E agora substituindo y(x) por 1 − x2 , se y > 0, ou por y = − 1 − x2 se y < 0,
temos:
−2x −x
y ′(x) = =√ , se y > 0,
2y(x) 1 − x2
ou
−2x x
y ′(x) = =√ , se y < 0.
2y(x) 1 − x2
1Não
confunda essa afirmação com o fato do cı́rculo ser uma curva de nı́vel r2 da função F :
R → R, F (x, y) = x2 + y 2 .
2

207
1. CURVAS VERSUS GRÁFICOS 208

No Caso 2 podemos obter a derivada da função x = x(y), para y num intervalo , do


seguinte modo: derivo a expressão (x(y))2 + y 2 = r 2 em y, pela regra da composta:
( (x(y))2 + y 2 )′ = (r 2 )′ ⇔ 2x(y)x′ (y) + 2y = 0 ⇔

−2y
⇔ x′ (y) = .
2x(y)
p p
E agora substituindo x(y) por 1 − y 2, se x > 0, ou por x = − 1 − y 2 se x < 0:
−2y −y
x′ (y) = =p , se x > 0,
2x(y) 1 − y2
ou
−2y y
x′ (y) = =p , se x < 0.
2x(y) 1 − y2

Isso que fizemos se chama derivação implı́cita. É útil mesmo quando não sabemos
a expressão explı́cita de y = y(x) ou de x = x(y).

Por exemplo, se nos damos uma curva no plano através de uma equação do tipo:
x2 y 2 − 3y 2 + y 4 − 8y + 2y 3 − 4 = 0
verificamos facilmente que (0, 2) é um ponto dessa curva.
Será que, num pequeno trecho perto de (0, 2) temos a curva dada como um gráfico
y = y(x) ? Ou seja, ∀x num intervalo aberto centrado em x = 0, será que
x2 y(x)2 − 3y(x)2 + y(x)4 − 8y(x) + 2y(x)3 − 4 = 0 ?.
Veremos que neste Exemplo esse é o caso (graças ao Teorema 2.1 a seguir).
Então supondo por um momento que sabemos que há um gráfico y = y(x) perto
de (0, 2) qual o valor de y ′ (x) em (x, y) = (0, 2) ?
Fazemos a derivada em x:
(x2 y(x)2 − 3y(x)2 + y(x)4 − 8y(x) + 2y(x)3 − 4)′ = 0 ⇔

2xy(x)2 + x2 2y(x)y ′(x) − 6y(x)y ′(x) + 4y(x)3y ′ (x) − 8y ′(x) + 6y(x)2y ′ (x) = 0

⇔ 2xy(x)2 + y ′ (x)[x2 2y(x) − 6y(x) + 4y(x)3 − 8 + 6y(x)2] = 0

′ −2xy(x)2
⇔ y (x) = 2
x 2y(x) − 6y(x) + 4y(x)3 − 8 + 6y(x)2
que dá em (x, y) = (0, 2)
0
y ′ (0) =
= 0,
48
ou seja que o gráfico y = y(x) em torno de (x, y) = (0, 2) tem reta tangente horizontal
nesse ponto.
CAPÍTULO 15. DERIVADAS DE FUNÇÕES IMPLÍCITAS 209

2. Teorema da função implı́cita


Como saberemos se lidamos com y = y(x) ou x = x(y) em torno de um ponto
P = (x, y) de uma curva F (x, y) = 0 ?
O Teorema 2.1 a seguir dá uma resposta (sua prova se vê em Análise Matemática):
Para poder enunciá-lo vamos introduzir um sı́mbolo novo: dada uma expressão
F (x, y) em duas variáveis, defino ∂F∂x
(x,y)
como sendo a derivada dessa expressão em
x (se houver), onde se considera y fixado. Por exemplo: se F (x, y) = yx2 + y 2 então
∂F (x,y)
∂x
= 2yx. Se F (x, y) = y 2 então ∂F∂x (x,y)
≡ 0. Se F (x, y) = exp(x)y 2 , então
∂F (x,y)
∂x
= exp(x)y 2 .
E analogamente, ∂F∂y(x,y)
se define como a derivada dessa expressão em y (se hou-
ver), onde se considera x fixado.
Teorema 2.1. (Teorema da função Implı́cita).
Seja F (x, y) um polinômio em duas variáveis.2
Suponha que exista (x, y) com F (x, y) = 03
Se ∂F∂y
(x,y)
6= 0 quando avaliada em (x, y), então para x, y em (possivelmente pe-
quenos) intervalos abertos centrados em x, y:
• a curva F (x, y) = 0 é um gráfico do tipo y = y(x) e
∂F (x,y)
• y ′ (x) = − ∂F∂x
(x,y) .
∂y
∂F (x,y)
Se 6= 0 quando avaliada em (x, y), então para x, y em (possivelmente pe-
∂x
quenos) intervalos abertos centrados em x, y::
• a curva F (x, y) = 0 é um gráfico do tipo x = x(y) e
∂F (x,y)

• x′ (y) = − ∂F∂y
(x,y) .
∂x

Esse Teorema tem vários detalhes, que se vêem melhor nos Exemplos.

∂F (x,y)
Exemplo 2.1. No cı́rculo F (x, y) = x2 + y 2 − r 2 = 0 temos ∂y
= 2y 6= 0 se y 6= 0.
Nesse caso:
∂F (x,y)
2x
y ′(x) = − ∂F∂x
(x,y)
= − ,
2y(x)
∂y
como vimos antes.
Mas se P no cı́rculo tem y = 0 então P = (−r, 0) ou P = (r, 0) e nesse caso
∂F (x,y)
∂x
= 2x 6= 0. Então é preciso usar funções x = x(y) para descrever o cı́rculo
como gráfico.
O Teorema 2.1 tem sutilezas que ficam evidentes no Exemplo a seguir:
2háversões mais gerais desse enunciado, onde F é muito geral, sujeito apenas a certas exigências
de derivabilidade
3Não queremos ter conjuntos vazios como F (x, y) = x2 + y 2 + 3 = 0.
2. TEOREMA DA FUNÇÃO IMPLÍCITA 210

Exemplo 2.2. Voltando ao exemplo que analisamos acima,


F (x, y) = x2 y 2 − 3y 2 + y 4 − 8y + 2y 3 − 4 = 0
temos
∂F (x, y)
= 2xy 2 ,
∂x
que se anula em P = (0, 2), mas temos
∂F (x, y)
= x2 2 y − 6 y + 4 y 3 − 8 + 6 y 2
∂y
que não se anula em P = (0, 2). Logo há um gráfico y = y(x) em torno de (0, 2) e já
calculamos y ′(0) = 0 acima.
Até agora não comentei o fato de que P = (0, −1) também satisfaz:
x2 y 2 − 3y 2 + y 4 − 8y + 2y 3 − 4 = 0.
Isso é interessante pois diz que para o mesmo valor x = 0 há dois valores y que
satisfazem F (x, y) = 0 !
Ou seja que é só num pequeno entorno de (0, 2) que pode ser descrito como gráfico
de y = y(x) , mas não todo o conjunto F (x, y) = 0.
Por outro lado, em (0, −1) tanto ∂F∂x
(x,y)
= 2xy 2 quanto
∂F (x, y)
= x2 2 y − 6 y + 4 y 3 − 8 + 6 y 2
∂y
se anulam !
Nessa caso o Teorema 2.1 não tem nada a dizer ! Ele não pode garantir nenhum
tipo de gráfico local y = y(x) ou x = x(y).
Ainda bem que o Teorema se calou nessa caso, pois em (0, −1) a curva F (x, y) = 0
tem uma espécie de laço, que não se deixa descrever nem como gráfico de y = y(x)
nem como gráfico de x = x(y).

A Figura a seguir dá uma idéia da curva, que não por acaso se chama conchóide:

y 0
-4 -2 0 2 4
-1x

-2
CAPÍTULO 15. DERIVADAS DE FUNÇÕES IMPLÍCITAS 211

Figura: Em (0, 2) vemos um pequeno gráfico horizontal y = y(x). Mas


em (0, −1) forma-se um laço.

Exemplo 2.3. O caso de

3x2
x3 + xy 2 − − y2 = 0
2
expõe outra sutileza do Teorema 2.1.
Note que essa curva tem sobre o eixo dos x exatamente dois pontos: (0, 0) e (0, 23 ).
Em (0, 32 ) temos (como o leitor pode verificar)

∂F (x, y) ∂F (x, y) 9
= 0, =
∂y ∂x 4

e o Teorema 2.1 diz que a curva F (x, y) = 0 se representa localmente como gráfico
x = x(y). Ademais calcula x′ ( 32 ) como

3 0
x′ ( ) = − 9 = 0,
2 (4)

ou seja que o gráfico é vertical.


Mas em (0, 0) temos
∂F (x, y) ∂F (x, y)
= = 0.
∂y ∂x
De fato esse ponto é completamente isolado do resto da curva ! Ou seja, não pode
ser visto como gráfico de uma função cujo domı́nio é um intervalo aberto em torno de
x = 0.
Na Figura a seguir o Maple não enxerga o (0, 0) na curva !

y 0
1,1 1,2 1,3 1,4 1,5
x
-1

-2

-3
3. RETA TANGENTE DE CURVA E PLANO TANGENTE DE SUPERFÍCIE212

3. Reta tangente de curva e plano tangente de superfı́cie


O Teorema 2.1 nos diz que, se uma curva F (x, y) = 0 é localmente, em torno de
(x, y), da forma y = y(x) então
∂F
′ ∂x
(x, y)
y (x) = − ∂F .
∂y
(x, y)
A reta tangente em (x, y) ao pedaço de gráfico y = y(x) foi definida na Seção 2 do
Capı́tulo 8 como:
y = y ′(x) + (y − y ′ (x) · x),
ou seja,
∂F ∂F
∂x ∂x
y= − ∂F · x + (y − ∂F
· x).
∂y ∂y
∂F
Multiplicando por ∂y
(x, y) e simplificando obtemos:
∂F ∂F
(x, y) · (x − x) + (x, y) · (y − y) = 0,
∂x ∂y
por isso defino:
∂F
Definição 3.1. Seja F (x, y) = 0 curva contendo o ponto (x, y) para o qual ∂x
(x, y) 6=
0 ou ∂F
∂y
(x, y) 6= 0. Então sua reta tangente em (x, y) é definida por:
∂F ∂F
(x, y) · (x − x) + (x, y) · (y − y) = 0,
∂x ∂y

Podemos dar uma definição análoga quando ao invés de uma curva no plano (x, y)
tivermos uma superfı́cie no espaço (x, y, z), dada em forma implı́cita pela equação
F (x, y, z) = 0:
Definição 3.2.
Seja F (x, y, z) = 0 contendo o ponto (x, y, z).
Se ∂F
∂x
(x, y, z)) 6= 0 ou ∂F
∂y
(x, y, z) 6= 0 ou ∂F
∂y
(x, y, z) 6= 0, então seu plano tangente
em (x, y, z) é definido por:
∂F ∂F ∂F
(x, y, z) · (x − x) + (x, y, z) · (y − y) + (x, y, z) · (z − z) = 0.
∂x ∂y ∂z

Exemplos:
• por essa definição a esfera de raio 1 dada por x2 + y 2 + z 2 − 1 = 0 tem em
(0, 0, 1) o plano tangente
∂F
(0, 0, 1) · (z − 1) = 2 · (z − 1) = 0,
∂z
que é o mesmo que o plano horizontal z = 1 no espaço (x, y, z).
CAPÍTULO 15. DERIVADAS DE FUNÇÕES IMPLÍCITAS 213

• a equação z 2 − x2 − y 2 = 0 define uma superfı́cie conhecida como cone de


duas folhas. No ponto (0, 0, 0):
∂F ∂F ∂F
= = = 0,
∂x ∂y ∂x
e nele portanto não está definido um plano tangente. Por isso esse ponto é
especial ou singular.

4. Tangentes, pontos racionais de cúbicas e códigos secretos


Consideremos uma cúbica em forma implı́cita, ou seja, uma curva dada por:
y 2 − x3 − b x − a = 0, a, b ∈ R,
ou equivalentemente:
y 2 = x3 + b x + a a, b ∈ R.
Quando se trabalha com computadores, o melhor dos mundos é lidar com números
Racionais. E duas questões muito importantes e atuais, que estão relacionadas com
a aplicação da matemática à criptografia, são:

Questão 1: Seja a curva dada por


y 2 = x3 + b x + a a, b ∈ Q.
Quem são ou quantos são os pontos P = (x, y) da curva que têm ambas coordenadas
Racionais ?

Questão 2: Dado um ponto P dessa curva com coordenadas Racionais, como


produzir outros pontos dela que também tenham coordenadas Racionais ?

Usaremos a notação P = (x, y) ∈ Q × Q para dizer que ambas as coordenadas são


Racionais.
A seguinte Afirmação é um método para atacar a segunda questão:
Afirmação 4.1. (Método das secantes e das tangentes)
Considere uma cúbica com coeficientes Racionais da forma
F (x, y) = y 2 − x3 − b x − a a, b ∈ Q.
• i) sejam P1 = (x1 , y 1 ) ∈ Q × Q e P2 = (x2 , y 2 ) ∈ Q × Q de F (x, y) = 0,
distintos. Se a reta que os liga não é vertical então ela intersecta a cúbica
em P3 = (x3 , y 3 ) ∈ Q × Q.
• ii) Suponha que ∂F ∂y
= 2y não se anula em P = (x, y) ∈ Q × Q. Então a reta
tangente a F (x, y) em P intersecta a cúbica num ponto Q que também tem
coordenadas Racionais.
Demonstração.
De i):
4. TANGENTES, PONTOS RACIONAIS DE CÚBICAS E CÓDIGOS
SECRETOS 214

A reta ligando P1 e P2 é:


y − y1 x2 y 1 − x1 y 2
y=( 2 )·x+ =
x2 − x1 x2 − x1
= A · x + b,
ou seja, tem coeficientes angular A e linear B Racionais.
Queremos resolver a equação
(A x + B)2 − x3 − b x − a = 0,
mas
(A x + B)2 − x3 − b x − a = (x − x1 ) · (x − x2 ) · q(x),
onde o grau do polinômio q(x) é 3 − 2 = 1.
Mas, como se viu na prova do Teorema 7.1 do Capı́tulo 6 e na Digressão que se
seguiu, os coeficientes de q(x) são Racionais.
Logo a terceira solução é a raı́z de
p1 p2
p(x) = ·x+ =0
q1 q2
e portanto produz um ponto P3 da cúbica com coordenadas Racionais.

De ii):
Pelo Teorema 2.1, F (x, y) localmente em torno de P é um gráfico de y = y(x),
com
∂F
−3x2 − b
y ′(x) = − ∂F
∂x
=− .
∂y
2y
Como b, x, y ∈ Q então y ′(x) avaliada em P = (x, y) é um número Racional, que
denoto aqui de A.
A equação da reta tangente é do tipo:
rP : y = Ax + B
onde o valor do coeficiente linear B se obtêm de:
y = Ax+ B ⇔ B = y − A x,
e portanto B também é um número Racional.
As coordenadas x dos pontos na intersecção F (x, y) ∩ rP são as soluções de:
F (x, y) = 0 e y = A x + B,
ou seja, soluções de
(A x + B)2 − x3 − b x − a = 0,
ou, equivalentemente,
−x3 + A2 x2 + (2AB − b) x + B 2 − a = 0.
Agora é o momento de lembrar que a coordenada x de P = (x, y) é uma raı́z dupla
ou tripla desse polinômio, já que rP é tangente à curva F (x, y) nesse ponto (tripla
seria o caso de um ponto de inflexão).
CAPÍTULO 15. DERIVADAS DE FUNÇÕES IMPLÍCITAS 215

No caso em que x é raı́z dupla exatamente, pelo Teorema 4.1 do Capı́tulo 13:
−x3 + A2 x2 + (2AB − b) x + B 2 − a = (x − x)2 · q(x).
onde o grau do polinômio q(x) é 3 − 2 = 1. Ademais os coeficientes de q(x) são
Racionais (Teorema 7.1, Capı́tulo 6 e Digressão).
Ou seja, q(x) = q1 x + q0 , com q0 , q1 ∈ Q e a raı́z de q(x) é
−q0
.
q1
O ponto Q 6= P buscado é portanto:
−q0 −q0
Q=( , A( ) + B ),
q1 q1
que nitidamente tem coordenadas Racionais.
Se P é ponto de inflexão, então Q = P , ou seja,
rP ∩ F (x, y) = {P, Q} = {P }.


Exemplo 4.1. Considere a curva analisada por Billing, em 1937:


y 2 − x3 + 82 x = 0.
Fora o óbvio (0, 0) há três pontos com coordenadas Racionais relativamente simples
49 231
P1 = (−1, 9), P2 = (−8, 12), P3 = ( , ).
4 8
A Figura a seguir mostra como o Maple plota para essa curva:

100

50

y 0
-5 0 5 10 15 20
x

-50

-100

Vou implementar neste Exemplo o que a prova da Afirmação 4.1 nos ensinou (as
contas tediosas foram feita com o Maple).
4. TANGENTES, PONTOS RACIONAIS DE CÚBICAS E CÓDIGOS
SECRETOS 216

A reta tangente ao gráfico local y = y(x) de F (x, y) = 0 em P1 = (−1, 9) é:

79 83
rP 1 : − x+ .
18 18

A intersecção rP1 ∩ F (x, y) = {P1 , Q1 } tem

6889 517339
Q1 = ( ,− ) ∼ (21, −88).
324 5832

Ver a Figura:

100

50

y 0
-10 -5 0 5 10 15 20
x

-50

-100

Agora podemos continuar o processo.


Tomo Q1 , a tangente rQ1 e determino rQ1 ∩ F (x, y) = {q1 , Q2 } onde Q2 terá
coordenadas Racionais.
Faço as contas e obtenho:

44588977 4653507299
rQ 1 : − x+
6208068 72701712

3143435938720609 6994054838592555031151
Q2 = ( ,− ) ∼ (9, −1).
346860974633616 6460009551215289641664

A Figura a seguir mostra isso:


CAPÍTULO 15. DERIVADAS DE FUNÇÕES IMPLÍCITAS 217

100

50

y 0
-10 -5 0 5 10 15 20
x

-50

-100

Um Teorema de Billing diz que se continuamos o processo, agora em Q2 e assim


sucessivamente, produzimos uma infinidade de pontos da curva com coordenadas
Racionais.
O mesmo ocorreria se tivéssemos começado com P2 ou P3 .

4.1. Códigos secretos.


Agora imagine que alguém quer criar uma operação de duplicação muito estranha.
Poderia definir que, para4
P1 := (−1, 9),
6889 517339
2 ⋆ P1 := Q1 = ( ,− ).
324 5832
E depois, do mesmo modo5
2 ⋆ Q1 := Q2
Ou seja:
3143435938720609 6994054838592555031151
4 ⋆ P1 = ( ,− ).
346860974633616 6460009551215289641664
Agora note que:
• 4 ⋆ P1 é obtido a partir de P1 de modo exato (por ser Racional), computa-
cionalemte de modo rápido, apesar de ser completamente diferente de P1
• mas a natureza de 4 ⋆ P1 torna-se impenetrável se não digo quem é P1 ou
qual a equação da cúbica que usei.
4Defato na teoria de curvas elı́pticas se tomaria no lugar de Q1 o ponto da cúbica que é simétrico
de Q1 em relação ao eixo dos x.
5Novamente, se usa de fato que o ponto da cúbica que é simétrico de Q em relação ao eixo dos
2
x.
5. DERIVAÇÃO IMPLÍCITA DE SEGUNDA ORDEM 218

• essa enorme assimetria entre a passagem


P1 7→ 4 ⋆ P1
e a passagem
4 ⋆ P1 7→ P1
é a base de um código secreto poderoso.
O leitor que se sentiu instigado deve procurar então estudar a teoria de criptografia
sobre as chamadas cúbicas na forma de Wierstrass.

5. Derivação implı́cita de segunda ordem


Na Seção 5 do Capı́tulo 3 associamos a Figura:

y 0
-1 -0,5 0 0,5 1 1,5 2
x

-1

-2

à curva y 2 − x3 − 1 = 0. Mas tem algo que não ficou plenamente justificado. Parece
na Figura que há 2 pontos de inflexão, em torno de x ∼ 0.8.
Vamos considerar ao invés daquela curva, outra bem parecida (mas mais adequada
para nossas contas):
F (x, y) = y 2 − x3 − 4x = 0.
A inflexão deve aparecer onde a segunda derivada y ′′ (x) muda de sinal, ou seja
onde y ′′ (x) = 0.
Só que já sabemos que aqui não se trata de um gráfico, mas apenas de uma curva.
Por isso precisamos da derivação implı́cita, só que agora para calcular a segunda
derivada.
Já sabemos que se y 6= 0:
∂F
′ ∂x 3x2 + 4
y (x) = − ∂F = .
∂y
2y
Então calculo
3x2 + 4 ′
y ′′ (x) = ( )
2y
pela regra do quociente, obtendo:
12x · y − (3x2 + 4) · 2y ′(x)
y ′′(x) = =
4y 2
CAPÍTULO 15. DERIVADAS DE FUNÇÕES IMPLÍCITAS 219
2
12x · y − (3x2 + 4) · 2( 3x2y+4 )
= =
4y 2

12xy 2 − 9x4 − 24x2 − 16


= .
4y 3
Preciso ver as raı́zes de y ′′(x), ou seja, as raı́zes de

12x(x3 + 4x) − 9x4 − 24x2 − 16

já que posso substituir


y 2 = x3 + 4x.

Ora,
12x(x3 + 4x) − 9x4 − 24x2 − 16 = 3x4 + 24x2 − 16,

que sabemos resolver (pense em z = x2 e resolva 15z 2 + 72z − 16 = 0).


Assim obtenho as raı́zes:
q q q q
2 √ 2 √ 2 √ 2 √
− −9 + 6 3, −9 + 6 3, − −9 − 6 3, −9 − 6 3,
3 3 3 3
das quais a única Real e positiva é
q
2 √
x := −9 + 6 3 ∼ 0.78.
3

Para este valor de x há dois valores de y na curva y 2 = x3 + 4x:


r q
2 √ √
6(−9 + 6 3)3/2 + 54 −9 + 6 3 ∼ 1.9
9
e
r q
2 √ √
− 6(−9 + 6 3)3/2 + 54 −9 + 6 3 − 1.9
9
Agora, já que já temos y ′ (x), é um trabalho tedioso achar a equação da reta tangente
em por exemplo:
q r q
2 √2 √ √
( −9 + 6 3 , 6(−9 + 6 3)3/2 + 54 −9 + 6 3 ).
3 9
Com essa equação posso plotar a cúbica e sua tangente, que mostra bem que há
uma inflexão nesse ponto:
6. EXERCÍCIOS 220

y 0
-2 -1 0 1 2 3 4 5
x

-4

-8

6. Exercı́cios
Exercı́cio 6.1. (resolvido)
Considere F (x, y) = y 2 − x3 = 0. Considere o ponto (1, 1) dessa curva.
i) usando o Teorema 2.1 verifique que perto de (1, 1) essa curva é o gráfico de uma
função y = y(x).
ii) calcule a derivada da função do item i) em (1, 1).
iii) note que (1, −1) também está na curva F (x, y) = y 2 − x3 = 0 e portanto ela
não é globalmente um gráfico de y = y(x).
Exercı́cio 6.2. Considere a cúbica F (x, y) = y 2 − x3 − 4x = 0.
Um fato muito bonito é que esta curva só tem 3 pontos com coordenadas Racionais:
(0, 0), (2, 4) e (2, −4).
Suponha esse fato.
Por outro lado ∂F∂y (x,y)
= 2y não se anula em (2, 4) nem em (2, −4), o que nos dá
a oportunidade de usar o método das tangentes (Afirmação 4.1) para obter pontos
racionais a partir deles.
i) conclua sem fazer nenhuma conta que as retas tangentes a F (x, y) em (2, 4) e
em (2, −4) passam pela origem (0, 0).
ii) faça as contas e obtenha as equações dessas duas retas tangentes.
CAPı́TULO 16

Funções inversas e suas derivadas

Vimos na Seção 1.2 do Capı́tulo 5 da Parte 1, que quando referidos ao mesmo


sistema cartesiano os gráficos de y = f (x) e de sua inversa y = f −1 (x) , então elas se
relacionam por uma reflexão na diagonal y = x.
Logo uma reta tangente ao gráfico y = f (x) de coeficiente angular a = B/A 6= 0 se
transforma numa reta tangente ao gráfico refletido, mas agora de coeficiente angular
1
a
= A/B (já que os acréscimos na coordenada x e y que definem A e B ficam
invertidos quando refletimos na diagonal). Ilustro isso nas Figura a seguir:

0,8

0,6

0,4

0,2

0
0 0,2 0,4 0,6 0,8

-0,2 x

-0,4

Figura: Reflexão na diagonal de um gráfico e de sua reta tangente

Quero motivar com isso o seguinte fato:


Teorema 0.1. Seja y = f (x) derivável com f ′ (x) 6= 0 e com uma função inversa
f −1 (x) também derivável. Então:
′ 1
f −1 (x) = ′ −1 .
f (f (x))
Demonstração. Considero a composição entre f e g = f −1 , que resulta em uma
anular o efeito da outra:
(f ◦ f −1 )(x) ≡ x.
Então o Teorema 1.1 dá:

(f ◦ f −1 )′ (x) = f ′ (f −1 (x)) · (f −1 ) (x).
Mas por outro lado:
1 ≡ (f ◦ f −1 )′ (x)
221

1. DERIVADA DE Y = X 222

pois (f ◦ f −1 )(x) ≡ x. Asim que:



1 ≡ f ′ (f −1 (x)) · (f −1) (x),
de onde
′ 1
(f 1 ) (x) = .
f ′ (f −1 (x))



1. Derivada de y = x

Vejamos o que é a derivada
√ >0 de y = x de dois modos distintos, um pela definição
e outro lembrando que :R → R é a inversa de y = x2 : R>0 → R>0 .
>0

Pela definição temos:


√ √
√ ′ x+h− x
x (x) := lim
h→0 h
e para x > 0 e h com |h| suficientemente pequeno para que x + h > 0, escrevo:
√ √ √ √ √ √
x+h− x x+h− x x+h+ x
lim = lim ·√ √ .
h→0 h h→0 h x+h+ x
Agora uso que ( + △) · ( − △) = 2 − △2 , para obter que:
√ ′ x+h−x
x (x) = lim √ √ =
h→0 h · ( x + h + x)
1
= lim √ √ .
h→0 x+h+ x

E agora uso a continuidade de y = x (por ser inversa de função contı́nua definida
num intervalo) para fazer:
√ ′ 1 1
x (x) = lim √ √ = √ .
h→0 x+h+ x 2· x
Observe que
1
lim √ = +∞
xց0 2 · x

o que diz que o gráfico de y = x fica vertical na origem.
Agora quero comparar esse resultado com o que obtemos pelo Teorema 0.1 sobre
a derivada da inversa. √
Seja f : R>0 → R>0 dada por f (x) = x2 e sua inversa f −1 (x) = x. Como
f ′ (x) = 2x, então √ √
f ′ ( x) = 2 · x
e portanto pelo Teo 0.1:
√ ′ 1
x (x) = √ ,
2· x
como querı́amos.
CAPÍTULO 16. FUNÇÕES INVERSAS E SUAS DERIVADAS 223

2. Distância versus quadrado da distância


No Capı́tulo 11 usamos a função que dava o quadrado da distância desde um
ponto, ao invés da distância ela mesma, para evitar derivar a raı́z quadrada, que
aparece na definição de distância (euclidiana) entre dois pontos.
A Afirmação a seguir justifica isso:
Afirmação 2.1. Seja f : [a, b] → R, derivável, com f (x) > 0 ∀x ∈ [a, b].
Então f tem ponto de mı́nimo/máximo global em x ∈ [a, b] se e somente se f 2 (x)
tem tem ponto de mı́nimo/máximo global em x ∈ [a, b].
Demonstração.
Se a é tal que 0 < f (a) ≤ f (x) ∀x ∈ [a, b] então 0 < f 2 (a) ≤ f 2 (x), pois a função
y = z 2 é estritamente crecente em (0, +∞).
Se a é tal que 0 < f 2 (a) ≤ f 2 (x) ∀x ∈ [a, b] então
p p
0 < f 2 (a) ≤ f 2 (x),

pois a função y = z é estritamente crescente em (0, +∞), já que sua derivada é
1

2 z
> 0. Ou seja, 0 < f (a) ≤ f (x) ∀x ∈ [a, b].
Analogamente para o caso 0 < f (x) ≤ f (a) e para o caso do outro extremo b de
[a, b].
Se x é ponto do intervalo aberto (a, b) que é mı́nimo global de f então f ′ (x) = 0,

f (x) ≤ 0 num pequeno intervalo à esquerda de x e f ′ (x) ≥ 0 num pequeno intervalo
à direita de x. Mas então
(f 2 )′ (x) = 2 · f (x) · f ′ (x) = 0
e (f 2 )′ tem os mesmo sinais que f ′ próximos de x. Logo x é mı́nimo global de f 2 (x).
Reciprocamente, se x ∈ (a, b) é mı́nimo global de f 2 (x) então (f 2 )′ (x) = 0, com
(f 2 )′ ≤ 0 à esquerda de x e (f 2 )′ ≥ 0 à direita de x. Mas como
(f 2 )′ (x) = 2 · f (x) · f ′ (x) e f (x) > 0,
então f ′ (x) = 0 e os sinais de f ′ próximo a x são os mesmos de (f 2 )′ : concluo que x
é mı́nimo global de f (x).
Analogamente para ponto do intervalo aberto (a, b) que seja máximo global de f
ou f 2 . 

O Exercı́cio 6.10 usa de outro modo o que aprendemos na prova da Afirmação 2.1.
1 m −m
3. Derivada da “função”x n , de x n e de x n

Seja a função f (x) = xn . Se n é par, precisamos restringir f a um semi-eixo para


termos uma função inversa f −1 (uma raı́z n-ésima).
Com essa ressalva, considere g = f −1 a inversa de f (x) = xn . Ou seja g(f (x)) = x.
1
A notação usual para g(x) é g(x) = x n , feita de propósito a que valha
1 n
g(f (x)) = (xn ) n = x = x n .
1 M −M
3. DERIVADA DA “FUNÇÃO”X N , DE X N E DE X N 224
1
Afirmação 3.1. Considere a função x n , para n ∈ N, (com a ressalva acima). Então
para x 6= 0 vale que
1 ′ 1 1
(x n ) (x) = x n −1 .
n
Demonstração.
O Teorema 0.1 diz que para x 6= 0, combinado com a derivada de xn , dá:
1 ′ 1
(x n ) = 1 n−1 .
n · (x n )
1 k
De aı́ em diante basta fazer algumas manipulações (usando (x n )k = x n ):
1 ′ 1 1 1 n−1
xn = · n−1 = · x− n = .
n x n n

1 1−n 1 1
= · x n = · x n −1 .
n n


m
Podemos agora derivar funções do tipo x n com m, n ∈ N usando as regras da
composta e da inversa, pois
m 1
x n = (x n )m .
1
Então pelo Teorema 1.1 (a regra da composta) e o que já sabemos para x n :
1 m′ 1 m−1 1 1
(x n ) = m · (x n ) ·( · x n −1 ) =
n
m m−1 1 m m −1
= · x n · x n −1 = ·xn
n n
m
Para podermos derivar funções do tipo x− n com m, n ∈ N podemos escrever
−m m
x n = 1mn e usar o que sabemos de quocientes e de x n :
x
m
1 ′ − m x n −1 m m 2m
( m ) = n 2m = − · x n −1− n =
xn xn n
m −m −1
− ·x n .
n
α ′ α−1
Qual o sentido de dizermos que em √ geral se f (x) = x então f (x) = αx ?
E se α 6∈ Q? Por exemplo α = 2 ou α = π? Após darmos um sentido a essa
expressão (e precisaremos da função exponencial para isso), será que essa função é
derivável ? Será que sua derivada também é α · xα−1 ? Voltaremos...
CAPÍTULO 16. FUNÇÕES INVERSAS E SUAS DERIVADAS 225

4. Derivadas do arcoseno e do arcocosseno


É claro que o seno visto como função periódica sin : R → R ou mesmo visto em
sin : [0, 2π] → R não tem uma função inversa.
Mas sua restrição sin : (− π2 , π2 ) → (−1, 1) mostrada na Figura a seguir sim tem
função inversa ! De fato, nessa região (− π2 , π2 ) o seno é uma função injetora, pois sua
derivada sin′ (x) = cos(x) é sempre positiva em (− π2 , π2 ), logo sin(x) é estritamente
crescente e portanto uma função injetora.

0,5

0
-1,5 -1 -0,5 0 0,5 1 1,5
x
-0,5

-1

Figura: Restrição do seno ao intervalo ((− π2 , π2 ).

A inversa de sin : (− π2 , π2 ) → R é chamada de valor principal do arco seno ou


apenas arcoseno, no sentido de que dado sin(θ) em (−1, 1) ela diz de que arco θ ele
proveio, π2 < θ < π2 .
1
É denotada arcsin. Guardaremos o sı́mbolo sin(x)−1 para denotar sin(x) .

1,5

0,5

0
-1 -0,5 0 0,5 1
x
-0,5

-1

-1,5

Figura: Gráfico de arcoseno, domı́nio (−1, 1) e imagem (− π2 , π2 ).

Como explicado no Teorema que trata da inversa de funções contı́nuas, o arcoseno


e o arcocosseno são funções contı́nuas. Mas vamos assumir que seja derivável, para
calcularmos sua derivada.
Agora considere na Figura a seguir a restrição do cosseno ao intervalo [0.π].
4. DERIVADAS DO ARCOSENO E DO ARCOCOSSENO 226

0,5

0
0 0,5 1 1,5 2 2,5 3
x
-0,5

-1

É uma função estritamente decrescente, cuja inversa (também estritamente de-


crescente) é denotada arccos : [−1, 1] → [π, 0].
Afirmação 4.1.
i) A derivada de arcsin : (−1, 1) → (− π2 , π2 ) é
1
arcsin′ (x) = √ .
1 − x2
Para a > 0, a derivada de arcsin( xa ) : (−a, a) → (− π2 , π2 ) é:
x 1
arcsin′ ( ) = √ .
a a2 − x2
ii) A derivada de arccos : (−1, 1) → [π, 0] é
1
arccos′ (x) = − √ .
1 − x2
iii) arccos(x) = π2 − arcsin(x), ∀x ∈ [−1, 1].
Demonstração.

De i):

Pelo Teorema 0.1:


1
arcsin′ (x) = . ′
sin (arcsin(x))
Mas já sabemos que a derivada do seno é o cosseno, logo:
1
arcsin′ (x) = .
cos(arcsin(x))
Agora uso a relação trigonométrica
cos2 (arcsin(x)) + sin2 (arcsin(x)) ≡ 1
e
sin2 (arcsin(x)) = ( sin(arcsin(x) )2 = x2
para obter:
cos2 (arcsin(x)) = 1 − x2 ,
e como cos(arcsin(x)) > 0 quando arcsin(x) ∈ (− π2 , π2 ) então obtenho:

cos(arcsin(x)) = + 1 − x2
CAPÍTULO 16. FUNÇÕES INVERSAS E SUAS DERIVADAS 227

e portanto
1
arcsin′ (x) = √ ,
1 − x2
como querı́amos.
Quando tomo a > 0, então pela regra da derivada da composta:
x 1 1
arcsin′ ( ) = p · =
a 1 − ( xa )2 a
1 1 1
=√ p x 2
=√ .
a 2 1 − (a) a − x2
2

De ii):
Pelo Teorema 0.1:
1
arccos′ (x) = .
cos′ (arccos(x))
Mas já sabemos a derivada do cosseno, logo:
−1
arccos′ (x) = .
sin(arccos(x))
Exatamente como fizemos antes, a relação trigonométrica entre seno e cosseno e o
fato de que o seno restrito a [0, π] é ≥ 0, dão:
−1
arccos′ (x) = √ .
1 − x2

De iii):

Os itens i) e ii) já provados dão que:


arccos′ (x) = − arcsin′ (x), ∀x ∈ (−1, 1).
Portanto existe uma constante C ∈ R tal que:
arccos(x) = − arcsin(x) + C, ∀x ∈ (−1, 1).
Mas
π
= arccos(0) = − arcsin(0) + C = 0 + C,
2
o que nos diz que
π
C= .
2
Ademais também:
π π π
π = arccos(−1) = + = − arcsin(−1) + ,
2 2 2
bem como:
π π π
0 = arccos(1) = − + = − arcsin(1) + .
2 2 2

5. DERIVADA DO ARCOTANGENTE 228

O Exercı́cio 6.8 propõe comprovar geometricamente (qualitativamente ao menos)


que arccos(x) = − arcsin(x) + π2 .
1
Note agora que a função √1−x 2 para x ∈ (−1, 1) é sempre positiva, vale 1 na

origem e tem
1 1
lim √ = +∞, e lim √ = +∞.
xր1 1−x 2 xց1 1 − x2
Tudo isso se vê na figura abaixo, onde plotei o arcoseno e sua derivada, para
x ∈ [−0.95, 0.95] (não posso me aproximar demais de −1 ou de 1 se não o gráfico fica
muito alto !)

0
-0,8-0,4 0 0,4 0,8
x

-1

Figura: Gráfico de y = arcsin(x) (vermelho) e de sua derivada y = √ 1 (verde).


1−x2

Essa figura é tão parecida (qualitativamente) com a que já vimos no Capı́tulo
anterior da função y = tan(x) e sua derivada que resolvi plotá-las juntas, para que o
leitor possa fazer comparações:

0
-0,8-0,4 0 0,4 0,8
x

-1

Figura: y = tan(x) (vermelho), sua derivada (verde), y = arcsin(x)


(amarelo) e sua derivada (azul) restritas a (−0.9, 0.9).

5. Derivada do arcotangente
Se x ∈ (− π2 , π2 ) então
1
tan′ (x) = > 0,
cos2 (x)
CAPÍTULO 16. FUNÇÕES INVERSAS E SUAS DERIVADAS 229

o que diz que para x ∈ (− π2 , π2 ) a função y = tan(x) é estritamente crescente.


Logo é injetora e tem função inversa denotada:
π π
arctan : R → (− , ).
2 2
Afirmação 5.1.
1
arctan′ (x) = , ∀x ∈ R
1 + x2
e para a > 0 :
1 x 1
· arctan′ ( ) = 2 , ∀x ∈ R
a a a + x2
Demonstração.
Pelo Teorema 0.1 e pela derivada da função tan(x):
1
arctan′ (x) = =
tan′ (arctan(x))
1
= 1 =
( cos2 (arctan(x)) )
= cos2 (arctan(x)).
Agora arctan(x) é um arco/ângulo e portanto vale para ele a relação trigonométrica
básica:
sin2 (arctan(x)) + cos2 (arctan(x)) = 1
e daı́, dividindo por cos2 (arctan(x)) > 0, temos:
sin2 (arctan(x)) 1
2
+1= 2
cos (arctan(x)) cos (arctan(x))
ou seja
1
tan2 (arctan(x)) + 1 = ,
cos2 (arctan(x))
e como
tan2 (arctan(x)) = (tan(arctan(x)))2 = x2 ,
1
x2 + 1 =
cos2 (arctan(x))
quer dizer:
1
cos2 (arctan(x)) =
1 + x2
Logo
1
arctan′ (x) = .
1 + x2
Se a > 0 a derivada da composta dá:
x 1 1 1
arctan′ ( ) = x 2 · =a· 2 .
a 1 + (a) a a + x2

5. DERIVADA DO ARCOTANGENTE 230

1
0,5
0
-3 -2 -1 0 1 2 3
-0,5
x
-1

Figura: A função arcotangente (vermelho) e sua derivada


(verde) restritas a (−4, 4)

Exemplo:
Para completar essa Seção, vou mostra neste Exemplo como informação qualita-
tiva pode servir para dar informação quantitativa !
Considere
x x
y = F (x) = − 2 arctan( ).
2 2
A pergunta é: em que pontos F (x) se anula, além do x = 0 ? Ou pelo menos, como
dar uma aproximação dessas raı́zes ? Nem pensar em tentar resolver explicitamente
F (x) = 0 ...
Já inicialmente é bom observar que F (x) é uma função ı́mpar, F (−x) = −F (x).
Portanto vamos pensar no eixo x > 0 apenas, depois fica fácil o eixo x < 0.
Note que
1 1 1 1 4
F ′ (x) = − 2 · · x 2 = − 2
2 2 1 + (2) 2 x +4
e esta última função teve seu gráfico esboçado na Seção 4 do Capı́tulo 14.
Vimos lá naquela Seção que F ′ (x) se anula, no eixo x > 0, em x = 2, que F ′ (x) < 0
em (0, 2) e que F ′ (x) > 0 em (2, +∞).
Então, como F (0) = 0, concluo que y = F (x) < 0 em (0, 2), assume um mı́nimo
em x = 2 e depois começa a crescer.
Como
x π
lim arctan( ) =
x+∞ 2 2
temos
lim F (x) = +∞.
x+∞

Ou seja, como F (x) é contı́nua, tem que voltar a se anular em algum ponto à direita
de x = 2.
Só que, para x > 0,
x x x π
F (x) = − 2 arctan( ) > − 2 · .
2 2 2 2
CAPÍTULO 16. FUNÇÕES INVERSAS E SUAS DERIVADAS 231

Como a reta y = x2 − π corta o eixo x > 0 em x = 2π ∼ 6.3, concluo que F (x) se


anula1 em x ∈ (2, 6.3).
Pela propriedade ı́mpar, F (x) se anula em −x ∈ (−6.3, 2).
Note que:
1
lim F ′ (x) = lim F ′ (x) =
x+∞ x−∞ 2
ou seja que a inclinação tende a 1/2 quando |x| → ∞.
Como
x π
lim arctan( ) = −
x−∞ 2 2
vemos que o gráfico de y = F (x) se aproxima de
x
y = +π
2
quando x → −∞.
A figura a seguir ilustra F (x) em vermelho, F ′ (x) em verde, y = y = x2 + π em
azul e y = x2 − π em amarelo.

0
-10 -5 0 5 10
x

-4

-8

6. Exercı́cios
Exercı́cio 6.1. (resolvidos: iii, iv, v, xv.)

Derive usando regras de derivação de +, −, x, /, e a derivada da composta:
p
i) sin(x3 ), se sin(x3 ) > 0 ii) cos5 (x) + sin(x5 ),
1Com o método de Newton do Capı́tulo 18, começando com 6.3 obtive na quinta iteração x ∼
4.662244741
6. EXERCÍCIOS 232

x4 + x2 + 1
iii) sin3 (x3 ), iv) sin(x) cos(x), v) ,
3x4 + 4x2 + 1

vi) 1 − x2 , se |x| < 1, vii) sin(x3 ), viii) cos3 (x) + sin3 (x),

x7 − x2 − 1 x3 − x + 1
ix) , x) ,
x4 + 4x2 + 8 x4 − x3 + x2 − 1
2
xi) sin3 (x) − sin(x3 ), xii) , 0 < x,
x3
xiii) (sin(x) · cos2 (x))2 , xiv) (x + 3)100 , xv) (3x + 4)100 .
Exercı́cio 6.2. Determine o domı́nio de cada uma das quatro funções a seguir e em
que que pontos do domı́nio existe a derivada. Derive-as usando as regras de derivação
(produto, soma, composição, etc).

x 1
i) y = , ii) y = ,
x2 − 1 sin(x)
1
iii) y = tan(x) · sin(cos(x)), iv) y = x4 · x 4 .
Exercı́cio 6.3. No Capı́tulo 28 vamos definir
| f ′′ (x) |
κ(x) := 3
(1 + (f ′ (x))2 ) 2
como sendo a curvatura do gráfico de y = f (x) em cada ponto x.
Verifique que
i) κ(x) ≡ 0 para uma reta y = a · x + b e
ii) κ(x) ≡ 1r para a parte do cı́rculo x2 + y 2 = r 2 que fica no primeiro quadrante.
Exercı́cio 6.4. Suponha que você só conhece a reta tangente ao Cı́rculo como o
fizemos aqui neste curso de Cálculo, ou seja, como reta cujo coeficiente angular é
dado por uma derivada, etc.
Prove que essa reta tangente é ortogonal ao raio do Cı́rculo, ou seja, que coincide
com a definição do Ensino Médio (dica: basta considerar pontos do cı́rculo x2 +y 2 = 1
com coordenada y > 0).
Exercı́cio 6.5. Considere a função f : R>0 → [−1, 1] dada por f (x) = sin( x1 ).
i) derive-a pela regra da composta, ii) comprove que |f ′(x)| fica arbitrariamente
grande quando x tende a zero, iii) interprete geometricamente o resultado, sobre o
que acontece com o gráfico de f próximo à origem, iv) agora considere a função dada
por f (x) = x2 · sin( x1 ) (para x > 0). v) derive-a , vi) veja se o módulo da derivada
f ′ (x) fica arbitrariamente grande próximo à origem, ou não.
Exercı́cio 6.6. Considere a Figura a seguir, que dá o gráficos de f (x) = arctan(x)
1
(função inversa da tangente), de sua derivada f ′ (x) = 1+x 2 (assuma que sua derivada
CAPÍTULO 16. FUNÇÕES INVERSAS E SUAS DERIVADAS 233

é essa) e de sua segunda derivada f ′′ (x), restritas ao eixo positivo x > 0.

0,5

0
0 0,5 1 1,5 2 2,5 3 3,5
x
-0,5

1
Vemos que o gráfico de f ′ (x) = 1+x 2 tem um ponto de inflexão, ou seja, onde as

inclinações de suas tangentes tem um mı́nimo e depois vão aumentando, ficando cada
vez mais próximas de zero quando x >> 1. Dito de outro modo, um ponto onde a
segunda derivada f ′′ (x) = (f ′ (x)′ ) têm um mı́nimo.
Para encontrar onde é esse mı́nimo de f ′′ (x), calcule pela regra do quociente a
terceira derivada f ′′′ (x) e procure por seus zeros ! (Vão ser duas soluções, uma positiva
1
e outra negativa, pois o gráfico de f ′ (x) = 1+x 2 é simétrico em relação ao eixo dos y).

Exercı́cio 6.7. Considere a função g : (−1, 1) → R dada por


y
g(y) = , se y ∈ [0, 1),
1−y
y
g(y) = , se y ∈ (−1, 0].
1+y
(Chamo a variável de y pois foi assim que a vimos na Parte 1 do Curso). Já vimos
que g é uma tremenda expansão, pois a imagem do intervalo pela g é toda a reta R !
1
Prove que a derivada da g em y ∈ [0, 1) é (1−y)2 e que a derivada da g em y ∈ (−1, 0]
1
é de (1+y)2 . Chamamos essas derivadas de taxas de expansão.
Exercı́cio 6.8. Comprove geometricamente que:
π
arccos(x) = − arcsin(x) + , ∀x ∈ [−1, 1].
2
Para isso:
i) faça o gráfico qualitativamente correto do seno restrito a [− π2 , π2 ],
ii) reflita o gráfico de i) na diagonal para obter o de arcsin.
iii) reflita no eixo dos x o gráfico de ii) para obter o de − arcsin
iv) Translade o gráfico de iii) verticalmente por π2 para obter o de − arcsin + π2 .
v) reflita o gráfico de iv) na diagonal para obter um gráfico qualitativamente
correto do cosseno a [0, π].
1 1 1
Exercı́cio 6.9. Descreva de modo qualitativamente correto a curva x 2 + y 2 = a 2 ,
para a > 0 fixado e x, y ≥ 0.
Para isso mostre que:
1 1
i) y = y(x) = (a 2 −x 2 )2 é derivável para 0 < x ≤ a e tem y ′(x) ≤ 0 em 0 < x ≤ a.
ii) y ′(a) = 0, ou seja, o gráfico tangencia o eixo x em x = a.
1 1
iii) por simetria se obtém o mesmo tipo de fenômeno para x = x(x) = (a 2 − y 2 )2 .
6. EXERCÍCIOS 234

iv) a inclinação da curva no ponto ( a4 , a4 ) é −1.


v) sempre o gráfico y = y(x) tem concavidade para cima.
Exercı́cio 6.10. Se alguém pede para traçarmos qualitativamente o gráfico de y =
x6 − 6x4 + 9x2 pode parecer muito difı́cil.
Mas se notamos que y = x6 − 6x4 + 9x2 = (x3 − 3x)2 então o que aprendemos na
prova da Afirmação 2.1 torna a tarefa fácil, desde que saibamos o de y = x3 − 3x.
CAPı́TULO 17

Taxas relacionadas

Uma utilidade da regra da derivada da composta é a de permitir estabelecer de


modo quantitativamente exato como a variação de uma grandeza afeta a variação de
outra.

1. Como varia um ângulo


Vou considerar primeiro uma interessante aplicação da derivada do arcotangente,
que vimos no Capı́tulo anterior.
Um objeto tem posição P (t) = (x(t), y(t)) no plano em cada instante t. Ambas
coordenadas podem mudar com o tempo e suas velocidades em cada instante - suas
derivadas - são denotadas x′ (t) e y ′ (t) (que suponho existem).
Na origem alguém observa o objeto com uma câmera e o ângulo anti-horário que a
câmera faz com o eixo dos x será denotado θ(t). Que suponho é uma função derivável
de t.
Como mostra a figura, onde o vetor em preto dá a posição em cada instante e o
vetor em vermelho indica a velocidade em cada instante:

A questão é: como muda a câmera quando o objeto muda de posição ? Ou seja,
como x′ (t) e y ′ (t) e a posição do objeto em cada instante afetam θ′ (t) ?
Supondo para simplificar que
π
x(t) > 0, y(y) ≥ 0 e 0 ≤ θ(t) < ∀t,
2
então:
y(t)
θ(t) = arctan( ).
x(t)
Derivo em t, pela regra da composta:
y(t) 1 y(t) ′
θ′ (t) = arctan′ ( )= y(t)
·( ) (t) =
x(t) 1 + ( x(t) ) 2 x(t)
235
2. COMO VARIA UMA DISTÂNCIA 236

y ′ (t) · x(t) − y(t) · x′ (t)


= .
x(t)2 + y(t)2
Essa fórmula dá várias informações, que servem para resolver vários problemas
práticos:
• se o objeto se move apenas verticalmente, então x ≡ x > 0, x′ (t) ≡ 0 e
quando está numa altura y(t) num instante t:
y ′(t) · x
θ′ (t) = ,
x2 + y(t)2
o que se simplifica ainda mais quando y(t) = 0 para:
y ′ (t)
θ′ (t) = .
x
• se o objeto se move apenas horizontalmente, então y ≡ y ≥ 0, y ′ (t) ≡ 0 e
quando está numa posição x(t) num instante t:


−y · x′ (t)
θ (t) = .
x(t)2 + y 2
• quando o objeto se move radialmente temos:
y ′ (t) y(t)

=
x (t) x(t)
e então:
θ′ (t) = 0.
• quando objeto se move num cı́rculo de raio r > 0 centrado na origem então:
y ′ (t) · x(t) − y(t) · x′ (t)
θ′ (t) = .
r2
Há vários modos de descrever esse movimento, por exemplo com:
(x(t), y(t)) = (r · cos(k · t) , r · sin(k · t)), k∈R
pois claramente x2 (t)+y 2(t) ≡ r 2 . Então nesse caso teremos, usando de novo
a regra da derivada da composta:
y ′ (t) · x(t) − y(t) · x′ (t)
θ′ (t) = = k, ∀t
r2

2. Como varia uma distância


Imagine dois objetos cujas posições P1 = (x1 (t), y1(t)) e P2 = (x2 (t), y2(t)) variam
ao longo de segmentos de retas c1 e c2 que se encontram em ângulo α (constante)
num ponto I, como na figura a seguir:
CAPÍTULO 17. TAXAS RELACIONADAS 237

P1

c1

α
I

c2

P2

A questão é: como variam as distâncias relativas umas às outras ?


Denoto d(t) a distância entre P1 e P2 . Temos pela lei dos cossenos (Afirmação
3.1, na próxima Seção):

d2 (t) = c21 (t) + c22 (t) − c1 (t) · c2 (t) cos(α).


Note que se α = π2 (ângulo reto) o tamanho d(t) é o que se espera por Pitágoras. Se
0 < α < π2 (ângulo agudo) então d(t) fica menor que o que se espera por Pitágoras,
mas se π2 < α < π (ângulo obtuso) então d(t) fica maior que o que se espera por
Pitágoras.
Então:
2 · d(t) · d′ (t) = 2 · c1 (t) · c′1 (t) + 2 · c2 (t) · c′2 (t) − [c′1 (t) · c2 (t) + c1 (t) · c′2 (t)] · cos(α),
ou seja:
cos(α)
c1 (t) · c′1 (t) + c2 (t) · c′2 (t) − · [c′1 (t) · c2 (t) + c1 (t) · c′2 (t)]
d′ (t) = 2
.
d(t)
Essa fórmula se presta para resolver vários problemas práticos, mesmo em casos
bem particulares:
• Se
π
c2 (t) ≡ C e α = .
2

Então c2 (t) ≡ 0 e cos(α) = 0 e obtemos da expressão acima:
2 · d(t) · d′ (t) = 2 · c1 (t) · c′1 (t),
ou seja,
c1 (t) ′
d′ (t) =
· c (t).
d(t) 1
• quando uma escada desliza ao longo de uma parede então d(t) ≡ d > 0 é o
tamanho da escada e α = π2 . Então a expressão acima vira:
0 = c1 (t) · c′1 (t) + c2 (t) · c′2 (t)
que diz como o aumento/diminuição da posição de um extremo repercute no
outro extremo da escada.
3. LEI DOS COSSENOS E PRODUTO ESCALAR DE VETORES 238

3. Lei dos cossenos e produto escalar de vetores


Falta explicar de onde surge a:
Afirmação 3.1. (Lei dos cossenos)
Considere um triângulo △ABC com ângulo α em A.
Então
BC 2 = AB 2 + AC 2 − 2 · AB · AC · cos(α).
Demonstração.
Como para ângulo reto a fórmula é o Pitágoras, o correto seria considerar ângulos
agudos e obtusos. Por brevidade considero apenas o caso de ângulo agudo α e deixo
o caso de obtuso como exercı́cio para o leitor.
Escolho H no segmento AC tal que BH seja ortogonal a AC em H, como mostra
a figura:

α
A C
H
Então Pitágoras se aplica em dois triângulos retângulos:
AB 2 = BH 2 + AH 2 e BC 2 = BH 2 + CH 2 .
De onde:
BC 2 − AB 2 = CH 2 − AH 2 .
Mas
CH = CA − AH
e portanto:
BC 2 − AB 2 = (CA2 − 2 · CA · AH + AH 2 ) − AH 2 = CA2 − 2 · CA · AH,
ou seja:
BC 2 = AB 2 + AC 2 − 2 · AC · AH.
Para terminar note que:
AH = AB · cos(α).


A lei dos cossenos embasa as propriedades do produto escalar de vetores.


Definição 3.1. Dados vetores v1 = (x1 , y1 ) e v2 = (x2 , y2) defino seu produto escalar
como:
v1 · v2 = x1 · x2 + y1 · y2 .
CAPÍTULO 17. TAXAS RELACIONADAS 239

Observação:
Quando usar · entre vetores se trata desse produto. Mas. quando fizer, para
λ ∈ R, o produto λ · v trata-se então de multiplicar cada coordenada de v por λ.
Afirmação 3.2.
i):

v1 · v2 = v2 · v1 , v1 · v1 = ||v1 ||2 , e v1 · (v2 + v3 ) = v1 · v2 + v1 · v3 .


ii) Dados vetores v1 = (x1 , y1) e v2 = (x2 , y2), então
v1 · v2 = ||v1|| · ||v2 || · cos(θ)
onde θ é o ângulo orientado de v1 para v2 (como cos(−θ) = cos(θ) dá o mesmo que
considerar o ângulo de v2 para v1 )
iii) Se ||v2 || = 1 então
(v1 · v2 ) · v2
é o vetor que corresponde à projeção ortogonal de v1 no eixo orientado gerado por v2 .
Demonstração.
O item i) é imediato das definições de módulo, produto escalar e de soma de
vetores.

De ii):
O item i) aplicado ao vetor diferença v1 − v2 :
||v1 − v2 ||2 = (v1 − v2 ) · (v1 − v2 ) = v1 · v1 + v2 · v2 − 2 · v1 · v2 =
= ||v1||2 + ||v2 ||2 − 2 · v1 · v2 ,
ou seja:
v1 · v2 = ||v1 − v2 ||2 − ||v1 ||2 − ||v2 ||2 .
Mas como mostra a figura a seguir posso aplicar a Lei dos cossenos para ter o
módulo de v1 − v2 :

v1 − v2
v2

v1

||v1 − v2 ||2 = ||v1 ||2 + ||v2 ||2 − 2 · ||v1 || cot ||v2 || · cos(θ),
de onde sai ii).
De iii):
O item ii) aplicado a um vetor unitário v2 dá
v1 · v2 = ||v1 || · cos(θ).
3. LEI DOS COSSENOS E PRODUTO ESCALAR DE VETORES 240

Então
(v1 · v2 ) · v2
está no eixo gerado por v2 e tem módulo:

||v1 || · | cos(θ)|.

Para comprovar que (v1 · v2 ) · v2 é realmente a projeção ortogonal de v1 sobre o eixo


gerado por v2 , podemos fazer uma conta:

v2 · [v1 − (v1 · v2 ) · v2 ] = v2 · v1 − (v1 · v2 ) · v2 · v2 = v2 · v1 − v1 · v2 = 0

o que diz pelo item ii) que v2 e v1 − (v1 · v2 ) · v2 são ortogonais.


Ilustro a seguir:

v1 − (v1.v2).v2
(v1.v2) . v2

v2 θ

v1

3.1. Uma interpretação vetorial da Seção 1. A fórmula


y ′(t) · x(t) − y(t) · x′ (t)
θ′ (t) =
x(t)2 + y(t)2
que demos na Seção 1 deste Capı́tulo admite uma interpretação vetorial importante,
que será retomada na Seção 5 do Capı́tulo 39.
Considero o vetor velocidade V := (x′ (t), y ′ (t)) e o vetor unitário
(−y(t), x(t))
N := p ,
x(t)2 + y(t)2

que é ortogonal
p ao vetor posição P := (x(t), y(t)). O módulo do vetor posição é
||P || := x(t)2 + y(t)2 .
O produto escalar de vetores:
(−y(t), x(t)) y ′ (t) · x(t) − y(t) · x′ (t)
V · N = (x′ (t), y ′(t)) · p := p
x(t)2 + y(t)2 x(t)2 + y(t)2

dá a projeção do vetor V := (x′ (t), y ′(t)) na direção do vetor unitário N (item iii) da
Afirmação 3.2). Veja a figura a seguir:
CAPÍTULO 17. TAXAS RELACIONADAS 241

N
V

E podemos então escrever na linguagem vetorial:


1
θ′ (t) = ·V ·N =
||P ||
y ′ (t) · x(t) − y(t) · x′ (t)
= .
x(t)2 + y(t)2
4. Exercı́cios
Exercı́cio 4.1. Considere um paralepı́pedo reto (ou seja, um objeto com a forma de
um tijolo maciço), cuja largura x(t), profundidade 2x(t) e altura y(t) mudam com o
tempo t.
Suponha que, em um instante t0 , sua altura é 1 cm e aumenta na taxa de 7 cm/s
e sua largura é 4 cm e decresce na taxa de −1 cm/s.
Qual a taxa de variação do Volume no instante t0 ? O Volume está aumentando
ou diminuindo em t0 ?
CAPı́TULO 18

O Método de aproximação de Newton

No Exercı́cio 9.11 do Capı́tulo 6 vimos que o polinômio


y = x5 − 2x4 + x3 + x2 + 1
tem uma raı́z no intervalo [−1, 1]. Mas para isso de usa o Teorema do Valor Inter-
mediário, que não diz quanto é a raı́z, apenas que ela existe.
Imagine quantas vezes Newton se viu defrontado com equações como essa, além
de outras não-polinomiais,1 por exemplo:
cos(x) + x · sin(x) − 1 = 0,
e certamente ele precisava ter informação sobre essas Raı́zes.
A idéia do método é bastante geométrica. Se queremos determinar uma raı́z de
f (x) = 0, trata-se de:
• escolher um ponto no eixo x, chamado de x0 , tal que f ′ (x0 ) 6= 0.
• determinar a reta tangente r0 ao gráfico de y = f (x) em (x0 , f (x0 ))
• intersectar r0 com o eixo dos x, chamando essa intersecção de x1
• recomeçar o processo a partir do ponto obtido.

Afirmação 0.1. O x1 obtido pelo método é da forma:


f (x0 )
x1 = x0 − .
f ′ (x0 )
Demonstração.
A reta tangente r0 ao gráfico de y = f (x) em (x0 , f (x0 )) tem equação:
y = f ′ (x0 ) · x + (f (x0 ) − f ′ (x0 ) · x0 ).
Intersectá-la com y = 0 dá:
f ′ (x0 ) · x0 − f (x0 )
x= =
f ′ (x0 )
f (x0 )
= x0 − .
f ′ (x0 )


1Como salienta S. Chandrasekhar na página 142 do seu livro Newton’s Principia for the common
reader, Oxford University Press , 1995.
243
244

Se a tangente num ponto (x, f (x)) do gráfico for uma reta horizontal então
terı́amos que resolver a equação:
f (x) = f (x),
que é tão difı́l como o problema original em geral. Ou seja, o método pode parar se
f ′ (x) = 0.

Exemplos:
• Para a raı́z de
y = x5 − 2x4 + x3 + x2 + 1
em [−1, 1] começo com
x0 := 1
e obtenho
x1 = 0.

Mas f (0) = 0 e páro.
Nova tentativa, partindo agora de
x0 := 1/2,
obtenho
x1 := −0.7058823529, x2 := −0.8206076715,
x3 := −0.7982163995, x4 := −0.7970632182, x5 := −0.7970602776,
e a partir daı́ a calculadora não muda mais o resultado. Então essa é a
aproximação buscada da raı́z.
A Figura a seguir indica como é o gráfico do polinômio.

0
-1 -0,5 0 0,5 1
x

-1

-2

• Agora quero uma raı́z de cos(x)+x·sin(x)−1 = 0 no intervalo [0, π] e começo


com x0 = 3.14.
Então:
x1 := 2.504649576, x2 := 2.348555437,
x3 := 2.331341479, x4 := 2.331122406, x5 := 2.331122370
a partir daı́ a calculadora passa desse valor para
x6 := 2.331122371
CAPÍTULO 18. O MÉTODO DE APROXIMAÇÃO DE NEWTON 245

e depois volta para o x5 , sucessivamente.

0,5
x
0 0,5 1 1,5 2 2,5 3
0

-0,5

-1

-1,5

-2

y = cos(x) + x · sin(x) − 1, x ∈ [0, π].


CAPı́TULO 19

O Princı́pio de Fermat e a refração da luz

1. Princı́pio de Fermat
Suponhamos dois pontos P1 = (x1 , y 1 ) e P2 = (x2 , y 2 ) com coordenadas y > 0.
O problema é: Encontrar o ponto P = (x, 0) no eixo dos x que minimiza a soma
das distâncias P P1 + P P2 .
Não é uma perda de generalidade muito grande supôr que P1 = (0, 1) (basta
escolher sistema de coordenadas adequado).
Chamemos o ângulo 1) formado em P pelo eixo dos x e a reta P P1 de ângulo de
incidência; e de ângulo refletido o ângulo formado pelo eixo dos x e a reta P P2 .
Afirmação 1.1. (Princı́pio de Fermat)
• i) o ponto no eixo dos x que minimiza a soma de distâncias a P1 := (0, 1) e
a P2 := (x2 , y 2 ), com y 2 > 0, é
x
P = (x, 0) = ( 2 , 0).
1 + y2
• ii) os ângulos de incidência e refletido formados nesse P são iguais.

2,5

1,5

0,5

0
0 0,5 1 1,5 2 2,5 3
x

Figura: Três exemplos do princı́pio de Fermat, com P1 = (0, 1)


P2 : (3, 1), (3, 2), (3, 3) e P : ( 32 , 0), (1, 0), ( 34 , 0) respectivamente.
Demonstração.
Do Item i):
Queremos encontrar o ponto P = (x, 0) no eixo dos x que minimiza a função:
p q
d(x) := (x − 0)2 + (0 − 1)2 + (x − x2 )2 + (0 − y 2 )2 =
1convexo, ou seja, 0 ≤ θ ≤ π, e não-orientado, ou seja, não distingo entre ângulos horários e
anti-horários.
247
1. PRINCÍPIO DE FERMAT 248
√ q
= x2 + 1 + (x − x2 )2 + y 22 .
Queremos usar o critério da segunda derivada (Afirmação 2.1 do Capı́tulo 10)
para determinar o mı́nimo de d(x).
Para isso precisamos calcular d ′(x), o que ainda não sabemos fazer.
Então, adiantando o que aprenderemos sobre derivadas de funções compostas e
da raı́z quadrada, Afirmo que:
x x − x2
d ′ (x) = √ +q =
2
x +1 (x − x2 )2 + y 2 2
q √
x · (x − x2 )2 + y 22 + (x − x2 ) · x2 + 1
= √ q ,
x2 + 1 · (x − x2 )2 + y 22
e claramente:
q √
d ′ (x) = 0 ⇔ x· (x − x2 )2 + y 22 + (x − x2 ) · x2 + 1 = 0.
Ao invés de resolver diretamente:
q √
x · (x − x2 )2 + y 22 = (x2 − x) · x2 + 1,
elevo ambos os lados ao quadrado, obtendo:
x2 · [(x − x2 )2 + y 22 ] = (x2 − x)2 · (x2 + 1),
o que equivale, após simplificações, a resolver:
(y 22 − 1) x2 + 2x2 x − x22 = 0.
Aqui há dois casos a considerar (dos quais daremos o significado geométrico a seguir):
Caso y 22 − 1 = 0, ou seja, y 2 = ±1, então a solução buscada é
x
P = (x, 0) = ( 2 , 0).
2
2
Caso y 2 − 1 6= 0, então temos uma equação quadrática em x, cujas soluções são:
x2 x2
e .
1 + y2 1 − y2
x
Note que o ponto Q := ( 1−y2 , 0) é colinear com (0, 1) e (x2 , y 2 ) (basta calcular os
2
coeficientes angulares das retas por dois deles). Então essa solução não nos interessa.
Porém a solução
x
P = (x, 0) = ( 2 , 0)
1 + y2
x
é interessante. Note que se y2 = 1 esse ponto se reduz a P = ( 22 , 0), ou seja, coincide
com a solução obtida no caso y 22 − 1 = 0.
x2 x2
Temos d ′ ( 1+y ) = 0 e agora precisarı́amos ver que d ′′ ( 1+y ) > 0, para termos um
2 2
mı́nimo de d(x).
A segunda derivada d ′′ (x) existe, como veremos nos Capı́tulos seguintes sobre
regras de derivação.
CAPÍTULO 19. O PRINCÍPIO DE FERMAT E A REFRAÇÃO DA LUZ 249

O cálculo de d ′′ (x) é tedioso e ainda mais tedioso2 é obter:


x2 (1 + y 2 )4
d ′′ ( )= q ,
1 + y2 y 2 (x22 + 1 + 2y 2 + y 22 )3
x
e vemos que d ′′( 1+y2 ) é positivo se y 2 > 0.
2
Está provado que o ponto minimiza a soma de distâncias.

Do Item ii):
Calculo o coeficiente angular da reta P P1 :

1−0 (1 + y 2 )
a := x2 = − .
0 − 1+y x2
2

Agora calculo o coeficiente angular da reta P P2 :

y2 − 0 1 + y2
a′ := x2 = ,
x2 − 1+y 2
x2
logo a′ = −a, ou seja, formam o mesmo ângulo (não-orientado) com a reta vertical.
Portanto também há igualdade de ângulos formados em P com a horizontal.


2. Refração, distâncias ponderadas e Lei de Snell


Na Seção anterior buscamos minimizar a soma das distâncias
P P1 + P P2 ,
onde P1 , P2 estão no semi-plano superior e P no eixo dos x
Agora imaginemos um problema um pouco mais geral.

Suponha que no semiplano superior nos movimentamos com uma velocidade con-
stante v1 enquanto no semiplano inferir nos movimentamos com uma velocidade con-
stante v2 . E que queremos sair de P1 no semiplano superior, atingir P no eixo dos x
e daı́, no semiplano-inferior, ir até P2 , fazendo isso no menor tempo possı́vel. Como
escolher P ?

Esse problema está ainda relacionado com o princı́pio de Fermat, que em geral não
é simplesmente de minimar distância entre dois pontos, mas de minimizar o tempo
gasto para ir de um a outro ponto.
Na prática é o problema do salva-vidas, que, estando em P1 , tem correr pela
areia (com velocidade v1 ) e escolher o ponto P na praia de onde sair nadando (com
velocidade v2 < v1 ) até chegar em algum banhista P2 . Veja Exercı́cio 3.1 abaixo.
2É útil para essas contas tediosas usar algum programa como o Maple.
2. REFRAÇÃO, DISTÂNCIAS PONDERADAS E LEI DE SNELL 250

Claro que se vv21 = 1, a solução é seguir a reta que liga P1 a P2 . E se vv12 << 1,
o ponto P ficará cada vez mais próximo da projeção vertical de P2 no eixo dos x.
Porém a resposta não é tão clara se vv21 ∼ 1.
Como distância é o mesmo que velocidade multiplicada pelo tempo, podemos
pensar que no semiplano superior e inferior as medidas de distância são diferentes.
Como se tivéssemos diferentes réguas para medir distância: um certo trecho que mede
d no semiplano superior (onde sou mais rápido) dever ser considerado como medindo
k · d > d no semiplano-inferior, onde sou mais lento.
Podemos então reformular o problema do seguinte modo:
Como minimizar a soma das distâncias ponderadas
d1,k (x) := P P1 + k · P P2 ?
(onde P1 , P2 estão em semi-planos diferentes e P no eixo dos x)
Isso é o que acontece quando a luz passa de um meio para outro. Por exemplo, a
razão entre velocidade da luz no ar (v1 ) e na água (v2 ) é da ordem de
v2 1
= ,
v1 1.33
ou seja, devemos usar a soma de distâncias ponderadas 3:
d1,1.33 (x) := P P1 + 1.33 · P P2,
(onde P1 está no ar e P2 na água).
Suponha que P1 = (0, 1) e que por exemplo
P2 = (x2 , −1), x2 > 0.
Imitando o que fizemos na Seção anterior, vamos querer derivar d1,k (x) e saber onde
d1,k ′ (x) = 0.
Agora, derivando obtemos:
x (x − x2 )
d1,k ′ (x) = √ +k p =
+1 x2 (x − x2 )2 + 1
p √
x · (x − x2 )2 + 1 + k x2 + 1 · (x − x2 )
= √ p .
x2 + 1 · (x − x2 )2 + 1
Como
x (x − x2 )
d1,k ′′ (x) = ( √ )′ + (k p )′ =
2
x +1 2
(x − x2 ) + 1
1 k
2 3/2
+ 2 > 0,
(x + 1) (x2 − 2x2 x + x2 + 1)3/2
a solução de d1,k ′ (x) = 0 será um ponto de mı́nimo de d1,k .
Mas
p √
d1,k ′ (x) = 0 ⇔ x · (x − x2 )2 + 1 = k x2 + 1 · (x2 − x)
3O chamado optical path length- OPL é definido como o produto da distância usual pelo ı́ndice
de refração - suposto constante - do meio onde a luz se propaga. Então no nosso caso d1,1.33 (x) =
OPL( ar ) + OPL( água )
CAPÍTULO 19. O PRINCÍPIO DE FERMAT E A REFRAÇÃO DA LUZ 251

e elevando ao quadrado ambos os lados, obtenho:


x2 ( (x − x2 )2 + 1 ) = k 2 (x2 + 1) (x2 − x)2 ,
ou seja, temos que resolver uma equação de grau 4:
(1 − k 2 ) x4 + (−2x2 + 2k 2 x2 ) x3 + (x22 + 1 − k 2 x22 − k 2 ) x2 + 2k 2 x2 x − k 2 x22 = 0.
Claro que se k = 1 (ou seja, d1,1 (x) é a soma de distâncias usuais), a equação
acima vira uma equação quadrática:
x
2x2 x − x2 = 0 ⇔ x = 2 .
2
x2
Logo P = ( 2 , 0) está na reta ligando P1 e P2 .
Mas se k 6= 1 temos uma verdadeira equação de grau 4.
Resovi fazer três exemplos, com o k = 1.33 (ı́ndice de refração da água) onde
sempre P1 = (0, 1), mas P2 assume três valores
(2, −1), (3, −1), (4, −1).
Nesses três casos o Maple resolve as equações de grau 4 acima4, dando em cada
caso um par de soluções complexas, uma solução real negativa e uma real positiva.
Listo as soluções reais positivas de cada um dos três casos:
se P2 = (2, −1), P = (1.268409214, 0),
se P2 = (3, −1), P = (2.078744326, 0),
se P2 = (4, −1), P = (2.983414222, 0).
A Figura a seguir representa as linhas quebradas ligando P1 a P e daı́ passando
por P2 , em cada um dos três casos, com k = 1.33:

x
0 1 2 3 4
0

-1

-2

-3

A figura a seguir dá os gráficos das d1,1.33 para


P2 = (2, −1), (3, −1), (4, −1).
4Pois existe a fórmula de Tartaglia para equações de grau 4.
2. REFRAÇÃO, DISTÂNCIAS PONDERADAS E LEI DE SNELL 252

6,5

5,5

4,5

3,5

0 1 2 3 4
x

Gráficos de y = d1,1.33 (x) para três escolhas de P2

Voltando ao que obtivemos como derivada:


p √
d1,k ′ (x) = 0 ⇔ x · (x − x2 )2 + 1 = k x2 + 1 · (x2 − x),
note que essa última expressão equivale a:
x (x − x)
√ =kp 2 .
2
x +1 (x − x2 )2 + 1
Agora note que
x
sin(α) = √
x2+1
onde α é o ângulo em P = (x, 0) do triângulo
∆ P P1 (x, 1).
E veja que
(x − x)
sin(β) = p 2
(x − x2 )2 + 1
onde β é o ângulo em P = (x, 0) do triângulo
∆ P P2 (x, −1).
Essa é a lei de refração de Snell :
sin(α) = k · sin(β).
Para uso posterior, podemos reescrever a lei de Snell assim:
v1
sin(α) = ,
v2
ou seja
sin(α) sin(β)
= .
v1 v2
CAPÍTULO 19. O PRINCÍPIO DE FERMAT E A REFRAÇÃO DA LUZ 253

Para terminar, é natural nos perguntarmos que acontece com a trajetória da luz
ao viajar por um meio com ı́ndice de refração variável. Qual o formato da trajetória
da luz, qual a sua equação ?
A resposta a esse tipo de pergunta depende de mais teoria matemática, por ex-
emplo do Cálculo de Variações.
3. Exercı́cios
Exercı́cio 3.1. (O Problema do salva-vidas)

Estando no ponto (8, 0), na areia da praia, o salva-vidas tem que sair correndo
para salvar alguém que se afoga no ponto B = (0, 5), dentro do mar. Veja a Figura.

Suponha que a velocidade do salva-vidas na praia é v1 m/s e na água é v2 < v1 ,


com razão:
v2
k := < 1.
v1
A questão é a seguinte: para que ele chegue o mais rápido possı́vel, até que ponto
(x, 0) com x ∈ [0, 8] ele deve correr pela praia, para daı́ então ir em linha reta nadando
até B ?
Na solução a coordenada x do ponto buscado será função de k, ou seja, x(k).
Também mostre que:
i) se k verifica k 2 · (k 2 − 1) < 0 então sair já de (8, 0) nadando não é a melhor
estratégia para o salva-vidas.
ii) mostre que limk→0 x(k) = 0. Ou seja, para valores de k muito pequenos o
melhor é correr pela areia até quase a origem e dali sair nadando em ângulo reto.
iii) Para um salva-vidas que corresse como Usain Bolt e nadasse como César Cielo
terı́amos k ∼ 0.22. Mas se nadasse como Cielo e corresse como uma pessoa normal,
então5 k ∼ 0.55.
Confirme que nesses dois casos
x(k) = x(0.22) ∼ 1.12 e x(k) = x(0.55) ∼ 3.34.

5Esses valores de k foram calculados pelo estudante Rafael Kuch, a quem agradeço
CAPı́TULO 20

As Cônicas e suas propriedades refletivas

1. Distância até uma parábola


Começo este Capı́tulo considerando o seguinte problema: dada uma parábola
y = C · x2 , com C > 0 fixado, e dado um ponto (0, a) no eixo positivo dos y, qual a
distância mı́nima entre ele e os pontos do gráfico da parábola ? Já o caso C = 1 é
interessante:

Afirmação 1.1. Seja o ponto (0, a) do eixo dos y com a > 0 e seja da (x) a distância
entre esse ponto e os pontos (x, x2 ) do gráfico da parábola y = x2 .
• i) se a > 21 então da (x) tem √
um máximo local em x = 0 e dois pontos de
2a−1
mı́nimo absoluto em x = ± √2 .
• ii) se a ≤ 12 então da (x) tem apenas um ponto de mı́nimo absoluto, em x = 0.
Ademais, se a = 14 então d 1 (x) = x2 + 14 .
4

A Figura a seguir ilustra a Afirmação: em vermelho y = d 3 (x), em verde y =


4
d 1 (x), em amarelo y = d 1 (x), em azul y = d 1 (x) e em lilás y = d 1 (x).
2 3 4 9

1,4

1,2

0,8

0,6

0,4

0,2

-1 -0,5 0 0,5 1
x

Veremos na próxima Seção 2, Definição 2.1, que


1
(0, a) = (0, )
4
2 1
é o foco da parábola y = x e que y = − 4 é a sua reta diretriz.
Demonstração.
255
1. DISTÂNCIA ATÉ UMA PARÁBOLA 256

Temos
p p
da (x) := (x − 0)2 + (x2 − a)2 = x2 + (x2 − a)2 ,
cujo domı́nio são todos os Reais.
Então máximos/mı́nimos são detectados por
x · (2x2 + 1 − 2a)
d′a (x) = p = 0.
x2 + (x2 − a)2
Ou seja, d′a (x) = 0 em

2a−1
• i) x = 0 e em mais dois pontos x = ± √
2
, desde que 2a − 1 > 0
• ii) apenas em x = 0, se 2a − 1 ≤ 0.
Podemos usar o Critério da primeira derivada para detectar máximos/mı́nimos
locais. Como claramente
lim da (x) = lim da (x) + ∞
x→+∞ x→−∞

os mı́nimos locais serão também globais.


No caso i),

2a − 1
d′a (x) < 0 se 0 < x < √
2
e

2a − 1
d′a (x) > 0 se − √ < x < 0.
2
o que diz que x = 0 é ponto de máximo local de da (x).
Ainda no caso i),

′ 2a − 1
da (x) > 0 se √ <x
2
e

′ 2a − 1
da (x) < 0 se x < − √ ,
2

o que diz que x = ± 2a−1

2
são pontos de mı́nimo local da da (x).
Já no caso ii), temos 2x2 + 1 − 2a ≥ 0 e o sinal de d′a (x) é o mesmo sinal de x:
d′a (x) > 0 se 0 < x
e
d′a (x) < 0 se x < 0,
o que diz que x = 0 é ponto de mı́nimo local.

CAPÍTULO 20. AS CÔNICAS E SUAS PROPRIEDADES REFLETIVAS 257

2. Definição unificada das cônicas


No colégio se insiste em apresentar cada cônica separadamente, sem que se dê
uma definição unificada.
A Definição 2.1 a seguir englobará todas as cônicas, menos uma, o Cı́rculo. Mas
veremos em seguida que a Definição 2.1 compreende a Definição 2.3, a qual se estende
naturalmente ao Cı́rculo.
Lembre que a distância de um ponto P a uma reta r, denotada P r a seguir, é a
distância do ponto P ao pé da perpendicular a r traçada desde P .
Definição 2.1. Fixe uma reta r e um ponto F ∈ / r. Uma cônica é o lugar geométrico
no plano dos pontos P cuja distância P F está numa razão constante para a distância
P r. Ou seja:
PF
= e, e > 0.
Pr
A grandeza e será chamada de excentricidade da cônica, F , de foco e r, de diretriz.

Afirmação 2.1. Considere uma cônica de foco F , diretriz r e excentricidade e. Então


existe um sistema cartesiano de coordenadas em que
• a origem (0, 0) pertence à conica,
• a diretriz vira a reta vertical x = −ρ, com ρ > 0,
• o foco é F = (eρ, 0)
• os pontos P = (x, y) da cônica satisfazem a equação:
(1 − e2 ) · x2 − 2e(1 + e)ρ · x + y 2 = 0.
Ademais, se e = 1 a equação vira:
1
x= · y2

assim como o foco vira F = (ρ, 0) e a diretriz, x = −ρ.
Se e < 1 , a equação geral vira
x2 2 y2
− · x + = 0,
a2 a b2
onde
eρ p
a := > 0 e b := a2 · (1 − e2 ) > 0.
1−e
Se e > 1, a equação geral vira:
x2 2 y2
+ · x − = 0,
a2 a b2
onde
eρ p
a := >0 e b := a2 (e2 − 1) > 0.
e−1
2. DEFINIÇÃO UNIFICADA DAS CÔNICAS 258

Definição 2.2. A cônica


1
x= · y 2,

do caso e = 1 da Afirmação 2.1, é chamada parábola.
• Ela tem óbvia simetria no eixo dos y e o eixo x é chamado de eixo da parábola.
• Um reta vertical pelo foco F = (ρ, 0) intersecta a parábola em dois pontos
(ρ, ±2ρ). A distância de F a cada um deles, que é 2ρ, é chamada semi-latus
rectum 1 da parábola.
• Num novo sistema cartesiano (x, y) em que o vértice P0 está em (x, y) = (h, k)
e o foco está na reta y = k a parábola
y 2 = 4ρx
se escreve como:
(y − k)2 = 4ρ(x − h)
que expandido dá:
y 2 − 2ky − 4ρx + k 2 + 4h = a1 y 2 + a2 y + a3 x + a4 = 0.
Em Exercı́cios pode se pedir para, a partir de uma equação do tipo:
a1 y 2 + a2 y + a3 x + a4 = 0
determinar a parábola, com o vértice, o foco e a diretriz.
Também o papel de x e y pode estar trocado.
• A pista para chegar na parábola está em que só há grau 2 em uma das
coordenas.

Para entendermos melhor as cônicas nos casos e 6= 1:

Afirmação 2.2. No caso 0 < e < 1 da Afirmação 2.1, existe um novo sistema de
coordenadas (x, y) dado por
x=x−a e y=y
em que a equação vira:
x y
2
+ 2 =1
a b
e no qual as coordenadas do foco são

F = (− a2 − b2 , 0),
para
eρ p
a := > 0 e b := a2 · (1 − e2 ) > 0.
1−e
Ademais2: √
a2 − b2
e= .
a
1semi largura ortogonal
2Na

apostila c := a2 − b2 para elipses
CAPÍTULO 20. AS CÔNICAS E SUAS PROPRIEDADES REFLETIVAS 259

No caso 1 < e da Afirmação 2.1, existe um novo sistema de coordenadas (x, y)


dado por
x=x−a e y =y
em que a equação vira:
x y
2
− 2 =1
a b
e no qual as coordenadas do foco são

F = ( a2 + b2 , 0),
onde
eρ p
a := >0 e b := a2 (e2 − 1) > 0.
e−1
Ademais3: √
a2 + b2
e= .
a

Definição 2.3. A cônica do caso 0 √


< e < 1 da Afirmação 2.2 é chamada elipse.
√Um reta vertical por F1 = (− a2 − b2 , 0) intersecta a elipse em dois pontos
b2 2
(− a − b , ± a ). A distância de F1 a cada um deles, que é ba , é o semi-latus rectum
2 2

da elipse.
Note que:
• A elipse tem simetria tanto no eixo dos x como no eixo dos y. Daı́ se obtem
que
√ ela poderia ser definida também com base num √ segundo foco F2 :=
( a2 − b2 , 0) como o foi com base em F1 := F = (− a2 − b2 , 0). Haverá
uma segunda diretriz, cuja distância ao foco F2 é a mesma da primeira diretriz
a F1 .

r1 r2
b
F1 F2

ρ a a ρ
b

• Se na equação
x2 y 2
+ 2 =1
a2 b
3Na

apostila, c := a2 + b2 para hipérboles
2. DEFINIÇÃO UNIFICADA DAS CÔNICAS 260

fazemos a = b então os dois focos coincidem em (0, 0) e temos o Cı́rculo de


raio a.
2
• O raio a = aa do cı́rculo é um caso particular de semi-latus rectum.
• Num novo sistema cartesiano (x, y) em que o vértice P0 está em (x, y) = (h, k)
e os focos estão na reta y = k, a elipse
x2 y 2
+ 2 =1
a2 b
se escreve como:
(x − h)2 (y − k)2
+ =1
a2 b2
que expandido dá uma expressão do tipo:
a1 x2 + a2 x + a3 y + a4 y 2 + a5 = 0.
Em Exercı́cios pode se pedir para, a partir de uma equação de elipse do tipo
a1 x2 + a2 x + a3 y + a4 y 2 + a5 = 0
determinar focos, eixos e a excentricidade.
Também o papel de x e y pode estar trocado.
2 2
• A pista para chegar na elipse na forma (x−h)
a2
+ (y−k)
b2
= 1 está em completar
os quadrados, ou seja, agrupar os termos em x separadamente dos em y e
forçar a parecer binômios (x − h)2 e (y − k)2
Definição 2.4. A cônica do caso 1 < e da Afirmação 2.2 é chamada hipérbole e tem
simetria4 no eixo x e no eixo y.√
Um reta vertical por F1 = ( a2 + b2 , 0) intersecta a elipse em dois pontos
√ b2
( a2 + b2 , ± ).
a
b2
A distância de F1 a cada um deles, que é a , é o semi-latus rectum da hipérbole.

Demonstração. (da Afirmação 2.1)


Seja então R ∈ r o pé da perpendicular a r traçada desde F . Considere o segmento
de reta RF .
Afirmo que existe apenas um ponto5 P0 no segmento RF tal que
P0 F = e · P0 r.
De fato, se identificamos a reta RF com os Reais, e se usamos a coordenada 0
para R e f > 0 para F , queremos resolver a equação:
f − x = e · (x − 0) = e · x,
o que dá:
(e + 1) · x = f,
f
cuja única solução é x0 = e+1
. Noto que 0 < x0 < f , pois e > 0.
4Daı́ se obtem que

poderia ser definida
√ também com base num segundo foco F2 := (− a2 + b2 , 0)
como o foi com base em F1 := F = ( a2 + b2 , 0).
5Será chamado de vértice
CAPÍTULO 20. AS CÔNICAS E SUAS PROPRIEDADES REFLETIVAS 261

Escolho como sistema cartesiano de coordenadas (x, y) aquele que tem origem em
P0 , eixo horizontal P0 F (orientado de R para F ) e eixo vertical a perpendicular a
P0 F por P0 .
Nesse sistema, P0 = (0, 0) e se ρ := P0 r > 0 a diretriz é
x = −ρ e F = (eρ, 0).
Ademais, pela sua Definição, qualquer ponto P = (x, y) da cônica verifica:
p p
(x − eρ)2 + y 2 = e · (x + ρ)2 ,
p p
pois P F = (x − eρ)2 + y 2 e P r = (x + ρ)2 . Portanto os pontos da cônica satis-
fazem:
(x − eρ)2 + y 2 = e2 · (x + ρ)2 ,
ou seja, após simplificar:
(1 − e2 ) · x2 − 2e(1 + e)ρ · x + y 2 = 0.
Caso e = 1:
Nesse caso a equação acima vira:
4ρ · x = y 2 ,
com F = (ρ, 0) e a diretriz vira x = −ρ.

Caso 0 < e < 1:


Nesse caso podemos dividir a equação
(1 − e2 ) · x2 − 2e(1 + e)ρ · x + y 2 = 0
por 1 − e2 obtendo:
2 2eρ y2
x − ·x+ = 0.
1−e 1 − e2
Introduzo uma constante a e depois uma b pela regra:
eρ p
a := e b := a2 · (1 − e2 ).
1−e
Já é bom notar que:
0 < b < a, pois 0 < 1 − e2 < 1.
Então a última equação vira:
2 a2 2
x − 2ax + 2 · y = 0
b
que dividida por a2 dá:
x2 2 y2
− · x + = 0.
a2 a b2
Caso 1 < e: Nesse caso, analogamente ao que fizemos no Caso anterior, mas com
eρ p
a := > 0 e b := a2 (e2 − 1) > 0
e−1
obtemos a equação:
x2 2 y2
+ · x − = 0.
a2 a b2
2. DEFINIÇÃO UNIFICADA DAS CÔNICAS 262

Demonstração. (da Afirmação 2.2)


No caso 0 < e < 1 já temos a equação
x2 2 y2
− · x + =0
a2 a b2
para a cônica, onde

a := > 0.
1−e
Portanto vemos que essa cônica intersecta a reta y = 0 em P0 = (0, 0) e em
P1 := (2a, 0).
Considere o ponto médio do segmento P0 P1 :
C := (a, 0).
Vamos transladar a origem do sistema de coordenadas para C. Para isso esta-
beleçamos um novo sistema de coordenadas (x, y) onde:
x = x − a e y = y.
Então a equação da cônica vira:
(x + a)2 2 y2
− · (x + a) + = 0,
a2 a b2
ou seja:
x2 y 2
+ 2 = 1.
a2 b
O foco F tinha coordenada x dada por eρ e agora, no novo sistema, terá coorde-
nada x dada por:
eρ e2 ρ
eρ − a = eρ − =− =
1−e 1−e
p p
e4 ρ2 e2 ρ2 − e2 ρ2 (1 − e2 )
=− =− =
1−e 1−e
s
e2 ρ2 e2 ρ2 (1 − e2 )
=− − =
(1 − e)2 (1 − e)2

= − a2 − b2 .
Das duas primeiras igualdades acima temos:
eρ − a = −ae
e do anterior: √
a2 − b2
e= .
a
Já no caso 1 < e temos a equação
x2 2 y2
+ · x − =0
a2 a b2
CAPÍTULO 20. AS CÔNICAS E SUAS PROPRIEDADES REFLETIVAS 263

para a cônica.
Portanto essa cônica intersecta a reta y = 0 em P0 = (0, 0) e em
P1 := (−2a, 0).
Considere o ponto médio do segmento P0 P1 :
C := (−a, 0).

r ’ r

ρ ρ
C
F ’ a a F

Vamos transladar a origem do sistema de coordenadas para C. Para isso usamos


um novo sistema de coordenadas (x, y) onde:
x = x + a e y = y.
Então a equação da cônica vira:
(x − a)2 2 y2
+ · (x − a) − 2 = 0,
a2 a b
ou seja:
x2 y 2
− 2 = 1.
a2 b
O foco F tinha coordenada x dada por eρ e agora, no novo sistema, terá coorde-
nada x dada por:
eρ e2 ρ
eρ + a = eρ + = =
e−1 e−1
p p
e4 ρ2 e2 ρ2 + e2 ρ2 (e2 − 1)
= = =
e−1 e−1
s
e2 ρ2 e2 ρ2 (e2 − 1)
= + =
(e − 1)2 (e − 1)2

= a2 + b2 .
2. DEFINIÇÃO UNIFICADA DAS CÔNICAS 264
2 2
A simetria no eixo x da equação xa2 − yb2 = 1 indica que a hipérbole poderia ser

definida em relação a um foco F ′ = (− a2 + b2 , 0) e uma diretriz r ′ , como mostra a
Figura acima. √
2 2
A relação e = a a+b é imediata das definições de a e b.


Uma observação final. Como para as elipses


a2 − b2
e=
a

e para as hipérboles


a2 + b2
e= ,
a

vemos que as expansões/contrações dadas por

φ(x, y) = (λ · x, λ · y), λ>0

não mudam a excentricidade. A figuras a seguir mostram elipses e hipérboles com a


mesma excentricidade:

4
2

y 0
-10 -5 0 5 10
-2x
-4
CAPÍTULO 20. AS CÔNICAS E SUAS PROPRIEDADES REFLETIVAS 265

9−1
Figura: Elipses de excentricidade igual a e = 3

4
2
y 0
-15 -10 -5 0 5 10 15
-2
-4x


9+1
Figura: Hipérboles de excentricidade igual a e = 3

Voltaremos ao estudo das cônicas na Seção 7 do Capı́tulo 39, onde as descrevere-


mos em coordenas polares. Papel especial será desempenhado pelas elipses.

3. A Parábola e sua propriedade refletiva


A parábola também aparecerá com destaque mais adiante, na Seção 8 do Capı́tulo
35, associada à balı́stica.
Um dos casos mais simples em que a reta tangente muda de acordo com o ponto
escolhido no gráfico é o caso das parábolas.
Mesmo assim já podemos obter algumas informações interessantes, como o mostrarão
as Seções seguintes, desde que soubermos calcular essas tangentes.
Afirmação 3.1. Um ponto P satisfaz a equação
y = Cx2 , C∈R
1 1
se e somente se P equidista da reta horizontal y = − 4C e do ponto F = (0, 4C )
(chamado de foco).
Demonstração.
Para provarmos isso, basta usarmos o caso e = 1 da Afirmação 2.1, trocando x
1
por y e fazendo C = 4ρ .
Mas também podemos fazer uma conta explı́cita, como segue.
Temos para P = (x, Cx2 ):
r
1 2
P F = (x − 0)2 + (Cx2 − ) =
4C
r
x2 1
= x2 + C 2 x4 − + 2 2 =
2 4C
3. A PARÁBOLA E SUA PROPRIEDADE REFLETIVA 266
r
x2 1
= C 2 x4 + + 2 2 =
2 4C
r
1 2
= (Cx2 + )
4C
1
e a distância de P até a reta y = − 4C é dada pelo tamanho
r
1 2
(Cx2 + ) .
4C
Reciprocamente, se P = (x, y) satisfaz
r r
1 2 1 2
x2 + (y − ) = (y + )
4C 4C
então
1 2 1 2
x2 + (y − ) = (y + )
4C 4C
de onde
y 1 y 1
x2 + y 2 − + 2 2 = y2 + + 2 2,
2C 4 C 2C 4 C
de onde:
y
x2 = e y = Cx2 .
C


1
Considere então a parábola y = Cx2 , com foco F := (0, 4C ) e reta diretriz hori-
1
zontal y = − 4C .
Dado um ponto P = (x, Cx2 ) qualquer de seu gráfico, denote p sua a projeção
vertical na reta diretriz:
1
p := (x, − ).
4C
Afirmação 3.2.
1 1
A reta rx que liga os pontos p = (x, − 4C ) e F = (0, 4C ) é ortogonal à reta tangente
2 2
Tx ao gráfico de y = Cx em P = (x, Cx ).
Ademais, rx e Tx se intersectam em Mx := ( x2 , 0), que é o ponto médio do segmento
de p e F .
Em suma, Tx é a reta mediatriz do segmento ligando p e F .

As Figuras a seguir ilustram a Afirmação:


CAPÍTULO 20. AS CÔNICAS E SUAS PROPRIEDADES REFLETIVAS 267

0
-4 -2 0 2 4
x

-2

-4

2
Fig: y = x4 , tangente y = x − 1 em P = (2, 1),
onde F = (0, 1), M = (1, 0) e p = (2, −1).

2
x
-4 -2 0 2 4
0

-2

-4

-6

-8

Fig: A Figura de antes e ademais a tangente y = 32 x − 9


4
em P = (3, 1), M = ( 32 , 0) e p = (3, −1).

Demonstração.
Já sabemos que a reta tangente Tx tem equação:
y = (2Cx) · x − Cx2 .
E a reta rx ligando p e F tem coeficiente angular:
1
4C
− −1
4C −1
= ,
0−x 2Cx
logo rx e Tx são ortogonais.
1
Por passar por F = (0, 4C ) a equação de rx é:
−1 1
rx : y = ·x+ .
2Cx 4C
Avaliando ambas as equações de retas em Mx = ( x2 , 0) vemos que Tx e rx contêm
Mx = ( x2 , 0).
3. A PARÁBOLA E SUA PROPRIEDADE REFLETIVA 268
1
Ademais as coordenadas de Mx são média aritmética das coordenadas de (x, − 4C )
1
e (0, 4C ), logo Mx é ponto médio do segmento que os une.


Agora vamos extrair consequências da Afirmação 3.2.

Note que os triângulos retângulos ∆F P Mx e ∆p P Mx são congruentes: de fato,


P F = P p já que P está na parábola, F Mx = Mx p por Mx ser ponto médio e P Mx
ser lado comum a ambos.
Logo os ângulos ∠F P Mx e ∠Mx P p são congruentes.
Considere em torno de P os ângulos ∠Mx P p e seu ângulo oposto pelo vértice.
Como são congruentes, temos que o ângulo que a reta vertical pP faz com a tangente
Tx é congruente com o ângulo ∠F P Mx .

F
P

Em Ótica se postula que a luz se reflete numa curva da seguinte forma:

o ângulo de incidência que se forma entre o raio de luz e a tangente da curva é


igual ao ângulo (não orientado) formado pelo raio refletido e a tangente da curva.

Pelo que vimos acima, isso quer dizer que raios de luz que chegam verticalmente
1
devem refletir na parábola y = Cx2 e passar todos pelo ponto F = (0, 4C ) que por
isso merece o nome de foco, por concentrar a luz. Esse fato é usado em antenas,
microfones, espelhos de formato parabólico, para concentrar ondas, som, calor, luz
em um ponto, que é o Foco.
Como não posso plotar retas verticais, não pude fazer o Exemplo a seguir na
posição vertical. Tive que colocar na horizontal. E só pude usar metade da parábola,
para ter um gráfico. Então a Figura a seguir ilustra a concentração de 5 raios hori-
zontais refletidos no Foco:
CAPÍTULO 20. AS CÔNICAS E SUAS PROPRIEDADES REFLETIVAS 269

2,5

1,5

0,5

0
0 0,20,40,60,8 1
x

y2
Figura: Braço da parábola x = 4
refletindo 5 raios horizontais no Foco F = (1, 0).

4. Prova analı́tica da propriedade do foco


Vou dar uma prova analı́tica do fato de que os raios verticais que incidem numa
parábola são todos refletidos para o foco.
A afirmação a seguir será útil em outros contextos6:

Afirmação 4.1. Seja (x, y) ponto do gráfico de y = f (x) em que o gráfico não tem
inclinação zero.
Se uma reta vertical por esse ponto é refletida no gráfico de tal modo que o ângulo
de incidência que forma com a reta tangente é igual ao ângulo que a reta refletida
forma coma reta tangente, então a equação da reta refletida é:

f ′ (x)2 − 1 f ′(x)2 − 1
y=( ) · x + f (x) − ( ) · x.
2f ′(x) 2f ′ (x)

Demonstração.
Na figura a seguir em azul estão os ângulos de incidência e de reflexão, supostos
iguais (congruentes). A reta horizontal é h.
Também t e n são as retas tangente e normal. Dois ângulos retos dão indicados.

6Aprendiisso no Tomo 3 do Traité des courbes speciales remarquables, planes et gauches, de F.


Gomes Teixeira, 1971, Chelsea Publishing Company
4. PROVA ANALÍTICA DA PROPRIEDADE DO FOCO 270

y = f(x)

Na figura a seguir veja: α = f ′ (x) o ângulo que a reta tangente t faz com o eixo
horizontal, β o ângulo que o raio refletido faz com o eixo horizontal, α1 o ângulo que
a normal faz com a vertical e α2 o ângulo que o raio refletido faz com a normal.

y = f(x)

α t
1
α2 β

α
h

Note que que α1 é congruente com α. Ademais, da hipótese sai que α2 ≡ α1 E


daı́:
α2 ≡ α1 ≡ α.
Então
π π
β= + α1 + α2 = + 2 · α.
2 2
Na linha a seguir uso algumas identidades trigonométricas:
π 1
tan(β) = tan( − (−2α)) = cot(−2α) = − cot(2α) = − .
2 tan(2α)
CAPÍTULO 20. AS CÔNICAS E SUAS PROPRIEDADES REFLETIVAS 271

Ou seja, usando agora a fórmula da tangente de 2α,


1
tan(β) = − 2 tan(α) .
( 1−tan(α)2 )
Então o coeficiente angular da reta refletida é:
tan(α)2 − 1 f ′ (x)2 − 1
tan(β) = =
2 tan(α) 2f ′ (x)
e o coeficiente linear é imediato.


No caso da parábola y = C · x2 a equação da reta refletida, de acordo com a


Afirmação 4.1, é então:
4C 2 x2 − 1 4C 2 x2 − 1
y=( ) · x + Cx2 − =
4Cx 4C
4C 2 x2 − 1 1
=( )·x+ ,
4Cx 4C
1
portanto todas passam por (0, 4C ), o foco.

5. A Elipse e sua propriedade refletiva


Afirmação 5.1. Um ponto P = (x, y) satisfaz a equação
x2 y 2
+ 2 =1
a2 b
se e somente se
P F1 + P F2 = 2a,
onde F1 = (−c, 0) e F2 = (c, 0) são os dois focos e
a2 = b2 + c2
.

Observe que esta Afirmação 5.1 dá um método prático para traçar uma elipse: fixe
dois pontos F1 e F2 , com dois pregos, e ligue-os por um cordão maior que a distância
F1 F2 . Com um lápis estique o cordão e agora mova o lápis, sempre mantendo o
barbante esticado, traçando pontos P . Você traçará uma elipse, pois F1 P + P F2 é
constante.
Demonstração. (da Afirmação 5.1)
Como notamos após a Definição 2.3, uma elipse pode ser definida com relação a
dois pares Foco/diretriz: F, r ou F ′ r ′ .
Para qualquer ponto P da elipse temos
PF = e · P r e P F ′ = e · P r′,
onde r, r ′ são as retas diretrizes.
5. A ELIPSE E SUA PROPRIEDADE REFLETIVA 272

r r’

F F’
ρ a a ρ

Logo
P F + P F ′ = e · r r′,
onde r r ′ é a distância entre essas duas retas (paralelas).
Ou seja, que P F + P F ′ ≡ C é constante para pontos na elipse.
Na descrição que demos, a excentricidade e da elipse verifica:

a=
1−e
ou seja, 2a − 2ae = 2eρ e portanto
2a = e · (2a + 2p).
Ora, como nos lembra a Figura acima:
2a + 2ρ = r r ′
é a distância entre as duas retas diretrizes da elipse. Logo
P F + P F ′ ≡ 2a.
A Afirmação 2.2 e a simetria no eixo x dão que as coordenadas dos focos são
F1 = (−c, 0) e F2 = (c, 0), onde

c = a2 − b2 .


A elipse tem a notável propriedade seguinte:

se P é um ponto da elipse e P F1 , P F2 duas semiretas que ligam P aos focos,


então os ângulos formados por P F1 e a tangente em P e o formado por P F2 e a
tangente em P são iguais.
Em outras palavras, se um raio de luz sai de um foco e reflete na elipse então
ele passa no outro foco.
Para provar isso, notamos primeiro o seguinte:
CAPÍTULO 20. AS CÔNICAS E SUAS PROPRIEDADES REFLETIVAS 273

Afirmação 5.2. Se uma reta só intersecta uma elipse num único ponto P , então
essa reta é a reta tangente à elipse em P .
Demonstração.
2 2
Considerarei apenas pontos da elipse xa2 + yb2 = 1 com coordenada y > 0, ou seja,
onde posso representar a elipse pelo gráfico de
r
x2
y = b · 1 − 2,
a
pois para os outros é análogo, usando outros gráficos
q do tipo y = y(x) ou x = x(y).
2
Uma reta y = A · x + B que passa por (x, b · 1 − xa2 ) tem equação:
r
x2
y = A x + (b · 1 − 2 − Ax).
a
x2 y2
Se a intersecto com a elipse a
+ = 1 obtemos:
b2
q
2
x2 (A x + b 1 − xa2 − Ax)2
+ − 1 = 0,
a2 b2
que é uma equação quadrática em x:
q
x 2
A2
1 −2A x 2 2 1− a2
A a2 x2 x2
2
( 2 + 2) · x + ( + ) · x + − 2 =0
b a b2 b b2 a
A2 1
(note que de fato é quadrática em x, pois b2 + a2 > 0).
O dicriminante desta função quadrática em x é:
q
2
4(−a A + a A x − 2a b 1 − xa2 Ax − b2 x2 )
4 2 2 2 2 2
,
b2 a4
e procuramos valores de A tais que, ∀x, anulem esse discriminante (pois isso dirá que
para esses valores de A há apenas 1 intersecção da reta com a elipse).
Ou seja, buscamos A que anulem o numerador
r
x2
−a4 A2 + a2 A2 x2 − 2a2 b 1 − 2 Ax − b2 x2 .
a
Uma conta tediosa prova que:
r
x2
−a4 A2 + a2 A2 x2 − 2a2 b 1 − 2 Ax − b2 x2 =
a
bx
= (−a4 + a2 x2 ) · ( A + q )2
2
a2 1 − xa2
e portanto
−b x
A= q
2
a2 1 − xa2
é o valor de A que anula o discriminante acima, ∀x.
5. A ELIPSE E SUA PROPRIEDADE REFLETIVA 274

Por outro lado reconhecemos que


−bx
q = f ′ (x),
x2
a2 1 − a2

onde r
x2
f (x) = b · .1−
a2
Logo a reta que só corta a elipse em P é de fato a sua reta tangente.


A seguinte afirmação explica o fato de que um raio e luz saindo de um foco da


elipse e refletindo na elipse passará necessariamente pelo outro foco:
Afirmação 5.3. As semiretas que ligam um ponto P da elipse aos dois focos F1 , F2
formam os mesmos ângulos (não-orientados) com a tangente à elipse passando por
P.
Demonstração.
Considere P na elipse e o triângulo ∆F1 P F2 .
Tome um ângulo externo α desse triângulo (veja a Figura).

F2 ’

F1 F2

Considere a bissectriz desse ângulo (ou seja, uma semireta que o divide em dois
ângulos iguais, de valores α2 ).
Marque um ponto F2′ no ângulo externo, cuja distância até P seja a mesma de F2
(denote essas distâncias por P F2 = P F2′ ). Veja a Figura:

r
F2 ’

α/2
β
α/2
Q

F1 F2
CAPÍTULO 20. AS CÔNICAS E SUAS PROPRIEDADES REFLETIVAS 275

Tome qualquer ponto Q da reta r que contém essa bissectriz, Q 6= P . Já que o Q
não está alinhado com F1 e F2′ , temos:
F1 Q + QF2′ > F1 P + P F2′ =
= F1 P + P F2 .
Já que a elipse é o lugar dos pontos P com
F1 P + P F2 ≡ 2a
vemos que Q não está na elipse.
Ou seja que o único ponto da reta r que está na elipse é P .
A Afirmação 5.2 anterior garante então que r é a tangente por P .
Mas o ângulo β é oposto pelo vértice ao ângulo que mede α2 .
Ou seja que as semiretas ligando P aos focos determinam ângulos com reta tan-
gente que medem ambos α2 .


6. A Hipérbole e o análogo da propriedade refletiva


Afirmação 6.1. Um ponto P = (x, y) satisfaz a equação
x2 y 2
− 2 =1
a2 b
se e somente se
| P F1 − P F2 | = 2a,
onde F1 = (−c, 0) e F2 = (c, 0) são os dois focos e b2 = c2 − a2 .
Demonstração.
Por exemplo suponhamos que P F1 − P F2 ≥ 0, como na Figura a seguir:.

ρ ρ

F1 a a F2

Por definição
P F1 − P F2 = e · P r1 − e · P r2 .
= e · r1 r2
logo P F1 − P F2 ≡ C é constante.
6. A HIPÉRBOLE E O ANÁLOGO DA PROPRIEDADE REFLETIVA 276

Pela Afirmação 2.2,



a= ,
e−1
ou seja 2ae − 2a = 2eρ e
2a = e · (2a − 2ρ).
Mas
2a − 2ρ = r1 r2 ,
como se vê na Figura acima.
Também a Afirmação 2.2 e a simetria da hipérbole no eixo x dão que os focos têm
essas coordenadas.


A hipérbole tem uma propriedade do mesmo tipo da elipse, a saber:

Os segmentos de reta que ligam um ponto de uma hipérbole aos seus dois focos
ficam bissectados pela reta tangente naquele ponto.

Para provarmos isso, como fizemos no caso da elipse, primeiro provaremos o


seguinte:
x2 y2
Afirmação 6.2. Se uma reta só intersecta uma hiperbole de equação a2
− b2
=1(
a, b > 0 ) num único ponto P , então
• i) essa reta é reta tangente à hiperbole em P ou
• ii) é uma reta paralela à reta y = ab · x ou
• iii) é uma reta paralela à reta y = − ab · x.

3
2
1
y 0
-6 -4 -2 0 2 4 6
-1
x
-2
-3

2
Figura: a hipérbole x22 − y 2 = 1 e retas paralelas
às retas y = 21 · x e y = − 21 · x.
Demonstração. (Afirmação 6.2)
CAPÍTULO 20. AS CÔNICAS E SUAS PROPRIEDADES REFLETIVAS 277
2 2
Considero pontos da hipérbole xa2 − yb2 = 1 com coordenada y > 0, ou seja, onde
posso representar a hipérbole pelo gráfico de
r
x2
y =b· − 1.
a2
Quero intersectar com a hipérbole uma reta qualquer y = A · x + B que passa por
r
x2
P = (x, b · − 1),
a2
ou seja, uma reta da forma:
r
x2
y = A·x+b − 1 − Ax.
a2
Obtenho então de
q
2
x2 (A · x + b 1 − xa2 − Ax)2
− − 1 = 0,
a2 b2
a equação em x:
q q
x2 x2
1 A 2
2A x2 2 a2
− 1 A x 2 2
A x2 2 a2
− 1 Ax
2
( 2 − 2 )x +( 2 − )x− 2 − 2 + = 0.
a b b b a b b2
Essa equação deixa de ser uma equação quadrática em x quando
1 A2
− = 0.
a2 b2
Ou seja, as retas passando por P com coeficientes angulares
b
A=±
a
só cortam a hipérbole em P .
2
Quando a12 − Ab2 6= 0 e a equação é quadrática, para termos P como única inter-
secção da reta e da hipérbole precisamos ter a anulação do dicriminante da função
quadrática em x. Ou seja, buscamos a condição:
q
2
4(−a4 A2 + a2 A2 x2 − 2a2 b xa2 − 1 Ax + b2 x2 )
= 0,
b2 a4
onde procuramos por coeficientes angulares A tais que, ∀x, seja nulo esse discrimi-
nante.
Ou seja, queremos A que anule o numerador
r
x2
−a4 A2 + a2 A2 x2 − 2a2 b − 1 Ax + b2 x2 .
a2
Mas uma conta tediosa mostra que:
r
4 2 2 2 2 2 x2
−a A + a A x − 2a b − 1 Ax + b2 x2 =
a2
6. A HIPÉRBOLE E O ANÁLOGO DA PROPRIEDADE REFLETIVA 278

bx
= (−a4 + a2 x2 ) · ( A − q )2
x2
a2 a2
−1
e portanto
bx
A= q
x2
a2 a2
−1
é o valor de A que anula o discriminante acima, ∀x.
Por outro lado reconhecemos que
bx
q = f ′ (x),
x2
a2 a2
−1
onde r
x2
− 1.
f (x) = b ·
a2
Logo, se uma reta corta a hipérbole em um único P , então é a reta tangente em P
ou paralelas a y = ab · x ou y = − ab · x.


2 2
Afirmação 6.3. Quando |x| → ∞ os pontos da hiperbole xa2 − xy 2 = 1 se aproximam
das reta y = ab · x ou da reta y = − ab · x (chamadas de assı́ntotas).

Com esta Afirmação e a Afirmação 6.2 podemos dizer:

fora as tangentes, as únicas retas que só cortam a hipérbole em 1 ponto são as
retas paralelas às assı́ntotas da hipérbole dada.

Demonstração. (Afirmação 6.3)


x2 y2
Cada ponto da hipérbole a2
− b2
= 1 pode ser descrito ou como ponto do gráfico
de r
x2 b √ 2
f1 (x) = b · − 1 = · x − a2 ,
a2 a
ou como ponto do gráfico de
r
x2 b √ 2
f2 (x) = −b · − 1 = − · x − a2 .
a2 a
Se vamos fazer |x| → ∞, obviamente podemos supôr |x| = 6 0 e escrever:
r r
b a2 b a2
f1 (x) = x2 (1 − 2 ) = |x| 1 − 2 ,
a x a x
r r
b a2 b a2
f2 (x) = − x2 (1 − 2 ) = − |x| 1 − 2 ,
a x a x
CAPÍTULO 20. AS CÔNICAS E SUAS PROPRIEDADES REFLETIVAS 279

e claramente: r
a2
lim 1− = 1.
|x|→+∞ x2
b
Ou seja, quando |x| → ∞ o gráfico de f1 tende ao gráfico de y = a
· |x| enquanto que
o de f2 tende ao de y = − ab · |x| .

Podemos ser mais detalhados:


Se x → +∞, temos o gráfico de f1 (x) se aproximando do de y = ab · x. Mas se
x → −∞ temos f1 (x) se aproximando de
b b
y = · (−x) = − · x.
a a
Se x → +∞, temos o gráfico de f2 (x) se aproximando do de y = − ab x. Mas se
x → −∞ temos f2 (x) se aproximando do de
b b
y = − · (−x) = · x.
a a


Afirmação 6.4. As semiretas que ligam um ponto P da hipérbole aos dois focos
F1 , F2 formam os mesmos ângulos (não-orientados) com a tangente à hipérbole em
P.
Demonstração.
Considere P um ponto da hipérbole. Como | P F1 − P F2 | ≡ C > 0 posso supor
que tomei P no ramo da hipérbole onde P F1 − P F2 ≡ C > 0 (seria análogo o outro
caso, trocando os papéis de F1 e F2 ).

F2 ’
α/2 α/2

F1 F2

Marque no segmento de reta [F1 P ] o ponto F2′ que tem P F2 = P F2′ .


Considere a bissectriz r do ângulo α em P que faz parte do triângulo ∆F1 P F2 .
6. A HIPÉRBOLE E O ANÁLOGO DA PROPRIEDADE REFLETIVA 280

Tome um ponto Q ∈ r, Q 6= P .

Caso 1: Suponhamos QF1 ≥ QF2′ :

Então como Q não está alinhado com F1 , F2′ , P , temos:

QF2′ + F2′ F1 > F1 Q,


e portanto:
F2′ F1 > F1 Q − QF2′ ≥ 0.
Note que a nossa reta r funciona também como mediatriz do segmento [F2′ F2 ] (por
ser a bissectriz do triângulo isósceles ∆F2′ P F2′ ). Logo

QF2′ = QF2
e portanto:
F2′ F1 > F1 Q − QF2 .
Por outro lado, já que o ponto F2′ está no segmento [F1 P ], temos:

F2′ F1 = P F1 − P F2′ =

= P F1 − P F2 .
Como este último valor é positivo, pela escolha de P ,
| P F1 − P F2 | = P F1 − P F2 ≡ C > 0
e
| P F1 − P F2 | > F1 Q − QF2 ≥ 0
nos faz concluir que Q não pertence à elipse.
Ou seja, que da reta r somente o ponto P está na elipse.
Vemos em seguida que r não é paralela a nenhuma das assı́ntotas da hipérbole.
Portanto, pela Afirmação 6.2, conclı́mos que r é a tangent à hipérbole no ponto P .

Caso 2: Suponhamos QF2′ ≥ QF1 :

Então como Q não está alinhado com F1 , F2′ , P , temos:

QF1 + F1 F2′ > QF2′ ,


e portanto:
F2′ F1 > QF2′ − QF1 ≥ 0.
O Resto da prova neste Caso 2 é exatamente igual ao do Caso 1.

CAPÍTULO 20. AS CÔNICAS E SUAS PROPRIEDADES REFLETIVAS 281

7. Famı́lia de cônicas co-focais ortogonais


Considere a seguinte famı́lia de cônicas:
x2 y2
+ = 1, k > 0,
λ λ − k2
com k fixado e o parâmetro λ > 0, λ 6= k 2 .
A Figura a seguir ilustra o caso em que k = 2, onde escolhi 10 valores
λ = 15, 10, 8, 6, 5, 3.5, 3, 2, 1, 0.3

y 0
-4 -2 0 2 4
x

-2

-4

A Afirmação a seguir descreve a famı́lia em detalhe. O item iv) é surpreendente !

Afirmação 7.1.
• i ) todas as cônicas dessa famı́lia têm os mesmos Focos (k, 0) e (−k, 0). Se
λ − k 2 > 0 a cônica correspondente ao λ é uma elipse com excentricidade
√k . Se λ − k 2 < 0 a cônica correspondente ao λ é uma hipérbole com
λ
excentricidade √kλ .
7. FAMÍLIA DE CÔNICAS CO-FOCAIS ORTOGONAIS 282

• ii) em cada ponto (x, 0) do eixo dos x, diferente dos dois Focos (k, 0) e (−k, 0)
e da origem, só passa um elemento da famı́lia de cônicas. De fato, se |x| > k
então passa só uma elipse cujo parâmetro é λ = x2 e cuja excentricidade é
a
e = |x| < 1. E se |x| < k então só passa uma hipérbole cujo parâmetro é
2 a
λ = x e cuja excentricidade é e = |x| > 1.
• iii) em cada ponto (0, y) do eixo dos y, diferente da origem só passa uma
elipse da famı́lia, com parâmetro λ = k 2 + y 2 e excentricidade √ k
k 2 +y 2
• iv) em cada ponto (x, y) com x · y 6= 0 passam dois elementos da famı́lia,
uma elipse e uma hipérbole, e a intersecção é ortogonal7
Demonstração.
Do item i):
Basta aplicar a Afirmação 2.2 para encontrar os focos e a excentricidade. Note
que se λ − k 2 < 0 as hipérboles são:
x2 y2
− 2 = 1.
λ k −λ

De ii):
Dado o ponto (x, 0) a expressão:
x2 y2
+ = 1, k>0
λ λ − k2
produz a seguinte equação quadrática em λ:
λ2 − λ · (k 2 + x2 ) + k 2 · x2 = 0.
Se x2 − k 2 > 0 (ou seja, |x| > k) o discriminante dessa equação vira:
x2 − k 2
e obtemos duas soluções:
λ = x2 e λ = k 2
mas por hipótese excluı́mos λ − k 2 . Analogamente se x2 − k 2 < 0.

De iii): Para um ponto (0, y) equação em λ agora é linear:


y2
= 1 ⇔ λ = k2 + y2.
λ − k2

De iv):
Deixo para o leitor verificar que para cada ponto (x, y) com x · y 6= 0 passam duas
cônicas diferentes, uma com excentricidade > 1 e a outra < 1. A única coisa que
quero destacar é que os parâmetros λ1 , λ2 são as soluções da equação quadrática em
λ:
λ2 − λ · (k 2 + x2 + y 2 ) + x2 · k 2 = 0
7Quando duas curvas se intersectam, o ângulo que formam é medido com base no ângulo formado
por suas retas tangentes.
CAPÍTULO 20. AS CÔNICAS E SUAS PROPRIEDADES REFLETIVAS 283

que sai de
x2 y2
+ = 1.
λ λ − k2
Lembro que:
λ1 + λ2 = k 2 + x2 + y 2 e λ1 · λ2 = x2 · k 2 ,
já que
λ2 − λ · (k 2 + x2 + y 2 ) + x2 · k 2 = (λ − λ1 ) · (λ − λ2 ).
Nesses pontos (x, y) com x · y 6= 0, as duas curvas da famı́lia que passam pelo
ponto não são verticais, ou seja, localmente em torno de cada ponto as duas curvas
são gráficos da forma y = fλ1 (x) e y = fλ2 (x). De fato,
2 y2
∂( xλ + λ−k 2
− 1)
=0⇔y=0
∂y
e podemos usar o Teorema 2.1 do Capı́tulo 15.
Também por esse mesmo Teorema calculo:
( 2x
λ1
) −x λ1 − k 2
fλ′ 1 (x) =− = ·( ),
( λ12y
−k 2
) y λ1
enquanto que
−x λ2 − k 2
fλ′ 2 (x) = ·( ).
y λ2
Agora noto que termos a condição:
−1
fλ′ 1 (x) =
fλ′ 2 (x)
equivale a termos
(x2 + y 2) · λ1 · λ2 − x2 · k 2 · (λ1 + λ2 ) + x2 · k 4 = 0,
o que conseguimos que seja verdade se usamos:
λ1 · λ2 = x2 · k 2 e λ1 + λ2 = k 2 + x2 + y 2.
Ora,
−1
fλ′ 1 (x) =
fλ′ 2 (x)
é a condição de ortogonalidade, por isso cada par elipse-hipérbole que se encontra
num ponto é ortogonal.


Para vermos exemplos de famı́lias de cúbicas ortogonais precisaremos da Seção 3


do Capı́tulo 50.
8. EXERCÍCIOS 284

8. Exercı́cios
Exercı́cio 8.1. 2
2
Chamamos uma hipérbole xa2 − yb2 = 1 de retangular se suas assı́ntotas são ortog-
onais entre si.
Qual a relação entre a e b que é necessária e suficiente para termos uma hipérbole
retangular ?
Exercı́cio 8.2. (resolvido)
Um planeta de move em trajetória elı́ptica, em que o Sol é um dos focos da elipse.
Observado a partir de um ponto (x, y) = (0, 0), o planeta está, num certo instante
t0 , na posição (x0 , y0 ), onde x0 > y0 > 0.
Ademais, sua coordenada x tem em t0 uma taxa de variação de −1 UA/s, enquanto
que sua coordenada y tem taxa de variação de 1 UA/s.
i) Determine a equação (padrão) da elipse que descreve sua trajetória.
ii) Determine as posições possı́veis do Sol.
iii) A distância do foco onde está o Sol até o vértice mais próximo é chamado de
perihélio do planeta. Determine-o.
CAPı́TULO 21

Integração e o Primeiro Teorema Fundamental

1. Área sob um gráfico positivo

Dado um gráfico de uma função contı́nua y = f (x) ≥ 0 quero entender qual a


Área compreendida sob esse gráfico e acima do eixo x, da vertical x = a até a vertical
x = b.
Se y = f (x) = ax+b é uma reta tudo ok, já sabemos o que são áreas de triângulos,
retângulo, trapézios, etc. Mas e se y = f (x) não for uma reta ? Se f (x) não é a
equação de uma reta, vemos que realmente precisamos definir de maneira matemati-
camente correta a intuição que temos de que há uma figura sob esse gráfico e que ela
tem uma certa área.
A idéia de Bernard Riemann é de ir subdividindo o domı́nio da f e colocando lado
a lado retângulos sob o gráfico (vou chamá-los de retângulos justapostos sob o gráfico).
A soma das áreas desses retângulos é menor que a área buscada, mas a medida que
se refina a subdivisão do domı́nio a soma de áreas dos retângulos justapostos sob o
gráfico se aproxima de um certo valor.
Isso funciona bem por exemplo se f : [a, b]] → R é contı́nua.
Se f não fosse contı́nua em [a, b], quem sabe os valores da f ficassem tão altos
quanto quiséssemos, o que levaria em muitos casos a que a área da região sob seu
gráfico devesse ser considerada infinita, não um número determinado. 1

1Veremos mais adiante, quando tratarmos de integrais impróprias que, às vezes, a integração
consegue domar o infinito, tanto do tamanho do intervalo onde se integra, quanto dos valores da
função em [a, b].
285
2. QUAL FUNÇÃO DESCREVE AS ÁREAS SOB GRÁFICOS? 286

Figura: Cinco retângulos sob o gráfico, de mesma largura (1/5 do intervalo).

1
Figura: 12 retângulos sob o gráfico, de mesma largura ( 12 do intervalo).

1
Figura: 24 retângulos sob o gráfico, de mesma largura ( 24 do intervalo).

Nem precisam ser retângulos de mesma largura, como nas Figuras acima. Basta
que o máximo das larguras dos retângulos tenda a zero à medida que refinamos as
escolhas dos retângulos.
Isso parece ainda um pouco vago, mas na Seção 2 a seguir faremos alguns Exemplos
explı́citos, onde fazemos a partição da base ficar cada vez mais fina e obtemos, via um
limite, um valor bem determinando, que será a área. É possı́vel provar um teorema
geral do seguinte tipo:
Afirmação 1.1. (B. Riemann)2 Seja f : [a, b] → R, f (x) ≥ 0 contı́nua.
Esse número é por definição a Área sob o gráfico de f , de a até b, denotada por
Af,a (b).

2. Qual função descreve as Áreas sob gráficos?

Dado uma função y = f (x) não-negativa, fixado um ponto inicial a de seu domı́nio
definimos acima a área sob seu gráfico até b.
Vamos agora fixar a e mudar o nome de b, passando a chamar-se agora x para
significar que vamos variar o b.
Então a área sob o gráfico vira uma nova função Af,a (x), que para cada valor de
x dá um resultado de Área.
Qual é essa função A(x)? E que propriedades ela tem?
Certamente é uma função crescente, será que Af,a (x) é contı́nua? Será que ela é
derivável ?
Com o que sabemos do colégio, só consigo ver dois tipos de exemplos simples de
f , onde responderı́amos facilmente sobre Af,a (x):
2Observo desde já que se pode dar versões bem mais fortes desse teorema de Riemann.
CAPÍTULO 21. INTEGRAÇÃO E O PRIMEIRO TEOREMA FUNDAMENTAL
287

• Exemplo 1 : Se y = C ≥ 0 é constante e a = 0, então AC,0 (x) é a área de um


retângulo de largura x e altura C. Podemos tomar como um Axioma que
sua área é dada por
AC,0 (x) = C · x.
• Exemplo 2 : Se y = Cx e a = 0 então ACx,a (x) é a área de um triângulo de
largura x e altura Cx. Sabemos da geometria elementar que área é dada por
C · x2
ACx,a (x) = .
2
Mas que tal re-obter esse valor agora de um jeito novo, que servirá para
entender a área de muitos outros exemplos?
Particione o intervalo [0, x] em n intervalos de mesmo tamanho:
x x 2x (n − 1)x nx
[0, x] = [0, ] ∪ [ , ] ∪ . . . ∪ [ , ].
n n n n n
Tome um primeiro retângulo posto sob o gráfico de y = C · x, de base [ nx , 2x n
]
x 2x 3x 2x
e altura C · n , um segundo retângulo de base [ n , n ] e altura C · n e assim
até um (n − 1)-ésimo retângulo, cuja base é [ (n−1)x
n
, nx
n
] e altura C · (n−1)x
n
.
Dado n ∈ N, a soma das áreas dos (n − 1) retângulos acima é:
x x x 2x x (n − 1)x
·C · + ·C · + ...+ ·C · =
n n n n n n
x2
= C · 2 · [1 + 2 + . . . (n − 1)] =
n
x2 (n − 1) · n
=C · 2 ·[ ],
n 2
onde na última linha usamos o item i) da Afirmação 1.1, do Capı́tulo 13.
Se fazemos n → +∞ estamos cada vez mais nos aproximando da área do
triângulo, de fato:
x2 (n − 1) · n C · x2
lim C · · [ ] = .
n→+∞ n2 2 2
• Exemplo 3: Seja y = C · x2 , C ≥ 0, a = 0 escolha um x, 0 < x.
Faça a partição do intervalo [0, x] como no Exemplo anterior. Tome como
primeiro retângulo sob o gráfico de y = C · x2 o retângulo de base [ nx , 2x n
]e
x 2 2x 3x x 2
altura C( n ) , o segundo retângulo de base [ n , n ] e altura C(2 n ) e assim
até o (n − 1)-ésimo retângulo, cuja base é [ (n−1)x
n
, nx
n
] e altura C((n − 1) nx )2 .
Como esses retângulos estão sob o gráfico, a soma de suas áreas é certa-
mente menor que a área real sob o gráfico.
Mas se fazemos n cada vez maior, a soma de área de retângulos vai tender
à área real, que queremos conhecer.
De fato, dado n ∈ N, a soma das áreas dos (n − 1) retângulos é:
x x2 x 22 x 2 x (n − 1)2 x2
· C · 2 + · C · 2 + ...+ · C · =
n n n n n n2
2. QUAL FUNÇÃO DESCREVE AS ÁREAS SOB GRÁFICOS? 288

x x2
=C· · 2 · [12 + 22 + . . . (n − 1)2 ].
n n
No item iii) da Afirmação 1.1 vimos a fórmula:
n(n + 1)(2n + 1)
12 + 22 + . . . + n2 = , ∀n ∈ N,
6
que dá quando aplicada ao nosso n − 1:
(n − 1)(n − 1 + 1)(2(n − 1) + 1)
12 + 22 + . . . + (n − 1)2 = =
6
(n − 1)n(2n − 1)
= =
6
2n3 − 3n2 + n
= , ∀n ∈ N.
6
Ora, então a soma de áreas dos (n − 1) retângulos é de fato:
x x2 2n3 − 3n2 + n 2n3 − 3n2 + n
C· · 2· = Cx3 .
n n 6 6n3
Mas pelo que já vimos na Parte 1 (já que C e x não mudam com n):
2n3 − 3n2 + n Cx3
lim C · x3 · = .
n→+∞ 6n3 3
Cx3
Então é ACx2 ,0 (x) = 3
.

• Exemplo 4: Seja y = C · x3 , C ≥ 0. Mais uma vez, faça a partição do


intervalo [0, x] como no Exemplo anterior. Tome como primeiro retângulo
sob o gráfico o retângulo de base [ nx , 2xn
] e altura C( nx )3 , o segundo retângulo
de base [ 2x , 3x ] e altura C(2 nx )3 e assim até o (n − 1)-ésimo retângulo, cuja
n n
base é [ (n−1)x
n
, nx
n
] e altura C((n − 1) nx )3 .
Dado n ∈ N, a soma das áreas desses (n − 1) retângulos é:
x x3 x 23 x 3 x (n − 1)3 x3
· C · 3 + · C · 3 + ...+ · C · =
n n n n n n3
x x3
= C · · 3 · [13 + 23 + . . . (n − 1)3 ].
n n
Os itens i) e ii) da Afirmação 1.1 dão juntos a fórmula:
n(n + 1) 2
13 + 23 + . . . + n3 = ( ) , ) ∀n ∈ N,
2
que dá quando aplicada ao nosso n − 1:
(n − 1)2 (n)2 n4 − 2n3 + n2
13 + 23 + . . . + (n − 1)3 = = , ∀n ∈ N.
4 4
Ora, então a soma de áreas dos (n − 1) retângulos é de fato:
x x3 n4 − 2n3 + n2 n4 − 2n3 + n2
C· · 3· = Cx3 · .
n n 4 4n4
CAPÍTULO 21. INTEGRAÇÃO E O PRIMEIRO TEOREMA FUNDAMENTAL
289

Mas pelo que já vimos na Parte 1 (já que C e x não mudam com n):
n4 − 2n3 + n2 Cx4
lim Cx3 · = .
n→+∞ 4n4 4
4
Então ACx3 ,0 (x) = Cx4 .
• Exemplo 5) Também podemos combinar dois Exemplos desses de acima, por
exemplo perguntar pela área sob o gráfico de
y = C1 x2 + C2 x3 , C1 , C2 ≥ 0,
de 0 até x. A soma de área de retângulos sob o gráfico será:
x x2 x3 x (n − 1)2 x2 (n − 1)3 x3
· (C1 2 + C2 3 ) + . . . + · (C1 + C 2 )=
n n n n n2 n3
x3 2 2 2 x4 3 3 3
= C1 · (1 + 2 + . . . + (n − 1) ) + C 2 4 · (1 + 2 + . . . + (n − 1) ),
n3 n
e pelo que vimos nos dois exemplos anteriores 3),4) (e pelo limite de somas):
x3 2 2 2 x4 3 3 3
lim C1 · (1 + 2 + . . . + (n − 1) ) + C 2 4 · (1 + 2 + . . . + (n − 1) ) =
n→+∞ n3 n
x3 x4
= C1
+ C2 .
3 4
Nos 5 Exemplos acima há, digamos assim, uma coincidência notável:

A Área como função de x é uma função derivável e ademais a derivada da Área


é a função de partida
Cx2
A(x) = Cx ⇒ A′ (x) = C, A(x) = ⇒ A′ (x) = Cx,
2
Cx3 Cx4
A(x) = ⇒ A′ (x) = Cx2 , A(x) = ⇒ A′ (x) = Cx3 .
3 4
C1 x3 C2 x4
A(x) = + ⇒ A′ (x) = C1 x2 + C2 x3 .
3 4
Como veremos isso não é uma coincidência ! O fato geral por trás disso, de que
derivando a função Área sob o gráfico voltamos na função que dá o gráfico, será o
Primeiro Teorema Fundamental do Cálculo.
E de fato é a chave para se calcular áreas sob gráficos incrivelmente complicados
(no Segundo Teorema fundamental do Cálculo).

3. Primeira Versão do Primeiro Teorema fundamental do Cálculo


A princı́pio não sabemos muito sobre o gráfico de Af,a (x), porém o próximo teo-
rema vai nos dizer muito.
Para demonstrarmos o Teorema, começo com uma Afirmação, ilustrada na figura
que segue:
3. PRIMEIRA VERSÃO DO PRIMEIRO TEOREMA FUNDAMENTAL DO
CÁLCULO 290

Afirmação 3.1. Suponha f : [a, b] → R é contı́nua e f (x) ≥ 0.


Tome x ∈ [a, b) e h > 0 suficientemente pequeno para que x + h ∈ [a, b]. Então:
Af,x (x + h) = f (ξ) · h,
para algum ponto ξ ∈ [x, x + h].

M_f

f (ξ)

m_f

Figura: A área sob o gráfico é igual à do retângulo de altura f (ξ), mf < f (ξ) < Mf
Demonstração.
Começo observando que, dado o h > 0, o valor Af,x (h) tem que estar entre:
mf · h ≤ Af,x (x + h) ≤ Mf · h
onde mf · h é a Área de uma retângulo com base h e altura mf (o mı́nimo de f em
[x, x + h]) e Mf · h é a Área de uma retângulo com base h e altura Mf (o máximo de
f em [x, x + h]).
Divido por h > 0:
Af,x (x + h)
mf ≤ ≤ Mf ,
h
A (x+h)
e portanto f,x h é um valor intermediário da f : [a, b] → R, um valor entre seu
mı́nimo e seu máximo.
Logo pelo T.V.I. existe ξ ∈ [x, x + h] tal que
Af,x (x + h)
= f (ξ),
h
logo Af,x (x + h) = f (ξ) · h.


O Teorema a seguir diz que sempre a derivada da função que mede áreas sob um
gráfico é a função original que dá o gráfico.
Também pode ser lido assim: a operação de derivar cancela o efeito da operação
de tomar área sob o gráfico:
Teorema 3.1. (Primeira versão)
Seja f : [a, b] → R contı́nua, f ≥ 0 e x ∈ [a, b). Então
A′f,a (x) = f (x).
CAPÍTULO 21. INTEGRAÇÃO E O PRIMEIRO TEOREMA FUNDAMENTAL
291

Demonstração.
Como essa ainda é uma versão light do Primeiro Teorema, me permito mostrar
apenas que a derivada à direita da Área é igual a f (x), ou seja, que fixado x ∈ [a, b]
vale:
Af,a (x + h) − Af,a (x)
lim = f (x)
hց0 h
Ora, pela aditividade da Área, para h > 0:
Af,a (x + h) = Af,a (x) + Af,x (x + h),
portanto
Af,a (x) + Af,x (x + h) − Af,a (x)
lim =
hց0 h
Af,x (x + h)
= lim .
h→0 h
Agora uso a Afirmação 3.1 acima, de que
Af,x (x + h) = f (ξ) · h,
onde ξ ∈ [x, x + h]. Então juntando tudo:
Af,x (x + h)
lim =
h→0 h
f (ξ) · h
lim =
h→0 h
= lim f (ξ).
h→0
Para terminar basta ver que
lim f (ξ) = f (x).
h→0
Ora, quando h tende a zero, ξ ∈ [x, x + h] tende a x.
Logo f (ξ) tende a f (x), porque f é contı́nua.


4. A Integral e suas propriedades


Até aqui só falamos de funções contı́nuas que são f ≥ 0, pois queriamos falar de
áreas sob seu gráfico e acima do eixo dos x.
Mas é claro que se f < 0 na região [a, b] faz sentido definir a área da região
compreendida entre o eixo dos x e seu gráfico, que denotaremos ainda por Af,a (b).
Sem entrar em detalhes técnicos, quero apresentar uma operação chamada integral
definida de f de a até b, de uma função f contı́nua definida em [a, b] denotada:
Z b
f (x)dx.
a
Dada y = f (x) contı́nua em [a, b] escolha uma lista de pontos, começando em a e
terminando em b:
a = x0 < x1 < . . . < xn = b,
4. A INTEGRAL E SUAS PROPRIEDADES 292

que chamamos de partição de [a, b].


Chamamos de norma dessa partição o máximo dos tamanhos |xi − xi−1 |. dizer
que a norma fica pequena é dizer que aumenta o número de pontos xi e também que
eles ficam bem distribuı́dos em [a, b].
Dada uma partição, escolha uma lista de pontos ξi ∈ [xi , xi + 1]. Tome os valores
da f nesses ξi e faça a soma:
(x1 − x0 ) · f (ξ0 ) + (x2 − x1 ) · f (ξ1 ) + . . . + (xn − xn−1 ) · f (ξn−1 )
que chamaremos de somas de Riemann.
Note que agora pode haver parcelas negativas nessa soma, se f < 0.

Fig.: Retângulos na parte y > 0 contribuem sua área na soma de Riemann,


enquanto os na parte y < 0 contribuem com o negativo da área

Se acontecer de f ≥ 0 então essa soma se parece muito com as somas de áreas de


retângulos sob o gráfico, que fizemos na Seção 2.
É possı́vel refinarmos as partições [a, b], colocando mais pontos xi e escolhendo
mais pontos ξi . Isso produz novas somas de Riemann, como acima.
E podemos passar ao limite, fazendo a norma das partições tender a zero (ou seja,
o número n de pontos é feito n → +∞).
Teorema 4.1. (Integral e suas propriedades)
Seja f (x) contı́nua em [a, b]. Então
• i) passando ao limite, com as normas das partições tendendo a zero, as somas
de Riemann
(x1 − x0 ) · f (ξ0 ) + (x2 − x1 ) · f (ξ1 ) + . . . + (xn − xn−1 ) · f (ξn−1 )
Rb
convergem para um número denotado a f (x) dx.

• ii) esse limite não depende do tipo particular de soma de Riemann, apenas
de que as normas das partiões de [a, b] tendam a zero.
Rb
• iii) se f ≥ 0 então a f (x)dx = Af,a (b).
Rb
• iv) se f < 0 então a f (x)dx = −Af,a (b), onde esta área Af,a (b) é compreen-
dida entre o eixo dos x e o gráfico.
CAPÍTULO 21. INTEGRAÇÃO E O PRIMEIRO TEOREMA FUNDAMENTAL
293
Rc
• v) c
f (x)dx = 0 para qualquer c ∈ [a, b].

• vi) se escolhemos c com a < c < b então vale


Z c Z b Z b
f (x)dx + f (x)dx = f (x)dx.
a c a
Ra Rb
• vii) b
f (x)dx = − a
f (x)dx.
Rb Rb
• viii) | a
f (x) dx | ≤ a
| f (x) | dx.

• ix) Se f, g são contı́nuas em [a, b] e c1 , c2 ∈ R, então


Z b Z b Z b
(c1 · f (x) ± c2 · g(x)) dx = c1 · f (x) dx ± c2 · g(x) dx.
a a a

Observações:
• Complementando os itens iii) e iv), se f tem valores positivos e negativos,
Rb
então a integral a f dx dá a área lı́quida da região compreendida entre o eixo
dos x e o gráfico da f .
Um exemplo importante R a disso é quando uma função f é ı́mpar (isto é,
f (x) = −f (−x)) que terá −a f (x)dx = 0.
Rb
Chamo a atenção que quando tivermos a f (x)dx = 0 isto não dirá em
geral que f ≡ 0. Por exemplo se tomo [a, b] = [0, 2π] e f (x) = sin(x), então
o fato que veremos a seguir:
Z 2π
sin(x)dx = 0
0
significa que a área sob o gráfico do seno, de [0, π], é a mesma área da região
sobre o gráfico, de [π, 2π].
• Se f e g são contı́nuas e definidas em [a, b] em geral:
Z b Z b Z b
f (x) · g(x)dx 6= f (x)dx · g(x)dx,
a a a
x3
o que se vê comparando áreas Ax2 ,0 (x) = com o produto de áreas Ax,0 (x) ·
3
x2 x2
Ax,0 (x) = 2 · 2 . Veremos mais tarde uma técnica para fazer as
Z b
f (x) · g(x)dx
a
chamada integração por partes.
Demonstração. (do Teorema 4.1)
Me contentarei com dar algumas idéias sobre cada item. Os detalhes se vêem em
cursos de Análise Matemática.
i), ii) e iii) são técnicas, e nos dão a liberdade na escolha das partições.
iv): óbvia se sabemos iii).
v): óbvia, pois posso pensar em no domı́nio [a′ , b′ ] := {c}.
5. TEOREMA DO VALOR MÉDIO DE INTEGRAIS 294

vi): decorre da liberdade que temos nas partições de [a, b] = [a, c] ∪ [c, b].
vii): pode ser tomado como uma definição.
viii): Decorre da desigualdade triangular que:
| (x1 − x0 ) · f (ξ0) + (x2 − x1 ) · f (ξ1 ) + . . . + (xn − xn−1 ) · f (ξn−1) | ≤
≤ | (x1 − x0 ) · f (ξ0) | + | (x2 − x1 ) · f (ξ1 ) | + . . . + | (xn − xn−1 ) · f (ξn−1) | =
= (x1 − x0 ) · |f (ξ0) | + (x2 − x1 ) · | f (ξ1) | + . . . + (xn − xn−1 ) · | f (ξn−1) |,
e reconhecemos que esta última expressão é uma soma de Riemann da função
| f (x) |.
Logo ao passar ao limite obtemos a desigualdade entre as integrais.
ix) Decorre de
(x1 − x0 ) · ( c1 f (ξ0) ± c2 g(x0 ) ) + . . . + (xn − xn−1 ) · ( c1 f (ξn−1) ± c2 g(xn−1 )) =
= c1 · [(x1 − x0 ) · f (ξ0 ) + . . . + (xn − xn−1 ) · f (ξn−1 )]±
± c2 · [(x1 − x0 ) · g(ξ0) + . . . + (xn − xn−1 ) · g(ξn−1)].


5. Teorema do valor médio de integrais


O Lema 3.1 pode ser retomado, e a nova prova é análoga:
Afirmação 5.1. (Teorema do Valor Médio para integrais)
Seja f : [a, b] → R contı́nua. Então existe um ponto ξ ∈ [a, b] tal que:
Rb
f (t)dt
f (ξ) = a .
b−a
Demonstração.
Sejam
m := min{f (x); x ∈ [a, b]} = f (x1 )
e
M := max{f (x); x ∈ [a, b] = f (x2 ),
(ambos números existem pois f é contı́nua e [a, b] é fechado).
Então Z b
m · (b − a) ≤ f (t)dt ≤ M · (b − a),
a
Rb
o que se vê se lembramos que a f (t)dt é um limite de somas de Riemann.
Então dividindo por b − a > 0:
Rb
f (t)dt
f (x1 ) = m ≤ a ≤ M = f (x2 ),
b−a
Rb
f (t)dt
o que diz que o número a
b−a
é uma valor intermediário da função contı́nua f . Ou
Rb
f (t)dt
seja, pelo T.V.I. existe algum ξ ∈ [a, b] tal que f (ξ) = a
b−a
como afirmamos.

CAPÍTULO 21. INTEGRAÇÃO E O PRIMEIRO TEOREMA FUNDAMENTAL
295

Esse valor f (ξ) que aparece na Afirmação 5.1 pode ser interpretado como uma
generalização da média aritmética de um número finito de valores da f :
f (ξ1 ) + . . . f (ξn )
.
n
Isso se justifica claramente se os pontos ξi forem escolhidos bem distribuı́dos no in-
tervalo [a, b]. Pois tomando partições de [a, b] do tipo:
(b − a) n(b − a)
x0 := a < x1 := a + < . . . < xn := a + = b,
n n
f (ξ1 )+...f (ξn )
afirmo que podemos ver n
como uma soma de Riemann da integral
Rb Z b
a
f (t)dt f (t)
= dt.
b−a a b−a

De fato, como
b−a
xi − xi−1 =
n
temos
1 1 f (ξ1 ) f (ξn )
+ . . . f (ξn ) · =
f (ξ1 ) · · (x1 − x0 ) + . . . + · (xn − xn−1 ).
n n b−a b−a
Rb f (t)
e supondo ξi ∈ [xi−1 , xi ] a expressão da direita é uma soma de Riemann de a b−a
dt.

6. A integral indefinida e o Primeiro Teorema fundamental


O Teorema 3.1 que vimos acima, tem uma versão mais geral que usa, ao invés de
Af,a (x), a noção de integral indefinida. Trata-se de uma função do tipo:
Z x
F (x) := f (t)dt
a

que realmente depende de x. Note que usei t em f (t) dt para deixar x indicando o
ponto escolhido.
Teorema 6.1. (Primeiro Teorema fundamental do Cálculo)
Seja f : [a, b] → R contı́nua e x ∈ [a, b]. Então
Z x
( f (t)dt )′ (x) = f (x).
a

Observações:
Rx
• O Teorema diz que F (x) := a f (t)dt é uma primitiva de f , pois F ′ (x) =
f (x). Já sabemos que duas primitivas F1 , F2 da f definidas num mesmo inter-
valo
R x só diferem por uma constante
R F1 (x) ≡ F2 (x) + C. Então podemos usar
a
f (t)dt ou abreviadamente f dx como sı́mbolo para todas as primitivas de
f.
6. A INTEGRAL INDEFINIDA E O PRIMEIRO TEOREMA FUNDAMENTAL
296

• Alguns estudantes confundem duas coisas diferentes:


Z b Z x

( f (x)dx ) 6= ( f (t)dt )′ (b).
a a
Rb
Mas a da esquerda ( a f (x)dx )′ é a derivada
Rx em x de um número e sempre

será zero. Enquanto
R x que a da direita ( a f (t)dt ) (b) é a derivada em x da
função G(x) := a f (t)dt, ou seja, f (x), que é depois avalida em x = b,
dando f (b). E só dará zero se f (b) = 0.
Demonstração. (do Teorema 6.1)
Seja fixado x ∈ [a, b]. Rx
Queremos saber se para F (x) := a f (t)dt vale que
F ′ (x) = f (x).
Ou seja, se
R x+h
Rx
f (t)dt − a f (t)dt
a
lim = f (x).
h→0 h
Se x = a ou x = b podemos considerar apenas h > 0 ou h < 0. Mas para x ∈ (a, b)
precisamos considerar as duas possibilidades.

Caso h > 0:

Como x + h > x ≥ a:
Z x+h Z x Z x+h
f (t)dt − f (t)dt = f (t)dt.
a a x

A Afirmação 5.1 diz que:


Z x+h
f (t)dt = h · f (ξh ), ξh ∈ [x, x + h].
x

Então R x+h Rx
a
f (t)dt − a f (t)dt h · f (ξh )
lim = lim =
h→0 h h→0 h
= lim f (ξh ) = f (x),
h→0
por ser f contı́nua e por estarem ξh ∈ [x, x + h].

Caso h < 0:

Como agora a ≤ x + h < x, então


Z x+h Z x Z x
f (t)dt + f (t)dt = f (t)dt,
a x+h a
portanto:
Z x+h Z x Z x
f (t)dt − f (t)dt = − f (t)dt =
a a x+h
CAPÍTULO 21. INTEGRAÇÃO E O PRIMEIRO TEOREMA FUNDAMENTAL
297
Z x+h
= f (t)dt,
x
que foi a mesma conclusão do caso h > 0.
Por outro lado, a Afirmação 5.1 diz que:
Z x
f (t)dt = −h · f (ξh ), ξh ∈ [x + h, x].
x+h

Então Z x+h
f (t)dt = h · f (ξh ), ξh ∈ [x + h, x],
x
que é a mesma conclusão do caso h > 0, exceto que agora ξh está em [x + h, x].
O resto do argumento é igual ao do caso h > 0.


O Teorema 6.1 admite uma generalização, que é útil:


Afirmação 6.1. Seja g(x) função derivável e f (x) contı́nua.
Z g(x)
( f (t)dt )′(x) = f (g(x)) · g ′ (x).
a

Demonstração.
R g(x)
Considere a
f (t)dt como uma composição F ◦ g onde
Z u
F (u) := f (t)dt.
a
Então pela derivada da composta:
(F (g(x))′ (x) = F ′ (g(x)) · g ′(x).
Mas pelo Primeiro Teorema do Cálculo:
F ′ (u) = f (u).


7. Existem funções com primeira derivada, mas sem segunda derivada


Acostumados com os polinômios, que têm derivadas de todas as ordens (mesmo
que ≡ 0 a partir de um a certa ordem), poderı́amos pensar que sempre que uma
função tem alguma derivada tenha também as de ordem seguinte.
Isso é falso. Por exemplo, considere a função
Z x
F1 : [−1, 1] → R, F1 (x) := | t | dt.
−1

Pelo Primeiro Teorema Fundamental, = | x |. F1′ (x)


′′
Logo F1 não terá F (0) (já que sabemos que | x | não tem derivada em x = 0).
8. EXERCÍCIOS 298

Agora façamos,
Z x
F2 : [−1, 1] → R, F2 (x) := F1 (t) dt.
−1

Pelo Primeiro Teorema fundamental, F2′ (x) = F1 (x) e F2′′ (x) = | x |. Logo F2 tem
primeira e segunda derivadas em todos os pontos de seu domı́nio, mas não terá F2′′′ (0).
E assim sucessivamente, podemos definir Fn , que vai bem até as derivadas de
ordem n, mas que não terá F (n+1) (0).

8. Exercı́cios
Exercı́cio 8.1. (resolvido)
O computador da as seguintes aproximações para:
π π
x1 := · (sin( ) + sin(π) ) = 1.570796327,
2 2
π π 2π
x2 := · (sin( ) + sin( ) + sin(π) ) = 1.813799365,
3 3 3
π π 2π 3π
x3 := · (sin( ) + sin( ) + sin( ) + sin(π) ) = 1.896118898,
4 4 4 4
π π 2π
x4 := · (sin( ) + sin( ) + . . . + sin(π) ) = 1.933765598.
5 5 5
i) qual uma possibilidade de termo geral da sequência xn da qual exibimos os
quatro primeiros termos ?
ii) Por quê os itens i) e ii) do Teorema 4.1 implicam que existe limn→∞ xn ?
Exercı́cio 8.2. Digo que g : I → R é uma função ı́mpar se g(x) = −g(−x) ∀x, −x ∈
I. E digo que é uma função par se g(x) = g(−x) ∀x, −x ∈ I.
Prove que:
i) Se f (x) é uma função ı́mpar, qualquer primitiva F (x) dela é uma função par.
ii) Se f (x) é uma função par, qualquer primitiva F (x) dela é uma função ı́mpar.
Dê exemplos onde f (x) é polinomial ou trigonométrica.
Exercı́cio 8.3. (resolvido)
i) Descreva a função F : [−1, 1] → R dada por
Z x
F (x) = | t |dt,
−1

onde | t | é o módulo.
Como é o gráfico de F (x) ?
Exercı́cio 8.4. Ao invés de ser 1 exercı́cio, este aqui serve de protótipo de uma
infinidade de exercı́cios.
R xuma função f : [a, b] → R contı́nua dada.
Suponha que você tem informação sobre
E considere a integral indefinida G(x) := a f (t)dt.
Suponha que te pedem pra encontrar máximos/mı́nimos de G(x).
Ataque o problema assim:
CAPÍTULO 21. INTEGRAÇÃO E O PRIMEIRO TEOREMA FUNDAMENTAL
299

• Note que G : [a, b] → R é contı́nua e que [a, b] fechado e limitado. Logo


existem máximos e mı́nimos globais da G(x).
• Esses pontos estão nos extremos a, b ou em (a, b).
• Mas os que estão em (a, b) são pontos crı́ticos da G, ou seja G′ (x) = 0 nesses
pontos.
• Ora, G′ (x) = f (x) e f foi dada.
Rx
Exercı́cio 8.5. Defina F : [0, π] → R como F (x) = 0 sin(t2 ) dt.
Usando o Primeiro Teorema do Cálculo, determine os 4 pontos de [0, π] onde
F ′ (x) = 0.
Um deles é ponto de mı́nimo global da F . Pelo Teste da segunda derivada, deter-
mine quais dos três outros são mı́nimos ou máximos locais.
Exercı́cio 8.6. (resolvido) Verifique que
x√ 1
F (x) = 1 − x2 + arcsin(x)
√ 2 2
2
é primitiva de y = 1 − x , para x ∈ [0, 1].
CAPı́TULO 22

Logaritmo natural e sua inversa, a exponencial

1. Existe uma função f 6≡ 0 que seja imune à derivação ?


Exceto pela função f ≡ 0, todas as funções que vimos até agora mudam ao serem
derivadas (os polinômios perdem grau, etc). Como poderı́amos criar uma função f (x)
imune à derivada ? Ou seja, com
f ′ (x) = f (x) ?
Imagine que tivéssemos uma função f : R> 0 → R com
1
f ′ (x) = .
x
Então f (x) > 0 ∀x ∈ R> 0 e daı́ f (x) é estritamente crescente. Logo f −1 : R → R>0

existiria e se fosse derivável, pelo Teorema 0.1 da derivada da inversa, terı́amos:


′ 1
(f −1 ) (x) = ′ −1 =
f (f (x))
1
= =
( f −11(x) )
= f −1 (x).

Ou seja (f −1 ) = f −1 : voilà a função imunizada.
Ou seja a sonhada função imune será a inversa daquela f (x) que tem f ′ (x) = x1 .
Mas será que já não temos uma função com f ′ (x) = x1 em nossa lista de funções
já conhecidas ?
Se quiséssemos ao invés de f ′ (x) = x−1 algo do tipo f ′ (x) = x−k , k 6= 1, bastaria
tomar
1
f (x) = · x−k+1
−k + 1
1
e pelo que já aprendemos f ′ (x) = x−k . Mas, justamente, não podemos escrever −k+1
se k = 1.

Assim como vimos que há leis fı́sicas importantes modeladas a partir da pro-
priedade f ′′ (x) = −f (x) do seno e do cosseno, há processos muito importantes mod-
elados matematicamente pela relação:
f ′ (x) = f (x).
Essa relação entre a derivada e a função diz por exemplo que quanto mais f (x) fica
positivo mais aumenta sua velocidade. É a modelagem de algum processo que tem
um crescimento extraordinário.
301
1. EXISTE UMA FUNÇÃO F 6≡ 0 QUE SEJA IMUNE À DERIVAÇÃO ? 302

Por exemplo, f (x) pode ser uma população em um certo tempo, e que quanto
mais elementos tem mais cruzamentos efetua, aumentando a população, e assim por
diante. Ou por exemplo uma dı́vida, sobre a qual incidem juros que aumentam a
dı́vida e sobre ela mais juros incidem, assim por diante.

1.1. Quantas funções são imunes à derivação ?


Acima propusemos um método para criar uma função imune à derivação (como
inversa de uma outa função) Chamemos nossa função imune f1 (x) (com f1′ (x) = f1 (x)
∀x portanto).
Suponhamos por um momento que f1 (x) nunca se anula (será verdade!).
Será que há alguma outra função f2 (x) com f2′ (x) = f2 (x) ∀x, bem diferente
da nossa f1 (x) e que quem sabe será criada por um outro método completamente
diferente desse nosso? A resposta é que essencialmente não !
E o argumento é o seguinte. Suponha outra f2 (x) com f2′ (x) = f2 (x) ∀x e defina:
f2 (x)
.
f1 (x)
Então a derivada do quociente dá:
f2 (x) ′ f ′ (x) · f1 (x) − f2 (x) · f1′ (x)
( ) (x) = 2 =
f1 (x) f12 (x)

f2 (x) · f1 (x) − f2 (x) · f1 (x)


=
f12 (x)
0
= ≡ 0.
f12 (x)
Mas então pela Parte 1 do Curso concluı́mos que
f2 (x)
≡C
f1 (x)
onde C é uma constante. Dito de outro modo f2 (x) = C · f1 (x) ou seja que f2 é
apenas f1 multiplicada por uma constante.
Note que se C = 0 então f2 (x) ≡ 0 é imune à derivação.
Então mãos à obra:
Definição 1.1. Considere a função
1
f : R>0 → R>0 , f (x) = .
x
A função de R>0 → R dada por
Z x
1
ln(x) := dx
1 x
é o logaritmo natural de x.
CAPÍTULO 22. LOGARITMO NATURAL E SUA INVERSA, A
EXPONENCIAL 303

Pelo Primeiro Teorema Fundamental(Teorema 6.1, Capı́tulo 21) ln(x) tem a pro-
priedade de que
1
ln′ (x) = ,
x
o que precisávamos.
Sua inversa (como ln′ (x) = x1 > 0, o ln(x) é uma função estritamente crescente)
então será a função imune a derivações.
Observe que:
• ln(1) = 0
• se 1 < x então ln(x) = A 1 ,1 (x) > 0.
x
• se x < 1 então
Z x Z 1
1 1
dx = − dx
1 x x x
R1
e x x1 dx = A 1 ,x (1) > 0 é uma área. Logo ln(x) < 0 se 0 < x < 1.
x
• como ln′′ (x) = − x12 < 0 é uma função com concavidade para baixo.
• na Afirmação 6.1 veremos que limx→+∞ ln(x) = +∞ e que limxց0 ln(x) =
−∞.
A importância prática dos logaritmos é enorme, devido a algumas propriedades
básicas que veremos nas próximas Seções.
Denoto a função inversa do logaritmo natual, definida de R → R>0 , por exp(y):

exp(ln(x))) = x, ∀x ∈ R>0 .

Em particular o número exp(1) será denotado por e, ou seja

ln(e) = ln(exp(1)) = 1.

A área sob o gráfico de x1 , desde 1 até 2, é menor que a área do quadrado de base
1 e altura 1. Logo
2 < e.
1
Considere agora a reta tangente ao gráfico de y = x
que passa pelo ponto (2, 12 ):
x
y = − + 1.
4
Ela passa por (1, 43 ) e por (3, 41 ). Então área sob o gráfico de x1 , desde 1 até 3, é maior
que a área do trapézio de base 2 formado pelos pontos (1, 43 ), (1, 0), (3, 0) e (3, 41 ).
Mas a área desse trapézio é a mesma do retângulo de base 2 e altura 12 (basta
pivotar no ponto (2, 21 ) a reta ligando (1, 43 ) e (3, 14 ), veja a Figura). Logo

e < 3.
2. PROPRIEDADES FUNDAMENTAIS DO LOGARITMO E DA
EXPONENCIAL 304

1
0,9
0,8
0,7
0,6
0,5
0,4
0,3
1 1,5 2 2,5 3
x

2. Propriedades fundamentais do logaritmo e da exponencial


Afirmação 2.1. No que segue x, x1 , x2 são positivos enquanto que y, y1, y2 são quais-
quer.
• i) ∀x1 , x2 > 0 vale ln(x1 · x2 ) = ln(x1 ) + ln(x2 ).
• ii) ∀x, ln( x1 ) = − ln(x).
m
• iii) ∀m, n ∈ N ln(x n ) = m n
· ln(x).
−m
−m
• iv) ∀m, n ∈ N ln(x n ) = n · ln(x).
• v) exp(y1 + y2 ) = exp(y1 ) · exp(y2 )
1
• vi) exp(−y) = exp(y) .
m m m
• vii) exp( n ) = exp(1) n = e n .
Demonstração.
De i):
Para recairmos em uma variável fixe x2 e olhe a função diferença:
φ(x1 ) := ln(x1 · x2 ) − ln(x1 ) − ln(x2 ),
como função de x1 apenas.
Temos pela regra da composta e pelo Primeiro Teorema Fundamental:
1 1
φ′ (x1 ) = · x2 −
x1 · x2 x1
onde derivei x1 ·x2 como função apenas de x1 , para cada x2 fixado, obtendo (x1 ·x2 )′ =
x2 . Ora então φ′ (x1 ) ≡ 0, portanto φ(x1 ) ≡ C.
Qual C ? Avalio em x1 = 1: φ(1) = ln(1x2 ) −0 −ln(x2 ) = 0, logo C = e φ(x1 ) ≡ 0
como querı́amos.
De ii):
Análoga à de i), derivando agora a função diferença
1
φ(x) := ln( ) + ln(x),
x
CAPÍTULO 22. LOGARITMO NATURAL E SUA INVERSA, A
EXPONENCIAL 305

que é:
(−1) 1
φ′ (x) = x · + ≡ 0.
x2 x
De iii):
Análoga, derivando agora:
m m
φ(x) := ln(x n ) − · ln(x),
n
−m m m −1 m −1
φ′ (x) = x n · ·xn − · x ≡ 0.
n n
De iv): sai de ii) e iii), já provadas.
De v):
Usando que exp é inversa de ln e a propriedade i) obtemos:

exp(y1 + y2 ) = exp(ln(x1 ) + ln(x2 )) = exp(ln(x1 · x2 )) =

= x1 · x2 = exp(y1 ) · exp(y2 ).
De vi):
Se aplicamos a v), já provada, para y1 = −y e y2 = y:

exp(−y + y) = exp(−y) · exp(y).


1
Mas exp(−y + y) = exp(0) = 1. Logo exp(−y) = exp(y) .
De vii):
Obviamente:
m m
ln(exp( )) = .
n n
Ou seja,
n m
· ln(exp( )) = 1.
m n
Por iii) temos então:
m n
ln(exp( ) m ) = 1.
n
Logo pela injetividade de y = ln(x):
m n
exp( ) m = exp(1),
n
ou seja:
m m
exp( ) = exp(1) n .
n

3. LOGA X , ∀A > 0 E LN | X | 306

3. loga x , ∀a > 0 e ln | x |
Podemos definir:

ln(x)
Definição 3.1. Defino ∀x > 0 e a > 0, a 6= 1, loga (x) := ln(a)

Na Biologia e na Quı́mica é importante a base 10, por exemplo.

Afirmação 3.1. Para x > 0 e a > 0, a 6= 1:

• o) loga (1) = 0 e loga (a) = 1.


1
• i) (loga (x))′ (x) = ln(a)·x , portanto loga (x) é estritamente crescente se a > 1
e loga (x) é estritamente decrescente se 0 < a < 1.
−1
• ii) (loga (x))′ (x) = ln(a)·x 2 , portanto o gráfico de loga (x) tem concavidade para

baixo se a > 1 e concavidade para cima se 0 < a < 1.


• iii) ∀x1 , x2 > 0 vale loga (x1 · x2 ) = loga (x1 ) + loga (x2 ).
• iv) ∀x, loga ( x1 ) = − loga (x).
m
• v) ∀m, n ∈ N loga (x n ) = m n
· loga (x).
−m
−m
• vi) ∀m, n ∈ N loga (x n ) = n · loga (x).
ln(a1 )
• vii) Se a1 , a2 > 0: loga2 (x) = ln(a 2)
· loga1 (x).
• viii): a função ln | x | está definida ∀x 6= 0 e sua derivada é (ln | x |)′(x) = x1

0
0,40,81,21,6 2
x
-1

-2

Figura: Gráficos de y = ln(x) (vermelho),


y = log0.5 (x) (verde) e y = log10 (x) (amarelo), x ∈ [0.1, 2].
CAPÍTULO 22. LOGARITMO NATURAL E SUA INVERSA, A
EXPONENCIAL 307

x
-4 -2 0 2 4
0

-2

-4

-6

Figura: O gráfico de y = ln | x |.

Demonstração. (da Afirmação 3.1)


De o):
ln(1) ln(a)
loga (1) := = 0, e loga (a) := = 1.
ln(a) ln(a)
1
De i): ao derivar a constante ln(a) sai.
De ii): derive a expressão de i).
De iii) páro x2 e considero a função diferença:
φ(x1 ) := loga (x1 · x2 ) − loga (x1 ) − loga (x2 ),
como função só de x1 .
Então já usando i) e a regra da composta:
1 1
φ′ (x1 ) = · x2 − ≡ 0.
ln(a) · x1 · x2 ln(a)x1
Logo
φ(x1 ) := loga (x1 · x2 ) − loga (x1 ) − loga (x2 ) ≡ C
e avaliando em x1 = 1 obtenho C = 0.
Deixo para o leitor a prova de iv) - vi), pois são análogas.
De vii): imediata, das definições.
De viii): se x > 0 já sabemos que ln′ (x) = x1 pelo Primeiro Teorema Fundamental do
Cálculo.
Se x < 0, então |x| := −x e temos pela regra da composta
1 1
(ln(−x))′ = · (−1) = , onde − 1 = (−x)′ ,
(−x) x
como querı́amos.

4. AS FUNÇÕES E X E AX , PARA A > 0 308

4. As funções ex e ax , para a > 0


Vimos no item vi) da Afirmação 2.1 que:
m m m
exp( ) = exp(1) n = e n , ∀m, n ∈ N
n
Isso motiva definir:
ex := exp(x), ∀x ∈ R.
Com essa definição e o item v) da Afirmação 2.1 temos garantida:
ex1 +x2 = ex1 · ex2 , ∀x1 , x2 ∈ R.
Definição 4.1. Para qualquer número Real positivo a > 0, defina:
ax := ex ln(a) .
Afirmação 4.1. Seja a número Real positivo.
• i) loga (ax ) = x.
• ii) ax1 +x2 = ax1 · ax2
• iii) (ax1 )x2 = ax1 ·x2
• iv) (ax )′ (x) = ln(a) · ax .
• v): ax é estritamente decrescente se a < 1, constante = 1 se a = 1 e ax é
estritamente crescente se a > 1.
• vi) os gráficos de ax sempre têm concavidade para cima.

10

0
-3 -2 -1 0 1
x

Figura: Os gráficos de y = ex em vermelho, de y = (0.5)x em verde


e de y = 10x em amarelo, x ∈ [−3, 1].

Demonstração.
De i):
ln(ax )
loga (ax ) := =
ln(a)
CAPÍTULO 22. LOGARITMO NATURAL E SUA INVERSA, A
EXPONENCIAL 309

ln(ex·ln(a) )
= = x.
ln(a)
De ii): Pela definição e pela propriedade de ex :

ax1 +x2 := e(x1 +x2 )·ln(a) = ex1 ·ln(a)+x2 ·ln(a) =

= ex1 ·ln(a) · ex2 ·ln(a) =: ax1 · ax2 .


De iii): Aqui uso duas vezes a definição :

(ax1 )x2 := (ex1 ·ln(a) )x2 :=


x1 ·ln(a) )
:= ex2 ·ln(e =

= ex2 ·x1 ln(a) =: ax1 ·x2 .


De iv): para derivar uso a regra da composta:

(ax )′ (x) := (ex ln(a) )′ (x) = ex ln(a) · ln(a) =: ln(a) · ax .


De v): O sinal de ax )′ (x) só depende do sinal de ln(a).
De vi): Devido a que:

(ax )′′ (x) = ln2 (a) · ax > 0, ∀x ∈ R




5. xa e sua derivada, a ∈ R.
Para sermos coerentes com a Definição 4.1 vamos definir:
Definição 5.1. Para x > 0 e a um Real qualquer, defino
ln(a)
xa := ea ln(x) e logx (a) := ,
ln(x)
onde x 6= 1 na última definição.

O leitor verá a importância dessas funções para resolver equações diferenciais na


Seção 1 do Capı́tulo 40.
Afirmação 5.1. Para x > 0 e a qualquer:
• i) (xa )′ (x) = a · xa−1
• ii) ln(xa ) = a · ln(x)
• iii) logx (xa ) = a.
6. CRESCIMENTO LENTO DO LOGARITMO E RÁPIDO DA EXPONENCIAL
310

Por exemplo, o gráfico de xπ é muito parecido com o de x3 , mas xπ só faz sentido
para x > 0:

0,8

0,6

0,4

0,2

0
0 0,2 0,4 0,6 0,8 1
x

Figura: O gráfico de y = xπ em vermelho e de y = x3 em verde, x ∈ (0, 1]

Demonstração.
De i):
a
(xa )′ (x) := (ea ln(x) )′ = ea ln(x) · = a · xa−1 .
x
De ii):
ln(xa ) := ln(ea ln(x) ) = a · ln(x).
De iii): Basta concatenar definições:
ln(ea ln(x) )
logx (xa ) := logx (ea ln(x) ) := = a.
ln(x)


6. Crescimento lento do logaritmo e rápido da exponencial


A Afirmação a seguir diz que o logaritmo natural cresce, mas cresce mais lenta-
mente até que y = x. E que, por outro lado, a exponencial cresce mais rápido que
qualquer n, n ∈ N:
Afirmação 6.1.
i) lim ln(x) = +∞, e lim ln(x) = −∞,
x→∞ xց0

ln(x)
ii) lim =0 e lim x · ln(x) = 0
x→∞ x xց0

Por outro lado, para qualquer n ∈ N:


xn
iii) lim = 0.
x→∞ ex
CAPÍTULO 22. LOGARITMO NATURAL E SUA INVERSA, A
EXPONENCIAL 311

Demonstração.
De i): Por definição ln(x) para x > 1 é a área sob o gráfico de x1 , de x = 1 até x.
Precisamos mostrar que à medida que x cresce a área cresce ano quanto quisermos.
Dito de outro modo, precisamos mostrar que a área sob o gráfico de x1 à direita de
x = 1 é tão grande quanto quisermos, desde que avancemos para a direita o suficiente.
Note que posso tomar os retângulos justpostos
1 1 1
[1, 2] × [0, ] ∪ [2, 3] × [0, ] ∪ . . . ∪ [n − 1, n] × [0,
2 3 n
cuja soma de áreas é
1 1 1
+ + ...+ .
2 3 n
Agora vamos ver que essa soma se faz tão grande quanto quisermos, quando n cresce,
o que implica que a área sob o gráfico à direita de 1 fica tão grande quanto quisermos.
De fato, denote:
1 1 1
sn := + + . . . +
2 3 n
e portanto com essa notação:
1 1 1 1 1 1 1
s2n := + ( + ) + ( + + + ) + . . . +
2 | 3 {z 4 } | 5 6 {z 7 8 }
21 parcelas 22 parcelas

1 1 1
+ ( n−1 + n−1 + ... n).
|2 + 1 2 {z + 2 2 }
2n−1 parcelas

Olhando para o menor termo em cada grupo destacado, acima, vemos que
1 1 1 2n−1 1
s2n ≥ + 2 · 2 + 22 · 3 + . . . + n = n · .
2 2 2 2 2
n
Ora como limn→+∞ 2 = +∞ obtemos que limn→+∞ s2n = +∞ e portanto limn→+∞ sn =
+∞. Isso diz que 21 + 31 + . . . + n1 fica tão grande quanto eu quiser, se n crescer o
suficiente.
Para vermos o que acontece com
lim ln(x)
xց0

note que
1
lim ln(x) = lim ln( ) =
xց0 z→+∞ z
= lim − ln(z) = − lim ln(z) = −∞.
z→+∞ z→+∞

De ii):
Só com a definição de ln(x) é imediato que:
ln(x) < x − 1, ∀x > 1,
pois x − 1 é quanto vale a área do retângulo de altura 1 e base [1, x].
6. CRESCIMENTO LENTO DO LOGARITMO E RÁPIDO DA EXPONENCIAL
312

E como x − 1 < x concluo:


0 < ln(x) < x, ∀x ≥ 1.
Por outro lado é claro que
1
x > 1 ⇔ x2 > 1
(passe da esquerda para a direita tirando a raı́z quadrada, e da dirita para a esquerda
elevando ao quadrado).
Ou seja:
1 1
0 < ln(x 2 ) < x 2 , se x > 1,
e pela propriedade do logaritmo:
1 1
0 < ln(x) < x 2 , se x > 1.
2
Agora eleve tudo ao quadrado obtendo:
(ln(x))2
0< < x, se x > 1
4
e daı́
ln(x) 4
0< < , se x > 1.
x ln(x)
Como sabemos que
4
=0 lim
x→+∞ ln(x)

fazendo x → +∞ na desigualdade obtemos:


ln(x)
0 = lim .
x→∞ x
Agora trato de
lim x · ln(x).
xց0

Note que:
ln(x) − ln(x) ln( x1 )
x · ln(x) = = = − .
( x1 ) ( −1
x
) ( 1
x
)
1
Se faço z := x
temos:
− ln(x) ln( x1 ) ln(z)
lim −1 = − lim 1 = − lim = 0,
xց0 (
x
) xց0 ( )
x
z→+∞ z
pelo que já sabemos de ii).
De iii):
Agora vamos ver que do ponto de vista de sua inversa temos o efeito contrário,
ou seja, que a exponencial cresce mais rápido que qualquer polinômio.
Como observamos acima, ln(x) < x − 1, se x > 1. Um tal x > 1 se escreve como
x = 1 + x com x > 0. Ou seja, obtenho:
ln(1 + x) < (1 + x) − 1 = x, se x > 0.
CAPÍTULO 22. LOGARITMO NATURAL E SUA INVERSA, A
EXPONENCIAL 313

Agora que já sei isso volto à notação anterior, escrevendo:


ln(1 + x) < x, se x > 0.
x
Já que isso vale ∀x > 0 uso para n+1
> 0 obtendo:
x x
ln(1 +)< , se x > 0.
n+1 n+1
Agora tomo exponencial, obtendo:
x x
1+ < e n+1
n+1
e portanto:
x x
< e n+1 .
n+1
Elevo tudo à n + 1:
x n+1 x
( ) < (e n+1 )n+1
n+1
x x
e usando a propriedade da exponencial (e m )m = em m = ex obtemos
xn+1
n+1
< ex , ∀x > 0
(n + 1)
e portanto
x
xn · < ex , ∀x > 0
(n + 1)n+1
e finalmente:
xn (n + 1)n+1
< , ∀x > 0.
ex x
Mas n é fixado e x cresce, logo:
xn
lim = 0,
x→+∞ ex

como querı́amos. 

7. Uma observação sobre o termo geral de uma série infinita


Vimos na prova do item i) Afirmação 6.1 que apesar de que:
1
lim =0
n→+∞ n
P+∞ 1
a série n=1 n fica tão grande quanto quisermos, ou seja,
+∞
X 1
= +∞.
n=1
n
8. UM PROBLEMA DA PUTNAM COMPETITON, N. 11, 1951 314

Definição 7.1. Diremos que uma soma infinita


+∞
X
an
n=1
converge se existe o limite
lim sn = L ∈ R,
n→+∞
onde a sequência sn é dada por:
sn := a1 + a2 + . . . + an .
P
Afirmação 7.1. Se a série infinita +∞n=1 an converge então necessariamente:
lim an = 0.
n→+∞

Demonstração.
Como
lim sn = L ∈ R,
n→+∞
então também vale:
lim sn−1 = L ∈ R.
n→+∞
Portanto pela propriedade do limite da diferença de duas sequências:
0 = lim (sn − sn−1 ) = lim an .
n→+∞ n→+∞

8. Um problema da Putnam Competiton, n. 11, 1951

Problema: Prove que vale:


1 1
ln(1 + ) > , ∀x > 0.
x 1+x

Solução:
Considere a função:
1 1
φ(x) := ln(1 + ) −
x 1+x
e note que
x+1 1 1
φ(x) = ln( )− = ln(x + 1) − ln(x) − .
x 1+x 1+x
Temos
lim φ(x) = +∞.
xց0
Portanto para x > 0 e pequeno vale φ(x) > 0.
Mas suponha por absurdo que para algum ponto x suficientemente grande aconteça
que
φ(x) ≤ 0.
CAPÍTULO 22. LOGARITMO NATURAL E SUA INVERSA, A
EXPONENCIAL 315

Como:
1 1 1 ′ 1
φ′ (x) = − −( ) =− <0
1+x x 1+x x · (1 + x)2
se x > 0 então φ(x) é uma função estritamente decrescente.
Portanto
φ(x) < φ(x) ≤ 0, ∀x > x.
Mas
1 1
lim φ(x) = lim [ln(1 + ) − ] = 0,
x→+∞ x→+∞ x 1+x
portanto não pode acontecer que
φ(x) < φ(x) ≤ 0, ∀x > x
pois os valores φ(x) têm que se aproximar de zero tanto quanto quisermos.
Essa contradição prova que φ(x) > 0 ∀x > 0, como querı́amos.

9. A regra de L’Hôpital
O Teorema de L’Hôpital é apresentado em muitos textos de Cálculo logo no inı́cio
e sem absolutamente nenhuma justificação.
É um exemplo tı́pico de um tópico de Matemática Superior ensinado do pior modo
possı́vel.
Teno visto alunos justificarem limites absolutamente simples como:
x2 + 1
lim = 1,
x→ +∞ x2
através do L’Hôpital decorado.
Por isso resolvi explicar (como se aprende no Spivak) pelo menos as formulações
mais fundamentais dessa regra.
A utilidade da regra de L’Hôpital é dar um critério para decidir o que acontece
quando, num quociente, tanto o numerador quanto o denominador tendem a zero.
Ou, como se diz, quando há uma indeterminação do tipo 00 .
Afirmação 9.1. (versão , 00 , x ∈ R, L ∈ R)
Sejam1 f : I \ {x} → R e g : I \ {x} → R onde I é um intervalo centrado em x.
Suponha:
• limx→x f (x) = limx→x g(x) = 0
• f ′ (x) e g ′ (x) estão definidas em I \ {x} e g ′ (x) 6= 0 em I \ {x}.
′ (x)
• limx→x fg′ (x) = L ∈ R.
Então:
• g(x) 6= 0 em I \ {x} e
• limx→x fg(x)
(x)
= L ∈ R.

O mesmo vale se nas hipótese e conclusões trocamos os limites plenos por algum
limite lateral como x ց x ou x ր x.
1 Dizer que uma função está definida em I \ {x} não quer dizer que ela também não possa estar
definida em x. Mas apenas que só precisamos que ela esteja definida num certo entorno de x.
9. A REGRA DE L’HÔPITAL 316

Demonstração.
Se f ou g não estão definidas em x ou mesmo se o valor de alguma delas em x
não é zero, redefina-as em x como:
f (x) = g(x) = 0,
2
deixando-as inalteradas em I \ {x}.
Com essa (re-)definição em x, as funções f, g são contı́nuas em x, ademais de
serem contı́nuas em I \ {x}, já que aı́ são até deriváveis.
Considere h > 0 pequeno para que
(x, x + h) ⊂ (I \ {x})
e note que g(x) não pode se anular em nenhum ponto x ∈ (x, x + h): caso contrário,
terı́amos g(x) = g(x) = 0 e o Teorema de Rolle aplicado ao intervalo [x, x] diria que
existe algum
ξh ∈ (x, x) ⊂ (I \ {x})
onde g (ξh ) = 0, contrariando uma hipótese de que g ′ (x) 6= 0 em todo I \ {x}.

Portanto faz sentido o quociente:


f (x)
, ∀x ∈ (x, x + h) ⊂ (I \ {x}).
g(x)
Agora aplico o T. V. Médio de Cauchy (Afirmação 1.3 Capı́tulo 10) a f, g restritas
ao intervalo [x, x] . Então existe
ϑx ∈ (x, x)
com :
f ′ (ϑx ) f (x) − f (x) f (x)
= = .
g ′(ϑx ) g(x) − g(x) g(x)
A hipótese
f ′ (x)
L = lim ′
x→x g (x)
f ′ (x)
diz que para qualquer tipo de ponto x que tende a x, o quociente g ′ (x)
tende a L.
Ora, quando x ց x temos ϑx ց x. Portanto
f ′ (x) f ′ (ϑx )
L = lim ′ = lim ′ .
x→x g (x) xցx g (ϑx )

Mas então
f ′ (ϑx ) f (x)
L = lim ′ = lim .
xցx g (ϑx ) xցx g(x)
f (x)
Analogamente para mostrar que L = limxրx g(x)
. 

Afirmação 9.2. (versão 00 , x = ∞, L ∈ R)


Suponha:
2Issonão vai alterar os cálculo dos limites, pois como sabemos limites só dependem do compor-
tamento em pontos próximos de x.
CAPÍTULO 22. LOGARITMO NATURAL E SUA INVERSA, A
EXPONENCIAL 317

• limx→+∞ f (x) = limx→+∞ g(x) = 0


• f ′ (x) e g ′ (x) estão definidas para x > K e g ′(x) 6= 0 para x > K.
′ (x)
• limx→+∞ fg′ (x) = L ∈ R.
Então:
• g(x) 6= 0 se x > K e
• limx→+∞ fg(x)
(x)
= L ∈ R.
Demonstração.
Vou fazer essa Afirmação recair na Afirmação 9.1 (para o limite lateral x ց x),
já provada.
Para isso defina:
1 1
fˆ(x) := f ( ) e ĝ(x) := g( ).
x x
Com essas definições, nossas hipóteses sobre f e g se traduzem nas seguintes hipóteses
sobre fˆ e ĝ:
• limxց0 fˆ(x) = limxց0 ĝ(x) = 0
f ′( 1 ) g′ ( 1 )
• fˆ′ (x) = − 2x e ĝ ′ (x) = − 2x estão definidas para x da forma 0 < x < 1 .
x x K
1
E ademais ĝ ′(x) 6= 0 se 0 < x < K
.
fˆ′ (x)
• limxց0 ĝ ′ (x)
= L ∈ R.
Então a Afirmação 9.1 (adaptada para limite lateral x ց 0) quando aplicada a fˆ
e ĝ e x = 0 dá que:
• ĝ(x) 6= 0 não se anula para 0 < x < K1
fˆ(x)
• limxց0 ĝ(x)
=L
Ou seja, g(x) 6= 0 se x > K e
f (x)
lim = L.
x→+∞ g(x)


Se examinamos as provas das duas Afirmações 9.1 e 9.2 vemos que valeriam
também se L = ∞. Nos referiremos a essas adaptações como versões 00 e L = ∞
do L ’Hopital.
Há também versões análogas, cuja prova exige algumas adaptações, para tratar
casos em que
lim |f (x)| = lim |g(x)| = +∞,
x→x x→x

ou como se diz, em que a indeterminação é do tipo ∞ .
Exemplos:
• Com a Afirmação 9.2 aplicada n + 1-vezes obtemos:
xn n · xn−1
lim = lim = ... =
x→∞ ex x→∞ ex
9. A REGRA DE L’HÔPITAL 318

n! 0
= lim = lim = 0.
x→∞ ex x→∞ ex
x
• Considere a composição ee . Vejamos que ela cresce mais rápido que a
própria exponencial. Pela Afirmação 9.2 adaptada para a indeterminação


se obtêm:
ex ex 1
lim x = lim ex x = lim ex = 0.
x→∞ ee x→∞ e · e x→∞ e

• quando numa expressão que é uma soma, uma parcela tende a +∞ e a outra
tende a −∞ nitidamente há uma indeterminação, chamada ∞−∞. Vejamos
um exemplo em que essa indeterminação se reduz a outra do tipo 00 , que pode
ser considerada via aplicação de L’Hôpital por duas vezes. Considere:
1 1 ex − 1 − x
lim ( − x ) = lim =
xց0 x e −1 xց0 x · (ex − 1)

ex − 1
= lim =
xց0 ex − 1 + x · ex
ex 1
= lim x = .
xց0 e + ex + x · ex 2
• quando numa expressão que é um produto, um fator tende a ∞ e o outro
tende a 0 nitidamente há uma indeterminação, chamada ∞ · 0. Vejamos um
exemplo em que essa indeterminação se reduz a outra do tipo ∞∞
, que pode
ser considerada via L’Hôpital. Considere:
ln(x)
lim ln(x) · tan(x) = lim =
xց0 xց0 ( 1 )
tan(x)

( x1 ) − sin2 (x)
= lim 2
sec (x)
= lim =
xց0 (− tan 2 (x) )
xց0 x
− sin(x)
= lim · sin(x) = −1 · 0 = 0.
xց0 x
• note que não há indeterminação nenhuma se ambas parcelas de uma soma
tendem a +∞ ou se ambas tendem a −∞.
• também não há indeterminação se numa soma ou subtração uma parcela
tende a zero e a outra também. Pois, se ǫ1 > 0 e ǫ2 > 0 são pequenos temos
|ǫ1 ± ǫ2 | ≤ ǫ1 + ǫ2 que é pequeno também.
Veremos na Seção 13 exemplos difı́ceis que precisam da regra de L’Hôpital.
Mas às vezes, em exemplos relativamente simples, não é claro se é mellhor usá-la
ou fazer diretamente. Por exemplo3:
√ √
lim a · x2 + b · x − a · x, a, b > 0.
x→+∞

Diretamente: √ √
lim ( a · x2 + b · x − a · x) =
x→+∞

3agradeço ao estudante Daniel Manica por este exemplo


CAPÍTULO 22. LOGARITMO NATURAL E SUA INVERSA, A
EXPONENCIAL 319
√ √
√ √
a · x2 + b · x + a · x
= lim ( a · + b · x − a · x) · ( √
x2 √ )=
x→+∞ a · x2 + b · x + a · x
b·x b·x
= lim √ √ = lim q √ =
x→+∞ 2
a · x + b · x + ax x→+∞
x · ( a + b + a) x

b b
= lim q √ = √ .
x→+∞
a+ b
+ a 2 · a
x

Agora via L’Hôpital para o tipo 00 :


r
√ √ b √
lim ( a · x2 + b · x − a · x) = lim x · ( a+ − a) =
x→+∞ x→+∞ x
q √ −2
a+ b
− a ( −b·x
√ )
x 2· a+ xb
= lim = lim =
x→+∞ x−1 x→+∞ −x−2
b b
= lim q = √ .
x→+∞
2· a+ b 2· a
x

10. A função xx
A função y = f (x) = xx está definida por:
xx := ex·ln(x) , ∀x ∈ R.
Afirmação 10.1. Para todo x > 0:
• i) (xx )′ = (ln(x) + 1) · xx .
• ii) a concavidade do gráfico de xx é para cima
• iii) xx tem um mı́nimo global em e−1 .
• iv) limxց0 xx = 1
x
• v) limx→∞ xe x = 0; em particular, limx→+∞ xx = +∞.

0,8

0,6

0,4

0,2

0
0 0,2 0,4 0,6 0,8 1
x

Figura: O gráfico de y = xx para x ∈ (0, 1]

Demonstração.
10. A FUNÇÃO X X 320

De i):
(xx )′ := (ex·ln(x) )′ (x) = ex ln(x) · (x · ln(x))′ = (ln(x) + 1) · xx .
De ii):
Basta notar que
1 x
(xx )′′ (x) = · x + (ln(x) + 1)2 · xx > 0, ∀x > 0.
x
De iii): Notar que:
(xx )′ = 0 ⇔ ln(x) + 1 = 0 ⇔ x = e−1
e usar ii).
De iv): Pela continuidade de ex :

lim ex ln(x) = elimxց0 x ln(x) .


xց0

Mas pelo item ii) da Afirmação 6.1,

lim x ln(x) = 0,
xց0

portanto
lim ex ln(x) = e0 = 1.
xց0

De v):
O item iii) da Afirmação 6.1 implica que limx→+∞ ex = +∞. E

ex ln(x) ≥ ex , se x ≥ e.
ex ∞
Portanto limx→∞ xx
é uma indeterminação ∞
. Uso então a Afirmação 9.2 adaptada

para ∞ :
ex ex
lim = lim .
x→∞ xx x→∞ ex·ln(x) · (ln(x) + 1)
Mas:
ex ex
lim ≤ lim =
x→∞ ex·ln(x) · (ln(x) + 1) x→∞ ex · (ln(x) + 1)

1
= lim = 0,
x→∞ ln(x) + 1
onde a desigualdade vale desde que x ≥ e.


A Figura a seguir ilustra onde xx passa a ser maior que ex


CAPÍTULO 22. LOGARITMO NATURAL E SUA INVERSA, A
EXPONENCIAL 321

25

20

15

10

0
0 0,5 1 1,5 2 2,5 3
x

Figura: Gráficos de y = xx em vermelho e y = ex em verde, x ∈ (0, 3]

11. Um problema da Putnam Competition, n. 22, 1961

Problema: A curva no plano definida por xy = y x , para x, y > 0, consiste de duas


componentes, uma que é uma reta e de uma outra curva.
Encontre as coordenadas do ponto de intersecção da reta com a outra curva.

Solução:
Vou me ater apenas à pergunta, sem tentar descrever em mais detalhes a curva
definida por xy = y x , para x, y > 0.
Em primeiro lugar a curva em questão é:
F (x, y) = xy − y x := ex ln(y) − ey ln(x) = 0.
É imediato que a reta diagonal faz parte desa curva, pois sobre a diagonal temos:
xy − y x = xx − xx = 0.
Supondo o que foi dito, que a reta diagonal corta uma segunda componente, nesse(s)
ponto(s) de interseção(ões) deve valer
∂F ∂F
=0 e = 0,
∂x ∂y
pois o Teorema 2.1 do Capı́tulo 15 diz que se
∂F ∂F
6= 0 ou 6= 0
∂x ∂y
então a curva F = 0 é localmente um gráfico regular e portanto, em torno de cada
ponto da diagonal F = 0 é exatamente um pedaço da reta diagonal.
Ora,
∂F y
= ex ln(y) · ln(y) − ey ln(x) ·
∂x x
∂F x
= ex ln(y) · − ey ln(x) · ln(x)
∂y y
12. UM MODO DE APROXIMAR E POR NÚMEROS RACIONAIS 322

que ao serem avaliadas em pontos da diagonal y = x dão:


x
ex ln(x) · ln(x) − ex ln(x) · = ex ln(x) · (ln(x) − 1)
x
e essa expressão se anula exatamente se:
ln(x) = 1,
ou seja, o ponto de intersecção é (x, y) = (e, e).

12. Um modo de aproximar e por números Racionais


Com um pouquinho de geometria básica conseguimos já determinar que:
2 < e < 3.
Agora vamos mostrar um modo de aproximar e com a precisão que quisermos:
Afirmação 12.1.
1
e = lim (1 + x) x
x→0
Em particular4,
1 n
e = lim (1 + ) , onde n ∈ N.
n→+∞ n
Demonstração.
Antecipando a próxima Seção, defino
1 1
(1 + x) x := e x ·ln(1+x) , x > −1.
Antes de passar ao limite x → 0, tomo o logaritmo natural:
1 1 1
ln( (1 + x) x ) = ln(e x ·ln(1+x) ) = · ln(1 + x).
x
e tento entender primeiro o que acontece com:
1
lim · ln(1 + x).
x→0 x
Ora,
1 ln(1 + x) − ln(1)
lim · ln(1 + x) = lim =:
x→0 x x→0 x
=: (ln(1 + x))′ (0) = 1.
Tomando a exponencial, que é contı́nua, concluo que
1 ln(1+x)
lim (1 + x) x = lim e x =
x→0 x→0
ln(1+x)
= elimx→0 x = e1 = e.
A segunda afirmação é apenas uma discretização desse fato, ou seja, onde o modo
como x → 0 é através da sequência de números Racionais n1 com n → +∞.


4Se pode provar, via o Cálculo, que e 6∈ Q, apesar de e poder ser aproximado por Racionais,
como diz esta afirmação
CAPÍTULO 22. LOGARITMO NATURAL E SUA INVERSA, A
EXPONENCIAL 323

Na Seção 5 do Capı́tulo 30 analisaremos uma aproximação mais eficiente de e.

13. Funções f (x)g(x) em geral e suas indeterminações


Que sentido dar a funções do tipo f (x)g(x) ? Já vimos alguns casos particulares.
Defino:
f (x)g(x) := eg(x)·ln(f (x)) , desde que f (x) > 0.
Com essa definição garantimos propriedades como:
ln(f (x)g(x) ) = ln( eg(x)·ln(f (x)) ) = g(x) · ln(f (x)),
bem como:
f (x)g(x)+h(x) = e(g(x)+h(x))·ln(f (x)) =
= eg(x)·ln(f (x)) · eh(x)·ln(f (x)) = f (x)g(x) · f (x)h(x) .
Exemplos de indeterminações:
• Note que podem aparecer indeterminações do tipo 1∞ , como já vimos no
1
caso (1 + x) x . Vejamos outro exemplo desse tipo:
1
lim (ex + x) x .
xց0

Tome o logaritmo:
1 1
ln((ex + x) x ) = · ln(ex + x)
x
e examine primeiro
ln(ex + x)
lim
xց0 x
0
como uma indeterminação 0 . Então:
ex +1
ln(ex + x) ( x )
lim = lim e +x = 2.
xց0 x xց0 1
Logo, tomando exponencial:
1
lim (ex + x) x = e2 .
xց0

• Existem também indeterminações ∞0 , como é o caso de


1
lim (ex + x) x .
x→+∞

Novamente tomo logaritmo:


1 1
ln((ex + x) x ) = · ln(ex + x)
x
e examine primeiro
ln(ex + x)
lim
x→+∞ x
como uma indeterminação ∞

. Então:
ex +1
ln(ex + x) ( x )
lim = lim e +x = 1
x→+∞ x x→+∞ 1
14. DERIVADA LOGARÍTMICA 324

e tomando exponencial obteremos:


1
lim (ex + x) x = e.
x→+∞

• Note que não existem indeterminações do tipo 0∞ : de fato, suponha f (x) > 0
com limx→x f (x) = 0. Se ademais limx→x g(x) = −∞, então:
lim f (x)g(x) := lim eg(x)·ln(f (x)) = +∞,
x→x x→x

enquanto que se vale limx→x g(x) = +∞ então:


lim eg(x)·ln(f (x)) = 0.
x→x

14. Derivada logarı́tmica


Se f (x) > 0 a derivada da composição ln(f (x)) é:
1
ln(f (x)) ′ = · f ′ (x).
f (x)
Note que o lado direito da expressão, ou seja,
f ′ (x)
f (x)
faz sentido mesmo se f (x) < 0, basta que não seja nula.
Definição 14.1. Seja f (x) qualquer função derivável. Onde ela não se anula, chamamos
a expressão
f ′ (x)
f (x)
de derivada logarı́tmica de f (x)

A Afirmação a seguir diz, do item i) ao iv) que a derivada logarı́tmica tem um


comportamento análogo ao do logaritmo, com respeito a produtos, quocientes e ex-
poentes.
O item v) dá a utilidade da derivada logaritmica, para calcular a própria f ′ (x),
quando f (x) envolve produtos, quocientese expoentes.
Afirmação 14.1. Sejam f, f1 , . . . , fn diversas funções da variável x, deriváveis e que
não se anulam na região considerada.
Então:
′ f1′ f1′
• i) (f(f11·f·...·f n)
2 ·...·fn )
= f1
+ . . . f1
,
(f n )′ f′
• ii) fn
= n· f
.
f1 ′
(f ) f1′ f2′
• iii) f
2
= f1
− f2
.
( f1 )
2
(f a )′ f′
• iv) para qualquer a ∈ R e f (x) > 0, fa
=a· f
.
CAPÍTULO 22. LOGARITMO NATURAL E SUA INVERSA, A
EXPONENCIAL 325

• v): suponha f (x) := f1a1 · . . . fnan , onde os expoentes ai são números Reais
quaiquer (suponha fi > 0 se for necessário). Então:

′ f1′ fn′
f (x) = f (x) · (a1 · + . . . + an · ).
f1 fn
Demonstração.
De i): Basta derivar o produto e simplificar:
(f1 · . . . · fn )′
=
(f1 · f2 · . . . · fn )

f1′ · f2 · . . . · fn f1 · . . . fn−1 · ·fn′


+ ...+ =
(f1 · f2 · . . . · fn ) (f1 · . . . · fn−1 fn )
f1′ f′
= + ... + n.
f1 fn
De ii): Uso a derivada da composta e simplifico:
(f n )′ n · f n−1 · f ′ f′
= =n· .
fn fn f
De iii): Uso a derivada do quociente e simplifico:
( ff12 )′ f1′ · f2 − f1 · f2′ f2
=( )· =
( ff21 ) f22 f1

f1′ · f2 − f1 · f2′ f′ f′
= = 1 − 2.
f1 f2 f1 f2
De iv): análoga à de ii), só que derivando a composição f (x)a := ea·ln(x) .
De v): basta usar os itens anteriores, pois f é definida através de produto/quocientes
e expoentes.


Exemplos:
• Suponha que te pedem para derivar
sin2 (x) · x3
f (x) = .
e2x
Com o item v) da Afirmação 14.1 se obtém:
sin2 (x) · x3 cos(x) 3
f ′ (x) = ( 2x
) · (2 + − 2) =
e sin(x) x

2 sin(x) · cos(x) · x3 + 3 · sin2 (x) · x2 − 2 · sin2 (x) · x3


= .
e2x
15. UMA FUNÇÃO EXTREMAMENTE ACHATADA 326
R
• como fazer tan(x) dx. Note que:
sin(x) f ′ (x)
tan(x) := dx = − ,
cos(x) f (x)
onde f (x) = cos(x). Então:
Z Z ′
f (x)
tan(x)dx = − dx =
f (x)
= − ln ||f (x)|| + C = − ln || cos(x)|| + C =
1
= ln( || cos(x)||−1 ) + C = ln( || || ) + C =
cos(x)
= ln || sec(x)|| + C.

15. Uma função extremamente achatada


As funções y = f (x) = xn com n ∈ N se anulam em x = 0 e tem até a derivada
de ordem n − 1 nula em x = 0:
f (0) = f ′ (0) = . . . = f (n−1) (0) = 0.
Quando n ∈ N cresce cada vez mais o gráfico dessas funções se achata cada vez mais
em torno ao x = 0:

0,8

0,6

0,4

0,2

0
-1 -0,5 0 0,5 1
x

Figura: Os gráficos de y = x2 (vermelho), y = x4 (verde)


e y = x6 (amarelo) para x ∈ [−1, 1].

Seria possı́vel uma função (diferente da função nula, obviamente) que tenha derivadas
de todas as ordens nulas em x = 0 ? Será que se todas as (infinitas !) derivadas são
nulas em x = 0 mesmo assim a função consegue decolar ?
Vamos ver que sim, usando o que aprendemos na Seção 6.
A função que consideraremos é:
−2 −1
f (x) = e−x = e x2 , se x 6= 0, e f (0) = 0.
Vou me contentar em mostrar que sua primeira e segunda derivada são zero na origem,
mas o leitor verá que o que uso para isso servirá em todas as derivadas.
CAPÍTULO 22. LOGARITMO NATURAL E SUA INVERSA, A
EXPONENCIAL 327

Para calcularmos sua derivada fora da origem podemos usar a regra da derivada da
composta. Mas para calcular sua derivada em x = 0 vamos precisar usar a definiçãod
e derivada: −2
′ e−h − 0
f (0) = lim .
h→0 h
Ora isso é o mesmo que:
1
′ h
f (0) = lim 1
h→0 e h2
1
e mudando de notação com z = h é o mesmo que
z
f ′ (0) = lim z 2
z→∞ e

(deverı́amos considerar separadamente o caso h ց 0 e z → +∞ e a outra possibilidade


h ր 0 e z → −∞, mas veremos que o resultado final não se altera). Mas vimos acima
que
z
lim z = 0
z→∞ e
z2 z
e portanto, como e > e se |z| > 1, com mais razão:
z
lim z 2 = 0
z→∞ e

logo f (0) = 0.
Agora para a segunda derivada, lembro a definição:
f ′ (h) − f ′ (0)
f ′′ (0) = lim .
h→0 h
Se h 6= 0, o valor de f ′ (h) é dado pela regra da composta:
−2
f ′ (h) = 2e−h · h−3 .
Logo:
−2
′′ 2e−h · h−3
f (0) = lim =
h→0 h
1
h4
=2 1 .
e h2
1
Agora com a notação z = h2
temos
z2
f ′′ (0) = lim ,
z→+∞ ez
e já vimos que
z2
lim =0
z→+∞ ez
logo
f ′′ (0) = 0.
Deixo como exercı́cio para o leitor mostrar, do mesmo jeito, que f ′′′ (0) = 0 e assim
sucessivamente.
O Maple dá ao seu gráfico o seguinte formato:
15. UMA FUNÇÃO EXTREMAMENTE ACHATADA 328

0,35

0,3

0,25

0,2

0,15

0,1

0,05

0
-1 -0,5 0 0,5 1
x

Fig.: Como o Maple representa a função extremamente achatada, x ∈ [−1, 1].

Mas note que parece que ela é zero em todo esse intervalo. Se diminuo o intervalo
ainda assim o gráfico dado pelo programa é enganador : parece que se anula ainda
em todo esse intervalo.

0,016

0,012

0,008

0,004

0
-0,4 -0,2 0 0,2 0,4
x

Figura: Assim o Maple representa a função extremamente achatada...

Por isso é sempre importante a teoria junto com o uso do computador pois sabemos
que a função
−2
f (x) = e−x , se x 6= 0, e f (0) = 0
só se anula em x = 0 !
Para terminar, um comentário.
Em geral, dada uma função f com todas as derivadas, onde f (x) = f (0) (x) é
derivada de ordem 0 e f (i) (x) é a de ordem i, a série:
+∞
X f (i) (0) i
x,
i=0
i!
é a chamada série de Taylor de f em x = 0 (continuo este tema na Seção 3 do
Capı́tulo 31)
No nosso caso como f (0) = f (i) (0) = 0, ∀i ∈ N, então a sua série de Taylor de f
em x = 0 é identicamente nula. Como cada série de Taylor converge em um intervalo
CAPÍTULO 22. LOGARITMO NATURAL E SUA INVERSA, A
EXPONENCIAL 329

(pode se degenerar a um ponto) teremos que dizer que a série de Taylor de nossa f
achatada converge em toda a reta.
Mas no entanto essa série só coincide com o valor da f em x = 0 !

16. Exercı́cios
Exercı́cio 16.1. Derive:

i) ex ln(x) , ii) x2 ln(x2 ) + x, iii) ln( x2 + 1),
2
iv) ln(x2 + 1), v) x2 ln(x), se x > 0, vi)ex ln(x) , vii) ln(x4 ),
1
viii) ln( ), 0 < x ≤ 1, ix) ln(x6 + 4x2 ).
x
Exercı́cio 16.2. (resolvido)
O programa Maple plota y = ln(1+x) x
para x ∈ [−0.9, 2]:

2,5

1,5

-0,5 0 0,5 1 1,5 2


x

sem se questionar sobre o que fazer em x = 0. Explique o que está acontecendo, com
os conceitos do Cálculo. Dica: Existe:
ln(1 + x)
lim ?
x→0 x
Quanto vale? Por quê ?
Exercı́cio 16.3. (resolvido)
Vimos dois fatos importantes do Cálculo:
ln(x)
lim ln(x) = +∞ mas lim = 0.
x→+∞ x→+∞ x
Ou seja que o logaritmo natural cresce, mas cresce mais lentamente que a própria
função y = x. A Figura mostra o gráfico de y = ln(x)
x
, para x ∈ [1, 10], onde se ve
ln(x)
que há um ponto de máximo, depois dele a função y = x vai caindo para cada vez
mais próximo do zero.
Determine o ponto de máximo de y = lnxx
.

0,35

0,3

0,25

0,2

0,15

0,1

0,05

0
2 4 6 8 10
x
16. EXERCÍCIOS 330

Exercı́cio 16.4. Vimos que que:


xn
lim ex = +∞ e ainda = 0, ∀n ∈ N.
lim
x→+∞ x→+∞ ex

Ou seja, que a exponencial cresce e cresce mais rapidamente que qualquer polinômio
xn .
n
A Figura mostra o gráfico de y = xex , para n = 2, 3 e para x ∈ [0, 4], onde se vê
que que cada um deles tem um ponto de máximo, depois dele a função vai caindo
ficando cada vez mais próxima de zero.
Para cada n fixado, determine em que intervalos a função:
xn
f : [0, +∞) → R, f (x) = x
e
é crescente, em que intervalo é decrescente e qual seu ponto de máximo (as respostas
são em função de n).

1,2

0,8

0,6

0,4

0,2

0
0 1 2 3 4
x

Exercı́cio 16.5. Derive:


2
i) ex ,
ii) ecos(x) ,
6
iii) ecos (x) ,
1
iv) exx , se x > 0,
v) etan(x) ,
ex
vi) ee .
2 x
Exercı́cio 16.6. Mostre que a derivada de ln( cosx2 (x)·e
·e
), para x ∈ (0, π2 ), é
2 2 sin(x)
+
1+ .
x cos(x)
Conclua daı́, sem fazer a derivada do quociente, que :
x2 · ex ′ 2 2 sin(x) x2 · ex
( ) = (1 + + ) · .
cos2 (x) · e x cos(x) cos2 (x) · e
Exercı́cio 16.7. Vamos definir as seguintes funções
ex − e−x ex + e−x
f1 (x) := e f2 :=
2 2
Prove que vale:
f2 (x)2 − f1 (x)2 ≡ 1, ∀x
de dois modos:
i) só fazendo contas que usam potências e produtos de exponenciais.
CAPÍTULO 22. LOGARITMO NATURAL E SUA INVERSA, A
EXPONENCIAL 331

ii) usando a filosofia do Cálculo, ou seja, de derivar uma função, ver que sua
derivada é zero, logo a função é constante e essa constante é zero.

Exercı́cio 16.8. Seja um k > 0. Prove a equivalência:

lim ekx = +∞ ⇔ lim e−kx = 0.


x→+∞ x→+∞

2) Os gráficos a seguir são de funções f (x) = f (0) · e−x , para diferentes valores de
f (0).
i) Confira que esses gráficos nunca se intersectam, mesmo quando x fica muito
grande.
ii) mostre que em todos esses gráficos as inclinações tendem a zero quando x
cresce.
iii) Calcule em cada x qual é quociente das inclinações de dois desses gráficos.

2,5

1,5

0,5

0
0 1 2 3 4
x

Exercı́cio 16.9. Prove que:

lim ln(xn ) − x = −∞, n ∈ N.


x→+∞

Dica: aplique exponencial para transformar a diferença num quociente. Depois volte
na expresssão original tomando logaritmo natural.

sin(x2 )
Exercı́cio 16.10. Seja f : [0, +∞) → R dada por f (0) = 0 e por f (x) = x
se
x > 0.
Prove que:

lim f (x) = 0, f ′ (0) = 1 e lim f ′ (x) = 1.


x→0 xց0
16. EXERCÍCIOS 332

A Figura a seguir plota em vermelho f e em verde f ′ para x ∈ [0, 5]:

x
0 1 2 3 4 5
0

-1

-2

Exercı́cio 16.11. Usando a Regra de l’Hôpital prove por indução em n ∈ N que:


(ln(x))n
lim = 0, ∀n ∈ N.
x→+∞ x
Exercı́cio 16.12. Usando L’ Hôpital prove que:
1
lim (1 + )x = 1.
x→0 x
Exercı́cio 16.13. (resolvido)
2
A função y = f (x) = e−x (vermelho), sua derivada f ′ (x) (verde) e sua segunda
derivada f ′′ (x) (amarelo) são dadas na Figura a seguir, para x ∈ [−2, 2]:

0,5
x
-2 -1 0 1 2
0

-0,5

-1

-1,5

-2

i) Calcule f ′ (x), f ′ (0), f ′′ (x) e f ′′ (0).


Note que o gráfico de f ′ (x) tem um máximo local e um mı́nimo local (que são
pontos de inflexão da f , portanto).
ii) Determine os pontos de mı́nimo/máximo locais de f ′ (x) resolvendo f ′′ (x) = 0.
Exercı́cio 16.14. (resolvido)
Prove que a tangente ao gráfico de y = ln(x) no ponto (e, 1) é uma reta que passa
pela origem. Dica: equação de uma reta dado um ponto e o coeficiente angular.
Então conclua, de preferência sem fazer contas, que a tangente ao gráfico de y = ex
no ponto (1, e) também é uma reta que passa pela origem.
CAPÍTULO 22. LOGARITMO NATURAL E SUA INVERSA, A
EXPONENCIAL 333

1
x
0,5 1 1,5 2 2,5 3 3,5 4
0

-1

-2

-3

-4

Exercı́cio 16.15. (resolvido)


Neste exercı́cio trata-se de encontrar primitivas sem ajuda de técnica nenhuma.
Tenha em mente que a primitiva de um produto não é o produto de primitivas.
Quando aparecer um produto f · g, lembre que a derivada da composta faz aparecer
produtos ! Por exemplo (sin(x2 ))′ = cos(x2 ) · 2x.
sin(x) cos(x)
i) , ii) x sin(x2 ) cos(x2 ),
6
2x + cos(x)
iii) 2 , se x2 + sin(x) ≥ 1,
x + sin(x)
1+x m
iv) , se x > 0, v) x n , m, n ∈ N, vi)2x cos(x2 ),
x
x 2
vii) cos(x2 ), viii) xex , ix) ex cos(ex ),
2
x)f (x) = a0 xn + a1 xn−1 + . . . + an , ai ∈ R,
20
4x3 + 4x x19 ex
xi) 4 , xii) ,
x + 2x2 + 1 20
1
ex
xiii) 2 , xiv) sin(x) sin(cos(x)),
x
20
x n 6x5 + 4x x19 ex
xv) (e ) , n ∈ N xvi) 6 , xvii)
x + 2x2 + 1 20
7
xviii) 7 , xix) cos(x) cos(sin(x)).
x
CAPı́TULO 23

Segundo Teorema Fundamental e Áreas

1. A descoberta de Gregory e Sarasa sobre área


A propriedade ln(xy) = ln(x) + ln(y), que vimos na Seção 2 do Capı́tulo anterior,
tem uma contrapartida geométrica interessante.
Suponha x ≥ 1 e y ≥ 1. Como xy ≥ x e as áreas as áreas sob o gráfico de x1 são
aditivas, podemos escrever:
A 1 ,1 (xy) = A 1 ,1 (x) + A 1 ,x (xy).
x x x

Mas
ln(xy) := A 1 ,1 (xy), ln(x) := A 1 ,1 (x) e ln(y) := A 1 ,1 (y).
x x x

Obtemos pela propriedade do logaritmo:


A 1 ,1 (x) + A 1 ,1 (y) = A 1 ,1 (x) + A 1 ,x (xy)
x x x x

e portanto:
A 1 ,1 (y) = A 1 ,x (xy).
x x

Por exemplo, com x = 2 e y = 2, A 1 ,1 (2) = A 1 ,2 (4) (quem consegue consegue intuir


x x
isso na Figura abaixo?)

1
0,9
0,8
0,7
0,6
0,5
0,4
0,3
1 1,5 2 2,5 3 3,5 4
x

1
Figura: As áreas sob x
entre 1 e 2 ou entre 2 e 4 são iguais !.

335
2. SEGUNDO TEOREMA FUNDAMENTAL DO CÁLCULO 336

Como se aprende no livro C.H. Edwards, The historical development of the Cal-
culus, Springer, 1979 esta propriedade
A 1 ,1 (y) = A 1 ,x (xy),
x x

foi observada por Gregory St. Vincent e A.A. Sarasa, antes do Cálculo.
Será que conseguimos verificar que
A 1 ,1 (y) = A 1 ,x (xy)
x x

diretamente, apenas com a definição de Área da Seção 1 do Capı́tulo 21 ?


Para definir A 1 ,1 (y) a primeira etapa é partimos o intervalo [1, y] em n subinter-
x
valos de tamanho y−1 n
, e levantarmos retângulos com altura f (x) = x1 , somando as
suas Áreas. Depois a segunda etapa é passar ao limite n → +∞.
Façamos a primeira etapa:
y−1 y − 1 −1 2(y − 1) −1 n(y − 1) −1
· [(1 + ) + (1 + ) + . . . + (1 + ) ].
n n n n
Por outro lado, a primeira etapa da definição de A 1 ,x (xy) é levantarmos retângulos
x
de base xy−x
n
e somarmos suas áreas, ou seja:
xy − x xy − x −1 2(xy − x) −1 x + n(xy − x) −1
· [(x + ) + (x + ) + ...+ ( ) ]=
n n n n
y − 1 −1 (y − 1) −1 2(y − 1) −1 n(y − 1) −1
= x· · [x · (1 + ) + x−1 · (1 + ) + . . . + x−1 · (1 + ) ],
n n n n
que, após cancelar x, dá o mesmo de antes ! Por isso ao passar ao limite n → +∞
dará o mesmo e:
A 1 ,1 (y) = A 1 ,x (xy).
x x

2. Segundo Teorema Fundamental do Cálculo


Teorema 2.1. Seja f : [a, b] → R contı́nua. Então
Z b
f (x)dx = F (b) − F (a),
a

onde F (x) é qualquer função com


F ′ (x) = f (x), ∀x ∈ [a, b].
Ou seja,dito de outro modo
Z b
F ′ (x)dx = F (b) − F (a).
a

Essa função F com F ′ (x) = f (x) ∀x é chamada de primitiva da f .

Demonstração.
Tome uma F (x) com F ′ (x) = f (x) ∀x ∈ [a, b] (não importa como se achou).
CAPÍTULO 23. SEGUNDO TEOREMA FUNDAMENTAL E ÁREAS 337

R x Agora lembre que o Primeiro Teorema Fundamental 6.1 diz que a função G(x) :=
a
f (x)dx tem
G′ (x) = f (x), ∀x ∈ [a, b].
Então
F ′ (x) = G′ (x), ∀x ∈ [a, b],
o que diz que
F (x) = G(x) + C, ∀x ∈ [a, b],
pelo Teorema Fundamental das Equações diferenciais (ver Capı́tulo 7 da Parte 1 deste
Curso). em particular:
F (b) = G(b) + C.
Ra
Mas que constante C é essa ? Temos que G(a) = a f (x)dx = 0, logo
F (a) = 0 + C,
ou seja C = −F (a) e
F (b) = G(b) − F (a)
e portanto:
Z b
G(b) := f (x)dx = F (b) − F (a),
a
como querı́amos.


Exemplo: Agora podemos justificar que


Z 2π
sin(x) dx = 0,
0

pois pelo Teroema 2.1:


Z 2π
sin(x)dx = − cos(2π) − (− cos(0)) = −1 + 1 = 0.
0

3. Regiões entre dois gráficos


Começo com um exemplo:√ determine a área da pétala compreendida entre os
gráficos de y = xn e y = n x para x ∈ [0, 1].
Há duas maneiras de ver essa pétala:

• como uma região abaixo do gráfico de y = n x e acima do de y = xn
• como formada por duas metades de pétalade mesma área. A metade inferior
determinada pela região entre o gráfico da diagonal y = x e o de y = xn . A
pétala tem simetria na reta diagonal.
3. REGIÕES ENTRE DOIS GRÁFICOS 338

Visto do primeiro modo, a área da pétala é uma diferença do tipo:


Z 1 Z 1
√n
x dx − xn dx =
0 0
Z 1 Z 1
1
= x dx −
n xn dx =
0 0
1+n
x n xn+1
= ( 1+n )(1) − 0 − ( (1) − 0) =
n
n+1
n 1 n−1
= − = .
n+1 n+1 n+1
Claro que se n = 1 a área é zero, pois a pétala degenera a um segmento de reta.
Note também que se fazemos n → +∞ obtemos como limite das áreas o valor
n−1
1 = lim ,
n→+∞ n + 1

que é a área do quadrado do qual a pétala vai se aproximando. Veja as Figura:

0,8

0,6

0,4

0,2

0
0 0,2 0,4 0,6 0,8 1
x


Figura: y = x2 , y = x e y = x, x ∈ [0, 1]
1

0,8

0,6

0,4

0,2

0
0 0,2 0,4 0,6 0,8 1
x


Figura: y = x3 , y = 3
x e y = x, x ∈ [0, 1]

Do segundo modo, que é o mais fácil, tomamos a área de metade da pétala e a


multiplicamos por 2:
Z 1
1
2·[ − xn dx] =
2 0
1 1
2·[ − ]=
2 n+1
2 n−1
=1− = .
n+1 n+1
Uma maneira mais geral de tratar a área da região compreendida entre dois
gráficos é dada a seguir:
CAPÍTULO 23. SEGUNDO TEOREMA FUNDAMENTAL E ÁREAS 339

Afirmação 3.1. Suponha f, g duas funções contı́nuas tais que no intervalo [a, b]
tenham:
f (x) ≥ g(x), ∀x ∈ [a, b].
Então a área da região, de x = a até x = b, abaixo do gráfico de f (x) mas acima
do gráfico de g(x) é dada por:
Z b
f (x) − g(x) dx.
a

Demonstração.
Suponhamos primeiramente o caso em que
g(x) ≥ 0, ∀x ∈ [a, b].
Então f (x) ≥ 0, ∀x ∈ [a, b], já que f (x) ≥ g(x).
Rb
Por um lado, a f (x) dx é a Área da região de x = a até x = b abaixo do gráfico
de f (x) e acima do eixo dos x, já que f (x) ≥ 0.
Rb
Enquanto que a g(x) dx é a Área da região de x = a até x = b abaixo do gráfico
de g(x) e acima do eixo dos x, já que g(x) ≥ 0.
Por uma propriedade da Integral:
Z b Z b Z b
f (x) − g(x) dx = f (x) dx − g(x) dx
a a a
Rb
e, como f (x) ≥ g(x), a f (x) − g(x) dx dá área da região de x = a até x = b, abaixo
do gráfico de f (x) mas acima do gráfico de g(x).
Agora, no caso geral, pode acontecer que g(x) < 0 para algum ponto no intervalo
[a, b].
Como g(x) é contı́nua, ela tem um valor mı́nimo global em [a, b]. Chame-o de
−C < 0. Então as novas funções
f (x) := f (x) + C e g(x) := g(x) + C
têm
g(x) ≥ 0, ∀x ∈ [a, b],
(se não fosse assim para algum x ∈ [a, b] então g(x) + C < 0 e g(x) < −C, con-
tradizendo a escolha de −C como mı́nimo da g) e
f (x) ≥ g(x), ∀x ∈ [a, b].

0
-1 -0,5 0 0,5 1
x
-1

-2
4. UM PROBLEMA DA PUTNAM COMPETITION, N. 54, 1993. 340

Figura: f vermelho, g verde, f amarelo, g azul, [a, b] = [−1, 1].

Pelo que já vimos no primeiro caso da demonstração, agora aplicado a f , g, o valor
de
Z b
f (x) − g(x) dx
a

dá a área da região de x = a até x = b, abaixo do gráfico de f (x) mas acima do


gráfico de g(x).
Como os gráficos de f (x) = f (x) + C e g(x) = g(x) + C diferem dos de f (x) e
g(x) apenas por uma translação vertical, então
Z b
f (x) − g(x) dx
a

dá a área da região de x = a até x = b, abaixo do gráfico de f (x) mas acima do


gráfico de g(x).
Finalmente:
Z b
f (x) − g(x) dx =
a

Z b
(f (x) + C) − (g(x) + C) dx =
a

Z b
= f (x) − g(x) dx, ,
a

o que conclui a demonstração.




4. Um problema da Putnam Competition, n. 54, 1993.

Problema 1: A reta horizontal y = C > 0 corta a curva y = 2x − 3x3 no primeiro


quadrante como na Figura abaixo.
Encontre o valor de C que faz com que as áreas das duas regiões delimitadas pelos
gráficos sejam iguais.
CAPÍTULO 23. SEGUNDO TEOREMA FUNDAMENTAL E ÁREAS 341

0,6

0,5

0,4

0,3

0,2

0,1

0
0 0,2 0,4 0,6 0,8
x

Aproveito para resolver um problema um pouco mais geral do que esse:

Problema 2: A reta horizontal y = C > 0 corta a curva y = A · x + B · x3 , com A > 0


e B < 0, no primeiro quadrante como na Figura (basta exigir A > 0 e B < 0 para
termos qualitativamente a mesma figura).
Encontre o valor de C que faz com que as áreas das duas regiões delimitadas pelos
gráficos sejam iguais.

Solução dos Problemas 1 e 2:


A igualdade de áreas das duas regiões delimitadas pelos gráficos siginifica, pela
Afirmação 3.1, que:
Z x
(A · x + B · x3 − C) dx = 0,
0
onde o limite de integração x é solução de:
A · x + B · x3 − C = 0.
Mas pelo Segundo Teorema Fundamental:
Z x
x2 x4
(A · x + B · x3 − C) dx = A · +B· − Cx
0 2 4
Ou seja, vemos que x satisfaz duas equações:
x2 x4
A · x + B · x3 − C = 0 e A · +B· − Cx = 0.
2 4
A primeira dá C = A·x+B ·x3 , que pode ser substuı́do na segunda, dando a equação:
A 3B 2
x2 · (− − · x ) = 0.
2 4
Como certamente x 6= 0, então:

2· A
x=√ √ √ ,
2 3 −B
onde lembre que A > 0 e B < 0.
4. UM PROBLEMA DA PUTNAM COMPETITION, N. 54, 1993. 342

Agora
√ √
2· A 2· A
C = A · (√ √ √ ) + B · (√ √ √ )3 =
2 3 −B 2 3 −B
√ √ √
A3 · 2 3
= √ .
9 −B
No caso particular do Problema 1, onde A = 2 e B = −3 obtemos então
2 4
x= e C= .
3 9
Veja a Figura a seguir:

0,6

0,5

0,4

0,3

0,2

0,1

0
0 0,2 0,4 0,6 0,8
x

No Livro do Anton, Calculo v. 1, Exercı́cio 40 da Seção 7.1, ele propõe uma


variante desse problema, o Problema 3. Porém como o gráfico não é mais de função
polinomial a resposta não é exata, mas sim aproximada:

Problema 3: A reta horizontal y = C, C > 0 corta y = sin(x), com x ∈ [0, π], em


dois pontos.
Encontre o valor de C que faz com que as áreas das duas regiões delimitadas pelos
gráficos sejam iguais.

Solução do Problema 3:
Como antes, a igualdade de áreas quer dizer:
Z x
sin(x) − C dx = 0.
0

Pelo Segundo Teorema do Cálculo:


Z x
sin(x) − Cdx = (− cos(x) − Cx) − (− cos(0) − 0) =
0

= − cos(x) − Cx + 1.
CAPÍTULO 23. SEGUNDO TEOREMA FUNDAMENTAL E ÁREAS 343

Ou seja, x satisfaz as equações:


− cos(x) − Cx + 1 = 0 e sin(x) − C = 0.
A segunda dá C = sin(x) que colocado na primeira dá:
− cos(x) − sin(x) · x + 1 = 0.
Portanto preciso resolver esta equação e, de posse desse resultado, basta fazer C =
sin(x) para terminar o Problema.
A solução que daremos desta equação não será exata, mas sim aproximada. Pelo
Método de Newton, que foi exposto no Capı́tulo 18, o resultado que se obtém é
x ≈ 2, 33112237 e C ≈ 0, 7246113541.
Veja a Figura a seguir:

1
0,8
0,6
0,4
0,2
0
0 0,5 1 1,5 2 2,5 3
x

5. Integral e centro de gravidade


Quando descrevemos o efeito da gravidade sobre objetos, fizemos, e o faremos
mais algumas vezes neste Curso, a super simplificação de considerar esses objetos
como sendo pontos.
Suponhamos, um pouquinho mais realisticamente, que o objeto tenha pelo menos
dimensão 1 ou seja, seja dado por um intervalo [a, b] e que sua densidade ρ(x) dependa
de cada ponto x ∈ [a, b].
A massa do objeto [a, b] é então dada por:
Z b
m= ρ(x) dx.
a
A lei de Newton se expressa para [a, b] então como:
Z b Z b
F = ρ(x) dx · g = ρ(x) · g dx.
a a
Por outro lado, num objeto 1-dimensional do tipo [0, r] a grandeza interessante é
o momento em torno de 0 produzido pela força gravitacional. Essa grandeza não
5. INTEGRAL E CENTRO DE GRAVIDADE 344

depende somente do peso concentrado numa região mas da distância dela até 0 (por
isso é mais fácil abrir uma porta segurando pelo trinco do que junto da dobradiça).
Para um ponto x ∈ [0, r] com massa mx o momento em torno de 0 é definido
como:
mx · g · x.
É natural, num objeto do tipo [0, r], de densidade variável ρ(x), definir o momento
produzido pela gravidade por:
Z r
M := ρ(x) · g · x dx,
0

pois essa integral pode ser considerada limite de somas de Riemann do tipo:
n
X
ρ(xi ) · g · xi .
i=1

Quando fazemos a simplificação de pensar que o objeto não-pontual é pontual,


estamos concentrando todos o efeito da gravida sobre um ponto x ∈ [0, r]. Ou seja,
fazemos
M := F · x,
que significa:
Z r Z b
ρ(x) · g · x dx = ρ(x) · g dx · x,
0 a

ou seja:
Rr
0
ρ(x) · x dx
x= Rb .
a
ρ(x) dx
Exemplos:
• Se a densidade ρ(x) ≡ ρ é constante para o objeto [0, r] então:
Rr r2
ρ · 0 xdx r
x= R r = 2 = ,
ρ · 0 dx r 2
r
que é o ponto médio de [0, r]. O Exercı́cio 7.2 mostra que x = 2
pode
acontecer mesmo se ρ(x) não é constante.
• Se defino ρ(x) := C · x então:
Rr
C · x2 dx 2
x = R0 b = · r,
C · x dx 3
a

ou seja, o centro de gravidade se desloca do ponto médio para um ponto


situado a 32 do comprimento r do segmento.
Voltaremos a esses dois últimos exemplos na Seção 6.
CAPÍTULO 23. SEGUNDO TEOREMA FUNDAMENTAL E ÁREAS 345

6. Arquimedes e a parábola: prova versus heurı́stica


Na antiguidade se discutia o problema da quadradura de figuras planas. Ou seja,
de obter figuras retangulares ou triangulares com a mesma área que uma figura cur-
vada dada.
Na Afirmação a seguir damos uma prova completamente automática (graças ao
Teorema Fundamental do Cálculo) de um teorema de Arquimedes:
Afirmação 6.1. Seja a parábola y = C · x2 , com C > 0 e a reta y = a · x + b com
a, b > 0. Sejam P1 := (x1 , y1 ) e P2 ; = (x2 , y2 ) os dois pontos de intersecção da reta
com a parábola.
Seja P3 = (x3 , y3) ponto da parábola que tem reta tangente paralela ao segmento
P1 P2 . Então a área do setor compreendido entre a reta e a parábola é 34 da área do
Triângulo ∆P1 P2 P3 .
A Figura ilustra as hipóteses do Teorema:

0
0 0,5 1 1,5 2

-1 x

Demonstração.
As coordenadas x1 , x2 são as soluções de:
C · x2 − a · x1 − b = 0,
ou seja: √ √
a2 + 4Cb
a− a+ a2 + 4Cb
x1 = e .
2C 2C
O ponto P3 tem coordenada x3 que verifica
2 · C · (x3 ) = a,
ou seja,
a a
P3 = ( C · ( )2 ).
2C 2C
Note que então
x1 + x2 y1 + y2 a2 + 4 · b · C
x3 = e y3 = − .
2 2 4C
6. ARQUIMEDES E A PARÁBOLA: PROVA VERSUS HEURÍSTICA 346

A área do triângulo ∆P1 P2 P3 pode ser calculada como 21 ||D|| onde D é o determinante:

x1 y1 1

D = x2 y2 1
x3 y3 1
Esse determinante se calcula fácil, pois pela propriedade do determinante:

x1 y1 1
x1 y1 1

x2 y2 1 =
x 2 y 2 1 =

x3 y3 1 x3 − x +x y +y 1+1
1
2
2
y3 − 2 1 2
1− 2

x1 y1 1 3
a2 + 4 · b · C (a2 + 4Cb) 2
= 2
x y 2 1 = (x1 − x2 ) ·
=−
0 − a2 +4·b·C 0 4C 4C 2
4C
de onde:
3
1 (a2 + 4Cb) 2
||D|| = .
2 8C 2
Por outro lado a área compreendida entre a reta e a parábola é:
Z x2 3
2 (a2 + 4Cb) 2
(a · x + b − C · x ) dx = .
x1 6C 2
O que querı́amos.


A prova original de Arquimedes é totalmente diferente, lida com somas infinitas.


Mas a grande questão é:
Como foi que ele imaginou, conjecturou, que existia essa relação tão precisa entre
as duas áreas ?
Isso é parte da heurı́stica, a arte/ciência de se descobrir candidatos a teoremas,
ou seja, conjecturas razoáveis que depois se prova rigorosamente.
Um pouco da heurı́stica de Arquimedes pode ser explicada se consideramos uma
situação mais simples que a da Afirmação 6.1, mas claramente muito relacionada com
ela.
Imagine o triângulo ∆ formado pelos três pontos (0, 0), (x, 0), (x, C · x), onde
C > 0. Sua base é o segmento (0, 0) (x, 0), com ângulo reto em (x, 0), e sua altura é
C · x. Denote
x·C ·x
A∆ =
2
sua área.
E considere também o gráfico da parábola y = C · x2 para x ∈ [0, x]. Denote por
A a área da região sob o gráfico da parábola e acima do eixo dos x, para x ∈ [0, x]
Vamos ver qual a heurı́stica de Arquimedes para conjecturar que
2 2 C · x2 C · x3
A= · x · A∆ = · x · = .
3 3 2 3
CAPÍTULO 23. SEGUNDO TEOREMA FUNDAMENTAL E ÁREAS 347

Ele pensa numa figura plana como sendo um objeto de espessura negligenciável,
com densidade constante (vamos supor = 1), para o qual o peso é proporcional à
área. O intervalo [0, x] para ele é uma alavanca apoiada no (0, 0) que sofre o efeito
do peso do triângulo ∆. Sobre cada ponto x ∈ [0, x] há uma fatia (infinitamente fina)
do triângulo, de peso C · x · g. Dessa forma o momento relativo a (0, 0) produzindo
pelo peso da fatia acima de x ∈ [0, x] é:

x · (C · x · g).

Mas obviamente vale a igualdade

x · (C · x · g) = 1 · (C · x2 · g)

e portanto o momento produzido pela fatia de ∆ sobre x é igual ao momento produzido


pelo peso da fatia da parábola sobre x colocada a distância 1 da origem. Por exemplo
na posição (−1, 0) de uma alavanca [−1, 1] que se apoia em 0.
Como fatia por fatia estabelecemos uma igualdade de momentos, concluimos que
o momento exercido pelo triângulo ∆ todo é igual ao de toda a região sob a parábola
se fosse pendurada no ponto (−1, 0). A alavanca ficaria assim em equilı́brio, veja a
Figura:

Mas Arquimedes sabia que, quando se trata do efeito da gravidade, pode-se sub-
stituir ∆ todo por um ponto, pelo seu baricentro B.
Como vimos na Seção 4 do Capı́tulo 7, o baricentro se encontra a 32 da distância
entre o vértice e o ponto médio do lado oposto.
Como consequência do Teorema de Tales, a projeção vertical de B no intervalo
[0, x] é o ponto ( 2x
3
, 0): portanto podemos pensar que todo o peso do triângulo é
exercido nesse ponto, produzindo um momento relativo a (0, 0) da ordem de

2
· x · A∆ · g.
3
7. EXERCÍCIOS 348

O B

Pelo equilı́brio da alavanca [−1, 1] que já tinhamos obtido, concluimos que:
2x
1·A·g = · A∆ · g,
3
ou seja:
2
A = · x · A∆ ,
3
como querı́amos.
Vejamos ainda de outro modo a heurı́stica de Arquimedes.
A área do triângulo e a área da região sob a parábola são, na nossa linguagem:
Z x Z x
2
A := C · x dx e A∆ = C · x dx.
0 0
O que queremos entender é de onde saiu a conjectura:
Rx
C · x2 dx 2x
R0 x = .
0
C · x dx 3
Agora lembre, da Seção 5, que:
Rx
C · x2 dx
x = R0 x
0
C · x dx
é o centro de gravidade do objeto unidimensional [0, x] cuja função de densidade é
ρ(x) := C · x.
Essa função ρ(x) associaria a cada ponto no intervalo [0, 1] uma massa/peso corre-
spondente à altura do segmento vertical sobre x que faz parte do triângulo ∆.
Foi isso que Arquimedes fez !

7. Exercı́cios
Exercı́cio 7.1. O seguinte caso particular do Teorema de Arquimedes pode ser feito
sem dificuldade.
Seja um parábola y = Cx2 , C > 0 e a reta horizontal y = b, que a intersecta em
dois pontos P1 e P2 . Denote a origem por O = (0, 0). Então a área da região abaixo
da reta e acima da parábola é exatamente 43 da área do triângulo ∆P1 OP2 .
Exercı́cio 7.2. Considere um objeto 1-dimensional, que é um intervalo [0, r].
Suponha que sua densidade é dada por ρ(x) = r · x − x2 .
i) Mostre, calculando integrais, que o centro de gravidade x ainda é o ponto médio
r
2
.
CAPÍTULO 23. SEGUNDO TEOREMA FUNDAMENTAL E ÁREAS 349

ii) encontre uma explicação conceitual para i), que permitirá gerar outras funções
ρ(x) para as quais ainda x = r2 .

Exercı́cio 7.3. Usando o Segundo Teorema Fundamental do Cáculo determine a área


1
compreendida entre os gráficos de y = x3 e de y = x 3 .

1,5

0,5

0
0 0,2 0,4 0,6 0,8 1 1,2
x

Obs. Nesse tipo de questão é preciso verificar onde os gráficos se intersectam e


qual gráfico está por cima do outro.

Exercı́cio 7.4. (resolvido)


Determine a área da região em forma de (meia) pétala compreendida entre o
gráfico de y = 8x + 2 e o gráfico de y = x4 + 2.

Exercı́cio 7.5. (resolvido) √


−2+ 22
É um fato que para b = 3
∼ 0, 9 vale:
Z b
x − x2 − x3 dx = 0.
0

Interprete isso geometricamente, como sendo equivalente a uma igualdade entre duas
Áreas de duas regiões comprendidas
√ entre gráficos de certas funções.
Dica: podes ser útil saber que 5 ∼ 2.2.

Exercı́cio 7.6. Através do Teorema Fundamental, determine a área da região com-


preendida entre os gráficos de y = x2 e y = −x2 + 8.

Exercı́cio 7.7. Encontre a reta y = a · x adequada para que a área compreendida


entre seu gráfico e o de y = x2 seja exatamente 1. Dica: vá té o fim sem determinar
o a, ao final, peça que a área seja 1 e obtenha assim o a.

0
0 0,5 1 1,5 2
x

Exercı́cio 7.8. (resolvido)


7. EXERCÍCIOS 350

Determine o valor adequado de a para que a área da região comprendida entre os


gráficos de y = x4 e y = a seja exatamente A = 1.

1,5

0,5

0
-1 -0,5 0 0,5 1
x

Exercı́cio 7.9. A figura a seguir mostra os gráficos de y = xn , para n = 1, 2, 3, 4, 5, 6,


na região x ∈ [0, 1].
i) na região x ∈ [0, 1] o gráfico de y = xn está por cima ou por baixo do de
y = xn+1 ?
ii) Determine para qual n a região compreendida entre os gráficos de y = xn e
1
y = xn+1 tem área exatamente igual a 12 .
1

0,8

0,6

0,4

0,2

0
0 0,2 0,4 0,6 0,8 1
x

Exercı́cio 7.10. A figura a seguir mostra os gráficos de y = xn − xn+1 , para n =


1, 2, 3, 4, x ∈ [0, 1]. Determine para qual n a região sob o gráfico de y = xn − xn+1
1
tem área 20 .
0,25

0,2

0,15

0,1

0,05

0
0 0,2 0,4 0,6 0,8 1
x

Exercı́cio 7.11. A figura a seguir mostra os gráficos de y = fn (x) := xn − x2n , para


n = 1, 2, 3, 4, no domı́nio x ∈ [0, 1] (que se parecem com chicotes):
0,25

0,2

0,15

0,1

0,05

0
0 0,2 0,4 0,6 0,8 1
x

i) Calcule fn′ (x), ∀n ∈ N.


ii) Determine a equação y = ax + b da reta tangente ao gráfico de fn (x) no ponto
(1, 0).
CAPÍTULO 23. SEGUNDO TEOREMA FUNDAMENTAL E ÁREAS 351

iii) Explique o que acontece com os coeficientes angulares das retas de ii), quando
n cresce.
iv) Se vê que cada y = fn (x) tem um ponto de máximo em seu domı́nio [0, 1].
Determine-o (claro dependendo de n).
v) todas as fn valem o mesmo nos seus pontos de máximo, quanto ?
vi) Determine a área An da região sob o gráfico de y = fn (x) = xn − x2n , de x = 0
até x = 1.
vii) A quanto tendem essas áreas quando n aumenta? Ou seja, qual o
lim An ?
n→+∞

Exercı́cio 7.12. A figura a seguir mostra os gráficos de y = fn (x) := x − x2n+1 , para


n = 3, 6, 10, 50, x ∈ [0, 1]:

0,8

0,6

0,4

0,2

0
0 0,2 0,4 0,6 0,8 1
x

i) Calcule fn′ (x), ∀n ∈ N.


ii) Determine as equações y = ax + b das retas tangentes ao gráfico de fn (x) no
ponto (0, 0), ∀n.
iii) Determine as equações y = ax + b das retas tangentes ao gráfico de fn (x) no
ponto (1, 0), ∀n.
iv) O que acontece com as retas dos itens ii) e iii), quando n → +∞ ?
v) Se vê que cada y = fn (x) tem um ponto de máximo em [0, 1]. Determine-o
(dependendo de n).
vi) Determine a área An da região sob o gráfico de y = fn (x) = x − x2n+1 , de
x = 0 até x = 1.
vii) O que acontece com An quando n → +∞, ou seja, existe o limn→+∞ An ? Se
existe quanto é ?
CAPı́TULO 24

Integração por partes

Vamos explicar agora uma técnica útil para encontrar primitivas de funções e
expressá-las concretamente como funções.
Lembro primeiro que criamos uma função completamente nova ao fazermos
Z x
1
ln(x) := dx.
1 x
Rx
Uma pergunta
Rx natural é: será criamos algo radicalmente novo se fazemos a ln(x)dx
ou essa a ln(x)dx se pode expressar através de funções conhecidas ?
Veremos que sim, se pode expressar através de funções conhecidas, de fato:
Z x
ln(x) dx = x ln(x) − x + C.
a
Verificamos facilmente que (x ln(x) − x + C)′ = ln(x).
Mas como chegamos numa primitiva dessas? Há alguma técnica ? O Teorema
a seguir dá uma técnica útil, embora à primeira vista não pareça, para encontrar
primitivas:
′ ′
Teorema R0.1. Sejam f e g definidas
Rx num intervalo,
R x com f′ e g funções contı́nuas.
x ′
Então a f (x) · g(x)dx = a f (x) · g(x)dx − a f (x) · g (x)dx.
Demonstraç
R ão.x
Note que ( a (f (x) · g(x))′dx)′ (x) = (f (x) · g(x))′(x) pelo Primeeiro Teorema Fun-
damentalRdo Cálculo.
x
Logo a (f (x) · g(x))′ dx = f (x) · g(x) + C pelo Teorema Fundamnal da Equações
Diferenciais.
Mas pela derivado do produto:
(f (x) · g(x))′ = f ′ (x) · g(x) + f (x) · g ′ (x).
Logo pelas propriedades aditivas da integral:
Z x Z x

(f (x) · g(x)) dx = (f ′ (x) · g(x) + f (x) · g ′(x))dx =
a a
Z x Z x

= f (x) · g(x)dx + f (x) · g ′(x)dx
a a
e portanto:
Z x Z x

f (x) · g(x)dx = f (x) · g(x) − f (x) · g ′(x)dx + C
a a
353
354

como querı́amos 

Vamos aplicá-lo nos exemplos a seguir, onde se vê que


• cuidado ao escolher quem fará o papel de f ′ e quem será g
• pode ser preciso usá-lo mais de uma vez
R
Exemplo 0.1. i) ln(x) dx:
Z Z
1
1 ln(x) dx = x ln(x) − x dx =
| {z } | {z } x
|{z}
f ′g fg
f g′

= x ln(x) − x + C.
R
ii) x ln(x) dx:
Z Z
x2 x2 1
x ln(x) dx = ln(x) − dx =
| {z }
f ′g
|2 {z } 2 x
|{z}
fg f g′

x2 x2
= ln(x) − + C.
R 2 4
ln(x)
iii) x
dx:
Z Z
1 1
ln(x) dx = ln(x) ln(x) − ln(x) dx.
|x {z } | {z }
fg
| {z x}
f ′g f g′
Logo: Z
ln(x)
2· dx = ln2 (x) + C
x
ou seja
Z
ln(x) ln2 (x)
dx = + C,
x 2
R
( 21 · C é outra constante, mas que sigo chamando de C). iv) ln(x) x2
dx:
Z Z
1 −1 −1 1
2
ln(x) dx = ln(x) − dx =
x
| {z } x
| {z } x x
| {z }
f ′g fg f g′
Z
− ln(x) 1
= + dx =
x x2
− ln(x) 1
= − + C.
R x x
v) cos2 (x) dx:
Z Z
cos(x) cos(x) dx = sin(x) cos(x) − sin(x)(− sin(x)) dx =
| {z } | {z } | {z }
f ′g fg f g′
CAPÍTULO 24. INTEGRAÇÃO POR PARTES 355
Z
= sin(x) cos(x) + sin2 (x)dx =
Z
= sin(x) cos(x) + (1 − cos2 (x))dx =
Z
= sin(x) cos(x) + x + C − cos2 (x)dx.
Logo Z
2· cos2 (x)dx = sin(x) cos(x) + x + C
e portanto: Z
sin(x) cos(x) + x
cos2 (x)dx = + C.
2
R
vi) cos3 (x) dx:
Z Z
2 2
cos(x) cos (x) dx = sin(x) cos (x) − sin(x)(−2 cos(x) sin(x)) dx =
| {z } | {z } | {z }
f ′g fg f g′
Z
= sin(x) cos (x) + 2 sin2 (x) cos(x)dx =
2

Z
= sin(x) cos (x) + 2 (1 − cos2 (x)) · cos(x)dx =
2

Z Z
= sin(x) cos (x) + 2 cos(x)dx − 2 cos3 (x)dx.
2

Logo
Z Z
3 2
3· cos (x)dx = sin(x) cos (x) + 2 cos(x)dx = sin(x) cos2 (x) + 2 sin(x) + C,

e portanto: Z
sin(x) cos2 (x) + 2 sin(x)
cos3 (x)dx = + C.
3
R
vii) x2 cos(bx) dx:
Z Z
sin(bx) 2
2 sin(bx)
cos(bx)x dx = x − 2x dx =
| {z }
f ′g
| b{z } | b{z }
fg f g′
Z
sin(bx) 2 2
=x − sin(bx)x =
b b
Z
sin(bx) 2 2
x − sin(bx) · x dx =
b b | {z }
F ′G
Z
sin(bx) 2 2 cos(bx) cos(bx)
= x − [− · x− − · 1 dx =] =
b b| b{z } | b
{z }
FG F ′G
1. EXERCÍCIOS 356

sin(bx) 2 2 2
= x + 2 cos(bx) · x − 3 sin(bx) + C.
R b b b
viii) eax cos(bx) dx:
Z Z
ax sin(bx) ax sin(bx) ax
cos(bx)e dx = e − ae dx =
| {z } | b{z } | b {z }
f ′g
fg f g′
Z
sin(bx) ax a
= e − sin(bx)eax dx =
b b| {z }
F ′G
Z
sin(bx) ax a − cos(bx) ax − cos(bx) ax
= e − [ e − ae ].
b b | b{z } | b {z }
FG F G′
Logo Z
a2 sin(bx)eax a
(1 + 2 ) · cos(bx)eax dx = + 2 cos(bx)eax + C
b b b
e Z
ax 1 sin(bx)eax a
cos(bx)e dx = a2
( + 2 cos(bx)eax ) + C.
1 + b2 b b

1. Exercı́cios
Exercı́cio 1.1. Dê um argumento para provar que ∀n ∈ N:
Z π
t · cos(nt)dt = 0
−π
sem fazer contas !
Integrando por partes, prove que:
Z π
2·π
t · sin(nt) dt = (−1)n+1 · ,
−π n
Exercı́cio 1.2.
i) verifique que se x ∈ [0, π2 ] então
x ≥ x sin(x) ≥ 0.
ii) Usando integração por partes e o segundo teorema fundamental, calcule a área
da região compreendida entre os gráficos de y = x e de y = x sin(x) de x = 0 até
x = π2 , mostrada na figura a seguir:

1,6

1,2

0,8

0,4

0
0 0,2 0,4 0,6 0,8 1 1,2 1,4
x
CAPÍTULO 24. INTEGRAÇÃO POR PARTES 357

Exercı́cio 1.3.
Se f ′ (x) = x2 · ln(x) e ademais f (e) = 0, qual é a f (x) ?
Exercı́cio 1.4. Prove que:
Z π Z π
2n
sin2n+1 (θ) dθ = · sin2n−1 (θ) dθ.
0 2n + 1 0
CAPı́TULO 25

Integração por substituição

Suponha uma f : J → R contı́nua e uma g : I → J contı́nua também. A variável


do domı́nio de f será u, f = f (u), e no domı́nio de g será x, g = g(x).
Como g(I) ⊂ J, então u = g(x) e faz sentido a composição de funções f (g(x)).
Note que em geral:
Z b Z g(b)
f (g(x)) dx 6= f (u) du.
a g(a)
2
Por exemplo, se f (u) = u e u = g(x) = x então:
Z b Z b2
b3 − a3 2 b4 − a4
= x dx 6= u du =
3 a a2 2
O que precisamos para corrigir esse erro é dado pelo seguinte Teorema:
Teorema 0.1. Seja f : J → R contı́nua e g : I → J derivável, u = g(x) com g ′ (x)
contı́nua. Então:
• faz sentido a composição f (g(x)),
• f (g(x))g ′(x) é integrável e de fato
Z b Z g(b)

f (g(x)) g (x) dx = f (u) du.
a g(a)

Supondo por um momento esse resultado, corrigimos o erro anterior:


Z b Z b2
b4 − a4 2 b4 − a4
2( )= x 2x dx = u du = .
4 a a2 2
O Teorema 0.1
Z b Z g(b)

f (g(x)) g (x) dx = f (u) |{z}
du .
a | {z } g(a)

sugere uma notação:


du = g ′ (x) dx,
que sugere por sua vez, para u = g(x), a notação:
du
= g ′(x).
dx
O lado esquerdo du
dx
é o modo como Leibniz se referia à derivada de u = g(x),
que na notação do Newton é g ′ (x). Ou seja, a última expressão que escrevemos
corresponde a dois modos de se escrever a mesma coisa.

359
360

Demonstração. (do Teorema 0.1)


Note que pelo Segundo Teorema do Cálculo:
Z g(b)
f (u)du = F (g(b)) − F (g(a)),
g(a)

onde F (u) é uma primitiva de f (u). Mas por outro lado, pela regra da composta:
(F (g(x)))′ = F ′ (g(x))g ′(x) = f (g(x))g ′(x)
ou seja que F (g(x)) é primitiva da função:
f (g(x))g ′(x).
Portanto se aplico o Segundo Teorema para calcular
Z b
f (g(x))g ′(x)dx
a
tenho Z b
f (g(x))g ′(x)du = F (g(b)) − F (g(a)).
a
Logo
Z g(b) Z b
f (u)du = f (g(x))g ′(x)dx.
g(a) a


Exemplo 0.1. Vamos provar aqui que a área sob o gráfico de 2 ln(x)
x
, de x = 1 até
x = e := exp(1) vale exatamente 1.
Ou seja, que Z e
2 ln(x)
dx = 1.
1 x
Faço u = ln(x), du = x1 dx e acerto os liitesd e integração:
Z e Z 1
2 ln(x) u2 u2
dx = 2 u du = 2 [ (1) − (0)] = 1.
1 x 0 2 2

Vamos ver como a linguagem da Integração por Substituição se aplicaria pra


encontrar algumas primitivas.
Exemplo 0.2. Por exemplo, para começar, primitivas de
sin(x) · cos(x).
Deixando de lado os limites de integração estamos deixando livre a escolha da con-
stante C. Portanto com:
u = sin(x), du = cos(x)dx
temos pelo Teorema 0.1:
Z Z
sin(x) · cos(x) dx = u du =
CAPÍTULO 25. INTEGRAÇÃO POR SUBSTITUIÇÃO 361

u2
= +C =
2
sin2 (x)
= + C.
2
Se quisermos destacar os limites de integração então faremos:
Z b Z sin(b)
sin(x) · cos(x) dx = u du =
a sin(a)

sin2 (b) sin2 (a)


= − .
2 2
Exemplo 0.3. Agora primitivas de
sinn (x) · cos(x), n ∈ N.
Sem nos fixarmos em limites de integração. com:
u = sin(x), du = cos(x)dx
temos pelo Teorema 0.1:
Z Z
n
sin (x) · cos(x) dx = un du =

un+1
= +C =
n+1
sinn+1 (x)
= + C.
n+1
Se atentamos aos limites de integração:
Z b Z sin(b)
n
sin (x) cos(x) dx = un du =
a sin(a)

sinn+1 (b) sinn+1 (a)


= − .
n+1 n+1
Exemplo 0.4. Agora quero as primitivas de
4x3 + 4x
.
x4 + 2x2 + 1
Para isso faço
u = x4 + 2x2 + 1, du = (4x3 + 4x) dx
e portanto pelo Teorema 0.1:
Z Z
4x3 + 4x 1
dx = du =
x4 + 2x2 + 1 u
= ln(u) + C =
= ln(x4 + 2x2 + 1) + C.
1. A SUBSTITUIÇÃO TRIGONOMÉTRICA X = SIN(θ) 362

Exemplo 0.5. Z

x3 · x − 5 dx, x − 5 > 0.
Faço
u = x − 5, du = dx
e escrevo x3 = (u + 5)3 . Daı́:
Z Z
√ 1
3
x · x − 5 dx = (u + 5)3 u 2 du =
Z
1
= (u3 + 15u2 + 75u + 125)u 2 du =
7 5 3 1
= u 2 + 15u 2 + 75u 2 + 125u 2 du =
2 9 30 7 5 250 3
= u 2 + u 2 + 30u 2 + u2 + C =
9 7 3
2 9 30 7 5 250 3
= (x − 5) 2 + (x − 5) 2 + 30(x − 5) 2 + (x − 5) 2 + C.
9 7 3
Exemplo 0.6. Z
1
√ √x dx, x > 0.
xe
Faço
√ 1
u = x, du = √ ,
2 x
logo Z Z
1
√ √x dx = e−u 2 du =
xe
1
= 2 (−e−u ) + C = −2 √x + C.
e

1. A substituição trigonométrica x = sin(θ)


A integral por substituição que quero tratar agora é (r > 0):
x
x = r · sin(θ) ou seja θ = arcsin( ),
r
para
π π x
− <θ< e − 1 < < 1.
2 2 r
O primeiro uso dela é obter de novo que:
Z Z
1 1
√ dx = p cos(θ) dθ =
1−x 2
1 − sin2 (θ)
Z
cos(θ)
= dθ = θ + C = arcsin(x) + C.
cos(θ)
CAPÍTULO 25. INTEGRAÇÃO POR SUBSTITUIÇÃO 363

2. Áreas do Cı́rculo e Elipse


Até aqui usamos as substituições u = g(x) e du = g ′(x) dx para simplificar a ex-
pressão que estamos integrando. A seguir usamos o Teorema 0.1 de um jeito diferente,
que parece complicar o integrando: mas no final tudo acaba bem !
Por ter sido demonstrado há tanto tempo por Arquimedes que a área do cı́rculo
de raio r é πr 2 , acabamos por trivializar esse fato notável.
Vejamos o que dá se tento calcular a área do Cı́rculo usando integrais/primitivas.
Vamos fazer o seguinte, vamos calcular primeiro a área de um quarto de Cı́rculo
de raio r, aquele que fica no primero quadrante e multiplicar depois o resultado por
4.
√A área do Cı́rculo no primeiro quadrante é a área sob o gráfico de y = f (x) =
+ r 2 − x2 , para x ∈ [0, r]. Quero calcular portanto:
Z r√
r 2 − x2 dx.
0
Faço a substituição:
x = r sin(θ).
Pelo Teorema 0.1 acima tenho que calcular:
Z π q Z r=r sin( π2 ) √
2
2
r 2 − r 2 sin (θ) · r cos(θ) dθ = r 2 − x2 dx.
0 0=r sin(0)
π
Ora como na região 0 ≤ θ ≤ 2
temos cos(θ) ≥ 0 posso dizer que:
q
cos(θ) = 1 − sin2 (θ)
então escrevo:
Z π q Z π q
2 2
2 2
2 2
r − r sin (θ) · r cos(θ) dθ = r 1 − sin2 (θ) · cos(θ) dθ =
0 0
Z π
2
= r2 cos2 (θ) dθ.
0
Já fizemos no Capı́tulo 24 a integral:
Z
cos2 (θ) dθ

e obtivemos como primitiva1 de cos2 (θ):


sin(θ) cos(θ) + θ
.
2
1Outra 1+cos(2θ)
opção para continuar seria usar a fórmula trigonométrica: cos2 (θ) = 2 e depois
uma primitiva de 1+cos(2θ)
2 , que é naturalmente
θ sin(2θ) sin(θ) cos(θ) + θ
+ = .
2 4 2
2. ÁREAS DO CÍRCULO E ELIPSE 364

Logo o Segundo Teorema do Cálculo dá:


Z π
2 sin(θ) cos(θ) + θ π sin(θ) cos(θ) + θ
cos2 (θ) dθ = ( )( ) − ( )(0) =
0 2 2 2
π
= .
4
Logo a área do setor no primeiro quadrante é π4 r 2 e a área do cı́rculo é πr 2 .

É claro que podemos inverter a questão e, supondo que sabemos a área de cı́rculos,
usar isso para calcular integrais.
Por exemplo, para r > 0 e r 2 − x4 > 0, vamos provar que
Z √r √
8
π = 2· r 2 − x4 · x dx.
r 0

De fato fazendo u = x2 , du = 2x dx e acertando os limites de integração temos:


Z √r √ Z r√
2 4
du
r − x · x dx = r 2 − u2 =
0 0 2
1 1
= · · πr 2 ,
2 4
Rr√ 1
pois 0
r 2 − u2 du é área de 4
de Cı́rculo de raio r.

Agora mostro que uma pequena adaptação do que fizemos para calcular a área do
cı́rculo nos dá a área de Elipses.
2 2
Considere a Elipse xa2 + yb2 = 1.
Vamos primeiro considerar 14 de sua área, que é a área sob o gráfico de y =
q
2
b2 (1 − xa2 ), com x ∈ [0, a].
Então quero calcular:
Z ar
x2
b2 (1 − 2 ) dx
0 a
e o farei com a substituição:
x = a sin(u), dx = a cos(u) du,
que nos dá:
Z r Z π
a q
x2 2
b2 (1 − 2 ) dx = b2 (1 − sin2 (u))a cos(u) du =
0 a 0
Z π
2
= ab cos2 (u) du.
0
Mas pelo que já vimos acima:
Z π
2 π
cos2 (u) du =
0 4
CAPÍTULO 25. INTEGRAÇÃO POR SUBSTITUIÇÃO 365

e portanto r
Z a
x2 π
b2 (1 − 2
) dx = ab .
0 a 4
2
x2
Logo a área toda da elipse + yb2 = 1 é πab.
a2
Quando b = a temos um cı́rculo x2 + y 2 = a2 , cuja área é πa2 .
R√
3. r 2 − x2 dx

Note que se
x
x = r sin(θ) e θ = arcsin( ),
r
então:
sin(θ) cos(θ) + θ 1 x x x
= · [ · cos(arcsin( )) + arcsin( )] =
2 2 r r r

1 x 2
r −x 2 x
= ·[ · + arcsin( )],
2 r r r
onde a última igualdade fica clara se usarmos a Figura a seguir:

r
x

2 2
r−x

Ou seja, pelo que fizemos na Seção anterior:


Z √
r2 x √ 2 x
r 2 − x2 dx = · [ 2 · r − x2 + arcsin( )] + C
2 r r
ou finalmente
Z √ √
1 x
r 2 − x2 dx = · [x · r 2 − x2 + r 2 arcsin( )] + C.
2 r

4. Mais exemplos da substituição x = sin(θ)


Na integral a seguir note que faço a substituição
x
= sin(θ)
3
para ter:
Z Z Z
x2 x2 1 x2
√ dx = p dx = · p dx =
9 − x2 9 · (1 − ( x3 )2 ) 3 1 − ( x3 )2
Z Z
1 9 · sin2 (θ)
= p · 3 cos(θ) dθ = 9 · sin2 (θ)dθ
3 2
(1 − sin (θ))
4. MAIS EXEMPLOS DA SUBSTITUIÇÃO X = SIN(θ) 366

e esta última integral sabemos fazê-la: seja pelo método por partes do Capı́tulo 24
ou usando a relação trigonométrica:
1 − cos(2θ)
sin2 (θ) = .
2
Sai então:
Z
x2 θ sin(2θ) θ sin(θ) cos(θ)
√ dx = 9 · ( − )+C =9·( − )+C =
9 − x2 2 4 2 2

arcsin( x3 ) 1 x 9 − x2
=9·( − · · ) + C.
2 2 3 3
Na integral a seguir, faço
x = sin(θ)
para ter:
Z Z
x3 sin3 (x)
√ dx = p cos(θ) dθ =
1 − x2 1 − sin2 (θ)
Z Z
3
= sin (θ) dθ = sin2 (θ) · sin(θ) dθ =

Z Z Z
2
= (1 − cos (θ)) · sin(θ) dθ = sin(θ) θ + cos2 (θ)) · (− sin(θ)) dθ =

cos3 (θ)
= − cos(θ) + +C =
3
3
1 (1 − x2 ) 2 √ 1 − x2
= −(1 − x2 ) 2 + = 1 − x2 · (−1 + ) + C.
3 3
Agora faremos a próxima integral com a substituição x = 3 · sin(θ):
Z Z
1 1
√ dx = p 3 cos(θ) dθ =
2
x · 9−x 2
9 sin (θ) · 9 − 9 sin2 (θ)
2

Z
1 1
= · dθ =
9 sin2 (θ)
Z
1
= · csc2 (θ) dθ =
9

1 1 9 − x2
= − · cot(θ) + C = − · + C.
9 9 x
CAPÍTULO 25. INTEGRAÇÃO POR SUBSTITUIÇÃO 367

5. Substituição trigonométrica x = tan(θ)


A substituição
x = tan(θ) ou θ = arctan(x),
para:
π π

<θ< e x ∈ R,
2 2
permite reobter: Z Z
1 1
2
dx = 2
sec2 (θ) dθ =
x +1 tan (θ) + 1
Z
= dθ = θ + C = arctan(x) + C.

6. Mais exemplos da substituição x = tan(θ)


As integrais do tipo Z
x
√ dx
1 + x2
podem ser feitas com a substituição2:
x = tan(θ), dx = sec2 (θ) dθ.
Como q p π π
1 + tan2 (θ) = sec2 (θ) = sec(θ), se − <θ<
2 2
então Z Z
x tan(x)
√ dx = sec2 (θ) du =
1 + x2 sec(θ)
Z
=
tan(θ) sec(θ) du = sec(θ) + C =

= sec(arctan(x)) + C = 1 + x2 + C,
onde a última igualdade fica clara se usarmos a Figura a seguir:

1+ x2
x

As integrais do tipo Z
1
√ dx
1 + x2
são um bom exemplo da substituição:
x = tan(θ), dx = sec2 (θ) dθ.
2Apesar de que a substituição u = 1 + x2 e du = 2x dx dá o resultado imediatamente
6. MAIS EXEMPLOS DA SUBSTITUIÇÃO X = TAN(θ) 368

Como
q p π π
1 + tan2 (θ) = sec2 (θ) = sec(θ), se − <θ<
2 2
então Z Z
1 1
√ dx = sec2 (θ) du =
1+x2 sec(θ)
Z
= sec(θ) du.

Só que agora somos obrigados a saber fazer esta última integral.
Para isso vamos fazer uns pequenos malabarismos3:
Z Z
1
sec(u) du := du =
cos(u)
Z
1 + sin(u)
= du =
cos(u) (1 + sin(u))
Z
sin2 (u) + cos2 (u) + sin(u)
= du =
cos(u)(1 + sin(u))
Z
cos(u) sin(u)
= + du =
1 + sin(u) cos(u)
Z Z
cos(u) − sin(u)
= du − du ==
1 + sin(u) cos(u)

= ln | 1 + sin(u) | − ln | cos(u) | + C =

1 + sin(u)
= ln | |+C =
cos(u)

=: ln | sec(u) + tan(u) | + C.
Finalmente então podemos completar a integração anterior:
Z
1
√ dx = ln | sec(θ) + tan(θ) | + C =
1 + x2

= ln | sec(arctan(x)) + tan(arctan(x)) | + C = ln( x2 + 1 + x) + C.

3Adaptando esses passos se prova também que


Z
csc(u) du = − ln | csc(u) + cot(u)| + C
CAPÍTULO 25. INTEGRAÇÃO POR SUBSTITUIÇÃO 369
R√
7. r 2 + x2 dx
Faço a seguir a substituição x = r · tan(θ):
Z √ Z q
2
2 2
r + x dx = r · 1 + tan2 (θ) sec2 (θ)dθ =
Z
= sec3 (θ)dθ.
Agora para calcular esta integral faço por partes:
Z Z
sec (θ)dθ = sec(θ) · sec2 (θ) dθ =
3

Z Z
= sec(θ)dθ + sec(θ) · tan2 (θ) dθ =
Z Z
= sec(θ)dθ + sec(θ) · tan(θ) · tan(θ) dθ =
| {z } | {z }
g′ f
Z Z
= sec(θ)dθ + sec(θ) tan(θ) − sec(θ) sec2 (θ) dθ,
| {z } | {z } | {z } | {z }
g f g f′
portanto: Z Z
3 1
sec (θ)dθ = · [ sec(θ)dθ + sec(θ) · tan(θ)] + C.
2
R
Voltando ao que queremos, como θ = arctan( xr ) e como já temos sec(θ) dθ:
Z √ Z 2 Z
2 3 r
r 2 + x2 dx = r · sec (θ)dθ = · [ sec(θ)dθ + sec(θ) · tan(θ)] + C =
2
√ √
r2 x2 + r 2 x x2 + r 2 x
= · [ln( + )+ · ]+C =
2 √ r r r r
r 2 2
x +r 2 x 1 √
= · ln( + ) + · x x2 + r 2 + C.
2 r r 2
8. Substituição trigonométrica x = sec(θ)
Quando falamos em x = sec(θ) e θ = arcsec(x) vamos pensar que
π π
1 < |x| e θ ∈ [0, ) ∪ ( , π].
2 2
Onde ademais, se x > 1 então 0 < θ < π2 .
O primeiro uso desta substituição será, supondo x > 1 e r > 0:
Z
1
√ dx =
x · x2 − r 2
Z
1
= p r sec(θ) tan(θ)dθ =
r sec(θ) · r 2 sec2 (θ) − r 2
Z
1 1 1
= · dθ = · θ + C = arcsec(x) + C.
r r r
9. MAIS EXEMPLOS PARA A SUBSTITUIÇÃO X = SEC(θ). 370

9. Mais exemplos para a substituição x = sec(θ).


As integrais do tipo Z
1
dx √
−1 x2
para 1 < x são um bom exemplo para a substituição:
x = sec(θ), dx = sec(θ) tan(θ) dθ,
θ = arcsec(x)
onde
π
1<x e 0<θ< .
2
De fato, como
√ q
x2 −1= tan2 (θ) = tan(θ),
se 0 < θ < π2 , então
Z Z
1 1
√ dx = sec(θ) tan(θ) du =
x2 − 1 tan(θ)
Z
= sec(θ) dθ =
= ln(sec(θ) + tan(θ)) + C

= ln(x + tan( x2 − 1)) + C,
onde a última igualdade fica clara se usarmos a Figura a seguir:

x
x2 1

A integral a seguir
Z √
x2 − 9
dx =
x
com
x = 3 · sec(θ), dx = 3 · sec(θ) tan(θ) dθ,
vira: Z √ Z p
x2 − 9 9 sec2 (θ) − 9
dx = sec(θ) tan(θ) dθ =
x 3 sec(θ)
Z
= 3 · tan(θ) dθ =
Z
= 3 · (sec2 (θ) − 1) dθ =
= 3 · tan(θ) − 3 · θ + C =
CAPÍTULO 25. INTEGRAÇÃO POR SUBSTITUIÇÃO 371

x2 − 9 x
=3· − 3 · arcsec( ) + C.
3 3
R√
10. x2 − r 2 dx
A seguir |x| > r > 0. Faço a mudança x = r · sec(θ) e depois integro por partes:
Z √ Z
2 2 2
x − r dx = r · tan(θ) · sec(θ) tan(θ)dθ =
Z
= r · (tan(θ) sec(θ) − sec3 (θ) dθ).
2

Mas já calculamos


Z
1
sec3 (θ) dθ = · [tan(θ) sec(θ) − ln(sec(θ) + tan(θ))] + C.
2
Portanto:
Z √
r2
x2 − r 2 dx = · [tan(θ) sec(θ) − ln(sec(θ) + tan(θ))] + C =
2
√ √
r 2 x x2 − r 2 x2 − r 2 x
= ·[ − ln( + )+C =
2 r r r r

1 √ r 2 2
x −r 2 x
= x x2 − r 2 − · ln( + ) + C.
2 2 r r
R
11. E as da forma √Ax3 +Bx12 +Cx+D dx ?

Nas Seções anteriores tivemos sucesso ao integrarmos


Z
1
√ dx,
ax2 + bx + c
fazendo uma mudança de variável do tipo x = sin(θ), x = tan(θ) ou x = sec(θ).
Mas, em geral, ou seja, para polinômios Ax3 + Bx2 + Cx + D de grau três gerais,
as integrais Z
1
√ dx
Ax3 + Bx2 + Cx + D
não podem ser expressas em termos de funções conhecidas, são chamadas de integrais
elı́pticas.

12. Exercı́cios
R
Exercı́cio 12.1. Fizemos ln(x)
x
dx por partes.
Veja que, neste exemplo, é mais fácil fazer por substituição.
Calcule pelos dois métodos:
Z e3
ln(x)
dx.
e2 x
12. EXERCÍCIOS 372
R √
x
Exercı́cio 12.2. Para fazer e dx use uma substituição e depois uma integração
por partes.
Exercı́cio 12.3. Faça por substituição as integrais a seguir. Dica: O lado direito
das igualdades dá uma pista das substituições u = g(x) e du = g ′(x)dx adequadas.
Z Z
1
i) tan(x) dx = − · (− sin(x)) dx,
cos(x)
Z Z
1
ii) cot(x) dx = · cos(x) dx,
sin(x)
Z Z Z
1 sin(x) −1
iii) sec(x) tan(x) dx := dx = · (− sin(x)) dx
cos(x) cos(x) cos2 (x)
Z Z
1 1 1
iv) dx = · dx.
ln(x) x ln(x) x
Exercı́cio 12.4. Prove que ∀n ∈ N:
Z 1 Z π
2 n
(1 − x ) dx = (sin(θ))2n+1 dθ.
−1 0
CAPı́TULO 26

Integração de funções racionais

Não háR uma solução para o problema de como integrar quocientes em geral; por
exemplo, sin(x)
x
dx não pode ser expressa em termos de funções elementares.
A questão que vamos respoder nesta Seção é a de como integrar
Z
p(x)
dx
q(x)
onde p(x), q(x) são polinômios.
A técnica geral para integrar essa funções racionais (quocientes de polinômios)
é conhecida como integração por frações parciais (ou frações simples, elementares,
como alguns chamam).
Procederemos por etapas, começando com casos simples.
Mais adiante, na Seção 4, daremos enunciados gerais.
R
1. (ax2 + bx + c)−1 dx

Começo explicando o que fazer para calcular:


Z
1
2
dx, com 0 6= a, b, c ∈ R.
ax + bx + c
Há três casos a considerar, dependendo do discriminante b2 − 4ac:
• i) b2 − 4ac = 0, ou seja, ax2 + bx + c = (x − x)2 tem uma raı́z real dupla,

• ii) b2 − 4ac > 0, ou seja, ax2 + bx + c = (x − x1 ) · (x − x2 ) tem duas raı́zes


reais diferentes ou

• iii) b2 − 4ac < 0, ou seja, ax2 + bx + c tem duas raı́zes complexas conjugadas
(não tem raı́zes Reais).

No caso i):
Faço u = x − x, du = dx e
Z Z
1 1
2
dx = dx =
ax + bx + c (x − x)2
Z
1 −1 1
= du = + C = + C.
u2 u x−x
No caso ii):
373
R
1. (AX 2 + BX + C)−1 DX 374

Gostaria de escrever, para A e B números bem escolhidos:

1 1 A B
= = + ,
ax2 + bx + c (x − x1 ) · (x − x2 ) x − x1 x − x2

pois então terı́amos:


Z Z Z
1 A B
dx = dx + dx =
(x − x1 ) · (x − x2 ) x − x1 x − x2

Z Z
1 1
=A· du + B · dv,
u v
onde u = x − x1 e v = x − x2 e daqui chegamos em:
Z
1
dx = A · ln |x − x1 | + B · ln |x − x2 | + C.
(x − x1 ) · (x − x2 )

Como encontrar A e B como queremos ? Queremos que valha:

1 A B
= + ,
(x − x1 ) · (x − x2 ) x − x1 x − x2

ou seja, somando as frações à direita:

1 (A + B)x − Ax2 − Bx1


= .
(x − x1 ) · (x − x2 ) (x − x1 ) · (x − x2 )

Para que (A + B)x − Ax2 − Bx1 = 1 precisamos ter

B = −A e − Ax2 + Ax1 = 1,

ou seja, as escolhas de A e B são:


1 −1
A= e B= .
x1 − x2 x1 − x2

Em suma, no caso ii) (x1 , x2 raı́zes Reais distintas):


Z
1 1 1
2
dx = · ln |x − x1 | − · ln |x − x2 | + C.
ax + bx + c x1 − x2 x1 − x2

No caso iii):
Primeiro faço, já que a 6= 0:
Z Z Z
1 1 1 1
dx = b c
dx = · dx.
ax2 + bx + c a · (x2 + a x + a ) a x2 + ab x + c
a
CAPÍTULO 26. INTEGRAÇÃO DE FUNÇÕES RACIONAIS 375

Agora escrevo1:
b c b b2 c
x2 + x + = (x + )2 − 2 + =
a a 2a 4a a
b 2 4ac − b2
= (x + ) + .
2a 4a2
Então
Z Z
1 1 1
2
dx = · b 2 4ac−b2
dx.
ax + bx + c a (x + 2a
) + 4a2
Agora faço a substituição:
b
u=x+ e du = dx.
2a
Então (já que 4ac − b2 > 0):
Z Z
1 1 1
b 2 4ac−b2
dx = 4ac−b2
du =
(x + 2a
) + 4a2
a u2 + 4a2

1 1 u
= ·q · arctan( q ) + C,
a 4ac−b2 4ac−b2
4a2 4a2

conforme a Seção 5 do Capı́tulo 16. Simplificando:


Z
1 2 u
2
dx = √ · arctan( q ) + C.
ax + bx + c 4ac − b2 4ac−b2
4a2

R αx+β
2. ax2 +bx+c
dx

Agora trato o caso mais geral:


Z
αx + β
2
dx, α, β ∈ R.
ax + bx + c
1Se continuamos um pouquinho obteremos a fórmula de Báskara: já que a 6= 0,
b c b 4ac − b2
x2 + x + = (x + )2 + .
a a 2a 4a2
De onde, se queremos que 0 = x2 + ab x + ac ,
b 2 b2 − 4ac
(x + ) = ,
2a 4a2

b ± b2 − 4ac
x+ =± ,
2a 2a
e finalmente: √
−b ± b2 − 4ac
x= .
2a
R αX+β
2. AX 2 +BX+C
DX 376

Na situação discutida em iii), em que 4ac − b2 > 0, temos:


Z Z
αx + β 1 αx + β
dx = · 2 dx
ax2 + bx + c a (x + 2a ) + 4ac−b
b 2
4a2

e a mudança
b
u= x+ e du = dx
2a
produz:
Z b
1 α(u − 2a
)+β
· 4ac−b2
du =
a u2 + 4a2
Z Z
1 u α·b 1
= · [α · 4ac−b2
du + (β − )· 2 du] = .
a +u2 4a2
2a + 4ac−b
4a2
u2
A integral mais à direita já sabemos resolvê-la com a função arcotangente:
Z
1 1 x
4ac−b 2 du = q · arctan( q ) + C.
u2 + 4a2 4ac−b2 4ac−b2
4a2 4a2

Já Z Z
u 1 2u
4ac−b2
du = · 2 du
+ u2 4a2
2 u2 + 4ac−b
4a2
e aı́ reconhecemos uma derivada logarı́tmica; logo:
Z
1 2u 1 2 4ac − b2
· 2 du = · ln(u + )+C =
2 u2 + 4ac−b
4a2
2 4a2
1 b 4ac − b2
· ln((x + )2 +
= ) + C.
2 2a 4a2
Juntando esses resultados concluı́mos o resultado.
Já no caso ii) discutido antes, em que há duas raı́zes reais distintas x1 6= x2 , ou
seja: Z Z
αx + β αx + β
dx = dx,
axa + bx + c (x − x1 ) · (x − x2 )
vou tentar escrever:
αx + β A B
= + ,
(x − x1 ) · (x − x2 ) (x − x1 ) (x − x2 )
para A e B bem escolhidos, pois daı́ em diante saberemos fazer :
Z
A B
+ dx
(x − x1 ) (x − x2 )
usando o logaritmo natural. Como
A B (A + B) · x + (−Ax2 − Bx1 )
+ = ,
(x − x1 ) (x − x2 ) (x − x1 ) · (x − x2 )
preciso ter:
α =A+B e β = −Ax2 − Bx1 ,
CAPÍTULO 26. INTEGRAÇÃO DE FUNÇÕES RACIONAIS 377

que dão:
αx1 + β
A= e B = α − A.
x1 − x2
Resta o caso em que:
Z Z
αx + β αx + β
dx = dx,
axa + bx + c (x − x)2

que dá:
Z Z Z
αx + β x 1
dx = α · dx + β · dx =
(x − x)2 (x − x)2 (x − x)2
Z Z
1 x 1
=α· [ + ] dx + β · dx =
x − x (x − x)2 (x − x)2

1 1
= α · ln ||x − x|| − αx · −β· + C.
x−x x−x

R 1
3. Ax3 +Bx2 +Cx+D
dx

Agora quero tratar do quê fazer para calcularmos:


Z
1
3 2
dx, A 6= 0.
Ax + Bx + Cx + D

Vimos, na Proposição 6.1 do Capı́tulo 6 que sempre um polinômio de grau ı́mpar


com coeficientes Reais tem ao menos uma raı́z Real x = x1 .
Portanto há 4 caso possı́veis a considerar2:
• i) Ax3 + Bx2 + Cx + D tem uma raı́z tripla Real,
• ii) Ax3 + Bx2 + Cx + D tem uma raı́z dupla e uma simples, todas Reais,
• iii) Ax3 + Bx2 + Cx + D tem três raı́zes Reais distintas, x1 , x2 , x3 .
• iv) Ax3 + Bx2 + Cx + D tem apenas uma raı́z simples Real e duas raı́zes
complexas (conjugadas).
São representados na figura a seguir:

2Qual o análogo do discriminante b2 − 4ac de ax2 + bx + c no caso de Ax3 + Bx2 + Cx + D ?


Isso se trata no Capı́tulo 32. Mas e como encontrar raı́zes de Ax3 + Bx2 + Cx + D? Em geral, nos
Exercı́cios básicos, uma raı́z do polinômio de grau 3 é evidente. Ou pelo menos se pode usar o Teste
da Raı́z Racional (Afirmação 8.1 do Capı́tulo 6). Após fatoração dessa primeira raı́z Real (talvez
até Rational) sobra um polinômio de grau 2. Em geral, será preciso usar a fórmula de Cardano do
Capı́tulo 32
R 1
3. AX 3 +BX 2 +CX+D
DX 378

1
x
-1 -0,5 0 0,5 1
0

-1

-2

-3

-4

Figura: Casos i) em vermelho, ii) em verde, iii) em amarelo e iv) em azul.

No que segue suponhamos que conhecemos as raı́zes Reais do Ax3 + Bx2 + Cx + D


Então no caso i), já sabemos o que fazer:
Z Z
1 1 −1
3 2
dx = 3
dx = +C
Ax + Bx + Cx + D (x − x1 ) (x − x1 )2
No caso ii):
Z Z
1 1
dx = dx
Ax3 + Bx2 + Cx + D (x − x1 )2 · (x − x2 )
vamos ser otimistas e tentar escrever, para ci constantes bem escolhidas:
1 c1 c2 c3
2
= + 2
+
(x − x1 ) · (x − x2 ) (x − x1 ) (x − x1 ) (x − x2 )
pois então obterı́amos:
Z
1 −1
2
dx = c1 · ln |x − x1 | + c2 · + c3 · ln |x − x2 | + C.
(x − x1 ) (x − x2 ) x − x1
Para encontrarmos ci adequadas, façamos primeiro a soma de frações à direita:
c1 c2 c3
+ + =
(x − x1 ) (x − x1 )2 (x − x2 )
c1 (x − x1 )(x − x2 ) + c2 (x − x2 ) + c3 (x − x1 )2
= =
(x − x1 )2 (x − x2 )
(c1 + c3 )x2 + (c2 − c1 (x1 + x2 ) − 2c3 x1 )x + (c1 x1 x2 − c2 x2 + c3 x21 )
= .
(x − x1 )2 (x − x2 )
Como o numerador dessa última expressão tem que igual ao numerador de (x−x )12 (x−x )
1 2
otemos um sistema de três equações:
c1 + c3 = 0, c2 − c1 (x1 + x2 ) − 2c3 x1 = 0
e c1 x1 x2 − c2 x2 + c3 x21 = 1.
CAPÍTULO 26. INTEGRAÇÃO DE FUNÇÕES RACIONAIS 379

As duas primeiras equações dão:


c3 = −c1 , c2 = c1 (x2 − x1 ),
que, quando substituidas na terceira equação, dão:
1 −1
c1 = 2 2
= .
2x1 x2 − x1 − x2 (x1 − x2 )2
Ou seja encontramos assim c1 e com ele obtemos c2 e c3 , desde que conheçamos as
raı́zes Reais x1 6= x2 .
No caso iii):

Gostarı́amos de escrever :
1 c1 c2 c3
= + +
(x − x1 )(x − x2 )(x − x3 ) x − x1 x − x1 x − x3
pois então integrarı́amos usando a primitiva ln | |.
Somamos
c1 c2 c3
+ + =
x − x1 x − x1 x − x3
(c1 + c2 + c3 ) x2 − (c1 (x2 + x3 ) + c2 (x1 + x3 ) + c3 (x1 + x2 )) x
= +
(x − x1 )(x − x2 )(x − x3 )
c1 x x + c2 x1 x3 + c3 x1 x2
+ 2 3
(x − x1 )(x − x2 )(x − x3 )
e igualo seu numerador a 1, obtendo um sistema de três equações:
c1 + c2 + c3 = 0, c1 (x2 + x3 ) + c2 (x1 + x3 ) + c3 (x1 + x2 ) = 0,
c1 x2 x3 + c2 x1 x3 + c3 x1 x2 = 1.
Da primeira posso pôr c3 em função dos outros, da segunda posso por c2 em função
de c1
c1 (x3 − x1 )
c3 = −(c1 + c2 ), c2 = − ,
(x3 − x2 )
e substituindo na terceira determinamos o c1 .

Caso iv):
Aqui temos
Ax3 + Bx2 + Cx + D = (x − x1 ) · (ax2 + bx + c),
onde ax2 + bx + c não tem raı́zes Reais, apenas raı́zes complexas (conjugadas). Se
conhecemos x1 , também conhecemos a, b, c por divisão de polinômios.
Portanto no que segue considero conhecidos esses coeficientes a, b, c.
Seremos otimistas tentando escrever3, para c1 , c2 , c3 adequados:
1 c1 c2 x + c3
2
= + 2 .
(x − x1 ) · (ax + bx + c) x − x1 ax + bx + c
3Note que ∀c1 , c2 :
1 c1 c2
6= + 2 ,
(x − x1 ) · (ax2 + bx + c) x − x1 ax + bx + c
4. FRAÇÕES PARCIAIS EM GERAL 380

Como
c1 c2 x + c3 (ac1 + c2 )x2 + (bc1 − c2 x1 + c3 )x + (c1 c − c3 x1 )
+ 2 = ,
x − x1 ax + bx + c (x − x1 )(ax2 + bx + c)
temos que resolver as equações:
ac1 + c2 = 0, bc1 − c2 x1 + c3 = 0 e c1 c − c3 x1 = 1.
A primeira me permite escrever c2 = −ac1 e a segunda dá
c3 = −bc1 + x1 c2 = −bc1 − x1 ac1 .
Ou seja c3 é função de c1 . Substituido c3 na terceira equação
c1 c − c3 x1 = 1,
esta vira uma equação de grau um em c1 e descobrimos o valor de c1 .
Achados os c1 , c2 , c3 basta calcular
Z
c2 x + c3
dx,
ax2 + bx + c
(o que aprendemos no inı́cio da Seção 2) para termos então finalmente:
Z Z
1 c2 x + c3
3 2
dx = c1 · ln |x − x1 | + dx.
Ax + Bx + Cx + D ax2 + bx + c
4. Frações parciais em geral

A situação que deverı́amos tratar a seguir, após a Seção 3, seria:


Z
αx2 + βx + γ
dx.
Ax3 + Bx2 + Cx + D
Vamos tratá-la já num contexto geral.
Suponho que quero fazer Z
P (x)
dx
Q(x)
onde P (x) é polinômio de grau p e Q(x) de grau q, sem fatores em comum, com
p ≥ q.
Então divido P (x) por Q(x), obtendo:
P (x) = Q(x) · H1 (x) + R1 (x)
pois se por absurdo fazemos:
1 c1 c2
2
= + 2 =
(x − x1 )(ax + bx + c) x − x1 ax + bx + c
ac1 x2 + (bc1 + c2 )x + (c1 c − c2 x1 )
=
(x − x1 )(ax2 + bx + c)
poduzimos equações:
ac1 = 0 e bc1 + c2 = 0.
Como a 6= 0 neste caso, então c1 = 0 e daı́ obtemos c2 = 0, absurdo.
CAPÍTULO 26. INTEGRAÇÃO DE FUNÇÕES RACIONAIS 381

onde o grau do polinômio H1 (x) é h1 = p − q e onde o grau do resto R1 (x) é


r1 < p.
Se r1 ≥ q posso dividir de novo:
R1 (x) = Q(x) · H2 (x) + R2 (x)
onde h2 = r1 − q e r2 < r1 .
E assim por diante: o processo só pára quando algum resto Rk (x) tem grau rk < q
(note que Rk (x) 6≡ 0 pois P (x) e Q(x) foram supostos ser fator comum).
Então
P (x) Q(x) · (H1 (x) + H2 (x) + . . . + Hk (x)) + Rk (x)
= =
Q(x) Q(x)
Rk (x)
= H1 (x) + H2 (x) + . . . + Hk (x) + .
Q(x)
Ora, integrar o polinômio H1 (x) + H2 (x) + . . . + Hk (x) é fácil; logo, o problema se
reduz a integrar uma fração do tipo:
Rk (x)
,
Q(x)
onde o grau do numerador é menor que o do denominador.
Por isso essa será a situação daqui para diante: consideraremos P (x) de grau p e
Q(x) de grau q, com
p<q
e sem fatores comuns.
Queremos fazer: Z
P (x)
dx.
Q(x)
Claro que, se pudermos fazer
P (x) Q′ (x)
=
Q(x) Q(x)
então Z
P (x)
dx = ln ||Q(x)|| + C.
Q(x)
Mas e quando não for assim, o que fazer?
Se usam então dois fatos puramente algébricos, que já vimos funcionarem concre-
tamente em casos particulares:

Fato 1: (Teorema de Fatoração)


Há sempre uma fatoração de Q(x) em produtos de potências de fatores lineares
e/ou quadráticos:
mk n
Q(x) = Lm n1 j
1 · . . . · Lk · Q1 · . . . · Qj ,
1
mi , ni ∈ N,
onde
m1 + . . . + mk + 2 · (n1 + . . . + nj ) = q,
Li := ai x + bi e Qi := ci x2 + di x + ei , ai , . . . , ei ∈ R.
4. FRAÇÕES PARCIAIS EM GERAL 382

Note: bastam lineares ou quadráticos, não precisa mais do que isso.


O exemplo q(x) = x4 + 1 por exemplo se decompõe assim:
√ √
x4 + 1 = (x2 + 1)2 − 2x2 = (x2 − 2 · x + 1) · (x2 + 2 · x + 1) =: Q1 · Q2 ,
onde Q1 e Q2 são polinômios irredutı́veis sobre4 os Reais (i.e. não são produtos de
polinômios Reais de grau 1), já que seus disciminantes valem −2.
Depois se usa:

Fato 2: (Decomposição em Frações Simples)


Se P (x) tem grau p e Q(x) grau q, com p < q e se
mk
Q(x) = Lm n1 nr
1 · . . . · Lk · Q1 · . . . · Qr ,
1
mi , ni ∈ N
então existem números Reais Ai,j , Bi,j e Ci,j tais que:
P (x) A1,1 A1,m Ak,1 Ak,m
= + . . . + m11 + . . . + + . . . + mkk +
Q(x) L1 L1 Lk Lk
B1,1 · x + C1,1 B1,n1 · x + C1,n1 Br,1 · x + Cr,1 B1,nr · x + C1,nr
+ + ...+ n1 + + ... .
Q1 Q1 Qr Qn1 r

Agora temos do lado direito um soma de integrais para fazer:


Z Z
P (x) 1
dx = A1,1 · dx + . . .
Q(x) L1
O leitor pode conferir que, pelo que já expusemos neste Capı́tulo, conseguirı́amos
fazer cada uma das integrais do lado direito, exceto as do tipo:
Z
1
dx, para n ≥ 2,
Q(x)n
onde Q(x) é quadrático
R e irredutı́vel.
R
Note que (x2 +1)n dx = 12 · u1n du se faço u = x2 + 1 e portanto sabemos fazê-la.
x

Como esses polinômios Qi (x) = ax2 + bx + c se deixam escrever (como vimos na


Seção 2) como
b 2 4ac − b2 4ac − b2
Qi (x) = (x +) + , com > 0,
2a 4a2 4a2
b
o problema se reduz essencialmente (quer dizer, módulo substituições u = x + 2a
) a
integrar: Z
1
, para n ≥ 2.
(x + 1)n
2

4 Sobre os complexos sim são redutı́veis:


√ √ √ √
2
√ 2 2√ 2 2√
(x − 2x + 1) = (x − ( − −1)) · (x − ( + −1))
2 2 2 2
√ √ √ √
√ 2 2√ 2 2√
(x2 + 2x + 1) = (x − (− + −1)) · (x − (− − −1))
2 2 2 2
CAPÍTULO 26. INTEGRAÇÃO DE FUNÇÕES RACIONAIS 383

Isso trato na Seção 5 a seguir.


R 1
5. (1+x2 )n
dx, n ≥ 2
Vou fazer para n = 2 em detalhe e apenas enunciar o resultado geral ∀n ≥ 2.
Afirmação 5.1.
Z
1 1 1 x
dx = · arctan(x) + · 2 + C.
(x2 + 1) 2 2 2 x +1
Vou dar duas provas. a primeira é curta mas não ensina muito.
Demonstração. (Primeira demontração)
Para fazer Z
1
dx
(x + 1)2
2

escrevo (e o leitor confere):


Z Z
1 1 x2
= [ − ] dx =
(x2 + 1)2 x2 + 1 (x2 + 1)2
Z
1 1 1 1 x2
= [ · 2 + · 2 − 2 ] dx =
2 x + 1 2 x + 1 (x + 1)2
Z Z
1 1 1 1 x2
= · 2 dx + [ · 2 − 2 ] dx =
2 x +1 2 x + 1 (x + 1)2
1 1 x
= · arctan(x) + · 2 + C,
2 2 x +1
onde se verifica por derivação direta que 21 · x2x+1 é a primitiva certa.


A segunda é longa mas revisa várias coisas que aprendemos:


Demonstração. (Segunda demonstração - Do estudante Walter Ferreira Diniz
Júnior)
Fazemos uma integração por partes:
Z Z
1 1 x
dx = · dx =
(x2 + 1)2 x (x2 + 1)2
Z
1 1 1 1
= · (− 2
) − (− 2 ) · (− ) dx =
x 2(1 + x ) x 2(1 + x2 )
Z
1 1
=− 2
− dx.
2x · (1 + x ) 2x (1 + x2 )
2

E agora uso o Teorema de Frações simples:


Z Z
1 1 1 A A Cx + D
2 2
dx = − 2
− · ( + 2+ ) dx =
(x + 1) 2x · (1 + x ) 2 x x 1 + x2
onde se calcula sem muita dificuldade que:
A = 0, B = 1, C = 0 e D = −1.
6. EXEMPLOS 384

Então:
Z Z
1 1 1 1 1
2 2
dx = − 2
− · ( 2
− 2 ) dx =
(x + 1) 2x · (1 + x ) 2 x x +1
1 1 1
=− 2
+ + · arctan(x) + C =
2x · (1 + x ) 2x 2
1 1 x
= · arctan(x) + · 2 + C.
2 2 x +1


Em geral, há uma fórmula de redução válida ∀n ≥ 2:


Z Z
1 2n − 3 1 x
2 n
dx = · 2 n−1
dx + .
(x + 1) 2n − 2 (x + 1) (2n − 2) · (x2 + 1)n−1

6. Exemplos

Vimos alguns exemplos dessa escritura nas Seções anteriores, onde também se vê
que Ai,j , Bi,j e Ci,j são soluções de sistemas de equações que surgem ao se comparar
os coeficientes de polinômios.

Vejamos mais exemplos:


R 3 2 +40
• 3x x+5x
4 +2x2 dx. Quero escrever:
3x3 + 5x2 + 40 3x3 + 5x2 + 40
= =
x4 + 2x2 x2 · (x2 + 2)
A B Cx + D
= + 2+ 2 .
x x x +2
Somando essas frações temos:
A B Cx + D (A + C) · x3 + (B + D) · x2 + 2A · x + 2B
+ 2+ 2 = .
x x x +2 x2 · (x2 + 2)
Ou seja, quero:
A + C = 3, B + D = 5, 2A = 0 e 2B = 40.
Obtenho: A = 0, B = 20, C = 3 e D = −15. Então:
Z Z Z
3x3 + 5x2 + 40 20 3x − 15
dx = dx + dx =
x4 + 2x2 x2 x2 + 2
Z Z Z
1 3 2x 1
= 20 · 2
dx + · 2
dx − 15 · 2
dx =
x 2 x +2 x +2
−20 3 1 x
= + · ln(x2 + 2) − 15 · √ · arctan( √ ) + C.
x 2 2 2
CAPÍTULO 26. INTEGRAÇÃO DE FUNÇÕES RACIONAIS 385
R x+5
• x3 +4x2 +4x
dx. Quero escrever:
x+5 x+5 A B C
= = + + .
x3 2
+ 4x + 4x x · (x + 2) 2 x x + 2 (x + 2)2
Como:
A B C (A + B) · x2 + (4A + 2B + C) · x + 4A
+ + = ,
x x + 2 (x + 2)2 x · (x + 2)2
obtenho o sistema:
A + B = 0, 4A + 2B + C = 1 e 4A = 5,
de onde
5 −5 −3
A= , B= e C= .
4 4 2
Então:
Z Z Z Z
x+5 5 1 5 1 3 1
3 2
dx = · dx − · dx − · dx =
x + 4x + 4x 4 x 4 x+2 2 (x + 2)2
5 5 3 1
· ln ||x|| − · ln ||x + 2|| + ·
= + C.
4 4 2 x+2
• (do estudante Walter Ferreira Diniz Júnior)
Como estou resumindo o Exemplo do Walter, deixo para o leitor conferir
os coeficientes da decomposição em frações parciais:
Z Z
1 1
4
dx = √ √ dx =
x +1 (x − 2x + 1) · (x2 + 2x + 1)
2

Z −1
√ x+ 1 Z √ x+ 1
−1
2 2 2 2 2 2
= √ dx + √ dx =
2
x − 2x + 1 2
x − 2x + 1
Agora o problema se reduz a saber resolver:
Z
x
√ dx,
x2 − 2x + 1
Z
1
√ dx,
x2 − 2x + 1

(analogamente para o caso em que o denominador é x2 + 2x + 1). A última
é fácil, pois:
Z Z
1 1
√ dx = √ dx =
2
x − 2x + 1 (x − 22 )2 + 21
Z
1
= du
u2 + 21
e sabemos fazer esta com a função arcotangente.
Já Z Z
x x
√ dx = √ dx =
x2 − 2x + 1 (x − 22 )2 + 21
6. EXEMPLOS 386
Z √
u + 22
= du
u2 + 21

onde novamente fizemos u = x − 22 .
Ora,
Z √ Z Z √
u + 22 u 2
2
du = du + du =
u2 + 21 u2 + 21 u2 + 21
Z √ Z
1 1 2 1
= dv + · du,
2 v 2 u + 21
2

1
onde
R v = u2 + 2
e essas últimas já sabemos fazer.
x+2
• x6 +2x4 +x2 dx
Temos
x+2 x+2
=
x6 + 2x4 + x2 x2 · (x2 + 1)2
e queremos encontrar a escritura:
x+2 A B Cx + D Ex + F
2 2 2
= + 2+ 2 + 2 .
x · (x + 1) x x x +1 (x + 1)2
Somo o lado direito e obtenho:
(A + C)x5 + (B + D)x4 + (2A + C + E)x3 + (2B + D + F )x2 + Ax + B
,
x2 · (x2 + 1)2
que, ao ser igualada ao esquerdo, dá:
A = 1, B = 2, C = −1, D = −2, E = −1 e F = −2.
Portanto:
Z Z
x+2 1 2 x+2 x+2
dx = [ + − − ] dx =
x6 + 2x4 + x2 x x2 x2 + 1 (x2 + 1)2
Z Z Z
1 2 2
= dx + dx − dx−
x x2 x2 + 1
Z Z Z
x x 2
− dx − dx − dx.
x2 + 1 (x2 + 1)2 (x2 + 1)2
Dessas seis integrais por fazer, as primeiras quatro têm primitivas conhecidas
(a menos de somar uma constante C):
Z Z
1 2 −2
dx = ln |x|, 2
dx = ,
x x x
Z Z
2 x 1
= dx = 2 arctan(x) e dx = · ln(x2 + 1).
x2 + 1 x2 + 1 2
A quinta se faz com a substituição u = x2 + 1, du = 2x dx:
Z Z
x 1 1 −1 1
2 2
dx = · 2
du = · 2 + C.
(x + 1) 2 u 2 x +1
CAPÍTULO 26. INTEGRAÇÃO DE FUNÇÕES RACIONAIS 387

A última é Z
2 x
dx = arctan(x) + + C,
(x2 + 1)2 (x2 + 1)
pelo que vimos bem no final da Seção 4, no caso n = 2.
7. Exercı́cios
Exercı́cio 7.1. Pelo método das frações parciais faça:
Z
x2 + 30
dx
x3 + 11x2 + 30x
e Z
x2 + 24
dx.
x3 + 10x2 + 24x
CAPı́TULO 27

Integrais impróprias

1
Vimos na Afirmação 6.1 do Capı́tulo 22 que a área sob o gráfico de y = x
à direita
de x = 1 é infinita, ou em outras palavras:
lim ln(x) = +∞.
n→+∞

Mas uma conseguência do Teorema 2.1 escandalizou o filósofo Hobbes, no séc.


XVII: existem regiões ilimitadas cuja Área é finita !
Afirmação 0.1.
Seja k ∈ R com k > 1. Então:
• Z +∞
1 1
i) : k
dx = ,
1 x k−1
ou seja, a área da região que fica sob o gráfico de y = x1k , para x ∈ [1, +∞)
1
é k−1 .
• Z 1
1 1
ii) : 1 dx = 1 + ,
0 (1 − x) k k−1
1
ou seja, a área da região sob o gráfico de y = 1 1 para x ∈ [0, 1) é 1 + k−1 .
(1−x) k

Demonstração.

De i):

A área sob o gráfico de y = x−k , de a > 0 até um certo x, é pelo Segundo Teorema
Fundamental:
Z x
1 1
x−k dx = ( x−k+1 )(x) − ( x−k+1 )(a), onde k 6= 1.
a −k + 1 −k + 1
A área de toda a região à direita de a > 0 é:
1 1
lim [ ( x−k+1 )(x) − ( x−k+1 )(a)) ] =
x→+∞ −k + 1 −k + 1
1 1 1 k−1
= lim [ + a ]=
x→+∞ (−k + 1) xk−1 k−1
1 k−1
= a ,
k−1
onde na última igualdade usei que k > 1.
389
390
1
Para a = 1 obtenho k−1
.

De ii):
Vou dar duas demonstrações: uma calculatória, outra completamente geométrica.
Na primeira fazemos uma integral:
Z 1 Z a
− k1 1
(1 − x) dx := lim (1 − x)− k dx =
0 aր1 0

1 1
−(1 − x)− k +1 (1 − x)− k +1
= lim [ (a) + (0)] =
aր1 − k1 + 1 − k1 + 1

1 1
= =1+ .
− k1 +1 k−1

Na segunda, vemos que:


1
y = (1 − x)− k
1 1
dá y k = 1−x
e 1−x= yk
, ou seja:

1
x= 1− .
yk
R1 1
Então 0 (1 − x)− k dx é a área do quadrado de lado 1 somada com a área da região
à direita de y = 1 que fica sob o gráfico de x = 1 − y1k . Mas essa área é k−1
1
pelo item
i). 

A Figura é apenas uma ilustração disso, pois não consegui usar as mesmas escalas
nos eixos (o quadrado aparece como um retângulo, em verde):

2,5

1,5

1
0 0,2 0,4 0,6 0,8
x
CAPÍTULO 27. INTEGRAIS IMPRÓPRIAS 391
1
Figura: Ilustração para x = 1 − y2
, y ∈ [1, +∞)

0,8

0,6

0,4

0,2

1 1,5 2 2,5 3
x

1
Figura: Ilustração para y = x2
, x ∈ [1, +∞).

1. Um problema da Putnam Competition, n. 2, 1939

Problema: Avalie as integrais:


Z 3
1
p dx
1 (3 − x) · (x − 1)
e Z +∞
1
dx.
1 ex+1 + e3−x
Solução
Parte da questão é dar um sentido às integrais, pois numa o integrando não está
definido em x = 1 nem em x = 3 e na outra o intervalo de integração é infinito.
O sentido que se deve dar à primeira é, como vimos:
Z 3 Z 3−ǫ2
1 1
p dx := lim p dx.
1 (3 − x) · (x − 1) ǫ 1 ց0 , ǫ 2 ց0 1+ǫ1 (3 − x) · (x − 1)
Faço: Z 3−ǫ2
1
p dx =
1+ǫ1 (3 − x) · (x − 1)
Z 3−ǫ2
1
= p dx =
1+ǫ1 1 − (x − 2)2
Z 1−ǫ2
1
= du = √
−1+ǫ1 1 − u2
= arcsin(1 − ǫ2 ) − arcsin(−1 + ǫ1 ).
Então Z 3−ǫ2
1
lim dx =
p
ǫ1 ց0 , ǫ2 ց0 1+ǫ1 (3 − x) · (x − 1)
= lim [arcsin(1 − ǫ2 ) − arcsin(−1 + ǫ1 )] =
ǫ1 ց0 , ǫ2 ց0
2. AS PRIMEIRAS TRANSFORMADAS DE LAPLACE, A FUNÇÃO GAMA E
O FATORIAL 392
π π
− (− ) = π,
=
2 2
onde na última linha usei que arcsin(u) é contı́nua em todo [−1, 1], apesar de ser
derivável apenas em (−1, 1).
Na segunda, temos:
Z +∞ Z a
1 1
x+1 3−x
dx := lim x+1
dx.
1 e +e a→+∞ 1 e + e3−x
Agora faço:
1 1 1
= 1 = 2x−2 =
ex+1 + e3−x ex+1 + ex−3 ( e ex−3+1 )
ex−3 −2 ex−1
= 2x−2 = e · x−1 2
e +1 (e ) + 1
x−1
e integro via a substituição u = e :
Z a
−2 1
e · 2+1
du = e−2 · (arctan(a) − arctan(1))
1 u
e portanto:
π
lim e−2 · (arctan(a) − arctan(1)) = e−2 · ( lim arctan(a) − )=
a→+∞ a→+∞ 4
π π π
= e−2 · ( − ) = 2,
2 4 4e
o resultado.

2. As primeiras Transformadas de Laplace, a função Gama e o fatorial


Afirmação 2.1. Seja k ∈ R, k > 0.

i):
Z +∞
1
e−kx · dx =
0 k
ii): Suponha f : [0, +∞] → R contı́nua, f (x) ≥ 0 e que existam a, C, M > 0 tais
que
f (x) ≤ C · eax , ∀x ≥ M,
então existe a integral imprópria
Z +∞
e−kx f (x)dx
0
para qualquer k > a.
Demonstração.
Temos Z Z
+∞ +∞
−kx
e dx := lim e−kx dx =
0 b→+∞ 0
CAPÍTULO 27. INTEGRAIS IMPRÓPRIAS 393
Z +∞
e−kb 1 1
= lim + )= . (
b→+∞ 0 −kb k k
Para a segunda afirmação, escrevo para k > a:
Z +∞ Z M Z +∞
−kx −kx
e f (x)dx = e f (x)dx + e−kx f (x)dx
0 0 M
RM −kx
onde a primeira integral 0 e f (x)dx existe pois o integrando é uma função contı́nua.
Precisamos ver se existe
Z b
e−(k−a)M −kx
lim C· e f (x)dx.
b→+∞ M (k − a)
Primeiro observo que Z b
lim e−kx f (x)dx
b→+∞ M
não cresce arbitrariamente.
Ora, usando as hipóteses:
Z b Z b
−kx
lim e f (x)dx ≤ C · lim e−kx eax dx
b→+∞ M b→+∞ M
Z b
= C · lim e−(k−a)x dx =
b→+∞ M
−(k−a)b −(k−a)M
e e e−(k−a)M
= C · lim ( + )=C· .
b→+∞ −(k − a) (k − a) (k − a)
Rb −kx
Como M
e f (x)dx é uma função crescente de b (pois e−kx f (x) ≥ 0), então:
Z b
e−(k−a)M
e−kx f (x)dx ≤ C · , ∀b ≥ M.
M (k − a)
Isso garante1 que existe Z b
lim e−kx f (x)dx.
b→+∞ M


As integrais impróprias do item ii):


Z +∞
e−kx f (x)dx,
0
para qualquer k > a, são chamadas Transformadas de Laplace da f (x).

Portanto o item i) deu as Transformadas de f (x) ≡ 1, que são k1 .

A Afirmação 2.2 a seguir pode ser lida do seguinte modo:


para k = 1, a Transformada de Laplace de f (x) = xn é igual a n! (fatorial).

1deixo detalhes mais próprios de cursos de Análise


2. AS PRIMEIRAS TRANSFORMADAS DE LAPLACE, A FUNÇÃO GAMA E
O FATORIAL 394

Afirmação 2.2. Para n ∈ {0} ∪ N:


Z +∞
e−x xn dx = n!
0
Demonstração.
Para n = 0 uma aplicação imediata do Teorema Fundamental dá que:
Z b
lim e−x dx = lim (−e−b + 1) = 1.
b→+∞ 0 b→+∞

Para prová-la para n = 1, integro por partes:


Z +∞ Z b
−x
e x dx = lim e−x x dx =
0 b→+∞ 0
Z b
= lim [−e−b b −e−x dx] =
b→+∞ 0
Z b
−b
= − lim e b − lim e−x dx =
b→+∞ b→+∞ 0
= 0 − (−1) = 1.
Supondo válido até n − 1 a fórmula:
Z +∞
e−x xn−1 dx = (n − 1)!
0
obtemos Z Z
+∞ b
−x n
e x dx = lim e−x xn dx =
0 b→+∞
Z b 0
= lim [−e−b bn − n e−x xn−1 dx] =
b→+∞ 0
= 0 − n · (n − 1)! = n!


Definimos o valor da Função Gama em cada n + 1 por


Z +∞
Γ(n + 1) := e−x xn dx = n!
0
Afirmação 2.3. Para todo p ∈ R, p > −1, existe a integral imprópria:
Z +∞
e−x xp dx.
0
Demonstração.
Se p > 0, o conhecido limite
lim xp+2 · e−x = 0
x→+∞

implica que
xp 1
x
< 2,
e x
CAPÍTULO 27. INTEGRAIS IMPRÓPRIAS 395

se x > K (suficientemente grande).


Então para esse K > 0 escrevo:
Z +∞ Z K Z +∞
−x p −x p
e x dx = e x dx + e−x xp dx.
0 0 K

A integral de 0 até K existe pois p > 0. Mas para vermos que existe também a
integral
Z +∞
e−x xp dx
K

escrevo, para x > K:


Z +∞ Z +∞
−x p 1
e x dx ≤ dx < +∞
K K x2
(esta última conhecida da Seção 27 do Capı́tulo 23.)
Se
−1 < p < 0
o problema agora na integral
Z +∞
e−x xp dx
0

é quando x ց 0.
Faço, para 0 < a < J, a integração por partes:
Z J p+1 p+1 Z J
−x p −J J −a a xp+1
e x dx = e −e + e−x dx
a p+1 p+1 a p+1
e observo que agora
Z J p+1 p+1 Z J
−x p −J J −a a xp+1
e x dx = e − lim [e + e−x dx]
0 p + 1 aց0 p+1 a p+1
e esses limites existem pois 0 < p + 1.


Portanto o valor da Função Gama em cada p ∈ R, p > −1, é dado por


Z +∞
Γ(p + 1) := e−x xp dx
0

O mesmo argumento dado na prova da Afirmação 2.2 dá agora que:

Γ(p + 1) = p · Γ(p), ∀p ∈ R, p > 0.


4. EXERCÍCIOS 396

3. Fórmula de Euler para o fatorial


Afirmação 3.1. (L. Euler, 1730)
Z 1
n! = (− ln(u))n du.
0

Demonstração.
Com a substituição:
x := − ln(u) ou seja u = e−x , du = −e−x dx,
temos Z Z Z
1 0 +∞
n n −x
(− ln(u)) du = x (−e ) dx = xn e−x dx = n!
0 +∞ 0
onde na última igualdade usei a Afirmação 2.2.


4. Exercı́cios
x −x
Exercı́cio 4.1. Defina cosh(x) := e +e
2
, o cosseno hiperbólico.
Para a > 0 e k > a, mostre que a Transformada de Laplace:
Z +∞
e−kx cosh(ax)dx
0
k
vale k 2 −a2
.
Exercı́cio 4.2. Mostre que:
Z +∞
1
dx = +∞,
2 ln(x)
apesar de que
1
lim = 0.
x→+∞ ln(x)
CAPı́TULO 28

A curvatura dos gráficos

1. O comprimento de um gráfico
Considere o gráfico de uma função f : [a, b] → R. Gostarı́amos nesta Seção de
definir e calcular o comprimento desse gráfico.
Na prática imagine uma curva feita de um material não-elástico, como um arame,
que queremos desentortar e calcular seu comprimento.
Considere uma partição
a = t0 < t1 < . . . < tn = b
do domı́nio [a, b] e considere o comprimento da poligonal inscrita no gráfico de f
formada de n segmentos:
p p
pn := (t1 − t0 )2 + (f (t1 ) − f (t0 ))2 + . . . + (tn − tn−1 )2 + (f (tn ) − f (tn−1 ))2 .
Ou seja,
s s
f (t1 ) − f (t0 ) 2 f (tn ) − f (tn−1 ) 2
pn = 1+( ) · (t1 − t0 ) + . . . + 1+( ) · (tn − tn−1 ).
t1 − t0 tn − tn−1
Se usamos em cada sub-intervalo [ti−1 , ti ] da partição o Teorema do Valor Médio
de Lagrange, então:
f (ti ) − f (ti−1 )
= f ′ (ξi ), ξi ∈ (ti−1 , ti ).
ti − ti−1
Então
p p
pn = 1 + (f ′ (ξ1 ))2 · (t1 − t0 ) + . . . + 1 + (f ′(ξn ))2 · (tn − tn−1 ).
Refinando a partição esperamos estar inscrevendo uma poligonal cujo tamanho
cada vez mais aproxima o tamanho do gráfico de f . A passagem ao limite n → +∞,
com a norma da partição de [a, b] tendendo a zero, sugere que definamos
Definição 1.1. Suponha um gráfico de f : [a, b] → R, com f derivável e f ′ (x) uma
função contı́nua.
O comprimento do gráfico de (a, f (a)) até (b, f (b)) será definido pela integral
Z bp
1 + f ′ (x)2 dx.
a

A primeira coisa que vemos nessa Definição 1.1 é que provavelmente em muitos
casos não será fácil calcular esse comprimento, pois dará uma integral complicada (às
vezes irredutı́veis a funções elementares).
397
1. O COMPRIMENTO DE UM GRÁFICO 398

Mas como f ′ (x) é contı́nua se vê que de qualquer forma existe a integral que dá
o comprimento.
Exemplos:
• No caso y = f (x) = A · x + B uma reta, nossa definição é apenas o conteúdo
do teorema de Pitágoras:
Z bp √
1 + f ′ (x)2 dx = 1 + A2 · (b − a) =
a
p p
= (b − a)2 + (A(b − a))2 = (b − a)2 + (Ab + B − Aa − B))2 .
• No caso y = x2 já não é tão evidente quanto mede seu gráfico:
Z bp Z b√
′ 2
1 + f (x) dx = 1 + 4x2 dx.
a a
Faço:
u = 2x, e du = 2dx
e Z Z 2b √
b √ 1
1+ 4x2
dx = · 1 + u2 du.
a 2 2a

Uma primitiva de 1 + u2 é
u√ 1 √
1 + u2 + ln(u + 1 + u2 ).
2 2
Logo:
Z b√
1 2b √ 1 √
1 + 4x2 dx = · [ · 1 + 4b2 + ln(2b + 1 + 4b2 )−
a 2 2 2
2a √ 1 √
− · 1 + 4a2 − ln(2a + 1 + 4a2 )].
2 2
Para a = 0, b = 1 isso dá:
1 √ 1 √
· [ 5 + ln(2 + 5)] ∼ 1.478942857
2 2

• Como o segmento de reta de (0, 0) a (1, 1) mede 2 ∼ 1.414213562, e como
3
x2 < x 2 < x, se x ∈ [0, 1],
3
é natural que o comprimento do gráfico de y = x 2 de x = 0 até x = 1 seja
um valor entre 1.414213562 e 1.478942857.
De fato,
Z bp Z 1r
3 1
1 + f ′ (x)2 dx = 1 + ( x 2 )2 dx =
a 0 2
Z 1r
9
= 1 + x dx =
0 4
Z 13 3
4 4 √ 4 2 13 2
= · u du = · · [( ) − 1] ∼
9 1 9 3 4
CAPÍTULO 28. A CURVATURA DOS GRÁFICOS 399

∼ 1.439709873
m
• Note no exemplo anterior que, se tivéssemos tomado uma função do tipo x n
com (m, n) 6= (3, 2), não seria muito claro o que fazer. Cairı́amos na integral:
Z 1r
m2 m
1 + 2 · x2( n −1) dx
0 n
que não tem uma expressão através de funções conhecidas se (m, n) são escol-
hidos genéricamente. Veremos mais integrais intratáveis na Seção seguinte.

2. Um problema da Putnam Competition, n.2, 1939


Nem todos os problemas dessa competição são difı́ceis, este aı́ é bem direto:
Problema: Encontrar o comprimento da curva y 2 = x3 da origem até o ponto onde
a reta tangente faz um ângulo de 45 graus com o eixo dos x.

Solução:

Essa
√ curva associa √ a cada valor de x > 0√dois valores possı́veis de y, a saber:
y = x3 e y = − x3 . No ramo onde y = x3 estão localizados os pontos onde
a retas tangentes têm inclinação positiva. E como estamos buscando o ponto onde
a inclinação é 1 (pois queremos
√ 45 graus) podemos pensar que perto desse ponto a
curva é o gráfico de y = x . 3

Assim buscamos x > 0 que verifica:


′ 3x2 3 1
y (x) = p = x 2 = 1,
2 x 3 2
ou seja, 49 x = 1, que dá
4
x= .
9
Agora é só calcular:
Z 4 r Z 4 r
9 3 1 2 9 9
1 + ( x 2 ) dx = 1 + x dx =
0 2 0 4
Z 2
√ 4 4
= u du = · (F (2) − F (1))
1 9 9
3
2
onde F (u) = 3
· u2.

3. Curvas parametrizadas e seu vetor velocidade


Será muito útil mais adiante trabalharmos também com curvas parametrizadas,
ou seja, com aplicações
Γ : R → R2 , (x(t), y(t)), t ∈ [a, b]
que supomos ter coordenadas x(t) e y(t) deriváveis.
3. CURVAS PARAMETRIZADAS E SEU VETOR VELOCIDADE 400

O traço de uma curva parametrizada Γ é o conjunto imagem Γ([a, b]). Observo


que nem sempre Γ([a, b]) é gráfico de alguma função; por exemplo, Γ([0, 2π]) é um
cı́rculo inteiro, quando tomamos

Γ : R → R2 , (cos(t), sin(t)), t ∈ [0, 2π]

O vetor velocidade de Γ é definido por:

Γ′ (t0 ) := ( x′ (t0 ), y ′(t0 ) ).

Note que:
x(t0 + h) − x(t0 ) y(t0 + h) − y(t0 )
Γ′ (t0 ) := ( lim , lim ,)=
h→0 h h→0 h

1
= lim · [ (x(t0 + h), y(t0 + h)) − (x(t0 ), y(t0 ))],
h
h→0

onde a última igualdade é um pouco mais que uma definição.


A Figura a seguir ilustra os vetores

Γ(t0 ) = (x(t0 ), y(t0 )), Γ(t0 + h) = (x(t0 + h), y(t0 + h)) e Γ(t0 + h) − Γ(t0 ).

Γ ( t_0 + h )

Γ ( t_0 )

_
Γ ( t_0 + h ) Γ ( t_0 )

1
A próxima ilustra a posição limite de h
· (Γ(t0 + h) − Γ(t0 )), ou seja, Γ′ (t0 ).

Γ ( t_0 )

Γ ( t_0 )

E a Figura a seguir ilustra


Γ(t0 ) + Γ′ (t0 )
como vetor que pertence à reta tangente de Γ no ponto Γ(t0 ) = (x(t0 ), y(t0)).
CAPÍTULO 28. A CURVATURA DOS GRÁFICOS 401

Γ ( t_0 ) + Γ ( t_0 )

Γ ( t_0 )

Γ ( t_0 )

4. Integrais que ninguém pode integrar


Para curvas parametrizadas
Γ : R → R2 , (x(t), y(t)), t ∈ [a, b]
podemos definir seu comprimento por:
Z bp
s := (x′ (t)2 + (y ′ (t))2 dx.
a
Fazer integrais é um artesanato, onde é preciso ter um pacote de integrais conheci-
das e tentar recair numa dessas através de uma técnica ou outra (substituição , por
partes, etc.) Porém existem integrais que não tem uma primitiva razoável,elementar
como se costuma chamar. E essas integrais indomáveis rondam as conhecidas ...
Vejamos um exemplo fundamental.
Quando parametrizamos um cı́rculo de raio a > 0 por
(a cos(t), a sin(t))
seu comprimento é dado por:
Z 2π p Z 2π
2 2 2 2
a sin(t) + a cos(t) dt = a · dt = 2πa.
0 0
x2 y 2
Porém se nosso cı́rculo vira uma elipse a2
+ b2 = 1 com a > b, então uma parametrização é:
(a cos(t), b sin(t))
e seu comprimento é:
Z 2π q Z 2π q
2
2 2 2
a sin (t) + b cos (t) dt = a2 sin2 (t) + b2 (1 − sin2 (t)) dt =
0 0
Z 2π q
b2 + (a2 − b2 ) · sin2 (t) dt =
0
Z 2π r
a2
=b· 1 − (1 − 2 ) · sin2 (t) dt.
0 b
Eis uma integral sem primitiva elementar, chamada de integral elı́ptica.
O que se faz é dar aproximações dessa integral, desde uma bem inocente:
a+b
≈2·π·( )
2
5. VELOCIDADE DE UM GRÁFICO OU DE UMA CURVA 402

até uma que exige o gênio de S. Ramanujan:


p
≈ π · (3 · (a + b) − (a + 3b)(3a + b)).
Veremos na Seção 42 do Capı́tulo 40 que a função:
Z πq
2
E(x) := 1 − x2 · sin2 (t)dt
0

satisfaz uma equação diferencial e depois que tem um desenvolvimento em série in-
finita, cujos truncamentos darão portanto aproximações do comprimento da elipse,
que é, pela sua simetria:
r
a2
= 4 · b · E( 1 − 2 ).
b

5. Velocidade de um gráfico ou de uma curva


Como pelo Primeiro Teorema do Cálculo:
p Z xp
′ 2
1 + (f (x)) = ( 1 + f ′ (t)2 dt )′
a

é natural denotarmos
ds p
= 1 + (f ′ (x))2 .
dx
Essa grandeza será chamada velocidade do gráfico no instante x.
Note que sempre
ds
>0
dx
o que diz o comprimento do gráfico sempre é uma função estritamente crescente. E
ademais, isso diz que existe uma função inversa: x = x(s). Logo dado um compri-
mento desde f (a) = A determino univocamente x e daı́ um único ponto no gráfico.
Portanto existe uma função bem definida P = P (s) que descreve os pontos do gráfico.
Para curvas parametrizadas
Γ : R → R2 , (x(t), y(t)), t ∈ [a, b]
seu comprimento foi definido por:
Z bp
s := (x′ (t)2 + (y ′ (t))2 dx.
a

Como Γ′ (t) := (x′ (t), y ′(t)) é o vetor tangente a Γ então


Z b
s= || Γ′(t) || dt.
a

Também é natural considerar:


ds p
= || Γ′(t) || = (x′ (x)2 + (y ′(x))2 .
dt
CAPÍTULO 28. A CURVATURA DOS GRÁFICOS 403

6. Definição de curvatura e sua fórmula


A noção intuitiva de curvatura é a de uma medida de quanto mudam as direções
das retas tangentes (em relação a algum eixo fixado como referência).
Mas, para que a curvatura de um gráfico G seja um conceito geométrico, vamos
defini-la como uma medida de quanto mudam as direções das tangentes num trecho
de um gráfico em relação a quanto vale o comprimento da porção do gráfico.
Como critério de adequação de um possı́vel definição exigiremos que um cı́rculo
Cr de raio r tenha curvatura constante e de fato κ = 1r (para que os cı́rculo muito
grandes se curvem muito pouco).
Essa exigência é natural, pois quando percorremos todo o cı́rculo, percorremos
s = 2πr e o ângulo θ formado pelas retas tangentes variou 2π. Logo
∆θ 1
κ(Cr ) :=
= .
∆s r
Para motivarmos a Definição e Fórmula 6 abaixo, considero θ = θ(s) uma função
que mede como varia o ângulo formado pelas direções tangentes em relação ao com-
primento do gráfico percorrido.
Então a regra da derivada da composta diz1:
d tan(θ(s)) d tan(θ(s)) d θ(s)
= · =
ds dθ ds
d θ(s)
= sec2 (θ(s)) · .
ds
Por outro lado,
dy
(x(s)) = tan(θ(s))
dx
e a regra da composta dá:
d tan(θ(s)) d dd xy (x(s)) d x
= · (s) =
ds dx ds
d2 y dx
2
=(x(s)) · (s).
dx ds
A taxa de variação que queremos para definir curvatura é
d θ(s)
.
ds
Até agora temos:
d2 y
d θ(s) dx2
(x(s)) · dd xs (s)
= .
ds sec2 (θ(s))
Mas definimos na Seção 1 anterior:
Z r
x
dy 2
s(x) := 1+( ) dt,
a dx
1A notação de Leibniz deixa mas claro em relação a que variável derivamos
6. DEFINIÇÃO DE CURVATURA E SUA FÓRMULA 404

ou seja, pelo Primeiro Teorema do Cálculo:


s
ds dy 2
(x) = 1 + ( ) .
dx dx
Pela derivada da função inversa teremos:
dx 1
(s) = q .
ds 2
1+ ( dd xy )
E também podemos escrever:
r
dy 2
sec(θ(s)) = 1+( ) .
dx
Logo obtivemos:
d2 y
d θ(s) 2 (x(s))
= dx d y 3 .
ds (1 + ( d x )2 ) 2
Essa é a justificação da seguinte definição:
Definição 6.1. A curvatura2 do gráfico de y = f (x) é:
2
| ddx2y |
κ(x) := 3 .
(1 + ( dd xy )2 ) 2
A Figura a seguir dá um exemplo de como varia a curvatura:

0
-2 -1 0 1 2
x

Figura: Em vermelho y = x2 e em verde sua função curvatura.

Observação 6.1. Note que acima obtivemos:


dx
= cos(θ(s)).
ds
Como
dy
(x(s)) = tan(θ(s))
dx
2por enquanto não nos interessa ter sinais, por isso tomamos o módulo
CAPÍTULO 28. A CURVATURA DOS GRÁFICOS 405

então a regra da composta dá:


dy dy dx
= ·
ds dx ds
ou seja:
dy
= sin(θ(s)).
ds
Novamente, no caso de uma curva parametrizada, podemos estender a Definição
6.1 para:
Definição 6.2. Se
Γ : R → R2 , (x(t), y(t)), t ∈ [a, b]
é uma curva parametrizada então sua curvatura é dada por:
| x′ (t)y ′′(t) − x′′ (t)y ′(t) |
κ(t) := 3 .
(x′ (t)2 + y ′ (t)2 ) 2

Note que esta Definição 6.2 é realmente é uma estensão da Definição 6.1, pois
quando t = x, temos x′ (x) ≡ 1 e x′′ (x) ≡ 0.

7. Qual a curvatura de uma quina ?


A curvatura de uma reta certamente é zero, já que a segunda derivada é zero.
Mas numa linha quebrada, formada de pedaços de retas, que curvatura faria sentido
associar à um ponto que é uma quina ??
Após a Afirmação seguinte daremos uma resposta:
Afirmação 7.1. Considere um braço de hipérbole:
ǫ
y = fǫ (x) = , ∀x > 0,
x
onde ǫ > 0 é fixado. Então:
3
i) sua função curvatura é κ(x) = 42ǫ·x2 3 .
(x +ǫ ) 2
ii) limx→+∞ κ(x) = 0 e limxց0 κ(x) = 0. √
iii) o ponto de máximo de κ(x) é em x = ǫ. Nele a curvatura é:

2
√ .
2 ǫ

iv) limǫց0 κ( ǫ) = +∞.
Demonstração.
A função curvatura é para x > 0:

x3 2ǫ · x3
κ(x) = 2 3
= 3 .
(1 + xǫ 4 ) 2 (x4 + ǫ2 ) 2
Portanto:
2ǫ · x3 x3
lim 3 = lim =0
x→+∞ (x4 + ǫ2 ) 2 x→+∞ x6
7. QUAL A CURVATURA DE UMA QUINA ? 406
1 1
e, já que limxց0 3 = ǫ3
> 0, então claramente
(x4 +ǫ2 ) 2

2ǫ · x3
lim 3 = 0,
xց0 (x4 + ǫ2 ) 2
Para buscarmos mı́nimo de κ(x) a derivamos:
−6 ǫ · x2 · (x4 − ǫ2 )
κ′ (x) = ,
(x4 + ǫ2 )5/2
e vemos que:

κ′ (x) > 0 se 0 < x < ǫ,

κ′ (x) = 0 se x = ǫ,

κ′ (x) < 0 se ǫ<x

o que diz nitidamente que x = ǫ é o ponto de máximo de k(x). Que nele vale:

√ 2
κ( ǫ) = √ .
2 ǫ


A Figura a seguir dá o gráfico da curvatura para ǫ = 1:

2,5

1,5

0,5

0
0,5 1 1,5 2 2,5 3 3,5 4
x

1 √1
Figura: O gráfico de y = x
(vermelho), sua κ(x) (verde) e o valor y = 2
em azul

Quando ǫ → 0 o ponto x = ǫ tende a x = 0, assim como todo o gráfico de
y = fǫ (x) = xǫ tende à união de retas x · y = 0, pois:
y·x =ǫ
ao longo do gráfico de y = fǫ (x).
E pelo item iv) da Afirmação 7.1:

lim κ( ǫ) = +∞
ǫց0
CAPÍTULO 28. A CURVATURA DOS GRÁFICOS 407

Assim se fôssemos atribuir um valor de curvatura a (0, 0) como ponto da união de


retas
y·x=0
deverı́amos pôr: κ = +∞.
CAPı́TULO 29

Séries convergentes

1. Séries k-harmônicas, k > 1.


Consideremos novamente a Afirmação 0.1 do Capı́tulo 27, que dizia que:
Z +∞
1 1
dx = .
1 xk k−1
Essa é a área da região à direita de 1 sob o gráfico de y = x1k . Note que essa área
é maior que a soma de áreas dos retângulos justapostos
1 1 1
[1, 2] × [0, k ] ∪ [2, 3] × [0, k ] ∪ . . . ∪ [n, n + 1] × ∪[0, ]...
2 3 (n + 1)k
onde os três pontos significam que podemos ir colocando sempre retângulos à direita.
Mas a área desses retângulos todos é (ainda num sentido vago) uma soma infinita:
1 1 1
+ + . . . + ...
2k 3k nk
Pela Afirmação 0.1 -i), com a = 1 temos:
1 1 1 1
∀n ∈ N, k
+ k + ...+ k < .
2 3 n k−1
O que significa essa soma infinita:
1 1 1
k
+ k + ...+ k ... ?
2 3 n
Simplesmente quer dizer que existe o limite da sequência xn dada por
1 1 1
xn := k + k + . . . + k , k ≥ 2.
2 3 n
Aqui é importante que k ≥ 2, pois pelo que vimos na prova da Afirmação 6.1 a
soma infinita
1 1 1
+ + ...+ ...
2 3 n
tem um comportamento diferente, ela fica tão grande quanto quisermos.
Definição 1.1. As séries 21k + 31k + . . . + n1k . . . são chamadas k-harmônicas. A série
1-harmônica 21 + 31 + . . . + n1 . . . é chamada apenas de harmônica.
Como a Afirmação 0.1 diz que
1
∀n ∈ N, xn <
k−1
409
1. SÉRIES K-HARMÔNICAS, K > 1. 410
1
dizemos que a sequência (xn )n é limitada superiormente por k−1 (a definição de lim-
itada infeiormente é análoga). E nitidamente é crescente, ou seja:
xn ≤ xn+1
1
pois xn+1 = xn + (n+1)k
(a definição de decrescente é análoga).

Então a nossa (xn )n é um exemplo de sequência limitada superiormente e cres-


cente, se
1 1 1
xn := k + k + . . . + k , k ≥ 2.
2 3 n
A seguir dou princı́pios gerais e úteis para sequências e séries:
Teorema 1.1. i) toda sequência (xn )n limitada superiormente e crescente tem
lim xn .
n→+∞

ii) toda sequência (xn )n limitada inferiormente e decrescente tem


lim xn .
n→+∞

P+∞ P+∞
iii) sejam i=1 ai e i=1 bi com
0 < ai ≤ bi , ∀i ∈ N.
P+∞ P
Se i=1 bi converge também +∞ a converge.
P+∞ P+∞ i=1 i
Se i=1 ai diverge então i=1 bi diverge.
Demonstração.
A prova dos itens i) e ii) se discute em cursos de Análise matemática. A prova
não dá nenhuma pista em geral dePquanto vale esse limite, apenas que existe.
Já iii) segue de i): de fato, se +∞ i=1 bi converge então em particular fica limitada,
por exemplo ≤ K.
Mas então sn := a1 + . . . + an é uma sequência crescente, pois ai > 0, e limitada,
já que
+∞
X
a1 + . . . + an ≤ bi ≤ K.
i=1
P
Logo converge +∞ i=1Pai por i).
Agora, quando +∞ i=1 ai diverge então sn := a1 + . . . + an forma uma sequência
de
P+∞ números de tamanho tão grande quanto quisermos (caso contrário i) diria que
i=1 ai converge). Mas então
b1 + . . . + bn ≥ a1 + . . . + an
também forma
P uma sequência de números de tamanho tão grande quanto quisermos.
Portanto +∞i=1 bi diverge.

CAPÍTULO 29. SÉRIES CONVERGENTES 411

Somente no Exercı́cio 7.1 do Capı́tulo 46 conseguiremos provar que:


π2 1 1 1
= 1+ 2 + 2 + 2 + ...
6 2 3 4
2. A série geométrica
Afirmação 2.1. Seja r um número Real, com 0 ≤ |r| < 1. Defina a sequência cujo
xn := 1 + r + r 2 + . . . + r n . Então
n+1
• i) ∀n ∈ N, xn = 1−r 1−r
.

• ii) limn→+∞ |r|n = 0 e limn→+∞ r n = 0.


1
• iii) limn→+∞ xn = 1−r n
.
Demonstração.
Claro que se |r| = 0 então r = 0 e tudo que afirmamos é obviamente válido. Logo
no que segue 0 < |r| < 1.

Prova de i), por indução:


1−r 2
Se n = 1, então de fato vale 1 + r = 1−r
. Supondo a fórmula até n − 1:
1 − rn
1 + r + r 2 + . . . + r n−1 =
1−r
e
1 − r n r n · (1 − r)
1 + r + r 2 + . . . + r n−1 + r n = + =
1−r 1−r
1 − r n+1
= .
1 − rn
Para provar ii), note que 0 < |r| < 1 implica (multiplicando por r positivo):
0 < |r|2 < |r| < 1,
e assim obtemos por indução:
0 < |r|n < |r|n−1 < 1, ∀n ∈ N
Mas então a sequencia (|r|n )n é decrescente e obviamente limitada inferiormente pelo
0. Pelo Teorema 1.1) existe
lim |r|n = L.
n→+∞
Mas afirmo que L = 0 (a principio seria apenas 0 ≤ L ≤ |r| < 1).
Meu argumento agora usará uma analogia1: se uma fila completa de pessoas tende
a um lugar, as pessoas nas posições pares também tendem a esse lugar.
Ou seja, quero dizer que:
lim |r|n = L ⇒ lim |r|2n = L.
n→+∞ n→+∞

1Rigorosamente trata-se de argumentar com uma subsequência da sequência toda


3. O TESTE DA RAZÃO (QUOCIENTE) 412

Por outro lado


lim |r|2n = lim (|r|n )2
n→+∞ n→+∞
e pelo limite de produtos de sequências:
lim (|r|n )2 = lim |r|n · lim |r|n = L2 .
n→+∞ n→+∞ n→+∞

Então L = L2 . Logo L(L − 1) = 0 e L = 0 ou L = 1. Mas


|r|n < |r| < 1.
impede que seja L = 1, ou seja, temos L = 0.
Bom agora só resta obervar que também limn→+∞ r n = 0. Mas o que significa
limn→+∞ r n = 0 ? Significa que se n é suficientemente grande temos para qualquer ǫ
dado:
|r n − 0| < ǫ,
ou seja, pelas propriedades do módulo:
|r n | = |r|n < ǫ.
Mas temos já provado que
lim |r|n = 0
n→+∞
e isso diz que se n é suficientemente grande temos para qualquer ǫ dado:
| |r|n − 0 | < |r|n < ǫ,
como querı́amos. ou seja:

Prova de iii):
Do item i) já temos que
1 − r n+1
xn = , ∀n ∈ N
1−r
e do item ii) temos limn→+∞ r n = 0. Com as propriedades de limites de somas/produtos
obtemos:
1 − limn→+∞ r n 1
lim xn = = .
n→+∞ 1−r 1−r


3. O teste da razão (quociente)


Afirmação
P 3.1. (Teste da razão para séries positivas)
Seja +∞i=1 ai com 0 < ai e suponha que existe:
ai+1
lim = L.
i→+∞ ai
P P+∞
Se L < 1 a série +∞i=1 ai converge, mas se L > 1 a série i=1 ai diverge. Se L = 1
o teste nada afirma em geral.
CAPÍTULO 29. SÉRIES CONVERGENTES 413

Demonstração.
ai+1
No caso 1 > L := limi→+∞ ai
tomamos

1−L
ǫ := >0
2
e podemos supor, a partir de um certo i0 que
ai+1
∈ (−ǫ + L, L + ǫ), ∀i ≥ i0 ,
ai
ou seja,
ai+1
< r < 1 ∀i ≥ i0 .
ai
Então
ai0 +1 < r · ai0 , ai0 +2 < r · ai0 +1 < r 2 ai0
etc até que
ai0 +j < r j · ai0 , ∀j ∈ N.
P P+∞ j
Mas a série +∞i=1 r j
· ai 0 = ai 0 · i=1 r é uma série geométrica convergente, pois
r < 1. Então pelo item iii) do Teorema 1.1 a série
+∞
X
ai0 +j
j=1

converge e portanto a série toda:


+∞
X i0
X +∞
X
ai = ai + ai0 +j
i=1 i=1 j=1

converge.
No caso L > 1 se lida com a desigualdade
ai+1
1<r< , ∀i ≥ i0
ai
e analogamente o item iii) do Teorema 1.1 dará agora que
+∞
X
ai
i=1

diverge.

4. UM ARGUMENTO GEOMÉTRICO PARA A SÉRIE GEOMÉTRICA 414

4. Um argumento geométrico para a série geométrica


Arquimedes provava com um argumento geométrico que
1 1 1 1
+ ( )2 + ( )3 + . . . =
4 4 4 3
o que dá em seguida
1 1 1 1
1 + + ( )2 + ( )3 + . . . = 1 + =
4 4 4 3
4 1
= = ,
3 1 − 14
em perfeita concordância com nossa Afirmação 2.1.
Seu argumento é o seguinte. Tome um quadrado de lado 1 e inscreva nele um
quadrado de lado 21 (e área 14 portanto). a seguir a seguir é o maior quadrado em
vermelho. Note que à direita e acima desse quadrado vermelho há quadrados verde e
amarelos de mesma área 14 .

Figura: Três etapas do processo de Arquimedes

Agora justaponha ao quadrado vermelho um segundo quadrado vermelho, de lado


1
4
e área 412 = 16
1
, como mostra a figuraa seguir (note que aparecem então dois quadra-
1
dos de área 16 à direita e acima dele).
Assim sucessivamente, quadrados vermelhos de lado 21n e área 41n são justapostos,
∀n ∈ N.
Arquimedes argumenta que esse processo continuado preenche todo o quadrado
de lado 1 com infinitos quadrados vermelhos, verdes e amarelos. A soma das áreas
dos vermelhos é a mesma soma das áreas dos verdes e da dos amarelos. Mas então
1 1 1
3 · ( + 2 + 3 + . . .) = 1,
4 4 4
e portanto
1 1 1 1
+ 2 + 3 + ... = .
4 4 4 3
CAPı́TULO 30

Aproximação de Números e Funções importantes

Neste Capı́tulo mostro que o cálculo permite, através da iteração das operações
elementares +, −, /, x, obter aproximações com a precisão que se quiser de:
• funções fundamentais como arctan(x), ln(x), etc

• números como p (p primo), π, e = exp(1).
Ou seja, o Cálculo transforma a gente num McGiver , aquele personagem que
quase sem nenhum instrumento fabricava aparelhos incrı́veis em suas missões. Nós
só com as quatro operações faremos tudo (e aı́ a gente entende um pouco do que
acontece quando se usa uma calculadora cientı́fica ...).

1. Aproximações de raı́zes quadradas por números racionais

Pensando bem, é curiosa a nomenclatura números Reais, pois esses números não
estão próximos da nossa realidade nem são dados de forma natural. Quem aparece no
dia-a-dia são os Naturais, os Inteiros e os Racionais, esses sim presentes nas operações
matemáticas mais simples do dia a dia.
Quando falamos números Reais estamos nos referindo a um conjunto de números
muito maior que o conjunto dos números Racionais (isso s eprova nos cursos de
Análise
√ Matemática). Apesar de que só saibamos citar um ou outro exemplo decor :
2, π, etc.
De fato quando Arquimedes se refere a π no seu trabalho A medida do cı́rculo,
ele o define como quociente entre o perı́metro e o diâmetro de um cı́rculo. Ele não
prova que π ∈ / Q, mas por outro lado dá um método para aproximá-lo tanto quanto
se quiser por números racionais. E seu método, √ que é geométrico, usa em certos
momentos aproximações de números como 3 por números Racionais.
Essa é uma visão muito interessante (como todas as do gênio Arquimedes) de que
números Reais são limites de sequências de números Racionais. Um ponto de vista
bastante útil e prático para as aplicações da matemática e ao mesmo tempo um ponto
de vista que, convenientemente adaptado produz um construção lógica dos Reais (um
pouco mais adiante volto nisto).

2. Raı́zes quadradas que são irracionais

Que tal √
primeiro nos convercermos de que existem números Irracionais, por ex-
emplo, que 2 ∈ /Q? √
Suponha por absurdo que sim 2 = pq , onde p, q ∈ N com mdc(p, q) = 1 (máximo

divisor comum é um). Ou seja, uso por ex. por absurdo 2 = 1/3 ao invés de 2/6.
415
3. COMO TIRAR RAÍZ QUADRADA SÓ COM +, −, ×, / 416
2
Mas então obtenho: 2 = pq2 e portanto: 2 · q 2 = p2 . O número Natural p se escreve
como um produto de números primos, e nesse produto o fator 2 aparece um c k ≥ 0
de vezes. Por ex. no 12 = 22 · 3 o fator 2 aparece k = 2 vezes. Mas em p2 há 2k
fatores 2 e 2k é sempre um número Par. Por outro lado p2 = 2 · q 2 e na decomposiçao
do número 2 · q 2 em primos, o √ fator 2 aparece um número Ímpar de vezes. Essa
contradição surgiu de supor que 2 é racional. √
Se olharmos bem o argumento que demos para convencernos √ que 2∈
/ Q, notamos
que serviria para provar que qualquer número primo P tem P ∈ / Q.

3. Como tirar raı́z quadrada só com +, −, ×, /


Vamos aplicar alguns itens do Teorema 3.1 do Capı́tulo 4, que dá propriedades d
elimites de sequências, para fazer uma mágica.
Tome um número positivo A. Tome um número positivo arbitrário, qualquer
x > 0 e defina
x0 := x
e
1 A
x1 := · (x + ).
2 x
Daı́ em diante, recursivamente, defina
1 A
xn := · (xn−1 + )
2 xn−1
Afirmação 3.1. 1
Se a sequência
1 A
· (xn−1 +
xn := )
2 xn−1
tem limn→+∞ xn = L > 0 então de fato

L= A
(a raı́z positiva de A).

√ √
Em particular, se A for um número Irracional como por exemplo 2 e se x for
Racional, então estamos dando um método para aproximar o número irracional pelos
números Racionais
1 A
xn := · (xn−1 + ).
2 xn−1
Demonstração.
Para começarmos a prova da Afirmação 3.1, argumentaremos através de uma
analogia.2
1Uma afirmação mais forte - e verdadeira - é de que de fato a sequência definida recursivamente
tem um limite L e esse limite é um número positivo.
2Rigorosamente trata-se de argumentar com uma subsequência da sequência toda
CAPÍTULO 30. APROXIMAÇÃO DE NÚMEROS E FUNÇÕES IMPORTANTES
417

Imagine uma fila de pessoas e que a fila se move para algum lugar. Então vemos
elemento n-ésimo caminhando em direção a esse lugar e o elemento (n − 1)-ésimo que
o segue para lá. Isso quer dizer em linguagem do dia a dia que:

se limn→+∞ xn = L (como supomos) então limn→+∞ xn−1 = L também.


Para provar a Afirmação toda, note que o Teorema 3.1 do Capı́tulo 4 vai dando,
já que limn→+∞ xn−1 = L :
1 1
lim = ,
n→+∞ xn−1 L
A 1 A
lim =A· = ,
n→+∞ xn−1 L L
A 1
lim (xn−1 + )=L+
n→+∞ xn−1 L
1 A 1 1
lim · (xn−1 + ) = · (L + ).
n→+∞ 2 xn−1 2 L
Mas temos
1 A
xn = · (xn−1 + )
2 xn−1
e limn→+∞ xn = L; logo juntando temos:
1 A
L = · (L + ),
2 L
de onde obtemos
L2 + A
2L =
L √
2
e portanto L = A; como L > 0 temos que L = A.


Fiz um exemplo na Calculadora, onde a cada etapa a calculadora faz truncamen-


tos.
2
Pondo A = 2 e ∀n ≥ 1, xn := 21 · (xn−1 + xn−1 ):
x0 := 390, x1 := 195.0025641 x2 := 97.50641019,
x3 := 48.76346084, x4 := 24.40223758, x5 := 12.24209864,
x6 := 6.202734661, x7 := 3.262586543, x8 := 1.937798551,
x9 := 1.484948789, x10 := 1.415898291, x11 := 1.414214565,
x12 := 1.414213562
e aqui a calculadora não sai mais desse número Racional, que para ela é a própria

2.
De onde saiu esse formato:
1 A
xn := · (xn−1 + )
2 xn−1
da sequência ?
4. OS REAIS ATRAVÉS DE SEQUÊNCIAS DE NÚMEROS RACIONAIS 418

Simplesmente note que é o formato dado pela Afirmação 0.1, do Capı́tulo 18 -


Método de Newton - para a função
f (x) = x2 − A,
pois:
f (xn−1 ) x2n−1 − A
xn = xn−1 − ′ = xn−1 − =
f (xn−1 ) 2 · xn−1
1 A
= · (xn−1 + ).
2 xn−1

4. Os Reais através de sequências de números Racionais

Como sabemos, não se pode ver um buraco negro, pelo motivo de que ele atrai
até mesmo os raios de luz. Então como os astrônomos podem estar tão seguros de
que existem esses misteriosos objetos?
O que eles vêem são estrelas sendo sugadas para um certa região, onde se acumu-
lam milhares de estrelas, apertando-se cada vez mais numa pequena região do espaço.
Daı́ deduzem que ali há um buraco negro.
Voltando ao nosso tema, se um sequência de números xn tende a um número L,
então os seus termos vão se aproximando entre si :
Afirmação 4.1. Suponha limn→+∞ xn = L. Então dado ǫ > 0 existe um nǫ tal que
∀n1 ≥ nǫ e ∀n2 ≥ nǫ , |xn1 − xn2 | < ǫ.
Demonstração.
Pela definiçao de limn→+∞ xn = L, dado ǫ > 0, existe nǫ tal que ∀n ≥ nǫ temos
|xn − L| < 2ǫ .
Então ∀n1 , n2 ≥ nǫ temos (pela desigualdade triangular):
|xn1 − xn2 | = |xn1 − L + L − xn2 | ≤
ǫ ǫ
≤ |xn1 − L| + |xn2 − L| < + = ǫ.
2 2


Podemos também inverter as coisas !


Que tal lidarmos inicialmente apenas com números Racionais e fazermos o seguinte:
cada vez que vemos uma sequência de números Racionais cujos termos se aproximam
entre si tanto quanto quisermos (como ocorre na conclusão da Afirmação 4.1), que
tal imaginarmos, postularmos, que ali há um número Real que os atrai ?

Chamaremos as sequências de números Racionais cujos termos se aproximam entre


si de sequências fundamentais.
Claro que pode acontecer que duas ou mais sequências fundamentais se acumulem
na mesma região, e as imaginamos estarem sendo atraı́das pelo mesmo número Real.
CAPÍTULO 30. APROXIMAÇÃO DE NÚMEROS E FUNÇÕES IMPORTANTES
419

Diremos que duas sequências fundamentais xn e x′n são equivalentes se


lim (xn − x′n ) = 0.
n→+∞

Isso sugere então pensar que:

cada número Real é uma classe de equivalência de sequências fundamentais.

5. Aproximações de e por números Racionais


Esta Seção está descrita de modo auto-suficiente, sem fazer apelo ao resultado da
Seção 12 do Capı́tulo 22. Claro que o leitor tema liberdade de supôr aquele resultado
e considerar esta Seção apaenas uma discretização daquela.
A prova da irracionalidade de e = exp(1) é dada com detalhes no livro do M.
Spivak, Calculus. Aqui o que discuto é como aproximá-lo por números Racionais.
Primeiro veremos uma sequência que o aproxima, mas o faz de modo bastante
lento, depois indicaremos outro modo de aproximá-lo, este sim rápido.

Sabemos pelo Teorema Fundamental e pela definição de logaritmo natural que:


1
ln′ (x) = , ∀x > 0
x
e portanto:
1
ln′ (1) = = 1.
1
Se olhamos isso pela definição de derivada o que temos é que
ln(1 + h) − ln(1) ln(1 + h)
1 = lim = lim .
h→0 h h→0 h
Mas se isso vale para quaisquer números h tendendo a zero, podemos tomá-los da
forma:
1
h= com n → +∞.
n
Ou seja que limh→0 ln(1+h)
h
= 1 vira
ln(1 + n1 ) 1
1 = lim 1 = lim n · ln(1 + ).
n→+∞
n
n→+∞ n
Pela propriedade de que
ln(xn ) = n · ln(x), ∀x > 0, ∀n ∈ N
obtenho:
1 n
1 = lim ln( (1 + ) ).
n→+∞ n
Suponha por um momento que a sequência xn := (1 + n1 )n tem um limite L.
Então como o ln(x) é uma função contı́nua tenho
1 n 1
lim ln( (1 + ) ) = ln( lim (1 + )n ) = ln(L).
n→+∞ n n→+∞ n
5. APROXIMAÇÕES DE E POR NÚMEROS RACIONAIS 420

Aplicando exponencial:
exp(1) = exp(ln(L)) = L,
ou seja concluı́mos que xn := (1 + n1 )n é uma sequência de Racionais tendendo ao e.
Vamos dar agora uma prova de que a sequência xn := (1 + n1 )n converge para um
número entre 2 e 3:
Afirmação 5.1. A sequência xn := (1 + n1 )n tem
1
lim (1 + )n = L, com 2 < L < 3.
n→+∞ n
Demonstração.
Basta verificar que que essa sequência é limitada superiormentemente por um
número menor que 3. Pois como é nitidamente crescente e x1 = 2, o Teorema 1.1
garantirá que ela converge.
Começo escrevendo pela fórmula do binômio:
n  
1 n X n 1 j
(1 + ) = ( ) =
n j=0
j n
1 n(n − 1) 1 1
=1+n· + 2
+ ... + n.
n 2! n n
Agora vamos escrever essa soma de um jeito adequado ao que segue:
1
(1 + )n =
n
1 n(n − 1) 1 n(n − 1)(n − 2) . . . 2 1
=1+n· + 2
+ ...+ =
n 2! n n! nn
1 1 1 1 2 n−2
= 1 + 1 + (1 − ) + . . . + (1 − )(1 − ) . . . (1 − ).
2! n n! n n n
Agora vamos dar quotas superiores para cada parcela desta soma, obtendo:
1 1 1 1 2 n−2
1 + 1 + (1 − ) + . . . + (1 − )(1 − ) . . . (1 − )<
2! n n! n n n
1 1
< 1 + 1 + + ...+ .
2! n!
Para darmos novas cotas superiores a essa soma lembro um Exercı́cio de Indução:
n! ≥ 2n−1 ∀n ∈ N.
Então
1 1 1 1
1+1+ + ...+ ≤ 1 + 1 + . . . + n−1 .
2! n! 2 2
ou seja, que (1 + n1 )n é sempre estritamente menor que
1 1
1+1+ . . . + n−1 .
2 2
É nı́tido que esta última soma é o resultado de adicionar 1 a um pedaço da série
geométrica infinita:
1 1
1 + . . . + n−1 + . . . ,
2 2
CAPÍTULO 30. APROXIMAÇÃO DE NÚMEROS E FUNÇÕES IMPORTANTES
421

que já vimos vale:


1 1 1
1+ . . . + n−1 + . . . = 1 = 2.
2 2 1− 2
Logo ∀n ∈ N:
1 n 1 1
(1 + ) < 1 + (1 + . . . + n−1 + . . .) = 3,
n 2 2
como querı́amos.


Fiz algumas contas no computador, obtendo os primeiros 10 valores (truncados


na 10 casa após a virgula) para xn := (1 + n1 )n :

x1 = 2, x2 = 2.250000000, x3 = 2.370370370, x4 = 2.441406250,


x5 = 2.488320000, x6 = 2.521626372, x7 = 2.546499697,
x8 = 2.565784514, x9 = 2.581174792, x10 = 2.593742460,
e assim por diante, se vê que a sequência vai crescendo lentamente. Tive que ir
até n = 120 para obter

x120 = 2.707041491.

Se pode provar que a sequência x′n := 1 + 1/1! + 1/2! + . . . + 1/n! também tende
para e = exp(1).
Fiz as contas de n = 1 até n = 12 e já aqui o computador diz que cheguei no
limite, ou seja o erro entre e = exp(1) e x′12 está na décima-primeira casa decimal:

x′1 = 2, x′2 = 2.500000000, x′3 = 2.666666667,


x′4 = 2.708333333, x′5 = 2.716666667, x′6 = 2.718055556,
x′7 = 2.718253968, x′8 = 2.71827877, x′9 = 2.718281526
x′10 = 2.718281801, x′11 = 2.718281826, x′12 = 2.718281828.
Veja por comparação como a sequência anterior xn = (1 + 1/n)n é lenta em
sua covergência para e, pois x112 = 2.707041491 ainda está bem longe de x′12 =
2.718281828.

6. Arcotangente e cartografia
Nos mapas as curvas de nı́vel dão a informação de quanto variou a coordenada
vertical ∆y entre dois pontos e a escala do mapa te dá informação da variação da
coordenada horizontal ∆x.
∆y
Logo se obtém um valor tan(α) = ∆x e torna-se relevante calcular arctan(α).
Logo é importante sabermos calcular o arcotangente com a precisão que quisermos.
Mas o que a calculadora cientı́fica de fato faz, quando calcula essa função ?
E se eu tiver apenas uma calculadora que faz as 4 operações, será que consigo
calcular arctan(α) com a precisão que quiser ?
6. ARCOTANGENTE E CARTOGRAFIA 422

Vou explicar o que fazer, para dar o arctan(x) pelo menos para x ∈ (−1, 1), com
a ordem de precisão que se quiser, ou seja, com quantas casas quisermos depois da
vı́rgula, apenas fazendo repetidamente as 4 operações +, −, /, x.
Primeiro começo lembrando da fórmula (Seção 5 do Capı́tulo 16 ):
1
arctan′ (x) = , ∀x ∈ R.
1 + x2
Escrevendo:
1 1
2
= ,
1+x 1 − (−x2 )
podemos usar a Afirmação 2.1 na região x ∈ (−1, 1):
1
= 1 − x2 + x4 − x6 + . . . se |x| < 1.
1 + x2
Sabemos pelo Primeiro Teorema Fundamental que:
Z x
1
2
dt = arctan(x) − arctan(0) = arctan(x).
0 1+t

Agora vamos ser otimistas 3: vamos imaginar que podemos usar a propriedade
Z x Z x Z x
(f + g) dt = f dt + g dt
a a a
não apenas para a soma de duas funções f + g mas para a soma de uma infinidade
de funções.
Ou seja, com otimismo, asssumo que a integral de uma soma infinita de funções
é a soma infinita de integrais. Esse otimismo nos permitiria escrever:
Z x
x3 x5 x7
(1 − t2 + t4 − t6 + . . .) dt = x − + − + . . . , se |x| < 1.
0 3 5 7
O fascinante é que sim, podemos fazer isso ! pelo menos nessa situação especı́fica...
Ou seja, igualando o lado esquerdo com o direito:
x3 x5 x7
arctan(x) = x − + − + ..., se |x| < 1.
3 5 7
E é isso que a calculadora faz: ela trunca a soma
x3 x5 x7
x−
+ − + . . . , se |x| < 1
3 5 7
num grau suficientemente alto para termos a precisão desejada do arctan(x). E fazer
somas e produtos como os que aparecem em
x3 x5 x7
x− + − + . . . , se |x| < 1
3 5 7
é fácil para uma calculadora !
As Figuras a seguir comparam o gráfico real de arctan : (−1, 1) → R com os
3
gráficos dos truncamentos y = x : (−1, 1) → R, y = x − x3 : (−1, 1) → R e
3 5
x − x3 + x5 : (−1, 1) → R.
3Justificado na Afirmação 2.1 do Capı́tulo 31
CAPÍTULO 30. APROXIMAÇÃO DE NÚMEROS E FUNÇÕES IMPORTANTES
423

0,5

0
-0,8 -0,4 0 0,4 0,8
x

-0,5

-1

Figura: O gráfico de y = arctan(x) (vermelho) e y = x (verde) para x ∈ [−0.99, 0.99].

0,8

0,4

0
-0,8 -0,4 0 0,4 0,8
x

-0,4

-0,8

x3
Figura: O gráfico de y = arctan(x) (vermelho) e y = x − 3
(verde) para x ∈ [−0.99, 0.99].

0,8

0,4

0
-0,8 -0,4 0 0,4 0,8
x

-0,4

-0,8

x3 x5
Figura: O gráfico de y = arctan(x) (vermelho) e y = x − 3
+ 5
(verde)
para x ∈ [−0.99, 0.99].

7. A aproximação de π dada por Leibniz


Uma prova de que π é Irracional é dada no excelente livro Calculus, de M. Spivak,
usando com astúcia o Cálculo.
O que quero dar aqui é uma aproximação de π por Racionais, que remonta a
Leibniz.
Mostraremos aqui que a série
x3 x5 x7
arctan(x) = x − + − + ...
3 5 7
funciona para x = 1 ! E como arctan(1) = π4 , teremos:
π 1 1 1
= arctan(1) = 1 − + − + . . . ,
4 3 5 7
7. A APROXIMAÇÃO DE π DADA POR LEIBNIZ 424

de onde:
1 1 1
π = 4(1 − + − + . . .).
3 5 7
.
Essa aproximação de π, apesar de bonita, é lenta e é feita por falta e excesso, de
modo oscilante: de fato as somas parciais de ordem ı́mpar da soma são maiores que
π e decrescem:
1 1
s1 := 4 · 1 = 4, s3 := 4(1 − + ) = 3.466666667,
3 5
1 1 1 1
s5 = 4(1 −+ − + ) = 3.339682540, . . .
3 5 7 9
enquantos as somas parciais de ordem par são menores que π e crescem:
1 1 1 1
s2 := 4(1 − ) = 2.666666667, s4 := 4(1 − + − ) = 2.895238095,
3 3 5 7
1 1 1 1 1
s6 := 4(1 − + − + − ) = 2.976046176, . . .
3 5 7 9 11
Queremos provar que uma fila sn vai toda para algum lugar determinando quando
n cresce. Se mostro que as posições pares s2n a fila vão para o lugar L e se mostro
que as posições ı́mpares s2n+1 também vão para esse lugar L, então a fila toda vai.
É isso que queremos verificar, pois queremos mostrar que para
1 1 1
sn := 4(1 − + + . . . + (−1)n )
3 5 2n − 1
existe
lim sn = L.
n→+∞

Reparando no formato das somas sn , vemos que para n ≥ 2:


• s2n+1 < s2(n−1)+1 pois
1 1
s2n+1 = s2(n−1)+1 − 4( − )
2(2n + 1) − 3 2(2n + 1) − 1
e portanto as somas parciais ı́mpares s2n+1 formam elas mesmas uma sequência
decrescente,
• s2n > s2(n−1) pois
1 1
s2n = s2(n−1) + 4( − )
2n − 3 2(2n) − 1
e portanto as somas parciais pares s2n+1 formam elas mesmas uma sequência
crescente.
• s2n ≤ s1 = 4 e s2 = 4(1 − 31 ) < s2n+1
Logo o Teorema 1.1 aplicado separadamente às sequências (s2n )n e (sn+1 )n , diz
que ambas convergem:
lim s2n = L1 e lim s2n+1 = L2 .
n→+∞ n→+∞
CAPÍTULO 30. APROXIMAÇÃO DE NÚMEROS E FUNÇÕES IMPORTANTES
425

Mas para terminar note que L1 = L2 pois


4
| s2n+1 − s2n | =
2(2n + 1) − 1
e
4
lim = 0.
n→+∞ 2(2n + 1) − 1

8. Aproximações de logaritmos
Se |x| < 1 então 1 + x > 0 e posso tomar ln(1 + x). Pela regra da composta:
1
ln(1 + x) ′ = .
1+x
Agora escrevo:
1 1
=
1+x 1 − (−x)
e uso a Afirmação 2.1 para x ∈ (−1, 1):
1
= 1 − x + x2 − x3 + . . . , se |x| < 1.
1 − (−x)
O Teorema Fundamental do Cálculo dá:
Z x
1
dt = ln(1 + x) − ln(1 + 0) = ln(1 + x)
0 1+t

Vamos ser novamente otimistas novamente e supor que a integral de uma soma infinita
é uma soma infinita de integrais4, obtendo então:
Z x
x2 x3 x4
ln(1 + x) = (1 − t + t2 − t3 + . . .) dt = x − + − . . . , |x| < 1.
0 2 3 4

As Figuras a seguir comparam o gráfico real de ln(1 + x) : (−1, 1) → R com


2
os gráficos dos truncamentos y = x : (−1, 1) → R, y = x − x2 : (−1, 1) → R e
2 3
x − x2 + x3 : (−1, 1) → R.
Para que os gráficos ficassem mais destacados não usei a mesma escala nos eixos
x e y:

1
x
-0,8 -0,4 0 0,4 0,8
0

-1

-2

-3

-4

4Justificado na Afirmação 2.1 do Capı́tulo 31


9. APROXIMAÇÃO DE LOGARITMOS DE NÚMEROS QUAISQUER 426

Figura: O gráfico de y = ln(1 + x) (vermelho) e y = x (verde)


para x ∈ [−0.99, 0.99].

x
-0,8 -0,4 0 0,4 0,8
0

-1

-2

-3

-4

x2
Figura: O gráfico de y = ln(1 + x) (vermelho) e y = x − 2
(verde)
para x ∈ [−0.99, 0.99].

x
-0,8 -0,4 0 0,4 0,8
0

-1

-2

-3

-4

x2 x3
Figura: O gráfico de y = ln(1 + x) (vermelho) e y = x − 2
+ 3
(verde)

9. Aproximação de logaritmos de números quaisquer


Agora vamos ver o que fazer para aproximar ln(z) de um número z > 0 qualquer.
Se |x| < 1 então 1 − x > 0 e posso tomar ln(1 − x). Pela regra da derivada da
composta:
1 −1
ln(1 − x) ′ = (−1) =
1−x 1−x
Se |x| < 1 escrevo pela Afirmação 2.1:
1
= 1 + x + x2 + x3 + . . . , se |x| < 1
1−x
e se pode também escrever (ver Afirmação 2.1 da Seção 31):
−1
= −1 − x − x2 − x3 − . . . , se |x| < 1.
1−x
Pelo Teorema Fundamental:
Z x
−1
ln(1 − x) − ln(1 − 0) = ln(1 − x) = dt,
0 1−t
CAPÍTULO 30. APROXIMAÇÃO DE NÚMEROS E FUNÇÕES IMPORTANTES
427

e se formos otimistas trocaremos a integral de uma soma infinita pela soma de infinitas
integrais (ver Afirmação 2.1 do Capı́tulo 31):
Z x
x2 x3
ln(1 − x) = (−1 − t − t2 − t3 − . . .) dt = −x − − ... |x| < 1.
0 2 3

Agora vamos precisar de um truque:

Afirmação 9.1. Todo número z > 0 se escreve de modo único como:

1+x
z= , com |x| < 1.
1−x
Demonstração.
Dado z > 0 quero resolver em x a equação:

1+x
= z.
1−x

Para isso faço z · (1 − x) = 1 + x, logo −zx − x = 1 − z, ou seja, −x(1 + z) = 1 − z e


daı́:
z−1
x= .
z+1
Note que x < 1 pois z − 1 < z < z + 1.
Também note −1 < x pois −(z + 1) = −z − 1 < z − 1, já que 0 < z.
Ou seja, |x| < 1. 

Usando dessa Afirmação e da propriedade do logaritmo do quociente, escrevo:

1+x
ln(z) = ln( ) = ln(1 + x) − ln(1 − x) z > 0, |x| < 1
1−x

e portanto, pelo que já vimos:

x2 x3 x4 x2 x3
ln(z) = (x − + − . . .) − (−x − − . . .), |x| < 1.
2 3 4 2 3

Se as somas acima fossem finitas, poderı́amos subtrair termo a termo. Sejamos


otimistas e imaginemos que podemos subtrair termo a termo nas somas infinitas (ver
Afirmação 1.1 do Capı́tulo 31), obtendo (já que os termos de grau par se cancelam):

x3 x5 z−1
ln(z) = 2(x + + + . . .), onde z > 0, x= , |x| < 1
3 5 z+1
11. EXERCÍCIOS 428

0
10 20 30 40 50
z

Figura: O gráfico de y = ln(z) (vermelho), z ∈ [0.5, 50], y = 2x (verde)


3 3 5
y = 2(x + x3 ) (amarelo) e y = 2(x + x3 + x5 ) (azul), onde x = z+1
z−1
.

10. Aproximação de ln(2)


Lembro que só usando a definição já sabı́amos que
1
< ln(2) < 1.
2
Com os resultados anteriores, para z = 2 e portanto x = z−1 z+1
= 13 , obtemos ln(2) com
a precisão que quisermos:
1 11 11 11
ln(2) = 2( + 3 + 5 + 7 . . .).
3 33 53 73
Meu computador aproxima ln(2) ≈ 0.6931471806.
Enquanto isso, obtenho:
1 1 1 1
s1 := 2( ) = 0.6666666667, s2 := 2( + ) = 0.6913580247
3 3 3 33
1 1 1 1 1
s3 := 2( + 3
+ ) = 0.6930041152
3 33 5 35
1 1 1 1 1 1 1
s4 := 2( + 3
+ 5
+ ) = 0.6931347573.
3 33 53 7 37
1 1 1 1 1 1 1 1 1
s5 := 2( + 3
+ 5
+ 7
+ ) = 0.6931460474
3 33 53 73 9 39
1 1 1 1 1 1 1 1 1 1 1
s6 := 2( + 3
+ 5
+ 7
+ 9
+ ) = 0.6931470738.
3 33 53 73 93 11 311
11. Exercı́cios
Exercı́cio 11.1. Obtenha uma sequência definida recursivamente que tende para a
raı́z cúbica de A. Para isso:
i) levante (x0 , 0) verticalmente no gráfico de y = x3 − A
ii) encontre a tangente ao gráfico de y = x3 − A no ponto obtido em i),
iii) desça pela tangente até encontrar o eixo x, determinando x1 e assim sucessi-
vamente.
iv) teste a sequência obtida, numericamente, numa calculadora.
CAPı́TULO 31

Séries numéricas e de funções

1. Séries numéricas
Um série infinita é uma soma infinita:
x1 + x2 + x3 + . . .
O sentido preciso dos três pontinhos é o seguinte: considere uma soma parcial de orde
n:
sn := x1 + x2 + . . . + xn .
Quando cresce o n os números sn forma eles mesmos uma sequência infinta (sn )n .
Então
x1 + x2 + x3 + . . . := lim sn ,
n→+∞
que pode existir ou não.

Quando existe esse limite dizemos que a soma infinita x1 + x2 + x3 + . . . converge


e quando não existe dizemos que x1 + x2 + x3 + . . . diverge.

O sı́mbolo x1 + x2 + x3 + . . . não é muito conciso, por isso uso:


n
X +∞
X
sn := xi , e x1 + x2 + x3 + . . . = xi .
i=1 i=1

A Afirmação a seguir justifica alguns dos truques usados nas Seções anteriores:
Afirmação
P 1.1. P+∞
i) Se +∞i=1 xi converge e C ∈ R então i=1 C · xi também converge e
+∞
X +∞
X
C · xi = C · xi .
i=1 i=1
P P
ii) Se +∞
i=1 xi e +∞ yi são duas séries convergentes então também convergem
i=1P
P
as séries +∞
i=1 (xi + y i ) e +∞
i=1 (xi − yi ) e ademais:
+∞
X +∞
X +∞
X
(xi + yi ) = xi + yi ,
i=1 i=1 i=1

+∞
X +∞
X +∞
X
(xi − yi ) = xi − yi .
i=1 i=1 i=1

429
1. SÉRIES NUMÉRICAS 430
P
iii) Sejam xi > 0 e yi > 0. Se xi ≤ yi ∀i ∈ N e se +∞ i=1 yi converge então também
P+∞
coverge i=1 xi converge
P P+∞
iv) Se +∞i=1 |xi | converge então i=1 xi . A recı́proca não é verdadeira.

Demonstração.
P +∞
De i): Como i=1 xi converge, então existe
n
X
lim sn = L, onde sn := xi .
n→+∞
i=1

Mas pelas propriedades de limites de sequências:


+∞
X
lim C · sn = C · lim sn := C · xi
n→+∞ n→+∞
i=1

Pela distributividade do produto e soma (finita)


n
X n
X
C · sn := C · xi = C · xi ,
i=1 i=1

e portanto
+∞
X
lim C · sn = C · xi ,
n→+∞
i=1
como querı́amos.
De ii): P P
Denoto por sxn := ni=1 xi e syn := ni=1 yi . Temos por hipótese que existem
lim sxn = L1 e lim syn = L2 .
n→+∞ n→+∞

Então pelas propriedades de soma/diferença de sequências, aplicadas às sequências


(sxn )n e (syn )n , temos:
lim (sxn ± syn ) = lim sxn ± lim syn ,
n→+∞ n→+∞ n→+∞

que é o que queremos provar.


De iii): Sem
P+∞entrar m muitos detalhes,a idéia é que se consegui somar as P infinitas
parcelas de i=1 yi com mais razão poderei somas as infinitas parcelas de +∞ i=1 xi ,
já que xi ≤ yi .
De iv): Sem entrar em detalhes que se vêem em textos de Análise Matemática,
o que posso dizer é que se conseguimos somar todos os módulos |xi | > 0 é razoável
que consigamos também somar as parcelas xi , já que nessas há mudanças de sinais
de > 0 para < 0, que produzem subtrações e cancelamentos.
Sobre a recı́proca : a série 1 − 21 + 31 − 14 + . . . converge (e o argumento é análogo
ao que usamos na aproximação de π). Mas como vimos na prova da Afirmação 6.1,
1 + 21 + 31 + 14 + . . . fica tão grande quanto quisermos.

CAPÍTULO 31. SÉRIES NUMÉRICAS E DE FUNÇÕES 431

2. Séries de potências
Agora precisamos justificar que, sob certas condições, a integral de uma soma
infinita é a soma infinita de integrais. Por exemplo, o otimismo:
Z x
x2 x3
(−1 − t − t2 − t3 − . . .) dt = −x − − . . . |x| < 1,
0 2 3
que podemos reescrever, se preferirmos, numa nova notação:
Z xX+∞ +∞ Z x
X
i
−t dt = −ti dt =
0 i=0 i=0 0

+∞
X −xi+1
= , |x| < 1.
i=0
i+1
Esta última expressão é uma série infinita, mas que depende de cada x com |x| < 1
para dar um valor determinado.
Por isso se chama série infinita de funções, e pode ser pensada como uma fábrica
de séries de números, pois:
+∞
X −xi+1
x 7−→ ∈ R,
i=0
i+1
desde que |x| < 1.
Esse é só um exemplo, em geral uma série infinita de funções é algo do tipo:
+∞
X
fi (x)
i=0
e o principal problema é saber para quais x as séries numéricas
+∞
X
x 7−→ fi (x)
i=0
convergem.
No que segue nos limitaremos apenas a funções
fi (x) = ai xi
onde ai são números (chamadas séries de potências).
P
Afirmação 2.1. Suponha uma série de funções +∞ i
i=1 ai t tal que para um certo t =
x > 0 convirja a série numérica:
X+∞
|ai ||xi |.
i=1
Então:
• convergem também as séries
+∞
X +∞
X
i
|ai t | e ai ti , ∀t ∈ [−x, x].
i=1 i=1
2. SÉRIES DE POTÊNCIAS 432

• A função
+∞
X
f : [−x, x] → R, f (t) := ai ti
i=1

é integrável e
Z x X +∞ +∞ Z x +∞
i
X
i
X ai i+1
ai t dt = ai t dt = x .
0 i=1 i=1 0 i=1
i+1

Demonstração.
Temos para |t| ≤ x:
+∞
X +∞
X +∞
X
i i
|ai t | = |ai ||t | ≤ |ai |xi |
i=1 i=1 i=1

e está última série converge por hipótese. P+∞ i


Então também convergem as séries numéricas i=1 |ai t |, obtidas escolhendo t
com |t| ≤ x (para cada t, aplique a Afirmação 1.1 itemPiii)).
Então para cada t escolhido com |t| ≤ x convergem +∞ i
i=1 ai t (para cada t, aplique
a Afirmação 1.1 item iv)).
Logo a função
+∞
X
f : [−x, x] → R, f (t) := ai ti
i=1

está bem definida.


A integrabilidade dessa f se explica nos textos de Análise Matemática.
Me concentrarei apenas em mostrar que
Z x +∞ Z x
X
f (t) dt = ai ti dt,
0 i=1 0

ou seja que
Z x n Z
X x
f (t) dt = lim ai ti dt,
0 n→+∞ 0
i=1

ou ainda (já que integral de soma finita é a soma finita de integrais) que
Z x Z x Xn
f (t) dt = lim ( ai ti ) dt.
0 n→+∞ 0 i=1

Para isso tenho que mostrar que:

dado ǫ > 0 qualquer, se n for suficientemente grande, então


Z x Z x Xn
| f (t) dt − ( ai ti ) dt | < ǫ.
0 0 i=1
CAPÍTULO 31. SÉRIES NUMÉRICAS E DE FUNÇÕES 433

Ora, do item ix) do Teorema 4.1, Capı́tulo 21:


Z x Z x Xn Z x n
X
i
f (t) dt − ( ai t ) dt = (f (t) − ai ti ) dt.
0 0 i=1 0 i=1

Pelo item viii) do Teorema 4.1, Capı́tulo 21:


Z x n
X Z x n
X
i
| (f (t) − ai t ) dt | ≤ | f (t) − ai ti | dt.
0 i=1 0 i=1
P+∞ i
Agora, por definição f (t) := i=1 ai t , logo
n
X +∞
X
i
f (t) − ai t = ai ti
i=1 i=n+1
e portanto
n
X +∞
X
i
| f (t) − ai t | = | ai ti | ≤
i=1 i=n+1
+∞
X +∞
X
≤ |ai ||ti | ≤ |ai ||xi |, se |t| ≤ x
n+1 n+1
P
O que vem a ser esse termo +∞ n+1 |ai ||x | ?
i
P+∞ i
Se denoto n+1 |ai ||x | = L, então
+∞
X n
X
i
|ai ||x | = L − |ai ||xi |.
i=n+1 i=1
P
Mas as somas parciais sn := ni=1 |ai ||xi | convergem para o limite L, logo
+∞
X
|ai ||xi | = L − sn
i=n+1

se faz tão pequeno quanto quisermos, se n cresce o suficiente. Posso tomar n tal que
+∞
X ǫ
|ai ||xi | < , onde x > 0.
i=n+1
x
Em conclusão: Z Z n
x x X
| f (t) dt − ( ai ti ) dt | ≤
0 0 i=1
Z +∞
x X
≤ |ai ||xi | dt ≤
0 i=n+1
Z x
ǫ ǫ
dt = · x = ǫ,

0 x x
se n cresce o suficiente. Era o que querı́amos demonstrar.

3. SÉRIES DE TAYLOR E OS RESTOS DE LAGRANGE, CAUCHY E
INTEGRAL 434

Para usar a Afirmação anterior é preciso ter uma idéia de qual x tomar. Esse
intervalo
[−x, x]
onde a série converge é chamado de intervalo de convergência.
Para determinar x, para cada t faça1:
|ai+1 | · |t|i+1 |ai+1 | |ai+1 |
L(t) := lim i
= lim · |t| = |t| · lim
i→+∞ |ai | · |t| i→+∞ |ai | i→+∞ |ai |

e imponha que:
L(t) < 1.
P+∞ −i i
Por exemplo, para i=1 (i + 2 ) · t temos:
|ai+1 | |i + 2−i + 1 + 2−1 |
L(t) := |t| · lim = |t| · lim =
i→+∞ |ai | i→+∞ |i + 2−i |
1 + 2−1
= |t| · lim 1 + = |t|.
i→+∞ i + 2−i
Portanto uma escolha
0<x<1
P+∞ −i i
garante que a série i=1 (i + 2 ) · t converge ∀t ∈ [−x, x].

3. Séries de Taylor e os Restos de Lagrange, Cauchy e Integral


Definição 3.1. Dada uma função f (x) que se possa derivar quantas vezes quisermos,
o seu polinômio de Taylor de grau n em a é dado por:
f ′′ f (n)
pn,f,a := f (a) + f ′ (a) · (x − a) + (a) · (x − a)2 + . . . + (a) · (x − a)n .
2! n!

A seguinte Afirmação mostra em que medida f (x) é aproximada por seu polinômio
de Taylor. Há três modos de expressar a diferença entre f e seu polinômio de Taylor,
cada um com sua utilidade.
Afirmação 3.1. (Restos da expansão de Taylor)
Suponha que f tem derivadas de todas as ordens.

i): Um polinômio q(x) de grau n tem


q(a) = f (a), q ′ (a) = f ′ (a), . . . , q (n) (a) = f (n) (a) ⇔ q(x) = pf,n,a.

Nos itens a seguir trato do caso a < x, mas as conclusões são análogas se x < a,
agora com x < x < a.
ii): (Resto de Lagrange) Existe pelo menos um ponto x ∈ (a, x) tal que
f (n+1) (x)
f (x) = pn,f,a + · (x − a)n+1 .
(n + 1)!
1Háversões mais gerais em que nem precisamos que exista esse limite, mas por enquanto ficamos
com esta.
CAPÍTULO 31. SÉRIES NUMÉRICAS E DE FUNÇÕES 435

iii): (Resto de Cauchy) Existe pelo menos um ponto x ∈ (a, x) tal que
f (n+1) (x)
f (x) = pn,f,a + · (x − x)n · (x − a).
n!
iv): (Resto Integral):
Z x (n+1)
f (t)
f (x) = pn,f,a + · (x − t)n dt.
a n!
Demonstração.

De i):
Note que da definição pf,n,a (a) = f (a), (pf,n,a )′ (a) = f ′ (a) e assim, sucessivamente,
que
(pf,n,a )(i) (a) = f (i) (a), i = 0, . . . , n.
Por outro lado se
q(x) = a0 + a1 x + . . . + an xn
então q(a) = f (a) implica que a0 = f (a); q ′ (a) = f ′ (a) implica que a1 = f ′ (a);
q ′′ (a) = f ′′ (a) implica que
2 · a2 = f ′′ (a),
f ′′ (a)
ou seja, a2 = 2
e assim sucessivamente até
f (n)
an = .
n!
De ii)
Fixados a e x, considere2 a seguinte função de t:
φ : [a, x] → R,
f ′′ f (n)
(t) · (x − t)2 + . . . +
φ(t) := f (x) − [ f (t) + f ′ (t) · (x − t) + (t) · (x − t)n ].
2! n!
Temos claramente φ(x) = 0, mas em geral
φ(a) 6= 0
já que
φ(a) := f (x) − pn,f,a .
Se acontece que φ(a) = 0 então o Teorema de Rolle diz que existe x ∈ (a, x) com
φ′ (x) = 0. Mas
f ′′′ f ′′
φ′ (t) = −f ′ (t) − f ′′ (t) · (x − t) + f ′ (t) − (t) · (x − t)2 + 2 (t) · (x − t) + . . . +
2! 2!
(n+1) (n)
f f
− (t) · (x − t)n + n · (t) · (x − t)n−1 .
n! n!
Note como os termos aparecem repetidos, mas com sinais opostos. Portanto após
cancelamentos:
f (n+1)
φ′ (t) = − (t) · (x − t)n .
n!
2Se fosse x < a a função φ(t) seria definida do mesmo jeito, no domı́nio [x, a]
3. SÉRIES DE TAYLOR E OS RESTOS DE LAGRANGE, CAUCHY E
INTEGRAL 436

Como φ′ (x) = 0 e x 6= x então concluimos que


f (n+1) (x) = 0
e a Afirmação ii) vale.
Mas no caso geral em que φ(a) 6= 0 faço:
(n + 1)!
C := · φ(a).
(x − a)n+1
Então a nova função
ψ : [a, x] → R,
C
ψ(t) := φ(t) − · (x − t)n+1
(n + 1)!
agora sim tem:
ψ(x) = ψ(a) = 0.
Pelo Teorema de Rolle existe algum x ∈ (a, x) onde:
ψ ′ (x) = 0.
Ora,
C f (n+1) C
ψ ′ (t) = φ′ (t) + (x − t)n = − (t) · (x − t)n + (x − t)n .
n! n! n!
Logo ψ ′ (x) = 0 e x 6= x dão que:
f (n+1) (x) = C.
Voltando na definição de ψ, agora com o valor de C = f (n+1) (x), obtemos
0 = ψ(a) =
f ′′ f (n) f (n+1) (x)
= f (x)−[f (a)+f ′ (a)·(x−a)+ (a)·(x−a)2 +. . .+ (a)·(x−a)n ]− ·(x−a)n+1 ,
2! n! (n + 1)!
o que conclui a demonstração deste item.

De iii):
Defina φ(t) como no item ii), para a qual sabemos que:
f (n+1)
(t) · (x − t)n .
φ′ (t) = −
n!
Agora aplique o Teorema do Valor Médio para ter algum x ∈ (a, x) tal que:
φ(x) − φ(a) ′ f (n+1)
= φ (x) = − (x) · (x − x)n .
x−a n!
Como φ(x) = 0 sempre obtemos
φ(a) f (n+1)
= (x) · (x − x)n
x−a n!
e portanto:
f (n+1)
φ(a) = (x) · (x − x)n · (x − a).
n!
Ora, φ(a) = f (x) − pn,f,a .
CAPÍTULO 31. SÉRIES NUMÉRICAS E DE FUNÇÕES 437

De iv):
Fazendo como no item i), temos
f (n+1)
φ′ (t) = − (t) · (x − t)n
n!
e o Teorema Fundamental do Cálculo dá:
Z x
f (n+1)
φ(x) − φ(a) = − (t) · (x − t)n dt.
a n!
Como φ(x) = 0, isso dá:
Z x
f (n+1)
φ(a) = f (x) − pn,f,a = (t) · (x − t)n dt.
a n!


Chama-se de Resto de Lagrange de ordem n + 1 a expressão:


f (n+1) (x)
Rn+1 (x) := · (x − a)n+1 ,
(n + 1)!
onde tomo qualquer x ∈ (a, x) que verifica o item ii) da Afirmação 3.1.
Se
lim Rn (x) = 0
n→+∞
então escrevo:
+∞ (i)
X f (a)
f (x) = · (x − a)i := lim pf,n,a .
i=0
i! n→+∞

Exemplos:
• Na Seção 6 vimos que
x3 x5 x7
arctan(x) = x − + − + . . . , se |x| < 1,
3 5 7
ou seja, de uma função que é igual à sua série de Taylor em a = 0, pois como
o leitor pode verificar:
(arctan(x))′ (0) = 1, (arctan(x))′′ (0) = 0, (arctan(x))′′′ (0) = −2,
(arctan(x))(4) (0) = 0, (arctan(x))(5) (0) = 24
etc. Ademais, naquela Seção plotamos alguns polinômios de Taylor dessa
função.

• Na Seção 8 vimos
x2 x3 x4
ln(1 + x) = x − + − ..., |x| < 1,
2 3 4
3. SÉRIES DE TAYLOR E OS RESTOS DE LAGRANGE, CAUCHY E
INTEGRAL 438

função que é igual sua série de Taylor em a = 0, pois como o leitor pode
verificar:
(ln(1 + x))′ (0) = 1, (ln(1 + x))′′ (0) = −1, (ln(1 + x))′′′ (0) = 2, (ln(1 + x))(4) (0) = −6,
etc. Também naquela Seção plotamos alguns polinômios de Taylor dessa
função.
• Como sin(0) = 0, sin′ (0) = cos(0) = 1, sin′′ (0) = − sin(0) = 0, sin′′′ (0) =
− cos(0) = −1 e em geral:

sin(2i) (0) = 0 e sin(2i+1) (0) = (−1)i , i = 0...


então
n
X (−1)i
sin(x) = · xi + Rn+1 (x).
i=0
i!
Mas
sin(n+1) (x) n+1 xn+1
|Rn+1 (x)| = | ·x |≤
(n + 1)! (n + 1)!
e portanto:
lim Rn+1 (x) = 0.
n→+∞

Logo
+∞
X (−1)i
sin(x) = · x2i+1 , ∀x ∈ R.
i=0
(2i + 1)!
• De modo completamente análogo se obtém
+∞
X (−1)i
cos(x) = · x2i , ∀x ∈ R.
i=0
2i!

• Como exp(i) (x) = ex e exp(i) (0) = e0 = 1 temos


n
x
X 1 i
e = x + Rn+1 (x);
i=0
i!

mas como y = ex é uma função crescente, temos


ex ex xn+1
|Rn+1 (x) = | · (x − a)n+1 | ≤
(n + 1)! (n + 1)!
e novamente limn→+∞ Rn+1 (x) = 0.
Portanto
+∞
x
X 1 i
e = x, ∀x ∈ R.
i=0
i!
CAPÍTULO 31. SÉRIES NUMÉRICAS E DE FUNÇÕES 439

4. A série binomial e sua série de Taylor


A questão que tratarei aqui é expressar
(1 + x)r := er·ln(1+x) , r∈R
através de sua série de Taylor.
Como veremos, no caso geral em que r 6∈ N trata-se de uma série infinita de
potências de x convergente para todo x com |x| < 1.
Mas, no caso particular em que r = n ∈ N, a série infinita vira um polinômio de
Taylor de grau n em x. E esse polinômio tem como coeficientes os coeficientes usuais
dados como sı́mbolo combinatório.
Importantes exemplos para nós serão:
1
(1 + x) 2 e (1 + x)−1 .
O polinômio de Taylor de f (x) = (1 + x)r se obtêm facilmente, pois:
′ f ′′ (0) r · (r − 1) f ′′′ (0) r · (r − 1)(r − 2)
f (0) = 1, f (0) = r, = , =
2! 2! 3! 3!
e por indução:
f (n) (0) r · (r − 1) . . . (r − (n − 1))
= , ∀n ∈ N.
n! n!
Se r = n0 ∈ N teremos:
f (n) (0) r · (r − 1) . . . (r − n0 ) . . . (r − (n − 1))
= = 0, ∀n ≥ n0 + 1.
n! n!
Nesse caso em que r = n0 ∈ N lembramos do sı́mbolo combinatório:
 
r r! r · (r − 1) . . . (r − (n − 1))
:= = , ∀n ≤ n0 = r.
n (r − n)! n! n!
Mas podemos adotar esse sı́mbolo:
 
r r · (r − 1) . . . (r − (n − 1))
:=
n n!
mesmo se r 6∈ N, pois faz sentido como um número Real ∀r ∈ R.
Se usamos o Teste da Razão (cf. Seção 3 do Capı́tulo 29) podemos ver que a série
infinita:
+∞  
X r
· xn
n=0
n
converge em módulo se |x| < 1, pois:
r

| n+1
· xn+1 |
lim  =
n→+∞ | nr · xn |
|r − n|
= lim · |x| = |x|.
n→+∞ n+1
4. A SÉRIE BINOMIAL E SUA SÉRIE DE TAYLOR 440

Mas não está nada claro que essa série coincida com (1+x)r . Claro que se (1+x)r
tem um desenvolvimento em série infinita, então é esse. Mas falta ver que há esse
desenvolvimento.
Afirmação 4.1. Se r 6∈ N e se −1 < x < 1, então vale o desenvolvimento em série
infinita:
+∞  
r
X r
(1 + x) = · xn ,
n=0
n
onde  
r r · (r − 1) . . . (r − (n − 1))
:= .
n n!
Demonstração.

Caso 0 < x < 1:

Nesse caso o item ii) da Afirmação 3.1 (Resto de Lagrange) dá:


k  
r
X r f (k+1) (x) k+1
(1 + x) = · xn + · x , para x ∈ (0, x) ⊂ (0, 1)
n=0
n (k + 1)!
onde
f (k+1) (x) k+1 r · (r − 1) . . . (r − k)
·x = · (1 + x)r−k−1 · xk+1 .
(k + 1)! (k + 1)!
Observo que, para cada x fixado com |x| < 1, a sequência
r · (r − 1) . . . (r − k) k+1
| ·x |
(k + 1)!
tende para zero: de fato, o teste teste da razão diz que a série
+∞
X r · (r − 1) . . . (r − k) k+1
| · x |,
k=0
(k + 1)!
converge; logo a sequência dos termos gerais dessa série tende a zero.
E se k + 1 > r (o que mais cedo ou mais tarde vai acontecer):
lim (1 + x)r−k−1 = 0
k→+∞
1
já que 1+x < 1. Portanto o Resto de Lagrange tende a zero, quando k → +∞, para
cada x com 0 < x < 1.

Caso −1 < x < 0:

Nesse caso, se usássemos a mesma idéia do caso anterior, não saberı́amos o que
fazer na última etapa, pois agora:
1
> 1,
1+x
já que x < x < 0.
CAPÍTULO 31. SÉRIES NUMÉRICAS E DE FUNÇÕES 441

Precisei de uma dica do M. Spivak, Calculus, p. 675, para terminar esta prova. A
dica é combinar o o Lema 4.1 a seguir com o Resto de Cauchy (item iii da Afirmação
3.1).
Do seguinte modo. Tomo o resto de Cauchy:
f (k+1) (x)
· (x − x)k · x.
k!
Escrevo:
   
f (k+1) (x) r r−k−1 r−1
= (k + 1) · · (1 + x) =r· · (1 + x)r−k−1 ,
k! k+1 k
onde as igualdades sobre os sı́mbolos são fáceis de conferir.
Portanto:
 
f (k+1) (x) k r−1
| · (x − x) · x| = |r · · (1 + x)r−k−1 · (x − x)k · x| =
k! k
 
r−1 x−x k
= |r · ·( ) · (1 + x)r−1 · x| ≤
k 1+x
 
r−1
≤ |r · | · |x|k · M · |x|,
k
onde na desigualdade usei o Lema 4.1 a seguir.
O caso já justificado (0 < x < 1) nos deu pelo menos que:
 
r−1
lim | · xk | = 0, se |x| < 1.
k→+∞ k
Portanto:  
r−1
lim |r · | · |x|k · M · |x| = 0
k→+∞ k
e o resto de Cauchy tende a zero.


Lema 4.1. Se −1 < x < x < 0 então:


(1 + x)r−1 ≤ M,
onde
M := max{1, (1 + x)r−1 }.
E também:
x−x (1 − xx )
| | = |x| · ≤ |x|.
1+x 1+x
Demonstração.
Note que, se r − 1 ≥ 0, a função
ψ : [x, 0] → R>0 , ψ(x) := (1 + x)r−1
é crescente (incluindo o caso constante, se r = 1), portanto seu máximo é ψ(0) = 1.
5. UM DEVANEIO SOBRE OS NÚMEROS COMPLEXOS 442

Se r − 1 < 0 a função
ψ : [x, 0] → R>0 , ψ(x) := (1 + x)r−1
é decrescente, portanto seu máximo é ψ(x) = (1 + x)r−1 .
Por isso M := max{1, (1 + x)r−1 }.
Agora noto que:
(1 − xx )
0≤ ,
1+x
pois 0 < 1 + x e x ≤ x.
Para provar a segunda afirmação basta mostrar que:
(1 − xx )
≤1
1+x
pois o resto sai imediatamente.
Mas essa desigualdade é o mesmo que
x
1 − ≤ 1 + x,
x
já que 0 < 1 + x. E de fato:
x
− ≤ x ⇔ x · (x + 1) ≤ 0,
x
o que é verdade.


5. Um devaneio sobre os números Complexos


Como não pretendo justificar minhas afirmações, apresento esta Seção como um
devaneio.
Mas de fato tudo é verdade, pois a teoria de séries funciona ainda melhor sobre
os números complexos.

Considero I = −1 (uso I maiúsculo para distinguir do ı́ndice i dos somatórios).
Vamos definir, continuando o que obtivemos na Seção anterior,
+∞
Ix
X 1
e := (Ix)i , ∀x ∈ R
i=0
i!
supondo que faça sentido a convergência da série da direita.
Então, usando que I 2 = −1, I 3 = −I, I 4 = 1, I 5 = I, I 6 = −1, etc, supondo que
possamos agrupar de modos diferentes as parcelas da série e que possamos fatorar
constantes, obtemos:
+∞ +∞
Ix
X (−1)i X
2i (−1)i
e = ·x +I · · x2i+1 ,
i=0
2i! i=0
(2i + 1)!
quer dizer:
eIx = cos(x) + I · sin(x).
CAPÍTULO 31. SÉRIES NUMÉRICAS E DE FUNÇÕES 443

Em particular a notável fórmula:


eIπ = −1,
onde estão unificadas a geometria (π), o Cálculo (e), a álgebra (−1), através da
variável complexa (I).
Essa fórmulas são chamadas fórmulas de Euler.
Ademais, já que sonhar é livre que tal definir para a + Ib ∈ C:
ea+Ib := ea · eIb = ea · (cos(b) + I · sin(b)).
Veremos na Seção 2 do Capı́tulo 40 a importância dessas definições.
6. Exercı́cios
Exercı́cio 6.1. Se z := a + Ib ∈ C e defino
ez := ea+Ib := ea · eIb ,
será que essa estensão da exponencial aos C ainda é uma função injetora ?
Exercı́cio 6.2. Usando a fórmula de Euler para eIx e para e−Ix , escreva sin(x) e
cos(x) em função de eIx e e−Ix .
Compare o resultado com o modo como são definidos o seno hiperbólico e o cosseno
hiperbólico, sinh(x) e cosh(x).
CAPı́TULO 32

O discriminante de polinômios de grau 3

Neste Capı́tulo nos perguntamos sobre raı́zes múltiplas de polinômios. Ou seja


pontos x ∈ R onde não somente o polinômio y = f (x) se anula mas onde há tangência
do gráfico com o eixo dos x. Ou seja, pontos onde também valha f ′ (x) = 0.
No caso de um polinômio de grau 2, f (x) = ax2 + bx + c, o sistema

f (x) = f ′ (x) = 0

significa:
ax2 + bx + c = 0 e 2ax + b = 0.
−b
Da segunda equação temos x = 2a
e substituindo na primeira obtemos:

ab2 b2 b2 − 4ac
0= − + c =
4a2 2a 4a2
ou seja, obtemos que onde há raı́z dupla x é onde há a anulação do discriminante:

b2 − 4ac = 0.

A conhecida fórmula de Báskara dá a localização da raı́z dupla: x = −b


2a
O objetivo deste Capı́tulo é explicar que há um discriminante de polinômios
de grau 3 e que sua anulação determina a existência de uma raı́z Real dupla dos
polinômiso de grau 3.

1. Preparação para a fórmula de Cardano


Consideremos um polinômio de grau exatamente 3, que após divisão pelo seu
coeficiente de grau 3 pode ser escrito como:

f (x) = x3 + a1 x2 + a2 x + a3 , ai ∈ R.

É muito útil a mudança de coordenada


a1
x= x− .
3
Em termos geométricos, x = x− a31 desloca o gráfico horizontalmente, como mostra
a figura a seguir:
445
1. PREPARAÇÃO PARA A FÓRMULA DE CARDANO 446

20

10

x
-3 -2 -1 0 1 2
0

-10

-20

Figura: Os gráficos de y = x3 + 3x2 e de y = (x − 1)3 + 3(x − 1)2 .


a1
Mas em termos algébricos a mudança x = x − 3
produz o polinômio a seguir,
livre de monômio de grau 2:
a21 a1 a2 2a3
f (x) = x3 + (a2 − )x− + a3 + 1 .
3 3 27
Essa notação está pesada, por isso volto a usar como variável x e ponho
a21 a1 a2 2a3
b = a2 − a=− + a3 + 1 .
3 3 27
Ou seja que podemos nos restringir a considerar:
f (x) = x3 + bx + a.
Afirmação 1.1. Seja um polinômio de grau 3 da forma
f (x) = x3 + bx + a
(sem termo quadrático).
Então

i) f (x) tem uma raı́z múltipla (dupla ou tripla) se e somente se


4b3 + 27a2 = 0.

ii) Se vale i) então a raı́z simples é


r
3 −a
x1 = 2
2
e a raı́z dupla é
r
−a
x2 = − 3
.
2
Se vale i), as raı́zes dupla e simples coincidem, formando uma raı́z tripla, exata-
mente quando a = b = 0.
CAPÍTULO 32. O DISCRIMINANTE DE POLINÔMIOS DE GRAU 3 447

Demonstração.
Primeiro provemos que 4b3 + 27a2 = 0 é condição necessária para a existência de
raı́z múltipla.
Analisar as raı́zes Reais múltiplas de f (x) = x3 + bx + a é analisar x onde
f (x) = f ′ (x) = 0,
o que significa resolver o sistema:
x3 + bx + a = 0 3x2 + b = 0.
A segunda
b = −3x2
e substituindo na primeira obtemos:
−2x3 + a = 0
ou seja
a = 2x3 .
Então
b3 = −27x6 e a2 = 4x6
ou seja, que temos a anulação do seguinte discriminante:
4b3 + 27a2 = 0.
Agora vamos ver que a condição
4b3 + 27a2 = 0
nos permite encontrar as raı́zes de f (x) = x3 + bx + a e ainda determinar qual é a
raı́z múltipla.
Começo com a fórmula do binômio:

(v + u)3 = v 3 + 3v 2 u + 3vu2 + u3 =
= v 3 + u3 + 3uv(u + v).
Portanto posso escrever a identidade:
(v + u)3 − 3uv(v + u) − (u3 + v 3 ) ≡ 0.
Pensemos por um momento em x = v + u e busquemos v, u satisfazendo:
−3uv = b, e − (u3 + v 3 ) = a.
Se conseguimos estas duas últimas condições então
(v + u)3 − 3uv(v + u) − (u3 + v 3 ) ≡ 0
diria que x = v + u seria raı́z de
x3 + bx + a = 0.
Ora, a primeira condição:
−3uv = b,
dá (supondo u 6= 0)
−b
v=
3u
1. PREPARAÇÃO PARA A FÓRMULA DE CARDANO 448

e, substituindo isso na segunda, u3 + v 3 = −a, obtemos:


−b3
u3 + = −a.
27u3
Se multiplicamos isso tudo por u3 , obtemos uma equação:
b3
u6 + au3 − = 0.
27
Note que esta equação é do tipo:
b3
(u3)2 + a(u3 ) − = 0,
27
ou seja , uma equação quadrática na nova variável u3 .
Portanto as raı́zes u3 podem ser descobertas pela fórmula de Báskara:
q
3
−a ± a2 − 4 −b 27
3
u = =
2
q
4a2 3
−a 4
+ 4b 27
= ± =
2 2
r
−a a2 b3
= ± + .
2 4 27
Logo s r
3 −a a2 b3
u= ± +
2 4 27
2 3
Estamos supondo 27a + 4b = 0, o que dá no mesmo que
a2 b3
+ = 0.
4 27
Logo obtenho r
3 −a
u=
2
e a condição v 3 + u3 = −a dá r
3 −a
v= .
2
Logo
x=v+u=
r
−a
=2· 3 .
2
q
Esse ponto x1 = 2 · 3 −a
2
é raı́z de f (x) = x3 + bx + a, mas é raı́z simples se a 6= 0.
Observe agora que se denoto por x1 , x2 , x3 as raı́zes Reais ou complexas de f (x) =
3
x + bx + a, podendo ser repetidas no caso múltiplo (xi = xj ) temos:
x1 + x2 + x3 = 0.
CAPÍTULO 32. O DISCRIMINANTE DE POLINÔMIOS DE GRAU 3 449

Isso é fácil de se ver, pois se escrevo:


x3 + bx + a = (x − x1 )(x − x2 )(x − x3 ) =
= x3 + (−x1 − x3 − x2 ) · x2 + (x1 x3 + x1 x2 + x2 x3 ) · x − x1 x2 x3 ,
temos que concluir que x1 + x2 + x3 = 0.
Ou seja, no caso de raı́z dupla x2 temos que x1 + x2 + x2 = 0, ou seja,
−x1
x2 = .
2
Verifiquemos então que o ponto
r
−x1 −a
x2 = =−3
2 2
é de fato raı́z dupla de f (x) = x3 + bx + a, calculando primeiro f (x) nesse ponto:
r r
3 −a 3 −a
(− ) + b(− 3 )+a=
2 2
r r
a 3 27 a4 3 −a
= − − +a=
2 4 2
r
a 3
3 27 a a 3a
= − +a= − + a = 0.
2 8 2 2
E a seguir calculando f ′ (x) nesse ponto:
r r
2
3 −a 2 3 a
3( − ) +b=3 +b=
2 4
r
3
3 −b
3 + b = −b + b = 0
27
4 b3
Claro que se a = 0 e a4 + 27 = 0 então b = 0 e f (x) = x3 tem raı́z tripla em x = 0.
q q
E também é claro que se a raı́z dupla − 3 −a 2
coincide com a raı́z simples 2 3 −a
2
então
a = 0.


2. A fórmula de Cardano para as três raı́zes Reais: viagem nos


Complexos
A Seção anterior foi dedicada ao caso em que x3 + bx + a tem discriminante:
a2 b3
+
∆ := = 0.
4 27
Mas nesta estaremos considerando o caso:
a2 b3
∆ := + 6= 0.
4 27
2. A FÓRMULA DE CARDANO PARA AS TRÊS RAÍZES REAIS: VIAGEM
NOS COMPLEXOS 450

Retomemos a prova da Afirmação 1.1 desde o começo, com a notação que lá
introduzimos, até o ponto em que obtivemos:
s r
3 −a a2 b3
u= ± + .
2 4 27
Escolho por exemplo1 : s r
3 −a a2 b3
u= + + .
2 4 27
Lá tı́nhamos a relação:
v 3 + u3 = −a,
portanto s r
3 −a a2 b3
v = −a − ( + + )=
2 4 27
s r
3 −a a2 b3
= − + .
2 4 27
E também naquela prova:
x=u+v =
s r s r
3 −a a2 b3 3 −a a2 b3
= + + + − +
2 4 27 2 4 27
3
é indicada como Raı́z de x + bx + a = 0.

Caso ∆ < 0:
Ora é fácil dar um exemplo de um polinômio x3 + bx + a com três óbvias raı́zes
Reais distintas para o qual:
∆ < 0.
Tome
x3 − 7x + 6
com raı́zes −3, 1, 2 para o qual
−100
∆= .
27
Então a expressão anterior para a Raı́z x é um pouco estranha, pois parece ser um
número Complexo não Real.
Este é o casus irreducibilis do tratado de Cardano, a Ars Magna.
Note que se ∆ < 0:
−a √ −a √
z := + ∆ e z := − ∆
2 2
são números complexos conjugados, não-Reais. Então chamemos x de x1 e notemos
que ele é a soma de um número complexo com seu conjugado:
√ √
3
x1 := 3 z + z =
1se pode checar que obterı́amos os mesmos resultados finais com a escolha −
CAPÍTULO 32. O DISCRIMINANTE DE POLINÔMIOS DE GRAU 3 451

3

3
= z+ z
e portanto x1 ∈ R.
Mas se pensamos na operação de extrair raı́z cúbica que produziu:
r
−a √
u= 3 + ∆
2
como operação sobre os complexos, então há de fato três raı́zes complexas diferentes.
Essa propriedade se origina do fato de que, sobre os complexos, há três raı́zes
distintas da unidade:
√ √
√3

3 −1 3 √ √3 −1 3 √
1 = 1, 1 = τ1 := + · −1 e 1 = τ1 := − · −1,
2 2 2 2
onde τ1 e τ1 são conjugados.
Então podemos tomar também

u = τ1 · 3 z
e devido à relação
−b
u·v =
∈R
3
somos obrigados a tomar: √
3
v = τ1 · z,
para termos outra raı́z Real x2 := u + v, já que2

x2 := u + v =
√ √3
= τ1 · 3 z + τ1 · z =
√ √
= τ1 3 z + τ1 3 z
que é um número Real.
A terceira opção é: √
u = τ1 · 3
z
e √
3
v = τ1 · z,
que produz:
√3

3
x3 := τ1 · z + τ1 · z.
No exemplo x3 − 7x + 6 as raı́zes obtidas são
x1 = 2, x2 = −3 e x3 = 1.

Caso ∆ > 0:
Nesse se pode mostrar que a única Raı́z Real é
r r
3 −a
√ −a √
x= + ∆+ 3 − ∆
2 2
2Lembre

3

que ∀z1 , z2 ∈ C, z1 + z2 = z1 + z2 e que z1 · z2 = z1 · z2 . A propriedade z = 3 z sai
de z 3 = z 3 .
3. O DISCRIMINANTE COMO CURVA 452

e que há mais duas Raı́zes complexas conjugadas, as raı́zes do polinômio quadrático:

αx2 + βx + γ

da fatoração
x3 + bx + c = (x − x) · αx2 + βx + γ.

3. O discriminante como curva

Vamos interpretar geometricamente a Afirmação 1.1.


Pensemos num plano cujas coordenadas são (a, b) e o lugar de anulação 4b3 +
27a2 = 0. Isso define uma curva Γ no plano (a, b).
O traço da curva Γ : 4b3 + 27a2 = 0 é dado na Figura a seguir:

-0,2 -0,1 0 0,1 0,2


0

-0,1

-0,2

-0,3

-0,4

-0,5

-0,6

-0,7

Note que a imagem de

γ : R → R2 = (a, b), γ(t) := (2t3 , −3t2 )

satifaz
4( −3t2 )3 + 27( 2t3 )2 ≡ 0.
Por isso γ(t) é chamada de parametrização de Γ : 4b3 + 27a2 = 0.
Ou seja:

todas as cúbicas do tipo y = ft (x) = x3 − 3t2 x + 2t3 têm raı́z múltipla.

Pela Afirmação 1.1 a localização da raı́z dupla é


r
3
3 −2t
x2 = − = t,
2
enquanto a raı́z simples é
r
3 −2t3
x1 = 2 = −2t.
2
Fiz quatro Exemplos na Figura a seguir:
CAPÍTULO 32. O DISCRIMINANTE DE POLINÔMIOS DE GRAU 3 453

40

20

0
-4 -2 0 2 4
x
-20

-40

Figura: Gráficos de de y = ft (x) = x3 − 3t2 x + 2t3 , com t = −2, −1, 1, 2

Quando t → 0 a raı́z dupla de y = ft (x) = x3 − 3t2 x + 2t3 colide com a terceira


raı́z simples, formando a raı́z tripla de y = f0 (x) = x3 . Veja a Figura a seguir:

60

40

20
x
-4 -2 0 2 4
0

-20

-40

-60

Figura: Gráficos de de y = ft (x) = x3 − 3t2 x + 2t3 , com t = −1, −1


2
, −1
4

A curva discriminante Γ separa o plano (a, b) em duas regiões, uma onde 4b3 +
27a2 < 0, e que está acima da curva na Figura. Na figura a seguir escolhi 4 pontos
(a, b) nessa região e plotei as cúbicas y = x3 + bx + a resultantes:
4. A CURVA DISCRIMINANTE ENTRE AS CÚBICAS SINGULARES 454

100

50

0
-4 -2 0 2 4
x
-50

-100

A outra região do plano, determinada pela Γ, é onde 4b3 + 27a2 > 0, e que fica
abaixo da curva na Figura. Na figura a seguir escolhi 4 pontos (a, b) nessa região e
plotei as cúbicas y = x3 + bx + a resultantes:

800

400

0
-10 -5 0 5 10
x

-400

-800

4. A curva discriminante entre as cúbicas singulares


Os pares ordenados de parâmetros (a, b) formam um plano, que será para nós
agora um plano (x, y).
É possı́vel escolher novas coordenadas (x, y) nesse plano, para que a curva dis-
criminante
4y 3 + 27x2 = 0
seja dada por:
y 2 − x3 = 0,
√ √
De fato, basta fazer uma mudança do tipo y := 27 · x e x := − 3 4 · y.
CAPÍTULO 32. O DISCRIMINANTE DE POLINÔMIOS DE GRAU 3 455

Definição 4.1. Um ponto P = (x, y) é uma singularidade de uma curva F (x, y) = 0


se nesse ponto
∂F (x, y) ∂F (x, y)
F (x, y) = = = 0.
∂x ∂y
Por exemplo. se
F (x, y) = y 2 − x3 − b x − a = 0,
para termos singularidades dessas cúbicas temos que ter:
y 2 − x3 − b x − a = 0, y=0 e − 3x2 − b = 0,
ou seja (já que o sinal não vai importar):
x3 + b x + a = 0 e 3x2 + b = 0.
Se denoto f (x) = x3 + b x + a, as singularidades terão coordenada x verficando:
f (x) = f ′ (x) = 0,
quer dizer, raı́z multipla de f (x) = 0.
Mas então estamos recaindo no que aprendemos na Afirmação 1.1:

A condição para termos singularidades nas cúbicas y 2 = x3 + b x + a é dada por


4b3 + 27 a2 = 0.

A Figura a seguir é o que o Maple consegue plotar da cúbica


y 2 − x3 + 3 x − 2 = 0,
que tem singularidade, pois 4 · (−3)3 + 27 · 22 = 0.
De fato o formato correto é o de um laço e a singularidade é o ponto (1, 0).

y 0
-2 -1 0 1 2 3
x
-2

-4

-6

Figura: A curva y 2 − x3 + 3 x − 2 = 0.

A Figura a seguir é como o Maple plota a curva


y 2 − x3 + 3 x + 2 = 0,
que tem singularidade pois 4 · (−3)3 + 27 · (−2)2 = 0.
4. A CURVA DISCRIMINANTE ENTRE AS CÚBICAS SINGULARES 456

y 0
2 2,4 2,8 3,2 3,6
x
-2

-4

-6

Figura: Atenção: esta curva y 2 − x3 + 3 x + 2 = 0


tem um ponto isolado em (−1, 0), que é a singularidade !

De fato, (−1, 0) está na curva, y 2 − x3 + 3 x + 2 = 0, pois esta é:


y 2 − (x + 1)2 · (x − 2) = 0.
Ademais ∂F
∂y
= 2y e ∂F
∂x
= −3x2 + 3 se anulam em (−1, 0).
Os dois últimos exemplos são casos da seguinte situação:

Afirmação 4.1. Suponha y 2 = f (x) = x3 + bx + a com


(a, b) 6= (0, 0) e 4 b3 + 27 a2 = 0.
q
2 −a
• i) Se a < 0 então y = f (x) tem um ponto singular isolado em (− 3 2
, 0)
q
e todos os outros pontos da curva tem coordenada x ≥ 2 3 −a
2
.
2
• ii) Se
q a > 0 então y = f (x) tem forma de laço com singularidade no ponto
( − 3 −a
2
, 0 ).

Demonstração.
Se f (x) = x3 + bx + a tem
(a, b) 6= (0, 0) e 4b3 + 27 a2 = 0,
então a Afirmação 1.1 diz que f (x) tem uma raı́z dupla e uma simples, bem como
que a raı́z simples é r
−a
x1 = 2 3
2
enquanto que a raı́z dupla é r
−a
x2 = − 3 .
2
Logo no caso i):
a > 0 ⇒ x1 < x2 ,
CAPÍTULO 32. O DISCRIMINANTE DE POLINÔMIOS DE GRAU 3 457

enquanto que, no caso ii):


a<0 ⇒ x2 < x1 .

Caso i): como a < 0,


∂F ∂F
= 2y e = 3x2 + b
∂y ∂x
q
se anulam em (− 3 −a
2
, 0), pois
r r
−a 2 −a 2 b
3( − 3 ) +b=0 ⇔ ( 3
) =− ⇔
2 2 3
a2 b3
⇔ =− ⇔ 27 · a2 = −4 · b3 .
q 2 27
Logo (− 3 −a 2
, 0) é singularidade, cuja coordenada x negativa.
Note que
f (x) = x3 + bx + a = (x − x2 )2 · (x − x1 ).
Como y 2 = f (x), é necessário que
r
−a
x ≥ x1 = 2 3
2
para termos números Reais
p p
y = (x − x2 )2 · (x − x1 ) ou y = − (x − x2 )2 · (x − x1 ).
q
Ou seja, fora o ponto (− 3 −a 2
, 0) todos os outros pontos dessa curva tem coordenada
q
x ≥ 2 3 −a2
.

Caso ii): No caso a > 0 a verificação de que (x2 , 0) é ponto singular de y 2 = f (x)
é idêntica. O ponto (x1 , 0) não é singular para a curva, que tem tangente vertical
neste ponto.
Agora, neste caso, como x1 < x2 e
f (x) = (x − x1 ) · (x − x2 )2 ,
basta que x ≥ x1 para que estejam definidas nos Reais as raı́zes:
p p
y = (x − x2 )2 · (x − x1 ) ou y = − (x − x2 )2 · (x − x1 ).
As duas opções distintas de raı́zes se colapsam para o valor y = 0 em x = x1 . São
distintas raı́zes no intervalo (x1 , x2 ), pois nesse intervalo
(x − x2 )2 · (x − x1 ) > 0.
E voltam a se colapsar para o valor y = 0 em x = x2 . Para x > x2 há novamente
duas opções distintas de raı́zes para y. Por isso se forma o laço em (x2 , 0).

5. PARAMETRIZAÇÃO DOS PONTOS RACIONAIS DE CÚBICAS
SINGULARES 458

A Figura a seguir é um diagrama, onde a curva cuspidal em vermelho é a curva


discriminante no plano (a, b). O complemento dessa curva no plano é feito de duas
regiões desconexas. Em cada região está esboçada em azul o tipo de cúbica y 2 =
x3 + bx + a que é a curva no plano (x, y) que surge se tomamos o ponto (a, b) nessa
região. No ponto (0, 0) = (a, b) que é a singularidade da curva discriminante produz-
se a cúbica cuspidal y 2 = x3 em azul. Se (a, b) pertence ao ramo superior da curva
discriminante ou ao ramo inferior surgem no plano (x, y) cúbicas com laço ou com
ponto singular isolado (indicadas em azul).

5. Parametrização dos pontos racionais de cúbicas singulares


As cúbicas que foram apresentadas na Seção 4 do Capı́tulo 15 são da forma:
y 2 = x3 + b x + a,
mas para elas 4b3 + 27 a2 6= 0. Nesse tipo de cúbica pode haver infinitos pontos
com coordenadas racionais. Mas por um Teorema famoso de Mordell, esses pontos
todos podem ser obtidos com os métodos geométricos da Afirmação 4.1, a partir de
um número finito de pontos com coordenadas Racionais. Por exemplo, na curva de
Billing,
y 2 − x3 + 82 x = 0
a partir de
49 231
P1 = (−1, 9), P2 = (−8, 12) e P3 = ( , ).
4 8
Já nas cúbicas singulares como
y 2 − x3 + 3 x − 2 = 0
é muito mais fácil de encontrar todos seus pontos com coordenadas Racionais.
Para isso, tome qualquer reta r passando por (1, 0) (o ponto onde a cúbica tem
um laço) da forma:
p p p
r(x) = · x − , ∈ Q.
q q q
Então a intersecção de r(x) com a cúbica se dá no ponto:
−2q 2 + p2 p · (−3q 2 + p2 )
( , )
q2 q3
cujas coordenadas são Racionais (além é claro do (1, 0)).
CAPÍTULO 32. O DISCRIMINANTE DE POLINÔMIOS DE GRAU 3 459

Por outro lado se ( pq11 , pq22, ) é um ponto de coordenadas Racionais dessa cúbica,
então pertence à reta:
p p
r(x) = · x − ,
q q
onde
p ( pq22 )
= p1 .
q ( q1 − 1)
Ou seja, todos os pontos com coordenadas racionais surgem por intersecção com as
retas por (1, 0) com coeficiente angular pq ∈ Q.
Já na cúbica:
y 2 − x3 + 3x + 2 = 0,
cuja singularidade (−1, 0) está separada do resto da cúbica, qualquer reta r passando
por (−1, 0) da forma:
p p p
r(x) = · x + , ∈Q
q q q
intersecta a cúbica no ponto:
2q 2 + p2 p · (3q 2 + p2 )
( , )
q2 q3
cujas coordenadas são Racionais (além é claro do (−1, 0)). E todos os pontos Racinais
da cúbica são assim obtidos, como vimos acima.
6. Cúbicas singulares aparecem como seções com o plano tangente
Imagine a cúbica de Billing
y 2 − x3 + 82 x = 0
como uma seção da superfı́cie
F (x, y, z) = z 2 + y 2 − x3 + 82 x = 0,
obtida ao cortá-la com o plano z = 0 do espaço (x, y, z).
O que dá a intersecção da superfı́cie com seu plano tangente no ponto (−1, 9, 0) ?
Afirmação 6.1. A intersecção da superfı́cie
z 2 + y 2 − x3 + 82 x = 0
com o plano tangente em (−1, 9, 0) é a curva no plano (x, z) dada por:
6241 2 6727 6889
z2 + ·x + ·x+ − x3 = 0.
324 162 324
A totalidade dos pontos dessa curva com coordenadas racionais é dada pelos pontos
6889q 2 + 324p2 p · (7213q 2 + 324p2
(x, z) = ( , ), p, q ∈ Z,
324q 2 324q 3
além do (−1, 0), que é uma singularidade isolada do resto da curva.
Também podem surgir por intersecção de superfı́cies cúbicas com seus planos
tangentes outros três tipo de curvas singulares:
• com laço, do tipo visto acima,
6. CÚBICAS SINGULARES APARECEM COMO SEÇÕES COM O PLANO
TANGENTE 460

• cuspidais como y 2 − x3 = 0 e
• união de três retas concorrentes, como y · x · (y − ax) = 0.

Demonstração. (da Afirmação 6.1)


Este tipo de Afirmação pede que algumas das contas sejam checadas por exemplo
com o Maple ou WXMaxima. Como envolvem só números Racionais esses programas
as executam perfeitamente.
Como definimos na Seção 3 do Capı́tulo 15, o plano tangente dessa superfı́ce no
ponto (−1, 9, 0) é dado por:
∂F ∂F ∂F
· (x + 1) + · (y − 9) + · (z − 0) = 0
∂x ∂y ∂z
que nesse caso dá:
79x − 83 + 18y = 0.
O fato de que não aparece a variável z quer dizer que esse plano é obtido da reta
tangente em (−1, 9) à curva
y 2 − x3 + 82 x = 0
apenas levantando-a verticalmente no eixo z.
A equação
6241 2 6727 6889
z2 + ·x + ·x+ − x3 = 0
324 162 324
surge de substituir
79 83
y =− ·x+
18 18
na equação dada
z 2 + y 2 − x3 + 82 x = 0.
Seu significado geométrico é o da intersecção da superfı́cie com o plano tangente
79x − 83 + 18y = 0.
Após a mudança de coordenada
1 6241
x= x+ ·
3 324
que vimos na Seção 1, obtemos no plano (x, z) uma nova equação da curva livre do
termo em x2 :
52027369 375273412597
z2 + x+ − x3 = 0
314928 459165024
e a Afirmação 4.1 diz então que esta curva tem uma singularidade isolada no ponto:
7213
(x, z) = (− , 0).
972
Voltando às coordenadas (x, z) vemos então que:
7213 1 6241
(− + · , 0) = (−1, 0)
972 3 324
é uma singularidade isolada.
CAPÍTULO 32. O DISCRIMINANTE DE POLINÔMIOS DE GRAU 3 461

Cada reta
p p p
r(x) = ·x+ , ∈Q
q q q
intersecta essa curva no ponto de coordenadas racionais:
6889q 2 + 324p2 p · (7213q 2 + 324p2
(x, z) = ( , )
324q 2 324q 3
além do (−1, 0).
Como vimos no final da Seção anterior, todo ponto Racional se obtém inter-
sectando a cúbica com uma reta por (−1, 0) cujo coeficientes angular e linear são
Racionais.


100

50

y 0
-10 -5 0 5 10 15 20
x

-50

-100

Figura: A curva de Billing e sua reta tangente

40

20

z 0

-20

-40
40
020 y
-20
-40
-10 0 10 20 30
x

Figura: A superfı́cie que produz a curva de Billing como seção z = 0.


6. CÚBICAS SINGULARES APARECEM COMO SEÇÕES COM O PLANO
TANGENTE 462

40

20

y 0

-20

-40

40
-10 0 10 20 3020
0
-20
-40
x z

Figura: A superfı́cie e seu plano tangente.


CAPı́TULO 33

Discriminante dos polinômios de grau 4

Uma equação quártica geral (após dividir pelo coeficiente de x4 ):


x4 + dx3 + cx2 + bx + a = 0
pode ser levada numa equação que não tem a potência 3, através da transformação:
d
x= x− ,
4
a qual produz na nova variável x:
3d2 −cd d3 bd cd2 3d4
x4 + (c − ) · x2 + ( + + b) · x − +a+ − = 0.
8 2 8 4 16 256
Por isso vamos pensar no que segue que já lidamos com uma equação do tipo:
x4 + cx2 + bx + a = 0.

1. A andorinha: o discriminante como superfı́cie


O problema do discriminante desta equação
F (x) := x4 + cx2 + bx + a = 0
aparece quando nos perguntamos por quais parâmetros a, b, c, d produzem uma equação
F (x) com alguma raı́z múltipla.
O discriminante ∆ = 0 é uma equação no espaço 3-dimensional dos parâmetros
(a, b, c) = R3 , já que a ∈ R, b ∈ R, c ∈ R. Por isso ∆ = 0 determina uma superfı́cie,
ou seja, algo que intuitivamente é bi-dimensional.
Ao invés de obter essa equação ∆ = 0, vou descrever a superfı́cie que ela produz
como uma superfı́cie parametrizada, ou seja, vou dar uma aplicação:
Γ : R2 → R3 = (a, b, c)
cuja imagem satisfaz ∆ = 0.
Para isso começo considerando F (x) := x4 + cx2 + bx + a = 0 com uma raı́z
múltipla x, ou seja:
F (x) = 0 e F ′ (x) = 0.
Temos então da primeira equação:
a = −x4 − cx2 − bx
e da segunda:
b = −4x3 − 2cx.
ou seja,
a = −x4 − cx2 + x · (4x3 + 2cx) = 3x4 + 2cx2 .
463
1. A ANDORINHA: O DISCRIMINANTE COMO SUPERFÍCIE 464

Podemos então definir uma aplicação φ : R2 → R3 :

φ(x, c) = ( 3x4 + cx2 , −4x3 − 2cx, c ) = (a, b, c)

contida no discriminante ∆ = 0.
Mas a imagem dessa aplicação é uma superfı́cie singular no sentido de que em
certos pontos dela não está bem determinado o plano tangente, pois há quinas, bicos,
etc. Pelo seu formato ela é conhecida como andorinha ou rabo da andorinha.
As Figuras a seguir dão duas imagens da andorinha:

3
2,5 0
-0,2
2
-0,4
1,5 -0,6
1 -0,8
-1
0,5
-1,2
0 -1,4
-4 -2 0 2 4
CAPÍTULO 33. DISCRIMINANTE DOS POLINÔMIOS DE GRAU 4 465

3
2,5

2
1,5

0,5
0
0 -0,2
-0,4
-4 -0,6
-2 -0,8
0 -1
-1,2
2 -1,4
4

2. Discriminante como envelope de famı́lias de retas ou planos


O que fizemos para equações quadráticas e cúbicas no Capı́tulo 32 e agora para
quárticas é parte de um processo geral de buscar num espaço de parâmetros
(a0 , a1 , . . . , an−1 )
uma equação ∆ = 0 que dá a condição que devem satisfazer os parâmetros para que
o polinômios correspondente
F (x) = xn + an−1 xn−1 + an−2 · xn−2 + . . . + a0 = 0
tenha raı́z múltipla.
Essa equação ∆ = 0 surge de considerar o sistema
∂F
F = = 0.
∂x
Que tal se agora consideramos
F (x) = xn + an−1 xn−1 + an−2 · xn−2 + . . . + a0 = 0
de um outro ponto de vista. Pensemos nele como determinando:
• uma famı́lia de retas no plano (a, b) = R2 , com parâmetro x, se F (x) =
x2 + ax + b = 0; ou
• uma famı́lia de retas no plano (a, b) = R2 , com parâmetro x, se F (x) =
x3 + bx + a = 0; ou
• uma famı́lia de planos espaço (a, b, c) = R3 , com parâmetro x, se F (x) =
x4 + cx2 + bx + a = 0;
2. DISCRIMINANTE COMO ENVELOPE DE FAMÍLIAS DE RETAS OU
PLANOS 466

• e assim por adiante ...


Já que ∆ = 0 surge de considerar o sistema
∂F
F = = 0.
∂x
vemos que, no sentido como foi definido na Seção 11 do Capı́tulo 35:

o discriminante ∆ = 0 é o envelope das famı́lias de retas ou planos com parâmetro


x dadas por F (x) = 0.
CAPı́TULO 34

3
Apêndice: O expoente 4 comanda a vida !

Neste capı́tulo dou uma aplicação à Biologia do logaritmo, da série geométrica e


da teoria de mı́nimos do Cálculo. Não sou nenhum especialista em bio-matemática,
minha intenção é apenas mostrar como conceitos matematicamente simples podem
ser úteis em outras ciências.
Ademais, aqui exponho apenas um argumento para demonstrá-la, que usa hipóteses
fortes e na etapa final um tipo de limite no número de nı́veis de ramificação do sistema
circulatório.
Mas a lei de Kleiber se aplica até a seres unicelulares. Portanto deve haver um
argumento bem mais geral para demonstrá-la !
Minhas referências foram:
• R. Dawkins, A grande história da Evolução, Companhia das Letras, 2009.
• J. West, J. Brown, B. Enquist, A general model for the origin of allometric
scaling laws in biology , Science, 1997.
• M. Kleiber, Body size and metabolic rate, Physiological Reviews, vol. 27, n.4
, 1947.
• R. Etienne, M. Apol, H. Olff, Demystifying West, Brown, Enquist model of
the allometry of metabolism , Functional Ecology, 2006.
Essencialmente o objetivo do Apêndice é apresentar algumas idéias do último
artigo.

1. Metabolismo versus massa corporal

Questão 1: Quem produz mais calor ao longo de dia, estando em repouso, um


homem ou um rato ?

Questão 2: Quem tem a maior taxa de produção de calor por unidade de peso,
um homem ou um rato ?

Os biólogos se interessam por essas questões, ou seja, entender a relação entre o


crescimento da massa corporal e o crescimento do metabolismo basal dos organismos
vivos.
O metabolismo basal B é essencialmente o consumo de oxigênio por unidade de
tempo (medido em kcal/dia).
Em 1883 Rubner propôs um modelo geométrico para explicar essa relação:
467
3. RETA DE AJUSTE - MÉTODO DE MÍNIMOS QUADRADOS 468

• É preciso haver uma superfı́cie de área A para as trocas de O2 entre o organ-


ismo e o ambiente. Ou seja
B = τ1 · A,
(τ1 constante que não depende da massa).
• Por outro lado, a massa corporal M verifica
M = τ2 · V.
• Mas A = τ3 · L enquanto V = τ4 · L3 , onde L é uma medida de comprimento.
2

Ou seja
B = τ5 · L2 e M = τ6 · L3 .
Pelo modelo de Rubner já se prevê que não pode aparecer de uma hora para outra
uma aranha - Godzilla. Ela se sufocaria antes de destruir qualquer coisa !

2. Escalas log/log para um experimento


A massa de um elefante é 1021 vezes a massa de uma ameba. Por isso, quando se
plota M versus B se usa log10 (M) versus log10 (B). Pois então se poder desfrutar da
propriedade:
log10 (ak ) = k · log10 (a).
Escolha agora o grupo de seres vivos que mais lhe agrada (caninos, felinos, pri-
matas, mamı́feros, aves, peixes, crustáceos, plantas, etc). De preferência com bastante
variabilidade de massa corporal.
Plote os pares ( log10 (M) , log10 (B) ) obtidos por observação no grupo de seres
vivos escolhidos.
Suponha que voce tem então sua lista
( log10 (M1 ), log10 (B1 ) ), . . . , ( log10 (Mk ), log10 (Bk ) )

Agora o problema é definir a Reta que mais se ajusta a esses pontos, pois é dela
que trata a Lei de Kleiber.

3. Reta de ajuste - método de mı́nimos quadrados


Se o leitor já conhece esse conceito, pode ir para a Seção seguinte.
Chamo de distância vertical de um ponto (x, y) a uma reta y = ax + b o número
p
|(ax + b) − y| = (ax + b − y)2.
Como há uma raı́z quadrada, torna-se complicado derivar. Por isso vamos elevar ao
quadrado a distância e tentar minimizar o quadrado da soma de distâncias verticais
até uma reta.
Problema 2: Determinar reta y = ax + b que minimiza a soma dos quadrados das
distâncias verticais até k pontos dados.

Vamos mostrar apenas como obter um candidato a reta que minimiza a soma dos
quadrados das distâncias. a verificação completa depende de noções de Cálculo em
duas variáveis.
3
CAPÍTULO 34. APÊNDICE: O EXPOENTE 4
COMANDA A VIDA ! 469

Imagine para as retas a notação:


y = ξx + β,
já que os coeficientes angulares ξ e lineares β são os que queremos determinar. O que
quero dizer é que devemos pensar na função:

z = f (ξ, β) = (ξx1 + β − y 1 )2 + (ξx2 + β) − y 2 )2 + . . . (ξxk + β − y k )2 .


como função de duas variáveis ξ, β.
O gráfico de z = f (ξ, β) forma uma superfı́cie no espaço com coordenadas (ξ, β, z).

Figura: O gráfico de z = f (ξ, β)

O ponto (ξ0 , β0 ) que buscamos será um ponto de mı́nimo do gráfico de z = f (ξ, β),
portanto esperamos que ao intersectar essa superfı́cie com os planos ξ = ξ0 e com
β = β0 produzam gráficos de funções z = f (ξ, β0 e z = f (ξ0 , β) que tenham pontos
de mı́nimo.
Ou seja, esperamos que as derivadas de z = f (ξ, β0) e de z = f (ξ0 , β) sejam zero
em (ξ0 , β0 ). Ou seja, devemos parar a variável ξ e derivar em β e vice-versa, e buscar
pelos zeros dessas derivadas.
∂g
Quando paramos ξ = ξ0 e derivamos em β usamos o sı́mbolo ∂β . Quando paramos
∂g
β = β0 e derivamos em ξ usamos o sı́mbolo ∂ξ . Então
∂g
= 2(ξx1 + β − y 1 )x1 + 2(ξx2 + β) − y 2 )x2 + . . . 2(ξxk + β − y k )xk =
∂ξ
k
X k
X k
X
= 2 · (ξ ( x2i ) +β( xi ) − xi y i )
i=1 i=1 i=1
e

∂g
= 2(ξx1 + β − y 1 ) + 2(ξx2 + β) − y 2 ) + . . . 2(ξxk + β − y k ) =
∂β
k
X k
X
= 2(ξ ( xi ) + k · β − y i ).
i=1 i=1
4. A LEI EXPERIMENTAL DE KLEIBER 470

Fazendo
∂g ∂g
= =0
∂ξ ∂β
estamos criando um sistema não-homogêneo de duas equações lineares, com duas
incógnitas ξ, β:
k
X k
X k
X
ξ( x2i ) + β( xi ) = xi y i ,
i=1 i=1 i=1
k
X k
X
ξ( xi ) + k · β = yi.
i=1 i=1
Podemos usar a Regra de Cramer para resolvê-lo, pois o determinante formado com
os coeficientes do sistema é:
k
X k
X
2
k·( xi ) − ( xi )2 > 0,
i=1 i=1

pelo item ii) da Afirmação 6.1 do Capı́tulo 11.


Obteremos por Cramer:
P P P
k · ki=1 xi y i − ( ki=1 xi )( ki=1 y i )
ξ0 = P P
k · ki=1 x2i − ( ki=1 xi )2
e P P P P
( ki=1 x2i )( ki=1 y i ) − ( ki=1 xi )( ki=1 xi y i )
β0 = P P
k · ki=1 x2i − ( ki=1 xi )2
4. A Lei experimental de Kleiber
Se verifica experimentalmente (com as ressalvas como k suficientemente grande,
etc) que:

(Lei de Kleiber - 1947) O coeficiente angular da reta de ajuste independe do


grupo de seres vivos escolhidos e vale 43 .

Observo que 34 < 1 implica que há uma lentificação do metabolismo, à medida
que a massa corporal aumenta.
Evidências:
• M. Kleiber se baseia numa tabela de k = 26 pontos, com Massa M dada em
kg e B dado em kcal/dia.
• A tabela analisa mamı́feros. Começa com dados do camundongo, com (M, B) =
(0.021, 3.6), passa por exemplo pelo gato (M, B) = (3, 162) e vai até dados
da vaca (M, B) = (435, 8166).
• Usando sua tabela, se obtém (conferi !) a0 = 0.7497881511 ∼ 34 .
No livro de Dawkins (2004) a lei de Kleiber é aplicada em três grupos:
• organismos unicelulares,
• organismos de sangue frio e
• de sangue quente.
3
CAPÍTULO 34. APÊNDICE: O EXPOENTE 4
COMANDA A VIDA ! 471

Aı́ se vê que os coeficientes lineares b0 das retas de ajuste mudam bastante.

Além disso, Dawkins usa a lei de Kleiber para estudar outra correlação: massa
corporal versus massa cerebral.

Das retas de ajuste log10 (B) = 34 log10 (M) + b, obtemos:


3 3
B = 10b · M 4 = τ · M 4
onde τ depende do tipo de organismo (sangue frio x sangue quente, por ex.)
Vou introduzir a notação
3
B ∝ M4
para dizer só nos interessa o expoente de M e expressar a Lei de Kleiber.
2
Para termos uma comparação, a seguir plotei y = x (vermelho), y = x 3 (verde) e
3
y = x 4 (amarelo), para x ∈ [1, 10]

10

2 4 6 8 10
x

5. Justificação racional da Lei de Kleiber


Até 1997 não havia nenhuma justificação teórica da lei experimental de Kleiber.
Então o fı́sico West e os biólogos Brown e Enquist trataram de provar a lei de Kleiber,
em artigo publicado na Revista Science.
A idéia deles foi de que a eficiência de um sistema metabólico está intimamente
relacionada à eficiência do sistema respiratório/circulatório.
A ”demonstração”’ deles se baseou em:
• hipóteses sobre a geometria do sistema circulatório.
• hipóteses da fı́sica de fluidos, sobre a eficiência do processo de distribuição
(ou seja, minimização das perdas, resistência, etc)
O artigo WEB teve um grande impacto. Em 2004, R. Dawkins diz:

(...) A Lei de Kleiber, seja para plantas, animais ou até mesmo no nı́vel do
transporte dentro de uma única célula, encontrou finalmente sua base racional. Ela
pode ser derivada da fı́sica e da geometria das redes de suprimento.(...)
No entanto, houve crı́ticas. Fora debates sobre as ”contas”que fizeram, criticou-se
6. O ARGUMENTO 472

• que há hipóteses fortes sobre a geometria dos sistema circulatório (algumas
retomaremos mais adiante)
• que o postulado de eficiência do sistema circulatório parece sugerir que a
Evolução já acabou, já estarı́amos otimamente adaptados ...
O artigo de Etienne, Apol e Olff, de 2006, esclarece quais as suposições de WBE,
destaca pontos obscuros de WBE e permite dar uma versão light de WBE.
Seguirei EAO, mas visando apenas explicar algumas das muitas idéias de WBE,
aquelas que dispensam a fı́sica dos fluidos.

6. O argumento
6.1. Hipótese 1. Hip. 1: Os sistemas circulatórios são árvores, onde:
• Cada ramo de ordem k pode ser considerado um cilindro, de comprimento
lk , cuja base é um disco de raio rk .

r _k

l _k

• Há 1 =: N1 ramo de ordem 1 (a aorta), que se subdivide em ν1 ≥ 2 ramos


de ordem 2,
• cada ramo de ordem k se subdivide em νk ≥ 2 ramos de ordem k + 1. Há Nk
ramos de ordem k.

• Observe que
Nk N2
Nk = · ...· = νk−1 · . . . · ν1
Nk−1 1
6.2. Capilares.
• o processo de ramificação da aorta em artérias e depois arterı́olas continua
até ramos finais, chamados de capilares.
3
CAPÍTULO 34. APÊNDICE: O EXPOENTE 4
COMANDA A VIDA ! 473

• cuja ordem na ramificação será designada por C e cujo número total será
NC .
• Saiba que as paredes dos capilares são unicelulares ! 0 diâmetro externo de
um capilar é de 5 a 10 µ m (micrômetros, 10−6 m).
• Nos capilares se dão os processos fı́sicos como difusão, osmose, etc. Através
dos quais oxigênio / nutrientes passam para os tecidos enquanto gás carbônico/
dejetos passam para o sangue.
• esses dados dos capilares são praticamente universais.
• Se sabe que no ser humano há ≈ 20 bilhões de capilares.
• As hemáceas humanas tem 8 µ m de diâmetro. Para trafegarem pelos capi-
lares elas formam fila indiana !
• Para se ver o grau de ramificação do sistema circulatório, a aorta de uma
baleia pode chegar a 23 cm de diâmetro.
Nk+1
6.3. Relação com os Capilares. Como νk := Nk
, defino analogamente:
lk+1 rk+1
λk := e ρk := .
lk rk
Note que vale
rk+1 rC
rk · ρk · ρk+1 . . . · ρC−1 = rk · · ...· = rC ,
rk rC−1
Ou seja:
rC
rk = QC−1
i=k ρi
e exatamente do mesmo jeito se obtém:
lC NC
lk = QC−1 e Nk = QC−1
i=k λi i=k νi
Imagine cada ramo cheio de sangue ou de seiva (já pensamos em sistemas não-
pulsáteis ...)
Considere πrk2 · lk o volume de cada ramo de ordem k.
A soma de todos os volumes de ramos de nı́vel k é portanto:
NC · r 2 · lC
Vs,k := Nk · (πrk2 · lk ) = π QC−1 C 2 .
i=k νi ρi λi

Logo o volume total no sistema


C
X
Vs := Vs,k
k=1

é:
C
X 1
Vs = πNC · rC2 · lC · ( QC−1 ).
k=1 i=k νi ρ2i λi
6. O ARGUMENTO 474

6.4. Definição de S1 e de S2 . Para facilitar, chamar


C
X 1
S1 := QC−1 .
k=1 i=k νi ρ2i λi
Com essa nova notação temos:
Vs = πNC · rC2 · lC · S1 .
Considere
• Ak o quociente das somas de áreas de seções transversas dos ramos
• Ek o quociente de somas de volumes de esferas cujos diâmetros são o compri-
mento dos ramos.
2
Nk+1 πrk+1
Ak := = νk · ρ2k ,
Nk πrk2
Nk+1 34 π( lk+1 )3
Ek := 2
= νk · λ3k .
Nk 43 π( l2k )3
Essa esferas de volume 34 π( l2k )3 serão supostos os volumes servidos pelos ramos,
ou seja partes do corpo que recebem nutrientes dos ramos cilı́ndricos de ordem k, de
comprimento lk .

l _k

E agora defino outra grandeza:


C
X 1
S2 := Q 1 ,
1/3 C−1
k=1 Nk i=k Ai · Ei3
PC 1
Afirmação: S1 := k=1
QC−1 2 pode ser escrito como:
i=k νi ρi λi
1
S1 = NC3 · S2
1
De fato, como νi · ρ2i = Ai e λi = ( Eνii ) 3 :
C
X 1
S1 = QC−1 1 =
k=1 i=k Ai · ( Eνii ) 3
QC−1 1
C
i=k νi
X 3

= QC−1 1 =
k=1 i=k A i · E i
3
3
CAPÍTULO 34. APÊNDICE: O EXPOENTE 4
COMANDA A VIDA ! 475
1
C
X ( NNCk ) 3
= QC−1 1 =
k=1 i=k Ai · Ei3
C
1 X 1
= NC ·3
1 QC−1 1
k=1 Nk 3
i=k Ai · Ei3
o que prova a Afirmação. Portanto:
4
Vs = π NC · rC2 · lC · S1 = π NC3 · rC2 · lC · S2 .
Ou seja:
3
Vs 4
NC = ( 2 )
πrC · lC · S2

6.5. Hipótese 2. A hipótese a seguir faz mais sentido para sistemas circulatórios
não-pulsáteis. Mas tomemo-a para simplificar a exposição.

Hip. 2 O metabolismo basal B é proporcional ao fluxo total pela aorta Q1 :

B = τ Q1 ,
onde a constante τ não depende da massa M.
Se pode mostrar que a incompressibilidade do fluido (sangue/seiva) implica:
Q1 = Nk Qk , ∀k = 1, . . . C,
onde Qk é fluxo em cada ramo de ordem k.
Logo:
B = τ NC QC
onde QC é o fluxo por cada capilar.

6.6. Hipótese 3. Obtemos da expresão anterior de NC :


3
Vs 4
B = τ QC ( 2 ) .
πrC · lC · S2
Lembre que Vs é o volume total (sangue/seiva).
Em mamı́feros, o volume de sangue ocupa 6 − 7
Há evidências experimentais para:

Hip. 3 Vs = ηM, onde η não depende da massa M.


Ou seja, do anterior obtenho:
3
M4
B ∝ QC 3 .
(rC2 · lC · S2 ) 4
6. O ARGUMENTO 476

6.7. Hipótese 4. Aqui retomamos o que já dissemos antes sobre o caráter uni-
versal dos capilares:

Hip. 4 As grandezas QC , rC , lC não dependem da massa M.


• Esta hipótese tem evidências experimentais, diz por exemplo que os dados
dos capilares de uma baleia e de um rato são essencialente os mesmos !
• Isso deve estar ligado ao fato de que, a partir dos capilares, o sistema de
distribuição só se baseia em processos fı́sicos universais, como a difusão.
• Ou visto de outro modo, que os sistemas circulatórios todos começaram mod-
estamente como redes capilares ...
• Porém o número de nı́veis C e NC claramente depende de M: maior o animal,
maior o número de etapas de ramificação e maior o número de capilares.

6.8. S2 invariante. Ou seja, do anterior obtenho agora:


3
M4
B∝ 3 .
(S2 ) 4
EAO dão argumentos no sentido de que a dependência entre S2 e M é negli-
genciável, o que concluiria a dedução da Lei de Kleiber.
Mas eu gostaria de seguir a exposição na linha do argumento original de WBE,
onde há algumas hipóteses (fortes) a mais, com consequências sobre S2 .

6.9. Hipótese 5. A resistência ao fluxo de sangue/seiva fica diminuida pela su-


posição (natural para o sistema circulatório de plantas):
Hip. 5 A soma das áreas das seções transversais é preservada a cada ramificação.
Ou seja :
Ak = 1, ∀k = 1, . . . , C.

6.10. Hipótese 6. A hipótese a seguir diz uma soma de volumes ao redor dos
vasos permanece constante em cada etapa da subdivisão:

Hip. 6 As quantidades Nk · 34 π( l2k )3 são preservadas nas ramificações.


Ou seja:
Ek ≡ 1, ∀k = 1, . . . C.
Esta última hipótse deu origem a muita controvérsia.

Como mostra EAO, as Hipóteses 5 e 6 são fortes, poderiam ser enfraquecidas pois
em
C
X 1
S2 = Q 1 ,
1/3 C−1
k=1 Nk i=k Ai · Ei3
os Ai e Ei podem se compensar, mesmo que mudem a cada etapa.
3
CAPÍTULO 34. APÊNDICE: O EXPOENTE 4
COMANDA A VIDA ! 477

6.11. Hipótese 7. Com as Hipóteses 5 e 6, S2 se reduz a:


C
X
S2 = Nk −1/3 .
k=1

A hipótese a seguir diz que ou sempre há dicotomias, ou sempre tricotomias , etc:

Hipótese 7: νk = ν , ∀k = 1, . . . , C (onde o Natural ν ≥ 2 não depende de M).

6.12. Número de ramificações. Portanto da Hipótese 7,


Nk = ν k−1 , k = 1 . . . C.
Por exemplo, em seres humanos, NC ≈ 2 × 1010 . De
NC = ν C−1
obtemos:
ν = 2 ⇒ C ≈ 35 e ν = 3 ⇒ C ≈ 22.
Ou seja, chegamos da aorta ao capilar em 35 dicotomias !
Ou chegamos da aorta ao capilar em 22 tricotomias !

Voltando ao S2 , note que ele se transforma numa soma geométrica (finita):


C
X
S2 = Nk −1/3 =
k=1

C
X −(k−1)
= ν 3 =
k=1
−C
1−ν 3
= −1 .
1−ν 3

6.13. S2 como função de C.


O número de nı́veis C depende de M.
Portanto precisamos ver que a dependência entre S2 e C é negligenciável.
O argumento de EAO é o seguinte: vamos plotar S2 como função de C, bem como
sua assı́ntota horizontal:
−C
1−ν 3 1
lim −1 = −1 ,
C→+∞ 1−ν 3 1−ν 3

−1
(que existe pois ν 3 < 1). E vejamos se a função S2 = S2 (C) se aproxima rapidamente
de sua assı́ntota. Se isso acontecer, a conclusão será que a partir de uma certo C, S2
pouco muda com C.
Para ν = 2 obtemos y = S2 (C):
6. O ARGUMENTO 478

1
5 10 15 20 25 30 35
x

Note que a escala no eixo y é menor que no eixo x.

Para ν = 3 obtemos y = S2 (C):

2,5

1,5

1
5 10 15 20
x

Note que a escala no eixo y é menor que no eixo x.

A velocidade com que os gráficos se aproximam do limite é o que EAO consideram


”dependência negligenciável”entre S2 e C.
E obtemos de 3
M4
B∝ 3
(S2 ) 4
o resultado: 3
B ∝ M 4.
Parte 2

Equações diferenciais ordinárias e


Aplicações
CAPı́TULO 35

As primeiras equações diferenciais

1. A exponencial e as equações diferenciais


A função y = f (x) = ex já nasceu com a propriedade de satisfazer a equação:
f ′ (x) = f (x), ∀x ∈ R.
Vamos ver agora algumas pequenas modificações da exponenciale e que tipo de
equações satisfazem:
Afirmação 1.1. Seja y = f (x) derivável e suponha que para k ∈ R tenhamos
f ′ (x) = k · f (x), ∀x ∈ R.
• Dado o valor f (0), então:
f (x) = f (0) · ekx , ∀x ∈ R.
• Mais em geral, dado f (x) para algum x, então:
f (x) = f (x) · ek (x−x) , ∀x ∈ R.
A Figura a seguir ilustra as soluções de f ′ (x) = −2 f (x) para quatro diferentes
valores iniciais f (0): 0.5, 1, 2, 3.

2,5

1,5

0,5

0
0 0,5 1 1,5 2 2,5 3
x

Demonstração.
Vamos provar diretamente o caso geral, onde nos damos o valor f (x).
Se k = 0 então a hipótese vira f ′ (x) ≡ 0. Já sabemos que nesse caso f (x) ≡ C e
portanto f (x) = f (x). Ou seja,
f (x) = f (x) · 1 = f (x) · e0 ,
como querı́amos.
481
2. A DEFINIÇÃO ORIGINAL DE NAPIER PARA O LOGARITMO 482

Logo podemos supôr que k 6= 0.


Considero a função g(x) := ek(x−x) .
Note que g(x) = ek(x−x) > 0 para todo x ∈ R.
Verifico pela regra da derivada da composta que:
g ′ (x) = k · ek(x−x) = k g(x), ∀x ∈ R.
Se tomo qualquer outra função f satisfazendo f ′ (x) = k · f (x), faço o quociente
f
g
e derivo pela regra da derivada do quociente:
f f ′g − f g′
( )′ (x) = =
g g2
(kf )g − f (kg)
= ≡ 0,
g2
o que nos faz concluir que fg ≡ C. Ou seja, f (x) = C · g(x).
Para descobrir C avalio tudo em x:
f (x) = C · g(x) =
= C · ek·0 = C.
Portanto f (x) = f (x) · ek(x−x) como querı́amos.


2. A definição original de Napier para o logaritmo


A obra do escocês John Napier (1550-1617) é o começo da longa história do con-
ceito de logaritmo.
Seguindo a exposição de C.H. Edwards (op.cit), podemos entender a definição
original de logaritmo de Napier do ponto de vista do Cálculo, e qual a relação com o
ln(x).
Esse anacronismo serve para entender o que fez Napier, mas lembre que, histori-
camente, Napier trabalhou só com sua definição e conseguiu fazer tabelas imensas de
logaritmos !
A definição de Napier envolve dois pontos se movendo:
• N um segmento [P0 , O] de comprimento P0 O = 107 , determinamos a posição
x(t) de um ponto P (t) que se move de P0 até O através da distância P (t) O:
x(t) = P (t) O.
• supomos que que a velocidade x′ (t) de P (t) satisfaz ∀t
x′ (t) = −x(t).
• ou seja, a velocidade inicial de P (t) é x′ (0) = 107 = x(0), mas a velocidade
vai caindo e quando P (t) está chegando no ponto O ele está parando, pois
x′ (t) = −x(t) ≈ 0.
CAPÍTULO 35. AS PRIMEIRAS EQUAÇÕES DIFERENCIAIS 483

• Com esse mesmo parâmetro de tempo t, num segundo segmento de origem


Q0 , se move um um ponto Q(t), se afastando de Q0 e a posição de Q(t) é
Q(t) = 107 t (ou seja, Q(t) tem velocidade constante 107 ).
• Napier define o tamanho Q0 Q(t) como sendo o logaritmo de x(t) := P (t) O.
• Chamemos o logaritmo definido assim por Napier de Nog(x).
Vamos traduzir isso na linguagem do Cálculo e obter:
Afirmação 2.1.
7
i) Nog(x) = 107 ln( 10x ).
ii) Nog(x1 x2 ) = Nog(x1 ) + Nog(x2 ) − 107 ln(107 ).
Demonstração.
De i):
A solução de x′ (t) = −x(t) é x = x(0)e−t pela Afirmação 1.1, ou seja,
x = 107 e−t .
Tomando logaritmo natural:
ln(x) = ln(107) + ln(e−t )
logo
ln(x) − ln(107 ) = −t
e
107
t = ln( )
x
logo
107
Nog(x) := 107 t = 107 · ln( ).
x
De ii)

107
Nog(x1 x2 ) = 107 · ln( )=
x1 x2
= 107 (ln(107 ) − ln(x1 x2 )) =
= 107 ln(107) − 107 ln(x1 ) − 107 ln(x2 ) =
1 1
= 107 ln(107 ) + 107 ln( ) + 107 ln( ) =
x1 x2
1 1
= 107 ln(107 ) −2 · 107 ln(107 ) + 2 · 107 ln(107 ) +107 ln( ) + 107 ln( ) =
| {z } x1 x2
0
1 1
= −107 ln(107 ) + 107 ln(107 ) + 107 ln( ) + 107 ln(107) + 107 ln( ) =
x1 x2
7 7
10 10
= −107 ln(107 ) + 107 ln( ) + 107 ln( )=
x1 x2
= −107 ln(107 ) + Nog(x1 ) + Nog(x2 ).

3. DECAIMENTO RADIOATIVO E DATAÇÃO 484

3. Decaimento radioativo e datação


Algumas substâncias quı́micas tem estrutura nucleares diferentes mas compostam-
se do ponto de vista quı́mico do mesmo jeito. São os chamados isótopos diferentes da
mesma substância.
Uma das mais importantes, por estar na base das moléculas orgânicas, é o Car-
bono. O isótopo chamado Carbono 14 é radioativo enquanto o isótopo mais comum,
o Carbono 12 não é radioativo.
A radioatividade surge com a desintegração do núcleo e portanto as substâncias
radioativas são instáveis, se degradam com o passar do tempo. Por isso se fala em
decaimento da substância, a quantidade tende a zero com o tempo.
Por exemplo, quando um organismo morre, deixa de assimilar Carbono à sua
estrutura (madeira, ossos, etc) e a proporção entre o Carbono 14 e o Carbono 12 (de
um para um trilhão quando vivo) começa a mudar, já que o Carbono radioativo se
decompõe.
Se considero a função y = f (x) para descrever a quantidade de uma substância
radioativa no tempo x, começando num tempo que fixo como x = 0, então
• f é uma função decrescente,
• f ′ (x) é sempre negativa
• f (x) tende a zero
Mais precisamente, a quantidade y = f (x) de cada substância quı́mica radioativa
satisfaz uma equação:
f ′ (x) = −kf (x), k > 0,
onde x ∈ R é o tempo e o valor de k > 0 depende especialmente de cada substância.

Já sabemos pela Afirmação 1.1 que

f (x) = f (0)e−k x , ∀R
e também pelo que sabemos sobre a exponencial:

lim e−kx = 0, k > 0.


x→+∞

3.1. Carbono 14.


Para o Carbono 14, k ≈ 3.8394 × 10−12 m/s (unidades de massa por segundo).
Ora, isso dá um decaimento em unidade de massa por ano próximo de:
−12
{z· 10 } ·60 ·60 ·24 ·365 ≈ 0.0001210793184.
3.8394
|
m/segundo
| {z }
m/minuto
| {z }
m/hora
| {z }
m/dia
| {z }
m/ano
CAPÍTULO 35. AS PRIMEIRAS EQUAÇÕES DIFERENCIAIS 485

Define-se meia-vida como o tempo τ no qual a quantidade inicial f (0) de uma


substancia radioativa se reduz à metade, ou seja:
f (0)
f (τ ) := .
2
Mas também temos:
f (0)
= f (0) · e−kτ ,
2
e daı́:
1
= e−kτ .
2
E tomando logaritmo:
1
ln( ) = −kτ.
2
Como − ln( 12 ) = ln(2), obtemos:
ln(2)
τ= .
k
No caso do Carbono 14 temos:
ln(2)
τ= ≈ 5724.736394
0.0001210793184
(e textos de fı́sica certamente o leitor encontrará aproximações mais corretas dessa
meia-vida)

3.2. Potássio 40.


Uma meia-vida relativamente curta (na escala geológica !) como a do Carbono 14
serve para datar madeira ou a historia da humanidade (na arqueologia).
Mas para datar rochas é preciso substâncias com meia-vida muito maiores. Por
exemplo, a lava das erupções se esfria, cristalizando-se, formando rochas cujo surgi-
mento pode ser datado. Isso porque ocorre o decaimento do potássio 40 (radioativo)
em argônio 40 (estável), que é uma gás mas que fica retido na lava transformada em
cristal. A meia vida do potássio 40 é 1, 3 bilhão de anos e portanto rochas muito
antigas podem ser datadas1
Por coincidência, vendo um documentário sobre a Evolução aprendi o seguinte:
foram encontrados restos de um hominı́dio que fora um dos primeiros a andar em duas
patas, e que se conjecturava ter em torno de 4 milhões de anos, quase um milhão a
mais que a famosa Lucy. Mas sua idade certamente não seria datável via Carbono
14. Vieram então geólogos e determinaram que os restos de ossos estavam localizados
entre duas camadas distintas de sedimentos de erupçoes vulcânicas.
Pelo método potássio/argônio as duas camadas de sedimentos vulcânicos forma
datadas em torno de 4 milhões de anos. Logo esses ossos tinham essa idade !

1Aprendi isso no livro de Richard Dawkins, A grande história da evolução- Na trilha de nossos
ancestrais, Companhia das Letras, 2009.
4. EQUAÇÕES DIFERENCIAIS LINEARES COM COEFICIENTES
CONSTANTES 486

3.3. A meia-vida da luz das super-novas.


O Professor Vı́tor Pereira, da Geologia da UFRGS, me explicou alguns fenômenos
muito interessantes, que resumo a seguir.
As super-novas são explosões de estrelas, catástrofes que acontecem com algumas
estrelas, e que de tão grandes produzem luz que é percebida na Terra a olho nu ou
por por lentes de telescópios amadores.
Mas a quantidade de luz que chega a partir dessas explosões se reduz rapidamente:
para um tipo de super-nova se constata que existe uma meia-vida da intensidade de
sua luz, que se determinou em 56 dias.
Não deve ser apenas coincidência que essa seja a meia-vida do Califórnio Cf 254 .
Essa substância é produzida em grande quantidade nessas explosões. e isso se sabe
por análise do espectro da luz das super-novas.
As super-novas são os verdadeiros fornos cósmicos dos elementos quı́micos: quanto
maior a intensidade das explosões mais pesados são os elementos quı́micos produzidos.
Porém esses elementos pesados em geral têm núcleos atômicos instáveis, se desin-
tegram e terminam sendo menos abundantes no Universo.

4. Equações diferenciais lineares com coeficientes constantes


A Afirmação a seguir resolve uma equação diferencial um pouco mais geral do que
a que já resolvemos na Seção anterior:
Afirmação 4.1. Uma equação do tipo:
g ′ (x) = A · g(x) + B, ∀x, A, B ∈ R
tem como solução:
i) g(x) = B · x + g(0), se A = 0,
ii) g(x) = g(0) · eAx , se B = 0,
B B
iii) g(x) = (g(0) + ) · eAx − , se A · B 6= 0.
A A
Ademais, em iii) temos
B
lim g(x) = − , se A < 0
x→+∞ A
ou
B
lim g(x) = − , se A > 0.
x→−∞ A

Note que a solução no caso mais geral, que é o iii), é uma soma (superposição) da
solução
g1 (x) = c1 · eAx , c1 ∈ R
da equação
g1′ (x) = A · g1 (x)
com a solução particular g2 (x) ≡ − B A
do problema que tratamos
g ′ (x) = A · g(x) + B.
CAPÍTULO 35. AS PRIMEIRAS EQUAÇÕES DIFERENCIAIS 487

Demonstração. (Afirmação 4.1)


Os casos i) e ii) em que A = 0 ou B = 0 já nos são conhecidos. Por isso
suponhamos AB 6= 0, ou seja, o situação de iii).
Há uma solução constante do problema: f (x) ≡ −B
A
, já que:
−B
0≡A·( ) + B.
A
Então vamos considerá-la uma solução desinteressante e procurar por outras interes-
santes, ou seja, não constantes. Por isso vou supor
−B
g(x) 6≡
A
e, o que é uma suposição a princı́pio mais forte2, que de fato:
−B
g(x) 6= , ∀x.
A
Então escrevo:
B
g ′ (x) = A · (g(x) + ),
A
e agora, com a suposição extra de que ∀x: g(x) + B A
6= 0 obtenho:
g ′(x)
= A.
g(x) + B A
Agora tomo primitivas. O lado esquerdo reconheço ter como primitivas:
B
ln |g(x) + | + C1
A
onde C1 é qualquer constante e o lado direito tem como primitivas:
Ax + C2
onde C1 é qualquer constante. Ou seja, agrupando as constantes como C3 := C2 − C1 ,
obtenho tomando primitivas:
B
ln |g(x) + | = Ax + C3 .
A
Tomando exponencial:
B
e ln |g(x)+ A | = eAx+C3 ,
de onde
B
|g(x) + | = eAx · eC3 .
A
B
Como g(x) + A é uma função contı́nua, ela não pode mudar de sinal sem se anular
A
(Teorema Valor Intermediário) e como supusemos que g(x) + B nunca se anula, temos
que ∀x:
• ou bem g(x) + B A
= eAx · eC3 > 0
• ou bem g(x) + B A
= −eAx · eC3 < 0.
2Na verdade, através da Afirmação 3 do Capı́tulo 36 se mostra que são a mesma hipótese
4. EQUAÇÕES DIFERENCIAIS LINEARES COM COEFICIENTES
CONSTANTES 488

Por isso agora adoto uma nova constante C, que pode ser positiva se C = eC3 ou
neqativa se C = −eC3 e escrevo:
B
g(x) = CeAx − .
A
Para determinar C avalio tudo em x = 0:
B
g(0) = C − ,
A
e portanto:
B
C = g(0) + ,
A
o que dá
B B
g(x) = (g(0) + ) · eAx − .
A A
B
Agora volto à hipótese de que g(x) + A
6 0. Observe que se pomos C = 0 em
=
B
g(x) = CeAx −
A
temos
−B
g(x) ≡.
A
As observações sobre os limites de g(x) são imediatas das prpriedades da expo-
nencial.


Na figura a seguir plotei a solução especial g(x) = − B A


junto de soluções g(x) =
B Ax B
(g(0) + A ) · e − A para 4 esolhas de g(0). Note que, por ser A = −1, à medida
que x cresce os gráficos se aproximam da solução constante. Se tivéssemos escolhido
A > 0 os gráficos se afastariam da solução constante, à medida que x crescesce.

7,4

7,2

6,8

6,6

0 1 2 3 4
x

Fig.: Gráfico de y = 7 (vermelho) e gráficos de y = Ce−x + 7,


com C = − 14 , − 12 , 21 , 41 .
CAPÍTULO 35. AS PRIMEIRAS EQUAÇÕES DIFERENCIAIS 489

5. Objetos em queda-livre vertical


Vamos aplicar alguns conceitos que aprendemos para entender o que acontece
quando um corpo3 de massa m cai (desde um altura razoavelmente baixa).
Sejam y = f (x) a posição do corpo no instante x, que supomos aumenta4 à medida
que o corpo se aproxima da superfı́cie da Terra e f ′ (x) sua velocidade.
Segundo Newton a aceleração f ′′ (x) de um corpo é dada por
F
f ′′ (x) = ,
m
onde F é a força resultante sobre o corpo que cai e m sua massa (em geral F é uma
grandeza vetorial, mas nesta situação particular podemos pensá-la como escalar).
Agora vamos postular que a Força resultante F tem duas origens: uma depen-
dendo apenas da atração gravitacional e outra dependendo da resistência que surge
quando o objeto que se desloca atinge uma velocidade alta.
• Ao nı́vel do mar, para quedas de não muito alto, a aceleração g impressa
pela gravidade é da ordem de 9.8 m/s s
. Galileu já tinha estimativas dessa
aceleração e foi o primeiro a notar que essa aceleração não depende da massa
do corpo (desprezando-se o atrito).
• Já o atrito e a resistência do ar contam no segundo tipo de força, do tipo5
−γ · f ′ (x),
onde γ > 0 depende da forma do objeto, do peso, do material, etc e onde
o sinal negativo tem a ver com o fato que aqui nos opomos ao efeito da
gravidade.
Então obtemos a aceleração:
−γ ′
f ′′ (x) = f (x) + g
m
Queremos descobrir quem é f ′ (x) e depois f (x).
Como tratamos de uma queda-livre, ou seja, o objeto não deve ser empurrado,
vamos supor
f ′ (0) = 0
e também f (0) = 0 para começarmos a medir a distância percorrida a partir do
instante x = 0.
Vamos usar a Afirmação 4.1 da Seção 4, com:
−γ
g(x) = f ′ (x), A = , B=g
m
e
f ′ (0) = 0.
3Aqui entendido como um ponto. Na Seção 5 do Capı́tulo 23 explicamos um pouco do que fazer
no caso de um objeto não-pontual
4Também poderı́amos medir a posição desde o solo, e então adaptarı́amos a grandeza g que
aparecerá a seguir por −g, para indicar que a gravidade traz para o solo
5Esta é uma hipótese, pois em outros modelos se supõe da forma −γ · (f ′ (x))2 o que conduz a
uma equação diferencial não-linear.
5. OBJETOS EM QUEDA-LIVRE VERTICAL 490

Temos então

f ′ (x) = gx, se γ = 0,

ou
−gm −γ x gm
f ′ (x) = em + , se γ 6= 0.
γ γ

Agora vamos impor que f (0) = 0 pois queremos medir a distância percorrida no
tempo x > 0.
Se γ = 0 obtemos

g · x2
f (x) = .
2

Ma se γ 6= 0:
Z
−gm −γ t gm
f (x) = [ em + ] dt =
γ γ

−m −gm −γ x gm
= ( )e m + x+C
γ γ γ

e a imposição f (0) = 0 dá:

−m gm
C= ( )
γ γ

e portanto:

gm2 −γ
x gm
f (x) = − · (1 − e m ) + · x.
γ2 γ

Seria muito interessante para um pára-quedista ter sua posição f (x) dada por uma
2
função linear. Note que a função f (x) acima se aproxima da reta y = gm γ
· x − gm
γ2
,
−γ
pois e m x → 0.
Os valores de γ se determinam experimentalmente. Por exemplo, para m = 10 kg
pode-se6 atribuir o valor γ = 2 kg
s
. A Figura a seguir compara a queda sem resistência
(γ = 0) com a queda com resistência ( γ = 2 kg s
).

6Boyce e DiPrima, Equações diferencias elementares e problemas de valores de contorno, LTC.


CAPÍTULO 35. AS PRIMEIRAS EQUAÇÕES DIFERENCIAIS 491

1000

800

600

400

200

0
0 2 4 6 8 10 12 14
x
-200

g·x2 2 −γ
Fig.: Gráficos de y = 2
(vermelho) e y = − gm
γ2
· (1 − e m x ) + gm
γ
· x (azul) e
2
y= − gm
γ2
+ gm
γ
· x (verde), g = 9.8, m = 10, γ = 2.

A seguinte afirmação trata da conservação de energia7 na queda-livre:


Afirmação 5.1. Considere um objeto pontual de massa m que cai em queda-livre,
verticalmente, sem efeito de atrito. Se f (x) dá a distância vertical percorrida desde
que o objeto é largado em queda livre, então a grandeza chamada Energia Total:
(f ′ (x))2
m· − mg · f (x)
2
é constante ∀x.
Demonstração.
x2
De fato, como vimos acima quando γ = 0, então f ′ (x) = g · x e f (x) = g · 2
.


No que segue vamos supor a seguinte versão da:

(Lei de Newton) se dd xs é a velocidade de um ponto de massa m ao longo de um


gráfico, então a aceleração é:
d2 s F
2
= ,
dx m
onde F é a força resultante que atua sobre o corpo.

7Se medı́ssemos a posição desde o solo, a energia total seria uma soma, não uma subtração
5. OBJETOS EM QUEDA-LIVRE VERTICAL 492

Afirmação 5.2. Considere dois pontos A, B num plano posicionado verticalmente.


Suponha que B = (0, 0) é a origem de um sistema de coordenadas cartesiano e que
A = (a1 , a2 ), a1 6= 0, e a2 > 0.
Suponha que o gráfico Γ de y = f (x) (derivável) com f (a) = A a f (b) = B descreve
a trajetória de um corpo de massa m que cai ao longo de Γ, apenas sob o efeito
da gravidade, sem atrito, partindo de A no tempo x = a com velocidade inicial 0 e
chegando em B no tempo x = b.
Então é constante, ∀x ∈ [a, b], a grandeza
( dd xs )2
m· + g · m · f (x),
2
onde g = 9.8 m/s2 .
Demonstração.
Derivando
( dd xs )2

2
obtemos:
d s d ( dd xs ) d s d2 s
m· · =m· · .
dx d x d x d x2
Como vimos na Seção 5, podemos determinar a posição de um ponto P do gráfico
em função de quanto vale o comprimento do gráfico desde f (a) = A até f (x) = P .
Ou seja, há uma função P = P (s).
A força resultante F (P (s)) em cada ponto P (s) do gráfico Γ depende do efeito da
gravidade na direção da tangente do gráfico, ou seja, é da ordem de
F (P (s)) = −gm · sin(θ(s)),
onde θ(s) é o ângulo formado pela tangente de Γ em P (s) com a horizontal e o sinal
− se deve a que a força é no sentido oposto ao crescimento de y (se θ = π2 temos toda
a força gravitacional gm agindo verticalmente).
Lembrando a Observação 6.1, temos então:
F (P (s)) dy
= −g · sin(θ(s)) = −g ·
m ds
e com a Lei de Newton obtemos:
d2 s dy
2
= −g · .
dx ds
Logo a derivada de
ds
m( )2
dx
é:
ds dy dy ds
m· · (−g · ) = −mg · =
dx ds dsdx
dy
= −mg · ,
dx
se usamos na última igualdade a regra da derivada da composta.
CAPÍTULO 35. AS PRIMEIRAS EQUAÇÕES DIFERENCIAIS 493

Portanto, como y = f (x), a derivada de


ds 2
m( ) + gm · f (x)
dx
é zero, o que diz que essa grandeza é constante.


6. Queda ao longo de um gráfico


Agora vamos considerar uma situação de interesse prático. Imagine um objeto
pontual que cai, deslizando sem atrito, ao longo de um gráfico ou de uma curva,
apenas sob o efeito da gravidade.
Em geral um gráfico y = f (x) ou uma curva parametrizada
Γ : R → R2 , (x(u), y(u))
tem um variável natural que descreve seus pontos(x ou u), mas que não tem nada a
ver em geral com o tempo t que descreve a queda do objeto.
Então a primeira questão que queremos tratar é saber como re-parametrixar a
curva ou gráfico pelo tempo t de modo a descrever a queda do objeto ao longo do
gráfico ou da curva.
Para isso, usaremos a Afirmação 6.1 a seguir. Essa é uma estensão da Afirmação
5.2 e sua prova desta é essencialmente8 a mesma da Afirmação 5.2. A diferença está
apenas no uso de noções vetoriais, por isso a omitimos:
Afirmação 6.1. Considere dois pontos A, B num plano posicionado verticalmente.
Suponha que A = (0, 0) é a origem de um sistema de coordenadas cartesiano e que
B = (b1 , b2 ), b1 6= 0, e b2 < 0.
Suponha que a curva parametrizada
Γ : (x(t), y(t)), t ∈ [a, b]
com A = (x(a), y(a)) a B = (x(b), y(b)), que descreve a trajetória de um corpo de
massa m no instante t caindo ao longo de Γ, apenas sob o efeito da gravidade, sem
atrito, partindo de A no tempo t = a com velocidade inicial 0 e chegando em B no
tempo t = b.
Então é constante, ∀t ∈ [a, b], a grandeza
( dd st )2
m· + gm · y(t),
2
ds
p
onde g = 9.8 m/s2 e dt
= (x′ (t)2 + (y ′(t))2 .

Como usaremos essa Afirmação para reparametrizar o gráfico ou curva pelo tempo
t de queda ?
8De novo a gravidade atua no sentido oposto ao crescimento da coordenada y(u) ≤ 0, por isso
o sinal + na grandeza Energia total
6. QUEDA AO LONGO DE UM GRÁFICO 494

Do seguinte modo. Começo com uma parametrização qualquer:


Γ̂ : (x(u), y(u)), u ∈ [c, d]
do traço da curva Γ.
Denote t ∈ [a, b] o parâmetro de tempo de queda que queremos introduzir para
descrver os pontos da curva. A Afirmação 6.1, combinada com dd st (a) = 0 e y(a) = 0,
diz que
ds
( )2 = −2 · g · y(t), ∀t ∈ [a, b]
dt
ou seja,
ds p
= −2 · g · y(t)
dt
e portanto
dt 1
=p .
ds −2 · g · y(t)
Portanto
dt dt ds
= · .
du ds du
p
x′ (u)2 + y ′(u)2
=p
−2 · g · y(t(u))
e Z p ′ 2
x (u) + y ′(u)2
t= p du.
−2 · g · y(t(u))
Em particular o tempo necessário para sair de Γ̂(c) e chegar em Γ̂(d) é:
Z d p ′ 2
x (u) + y ′(u)2
t= p du.
c −2 · g · y(t(u))

6.0.1. Exemplo:
Vamos fazer um exemplo bem simples. Na Seção seguinte haverá uns mais inter-
essantes. Vamos aqui descrever a queda de (0, 0) até B = (b1 , b2 ) b1 6= 0 e b2 < 0 ao
longo de um segmento de reta. Para isso vamos parametrizar a reta que liga esses
pontos pelo tempo de queda.
O faremos de dois modos: um bem elementar, e o outro, como ensinamos acima,
que expressa o tempo t como uma integral.
A função de t que dá a posição a partir de A = (0, 0) é parecida com aquela da
2
queda-livre vertical: g · t2 (já que f ′ (0) = 0 e f (0) = 0 e a aceleração é constante
ao longo da semireta AB). Mas a diferença com aquele caso já estudado é que a
gravidade atua na semireta AB de acordo com a projeção de um vetor vertical de
módulo g nesta semireta; ou seja, com valor
g · sin(θ)
onde θ é o ângulo entre a semireta AB e uma reta horizontal. Ou seja, o efeito da
gravidade vira zero se θ = 0 e volta a ser máxima se θ = π2 .
CAPÍTULO 35. AS PRIMEIRAS EQUAÇÕES DIFERENCIAIS 495

Por isso se tomamos um sistema cartesiano em que


A = (0, 0), B = (b1 , b2 ), com b1 6= 0, b2 < 0,
então o deslizamento do objeto ao longo da semireta AB
t2
g · sin(θ) · .
2
será descrito pela curva parametrizada:
b1 t2 b2 t2
(x(t), y(t)) = ( p 2 · g sin(θ) · , p · g sin(θ) · ),
b1 + b22 2 b21 + b22 2
onde ( √ b21 , √ b21 ) é um vetor de módulo 1 que gera a semireta AB.
b1 +b22 b1 +b22
Já que
−b2
sin(θ) = p 2
b1 + b22
ficamos com:
−b1 · b2 t2 −b22 t2
(x(t), y(t)) = ( · g · , · g · ).
(b21 + b22 ) 2 (b21 + b22 ) 2
O tempo que leva para chegar em B se obtém igualando:
−b1 · b2 t2 −b22 t2
· g · = b1 ou · g · = b2 ,
(b21 + b22 ) 2 (b21 + b22 ) 2
o que dá: s
2 · (b21 + b22 )
t= .
−g · b2
Agora retomo esse mesmo exemplo, para expressar o tempo d equeda via uma integral.
Uma parametrização natural da reta é:
b1 b
Γ̂ : (x(u), y(u)) = ( p 2 · u, p 2 · u)
b1 + b22 b21 + b22
com q
u ∈ [ 0, b21 + b22 ].
Então p p
x′ (u)2 + y ′(u)2 4
b21 + b22
p =√ √
2 · g · y(t(u)) −2g · b2 · u
e Z p
4
b21 + b22
t= √ √ du =
−2g · b2 · u
√ p
2 4 b2 + b22 √
= √ 1 · u + C.
−g · b2
Mas t = 0 corresponde a u = 0 e daı́ C = 0. Ou seja:
−g · b2 t2
u= p 2 ·
b1 + b22 2
7. A CURVA QUE MINIMIZA O TEMPO 496

e portanto esta re-parametrização coincide com a obtida pelo método elementar.

7. A curva que minimiza o tempo


Considero o caso particular em que um objeto pontual de massa m = 1 cai pela
reta ligando
A = (0, 0) a B = (π, −2)

(e no qual uso para aceleração g o valor π 2 ≈ 9.869604404) Obtemos, segundo o


Exemplo da Seção 6, uma parametrização do segmento de reta pelo tempo de queda
t segundo a qual o tempo de queda é

π2 + 4
t= ≈ 1.185447061.
π

O objetivo desta Seção é dar explicitamente outras curvas β ligando A = (0, 0)


até B = (π, −2), parametrizadas pelo tempo de queda t, mas que cheguem em B num
tempo t < 1.18. p
É claro que o comprimento de β, de A até B, é maior que a distância b21 + b22
do segmento de reta, porém afirmo que deslizando por essas curvas β o objeto chega
antes a B do que se deslizasse pela reta AB !
Considere a curva

u5 u2 √ √
α: x(u) := √ , y(u) := − √
5
, u ∈ [0, 2· 5
π].
25 π2

Então

p √
x′ (u)2 + y ′ (u)2 25u6π 4/5 + 128
p = ,
2 · g · y(t(u)) 8π 6/5

onde usei π 2 ≈ g e daı́ se pode avaliar numericamente no Maple o tempo da queda


ao longo desta curva como:
Z √ √
2· 5 π

25u6π 4/5 + 128
t= du ≈ 1.008984423.
0 8π 6/5

O traço de α é a curva no plano dada por


2
2x 5
y=− 2 , x ∈ [0, π],
π5
dada na Figura a seguir.
CAPÍTULO 35. AS PRIMEIRAS EQUAÇÕES DIFERENCIAIS 497

x
0 0,5 1 1,5 2 2,5 3
0

-0,5

-1

-1,5

-2

Observe que α começa com inclinação vertical, o que aproveita bastante bem o
efeito da gravidade. Ademais note que só conseguimos fazer com que a integral não
tenha valor +∞ porque quando y(0) = 0 também dd us = 0.
A curva que considero a seguir é a ciclóide:
β(t) := ( πt − sin(πt) , cos(πt) − 1 ), t ∈ [0, 1]
que claramente sai de β(0) = A e chega em t0 = 1 em
β(1) = (π, −2) = B.
A figura a seguir compara o traço de α com o da ciclóide β:

0 0,5 1 1,5 2 2,5 3


0

-0,5

-1

-1,5

-2

Em vermelho α e em verde a ciclóide β.

O que precisamos verificar é se a β(t) pode descrever a posição do objeto que


desliza. Para isso uso a Afirmação 6.1.
Temos para esta curva:
ds 2
( ) = (x′ (t)2 + (y ′(t))2 = 2π 2 · (1 − cos(πt)).
dt
7. A CURVA QUE MINIMIZA O TEMPO 498

Usando para g o valor π 2 ≈ 9.869604404, após derivar e simplificar obtemos:


( dd st )2
d( 2
+ π 2 · y(t) )
≡ 0,
dt
onde y(t) = cos(π · t) − 1.
A sequência de Figuras a seguir mostra a corrida entre a reta (em verde) e a
ciclóide (em vermelho), para ir de (0, 0) até (π, −2). Cuide que as escalas dos eixos
x, y vão mudando de figura para figura.
Os tempos transcorridos são
t = 0.05, 0.1, 0.3, 0.5, 0.7, 1.0, 1.18,
e em t = 1 a ciclóide já chegou no ponto (π, −2).

0 0,001
0,002
0,003
0,004
0,005
0

-0,002

-0,004

-0,006

-0,008

-0,01

-0,012

0 0,0050,010,0150,02
0

-0,01

-0,02

-0,03

-0,04

0 0,05 0,1 0,15 0,2


0

-0,1

-0,2

-0,3

-0,4
CAPÍTULO 35. AS PRIMEIRAS EQUAÇÕES DIFERENCIAIS 499

0 0,1 0,2 0,3 0,4 0,5


0

-0,2

-0,4

-0,6

-0,8

-1

0 0,5 1 1,5 2 2,5


0

-0,5

-1

-1,5

0 0,5 1 1,5 2 2,5


0

-0,5

-1

-1,5

0 0,5 1 1,5 2 2,5 3


0

-0,5

-1

-1,5

-2
8. BALÍSTICA E O SUPER MÁRIO 500

0 1 2 3 4
0

-0,5

-1

-1,5

-2

Johann Bernoulli colocou, em 1696, o seguinte problema:

Problema da braquistócrona9:
Sejam dados dois pontos A, B num plano vertical. Se A e B não estão numa reta
vertical, encontrar qual a curva descrita por um corpo M que sai de A e chega em B
no menor tempo possı́vel, sob efeito apenas da gravidade.

É possı́vel provar, com recursos mais avançados dos que dispomos no momento,
que a curva que minimiza o tempo é uma ciclóide.

8. Balı́stica e o Super Mário


Vários cientistas do Renascimento foram defrontados com problemas fı́sico-matemáticos
ligados à balı́stica, por exemplo Galileu, Torricelli e outros. Naquela época os mecenas
eram os Reis e os Reis sempre foram belicosos...
Por isso vou explicar o problema mais básico de balı́stica, mas o leitor pacifista
pode adaptá-lo ao jogo Super Mário, mais de acordo com o espı́rito de nossa época.
Nesse jogo o personagem salta para nı́veis mais altos. O que pode ser interpretado
como o ponto mais alto da trajetória na Afirmação 8.1 a seguir.
O problema mais básico para açguém que atira com um canhão é: dado um
alvo encontrar o ângulo θ que se deve levantar um canhão para atingir o alvo.
Mais precisamente, imagine o alvo no eixo x > 0 e com coordenada (x, 0) enquanto
o canhão está na origem (0, 0). Em geral a velocidade escalar da bala do canhão não
pode ser alterada, o que se pode é alterar o ângulo 0 < θ < π2 que o canhão forma
com o eixo x > 0.
Também se supõe que a bala sofre apenas o efeito da gravidade (e que estamos a
nı́vel do mar), sem sofrer resistências extra ao seu deslocamento.
Se meditamos um momento vemos que, se x for grande demais em relação a v0
pode acontecer da bala nunca alcançar o alvo. Aı́ é preciso aproximar o canhão do
alvo.
A Figura a seguir mostra 4 tentativas frustradas de se atingir o alvo, onde v0 = 5
e x ≥ 3.
9braquistocrona vem do grego e significa menor tempo
CAPÍTULO 35. AS PRIMEIRAS EQUAÇÕES DIFERENCIAIS 501

1
0,8
0,6
0,4
0,2
0
0 0,5 1 1,5 2 2,5

Figura: A tentativa em verde é a de θ = π4 .


Afirmação 8.1. Seja v0 > 0 a velocidade escalar com que a bala sai do canhão e o
alvo em (x, 0), com x > 0.
• o ângulo θ a ser escolhido para o tiro atingir o alvo (x, 0) verifica
g·x
sin(2 · θ) = 2 ,
v0
onde g = 9.8 (m/s2 ).
• em geral, dado um 0 < θ < π2 , a trajetória da bala é descrita pela parábola
g
y=− 2
· x2 + tan(θ) · x.
2 · v0 · cos2 (θ)
Em particular, a partir da parábola vemos que:
• o ponto mais alto atingido pela bala tem coordenadas:
v02 · sin(θ) cos(θ) v02 · sin2 (θ)
( , ).
g 2g
• o ponto onde a bala atinge o chão tem coordenada
sin(2θ) · v02
x= .
g
Em particular o ponto mais longe que pode ser atingido tem coordenada
v02
x=
g
e corresponde à escolha θ = π4 .
• o ponto mais alto da trajetória se dá no tempo
v0 · sin(θ)
tM = .
g
O tempo que transcorre entre a saı́da da bala e sua chegada ao chão é 2 · tM .
8. BALÍSTICA E O SUPER MÁRIO 502

A Figura a seguir ilustra um tiro certeiro:

1,6
1,2
0,8
0,4
0
0 2 4 6 8
x

Figura: θ = π5 , v0 = 10, x ∼ 9.7, altura máxima ∼ 1.7.

Demonstração.
A velocidade v0 tem uma componente horizontal e uma vertical.
A horizontal é x′ (0) = v0 · cos(θ) e a vertical y ′ (0) = v0 · sin(θ).
Não há componente horizontal da força de gravidade. Portanto,10 se x(t) é a
coordenada horizontal da posição da bala:
x′′ (t) ≡ 0
o que dá:
x′ (t) ≡ C = x′ (0)
e portanto:
x(t) − x(0) = x′ (0) · t.
Como (x(0), y(0)) = (0, 0) temos:
x(t) = x′ (0) · t = v0 · cos(θ) · t, ∀t ≥ 0.
Mas a gravidade g afeta a componente vertical. De fato:
y ′′(t) = −g,
(onde o sinal vem da oposição entre o sentidos).
Logo
y ′ (t) − y ′ (0) = −g · t,
ou seja,
y ′(t) = y ′(0) − g · t,
e daı́ obtemos:
g · t2
y(t) − y(0) = y ′ (0) · t − .
2
Ou seja
g · t2
y(t) = v0 sin(θ) · t − .
2
10E se supõe que a bala não sofre resistência
CAPÍTULO 35. AS PRIMEIRAS EQUAÇÕES DIFERENCIAIS 503

Substituindo
x(t) x
t= ′
= ′
x (0) x (0)
em
g · t2
y(t) = v0 sin(θ) · t −
2
obtemos a parábola
g
y=− · x2 + tan(θ) · x,
2· v02 2
· cos (θ)
que é a descrição da trajetória da bala.
Sabemos encontrar o ponto de máximo de uma parábola y = ax2 + bx + c, onde
a < 0. Esse ponto é x = −b 2a
. No caso da parábola acima obtemos:

v02 · sin(θ) cos(θ)


x=
g
e daı́ obtemos a altura máxima.
O tempo tM em que se atinge essa altura máxima é obtido de igualar a componente
vertical da velocidade a zero:

0 = y ′(tM ) = y ′(0) − g · tM ,

portanto:
y ′(0)
tM = .
g
E o tempo tF > 0 no qual a bala atinge o alvo é obtido de igualar y(tF ) = 0 e resolver:

g · t2
0 = v0 sin(θ) · t −
2
cujas raı́zes são t = 0 e
2 · y ′(0)
tF = = 2 · tM .
g
A coordenada x do alvo atingido pode ser obtida ou avaliando x(t) em tF ou
vendo-se a intersecção da parábola acima com o eixo x. De ambos os modos obtêm-
se:
v 2 · sin(2 · θ)
x= 0 .
g

10. UM PROBLEMA DA PUTNAM COMPETITION, N.14, 1954 504

Deixo para o Exercı́cio 14.7 a prova de uma propriedade de balı́stica conhecida


por Galileu, exemplificada na Figura a seguir:

0
0 2 4 6 8 10

9. Equações diferenciais lineares em geral


Uma equação diferencial de primeira ordem linear geral é uma equação do seguinte
tipo:
f ′ (x) = a(x) · f (x) + b(x),
onde a incógnita é a função y = f (x).
Como veremos na Afirmação 11.1 a seguir (que generaliza a Afirmação 4.1) a
solução dessa equação não é única mas forma uma famı́lia de curvas, chamadas de
curvas integrais da equação. A curva solução só fica determinada quando impomos
que passe por algum ponto do plano.

10. Um problema da Putnam Competition, n.14, 1954


O que é interessante é que, antes de sabermos quem são as curvas integrais, já
podemos responder a um problema:
Problema: Se a famı́lia de curvas integrais da equação:
f ′ (x) + p(x) · f (x) = q(x), com p(x) · q(x) 6= 0
é cortada pela reta vertical x = k, então as retas tangentes às curvas integrais pelos
pontos de intersecção concorrem todas num mesmo ponto.

Solução:
Denoto por fα (x) e fβ (x) duas curvas integrais distintas.
Vou tomar duas retas tangentes às curvas integrais fα (x) e fβ (x) por pontos
distintos da reta x = k:
(k, fα (k)) e (k, fβ (k)).
A primeira verifica:
y − fα (k)
= fα′ (k) = −p(k) · fα (k) + q(k)
x−k
CAPÍTULO 35. AS PRIMEIRAS EQUAÇÕES DIFERENCIAIS 505

enquanto que a segunda:


y − fβ (k)
= fβ′ (k) = −p(k) · fβ (k) + q(k).
x−k
Ou seja, a primeira é a reta:
y = (−p(k) · fα (k) + q(k)) · x − k · (−p(k) · fα (k) + q(k)) + fα (k).
enquanto a segunda é:
y = (−p(k) · fβ (k) + q(k)) · x − k · (−p(k) · fβ (k) + q(k)) + fβ (k).
Quando consideramos a interseção dessas retas temos que resolver a equação:
−p(k) · fα (k) · x + (kp(k) + 1) · fα (k) = −p(k) · fβ (k) · x + (kp(k) + 1) · fβ (k)
ou seja:
(kp(k) + 1) · (fβ (k) − fα (k)) kp(k) + 1
x= = ,
p(k) · (fβ (k) − fα (k)) p(k)
que não depende das fα e fβ particulares que tomei. Portanto essa é a coordenada x
do ponto onde concorrem todas as retas tangentes.
Fiz um Exemplo, antecipando o resultado da próxima Seção sobre quem são as
curvas integrais da equação.
Tomei
2
f ′ (x) + p(x) · f (x) = q(x), com p(x) = , q(x) = cos(x), x ∈ [0.8, 6]
x
pois de fato quem não pode se anular é p(x) = x2 .
Escolhi k = 2 e tracei 11 curvas integrais, na próxima Figura:

0
1 2 3 4 5 6
x

-2

-4

Agora adicionei suas 11 retas tangentes nas interseções com x = 2. Segundo


2· 2 +1
nossas contas devem se encontrar no ponto cuja coordenada x vale 22 = 3, o que
2
se vê bem na Figura:
11. SOLUÇÕES DAS EQUAÇÕES LINEARES GERAIS 506

2
x
1 2 3 4 5 6
0

-2

-4

11. Soluções das equações lineares gerais


Agora vamos ver quem são as soluções das equações diferenciais lineares de primeira
ordem:
Afirmação 11.1.
Sejam a(x), b(x) e f (x) funções definidas num intervalo aberto e com valores em
R, tais que a(x) e b(x) são contı́nuas e f derivável, com f ′ (x) função contı́nua ao
menos.
• i) Se f ′ (x) = a(x) · f (x) então
R
a(x) dx
f (x) = C · e , com C ∈ R.
Dado f (x0 ) então
Rx
a(t) dt
f (x) = f (x0 ) · e x0
.
• ii) Se f ′ (x) = a(x) · f (x) + b(x) então
R
Z R R
a(t) dt
f (x) = e · e − a(t) dt · b(x) dx + C · e a(t) dt .

• iii) se a(x) ≡ a e b(x) ≡ b, então ii) vira:


e−ax b
f (x) = eax · · b + C · eax = − + C · eax .
(−a) a
CAPÍTULO 35. AS PRIMEIRAS EQUAÇÕES DIFERENCIAIS 507

Demonstração.

De i):
Usaremos a mesma idéia da prova da Afirmação 4.1.
Primeiro noto que a função f ≡ 0 é solução e corresponde a tomar C = 0.
Podemos então supôr no que segue que f 6≡ 0.
Faremos a suposição a princı́pio mais forte11 de que:
∀x ∈ R, f (x) 6= 0.
Então posso fazer:
f ′ (x)
= a(x).
f (x)
Tomando primitivas (e colocando as constantes do lado direito):
Z
ln ||f (x)|| = a(x) dx + C1 .

Logo R R R
||f (x)|| = e a(x) dx+C1 = e a(x) dx · eC1 = C2 · e a(x) dx .
Pelo T.V.I. sabemos que ou bem f (x) > 0 ∀x ou bem f (x) < 0 ∀x.
Então: R R
f (x) = C2 · e a(x) dx ou f (x) = −C2 · e a(x) dx .
Em qualquer dos casos,
R
a(x) dx
f (x) = C · e , com C 6= 0.
Se tomo x0 no domı́nio da f , acima poderı́amos ter escrito:
Z x
ln ||f (x)|| − ln ||f (x0 )|| = a(t) dt,
x0
e daı́ terı́amos:
Rx Rx
a(t) dt+ln ||f (x0 )|| a(t) dt
||f (x)|| = e x0
= ||f (x0 )|| · e x0
.
Em qualquer dos casos (f (x) > 0 ∀x ou f (x) < 0 ∀x):
Rx
a(t) dt
f (x) = f (x0 ) · e x0
.
De ii):
Agora temos:
f ′ (x) = a(x) · f (x) + b(x)
e o leitor em seguida vê que a idéia da prova da Afirmação 4.1 já não funciona aqui:
ou seja, não aparece mais uma derivada logarı́tmica do lado esquerdo.
O que faremos é multiplicar toda a equação dada por um fator µ(x) adequada-
mente escolhido para que do lado esquerdo apareça a derivada de algo, apesar de que
esse algo nem sempre será o logaritmo.
Faço
f ′ (x) − a(x) · f (x) = b(x)
11Na verdade, através da Afirmação 3 do Capı́tulo 36 se mostra que são a mesma hipótese
11. SOLUÇÕES DAS EQUAÇÕES LINEARES GERAIS 508

e
µ(x) · f ′ (x) − µ(x) · a(x) = µ(x) · b(x).
Quero que valha:
µ(x) · f (x) − µ(x) · a(x) = ( µ(x) · f (x) )′
e para isso temos que ter:
µ′ (x) = −a(x) · µ(x),
já que:
( µ(x) · f (x) )′ = µ(x) · f ′ (x) + µ′ (x) · f (x).
Ora, o item i) nos diz quem são as soluções µ(x) de µ′ (x) = −a(x) · µ(x) e tomo uma
com C = 1: R
µ(x) = e − a(t) dt .
Portanto: R R
− a(t) dt
(e · f (x) )′ = e − a(t) dt
· b(x).
Tomando primitivas e passando a constante para a direita:
R
Z R
− a(t) dt
e · f (x) = e − a(t) dt · b(x) dx + C

e portanto: Z
R R R
a(t) dt − a(t) dt a(t) dt
f (x) = e · e · b(x) dx + C · e .

Vejamos Exemplos para a Afirmação 11.1:


• Tomemos as equações do tipo
f ′ (x) = xk · f (x), com k ∈ Z, para x > 0.
Escolho o ponto x0 = 1. É claro que
Z x
xk+1 1
tk dt = − se k 6= −1
1 k+1 k+1
ou Z x
t−1 dt = ln(x) se k = −1.
1
Portanto pelo item i):
xk+1
e k+1
f (x) = f (1) · 1 , se k 6= −1
e k+1
ou
f (x) = f (1) · x, se k = −1.
CAPÍTULO 35. AS PRIMEIRAS EQUAÇÕES DIFERENCIAIS 509

• Agora considere as equações do tipo


−n
f ′ (x) = · f (x) + 2n · xn−1 , com n ∈ N, para x > 0
x
Temos pelo item ii):
R −n
Z R R −n
n
dt
f (x) = e t · e t dt · b(x) dx + C · e t dt .
mas agora: R n
dt
e= en·ln(x) = xn , onde x > 0
t
−n
R
enquanto que e t dt = x1n e daı́:
Z R Z
n
dt
e t · b(x) dx = 2n · x2n−1 dx = x2n .

Logo obtemos
1 C C
n
· x2n + n = xn + n .
f (x) =
x x x
A determinação de C depende da escolha de um valor f (x0 ), pois C =
xn0 · (f (x0 ) − xn0 ).

0
1 2 3 4 5
x
-2

-4

Fig. As curvas y = x + Cx com C = −3, −2, −1, 0, 1, 2, 3.


• Agora considere a equação
−2
f ′ (x) = · f (x) + cos(x), para x > 0
x
Pelo item ii):
R −2
Z R R −2
2
f (x) = e t · e t dt · cos(x) dx + C · e t dt ,
dt

onde, como antes,


R 2
R −2 1
dt
e t = x2 e e t
dt
= onde x > 0.
x2
E Z
x2 · cos(x) dx = x2 · sin(x) + 2x · cos(x) − 2 sin(x),
12. UM PROBLEMA DA PUTNAM COMPETITION, N. 49, 1958. 510

como vimos num dos Exemplos do Capı́tulo 24. Logo obtemos :


2 cos(x) 2 sin(x) C
f (x) = sin(x) + − + 2.
x x2 x

A Figura a seguir mostra essas curvas para C = −3,−2,−1,0,1,2,3.

0
2 4 6 8 10
x

-2

Note que à medida que x cresce essas as curvas todas se aproximam de


y = sin(x).

12. Um problema da Putnam Competition, n. 49, 1958.

Problema: Um erro comum no Cálculo é achar que:


(f (x) · g(x)) ′ = f ′ (x) · g ′(x).
2
Se f (x) = ex prove que existe uma g(x) 6≡ 0 definida num intervalo aberto tal que
para essas f e g vale:
(f (x) · g(x)) ′ = f ′ (x) · g ′(x).
Solução:
Queremos que
2 2
(ex )′ · g ′(x) = (ex · g(x))′ ,
mas por outro lado certamente:
2 2 2
(ex · g(x))′ = (ex )′ · g(x) + ex · g ′(x) =
2 2
= 2x · ex · g(x) + ex · g ′ (x).
Então obtemos:
2 2 2
2x · ex · g ′ (x) = 2x · ex · g(x) + ex · g ′ (x),
de onde
2x
g ′ (x) = · g(x),
2x − 1
supondo 2x − 1 6= 0.
CAPÍTULO 35. AS PRIMEIRAS EQUAÇÕES DIFERENCIAIS 511

Esse tipo de equação é tratada pelo item i) da Afirmação 11.1: se g(x) > 0 e se
2x − 1 > 0, então R 2x
g(x) = eC · e 2x−1 dx .
Ora:
2x 1
=1+
2x − 1 2x − 1
e portanto (módulo constantes)
Z
2x ln(2x − 1)
dx = x + ,
2x − 1 2
de onde
ln(2x−1) √ 1
g(x) = ex+ 2 = ex · 2x − 1, para x > .
2
13. As equações de Bernoulli e sua redução a equações lineares
Jakob Bernoulli considerou uma classe de equações diferenciais extremamente
úteis, como veremos em aplicações no Capı́tulo 38. Mas as equações dessa vez são
não-lineares (pois envolvem o termo f (x)r ).
O que é incrı́vel é que elas podem ser transformadas em equações diferenciais
lineares. O truque é do grande Leibniz !
Repare que os casos r = 0, 1 na Afirmação 13.1 a seguir já estão resolvidos pela
Afirmação 11.1 acima.
Afirmação 13.1. Sejam a(x), b(x) contı́nuas, f (x) derivável com f ′ (x) contı́nua.
Suponha12
f ′ (x) = a(x) · f (x) + b(x) · f (x)r , r 6= 0, 1, r ∈ R.
Então
• g(x) := f 1−r (x) satisfaz a equação diferencial linear:
g ′ (x) = (1 − r) · a(x) · g(x) + (1 − r) · b(x)
e portanto ou f (x) ≡ 0 ou13
R
Z R R 1
(1−r)a(t)dt
f (x) = [ e · e (r−1)a(t)dt · (1 − r)b(x) dx + C · e (1−r)a(t)dt ] 1−r

Demonstração.
Mais uma vez, após considerar a situação em que f ≡ 0, trocaremos a condição
f 6≡ 0 pela condição a princı́pio mais forte14
f (x) 6= 0, ∀x.
Noto que se g(x) := f 1−r (x) , então:
g ′(x) (1 − r) · f −r (x) · f ′ (x)
= =
g(x) f 1−r (x)
12dependendo do r ∈ R pode ser necessário supôr que f (x) > 0 para que faça sentido f (x)r .
13Onde aparece r − 1 na fórmula a seguir ao invés de 1 − r está correto, não inverta ...
14Na verdade, através da Afirmação 3 do Capı́tulo 36 se mostra que são a mesma hipótese
14. EXERCÍCIOS 512

f ′ (x)
= (1 − r) · =
f (x)
(1 − r) · a(x)f (x) + (1 − r) · b(x)f r
= =
f (x)
= (1 − r) · a(x) + (1 − r) · b(x)f r−1 =
b(x)
= (1 − r) · a(x) + (1 − r) · ,
g(x)
e portanto multiplicando por g(x):
g ′(x) = (1 − r) · a(x)g(x) + (1 − r) · b(x).
Como já sabemos resolver esta equação pela Afirmação 11.1, temos g(x) e daı́ a f (x).


Um Exemplo:
y ′(x) = x · y(x) + y(x)2 ,
cuja solução portanto é:
2
Z
x2 x2
− x2
y = [−e · e 2 dx + C · e− 2 ]−1 , C ∈ R.

14. Exercı́cios
Exercı́cio 14.1. (resolvido)
A função representada a seguir é estritamente decrescente e tende a zero. No
entanto, afirmo que ela não pode representar a desintegração de nenhuma substância
radioativa, devido a aspecto (s) qualitativo (s) de seu gráfico.
Explique quê aspecto qualitativo é (são) esse(s), usando os conceitos e a teoria
desenvolvida neste Curso.

35

30

25

20

15

10
0 1 2 3 4
x

Exercı́cio 14.2. Quanto tempo tem que ter passado para que uma mostra de osso
tenha menos que 10−3 vezes a quantidade original de C14 ?
Exercı́cio 14.3. Em quanto tempo duplica uma dı́vida que cresce segundo a equação
f ′ (x) = 2 · f (x) ?
CAPÍTULO 35. AS PRIMEIRAS EQUAÇÕES DIFERENCIAIS 513

Exercı́cio 14.4. (resolvido)


A 21 -vida é o tempo τ transcorrido para que uma substância radioativa tenha
massa f (τ ) igual à metade da massa inicial f (0).
i) Suponha que defino a 41 -vida como o tempo τ̂ transcorrido para que uma
substância radioativa tenha massa f (τ̂ ) igual a um quarto da massa inicial f (0).
Qual a relação entre τ̂ e τ ?
ii) Suponha agora que defino a √12 -vida como o tempo τ̌ transcorrido para que
uma substância radioativa tenha massa f (τ̌ ) igual f√(0)
2
. Qual a relação entre τ̌ e τ ?
1
iii) Mais geralmente, chamo agora de 1 -vida o tempo τn transcorrido para que
2n
f (0)
uma substância radiotiva tenha massa f (τn ) igual 1 . Qual a relação entre τn e τ ?
2n

Exercı́cio 14.5. Em 10 anos a quantidade inicial f (0) de uma substância radioativa


caiu para f (0)
3
.
i) qual o valor de k na equação f ′ (x) = −kf (x) do decaimento ?
ii) qual a meia-vida dessa substância (em função do k do item i) ?
Exercı́cio 14.6. (resolvido)
Considere a equação f ′ (x) = −kf (x), com −k < −1 e f (0) = 1. Note que então

f (0) = −k < −1.
Para qual tempo x temos que o coeficiente angular da tangente ao gráfico da
solução y = f (x) é exatamente −1 ?

Exercı́cio 14.7. A Figura a seguir ilustra em vermelho a trajetória de uma bala de


canhão que forma ângulo de π4 com o eixo x, atingindo o alcance máximo.
E em amarelo e verde dois lançamentos com ângulos π4 + 0.4 e π4 − 0.4, respecti-
vamente.

0
0 2 4 6 8 10

Por quê atingiram o mesmo ponto ?


Galileu já conhecia essa propriedade !
Exercı́cio 14.8. Suponha que um objeto com temperatura t0 é colocado num ambi-
ente com temperatura T (que é mantida constante). Suponha que t0 > T .
14. EXERCÍCIOS 514

A lei de esfriamento de Newton diz que a taxa de variação da temperatura do


objeto em cada instante é proporcional à diferença de temperatura entre o objeto e
o ambiente naquele instante.
Modele a equação diferencial do esfriamento e a resolva.
Tendo obtido a solução, mostre que quando t → +∞ a temperatura do objeto
tende à do ambiente.
Exercı́cio 14.9. Suponha que y(x) é a quantidade de indivı́duos de uma espécie e
que seu desenvolvimento é modelado pela equação:
y ′ (x) = a · y(x) − x, onde a > 0,
ou seja, onde supõe-se que os fatores adversos (ataques de predadores, escassez, etc)
dependem do tempo como a função −x.
a) Prove que a população no tempo verifica:
1 x 1
y(x) = 2 + + (f (0) − 2 ) · eax .
a a a
b): discuta as condições iniciais f (0) que produzem superpolação ou extinção a
longo prazo.
c): para todo a > 0, calcule y ′(0). Esboce as diferentes soluções.
Exercı́cio 14.10. (resolvido)
Suponha que y(x) é a quantidade de indivı́duos de uma espécie e que seu desen-
volvimento é modelado pela equação:
y(x)
y ′(x) = − x, x ≥ 0.
x+1
Ou seja, onde supõe-se que os fatores propı́cios (fertilidade, alimentos, etc) depen-
1
dem do tempo como x+1 enquanto que os fatores adversos (ataques de predadores,
escassez, etc) dependem do tempo como a função −x.
a) Prove que a população no tempo verifica:
y(x) = (1 + x) · [y(0) + ln(1 + x) − x], C ∈ R.
b): dê um argumento para provar que, não importa qual C, sempre:
lim y(x) = −∞,
x→+∞

ou seja, que essa população está fadada à extinção.


CAPı́TULO 36

Aspectos gerais das equações de primeira ordem

1. Equações diferenciais e metamorfoses de curvas


Quando temos uma equação diferencial:
y ′ (x) = f (x)
para f contı́nua e x num intervalo, sabemos que :
y(x) = F (x) + c
onde F (x) é uma primitiva de f (x).
Essa famı́lia de gráficos y = F (x)+c é bem trivial, pois é composta de translações
verticais do gráfico y = F (x).
Mas uma equação diferencial do tipo separável 1:
g(y) · y ′(x) = f (x)
já produz famı́lias de gráficos ou curvas bem interessantes.
Para começar a equação:
y · y ′(x) = −x
se resolve notando que ela se escreve como
2 2
d( y(x)
2
) d( x2 )
=−
dx dx
e daı́:
y(x)2 + x2 = c, c ∈ R
que é uma famı́lia de cı́rculos concêntricos quando c > 0.
Aqui não há gráficos, mas apenas curvas, e não há translações mas sim contrações
e expansões das curvas.
Agora vejamos o Exemplo:
2y · y ′(x) = 3x2 − 1,
que pode ser escrito como:
d(y(x)2 ) d(x3 − x)
= ,
dx dx
de onde:
y 2 = x3 − x + c, c ∈ R.
Essa famı́lia de cúbicas já foi estudada ao longo do Curso, por exemplo na Seção 5
do Capı́tulo 3. O caso c = 0 é ilustrado na figura a seguir:
1Veremos em detalhe este tipo de equação na Seção 4
515
1. EQUAÇÕES DIFERENCIAIS E METAMORFOSES DE CURVAS 516

y 0
-1 -0,5 0 0,5 1 1,5 2
x
-1

-2

-3

A Figura a seguir plota y 2 = x3 − x ao lado de y 2 = x3 − x + 1:

y 0
-1 -0,5 0 0,5 1 1,5 2
x
-1

-2

-3

A Figura a seguir plota y 2 = x3 − x, y 2 = x3 − x + 1 e y 2 = x3 − x − 1:


3

y 0
-1 -0,5 0 0,5 1 1,5 2
x
-1

-2

-3

A Figura a seguir plota y 2 = x3 − x + c para os valores


c = −4, −3, −2, −1, 0, 1, 2, 3, 4.

y 0
-1 0 1 2
x
-1

-2

-3

Note que:
CAPÍTULO 36. ASPECTOS GERAIS DAS EQUAÇÕES DE PRIMEIRA
ORDEM 517

• para c ∈ {−4, −3, −2, −1} ou c ∈ {4, 3, 2, 1} há apenas mudanças quantita-
tivas nas curvas, ou seja, quando a curva muda um pouco mas tem o mesmo
aspecto geral.
• mas quando c ∈ {−1, 0, 1} as curvas correspondentes passam por mudanças
qualitativas importantes.

De fato, como será explicado no Capı́tulo 32 o valor


2
c= √
3 3
é um divisor de águas nessa famı́lia de curvas. Para esse valor preciso de c a curva
tem o formato de um laço (que o Maple não plota muito bem...)
A Figura a seguir plota as curvas para c = −1, 0, 3√2 3 , 1:

y 0
-1 -0,5 0 0,5 1 1,5 2
x
-1

-2

-3

2. Equações diferenciais em forma normal e as curvas Isóclinas


Quando escrevemos uma equação diferencial de primeira ordem (i.e. onde só entra
a primeira derivada e a função) na forma:
y ′(x) = P (x, y),
ou seja, onde isolamos y ′, dizemos que a equação está na forma normal.
Quando se quer ter uma noção qualitativa grosseira das soluções da equação:
y ′ (x) = P (x, y)
se traçam as curvas isóclinas (mesma inclinação em grego), ou seja, as curvas dadas
implicitamente por:
P (x, y) = k,
que são as curvas no plano tais que as inclinações y ′ têm o mesmo valor k.
O Exemplo
y ′ (x) = x · y
é bom para começar, não só porque suas isóclinas são as hipérboles x · y = k (que à
medida que k → 0 se expremem sobre os eixos coordenados), mas também porque
cai no formato da Seção anterior g(y) · y ′ (x) = f (x):
1 ′
· y (x) = x, se y 6= 0.
y
2. EQUAÇÕES DIFERENCIAIS EM FORMA NORMAL E AS CURVAS
ISÓCLINAS 518

É possı́vel dar uma desenho qualitativo das curvas y = y(x) solução dessa equação
na Figura a seguir:

Os segmento verticais são pedaços das retas tangentes à curvas soluções. Por isso
pode ser chamado de campo de direções tangentes.
Como a equação y1 · y ′(x) = x pode ser escrita:
2
d ln |y(x)| d( x2 )|
=
dx dx
então
x2
ln |y(x)| = +c
2
de onde
x2 x2
|y(x)| = e 2 +c = C · e 2 , C>0
e
x2
y = y(x) = C · e 2 , C ∈ R \ {0}.
Só que na discussão que fizemos impusemos que
y 6= 0.
E com isso esquecemos a solução
y ≡ 0 de y ′(x) = x · y(x).
Como veremos na Afirmação 3.1 da próxima Seção, quando uma equação está na
forma normal
y ′ (x) = P (x, y)
e quando P (x, y) e ∂P
∂y
são funções contı́nuas no plano, como é o caso para
∂P
P (x, y) = x · y, = x,
∂y
há unicidade da solução por cada ponto. Em particular o gráfico de uma solução
y1 6≡ 0 não pode intersectar o eixo y ≡ 0, pois este é solução da mesma equação.
CAPÍTULO 36. ASPECTOS GERAIS DAS EQUAÇÕES DE PRIMEIRA
ORDEM 519

No próximo Exemplo se trata de uma Equação de Bernoulli :

y ′(x) = x · y(x) + y(x)2 .

É uma equação não-linear (termo quadrático em y(x)) que pode ser reduzida a uma
equação linear de primeira ordem, o que é raro e surpreendente, como vimos na Seção
13.1 do Capı́tulo 35. Vimos lá que as soluções são

2
Z
x2 x2
− x2
y = [−e · e 2 dx + C · e− 2 ]−1 , C ∈ R.

Note que

x · y + y2 = k

são hipérboles que se espremem sobre os eixos y = 0 e y + x = 0, já que x · y + y 2 =


y · (x + y). A Figura a seguir ilustra esses dois eixos, 4 isóclinas algumas soluções
(apenas qualitativamente).

O Exemplo

y ′(x) = x2 + y 2

é muito interessante. Aparenta ser mais fácil de tratar que o anterior. Mas não é !
Suas curvas isóclinas são sim imediatas, pois são cı́rculos ou a origem se k ≥ 0:

x2 + y 2 = k, k≥0

e feitas em detalhe dão uma boa idéia - qualitativa - das curvas que são soluções.
3. EXISTÊNCIA E UNICIDADE PARA Y ′ (X) = F (X, Y ) - MÉTODO DE
PICARD 520

Porém y ′ (x) = x2 + y 2 é a primeira equação de Riccati não-trivial na literatura,


estudada pelo Riccati e por Johan Bernoulli.
Suas soluções explı́citas y(x) não são funções que tenham sido apresentadas a
quem fez Cálculo 1 e 2. São funções não-elementares, são de fato composições de
funções de Bessel e suas derivadas.
Dedicarei um Capı́tulo às Riccati e a solução explı́cita de y ′ = x2 + y 2 se encontra
na Seção 4 do Capı́tulo 45. As funções de Bessel serão tratadas no Capı́tulo 43 (pelo
menos algum rudimento, pois têm uma vasta teoria).

3. Existência e unicidade para y ′(x) = F (x, y) - Método de Picard


O Teorema a seguir assegura existência e unicidade de soluções de equações de
primeira ordem na forma normal, sob certas condições. É muito importante como
fundamentação da teoria de equações diferenciais, embora não seja considerado com-
putacionalmente rápido.
Teorema 3.1. Seja uma equação diferencial do tipo y ′(x) = F (x, y), com F (x, y)
função de duas variáveis.
Suponha que as funções F (x, y) e ∂F∂y
são contı́nuas2 numa região U aberta do
plano contendo (a, b).
Então para cada ponto (a, b) ∈ U existe e é única a função y = y(x) verificando

y (x) = F (x, y(x)) e y(a) = b, para x ∈ Ia onde Ia é um intervalo aberto centrado em
a.
Em particular, se y ≡ C for solução da equação então as outras soluções nunca
assumem esse valor C.
Em particular, se y ≡ 0 for solução da equação então as outras soluções nunca se
anulam.

2O Apêndice deste Capı́tulo, Seção 15, explica bem esta noção


CAPÍTULO 36. ASPECTOS GERAIS DAS EQUAÇÕES DE PRIMEIRA
ORDEM 521

Não vejo exemplo mais simples para mostrar a importância das hipóteses deste
Teorema, do que a equação:
y
y ′ (x) = .
x
Ela é separável
y ′(x) 1
= , sex · y 6= 0
y(x) x
e se resolve como:
ln ||y|| = ln ||x|| + C1
ou seja:
y = C2 x.
Pela origem há uma infinidade de soluções e pelo eixo dos y, onde x = 0, não
há soluções. Pois é ao longo de x = 0 que não há continuidade da função de duas
variáveis F (x, y) = xy .

Idéia da prova do Teorema 3.1:


Uma prova perfeitamente legı́vel se encontra no livro de Bear. Mas posso indicar
ao menos algumas idéias da prova:
• primeiramente notar que y = y(x) é solução de y ′ (x) = F (x, y) e satisfaz
y(a) = b se e somente se
Z x
y(x) = b + F (t, y(t)) dt.
a

R x De fato, Rse y(x) é solução de y ′ (x) = F (x, y) então y(x)


R x − y(a) =
′ x
a
y (t) dt = a F (t, y(t)) dt. Reciprocamente, se y(x) = b + a F (t, y(t)) dt
então y ′ (x) = F (x, y(x)).
• A partir daı́ Picard considera uma sequência de funções yn (x) definida recur-
sivamente por:
Z x
y0 (x) ≡ b, yn (x) := b + F (t, yn−1 (t)) dt.
a

• aR condição de que F (x, y) é contı́nua garante que existam as integrais b +


x
a
F (t, yn−1(t)) dt e também garante que existe um intervalo Ia em torno de
a em que todas as yn (x) estão definidas.
• a condição ∂F∂y
é contı́nua vai ser usada para garantir que a sequência yn (x)
convirja uniformemente para uma função
y+∞ (x) := lim yn (x)
n→+∞

e que valha
Z x Z x
lim b + F (t, yn−1 (t)) dt = b + F (t, y+∞ (t)) dt.
n→+∞ a a

• para que haja unicidade, ou seja, para que qualquer solução Y (x) com Y (a) =
b seja da forma Y = y+∞ também é preciso que ∂F∂y
seja contı́nua.
3. EXISTÊNCIA E UNICIDADE PARA Y ′ (X) = F (X, Y ) - MÉTODO DE
PICARD 522

Exemplo:
Quando F (x, y) é um polinômio é fácil implementar o método. Vou implementar
as primeiras etapas da recursão no
Caso 1): y ′ = −y 2 , y(1) = 1
′ 2
Caso 2): y = −x + y , y(0) = b.
No caso 1):
y0 ≡ 1, y1 = 2 − x,
10 1
y2 = − 4x + 2x2 − x3 ,
3 3
323 100 40 2 88 3 41 4 4 5 2 6 1
y3 = − x + x − x + x − x + x − x7 .
63 9 3 9 9 3 9 63
Ou seja, o método está nos dando uma aproximação (não muito rápida, infelizmente)
de:
1 1
y= = = 1 + (1 − x) + (1 − x)2 + (1 − x)3 + . . . para |1 − x| < 1
x 1 − (1 − x)
pois
1 + (1 − x) = 2 − x, 1 + (1 − x) + (1 − x)2 + (1 − x)3 = 4 − 6x + 4x2 − x3 ,
1 + (1 − x) + . . . + (1 − x)7 = 8 − 28x + 56x2 − 70x3 + 56x4 − 28x5 + 8x6 − x7 .
A figura a seguir ilustra:

0
0,5 1 1,5 2 2,5 3
x

-1

Fig.: y = x1 em vermelho, y1 verde, y2 amarelo, y3 azul.


No Caso 2), o método de Picard começa com:
y0 ≡≈ 0.73,
(pelo que veremos mais adiante esse é o valor aproximado de y(0)) e faz
y1 ≈ 0.73 + 0.53x − 0.5x2 ,
CAPÍTULO 36. ASPECTOS GERAIS DAS EQUAÇÕES DE PRIMEIRA
ORDEM 523

y2 ≈ 0.73 + 0.53x − 0.1x2 − 0.15x3 − 0.13x4 + 0.05x5


y3 ≈ 0.73 + 0.53x − 0.11x2 + 0.04x3 − 0.08x4 − 0.06x5 − 0.006x6 + 0.01x7 +
+0.003x8 + 0.0003x9 − 0.001x10 + 0.0002x11 .

Veremos na Seção 6 do Capı́tulo 44 que a solução y(x) no Caso 2) não é uma


função já conhecida nossa; ou seja, não é elementar. Seu gráfico para x ∈ [−2.2, 4] é
do tipo:

x
-2 -1 0 1 2 3 4
0

-2

-4

-6

Na figura a seguir y(x) está comparado com as primeiras aproximações:

x
-2 -1 0 1 2
0

-1

-2

-3

Fig.: y(x) em vermelho, y1 verde, y2 amarelo, y3 azul.


3. EXISTÊNCIA E UNICIDADE PARA Y ′ (X) = F (X, Y ) - MÉTODO DE
PICARD 524

Exemplo:

De volta ao exemplo:
2y · y ′(x) = 3x2 − 1,
quando posto na forma padrão vira:
′ 3x2 − 1
y (x) = .
y
Se considero U = {(x, y); y > 0} (o semiplano superior), posso usar o Teorema 3.1 e
para cada ponto desse semiplano passa apenas uma solução y = y(x). Sabemos que
a equação é satisfeita pelas curvas y 2 = x3 − x + c, que não são gráficos, mas mas
restritas ao semiplano superior sim são gráficos do tipo y = y(x).
Ou seja, na Figura a seguir só devemos considerar a parte das curvas acima do
eixo horizontal.

y 0
-1 0 1 2
x
-1

-2

-3

Quando y = 0 aı́ não podemos usar o Teorema 3.1 e de fato, como vemos nessa
mesma figura, sobre o eixo dos x há:
• pontos onde as curvas são gráfico de x = x(y), não de y = y(x)
• pontos de onde saem mais de uma ramo de curva

Exemplo: Considero a a equação:


−y · cos(x)
y ′(x) = , x ∈ (0, π), y ∈ (−2, 1).
(y + 2) · sin(x)
Nessa região retangular aberta U = (0, π) × y ∈ (−2, 2) posso aplicar o Teorema 3.1.
−y·cos(x)
Antes de resolver a equação noto, só pela expressão y ′ (x) = (y+2)·sin(x) que:
• onde y ∼ 0, as inclinações y ′(x) dos gráficos ficam quase zero.
• onde y > 0 e x ∼ 0 as inclinações y ′ (x) ficam muito negativas (pois sin(x) ∼ 0
e cos(x) ∼ 1)
• onde y > 0 e x ∼ Π as inclinações y ′ (x) ficam muito positivas (pois sin(x) ∼ 0
e cos(x) ∼ −1)
• onde y < 0 e x ∼ 0 as inclinações y ′ (x) ficam muito positivas
• onde y < 0 e x ∼ Π as inclinações y ′(x) ficam muito negativas
• para x ∼ Π2 as inclinações ficam perto de zero (pois cos(x) ∼ 0).
CAPÍTULO 36. ASPECTOS GERAIS DAS EQUAÇÕES DE PRIMEIRA
ORDEM 525

• onde y ∼ −2 as inclinações ficam quase verticais.


Ilustro isso a seguir:

y(x)0
0 0,5 1 1,5 2 2,5 3
x

-1

-2

Quais as soluções dessa equação diferencial ? Veremos na Seção 4 a seguir.

4. Equações separáveis
Note que nos últimos exemplos da Seção anterior, as equações são de tipo especiais,
pois:
y ′(x) = F (x, y)
nesses exemplos pode ser escrita como:
f (x)
y ′(x) = .
g(y)
No Exemplo anterior:
3x2 − 1
y ′ (x) =
2y
e neste

( −sin(x)
cos(x)
)
y (x) = .
( y+2
y
)
Uma equação desse tipo
f (x)
y ′ (x) =
g(y)
é chamada de separável.
Para resolver uma equação separável em geral, noto que pela regra da cadeia posso
escrever3:
d (G(y(x)) − F (x))
g(y) · y ′(x) − f (x) = = 0,
dx
3Ou seja, uma equação separável é sempre exata no sentido da próxima Seção 7
4. EQUAÇÕES SEPARÁVEIS 526

desde que
d G(y) d F (x)
= g(y) e = f (x).
dy dx
E portanto a solução geral é da forma:

G(y(x)) − F (x) = C.

Num dos exemplos da Seção anterior, onde

−f (x) = −3x2 + 1 e g(y) = 2y

temos:
G(y(x)) − F (x) = y 2 − x3 + x = C
e no segundo onde

cos(x) y+2 2
−f (x) = e g(y) = =1+
sin(x) y y
temos:
G(y(x)) − F (x) = y + 2 ln |y| + ln | sin(x)| = C.
Para x ∈ (0, π) ploto a seguir

y + 2 ln |y| + ln | sin(x)| = C > 0

para alguns valores de C > 0, com y ∈ (−2, 2).

y 0
0,5 1 1,5 2 2,5 3
x

-1

-2

A seguir faço a união x ∈ (−π, 0) ∪ (0, π) e uso ainda y ∈ (−2.2), o que já nos dá
uma idéia da periodicidade das soluções:
CAPÍTULO 36. ASPECTOS GERAIS DAS EQUAÇÕES DE PRIMEIRA
ORDEM 527

y 0
-3 -2 -1 0 1 2 3
x

-1

-2

Outro exemplo: equações de Bernoulli a coeficientes constantes, como:


y ′ (x) = a · y(x) − b · y(x)2
são separáveis. É desse ponto de vista que as trataremos na Seção 4 do Capı́tulo 38.

5. A clepsidra
Considero aqui um exemplo de equação separável associado ao escomanto de um
lı́quido.
Imagine um recipiente em formato de superfı́cie de revolução em torno do eixo
dos y de um gráfico
x = f (y), y ∈ [0, y(0)]
onde y(0) é a altura do lı́quido que preenche o recipiente.
A chamada Lei de Torricelli diz que a velocidade com que o lı́quido sai pela base
do recipiente é proporcional à altura do lı́quido, da forma:
p u.m.
2g · y(t) .
t
onde g é a constante de aceleração gravitacional e u.m. é unidade de comprimento.
Se a abertura ba base tem área de A u.m.2 então a queda do volume V (t) do
lı́quido é de
dV p u.m.3
= −A · 2g · y(t) .
dt t
Seja V (y) o volume do lı́quido quando a altura é y. Esse é o volume do sólido de
revolução calculado integrando as fatias circulares horizontais:
Z y
V (y) = π · f (u)2 du.
0
Então pela regra da derivada da composta e pelo teorema fundamental:
dV dV dy
= · =
dt dy dt
6. EQUAÇÕES HOMOGÊNEAS 528

= π · f (y)2 · y ′ (t).
Então a altura em cada instante do lı́quido satisfaz a seguinte equação separável:

′ −A · 2g y
y (t) = .
π · f (y)2
Suponha agora que

x = f (y) = 4 y ou seja y = x4 .
Então a equação anterior vira:

′ A· 2g
y (t) ≡ − ,
π
que é constante.
Tomando
π
A= √ ,
A· 2g
temos
y(t) = y(0) − t
e portanto a altura y(t) serve como relógio para marcar o tempo ! Esses relógios de
água se chamam clepsidras.
6. Equações homogêneas
As equações
y ′(x) = F (x, y)
em que a função F tem a propriedade
F (x, y) = F (t · x, t · y), ∀t
são chamadas de4 homogêneas de grau 0.
Essas equações são resolvidas associando-se a elas uma equação separável.
Isso se faz do seguinte modo: tomando o t particular t = x1 posso dizer então que:
1 1 y
y ′(x) = F (x, y) = F ( · x, · y) = F (1, ) =: F (1, u),
x x x
chamando u := xy .
Temos u(x) = y(x) x
, ou seja,
u(x) · x = y(x)
e derivando:
u′ (x) · x + u(x) = y ′ (x) = F (1, u).
O que produz a equação separável nas variáveis u e x:
F (u) − u(x)
u′ (x) = .
x
Essas já sabemos resolver !

Um Exemplo que me pareceu interessante.


4Em geral diz-se que F (x, y) é homogênea de grau d se F (t · x, ·y) = td · F (x, y).
CAPÍTULO 36. ASPECTOS GERAIS DAS EQUAÇÕES DE PRIMEIRA
ORDEM 529

No Exercı́cio 10.8 - Capı́tulo 11 (resolvido) dávamos (A, B) no primeiro quadrante


e uma reta y = ax (com 0 < aA < B). Perguntamos qual a reta por (A, B) que
formava um triângulo de menor área com o eixo dos y > 0. A figura ilustra o
problema:
y
y=ax

(A,B)

Na resolução vimos que o coeficiente angular da reta apropriada é:


2Aa − B
λ= .
A
Agora posso perguntar: qual gráfico y = f (x) contendo (A, B) tem a propriedade de
que:
2xa − y
f ′ (x) =
x
e portanto tem retas tangentes que formam em cada ponto triângulos de menor área
com o eixo y > 0 e a reta y = ax.
Ora, essa equação diferencial é homogênea. Portanto recai na equação separável:
2a − u(x) − u(x) 2a − 2 · u(x) y
u′ (x) = = , u(x) := ,
x x x
ou seja,
1 u′(x) 1
· =− .
2 u(x) − a x
y
Notando que u − a = x
− a > 0 para que se formem realmente triângulos obtemos:
1
· ln(u(x) − a) = − ln(x) + C,
2
B
onde a constante C fica determinanda pela condição B = y(A), ou seja u(A) = A
.
Toemando exponencial e elevando ao quadrado obtenho:
(B
A
− a) 1
u(x) = · 2 + a,
A2 x
ou seja:
(BA
− a) 1
y= · + a · x.
A2 x
Há equações que apesar de não serem homogêneas de grau 0 podem ser transfor-
madas em equações homogêneas de grau 0, após mudança linear de coordenadas.
7. EQUAÇÕES EXATAS 530

Por Exemplo:
ax + by + c
y ′(x) = , com x 6= 0 ea · e − d · b 6= 0.
dx + ey + f
Se c = f = 0 já estamos num caso de equação homogênea de grau 0, pois:
at · x + bt · y ax + by a + b · xy
= = .
dt · x + et · y dx + ey d + e · xy
Se c 6= 0 ou f 6= 0 faço as mudanças de coordenadas:
v =y−β e u=x−α
onde ainda resta escolher quais serão os números α, β, mas pelo menos já temos:
dv dy
= ,
du dx
pois pela regra da composta escrita na notação de Leibniz:
dv dv dy dx dy
= · · =1· · 1.
du dy dx du dx
Ou seja,
dv ax + by + c a · (u + α) + b · (v + β) + c
= = =
du dx + ey + f d · (u + α) + e · (v + β) + f
au + bv + c + a · α + b · β
=
du + ev + f + d · α + e · β
e aı́ vemos que precisamos escolher α, β para que tenhamos:
c + a · α + b · β = 0 e f + d · α + e · β = 0,
ou seja, precisamos resolver o sistema linear não homogêneo (já que c 6= 0 ou f 6= 0):
a · α + b · β = −c

d · α + e · β = −f
Pela regra de Cramer tudo que precisamos é a condição: a · e − d · b 6= 0.
Com as soluções α, β desse sistema conseguimos uma equação homogênea, que já
sabemos resolver.

7. Equações exatas
As equações separáveis e algumas outras equações diferenciais que vimos recaem
em situações do tipo:
d U(x, y(x))
=C
dx
e daı́ as resolvemos como U(x, y(x)) = C · x + D.
CAPÍTULO 36. ASPECTOS GERAIS DAS EQUAÇÕES DE PRIMEIRA
ORDEM 531

Definição 7.1. Uma equação y ′ (x) = F (x, y) é exata se pode ser escrita como:
F1 (x, y) · y ′(x) + F2 (x, y) = C
onde F1 (x, y), F2(x, y) são contı́nuas em U e verificam
d U(x, y(x))
F1 (x, y) · y ′(x) + F2 (x, y) =
dx
para alguma função U(x, y) definida em U, cujas derivadas parciais de primeira e
segunda ordem são contı́nuas.
Afirmação 7.1. Seja a equação
F1 (x, y) · y ′(x) + F2 (x, y) = C
com (x, y) numa região U do plano.

i) se é uma equação exata então:


∂F1 (x, y) ∂F2 (x, y)
= .
∂x ∂y
ii) em U = R2 \ {(0, 0)} a equação
x y
2 2
· y ′ (x) − 2 =0
x +y x + y2
verifica
x y
∂( x2 +y 2) ∂(− x2 +y 2)
= .
∂x ∂y
mas no entanto não é exata.
iii) se [a, b] × [c, d] é um retângulo fechado está contido em U, então a condição
∂F1 (x, y) ∂F2 (x, y)
=
∂x ∂y
em U é suficiente para que F1 (x, y)·y ′(x)+F2 (x, y) = C seja exata. Ademais, podemos
tomar Z x Z y
U(x, y) := F2 (t, c) dt + F1 (x, t) dt
a c
d U (x,y(x))
para que dx
= F1 (x, y) · y ′(x) + F2 (x, y).
Demonstração.

De i):
Se existe uma função U(x, y) para a qual na região U:
d U(x, y(x))
F1 (x, y) · y ′ (x) + F2 (x, y) = ,
dx
então isso quer dizer pela regra da composta que:
∂U(x, y(x)) ∂U(x, y(x))
= F1 (x, y) e = F2 (x, y).
∂y ∂x
7. EQUAÇÕES EXATAS 532

Como as derivadas parciais de primeira e segunda ordem de U(x, y) são supostas


contı́nuas, podemos usar o Lema de Schwartz, que garante que as derivadas parciais
de segunda ordem não dependem da ordem em que derivamos, ou seja:
∂ 2 U(x, y) ∂ 2 U(x, y)
= .
∂x ∂y ∂y ∂x
Portanto:
∂F1 (x, y) ∂F2 (x, y)
= .
∂x ∂y
De ii):
Não poderei dar todos os detalhes desta prova, que exigiria mais técnica, mas
posso dar uma boa idéia de por quê essa equação não é exata.
Temos que U = R2 \ {(0, 0)} é o plano menos a origem. Nesse U é que vamos
considerar a equação:
x y
2 2
· y ′(x) − 2 = 0.
x +y x + y2
Note que
∂F1 (x, y) 1 · (x2 + y 2) − x · (2x) −x2 + y 2
= = ,
∂x (x2 + y 2 )2 (x2 + y 2)2
∂F2 (x, y) (−1) · (x2 + y 2 ) + y · (2y) −x2 + y 2
= = .
∂y (x2 + y 2 )2 (x2 + y 2 )2
Considere um ponto P = (x, y) de U e escolha dentre os possı́veis valores θ +k ·2π,
k ∈ Z um θ(x, y) para medir o ângulo anti-horário que P = (x, y) forma com o eixo
x > 0.
Temos
y
sin(θ(x, y)) = p
x2 + y 2
e se supomos que θ(x, y) é uma função derivável numa pequena região em torno de
P , teremos pela regra da composta:
∂θ(x, y) ∂ sin(θ(x, y))
cos(θ(x, y)) · = =
∂y ∂y
y
∂( √ ))
x2 +y 2 x2
= = 3 .
∂y (x2 + y 2 ) 2
Como
x
cos(θ(x, y)) = p ,
x2 + y 2
obtemos
∂θ(x, y) x
= 2 .
∂y x + y2
De modo completamente análogo obteremos:
∂θ(x, y) −y
= 2 .
∂x x + y2
CAPÍTULO 36. ASPECTOS GERAIS DAS EQUAÇÕES DE PRIMEIRA
ORDEM 533

Ou seja, que a função U(x, y) definida em U que buscamos (contı́nua, derivável, etc)
seria essencialmente uma estensão dessa θ(x, y) a toda a regio U.
Mas se pode mostrar que essa estensão é impossı́vel, pelo fato de U ser uma região
em torno da origem: pense em um cı́rculo em torno da origem, como poderı́amos
medir ângulos quando damos voltas nesse cı́rculo ? Isso levaria a mais de um valor
de ângulo para cada ponto (θ + k · 2π, k ∈ Z) e portanto U(x, y) = θ(x, y) não seria
uma verdadeira função bem definida,

De iii):
A expressão
Z x Z y
U(x, y) := F2 (t, c) dt + F1 (x, t) dt
a c
faz sentido no retângulo [a, b] × [c, d] e cada integral existe pois F1 e F2 são funções
contı́nuas. R
x
Como a F2 (t, c) dt não depende de y,
Rx
∂( a F2 (t, c) dt)
= 0.
∂y
Pelo Primeiro Teorema Fundamental:
Ry
∂( c F1 (x, t) dt)
= F1 (x, y).
∂y
Portanto
∂U(x, y)
= F1 (x, y).
∂y
Queremos agora derivar U(x, y) em x e em y. Para isso algumas observações são
importantes.
Usando o Primeiro Teorema Fundamental sabemos que
Rx
∂( a F2 (t, c) dt)
= F2 (x, c).
∂x
Ry
Mas como derivar c F1 (x, t) dt em relação a x ? Ry
Note que x funciona como um parâmetro para as diferentes integrais c F1 (x, t) dt,
ou seja, há uma aplicação:
Z y
x ∈ [a, b] 7→ F1 (x, t) dt
c

e não está claro como derivá-la em x.


Explicaremos na Seção 9 que, nas condições em que estamos, podemos afirmar:
Ry Z y
∂( c F1 (x, t) dt) ∂F1 (x, t)
= dt,
∂x c ∂x
ou seja, que a derivada passa sob o sinal da integral.
8. INTEGRAL AO LONGO DE UM CAMINHO 534

Tendo isso, veja agora o que se obtêm usando a hipótese


∂F1 (x, y) ∂F2 (x, y)
=
∂x ∂y
e o Primeiro Teorema Fundamental:
Z y
∂U(x, y) ∂F1 (x, t)
= F2 (x, c) + dt =
∂x c ∂x
Z y
∂F2 (x, t)
= F2 (x, c) + dt =
c ∂y
= F2 (x, c) + [F2 (x, y) − F2 (x, c)] =

= F2 (x, y)
como querı́amos.


8. Integral ao longo de um caminho


Seja Γ(t) = (x(t), y(t)), com t ∈ [A, B] uma curva parametrizada e derivável, no
mesmo sentido do Capı́tulo 28.
Então defino a integral ao longo da curva Γ por
Z Z B
F1 (x, y)dy + F2 (x, y)dx := [F1 (x(t), y(t)) · y ′(t) + F2 (x(t), y(t)) · x′ (t)] dt.
Γ A

Se Γ é uma união de um número finito de curvas deriváveis então defino a integral


ao longo de Γ como soma de integrais.
Afirmo que a integral
Z x Z y
F2 (t, c) dt + F1 (x, t) dt
a c

que aparece no item iii) da Afirmação 7.1 é uma integral ao longo de uma linha
quebrada Γ.
De fato, fixado o ponto (x, y), então Γ pode ser parametrizada por
t ∈ [a, x] ∪ [c, y]
da seguinte forma:
Γ(t) = (t , c ), se t ∈ [a, x]

Γ(t) = ( x , t ), se t ∈ [c, y]
Confira que Γ(a) = (a, c), Γ(x) = (x, c) = Γ(c) e Γ(y) = (x, y).
A figura ilustra essa linha quebrada:
CAPÍTULO 36. ASPECTOS GERAIS DAS EQUAÇÕES DE PRIMEIRA
ORDEM 535

(x,y)

(a,c) (x,c)

Então nessa linha quebrada:


Z
F1 (x, y)dy + F2 (x, y)dx :=
Γ

Z x
:= [F1 (x(t), y(t)) · y ′ (t) + F2 (x(t), y(t)) · x′ (t)] dt+
a

Z y
+ [F1 (x(t), y(t)) · y ′ (t) + F2 (x(t), y(t)) · x′ (t)] dt =
c

Z x Z y
= F2 (t, c) dt + F1 (x, t) dt,
a c

como afirmamos.

A Afirmação a seguir complementa o item iii) da Afirmação 7.1:

Afirmação 8.1. Suponha que U é uma região do plano com a propriedade de que
quaisquer dois de seus pontos possam ser ligados por alguma curva parametrizada
derivável.
Se a equação
F1 (x, y) · y ′(x) + F2 (x, y) = C

com (x, y) numa região U do plano é uma equação exata então


Z
F1 (x, y)dy + F2 (x, y)dx
Γ

independe da curva parametrizada Γ ⊂ U que liga (a, c) a (x, y). Ou seja, depende
apenas dos pontos iniciais e finais.
9. DERIVADA DA INTEGRAL EM RELAÇÃO AO PARÂMETRO -
FÓRMULAS DE LEIBNIZ 536

(x,y)

(a,c) (x,c)

Figura: A linha quebrada de antes e outra curva ligando (a, c) a (x, y).

Demonstração.

Z Z B
F1 (x, y)dy + F2 (x, y)dx := [F1 (x(t), y(t)) · y ′(t) + F2 (x(t), y(t)) · x′ (t)] dt =
Γ A
Z B
∂U(x(t), y(t)) ′ ∂U(x(t), y(t)) ′
= [ · y (t) + · x (t)] dt =
A ∂y ∂x
Z B
d U(x(t), y(x(t)))
= dt =
A dt
= U(B) − U(A),
onde após a definição, usamos que a equação é exata, depois a regra da derivada da
composta5, e por último usamos o Teorema Fundamental do Cálculo.


9. Derivada da integral em relação ao parâmetro - Fórmulas de Leibniz


Rb
Afirmação 9.1. Seja F (x) := a f (t, x) dt uma integral dependendo de um parâmetro
x ∈ [c, d] (intervalo fechado), onde os limites de integração a, b não dependem de x.
Suponha que existe ∂f ∂x
e que a função
∂f
: [a, b] × [c, d] → R
∂x
seja contı́nua (ver Def. 15.1).
Então:
Rb Z b
∂F ∂ a f (t, x) dt ∂f (t, x)
= = dt.
∂x ∂x a ∂x
5Para funções de duas variáveis
CAPÍTULO 36. ASPECTOS GERAIS DAS EQUAÇÕES DE PRIMEIRA
ORDEM 537

Demonstração.
Queremos provar que para cada x:
Z b
∂F ∂f (t, x)
(x) = (x) dt.
∂x a ∂x
Ou seja, queremos ver se
Z b
∂f (t, x) F (x + h) − F (x)
(x) dt = lim :=
a ∂x h→0 h
Rb Rb
a
f (t, x + h) dt − a f (t, x) dt
:= lim .
h→0 h
Para cada h posso escrever:
Rb Rb Z b
a
f (t, x + h) dt − a f (t, x) dt f (t, x + h) − f (t, x)
= dt
h a h
O que queremos saber é, finalmente, se dado ǫ > 0 existe δ (dependendo de ǫ e de x
possivelmente) tais que:
Z b Z b
f (t, x + h) − f (t, x) ∂f (t, x)
|h| < δ ⇒ | dt − (x) dt | < ǫ.
a h a ∂x
Vejamos como determinar esse δ. Temos
Z b Z b
f (t, x + h) − f (t, x) ∂f (t, x)
| dt − (x) dt | =
a h a ∂x
Z b
f (t, x + h) − f (t, x) ∂f (t, x)
=| ( − (x)) dt | ≤
a h ∂x
Z b
f (t, x + h) − f (t, x) ∂f (t, x)
≤ | − (x)| dt.
a h ∂x
O Teorema do Valor Médio de Lagrange no6 intervalo [x, x + h] dá que:
f (t, x + h) − f (t, x) ∂f (t, x)
= (x + τ · h), para algum 0 < τ < 1.
h ∂x
Portanto:
Z b Z b
f (t, x + h) − f (t, x) ∂f (t, x) ∂f (t, x) ∂f (t, x)
| − (x)| dt = | (x + τ · h) − (x)| dt.
a h ∂x a ∂x ∂x
Por hipótese
∂f (t, x)
: [a, b] × [c, d] → R
∂x
é contı́nua e
||(t, x + τ · h) − (t, x)|| ≤ |h|.
Portanto pela Afirmação 15.1 existe δ tal que
∂f (t, x) ∂f (t, x) ǫ
|h| < δ ⇒ | (x + τ · h) − (x)| <
∂x ∂x b−a
6para simplificar a exposição, me restrinjo a considerar h > 0, mas o caso h < 0 é análogo.
9. DERIVADA DA INTEGRAL EM RELAÇÃO AO PARÂMETRO -
FÓRMULAS DE LEIBNIZ 538

e portanto Z b
∂f (t, x) ∂f (t, x)
|h| < δ ⇒ | (x + τ · h) − (x)| dt < ǫ
a ∂x ∂x
como querı́amos.


Exemplo:

Seja: Z 1
x·t ex·t ex·t ex 1
F (x) := e dt = (1) − (0) = −
0 x x x x
e portanto
ex ex 1
F ′ (x) = − 2 + 2.
x x x
Por outro lado, Z Z 1
1
∂ ex·t
dt = ex·t · t dt
0 ∂x 0
e integrando por partes se obtêm:
Z 1 Z 1 x·t
x·t ex·t ex·t e
e · t dt = ( · t)(1) − ( · t)(0) − · 1 dt =
0 x x 0 x
ex ex 1
= − 2 + 2.
x x x
A Afirmação anterior 9.1 admite uma versão mais geral, que menciono agora, mas
que ainda não provo:
R b(x)
Afirmação 9.2. Seja F (x) := a(x) f (t, x) dt uma integral dependendo de um parâmetro
x ∈ [c, d] (intervalo fechado), onde os limites de integração a(x) e b(x) são funções
deriváveis de x.
Suponha que existe ∂f∂x
e que a função
∂f
: [a, b] × [c, d] → R
∂x
seja contı́nua (ver Def. 15.1).
Então:
Z b(x)
∂F db(x) da(x) ∂f (t, x)
= · f (t, x)|t=b(x) − · f (t, x)|t=a(x) + dt.
∂x dx dx a(x) ∂x

Por exemplo, se Z x
F (x) = et−x · t dt,
0
então, pondo a(x) ≡ 0 e b(x) = x, teremos pela Afirmação 9.2:
Z x
′ t−x t−x
F (x) = 1 · (e · t)t=x − 0 · (e · t)t=0 + (−et−x · t) dt =
0
CAPÍTULO 36. ASPECTOS GERAIS DAS EQUAÇÕES DE PRIMEIRA
ORDEM 539
Z x
= x− et−x · t dt.
0
Mas neste exemplo simples também se pode fazer a conta diretamente, pois:
Z x Z x
t−x −x
F (x) = e · t dt = e · et · t dt
0 0
de onde, pela regra do produto e pelo Teorema Fundamental:
Z x Z x
′ −x t −x x
F (x) = −e · e · t dt + e · e · x = x − et−x · t dt.
0 0

10. Fatores integrantes


A equação
x2 · y ′ (x) + (1 − x2 ) · y 2
não é exata, já que
∂x2 ∂((1 − x2 ) · y 2)
6= .
∂x ∂y
(item i) da Afirmação 7.1).
Mas se multiplico a equação toda por:
1
µ(x, y) := 2 2 , x · y 6= 0,
x ·y
então a nova equação:
1 1
2
· y ′(x) + 2 − 1 = 0
y x
verifica
∂( y12 ) ∂( 12 − 1)
≡0≡ x .
∂x ∂y
Logo o item iii) da Afirmação 7.1 me diz que essencialmente o que tenho que fazer
é definir: Z x Z y
1 1 1 1
U(x, y) = 2
− 1 dt + 2
dt = x − − + C1
a t c t x y
e que a solução geral é:
1 1
−x − − = C.
x y
Para reforçar isso, note que se U(x, y(x)) ≡ C, então
dU(x, y(x))
0= = µ(x, y) · [x2 · y ′ (x) + (1 − x2 ) · y 2 ],
dx
e como µ(x, y) 6≡ 0, então
U(x, y(x)) ≡ C
são as soluções de x2 · y ′ (x) + (1 − x2 ) · y 2 ≡ 0
Pondo y = y(x) temos
1 x −x
y= 1 = 2
= .
−C − x − x −C · x − x − 1 C · x + x2 + 1
10. FATORES INTEGRANTES 540

A solução y ≡ 0 de x2 · y ′ (x) + (1 − x2 ) · y 2 = 0 se perdeu no caminho, pois quando


usei µ(x, y) supus que y 6= 0. Por isso adjunto às soluções
−x
y=
C · x + x2 + 1
a solução y = 0.
O campo de direções para
1 1
2
· y ′(x) + 2 − 1 = 0
y x
é esboçado na Figura a seguir, com x ∈ [0.5, 5] e y = [−0.5, 0.5]

0,4

0,2

y(x) 0
1 2 3 4 5
x

-0,2

-0,4

Algumas curvas integrais


−x
y=
C · x + x2 + 1
são esboçadas na Figura a seguir, para x ∈ [0.5, 5]:

x
1 2 3 4 5
0

-0,1

-0,2

-0,3

-0,4

-0,5
CAPÍTULO 36. ASPECTOS GERAIS DAS EQUAÇÕES DE PRIMEIRA
ORDEM 541

Em geral achar um fator ntegrante µ(x, y) de um tipo bem geral é um problema


difı́cil, pois temos de resolver equações a derivadas parciais para encontrá-lo.
A tentativa mais otimista é buscar fatores integrantes que só dependam de uma
variável, ou seja µ = µ(x) ou µ = µ(y).
Se não der, buscar do tipo µ(x, y) = xa · y b, onde os valores corretos de a, b se
descobrem ao impôr-se:
∂xa · y b · F2 (x, y) ∂xa · y b · F1 (x, y)
= ,
∂x ∂y
o que produz um sistema de equações em a, b.

Exemplo:
Considero a equação:
n √
· x · y ′(x) + n x + y = 0, n ∈ N, n ≥ 2
n−1
para x 6= 0 e ademais x > 0 se n é par.
Essa equação não é exata. Multiplico-a por µ(x):
n √
· x · µ(x) · y ′ (x) + µ(x) · ( n x + y) = 0.
n−1
e quero ter:
n n
µ′ (x) · · x + µ(x) · = µ(x),
n−1 n−1
ou seja, para µ(x) 6= 0:
µ′ (x) 1 1
=− · .
µ(x) n x
Integrando e tomando exponencial obtenho:
1
−n 1
µ(x) = eln(x )
= x− n .
1
Então multiplicada por µ(x) = x− n a equação vira a nova equação exata:
n n−1 −1
· x n · y ′ (x) + 1 + x n · y = 0, n ∈ N, n ≥ 2
n−1
cuja solução geral é
Z x Z y
−n1 n n−1
U(x, y) = (1 + t · c) dt + · x n dt =
a c n−1
n n−1 n n−1 n n−1
= x+ · x n · c − C1 + ·x n ·y− ·x n ·c=
n−1 n−1 n−1
n n−1
= x+ · x n · y − C1 ,
n−1
ou seja, as soluções são:
n n−1
x+ · x n · y = C1 .
n−1
O Exercı́cio 16.1 no final do Capı́tulo consiste em encontrar fator integrante.
11. EQUAÇÕES IMPLÍCITAS, DISCRIMINANTES E ENVELOPES 542

10.1. Fatores integrantes de equações lineares. Aqui quero lembrar que,


no caso de equações diferenciais lineares, já tratamos de seus fatores integrantes na
Seção 9. Mas podemos retomar o que fizemos lá à luz desta teoria mais geral7.
Escrevo a equação linear como:
y ′ − a(x)y − b(x) = N · y ′ + M = 0
e busco µ(x) tal que:
∂[µ(x) · 1] ∂[µ(x) · (−a(x)y − b(x))]
= = −µ(x)a(x),
∂x ∂y
ou seja,
µ′ (x) = −a(x)µ(x).
R
−a(x)dx
Tomo µ(x) = e . Portanto
Z Z R R
U(x, y) = µ(x) dy = e −a(x)dx dy = e −a(x)dx · y + h(x)
e
∂U(x, y) R
= −a(x) · e −a(x)dx · y + h′ (x) =
∂x R
= µ(x) · (−a(x)y − b(x)) = e −a(x)dx · (−a(x)y − b(x))
ou seja, R
h′ (x) = −b(x) · e −a(x)dx

e Z R
−a(x)dx
h(x) = − b(x) · e dx + C.
Portanto Z
R R
−a(x)dx −a(x)dx
U(x, y) = e ·y− b(x) · e dx ≡ C,
que também dá: Z
R R
a(x)dx
y=e · [ b(x) · e −a(x)dx dx + C].

11. Equações implı́citas, discriminantes e envelopes


Nas Seções anteriores, para cada ponto de uma região U do plano está associado
um valor de y ′ (x) através da expressão:
y ′ (x) = F (x, y).
A situação que trataremos agora é diferente, pois nela haverá pontos do plano (x, y)
que não têm y ′(x) associada, outros que têm um valor bem definido e outros ainda
têm dois valores possı́veis !
O Exemplo para começar é:
(y ′ )2 − 4x · y ′ + 4y = 0,
na qual y ′ figura implicitamente.
7Agradeço ao estudante Luciano B. Barros por esta questão.
CAPÍTULO 36. ASPECTOS GERAIS DAS EQUAÇÕES DE PRIMEIRA
ORDEM 543

Se pensamos nessa equação diferencial como uma equação quadrática usual na


variável y ′ , então ela tem um discriminante:
∆ := 16x2 − 4 · 1 · (4y) = 16x2 − 16y,
ou seja, se num ponto (x, y) do plano ∆ < 0 , não há y ′ associado; se ∆ = 0 há
exatamente 1 valor y ′ associado e se ∆ > 0, então há duas possibilidades de y ′.
Note que ∆ = 0 equivale a termos y = x2 , ou seja, são pontos de uma parábola.
Que famı́lia de curvas satifaz essa equação diferencial implı́cita (y ′ )2 −4x·y ′ +4y = 0
? A famı́lia de retas tangentes à parabola y = x2 , que vem a ser a famı́lia de retas:
y = 2c · x − c2 .
Note que y ′ (x) = 2c e portanto:
y′
y = y ′ · x − ( )2 ,
2
de onde sai:
(y ′ )2 − 4x · y ′ + 4y = 0.

0,5
x
-1 -0,5 0 0,5 1
0

-0,5

-1

-1,5

-2

-2,5

Outro modo de se obter a parábola y = x2 desse Exemplo é eliminando-se c nas


duas equações:
∂(y − 2c · x + c2 )
y − 2c · x + c2 = 0 e = −2x + 2c = 0,
∂c
pois a segunda dá c = x, que quando posto na primeira dá: y − 2x2 + x2 = 0, ou seja
y = x2 .
É esse o processo de eliminação do parâmetro c retomado na Definição a seguir:
Definição 11.1. Considere uma famı́lia de curvas com equações F (x, y, c) = 0 de-
pendendo de um parâmetro c e que tenha ∂F∂c
.
A curva g(x, y) = 0 obtida por eliminação de c nas equações:
∂F (x, y, c)
F (x, y, c) = = 0
∂c
é o envelope da famı́lia de curvas dada.
11. EQUAÇÕES IMPLÍCITAS, DISCRIMINANTES E ENVELOPES 544

Exemplo: Considere agora a famı́lia de retas ortogonais à parábola y = x2 em


pontos diferentes da origem, ou seja:
−1 1
y= · x + c2 + , c 6= 0
2c 2
que pode ser reeescrita (multiplicando por 2c) como:
2c3 + c − x − 2c · y = 0
Nesse caso,
∂F (x, y, c)
= 6c2 + 1 − 2y
∂c
e o envelope da famı́lia surge de se eliminar c do seguinte modo (penso em c > 0):
r
2y − 1
c= , 2y − 1 > 0,
6
r r r
2y − 1 3 2y − 1 2y − 1
2·( ) + −x−2 ·y =0
6 6 6
ou seja:
r
2y − 1 2y − 1
·(2· + 1 − 2y ) − x = 0,
6 6
ou seja:
r
2y − 1 2
· (− · (2y − 1) ) = x
6 3
e
2 3
− √ · (2y − 1) 2 = x
3 6
ou seja:
2
(2y − 1)3 = x2 .
27
Isso pode ser escrito como
2 · (1 − 2y)3 + 27 · x2 = 0
ou dividindo por 4:
1 − 2y 3 x
∆ := 4 · ( ) + 27 · ( )2 = 0
2 2
e veremos no Capı́tulo 32 que ∆ é o discriminante da equação cúbica na variável c:
1 − 2y x
c3 + c · ( ) − = 0 ⇐⇒ 2c3 + c − x − 2c · y = 0,
2 2
onde (x, y) devem ser pensados como coeficientes.
A Figura a seguir ilustra o envelope 2 · (1 − 2y)3 + 27 · x2 = 0 da famı́lia de retas
ortogonais à parábola.
CAPÍTULO 36. ASPECTOS GERAIS DAS EQUAÇÕES DE PRIMEIRA
ORDEM 545

1,5

y 1

0,5

0
-1 -0,5 0 0,5 1
x

Exemplo: A parábola de segurança 8

Vimos na Afirmação 8.1 do Capı́tulo 35 que as trajetórias parabólicas de um


projétil, que parte com velocidade escalar v0 e ângulo 0 < α <
f racpi2 comv a horizontal, descrevem parábolas
g
y=− 2
· x2 + tan(θ) · x.
2 · v0 · cos2 (θ)
O envelope dessa famı́lia serve para determinar a região além da qual nenhum ar-
remesso pode passar.
Afirmo que esse envelope é a seguinte curva:
(v0 )2 g
y= − · x2
2g 2(v0 )2
que também é uma parábola.
Para obter a curva envelope derivo a famı́lia
g
H(x, y, θ) := y + 2
· x2 − tan(θ) · x = 0
2 · v0 · cos2 (θ)
em relação a θ obtendo:
g · sin(θ)
− + sec2 (θ) · x = 0
v02 · cos3 (θ)
Então:
g · tan(θ) · sec2 (θ)
− = − sec2 (θ) · x
v02
e portanto
v02
tan(θ) · x =
g
8Sugerido por Fábio Casula
11. EQUAÇÕES IMPLÍCITAS, DISCRIMINANTES E ENVELOPES 546

Substituindo esta expressão na famı́lia


g
H(x, y, θ) = y + · (1 + tan2 (θ)) · x2 − tan(θ) · x = 0
2 · v02
obtemos a parábola envelope.
A Figura a seguir mostra para v0 = 1 e g = 10 algumas trajetórias parabólicas.
1
Em vermelho a de alcance máximo x = 10 , para a = π4 . Em azul, duas com a = π4 +0.2
π
e a = 4 − 0.2, que atingem o mesmo ponto. Em verde, a parábola de segurança.

0,05

0,04

0,03

0,02

0,01

0
0 0,02 0,04 0,06 0,08 0,1
x

Após termos desenvolvido melhor a noção de discriminante, veremos no Capı́tulo


33 que há uma via de duas mãos entre envelopes de famı́lias de retas e discriminantes
de polinômios.

Vimos na seção 3 do Capı́tulo 15 que a reta tangente à curva F (x, y) = 0 no ponto


(x, y) é dada por:

∂F (x, y) ∂F (x, y)
· (x − x) + · (y − y) = 0.
∂x ∂y

Da definição de vetor tangente Γ′ (t) = (x′ (t), y ′(t)) a uma curva parametrizada
Γ dada na Seção 3 do Capı́tulo 28 e das explicações que demos lá, segue que Γ é
tangente a F (x, y) = 0 quando:

∂F (x(t), y(t)) ′ ∂F (x(t), y(t)) ′


· x (t) + · y (t) = 0.
∂x ∂y
CAPÍTULO 36. ASPECTOS GERAIS DAS EQUAÇÕES DE PRIMEIRA
ORDEM 547

Diremos que uma curva F (x, y) = 0 é não-singular se em cada ponto da curva es-
tiver definida sua reta tangente. Portanto isso equivale a que não aconteça a anulação
simultânea de ∂F∂x
(x,y)
e de ∂F∂y
(x,y)
em nenhum ponto da curva F (x, y) = 0.
Afirmação 11.1. Seja F (x, y, c) = 0 uma famı́lia de curvas com um parâmetro
c ∈ J, onde J é um intervalo. Suponha que para cada c a curva F (x, y, c) = 0 é
não-singular. Suponha que, ademais das derivadas ∂F (x,y,c)
∂x
e ∂F (x,y,c)
∂y
, esteja também
∂F (x,y,c)
definida a derivada ∂c
. Seja
Γ : I → R2 , Γ(t) = (x(t), y(t))
uma curva parametrizada, derivável, onde I é intervalo.
Suponha que para parâmetro c exista um valor bem determinado de t, chamado
de t(c), tal que Γ é tangente à curva F (x, y, c) = 0 no ponto Γ(t(c)). E suponha que
essa função t = t(c) seja derivável.
Então Γ está contida no envelope da famı́lia F (x, y, c) = 0.
Demonstração.
Como Γ(t(c)) é tangente à curva F (x, y, c) = 0 no ponto
Γ(t(c)) = (x(t(c)), y(t(c))) = (x(c), y(c)),
em particular temos:
F (x(c), y(c), c) ≡ 0, ∀c ∈ J.
Como t = t(c), x(t) e y(t) são deriváveis, então por composição x(t(c)) = x(c) e
y(t(c)) = y(c) também o são. Chamando
φ(c) = F (x(c), y(c), c) ≡ 0
obtemos derivando-a9:
0 ≡ φ′ (c) =
∂F (x(c), y(c), c) ′ ∂F (x(c), y(c), c) ′ ∂F (x(c), y(c), c)
= · x (c) + · y (c) + .
∂x ∂y ∂c
Segue do que vimos na seção 3 do Capı́tulo 15 que o fato de Γ ser tangente à
famı́lia em F (x, y, c) = 0 se escreve, para cada c, como:
∂F (x(c), y(c), c) ′ ∂F (x(c), y(c), c) ′
· x (c) + · y (c) ≡ 0.
∂x ∂y
Concluı́mos de 0 ≡ φ′ (c) que:
∂F (x(c), y(c), c)
0≡ .
∂c
Ou seja que Γ está contida na curva envelope, pois essa está definido por:
∂F (x, y, c)
F (x, y, c) = = 0.
∂c


9E usando uma versão da regra da composta para funções de mais de uma variável
12. UM PROBLEMA DA PUTNAM COMPETITION, N. 5, 1942 548

12. Um problema da Putnam Competition, n. 5, 1942

Problema: Considere a famı́lia de parábolas com um parâmetro c:


c3 2 a2
y= ·x + · x − 2c.
3 2
i) determine o lugar geométrico dos vértices.
ii) determine o envelope da famı́lia
iii) esboce o envelope e dois elementos tı́picos da famı́lia.
Solução:
De i): para encontrar o lugar geométrico dos vértices, farei primeiro a suposição
adicional de que
c>0
e depois discutirei o que acontece para c < 0.
Com c > 0 posso escrever:
c3 2 c2
y= · x + · x − 2c =
3 2
√ √
c3 3 √ 2 3
=( √ ·x+ · c ) − 2c − 2 · c =
3 4 4
√ √
c3 3 2 35
=( √ ·x+ ) − · c,
3 4 16
ou seja: √ √
35 c3 3 2
y+ ·c=( √ ·x+ ).
16 3 4
Então os vértices das parábolas são os pontos:
3 1 35
(x, y) = (− · , − · c).
4 c 16
Esses pontos satisfazem:
3 35
x·y = ·
4 16
e isso é uma hipérbole. O ramo dessa hipérbole que tem x < 0 e y < 0 descreve o
3 2
lugar dos vértices de y = c3 · x2 + c2 · x − 2c para c > 0, já que todas elas cortam o
eixo dos y em pontos de coordenadas negativas.
Já o ramo da hipérbole com x > 0 e y > 0 descreve os vértices das parábolas
3 2
y = c3 · x2 + c2 · x − 2c para c < 0.

De ii): O envelope satisfaz:


c3 2 c2
y= · x + · x − 2c e 0 = c2 · x2 + c · x − 2.
3 2
Suponha por um momento que c > 0 e que x > 0 e resolva
c2 · x2 + c · x − 2 = 0
CAPÍTULO 36. ASPECTOS GERAIS DAS EQUAÇÕES DE PRIMEIRA
ORDEM 549

como equação quadrática onde c é a variável e x é fixado. Então:


p
−x + x4 − 4 · x2 · (−2) 2x 1
c= 2
= 2 = ,
2x 2x x
1
e note que c = x
é solução de
c2 · x2 + c · x − 2 = 0
também para x < 0.
1 c3 c2
Substituindo c = x
em y = 3
· x2 + 2
· x − 2c e simplificando obtemos:
7 1
y=− · ,
6 x
que vem a ser o envelope ∆ = 0.
De iii): considerando c = 1 e c = −1 por exemplo o aspecto tı́pico é esboçado
na Figura a seguir, onde em verde está lugar dos vértices V e em vermelho o envelope
da famı́lia de cônicas:
y

c>0

V

c<0

Consegui depois fazer no Maple uma figura mais realista, porém restrita a peque-
nas regiões do plano, dessa famı́lia:

10

5
x
0,1 0,2 0,3 0,4 0,5 0,6
0

-5

-10

-15
13. EQUAÇÕES DE CLAIRAUT E DE LAGRANGE: ISÓCLINAS RETAS 550

15

10

0
-0,6 -0,5 -0,4 -0,3 -0,2 -0,1
x
-5

-10

A primeira figura é para x > e a segunda para x < 0, onde se vê parte da curva
envelope y = − 76 · x1 em vermelho.

13. Equações de Clairaut e de Lagrange: isóclinas retas


Lagrange10 considerou o problema seguinte: resolver as equações diferencias de
primeira ordem tais que as curvas isóclinas são todas retas.
dy
Em suma, já que as isóclinas surgem de fixarmos dx = C, trata-se do problema
de resolver equações diferenciais da forma:
dy
y = a(p) · x + b(p), .
onde p :=
dx
Precisamos nos acostumar a distinguir entre o subconjunto de pontos do plano
determinado por uma curva - o traço da curva - e as diferentes maneiras como podemos
percorrer esse subconjunto - as diferentes parametrizações. A idéia de Lagrange é dar
as curvas-soluções na forma de curvas parametrizadas por:
x = x(p) e y = y(p).
Quando falharia essa idéia ? Quando a inclinação p ≡ C ao longo de uma porção
da curva-solução. Mas nesse caso essa porção da curva-solução está contida em alguma
reta:
y = C · x + C2 (p).
E ademais, como começamos com
y = a(p) · x + b(p)
concluı́mos que
a(p) = C = p.
Em suma, (partes de) retas y = Cx + C2 são soluções de
dy
y = a(p) · x + b(p), onde p :=
dx
10 São chamadas Equações de D’Alembert no livro de E. Kamke, Differentialgleichungen- Lo-
sungsmethoden und losungen, T. I, Chelsea Publisinhg Company, 1948, pg. 31
CAPÍTULO 36. ASPECTOS GERAIS DAS EQUAÇÕES DE PRIMEIRA
ORDEM 551

quando houver solução de


a(p) − p = 0
Se ocorrer que a(p) ≡ p então genericamente as soluções são retas. É o caso das
equações que vimos na Seção 11:
(y ′ )2 − 4x · y ′ + 4y = 0,
ou seja,
(y ′ )2
y = x · y′ − ,
4
que vimos ter por soluções a famı́lia de retas
y = 2c · x − c2 .
Uma equação do tipo
y = y ′ · x + b(y ′ )
é uma Equação de Clairaut e é uma classe importante de equações. As retas
y = c · c + b(c), c∈R
são soluções.
De agora em diante suporemos então que
a(p) − p 6≡ 0.
Cada vez que tivermos uma raı́z de a(p) − p = 0 teremos (porções de) curvas-
soluções contidas em retas e a idéia de parametrizar a solução por x = x(p) e y = y(p)
deve ser abandonada.
Já que p varia ao longo das soluções, derivo em p a expressão
y = a(p) · x + b(p),
obtendo
dy da dx db
= · x + a(p) · + .
dp dp dp dp
Usando:
dy = p · dx
obtemos:
dx da dx db
p· = · x + a(p) · +
dp dp dp dp
e daı́, já que a(p) − p 6= 0:
da db
dx dp dp
− ·x= .
dp p − a(p) p − a(p)
Esta é em geral uma equação linear a coeficientes variáveis. Com o fator de
integração
R dp
da
− p−a(p) dp
µ(p) := e
a solução é:
Z db
−1 dp
x(p) = µ(p) ·( µ(p) · dp + K), K ∈ R.
p − a(p)
13. EQUAÇÕES DE CLAIRAUT E DE LAGRANGE: ISÓCLINAS RETAS 552

De y = a(p) · x + b(p) obtemos:


y(p) = a(p) · x(p) + b(p)
como querı́amos.

Exemplo:
Suponhamos que a(p) = αp, α 6= 1 e que b(p) ≡ C1 . Neste caso simples,
db
p − a(p) = (1 − α)p e =0
dp
portanto
da db
dx dp dp
− ·x =
dp p − a(p) p − a(p)
se reduz a:
dx α
= · x.
dp (1 − α)p
logo: R α α
dp
x(p) = C2 · e (1−α)p = C2 · ||p|| (1−α)p
e
α
y(p) = α · C2 · ||p|| (1−α)p · p + C1 .
Se p > 0 temos
1
y(p) = α · C2 · p 1−α + C1 .
Como neste caso simples a equação original é linear:
dy dy y C1
y = αx · + C1 ⇔ − =−
dx dx αx αx
R 1 1
− αx dx
sabemos resolvê-la e obtemos, com o fator de integração ν(x) := e = x− α , se
x > 0, e temos:
1
y(x) = K · x α + C1 , x > 0.
Para chegarmos de
1
y(x) = K · x α + C1 , x > 0, K 6= 0
em
1
y(p) = α · C2 · p 1−α + C1 , p>0
basta notar que
dy K 1−α
p= = ·x α ,
dx α
ou seja,
α α
x=( · p) 1−α
K
e escolhermos
α 1−α
1
C2 = ( ) .
K
Exemplo:
CAPÍTULO 36. ASPECTOS GERAIS DAS EQUAÇÕES DE PRIMEIRA
ORDEM 553

p2 dy
y= · x + 2p, p=
2 dx
é uma equação de Lagrange.
2
As duas soluções p = 0, 2 de p − a(p) = p − p2 = 0 dão origem a duas soluções
retas da equação original:
y = 2x + 4 e y ≡ 0.
Se p 6= 0 e p 6= 2, então da equação de Lagrange obteremos, como explicado, a
equação diferencial linear:
dx p 2
− p 2 · x = 2 .
dp p −
2
p− p 2
R 2
dp
Usando o fator de integração µ(p) = e = (p−2)2 , obteremos a solução geral:
p−2

1
x(p) = · (4 ln(p2 ) − 4p + K), K ∈ R.
(p − 2)2
e daı́
p2
y(p) = · x(p) + 2p.
2
14. Transformação de Legendre, dualidade e resolução de equações
diferenciais
Considere uma função y = y(x) tal que sua derivada y ′ = y ′ (x) seja ela mesma
uma função inversı́vel.11
Denote a função inversa de y ′ = y ′(x) por x = x(y ′ ).
Defino
X := y ′(x)
e a transformação de Legendre de y = y(x) é a função Y (X) dada por
Y (X) := x · y ′ (x) − y(x) = X · x(X) − y(x(X)).
Afirmo que:
dY
Y ′ (X) := = x(X).
dX
De fato,
′ d(x · y ′ (x) − y(x)) (x(X) · X − y(x))
Y (X) = := =
dX dX
dx(X) dy(x) dx
= x(X) + ·X − · =
dX dx dX
dx(X) dx
= x(X) + ·X −X · = x(X).
dX dX
Agora afirmo que:
y(x) = X · Y ′ (X) − Y (X),
11Isso pode ser garantido se y ′′ (x) > 0 ∀x num Intervalo I, ou seja, se y(x) for convexa, pois
então y (x) é estritamente crescente em I e segue que y ′ (x) é inversı́vel.

14. TRANSFORMAÇÃO DE LEGENDRE, DUALIDADE E RESOLUÇÃO DE
EQUAÇÕES DIFERENCIAIS 554

pois da definição que demos


Y (X) := x · y ′ (x) − y(x)
obtenho
y(x) = x · y ′(x) − Y (X) = Y ′ (X) · x − Y (X).
Reunindo o que temos:
X = y ′ (x) e x = Y ′ (X)
e
Y (X) = x · y ′ (x) − y(x) e y(x) = X · Y ′ (X) − Y (X).
Essa possibilidade de trocar Y por y (e vice-versa) e de trocar X por x (e vice-versa)
nas duas expressões acima e manter a verdade é um caso do princı́pio de dualidade.
Para ficar mais fundamentada essa dualidade, noto também que
y ′′(x) > 0 ⇒ Y ′′ (x) > 0.
De fato,
dY
d2 Y d( dX ) dx
Y ′′ (X) :=
2
:= = =
dX dX dX
1 1
= dX := ′′ > 0,
( dx ) y (x)
onde usei o Teorema da derivada da função inversa.
Se pode, ademais, provar que a transformação de Legendre é involutiva.

A idéia agora é usar a transformação de Legebdre para passar de uma equação


diferencial F (x, y, y ′) = 0 para outra equação F (X, Y, Y ′ (X)) = 0 que seja mais fácil
de resolver !
Feito isso, da soução Y = Y (X) de F (X, Y, Y ′ (X)) = 0 passamos à solução da
equação original via:
x = Y ′ (X), y = X · Y ′ (X) − Y (X)
que é um tipo de parametrização da solução de F (x, y, y ′) = 0.
O Exemplo a seguir12 já deve dar uma idéia da utilidade da transformação de
Legendre:

Exemplo:
Resolver:
(a2 · x + b2 · y + c2 ) · (y ′ )2 + (a1 · x + b1 · y + c1 ) · y ′ + a0 · x + b0 · y + c0 = 0,
onde ai , bi , ci ∈ R.
Solução: se faço as mudanças
y ′ = X, x = Y ′ (X), y = XY ′ (X) − Y,
12 Esses dois exemplos tirei de E. Kamke, Differentialgleichungen
CAPÍTULO 36. ASPECTOS GERAIS DAS EQUAÇÕES DE PRIMEIRA
ORDEM 555

que nada mais são que a transformação de Legendre, obtemos - basta expandir a
expressão obtida por composição e depois reunir os termos -
(A(X) + X · B(X)) · Y ′ (X) − B(X) · Y + C(X) = 0,
onde
A(X) := a2 X 2 + a1 X + a0 , B(X) := b2 X 2 + b1 X + b0 e C(X) := c2 X 2 + c1 X + c0 .
Ora, sabemos resolver esta equação diferencial linear de primeira ordem
B(X) C(X)
Y ′ (X) − ·Y = −
A(X) + X · B(X) A(X) + X · B(X)
via fator de integração
R B
− A+X·B dX
µ(X) = e .
Portanto teremos explicitamente:
R R
Z R
B
dX B
dX B C(X)
Y = Y (X) = K · e A+X·B −e A+X·B · e− A+X·B
dX
· dX.
A(X) + X · B(X)
E daı́ a solução geral x = Y ′ (X) e y = X · Y ′ (X) − Y (X) da equação original.

Exemplo:
Resolver:
x3 (y ′ )2 − 2x2 yy ′ + xy 2 − y ′ = 0.
Solução: Reescrevo-o como:
y ′ = x · (xy ′ − y)2 .
Com a transformação de Legendre
y ′ = X, x = Y ′ (X), Y (X) = xy ′ − y
essa equação vira a equação separada:
X = Y ′ (X) · Y (X)2 ,
que se resolve por:
X2 Y3
= + K, K ∈ R.
2 3
Ou seja,
3 1
Y (X) = ( X 2 + K) 3 .
2
Daı́ sai
x = Y ′ (X) y = X · Y ′ (X) − Y (X).
15. APÊNDICE: FUNÇÕES CONTÍNUAS DE DUAS VARIÁVEIS E
CONTINUIDADE UNIFORME 556

15. Apêndice: Funções contı́nuas de duas variáveis e continuidade


uniforme
Para a Seção 3 e para outras ainda por vir, precisamos esclarecer algumas noções.
Queremos determinar o que deve significar para uma função z = f (x, y) de duas
variáveis ser contı́nua num ponto (x, y) de seu domı́nio. Quando dissermos apenas
contı́nua significará em cada ponto de seu domı́nio.
Definição 15.1. Dizemos que z = f (x, y) é contı́nua num ponto (x, y) se dado ǫ > 0,
existe δ > 0 tal que
||(x, y) − (x, y)|| < δ ⇒ |F (x, y) − F (x, y)| < ǫ,
onde q
||(x, y) − (x, y)|| := (x − x)2 + (y − y)2
e onde possivelmente δ depende de ǫ e de (x, y).

Note que essa definição pede que haja aproximação do valor F (x, y), não impor-
tando em que direção no plano nos aproximemos de (x, y),
A função
(x + y)2
z = F (x, y) := , se (x, y) 6= (0, 0) e F (0, 0) = K
x2 + y 2
não é contı́nua em (0, 0) para nenhuma escolha de K ∈ R.
De fato, escolha um K. Se nos aproximamos de (0, 0) pela reta y = x a função
vale nesses pontos:
4x2
z = F (x, x) := = 2, se x 6= 0 e F (0, 0) = K
2x2
enquanto que se nos aproximamos de (0, 0) pela reta y = −x a função vale nesses
pontos:
z = F (x, −x) := 0, se x 6= 0 e F (0, 0) = K.
Logo ou |F (x, x) − K| não fica pequeno ou |F (x, −x) − K| não fica pequeno.
Já um polinômio de duas variáveis
z = a00 + a10 x + a0,1 y + a11 xy + . . . ann xn y n
de grau 2n é um bom exemplo de função contı́nua no sentido da Definição 15.1.
No Capı́tulo 6 vimos que
1
f : (0, +∞) → R, f (x) =
x
é uma função contı́nua.
Mas o Exemplo 2) da Seção 2 do Capı́tulo 5 já tinha mostrado o que a Figura
indica: que vai ficando mais difı́cl encontrar o δ > 0 adequado à medida que x se
aproxima do 0 para que tenhamos:
1 1
|x − x| < δ ⇒ | − | < ǫ.
x x
CAPÍTULO 36. ASPECTOS GERAIS DAS EQUAÇÕES DE PRIMEIRA
ORDEM 557

Figura: Para um mesmo ǫ, preciso cada vez menores valores de δ


1
O mesmo fenômeno acontece em duas variáveis, por exemplo f (x, y) = x2 +y 2 , com

(x, y) 6= (0, 0).


Mas se restringimos a função para o domı́nio:
1
f : [a, +∞) → R, f (x) = ,
x
onde
a > 0,
então tudo fica mais simples.
Se quero um δ com
1 1
|x − x| < δ ⇒ | − | < ǫ
x x
basta tomar:
δ := ǫ · a2
pois então, independentemente de x:
1 1 x−x |x − x| |x − x|
| − |=| |= ≤ ≤ ǫ,
x x xx xx a2
se |x − x| < ǫ · a2 .

A próxima afirmação dá uma resposta geral (sua prova é mais tı́pica dos cursos
de Análise):
Afirmação 15.1. Seja f um função em uma variável x ou em duas variáveis (x, y),
que é contı́nua em cada ponto de um intervalo fechado [a, b] ou de um retângulo
fechado [a, b] × [c, d].
Então a escolha de δ > 0 para que:
|x − x| < δ ⇒ |f (x) − f (x)| < ǫ,
ou para que
||(x, y) − (x, y)|| < δ ⇒ |f (x, y) − f (x, y)| < ǫ,
só depende de ǫ e não no ponto particular x ou (x, y).
16. EXERCÍCIOS 558

16. Exercı́cios
Exercı́cio 16.1. (resolvido)
Seja n ∈ N, com n ≥ 2 fixado.
Considere a equação diferencial:
((n + 1)xn−1 y n + n2 xn y n−1 ) · y ′ (x) + nxn−2 y n+1 + n(n + 1)xn−1 y n = 0
i) Encontre um fator integrante µ(x) para a equação.
ii) determine as curvas integrais.
CAPı́TULO 37

Curvas de Perseguição

Este capı́tulo consegue reunir temas distintos, que já tratamos, como equações
diferenciais separáveis, envelopes e cônicas. E dá uma aplicação prática, o que me
parece valioso. 1
1. O problema
Imagine um objeto P = P (t) que sai de
(0, y)
no eixo positivo dos y e que todo tempo persegue um outro objeto Q = Q(t) que se
desloca a partir da origem, no sentido do eixo dos x.
Perseguir aqui significa que todo tempo a reta tangente à curva descrita por P (t)
passa por Q(t).
A reta tangente faz então papel da visão do predador P (t), que está todo o tempo
fixada na presa Q(t).
Por isso o tema interessou A. Lotka, estudioso dos aspectos matemáticos da Ecolo-
gia, como veremos mais adiante neste Capı́tulo.
Se não colocamos nenhuma hipótese sobre as velocidades dos pontos o problema
é intratável, mas:
Afirmação 1.1. Imagine um predador P = P (t) que sai de
(0, y)
no eixo positivo dos y e que todo tempo persegue Q = Q(t) que se desloca a partir
da origem, no sentido do eixo dos x. Suponha que o vetor velocidade de P (t) tem
módulo constante v1 e que a velocidade de Q(t) é constante v2 .
i) Se r := vv12 < 1 então
y
• no tempo t = v1 ·(1−r2 ) o predador P (t) colide com a presa Q(t) no ponto do
ry
eixo dos x cuja coordenada é x = 1−r2
y
• o predador percorreu a distância 1−r2 .
• a curva descrita por P (t) tem equação
yr 1−r
y −r ry
x=− ·y + · y 1+r + .
2(1 − r) 2(1 + r) 1 − r2
1Aprendi essas coisas inicialmente com o livro The W. L. Putnam Mathematical Competition,
Problems and solutions, 1938-1964., Math. Association of America. e depois com artigos de A.
Bernhardt, Curves of pursuit, Scripta Mathematica, vol. 20, 1954, vol. 23, 1957 e vol. 24, 1959,
bem como com o de A. Lotka, Families of curves of pursuit, and their isochrones, The American
Mathematical Monthly, Vol. 35, No. 8 (Oct., 1928), pp. 421-424.
559
1. O PROBLEMA 560
v2
ii) Se r := v1
= 1 então
1
• o predador não alcança a presa, mas segue-a a uma distância que tende a y
quando t → +∞.
• a curva descrita pelo predador P (t) tem equação
y y y y y
x = − ln( ) + ( )2 − .
2 y 4 y 4
A figura a seguir ilustra um dia da caça e outro do caçador.
Cuide que o eixo dos y foi posto horizontalmente e as escalas não são as mesmas
para fica evidente o ponto de impacto.

20

15

10

0
0 1 2 3 4 5 6
y
1
Fig.: Com y = 6 e r = 2
a presa é apanhada em x = 4. Em verde a curva se r = 1.

Na prova da Afirmação usamos bastante a comodidade da notação de Leibniz para


as derivadas e para a regra da cadeia.
Demonstração.
A curva do predador P (t) será vista como uma curva parametrizada
γ(t) = (x(t), y(t)),
onde t é o tempo, com γ(0) = (0, y), com y > 0 fixado. E ademais Q(0) = (0, 0).
A equação x = f (y) do traço de γ(t) então tem
dx
(y) = 0,
dy
pois o predador P (t) olha verticalmente a presa Q(t) quando t = 0.
CAPÍTULO 37. CURVAS DE PERSEGUIÇÃO 561

Como Q(t) se desloca seguindo o eixo dos x, então


dx
(y) < 0, ∀y,
dy
ou seja, a coordenada y é estritamente decrescente com t.
Isso permite que pensemos na coordenada y de γ como função inversı́vel de t, ou
seja:
y = y(t) e t = t(y).
Quando usar
dt
dy
usarei também
dy dt
· ≡1
dt dy
para expressar as regras de derivada de composta/inversa.
Lembro que
dt
< 0 ∀y.
dy
A condição de perseguição diz que:
dx x(t) − v2 · t
= ∀t ≥ 0,
dy y(t)
ou seja,
dx
y(t) · = x(t) − r · v1 · t.
dy
Por hipótese
r
dx 2 dy
v1 ≡ ( ) + ( )2 ,
dt dt
de onde obtemos: r
dt dx dy dt
v1 · (− ) = ( )2 + ( )2 · (− ) =
dy dt dt dy
r s
dx dy dt
= ( )2 + ( )2 · ( )2 =
dt dt dy
s
dx dt dy dt
= ( · )2 + ( · )2 =
dt dy dt dy
s
dx
= ( )2 + 1.
dy
Como dissemos acima, temos t = t(y) e a equação pode ser escrita como
dx
y· = x(t(y)) − r · v1 · t(y).
dy
1. O PROBLEMA 562

Derivo-a em y obtendo:
dx d2 x dx dt
+y· 2 = − r · v1 · ,
dy dy dy dy
ou seja, s
2
d x dt dx 2
y· 2
= −r · v1 =r· ( ) + 1.
dy dy dy
Com a variável
dx
z :=
dy
o que temos então é a equação diferencial:
dz √
y· = r · z 2 + 1,
dy
que é separável:
1 dz r
√ − = 0.
z 2 + 1 dy y
A solução geral é: √
ln(z + z 2 + 1) − r · ln(y) = C1 ,
pois já vimos a primitiva
Z √
1
√ dz = ln(z + z 2 + 1)
z2 + 1
no Capı́tulo 25.
dx
A constante C1 fica determinada pela condição que em y = y temos z := dy
= 0:
−r · ln(y) = C1
ou seja a solução é:

ln(z + z 2 + 1) − r · ln(y) = −r · ln(y),
quer dizer: √
r · ln(y) − r · ln(y) = ln(z + z 2 + 1),
ou seja
y √
ln(( )r ) = ln(z + z 2 + 1)
y
e portanto:
y √
( )r = z + z 2 + 1.
y
Isso dá:
y
(( )r − z)2 = z 2 + 1
y
e daı́ isolo z:
1 y 1 y
z = − ( )−r + ( )r .
2 y 2 y
CAPÍTULO 37. CURVAS DE PERSEGUIÇÃO 563
dx
R
Como z = dy
então z dy = x + C e portanto, se
0 < r < 1,
então no item i) obtemos
y y y y
x + C2 = − · ( )1−r + · ( )1+r .
2 · (1 − r) y 2 · (1 + r) y
A constante C2 se determina com a condição de que quando x = 0 temos y = y:
y y r·y
C2 = − + =− .
2 · (1 − r) 2 · (1 + r) 1 − r2
Obtivemos então no caso 0 < r < 1 que
y y y y r·y
x=− · ( )1−r + · ( )1+r +
2 · (1 − r) y 2 · (1 + r) y 1 − r2
descreve o traço de γ, a trajetória do predador.
Tudo que fizemos acima era para y > 0. Mas quando y → 0 vemos que a coorde-
nada x(y) de γ verifica:
r·y
x(y) → ,
1 − r2
pois r < 1.
Por outro lado, como
dx 1 y 1 y
y· = y · (− ( )−r + ( )r ) =
dy 2 y 2 y
1 y 1−r 1 y 1+r
= − · −r + · r
2 y 2 y
dx
e como 0 < r < 1 vemos que y → 0 implica y · dy
→ 0, ou seja,
dx
x(y) − r · v1 · t(y) = y · → 0 quando y → 0.
dy
Já que a posição da presa em função do tempo é dada por
r · v1 · t(y),
o que vemos é que quando y → 0 também a posição da presa tende a
r·y
.
1 − r2
r·y
Logo o ponto no eixo dos x dado por 1−r2 é o ponto em que o predador pega a
presa.
O tempo transcorrido na caçada foi
y
.
v1 · (1 − r 2 )
O predador percorreu a distância
y y
v1 · 2
=
v1 · (1 − r ) 1 − r2
1. O PROBLEMA 564

Retomando agora o caso


r=1
do item ii), de
dx 1 y 1y
z := = − ( )−1 +
dy 2 y 2y
obtemos, integrando:
y y y y
x = − ln( ) + ( )2 + C
2 y 4 y
e C se determina com a condição de que, em x = 0, temos y = y:
y y y y y
x = − ln( ) + ( )2 − .
2 y 4 y 4
Temos
dx
x(y) − r · v1 · t(y) = y · =
dy
1 y 1 y2
= − · −1 +
2 y 2y
e portanto:
−1
x(y) − r · v1 · t(y) → − quando y → 0
y
(o sinal negativo significa que o predador está atrás da presa). Ou seja distância entre
presa e predador: p
(r · v1 · t(y) − x(y))2 + y 2
tende a y1 .


A Afirmação a seguir reúne algumas observações que eu pude fazer após entender
a Afirmação 1.1:
Afirmação 1.2. Imagine um predador P = P (t) que sai de
(x, y), com x ≥ 0 e y > 0
e que todo tempo persegue Q = Q(t) que se desloca a partir da origem, no sentido do
eixo dos x. Suponha que o vetor velocidade de P (t) tem módulo constante v1 e que a
velocidade de Q(t) é constante v2 .
Se r := vv12 < 1 então
• o predador P (t) colide com a presa Q(t) no ponto do eixo dos x cuja coorde-
nada é
y Ay
− +x
2A · (1 − r) 2(1 + r)
onde r
x x
A = + ( )2 + 1.
y y
CAPÍTULO 37. CURVAS DE PERSEGUIÇÃO 565

• a curva descrita por P (t) tem equação


yr 1−r
A · y −r y A·y
x=− ·y + · y 1+r + − + x.
2A · (1 − r) 2(1 + r) 2A · (1 − r) 2(1 + r)
• se fixamos y > 0 e perguntamos por qual a coordenada x do ponto de partida
do predador que faz com que o predador alcance a presa em menos tempo a
resposta é:
y·r
x= √ .
1 − r2
De fato, o ponto de impacto no eixo dos x também tem coordenada
y·r
x= √ .
1 − r2
A figura a seguir mostra as trajetórias de três predadores: Em vermelho o que sai
de (0, 6) √
e apanha a presa em (4, 0); em verde o que sai√ de (1, 6) e em amarelo o que
sai de (2 3, 6). Esse último apanha a presa no ponto (2 3, 6) e segundo a Afirmação
1.2 é o que minimiza o tempod e caçada.

0
0 1 2 3 4 5 6
y

Na figura a seguir faço um zoom da figura para ver as diferentes posições em que
apanham a presa:

3,6

3,2

2,8

2,4

0 0,1 0,2 0,3 0,4 0,5


y
2. AS ELIPSES ISÓCRONAS, SEGUNDO A. LOTKA 566

Demonstração.
Basta repetir a prova da Afirmação 1.1 mas levando em conta como devem ser
determinadas as constantes de integração C1 e C2 .
A constante C1 fica determinada agora pela condição que em y = y temos
dx x
z := = ,
dy y
pois a reta tangente de γ deve passar pela origem.
E depois a constante C2 fica determinada por x = x quando y = y.
Desse jeito se chega, como antes, na equação da curva γ:
yr A · y −r y A·y
x=− · y 1−r + · y 1+r + − + x,
2A · (1 − r) 2(1 + r) 2A · (1 − r) 2(1 + r)
que tende a
y A·y
− +x
2A · (1 − r) 2(1 + r)
quando y → 0, pois 0 < r < 1.
Fixado y e deixando variável apenas a coordenada x temos uma função
y A(x) · y
d(x) := − + x,
2A · (1 − r) 2(1 + r)
onde r
x x
A(x) = + ( )2 + 1,
y y
que dá a posição de impacto no eixo dos x. Se minimizamos essa posição de impacto
no eixo dos x estaremos minimizando o tempo da caçada (pois esse tempo é igual à
posição no eixo x dividido por v2 , a velocidade da presa).
Um cálculo mecânico dá que d′ (x) se anula em:
y·r
x= √ ,
1 − r2
e que d′′ (x) nesse ponto é positiva. Esse mı́nimo local de fato é o ponto de mı́nimo
global de d(x).


2. As elipses isócronas, segundo A. Lotka


Para entender o que fez A. Lotka vamos introduzir alguns objetos (o leitor pode
acompanhar na Figura a seguir)
• novas coordenadas (x, y) no ponto I de impacto entre predador e presa. Note
que x tem a orientação oposta de x.
• um sistema de coordenadas polares (ρ, θ) móvel, que dará informação do
movimento da presa Q = Q(t) em relação ao do predador P = P (t). O pólo
ˆ Então π ≤ θ ≤ π.
é em Q e θ = P QI. 2
CAPÍTULO 37. CURVAS DE PERSEGUIÇÃO 567

• o comprimento s da curva descrita pelo predador (ver Seção 1 do Capı́tulo


28) será medido desde o ponto I até P (t). Se r := vv21 < 1 é o quociente das
velocidades então a distância entre Q(t) e I é r · s.

y y

ρ s

Q r.s I
x x

Então, levando em contas sinais e orientações:


x = r · s − ρ · cos(θ) e y = ρ · sin(θ).
Todas essas grandezas dependem de s. Derivo em relação ao comprimento s:
dx dρ dθ
=r− · cos(θ) + ρ · sin(θ) ·
ds ds ds
e
dy dρ dθ
= · sin(θ) + ρ · cos(θ) · .
ds ds ds
Mas quando o parâmetro que descreve uma uma curva é seu próprio comprimento s,
temos: r
dx dy
( )2 + ( )2 ≡ 1.
ds ds
Ou seja que podemos escrever (levando em conta que x cresce com o crescimento de
s e que π2 ≤ θ ≤ π):
dx dy
= − cos(θ) e = sin(θ).
ds ds
Em suma, temos o sistema:
dρ dθ
− cos(θ) = r − · cos(θ) + ρ · sin(θ) ·
ds ds
e
dρ dθ
sin(θ) = · sin(θ) + ρ · cos(θ) · .
ds ds
Multiplicando a primeira equação do sistema por sin(θ), a segunda por − cos(θ) e
somando-as obtenho:

= 1 + r · cos(θ).
ds
3. UM ENVELOPE QUE É UMA CURVA DE PERSEGUIÇÃO 568

Já multiplicando a primeira do sistema por cos(θ) e a segunda por sin(θ) e somando-as
obtenho:

ρ· = −r · sin(θ).
ds
Agora é só juntar essas duas equações obtidas e temos a equação diferencial:
dρ dθ
(1 − r · cos(θ)) · + r · sin(θ) · ρ · = 1 − r2.
ds ds
Reconhecemos aı́ uma equação diferencial exata:
d [ (1 − r · cos(θ)) · ρ]
= 1 − r2 .
ds
Integrando-a temos:
(1 − r · cos(θ)) · ρ = (1 − r 2 ) · s + C.
A constante C fica determinada quando impomos que para s = 0 (ou seja, estando
em I) a distância entre P e Q é ρ = 0. Ou seja, C = 0.
Portanto
(1 − r 2 ) · s (1 − r 2 ) · s
ρ= = .
1 − r · cos(θ) 1 + r · cos(π − θ)
Ora, para cada s fixado
(1 − r 2 ) · s
ρ=
1 + r · cos(π − θ)
é uma elipse com excentricidade 0 < r < 1 e com (1 − r 2 ) · s de semi-latus rectus (veja
a Afirmação 7.1 do Capı́tulo 39).
Lembre que naquela descrição o ângulo θ := π − θ é medido com o eixo polar (eixo
dos x > 0) e que o pólo do sistema polar (ρ, θ) é o foco da cônica.
A interpretação que Lotka dá é a seguinte (sempre supondo velocidades v1 , v2
constantes e r = vv21 ).
Suponha que a presa Q segue em direção ao refúgio I que dista dela r · s. Se um
predador P seguindo uma curva de perseguição qualquer avista Q, então P consegue
pegar Q antes que este se refugie se P está no interior da elipse
(1 − r 2 ) · s
ρ= .
1 + r · cos(π − θ)
Essa elipse descreve todos os pontos em que P , seguindo curvas de perseguição, pega
Q em I.

3. Um envelope que é uma curva de perseguição


A observação desta Seção é de Gomes Teixeira, em seu Traité de courbes speciales
remarquables, vol. III, páginas 137-138.
Considere a famı́lia de retas que se forma por reflexão de retas verticais em pontos
(x, y) do gráfico de
y = f (x) = a · ln(x),
onde a 6= 0 é fixado.
CAPÍTULO 37. CURVAS DE PERSEGUIÇÃO 569

De acordo com a Afirmação 4.1 do Capı́tulo 20, a equação dessa retas refletidas
é:
f ′ (x)2 − 1 f ′ (x)2 − 1
y=( ) · x + f (x) − ( )·x=
2f ′ (x) 2f ′ (x)

a2 − x2 x2 − a2
= · x + a · ln(x) + .
2ax 2a
Isso se pode escrever também como:
F : y · (2ax) − (a2 − x2 ) · x = 2a2 x ln(x) − (a2 − x2 ) · x.
Como F é uma famı́lia de retas com parâmetro x, pode ser derivada em relação ao
parâmetro. Obtemos:
∂F
: 2a · y + 2x · x = 2a2 ln(x) + a2 + 3x2 .
∂x
Agora note que
∂F
F −x·
∂x

−(a2 − x2 ) · x = −2x · (a2 − x),
de onde
x = 2x.
Quando substituido em F , x = 2x dá:
x2 a
y = a ln(x) − + .
2a 2
Ou seja, a equação do envelope da famı́lia de retas F é:
x ( x )2 a
y = a ln( ) − 2 + ,
2 2a 2
ou seja, o envelope é:
x2 a
y = a ln(x) − + − a ln(2).
8a 2
Se reconhece aı́, trocando x por y, uma curva de perseguição do tipo do item ii)
da Afirmação 1.1.
A figura a seguir ilustra a situação, com a = 1, ou seja, y = f (x) = ln(x) (verde),
com 8 retas da famı́lia F e onde a curva envelope (em vermelho)
x2 1
y = ln(x) − + − ln(2)
8 2
persegue pontos no eixo vertical.
4. EXERCÍCIOS 570

0
1 2 3 4 5
x
-1

-2

-3

4. Exercı́cios
Exercı́cio 4.1. (resolvido)
3
Em 1687, Huygens observou que as curvas y = a · x 4 − x, para x ≥ 0, com a > 0
fixado, têm as seguintes propriedades:
a8
i) a área da região finita que fica entre seus gráficos e o eixo dos x tem área 14
.

ii) a tangente ao seu gráfico em (x, y) passa por (− x3 , x3 ), não importando qual o
a > fixado.
3
Prove i) e ii) e, ademais, esboce qualitativamente o gráfico de y = x 4 − x, para
a > 0. Ou seja, determine sinais e raı́zes, crescimento e decrescimento, concavidades
e se há assı́ntotas quando x → +∞.
3
A propriedade ii) diz então que as curvas y = a · x 4 − x são curvas de perseguição
dos pontos (− x3 , x3 ) que se movem na reta y = −x. O quociente entre as velocidades
não é constante neste exemplo.
CAPı́TULO 38

Cinética quı́mica e crescimento bacteriano

Quando saı́mos do campo das equações diferenciais lineares, em geral topamos


com equações difı́ceis de serem resolvidas explicitamente (ou mesmo impossı́veis ...).
Mas algumas equações diferenciais não-lineares bem especiais são ainda fáceis de
serem resolvidas e muito úteis.

1. Cinética quı́mica
Esta Seção expõe trechos de Notas do Professor Mark Thompson.
Infelizmente não exponho tudo que há em suas notas. Detalhei um pouco mais
algumas contas e acrescentei uns gráficos.

Já em 1850, L. F. Wilhelmy estudou a reação em que água e sacarose produzem


celulose e frutose:
H2 O + C12 H22 O11 −→ C6 H12 O6 + C6 H12 O6
e verificou que taxa de decrescimento da quantidade/concentração c(t) de sacarose
no tempo t era proporcional à quantidade/concentração do açúcar não-invertido:
c′ (t) = −k · c(t).
A constante k é chamada de taxa especı́fica da reação ou constante da reação.
Mas, em muitos casos, o decrescimento da quantidade cA (t) do reagente A não
depende somente da quantidade de A mas também da de outros reagentes B, C . . . , Z.
E pode acontecer do decrescimento ser dado por uma lei geral:
c′A (t) = −k · caA · cbB · . . . · czZ , onde a, b, . . . , z ∈ R
Chama-se ordem da reação a soma de expoentes:
a + b + c + . . . + z.
Alguns exemplos:
• i) A decomposição do pentóxido de nitrogênio:
2 N2 O5 −→ 4 NO2 + O2 ,
segue a lei
[N2 O5 ]′ (t) = −k · [N2 O5 ](t)
onde [N2 O5 ](t) é a concentração no instante t. Por isso é uma reação de
primeira ordem.
571
1. CINÉTICA QUÍMICA 572

• ii) Já a decomposição do dióxido de nitrogênio:


2 NO2 −→ 2 NO + O2 ,
segue a lei:
[NO2 ]′ (t) = −k · [NO2 ]2 (t)
, sendo portanto de segunda ordem.
• iii) A reação:
C2 H5 Br + (C2 H5 )3 N −→ (C2 H5 )4 NBr
segue também uma lei de segunda ordem, mas do tipo:
[C2 H5 Br]′ (t) = −k · [C2 H5 Br](t) · [(C2 H5 )3 N](t).
• iv) a ordem não precisa ser um número inteiro, por exemplo, a decomposição:
CH3 CHO −→ CH4 + CO,
segue a lei:
3
[CH3 CHO]′(t) = −k · [CH3 CHO] 2 (t).
Note que as formas estequiométricas de i) e ii) são iguais, mas as ordens de
reação são diferentes. Para se entender a ordem de uma reação é preciso entender o
mecanismo da reação.
A maioria das reações quı́micas não são simples do ponto de vista cinemático
e envolvem uma sequência de estágios entre os reagentes iniciais e os produtos fi-
nais. Cada uma das etapas é chamada de reação elementar. Reações complexas são
sequências de reações elementares.
Um conceito importante é o de molecularidade de uma reação. Por exemplo, a
decomposição do iodeto de hidrogênio:
2 HI −→ H2 + I2
acontece quando duas moléculas de HI se chocam com suficiente energia para produzir
um rearranjo das ligações quı́micas (de duas H − I ligações para uma H − H ligação
e uma I − I ligação). Como esse processo elementar envolve duas moléculas sua
molecularidade é 2.
Experimentalmente se observa que:
[HI]′ (t) = −k · [HI]2 (t).
Todas1 as reações de molecularidade 2 são de ordem 2. Esse princı́pio já nos garante
que a decomposição do ozônio:
2 O3 −→ 3 O2 ,
não tem molecularidade 2, já que se sabe que ela obedece à lei:
[O3 ]2 (t)
[O3 ]′ (t) = −k · .
[O2 ](t)
1mas nem toda reação de ordem dois é de molecularidade dois.
CAPÍTULO 38. CINÉTICA QUÍMICA E CRESCIMENTO BACTERIANO 573

de ordem 1. Essa lei mais complicada pode ser explicada analisando duas reações
elementares envolvidas na reação
2 O3 −→ 3 O2 .
São elas:
O3 ⇋ O2 + O e O + O3 −→ 2O2 .
A primeira delas é muito rápida e leva a um equilı́brio da forma:
[O3 ](t)
[O](t) = C · , C ∈ R>0
[O2 ](t)
enquanto que
O + O3 −→ 2O2
satifaz uma lei:
[O3 ]′ (t) = −k ′ · [O](t) · [O3 ](t).
Portanto
′ ′ [O3 ]2 (t) [O3 ]2 (t)
[O3 ] (t) = −k · C · = −k · .
[O2 ](t) [O2 ](t)
Existem muitas reações cuja cinética é plenamente conhecida, algumas com mecan-
ismos apenas razoavelmente estabelecidos e outras com mecanismos ainda discutidos
e pesquisados.

2. Equação diferencial de uma reação de primeira ordem


Considere a reação quı́mica da forma:
A −→ B + C.
Suponha que a concentração da substância A é dada inicialmente por f (0) = a
mol/litro e que após um tempo2 x haja a − f (x) mol/l de A e que se formaram f (x)
mols/l das substâncias B e C.
Então a função f (x) mede a taxa de formação de B e C a partir de A.
Afirmação 2.1. Suponhamos que f (x) com f (0) = a verifica:
f ′ (x) = k · (a − f (x)), k > 0.
Então
f (x) = a · (1 − e−k·x )
e noto que limx→+∞ f (x) = a.
Demonstração.
De fato,
f ′ (x) = ka − k · f (x) = −k · f (x) + k · a, k > 0
é uma equação do tipo estudado na Afirmação 4.1 da Seção 4 do Capı́tulo 35.
Aquela Afirmação dá a solução f (x) na forma:
ka ka
f (x) = (f (0) + ) · e−kx − =
(−k) (−k)
2Volto usar x para tempo, ao invés de t, para ser coerente com notações de Capı́tulos anteriores
3. EQUAÇÃO DIFERENCIAL DE UMA REAÇÃO DE SEGUNDA ORDEM 574

= (f (0) − a) · e−kx + a.
Mas f (0) = 0 e portanto: f (x) = a · (1 − e−kx ). 

3. Equação diferencial de uma reação de segunda ordem


Considere uma reação quı́mica:

A + B −→ C + D

em que as concentrações de A e B são dadas inicialmente por a e b e que, após um


tempo x, f (x) mols/l de A e B tenham reagido produzindo f (x) mols/l de C e D.

Afirmação 3.1. Suponha que a concentração f (x) de C e D verifica

a − f (x) > 0 e b − f (x) > 0 ∀x

e satisfaz:
f ′ (x) = k · (a − f (x)) · (b − f (x)), k > 0.
Então:
a · b · (1 − ek(a−b)·x )
f (x) = .
b − a · ek(a−b)·x
Ademais,

lim f (x) = b, se a > b e lim f (x) = a, se b > a.


x→+∞ x→+∞

As Figuras a seguir ilustram a Afirmação:

1,5

0,5

0
0 0,5 1 1,5 2 2,5 3
x

Figura: Caso k = 1, a = 2, b = 3
CAPÍTULO 38. CINÉTICA QUÍMICA E CRESCIMENTO BACTERIANO 575

2,5

1,5

0,5

0
0 0,5 1 1,5 2 2,5 3
x

Figura: Caso k = 1, a = 4, b = 3

Demonstração.
Note que de f ′ (x) = k · (a − f (x)) · (b − f (x)) obtenho, dividindo:
f ′ (x)
=k
(a − f (x)) · (b − f (x))
Como já vimos no item ii) da Seção 1 do Capı́tulo 26:
Z
f ′ (x)
dx =
(a − f (x)) · (b − f (x))
Z
−1 f ′ (x) 1 f ′ (x)
= [ · + · ] dx =
a − b (a − f (x)) a − b (b − f (x))
Z Z
1 −f ′ (x) 1 −f ′ (x)
= · dx − · dx =
a − b (a − f (x)) a − b (b − f (x))
Z Z
1 1 1 1
= · du − · dv =
a−b u a−b v
1 1
= · ln(u) − · ln(v) =
a−b a−b
1 1
= · ln(a − f (x)) − · ln(b − f (x)).
a−b a−b
Por outro lado,
1 1
· ln(a − f (x)) − · ln(b − f (x)) = k · x + C.
a−b a−b
Mas se x = 0 temos f (0) = 0, o que dá:
ln(a) − ln(b)
C=
a−b
e portanto:
1
· ( ln(a − f (x)) + ln(b) − ln(b − f (x)) − ln(a) ) = k · x,
a−b
4. CRESCIMENTO BACTERIANO 576

que dá:
1 b · (a − f (x))
· ln( ) = k · x,
a−b a · (b − f (x))
ou seja,
b · (a − f (x))
ln( ) = (a − b) · k · x
a · (b − f (x))
e aplicando exponencial temos:
b · (a − f (x))
= ek·(a−b)·x .
a · (b − f (x))
Agora é só isolar f (x), provando assim a afirmação sobre o formato da f (x).
Se a > b então
lim ek(a−b)·x = +∞
x→+∞
e daı́:
ab
lim f (x) = = b.
x→+∞ a
No caso b > a temos
lim ek(a−b)·x = 0
x→+∞
e daı́:
ab
lim f (x) = = a.
x→+∞ b


4. Crescimento bacteriano
Quando uma quantidade de bactérias é posta num meio de cultivo adequado,
inicialmente sua a população cresce muito rápido.
Mas, ao longo do tempo, quando começam a aparecer detritos e começa a haver
competição por nutrientes há uma desaceleração do crescimento e a população tende
a um platô. Ou seja, ainda nascem e morrem indivı́duos mas a população fica mais
ou menos estável.
Obtemos a mesma descrição no caso das populações humanas em paı́ses desen-
volvidos, que inicialmente cresceram muito mas atualmente atingiram platôs.
O tipo de equações diferenciais simples que modela o crescimento bacteriano é a
seguinte:
f ′ (x) = r · f (x) − s · f 2 (x), r > 0, s > 0.
onde f (x) é a população em cada instante.
Note que para f (x) < 1 temos f 2 (x) < f (x) e a contribuição de −sf 2 (x) pode ser
pouco relevante, mas à medida que f (x) aumenta, essa parte quadrática da equação
se manifesta.
É claro que f (x) ≡ rs é solução de
r r
0 ≡ f ′ (x) = r · ( ) − s · ( )2 ≡ 0.
s s
Por isso afirmamos:
CAPÍTULO 38. CINÉTICA QUÍMICA E CRESCIMENTO BACTERIANO 577

Afirmação 4.1. Seja f : I → R derivável com

r
0 < f (x) < , ∀x ∈ I
s

e satisfazendo ∀x ∈ I:

f ′ (x) = r · f (x) − s · f 2 (x), r > 0, s > 0.

Então
f (0) · rs · er·x
f (x) = r ,
s
− f (0) · (1 − er·x )

a qual tem
r
lim f (x) = .
x→+∞ s

Na Figura a seguir ploto a solução especial f (x) = rs ao lado de soluções não


constantes. Note que há pontos de inflexão nos gráficos, fenômeno inexistente nas
soluções que apareceram na Seção 3. a próxima Seção 5 discutirá a posição desses
pontos de inflexão.

10

0
0 0,2 0,4 0,6 0,8 1 1,2
x

Figura: O gráfico de y = 10 (vermelho) e os gráficos de


f (0)· rs ·er·x
y = r −f (0)·(1−e r·x ) , com r = 10, s = 1 e f (0) = 0.05, 0.5, 1.
s

Pode ser interessante para o leitor considerar um gráfico tı́pico de crescimento


bacteriano, ao lado do de suas derivadas, para acentuar a presença do ponto de
inflexão:
4. CRESCIMENTO BACTERIANO 578

2
x
0 0,5 1 1,5 2 2,5 3
0

-2

-4

-6

Figura: y = f (x) (vermelho), y = f ′ (x) (verde) e y = f ′′ (x) (amarelo)

Uma conta tediosa mostra que podemos re-escrever a função dada na Afirmação
4.1:
f (0) · rs · er·x
f (x) = r ,
s
− f (0) · (1 − er·x )
como
r
s r 1
f (x) = −r·x
, onde k := −1 + · .
1+k·e s f (0)
Este último tipo de função é chamada de função logı́stica. É usada nas mais
variadas áreas de conhecimento, da Biologia à Economia.
Demonstração. Note que esta equação
f ′ (x) = r · f (x) − s · f 2 (x), r, s > 0,
re-escrita como:
r
f ′ (x) = −s · (0 − f (x)) · ( − f (x))
s
é um caso particular da equação diferencial estudada na Seção 3:
f ′ (x) = k · (a − f (x)) · (b − f (x)),
pondo-se
r
k = −s, a=0 e b= .
s
Não podemos aplicar imediatamente a Afirmação 3.1 pois na prova daquela Afirmação
usamos f (0) = 0, coisa que não temos aqui.
Mas podemos reciclar aquela prova3, como segue.
De f ′ (x) = −s · (0 − f (x)) · ( rs − f (x)) obtenho, dividindo:
f ′ (x)
= −s.
(0 − f (x)) · ( rs − f (x))
3Note que a estamos resolvendo como equação separável.
CAPÍTULO 38. CINÉTICA QUÍMICA E CRESCIMENTO BACTERIANO 579

Então, como fizemos lá:


Z
f ′ (x)
dx =
(0 − f (x)) · ( rs − f (x))
Z
s f ′ (x) −f ′ (x)
= · [ + r ] dx =
r (0 − f (x) ( s − f (x))
Z
s f ′ (x) −f ′ (x)
= · [− + r ] dx =
r f (x) ( s − f (x))

s s r
= − · ln(f (x)) + ln(( − f (x))),
r r s
que fazem sentido pois 0 < f (x) < rs .
Por outro lado,
s r
· [− ln(f (x)) + ln( − f (x))] = −s · x + C.
r s
Avaliando em x = 0, com f (0) > 0:
s r
C= · [− ln(f (0)) + ln( − f (0)) ]
r s
e portanto:
s r r
· [− ln(f (x)) + ln( − f (x)) + ln(f (0)) − ln( − f (0)) ] = −s · x
r s s
que dá:
f (0) · ( rs − f (x))
ln( ) = −r · x,
f (x) · ( rs − f (0))
ou seja:
f (x) · ( rs − f (0))
ln( ) = r · x.
f (0) · ( rs − f (x))
Aplicando exponencial temos:
f (x) · ( rs − f (0))
r = er·x
f (0) · ( s − f (x))

Agora é só isolar f (x), obtendo o formato afirmado.


Ademais, como r > 0, temos limx→+∞ er·x = +∞ e do formato da f (x) é fácil de
ver que limx→+∞ f (x) = rs .

5. PONTO DE INFLEXÃO DA FUNÇÃO LOGÍSTICA 580

5. Ponto de inflexão da função logı́stica


Afirmação 5.1. A solução de
f ′ (x) = r · f (x) − s · f 2 (x), r > 0, s > 0,
dada por
r
s r 1
f (x) = , onde k := −1 + · ,
1+k· e−r·x s f (0)
tem um único ponto de inflexão cujas coordenadas são:
ln(k) r
( , ).
r 2s
Note que a segunda coordenada não depende de f (0).
f (0)· rs ·er·x
A figura a seguir mostra, com r = 10, s = 1, os três gráficos y = r −f (0)·(1−e r·x )
s
para diferentes condições iniciais: f (0): 0.05, 0.5, 1. Todos têm inflexão na altura 5:

10

0
0 0,2 0,4 0,6 0,8 1 1,2
x

Demonstração.
Cada solução y = f (x) terá ponto de inflexão onde a sua derivada f ′ (x) tem um
valor máximo ou mı́nimo.
Mas
f′ = r · f − s · f2
e se pensamos f agora como uma variável usual4, podemos usar o sabemos sobre o
gráfico de
z = r · u − s · u2 ,
r
é uma parábola com concavidade para baixo, com ponto de máximo em u = 2·s .
Ou seja que os pontos de inflexão de todas as soluções ocorrem em pontos
r
(x, f (x)) = (x, ).
2·s
4A idéia que uso agora se aplicará a qualquer equação diferencial autônoma, ou seja, y(x)′ =
P (y(x)) onde P não depende explicitamente de x, só de y(x)
CAPÍTULO 38. CINÉTICA QUÍMICA E CRESCIMENTO BACTERIANO 581

Mas o tempo x é diferente para cada solução. De fato,


r 2 · k · e−r·x
f ′ (x) = .
s · (1 + k · e−r·x )2
e
r 3 · k · e−r·x · (k · e−r·x − 1)
f ′′ (x) = .
s · (1 + k · e−r·x )3
Portanto f ′′ (x) = 0 exatamente onde
k · e−r·x − 1 = 0,
isto é, em:
ln(k) r 1
x := , onde k := −1 + ·
r s f (0)
′′ ′′
e ademais f (x) > 0 se x < x e f (x) < 0 se x > x.
Em suma, x é o único ponto de inflexão.


6. Equação de Bernoulli e reações quı́micas de ordem fracionária


A solução geral da Equação de Bernoulli
f ′ (x) = a(x) · f (x) + b(x) · f (x)r ,
dada na Afirmação 13.1 do Capı́tulo 35, no caso particular em que
r = 2, a(x) ≡ a e b(x) ≡ b,
nos permite re-obter os resultados das Seções 4 e 5, pois:
1
f (x) =
g(x)
onde Z
b
g(x) = e−ax · eax · (−b) dx + C · e−ax = − + C · e−ax .
a

já que g (x) = −a · g(x) − b. Ou seja,
1
f (x) = b ,
− a + C · e−ax
de onde se obtem, para f (0) 6= 0, o valor
1 b
C= + .
f (0) a
Logo
−a
1 b
f (x) = b aC·e−ax
= aC·e−ax
=
− a · (1 − b
) 1− b
−a −a
b b
= 1 = a =
a( f (0) + ab )·e−ax 1− e−ax − e−ax
1− b
b f (0)
6. EQUAÇÃO DE BERNOULLI E REAÇÕES QUÍMICAS DE ORDEM
FRACIONÁRIA 582
−a −a
b b
= = ,
1 + ( bf−a
(0)
− 1) · e−ax 1 + k · e−ax
onde
−a 1
k := −1 + · ,
b f (0)
e pondo
r := a e − s := b
temos exatamente a função logı́stica da Seção 5.
Mas, o que é importante, há reações quı́micas cuja cinética é expressa por Equações
de Bernoulli com expoente r fracionário:
f ′ (x) = a(x) · f (x) + b(x) · f (x)r , r ∈ Q.
Por exemplo, a decomposição do acetaldeı́do:
CH3 CHO → CH4 + CO
verifica (fase gasosa a 450 graus C):
3
[CH3 CHO]′(x) = −k · [CH3 CHO] 2 (x), k>0
onde uso x para o tempo.
Nessa situação r = 23 e pedimos que f (x) := [CH3 CHO](x) > 0.
Para a(x) ≡ 0 e b(x) ≡ −k, a prova da Afirmação 13.1 do Capı́tulo 35 diz que a
função
1
g(x) := f (x)− 2
verifica
k
g ′ (x) = ,
2
k
ou seja, g(x) = 2 · x + g(0) e portanto:
k 1
f (x) = ( ·x+ p )−2 .
2 f (0)
CAPı́TULO 39

Newton e a gravitação

(...) Halley colocou a questão diretamente para Newton em agosto de 1684:


supondo-se uma lei do inverso do quadrado da distância para a atração do Sol, que
tipo de curva faria o planeta ? Newton lhe disse, uma elipse. Disse-lhe que havia
calculado isso havia muito tempo. (..) que não conseguia achar os cálculos, mas
prometeu refazê-los e enviá-los mais tarde (...)
(trecho da biografia de Newton, de J. Gleick)

Este Capı́tulo explicará alguns dos cálculos que Newton queria mostrar a Halley...
Além de seu interesse intrı́nseco, serve de motivação ao tema das equações difer-
enciais de segunda ordem.

1. Atração segundo o inverso do quadrado da distância


Se lembramos como é enorme raio do globo terrestre, podemos pensar que a
distância entre os objetos caindo (em queda-livre ou arremessados, nas Seções ante-
riores) e o centro da Terra é muito próxima do valor do Raio da Terra1:
R ∼ 6.378 · (10)6 m.
Estabeleçamos a lei de atração universal, de Newton, que é formulada para dois
pontos com massa:

dois pontos de massa m0 e m se atraem recı́procamente com uma força da ordem


G·m0 ·m
de r2
,
onde G é uma constante universal e r é a distância entre eles.
Agora imaginemos a massa da Terra M ∼ 5.98 · 1024 concentrada no seu centro
(centro de gravidade). O que acontece quando queremos usar a lei de atração para
explicar a atração mútua exercida pelo centro de gravidade da Terra e um ponto de
massa m = 1?
Obteremos:
g G·M ·m
=g= ∼
m R2
G · 5.98 · 1024
∼ ,
(6.378)2 · (10)12
e portanto
G ∼ 6.67 · (10)−11 ,
em unidades m3 /(s2 kg).
1Os dados sobre a Terra obtive em R. Resnick e D. Halliday, Fı́sica, LTC.
583
2. TEMPO DE COLISÃO E VELOCIDADE DE ESCAPE 584
F
Ademais como a massa da Terra é enorme, sua aceleração M
pode ser considerada
nula.
2. Tempo de colisão e velocidade de escape
Agora que já colocamos os fenômenos de queda-livre e balı́stica no quadro da lei
geral da atração gravitacional, consideremos:
Afirmação 2.1. Suponha um ponto de massa M colocado na origem e outro ponto P
de massa m na posição (x(0), 0), com x(0) > 0. Suponha M tão grande que possamos
considerar o ponto na origem como parado.
Suponha que no instante t = 0 o vetor velocidade (x′ (0), y ′(0)) tenha componente
vertical nula y ′(0) = 0 (ou seja, caso estiver em movimento, o faz no eixo horizontal).
Então
• É constante ∀t a grandeza:2
(x′ (t))2 GM
− .
2 x(t)
• Se x′ (0) = 0 (velocidade inicial zero) então o tempo de colisão entre o ponto
P e a origem é de: r
π x(0) 3
· .
2 2GM
• Para escapar da atração do ponto na origem e se afastar tanto quanto quis-
ermos da origem (i.e. limt→+∞ x(t) = +∞), é necessário e suficiente que
s
2 · GM
x′ (0) ≥ .
x(0)
q
• ademais, se x′ (0) = 2·GM x(0)
então sua velocidade é sempre positiva mas tende
a zero (limt→+∞ x′ (t) = 0).
• em particular, para um foguete lançado da superfı́cie da Terra escapar da
atração da Terra e se afastar da Terra:
s
2 · GM
x′ (0) ≥ ∼ 11.184 m/s.
x(0)
Demonstração.
A Lei de Atração de Newton diz:
G·M ·m
m · x′′ (t) = − ,
x(t)2
onde o sinal − deve-se a que a atração é oposta ao sentido positivo dos x.
Logo
G·M
x′′ (t) = − ,
x(t)2
2chamada (x′ (t))2
de Energia total, onde 2 é chamada de energia cinética e − GM
x(t) de energia
potencial.
CAPÍTULO 39. NEWTON E A GRAVITAÇÃO 585

′′ ′ x′ (t)
x (t) · x (t) ≡ −Gm0 ,
x(t)2
e portanto
(x′ (t))2 ′ 1 ′
[ ] ≡ Gm0 · [ ],
2 x(t)
ou seja
(x′ (t))2 Gm0 ′
[ − ] ≡0
2 x(t)
e
(x′ (t))2 Gm0
− ≡ C.
2 x(t)
Se o corpo foi largado com velocidade inicial
x′ (0) = 0,
então obtenho
Gm0
C=− ,
x(0)
e portanto s
Gm0 Gm0
x′ (t) = − 2 · ( + )
x(0) x(t)
(onde tomo a raı́z negativa poque o ponto P se aproximará da origem).
Como x′ (t) < 0, para t > 0, a função x(t) é estritamente decrescente.
Logo posso considerar a função inversa t = t(x). A fórmula da derivada da função
inversa dá:
1
t′ (x) = − q .
2 · ( Gm 0
x(0)
+ Gm0
x
)
Para calcular o tempo t de colisão entre P e a origem podemos fazer a integral
Z t
t−0= dt =
0
Z 0
= t′ (x) dx,
x(0)
pois assim estaremos calculando o tempo que trancorre para sairmos de x(0) > 0 e
chegarmos em x = 0 (a origem).
Ou seja,
Z x(0) Z x(0)
′ 1
t=− t (x) dx = q dx.
0 0 2 · ( Gm 0
x(0)
+ Gm0
x
)
Se somamos frações, simplificamos, e usamos que as constantes saem da integral,
obtemos:
Z x(0) r Z x(0) √
1 x(0) x
q dx = · p dx,
0 2 · ( Gm 0
+ Gm0
) 2GM 0 x(0) − x
x(0) x

onde se nota que x(0) − x > 0.


2. TEMPO DE COLISÃO E VELOCIDADE DE ESCAPE 586

Agora faço a substituição para u > 0:


x = u2 e dx = 2u du,
obtendo:
r Z x(0) √ r Z √x(0)
x(0) x x(0) u2
· p dx = 2 · p du.
2GM 0 x(0) − x 2GM 0 x(0) − u2
u2
Não é difı́cil conferir que uma primitiva de √ é:
x(0)−u2

up x(0) u
− x(0) − u2 + · arcsin( p ).
2 2 x(0)
Portanto: r Z √x(0)
x(0) u2
t=2 · p du =
2GM 0 x(0) − u2
r p q p
x(0) x(0) p x(0) x(0)
=2 · [− x(0) − ( x(0))2 + · arcsin( p )] =
2GM 2 2 x(0)
r
x(0) x(0) π
=2 · · =
2GM r 2 2
π x(0) 3
= ,
2 2GM
como querı́amos demonstrar.
Agora consideremos a situação em que x′ (0) > 0.
Determinemos a condição necessária e suficiente sobre x′ (0) > 0 para que o ponto
P escape da atração do ponto na origem e se afaste tanto quanto quisermos da origem.
Já vimos que:
(x′ (t))2 GM
− ≡ C,
2 x(t)
ou seja
(x′ (t))2 GM
0≤ ≡C+ .
2 x(t)
Mas, se há um escape onde x(t) → +∞, então GM x(t)
→ 0 e daı́:
0 ≤ C.
Portanto:
(x′ (0))2 GM
− ≡ C ≥ 0,
2 x(0)
de onde s
2GM
x′ (0) ≥ .
x(0)
O caso s
2GM
x′ (0) =
x(0)
CAPÍTULO 39. NEWTON E A GRAVITAÇÃO 587

equivale a que
(x′ (t))2 GM
− ≡ 0,
2 x(t)
ou seja,
(x′ (t))2 GM
= .
2 x(t)
Portanto
√ 1
x′ (t) = 2GM p
x(t)
e p √
x(t) · x′ (t) = 2GM ,
que, integrando, dá:
2 3 √
x(t) 2 = 2GM · t + D, D ∈ R.
3
De onde:
3 √ 2
x(t) = ( · ( GM · t + D)) 3 .
2
Portanto
lim x(t) = +∞ mas lim x′ (t) = 0,
t→+∞ t→+∞
√ 1
pois x′ (t) = 32 ( 23 · ( GM · t + D))− 3 .


3. Nı́veis de energia
Na situação da Afirmação 2.1 vimos que
(x′ (t))2 GM
− ≡ C.
2 x(t)
Aprendemos na prova dessa Afirmação que o escape ocorre quando
(x′ (t))2 GM
− ≡C≥0
2 x(t)
e a colisão quando
(x′ (t))2 GM
− ≡ C < 0.
2 x(t)
Chamamos esses valores de C de nı́veis de energia.
No caso de colisão, a conservação de Energia Total implica que limx→0 x′ (t) = +∞,
Por isso as trajetórias de colisão são chamadas de singularidades do conjunto de
trajetórias possı́veis para um corpo que é atraı́do por outro de massa muito maior.
Se multiplicamos por 2 · x(t) obtemos das expressões anteriores:
(x′ (t))2 · x(t) − 2GM − C · x(t) ≡ 0.
Num plano (x, y) = (x(t), x′ (t)) essas curvas são as cúbicas:
y 2 · x − 2GM − C · x ≡ 0.
3. NÍVEIS DE ENERGIA 588

Elas são qualitativamente o seguinte (note que para C ≥ 0 são formadas de dois
ramos):

C>0

C<0

x
C=0

Ademais podemos pensar na equação diferencial de segunda ordem, que é do tipo:


1
x′′ = −
x2
como um campo vetorial (x′ , y ′), tangente a essas curvas, da forma:
1
x′ = y, y′ = −
x2
e a figura agora fica mais completa:

C>0

C<0

x
C=0

Essa figura nos diz que:


CAPÍTULO 39. NEWTON E A GRAVITAÇÃO 589

• No caso C < 0, um corpo arbitrariamente próximo da origem que parte com


velocidade positiva arbitrariamente alta atinge um ponto onde sua velocidade
se anula e começa a ser atraı́do, colidindo com velocidade arbitrariamente
nehgativa.
• No caso C = 0, se um corpo arbitrariamnte próximo da origem parte com
velocidade positiva arbitrariamente alta ele consegue escapar, com velocidade
positiva tendendo a zero. E também que poderia vir de arbitrariamente longe
um corpo com velocidade negativa arbitrariamente pequena e que colidisse
com velocidade arbitrariamente negativa.
• No caso C = 0, se um corpo arbitrariamnte próximo da origem parte com
velocidade positiva arbitrariamente alta ele consegue escapar. E também que
poderia vir de arbitrariamente longe e que colidisse com velocidade arbitrari-
amente negativa.

4. Órbitas planetárias
Na Seção anterior estudamos como se dá a colisão entre um corpo e outro de
massa muito maior, que o atrai de acordo com a lei de Newton.
Mas a situação mais interessante é quando o objeto de pequena massa (planeta,
satélite, cometa, etc) gravita em torno do de grande massa (estrela) sem colidir.
A princı́pio esta Seção usa dados do plano e de funções duas variáveis, portanto
seria mais natural num curso de Cálculo em duas variáveis, enquanto o nosso tem
sido em uma variável.
Mas ela é tão profundamente ligada à origem e ao objetivo do criador do Cálculo,
que se torna inevitável apresentá-la.
Vamos nos situar num plano onde suporemos que viaja o planeta em sua órbita,
para simplificar o problema.
De fato, a primeira etapa do problema geral é mostrar que, apesar de estar num
espaço 3-dimensional, a órbita do planeta é de fato plana. Ou seja, que cada planeta
não sai de uma fatia plana do espaço.
Para obter os resultados de Newton, começo lembrando que agora há duas coor-
denadas
P (t) = ( x(t) , y(t) ).
do planeta, que mudam com o tempo t.
Ademais a velocidade instantânea P ′ (t) será
P ′ (t) := ( x′ (t) , y ′(t) ),
como já explicamos na Seção 3 do Capı́tulo 28.
Enquanto que a aceleração instantânea será, pelo mesmo motivo,
P ′′ (t) := ( x′′ (t) , y ′′ (t) ).

5. Velocidade e aceleração expressas em coordenadas polares


Por um motivo que vai ficar claro um pouco mais adiante, vamos criar um novo
modo de descrever a posição P (t) = (x(t), y(t)), a velocidade P ′ (t) e a aceleração
P ′′ (t).
5. VELOCIDADE E ACELERAÇÃO EXPRESSAS EM COORDENADAS
POLARES 590

Estamos acostumados a encontrar um ponto especı́fico do plano através de um par


de informações sobre ele, a coordenada x e a coordenada y. Mas o sistema cartesiano
ortogonal é apenas um instrumento para determinar pontos no plano.
Podemos usar outro par de informações, por exemplo a distância r do ponto até
um ponto - chamado Pólo - e o ângulo anti-horário θ que o vetor posição forma com
uma semireta - chamada eixo polar. Essa descriçaõ dos pontos se chama sistema de
coordenadas polares.
Apesar da utilidade dessa nova descrição (r, θ) não se deve esquecer que θ fica
definido a menos da ambiguidade:
θ + k · 2π, k∈Z
A partir de agora sobrepomos ao sistema cartesiano (x, y) um sistema polar. Com
isso determinaremos um ponto P (t) do plano dizendo qual a distância r(t) que o
ponto tem da origem e qual o ângulo θ(t) (definido módulo k · 2π, k ∈ Z), que o vetor
(x(t), y(t)) forma com o eixo x > 0. Ou seja,
p x(t) y(t)
r(t) = x(t)2 + y(t)2 , cos(θ(t)) = e sin(θ(t)) = .
r(t) r(t)
Note que numa pequena região em torno do P (t) podemos escolher o ângulo θ(t)
sem ambiguidade. As funções cos(θ(t)) e sin(θ(t)) são deriváveis se r(t) 6= 0. E
também
y(t)
θ(t) = arcsin( )
r(t)
é derivável se r(t) 6= 0.
Temos também:
x(t) = r(t) · cos(θ(t)) e y(t) = r(t) · sin(θ(t))
e, pelas regras de derivação de produto e composta:
P ′ (t) := ( x′ (t) , y ′ (t) ) =
= ( r ′ (t) · cos(θ(t)) − r(t) · sin(θ(t)) · θ′ (t) , r ′ (t) · sin(θ(t)) + r(t) · cos(θ(t)) · θ′ (t) ).
Note que3
||P ′(t)||2 = x′ (t)2 + y ′ (t)2 = r ′ (t)2 + r(t)2 · (θ′ (t))2 .
A expressão de
P ′′ (t) := ( x′′ (t) , y ′′(t) )
é maior, como o leitor pode verificar.
Agora vem uma etapa engenhosa: vamos querer obter as projeções dos vetores
P (t) e P ′′ (t) em duas direções: numa direção paralela a P (t) e numa direção ortogonal

a P (t).
A direção paralela a P (t) é dada pelo vetor de módulo 1:
1
( cos(θ(t)) , sin(θ(t)) ) = · P (t).
r(t)
3O

módulo de um vetor v = (a, b) do plano é ||v|| = a2 + b 2
CAPÍTULO 39. NEWTON E A GRAVITAÇÃO 591

Já a direção ortogonal a P (t) será dada pelo vetor de módulo 1:


( − sin(θ(t)) , cos(θ(t)) ).
Vamos usar o item iii) da Afirmação 3.2 do Capı́tulo 17 como método para obter
projeções.
Então obtemos que a projeção de V = P ′ (t) na direção
v = ( cos(θ(t)) , sin(θ(t)) )
é dada por
r ′ (t) · ( cos(θ(t)) , sin(θ(t)) )
pois (sem t para simplificara notação) vale a igualdade:
r ′ = (r ′ cos(θ) − r sin(θ)θ′ ) · cos(θ) + (r ′ sin(θ) + r cos(θ)θ′ ) · sin(θ).
E do mesmo modos se obtêm que a projeção de V = P ′ (t) na direção
v = (− sin(θ(t)) , cos(θ(t)))
é dada por:
r(t) · θ′ (t) · (− sin(θ(t)) , cos(θ(t))).
Essa projeção diz que, para uma mesma mudança de ângulo θ′ (t), quanto maior
for r mais rápido vamos na direção ortogonal a P (t).
Uma conta um pouco maior4 dará que a projeção da aceleração P ′′(t) na direção
v = ( cos(θ(t)) , sin(θ(t)) )
é:
[r ′′ (t) − r(t) · (θ′ (t))2 ] · ( cos(θ(t)) , sin(θ(t)) ).
Note que se o movimento é perfeitamente circular, r(t) = r e o módulo dessa
projeção vira r · (θ′ (t))2 : esse termo está ligado à força centrı́peta, que aumenta com
o aumento de (θ′ (t))2 .
E uma conta mais longa dá que a projeção da aceleração P ′′ (t) na direção de
v = (− sin(θ(t)) , cos(θ(t)))
é:
[r(t) · θ′′ (t) + 2 · r ′ (t) · θ′ (t)] · (− sin(θ(t)) , cos(θ(t))).
Note agora que essa projeção da aceleração muda quando r(t) aumenta ou diminui:
isso é o que faz um patinador girando ao abrir ou fechar os braços, para diminuir ou
aumentar a velocidade do giro.
4Se
tivermos à disposição a notação Complexa P = r · eiθ e se soubermos que i · eiθ é ortogonal

a e , aı́ fica bem fácil:
P ′ = r′ · eiθ + ir · eiθ · θ′
e
P ′′ = r′′ · eiθ + i · r′ · eiθ θ′ + ir′ · eiθ · θ′ − r · eiθ · (θ′ )2 + ir · ·eiθ · θ′′ =
= eiθ · [r′′ − r · (θ′ )2 ] + i · eiθ · [2r′ θ′ + rθ′′ ].
e
6. GRANDEZAS CONSTANTES AO LONGO DAS TRAJETÓRIAS 592

6. Grandezas constantes ao longo das trajetórias


Afirmação 6.1. Suponha um ponto sendo atraı́do por força radialmente dirigida para
a origem. Suponha M tão grande relativo a m que possamos supôr o ponto na origem
tem aceleração nula. Suponha que r(0) 6= 0 e que θ′ (0) 6= 05.
Então:
i) o fato da força ser radialmente dirigida para a origem implica que ∀t é constante
a grandeza
r(t)2 · θ′ (t) ≡ C 6= 0.

ii) se adicionalmente supomos que o módulo da força radial, segundo Newton, é


GM m
r(t)2
então ∀t é constante a grandeza
m · ||P ′(t)||2 GMm
E := − ,
2 r(t)
chamada de Energia total, soma da energia cinética
||P ′(t)||2
Ec := m ·
2
e da energia potencial
GMm
Ep := − .
r(t)

Na Seção 9 vamos dar o sentido geométrico da parte i) desta Afirmação.


Demonstração. (da Afirmação 6.1)
Lidaremos com velocidade e aceleração em coordenadas polares, como explicamos
na Seção 5.
Prova de i):

A hipótese sobre a direção radial da força de atração se expressa, pelo que vimos
na Seção 5, como:
r(t) · θ′′ (t) + 2 · r ′ (t) · θ′ (t) ≡ 0.
Ou seja,
( r(t)2 · θ′ (t) )′ (t) = 2 · r(t) · r ′ (t) · θ′ (t) + r(t)2 · θ′′ (t) =
= r(t) · (2r ′ (t) · θ′ (t) + r(t) · θ′′ (t)) ≡ 0,
e portanto
r(t)2 · θ′ (t) ≡ C.
Ademais,
r(0)2 · θ′ (0) = C 6= 0,
pois supusemos r(0) 6= 0 e θ′ (0) 6= 0.
Prova de ii):
5essas hipóteses dizem que o momento angular m · r(0)2 · θ′ (0) não é nulo, o que implicará,
conforme veremos na prova da Afirmação, que o objeto não vai seguir uma trajetória radial - caso
já estudado na Seção 2
CAPÍTULO 39. NEWTON E A GRAVITAÇÃO 593

Elevando ao quadrado a expressão anterior temos r(t)4 · (θ′ (t))2 ≡ C 2 e daı́


C2
r(t) · (θ′ (t))2 = .
r(t)3
A hipótese sobre o módulo da força radial dá, conforme a Seção 5, que
GMm
m · (r ′′ (t) − r(t) · (θ′ (t))2 ) = −
r(t)2
(onde o sinal menos está ligado ao sentido da atração para a origem, oposto ao do
vetor posição P (t)).
Portanto:
C2 GM
r ′′ (t) − = −
r(t)3 r(t)2
ou seja,
′′ C2 GM
r (t) = 3
− .
r(t) r(t)2
Se r ′ (t) ≡ 0 então r(t) ≡ r constante. E como r 2 · θ′ (t) = C, concluimos que θ′ (t) = rC2
é constante. Então
2
′ 2 ′ 2 2 ′ 2 2 C C2
||P (t)|| = r (t) + r(t) · (θ (t)) = r · 4 = 2 .
r r
Portanto
||P ′(t)||2 GMm C2 GMm
m· − =m· 2 −
2 r(t) 2r r
é constante, como afirmamos.
Portanto posso considerar no que segue que r ′ (t) 6≡ 0. Daı́, multiplicando por
r ′ (t), e tomando primitivas temos:
Z t
r ′ (t)2
= r ′′ (s) · r ′ (s) · ds =
2 t0
Z t 2
C GM
= ( 3
− 2
) · r ′ (s) ds.
t0 r(s) r(s)
Reconhecemos aı́ uma fórmula de integração por substituição:
Z r(t) 2
r ′ (t)2 C GM
= ( 3 − 2 ) dr =
2 r(t0 ) r r
C2 GM
=− 2
+ + C2 ,
2 · r(t) r(t)
onde C2 é uma constante. Ou seja,
C2 2GM
r ′ (t)2 + 2
− ≡ C3 .
r(t) r(t)
onde C3 = 2 · C2 . Já observamos que:
x′ (t)2 + y ′(t)2 = r ′ (t)2 + r(t)2 · (θ′ (t))2
6. GRANDEZAS CONSTANTES AO LONGO DAS TRAJETÓRIAS 594

e também que
C2
r(t)2 · (θ′ (t))2 = .
r(t)2
Portanto
C2
x′ (t)2 + y ′ (t)2 = r ′ (t)2 + ,
r(t)2
que quando substituı́do na anterior dá:
2GM
x′ (t)2 + y ′ (t)2 − ≡ C3 .
r(t)
Se consideramos a velocidade inicial P ′ (0) concluı́mos que
2GM 2GM
x′ (t)2 + y ′(t)2 − = C3 = x′ (0)2 + y ′(0)2 − .
r(t) r(0)
Multiplicando por m2 , concluı́mos que é constante a grandeza:
m · ||P ′(t)||2 GMm
− .
2 r(t)


Afirmação 6.2.
Nas mesmas hipóteses da Afirmação 6.1 (anterior), a trajetória de P (t) = (r(t), θ(t))
pode ser descrita em coordenadas polares (r, θ) através de uma função r = r(θ).
De fato, precisamente:
C2
GM
r(θ) = √
m2 G2 M 2 +2mEC 2
1+ GM m
· cos(θ)
2 ′
onde m · C = m · r (t) · θ (t) é o momento angular e E = Ec + Ep é a energia total
da trajetória.

Na próxima Seção (Seção 7) explicaremos a geometria da trajetória r(θ) dada na


Afirmação 6.2.

Demonstração. (da Afirmação 6.2)


Já vimos que
r(t)2 · θ′ (t) ≡ C = r(0)2 · θ′ (0) 6= 0,
portanto6 θ′ (t) > 0 ∀t ou θ′ (t) < 0 ∀t.
Isto permite determinar a coordenada r de P (t) como função de θ, ao longo da
trajetória. De fato, θ(t) é ou bem uma função estritamente crescente (se θ′ (t) > 0 ∀t)
ou estritamente decrescente de t (se θ′ (t) < 0 ∀t). Assim t determina θ e θ determina
r.
1
Considero uma nova variável u(t) = r(t) .
6θ ′ (t) como função de t é contı́nua, pois de fato existe θ′′ (t).
CAPÍTULO 39. NEWTON E A GRAVITAÇÃO 595

Então
1
r ′ (t) = [r(θ(t))]′ (t) = [ ]′ (t) =
u(θ(t))
1 du dθ
=− 2
· · =
u(θ) dθ dt
dθ du du
= −r 2 · · = −C · ,
dt dθ dθ
onde C é o momento angular. Coloquemos
du
r ′(t) = −C ·

e
C
r(t) · θ′ (t) = =C ·u
r(t)
na fórmula da energia cinética:
||P ′(t)||2 (r ′ (t)2 + r(t)2 θ′ (t)2 )
Ec := m · =m· =
2 2
( du )2 + u(θ)2
= mC 2 · dθ ,
2
ou seja,
du 2Ec
( )2 + u(θ)2 = .
dθ mC 2
Ora,
GMm
Ec = E − Ep = E + =
r
= E + GMm · u.
Logo
du 2
( )2 + u(θ)2 = (E + GMm · u(θ)).
dθ mC 2
Lembro que a energia total E é constante ao longo da trajetória, portanto a
derivada de E como função de θ é zero ao longo da trajetória. Logo, derivando em θ
a expressão anterior, temos:
du d2 u du 2GM du
2· · 2 + 2u(θ) = .
dθ dθ dθ C 2 dθ
Ou seja,
du d2 u GM
2· · [ 2 + u(θ) − 2 ] = 0.
dθ dθ C
Conforme provaremos na Afirmação 8.1 da Seção 8, todas as soluções da equação
diferencial
d2 u GM
2
+ u(θ) − 2 = 0
dθ C
são do tipo:
GM
u(θ) = 2 + A · cos(θ − q)
C
onde A e q são constantes arbitrárias.
Suponhamos por um momento isso.
6. GRANDEZAS CONSTANTES AO LONGO DAS TRAJETÓRIAS 596

Então u′ (θ) = −A sin(θ − q) e portanto


(u′(θ))2 = A2 sin2 (θ − q)
e
GM
(u′(θ))2 + u(θ)2 = A2 sin2 (θ − q) + ( + A · cos(θ − q))2 =
C2
G2 M 2 GM
= A2 + 4
+ 2A · 2 · cos(θ − q)
C C
e por outro lado já tinhamos
2
(u′(θ))2 + u(θ)2 = (E + GMm · u(θ)) =
mC 2
2 GM
= (E + GMm · ( + A · cos(θ − q))) =
mC 2 C2
2E 2G2 M 2 GM
= + + 2A · · cos(θ − q).
mC 2 C4 C2
Reunindo isso obtenho:
G2 M 2 2E m2 G2 M 2 + 2mEC 2
A2 = + =
C4 mC 2 m2 C 4
o que dá:

m2 G2 M 2 + 2mEC 2
A=± .
mC 2
Logo

1 GM m2 G2 M 2 + 2mEC 2
= u(θ) = 2 ± · cos(θ − q).
r(θ) C mC 2
Como cos(θ − q + π) = − cos(θ − q) não precisamos manter o ± e módulo translação
em θ, podemos escrever:

1 GM m2 G2 M 2 + 2mEC 2
= 2 + · cos(θ),
r(θ) C mC 2
C2
e multiplicando tudo por GM
:

C2 1 m2 G2 M 2 + 2mEC 2
· =1+ · cos(θ),
GM r(θ) GMm
de onde finalmente:
C2
GM
r(θ) = √ .
m2 G2 M 2 +2mEC 2
1+ GM m
· cos(θ)

CAPÍTULO 39. NEWTON E A GRAVITAÇÃO 597

7. As órbitas como cônicas em coordenadas polares


Se o eixo polar é identificado com o dos x > 0 e o Pólo com (x, y) = (0, 0) então:
p y
r = x2 + y 2 e tan(θ) = .
x
No Capı́tulo 20 definimos a excentricidade e o semi-latus rectum de uma cônica
qualquer.
Afirmação 7.1. Seja uma cônica com foco F , semi-latus rectum l e excentricidade
e > 0.
Tome coordenadas polares cujo Pólo é F . Use o eixo da cônica como eixo dos x
e ponha como eixo polar o eixo x > 0.
Então nessa coordenada polar a cônica é dada por:
l
r(θ) = ,
1 + e · cos(θ)
onde θ é o ângulo medido com o eixo polar.
Em particular:
2 2
• as elipses xa2 + yb2 = 1 viram
b2
a
r(θ) = √ .
a2 −b2
1+ a
· cos(θ)
Essa descrição se estende ao cı́rculo x2 + y 2 = a2 , pondo e = 0, o que dá a
equação r(θ) = l = a.
2 2
• As hipérboles xa2 − yb2 = 1 viram
b2
a
r(θ) = √ .
a2 +b2
1+ a
· cos(θ)

• as parábolas y 2 = 4ρ · x viram r(θ) = 1+cos(θ)
.
Demonstração.
Como o Pólo é F , temos para um ponto P da cônica
r(P ) = e · P r
onde r é diretriz da cônica.
Considere x = −(ρ + eρ) a equação da diretriz, P0 = (−eρ, 0) vértice da cônica e
o foco F = (0, 0). Ou seja, que a distância entre a diretriz e o foco F é ρ + eρ.
Denote x(P ) a coordenada x de P (que pode assumir valores positivos ou nega-
tivos). Então
P r = (ρ + eρ) + x(P )
e portanto
r(P ) = e · (ρ + eρ + x(P ))
Um ponto P̂ da cônica com P̂ r = (ρ + eρ) está situado verticalmente sobre o foco.
Pela Definição 2.1 de cônica do Capı́tulo 20,
P̂ F = e · (ρ + eρ).
7. AS ÓRBITAS COMO CÔNICAS EM COORDENADAS POLARES 598

Mas o semi-latus rectum l foi definido como a distância P̂ F , ou seja, l = e · (ρ + eρ).


Ou seja, temos
r(P ) = l + e · x(P ).
Podemos tomar o ângulo θ̂ que o vetor posição faz com a semi-reta que sai de
F = (0, 0) e chega no vértice P0 = (−eρ, 0). Assim x(P0 ) = r(P0 ) cos(0). Assim em
geral,
x(P ) = r(P ) cos(θ̂) = −r(P ) cos(π − θ̂) = −r(P ) cos(θ)
onde θ é o ângulo formado com o eixo x > 0. Daı́
r(P ) = l − e · r(P ) cos(θ)
e portanto
l
r(P ) = r(θ) = .
1 + e · cos(θ)


Afirmação 7.2. A trajetória determinada na Afirmação 6.2 como


C2
GM
r(θ) = √
m2 G2 M 2 +2mEC 2
1+ GM m
· cos(θ)
C2
é uma cônica com semi-latus rectum GM e excentricidade

m2 G2 M 2 + 2mEC 2
e= .
GMm
Ademais, é uma elipse (cı́rculo), parábola ou hipérbole se respectivamente E < 0
2 2
(E = − mG2CM 2 ), E = 0 ou E > 0.
Demonstração.
A Afirmação 7.1 já demonstrada nos diz que se trata de uma cônica com essa
excentricidade e esse semi-latus rectum.
Agora noto que:
e<1 ⇔ m2 G2 M 2 + 2mEC 2 < G2 M 2 m2 ⇔
⇔ 2mEC 2 < 0 ⇔ E < 0.
E do mesmo modo
mG2 M 2
e=0 ⇔ E=− ,
2C 2
e=1 ⇔ E=0
e>1 ⇔ E > 0.


Exemplo:
As órbitas dos planetas dos sistema Solar tem excentricidade muito pequena.
Mercúrio é o planeta do sistema solar cuja órbita tem a maior excentricidade, da
ordem de e = 0.205630. Seu semi-latus rectus é 5.54430 × 1010 m.
CAPÍTULO 39. NEWTON E A GRAVITAÇÃO 599

4E10

2E10

-6E10 -4E10 -2E10 0E0 2E10 4E10


0E0

-2E10

-4E10

l
Figura: Elipse r(θ) = 1+e cos(θ)
, e = 0.205630 e l = 5.54430 × 1010 (notação 5.5 E 10).

8. Oscilador harmônico
A Afirmação a seguir prova um fato que já usamos na prova da Afirmação 6.2,
além de reforçar o conteúdo da Afirmação 2.1 do Capı́tulo 12:
Afirmação 8.1.
i) Todas as soluções do problema
f ′′ (x) = −k 2 · f (x) + H, ∀x ∈ R
onde k, H ∈ R, são da forma
H
f (x) = a · cos(k · x) + b · sin(k · x) +
k2
onde a, b são constantes arbitrárias. Essas constantes ficam determinadas por a =
f (0) e b = f ′ (0).
ii) Ademais7,
a · cos(k · x) + b · sin(k · x) ≡ A · cos(k · x − q)
onde √ a
A= a2 + b2 e cos(q) = .
a2 + b2
Demonstração.
Se k = 0 tudo é muito fácil. Por isso suponho k 6= 0.
H
De i): Derivando duas vezes as funções a cos(k · x) + b · cos(k · x) + k2
se verifica
facilmente que elas satisfazem:
f ′′ (x) = −k 2 · f (x) + H, H ∈ R.
7Note que (A, q) funciona como coordenadas polares do vetor (a, b). Essas novas grandezas são
úteis pois dizem que a solução é um gráfico do cosseno expandido verticalmente por A (amplitude),
deslocado horizontalmente por q e com frequência modificada pelo fator k.
8. OSCILADOR HARMÔNICO 600

O que precisamos provar é que não há outros tipos de função satisfazendo essa
equação.
Considere uma misteriosa função f que satisfaça
f ′′ (x) = −k 2 · f (x) + H, H ∈R
bem como a função muito simples g(x) ≡ kH2 , que certamente também verifica essa
equação.
Então a nova função φ := f − g = f (x) − kH2 satisfaz o problema:
φ′′ (x) = −k 2 · φ(x).
Se conseguirmos provar que as únicas soluções de φ′′ (x) = −k 2 · φ(x) são da forma
a·cos(k·x)+b·sin(k·x), com a, b constantes arbitrárias, então nossa outrora misteriosa
função vira:
H
f (x) =: φ(x) + g(x) = a · cos(k · x) + b · sin(k · x) + 2 ,
k
que é o que queremos provar.
Portanto recaı́mos num problema levemente mais fácil:
φ′′ (x) = −k 2 · φ(x).
Nessa direção, vamos provar primeiro o seguinte:
Caso 1: se φ(x) satisfaz φ′′ (x) = −k 2 · φ(x) e ademais φ(0) = φ′ (0) = 0 então
φ(x) ≡ 0.
De fato, terı́amos:
φ′′ (x) + k 2 · φ(x) ≡ 0
e portanto
2φ′ (x) · [φ′′ (x) + k 2 · φ(x)] ≡ 0
ou seja,
[(φ′ (x))2 + (k 2 φ(x))2 ]′ ≡ 0
e portanto
(φ′ (x))2 + (k 2 φ(x))2 ≡ C.
Mas φ(0) = φ′ (0) = 0 dão que (φ′ (x))2 + (k · φ(x))2 ≡ 0 e isso implica que φ′ (x) ≡
φ(x) ≡ 0, como querı́amos.
Agora atacaremos o caso geral:

Caso 2: φ(x) satisfaz φ′′ (x) = −k 2 · φ(x) mas a := φ(0) e b := φ′ (0) são arbitrários.
Derivando duas vezes se vê que ψ(x) := a · cos(k · x) + b · sin(kx) satisfaz ψ ′′ (x) =
2
−k · ψ(x). Então
(φ − ψ)(x) := φ(x) − ψ(x)
satifaz
(φ − ψ)′′ (x) = −k 2 · (φ − ψ)(x).
Mas agora (φ − ψ)(0) = 0 e (φ − ψ)′ (0) = 0 e pelo Caso 1 aplicado à função (φ − ψ)(x)
concluo que φ − ψ ≡ 0, ou seja φ = a · cos(k · x) + b · sin(kx) como querı́amos.
De ii):
CAPÍTULO 39. NEWTON E A GRAVITAÇÃO 601

Temos:
cos(k · x − q) = cos(k · x) · cos(−q) − sin(k · x) · sin(−q) =
= cos(k · x) · cos(q) + sin(k · x) · sin(q) =
a b
= cos(k · x) · √ + sin(k · x) · √ ,
2
a +b 2 a + b2
2

portanto com A = a2 + b2 sai o item ii).


9. Área em coordenadas polares e a lei de Kepler sobre as áreas


Vamos aqui dar o significado geométrico do item i) da Afirmação 6.1.
Como veremos, ele diz que à medida que um planeta percorre uma órbita cônica
tendo o Sol em um de seus focos, a taxa de variação da área do setor centrado no
foco é constante.
Para isso, primeiro preciso explicar como se calculam áreas em coordenadas po-
lares, pois foi nessas coordenadas que obtivemos as tajetória cônicas.
Quando se divide uma pizza circular de raio r cortando fatias que passam pelo
centro, todos acham uma divisão justa se as fatias têm o mesmo ângulo central.
Ou seja, a área de um setor circular (a fatia de pizza) é proporcional ao ângulo
θ central. Se a abertura é θ ∈ [0, 2π] a área é:
r2
Aθ = θ · ,
2
onde a área total é A(2π) = πr 2 .
Quando temos um setor delimitado pelo pólo e por uma curva em coordenada
polar r = r(θ) ≥ 0, com θ ∈ [a, b] , podemos começar a aproximação da área dessa
região pela soma de áreas as de setores circulares de abertura ∆θi := θi − θi−1 e raio
r(ξi ), onde ξi ∈ [θi−1 , θi ]:
n
X r(ξi )2
A(∆θ1 ) + A(∆θ2 ) + . . . + A(∆θn ) = ∆θi · .
i=1
2
Veja a Figura:

r(θ)

∆θ 4
∆θ 2 ∆θ 3
∆θ 1

O
10. EM TORNO DA PROPOSIÇÃO XXX DO PRINCIPIA 602

Se pensamos em refinar a partição do intervalo [a, b], fazendo n → +∞, temos


motivada a Definição a seguir:
Definição 9.1. A área do setor determinando pelo pólo O e a curva r(θ) ≥ 0 com
θ ∈ [a, b] é:
Z b 2
r (θ)
· dθ.
a 2

Agora, se θ = θ(t) é uma função estritamente crescente de t ∈ [c, d] podemos


escrever:
Z θ0 (t0 ) 2 Z t0 2
r (θ) r (θ(t)) ′
dθ = · θ (t) dt
a 2 c 2
e pelo Primeiro Teorema Fundamental do Cálculo:
Z θ0 2
r (θ) r 2 (θ(t0 )) ′
( dθ )′ (t0 ) = · θ (t0 ).
a 2 2
Na Afirmação 6.1 temos uma situação em que θ = θ(t) é uma função estritamente
crescente e lá obtivemos no item i):
r 2 (θ(t)) · θ′ (t) ≡ C,
ou seja:
r 2 (θ(t)) ′ C
· θ (t) ≡ .
2 2
Portanto durante as trajetória dos planetas a taxa de variação das áreas dos setores
descritos é constante.
Ou seja, a velocidade areal é constante, o que é conhecido como Lei de Kepler.

10. Em torno da proposição XXX do Principia


A obra fundamental de Newton, o Principia Mathematica de 1686, não é nada
fácil de ser lida, pois, além da complexidade do tema, lá se adota uma exposição num
estilo difı́cil de ser entendido.
Tanto pelo tom imperial do autor (do tipo, faça isso e isso e esta é a resposta.
ponto final ) como principalmente por ele ter feito grande parte da exposição no estilo
da geometria grega (sintética, não-analı́tica)
Dá para entender que ele não quisesse expôr fisica nova com matemática nova,
recém criada (por ele).
O grande fı́sico S. Chandrasekhar escreveu um livro para ajudar a quem quer ler
o Principia (Newton’s Principia for the common reader ) e baseado nele (p.131 em
diante) é que consegui entender a demonstração da proposição a seguir.
Também é de se notar que algumas afirmações de Newton só foram entendidas
pela comunidade fı́sico-matemática séculos depois, como o mostrou V. Arnold.
A Afirmação a seguir é o Corolário II da Proposição XXX do Principia (veja a
Figura)
CAPÍTULO 39. NEWTON E A GRAVITAÇÃO 603
1
Afirmação 10.1. Considere uma parábola de equação x = 4a · y 2, com vértice A =
(0, 0) e foco S = (a, 0). Tome a mediatriz m do segmento AS, dada portanto por
m : x = a2 . Denote G = ( a2 , 0). Considere pontos P da parábola e mP retas
mediatrizes dos segmentos SP . Determine o ponto HP := m ∩ mP (veja Figura a
seguir).
Então à medida que o ponto P se move na parábola atraı́do segundo a lei de
atração do inverso quadrado pelo ponto no foco S, o ponto HP se move na reta m
com velocidade constante. E a velocidade de Hp é igual a 83 do módulo da velocidade
que tem P ao passar pelo vértice A.

H
P

A G S

A prova a seguir é a de S. Chandrasekhar:


Demonstração.
Temos pela construção e por Pitágoras:
2 2 2 2 2
AG + GH = GS + GH = SH .
Como os triângulos ∆SZH e ∆P ZH são congruentes, então:
2 2 2
AG + GH = P H .
Sejam O a projeção vertical de P e H ′ a projeção horizontal em P O de H, como
mostra a figura a seguir:

H H’

Y
P
S’

A G S O
10. EM TORNO DA PROPOSIÇÃO XXX DO PRINCIPIA 604

Então:
2 2 2
P H = P H ′ + H ′ H = (P O − GH)2 + (AO − AG)2 =
2 2 2 2
= P O − 2P O · GH + AO − 2AO · AG + GH + AG .
Logo igualando e cancelando termos:
2 2
0 = P O − 2P O · GH + AO − 2AO · AG,
ou seja,
2 2
2P O · GH = P O + AO − 2AO · AG.
Como x = AO e y = P O, a equação
1
x= · y2
4a
permite escrever
1 2 1 2
AO = · PO = · PO ,
4AS 4 · 2 · AG
que dá
2
2 PO 1
2P O · GH = P O · [ 1 + − ]=
(4AS) 2 4
2
23 PO
= PO · [ + ]
4 (4AS)2
e dividindo por P O 6= 0:
2
3 PO
2 · GH = P O · [ + ]=
4 (4AS)2
3 AO
= PO · [ + ]
4 4AS
Multiplicando o queobtivemos por 64 · AS obtenho:
4 1
· GH · AS = · P O(AO + 3 · AS) =
3 6
1
= · P O(4 · AO − 3 · (AO − AS)) =
6
1
= · P O(4 · AO − 3 · OS) =
6
2
= · x(P ) · y(P ) − A(∆SOP ),
3
onde x(P ) e y(P ) são as coordenadas de P da parábola e A(∆SOP ) é a área do
triângulo. √ √
Agora notamos que a área sob o gráfico de y = 2 · a x, de x = 0 até x = x(P ),
é pelo Teorema Fundamental do Cálculo:
Z x
√ √ 4 √ 3
2 · a t dt = · a · x 2 =
0 3
2 √
= · x · 4ax =
3
CAPÍTULO 39. NEWTON E A GRAVITAÇÃO 605

2
= · x(P ) · y(P ).
3
O segmento parabólico SOP é a região obtida ao retirar o triângulo ∆SOP da região
sob o gráfico da parábola de A até o ponto O. O que obtivemos acima é que a área
desse segmento parabólico SOP , denotada A(SOP ), é:
4 4a
A(SOP ) = · GH · AS = · GH.
3 3
Ou seja,
3
GH = A(SOP ).
4a
Ora, a posição de P = P (t) e H = H(t) depende do tempo t que descreve a trajetória,
portanto:
d GH(t) 3 d A( SOP (t) ) 3 C
= · ≡ ,
dt 4a dt 4a 2
onde na última equivalência usei o item i) da Afirmação 6.1, como foi interpretada
na Seção 9 anterior.
Só falta ver que o módulo da velocidade vA de P ao passar por A vale
C
vA = ,
a
para então terminarmos a demonstração.
Lembre da Afirmação 6.1 que
C ≡ r 2 (θ(t)) · θ′ (t),
ou seja
C = r 2 (θ(0)) · θ′ (0) = a2 · θ′ (0).
Como vimos na Seção 5, a velocidade P ′ (t) de P tem duas projeções: uma radial, de
módulo:
r ′ (θ(t))
e outra ortogonal, de módulo:
r(θ(t)) · θ′ (t).
Mas A = A(0) é o vértice da parábola, logo é um ponto de mı́nimo de r(θ(t)) e
portanto r ′ (θ(0)) = 0. Portanto se o tempo for medido a partir da posição A:
vA = r(0) · θ′ (0) = a · θ′ (0).
Logo:
C
vA = ,
a
como querı́amos.

11. A EQUAÇÃO DE KEPLER PARA O MOVIMENTO PLANETÁRIO
ELÍPTICO 606

11. A Equação de Kepler para o movimento planetário elı́ptico


Obteremos aqui uma equação, cuja solução na Seção 6 do Capı́tulo 46 permitirá
dizer para onde devemos olhar no céu a cada instante para localizar um determinado
planeta. Ou seja, permitirá parametrizar a posição do planeta numa órbita elı́ptica
em função do tempo.
Minha referência para esta Seção é o livro Analytical Mechanics, de A. Fasano e
S. Marmi, Oxford University Press, 2006.
Afirmação 11.1. (Equação de Kepler)
Suponhamos que um determinado planeta se move numa trajetória elı́ptica E dada
em coordenadas cartesianas por:
X2 Y 2
+ 2 = 1, 0 < b < a.
a2 b
Trace o cı́rculo C de raio a centrado na origem O = (0, 0).
Dado um ponto P (T ) (T é o tempo percorrido desde o perihélio em A = (a, 0))
da trajetória elı́ptica, denoto Q ∈ C a projeção vertical de P (T ) no cı́rculo C.
Sejam (R, φ) as coordenadas polares de Q tendo pólo em O = (0, 0).
Então:

φ − e · sin(φ) = · T,
T0
onde T0 é o perı́odo da trajetória.
2π·T
A grandeza φ é conhecida como anomalia excentrica e M := T0
é a anomalia
média.
Na Figura a seguir os dados da elipse estão em vermelho; enquanto que os do
cı́rculo e de construções auxiliares que faremos etão em azul:
Q
Y

ϕ θ

p A X
O F

Demonstração.
Suponha que o perihélio está em A, com coordenada X(A) = a > 0. Sabemos
que a coordenada de F é (X, Y ) = (e · a, 0), onde 0 < e < 1 é a excentricidade.
Sejam (r, θ) coordenadas polares com pólo no Foco A da elipse, onde se encontra
o Sol, com θ = 0 o perihélio A. Dado um ponto P 6= A da trajetória elı́ptica, denoto
CAPÍTULO 39. NEWTON E A GRAVITAÇÃO 607

Q ∈ C a projeção vertical de P no cı́rculo C. E denoto por p a projeção de P no eixo


horizontal.
No que segue pensaremos em P no semiplano Y > 0 e nos gráficos do cı́rculo e da
elipse: √
YC (X) = a2 − X 2 ,
r
X2 b √
YE (X) = b2 · 1 − 2 = · a2 − X 2 .
a a
Uma observação sobre a área do setor da elipse e do cı́rculo:
b
Ar(AF P ) = · Ar(AF Q).
a
De fato,
Ar(AF P ) = Ar(ApP ) − Ar(∆F pP ) =
Z a
F p · pP
= YE (X) dX − =
X(p) 2
Z a
b √ 2 F p · pP
= · a − X 2 dX − .
X(p) a 2
e setor do cı́rculo,
Ar(AF Q) = Ar(ApQ) − Ar(∆F pQ) =
Z a
F p · pQ
= YC (X) dX − =
X(p) 2
Z a √
F p · pQ
= · a2 − X 2 dX − .
X(p) 2
Mas
b
pP = · pQ,
a
já que YE (X) = ab · YC (X).
Logo:
b
Ar(AF P ) = · Ar(AF Q).
a
Pela lei de Kepler para as áreas varridas,
Ar(AF P (T )) = C · T,
onde T é o tempo percorrido desde o periélio (T = 0) e 2C é o momento angular. Em
particular:
Ar(E) = π · ab = C · T0 ,
onde T0 denota o perı́odo.
Logo até aqui temos para P (T )
b
C · T = · Ar(AF Q).
a
Agora noto que, para O = (0, 0) e (R, φ) coordendas polares com pólo em O:
Ar(AF Q) = Ar(AOQ) − Ar(F OQ) =
11. A EQUAÇÃO DE KEPLER PARA O MOVIMENTO PLANETÁRIO
ELÍPTICO 608

b a2 F OpQ
=·[ ·φ− ]=
a 2 2
b a2 (e · a) · (a · sin(φ))
= ·[ ·φ− ]
a 2 2
onde F = (e · a, 0).
Concluı́mos que
ab
C ·T = · [φ − e · sin(φ)].
2
e portanto
2C 2π
φ − e · sin(φ) = ·T = · T =: M.
ab T0

CAPı́TULO 40

Equações diferenciais de segunda ordem

1. Redução de ordem
Quando queremos resolver uma equação de grau 4 do tipo:
a · x4 + b · x2 + c = 0
obviamente fazemos z := x2 e descobrimos as raı́zes desta equação quadrática. Depois
voltamos na variável original x.
Do mesmo modo uma equação diferencial de segunda ordem
2
x′′ − · x′ = t
t
pede que façamos
z(t) := x′ (t)
e resolvamos primeiro a equação de primeira ordem:
2
z′ − · z = t
t
R
para depois obtermos x = z dt. Isso é uma redução de ordem.
Há um tipo de redução de ordem que se aplica a equações autônomas (onde a
variável independente não figura explicitamente) de segunda ordem. Por exemplo, a
equação da Seção 2 do Capı́tulo 39
1
x′′ = − 2
x
é uma equação autônoma.
Como a velocidade x′ (t) pode ser pensada como uma função da posição x podemos
introduzir a variável:
z := x′
e pensarmos em z = z(x).
Daı́ então (com a notação de Leibniz para a regra da cadeia):
dx′ dz dz dx dz
x′′ (t) = = = · =: ·z
dt dt dx dt dx
e a equação vira:
dz 1
· z = − 2.
dx x
Ou seja,
z2 1
= + C1
2 x
609
2. HOMOGÊNEAS, A COEFICIENTES CONSTANTES 610

e daı́ r
2
z=± + 2C1
x
ou seja, r
′ 2
x =± + 2C1 .
x
Por exemplo, com C1 = 0, continuamos com
p √
x(t) · x′ (t) = 2
de onde
2 3 √
· x(t) 2 = ± 2 · t + C2 ,
3
de onde obtemos x(t).
Esta idéia permite por exemplo resolver a equação a seguir, que é autônoma de
segunda ordem mas não-linear:
x′′ + (x′ )2 = x
vira
z′ · z + z2 = x
se fazemos como antes
dz
z = x′ e · z = x′′ .
dx
Supondo z 6= 0 e dividindo por z temos:
dz x
+z = ,
dx z
ou seja,
dz
= −z + x · z −1 ,
dx
que é uma equação de Bernoulli com expoente r = −1. Agora trata-se de resolver
esta equação (o que já sabemos fazer) e depois voltar na variável x de partida.

2. Homogêneas, a coeficientes constantes


Na Afirmação 8.1 do Capı́tulo 39 resolvemos a equação
f ′′ (x) + k 2 · f (x) = 0, ∀x ∈ R
(e também o caso não homogêneo), de onde decorre que todas as soluções do problema
f ′′ (x) + f (x) = 0, ∀x ∈ R
são da forma
y = f (x) = a · cos(x) + b · sin(x)
onde a, b são constantes arbitrárias. Essas constantes ficam determinadas por
a = y(0) e b = y ′(0).
Agora quero tratar do problema mais geral:
f ′′ (x) + K · f ′ (x) + L · f (x) = 0, K, L ∈ R.
CAPÍTULO 40. EQUAÇÕES DIFERENCIAIS DE SEGUNDA ORDEM 611

do qual uma instância já apareceu quando tratamos da Lei de Hooke com atrito no
Capı́tulo 12.
Afirmação 2.1. A solução geral de
f ′′ (x) + K · f ′ (x) + L · f (x) = 0, K, L ∈ R
fica determinada pela natureza das soluções r1 , r2 da equação quadrática:
r 2 + K · r + L = 0.
• Se há duas raı́zes Reais r1 , r2 ∈ R distintas, então a solução geral é
y = f (x) = a · er1 x + b · er2 x
que ficam determinados por
y ′(0) − r2 y(0)
a= e b = y(0) − a.
r1 − r2
• Se há uma raı́z dupla r1 = r2 ∈ R a solução geral é
K K
y = a · x · e− 2 ·x + b · e− 2 ·x ,
que ficam determinados por
K
b = y(0) e a = y(0) · + y ′ (0).
2
√ √
−K 4−K 2 −K 4−K 2
• Se r1 = 2
+I · 2
e r2 = 2
−I · 2
são Complexos, então a solução
geral é
√ √
−K
x 4L − K 2 −K
x 4L − K 2
y =a·e 2 · cos( · x) + b · e 2 · sin( · x).
2 2
que ficam determinados por
2y ′(0) + Ky(0)
a = y(0) e b = √ .
4L − K 2
x −x
Observação: Como as funções hiperbólicas são definidas por cosh(x) := e +e 2
e
x −x
sinh(x) := e −e 2
e como
ex = cosh(x) + sinh(x)
é possı́vel expressar o resultado dessa Afirmação usando as funções hiperbólicas.

A Figura a seguir compara, com as mesmas condições iniciais y(0) = 8 e y ′(0) = 10,
as diferentes soluções de
y ′′ + K · y ′ + y = 0,
onde K vale:
• K = 0 em vermelho,
• K = 1/2 em verde,
• K = 2 em amarelo e
• K = 3 em azul.
2. HOMOGÊNEAS, A COEFICIENTES CONSTANTES 612

10

x
0 2 4 6 8 10 12
0

-5

-10

Demonstração.
A idéia para resolver:
f ′′ (x) + K · f ′ (x) + L · f (x) = 0
é buscar soluções do tipo:
y = erx
onde a natureza da constante r é a essência do problema.
Ou seja, queremos que valha:
(erx )′′ + K · (erx )′ + L · erx = 0,
isto é,
erx · (r 2 + K · r + L) = 0.
Como erx 6= 0 precisamos que r satisfaça a equação caracterı́stica associada:
r2 + K · r + L = 0
cujas raı́zes são:
√ √
−K + ∆ −K − ∆
r1 := e r2 := , onde ∆ = K 2 − 4L.
2 2
CAPÍTULO 40. EQUAÇÕES DIFERENCIAIS DE SEGUNDA ORDEM 613

Se
∆ > 0 ⇔ K 2 > 4L
temos r1 , r2 ∈ R e r1 6= r2 , daı́:
y = f1 (x) = er1 x e y = f2 (x) = er2 x
são soluções, assim como qualquer combinação linear:
y = f (x) = a · er1 x + b · er2 x .
Agora as condições y(0) e y ′(0) permitem determinar a, b, pois:
y(0) = a + b e y ′(0) = r1 a + r2 b,
ou seja:
y ′(0) − r2 y(0)
a= e b = y(0) − a.
r1 − r2
O problema começa a complicar quando ∆ = 0 e quando ∆ < 0 (este último foi
o caso que apareceu no Capı́tulo 12 sobre as Leis de Hooke, onde usei K = 0.1 ou
K = 0.3 e L = 1).
Quando
∆ = 0 ⇔ K 2 = 4L
temos
K
r := r1 = r2 = − ;
2
Precisamos buscar outra solução, diferente (linearmente independente) da solução
K
y = f (x) = e− 2 ·x . A idéia é buscar soluções do tipo1:
K
y = g(x) · e− 2 ·x .
Ou seja, quero que:
K K K2 K
(g(x) · e− 2 ·x )′′ + K · (g(x) · e− 2 ·x )′ + · g(x) · e− 2 ·x = 0,
4
o que produz, depois de uma bonita simplificação,
K
e− 2 ·x · g ′′ (x) = 0,
ou seja,
g ′′(x) ≡ 0.
Então g(x) = ax + b e
K K K
y = (ax + b) · e− 2 ·x = a · x · e− 2 ·x + b · e− 2 ·x
são soluções.
As condições y(0) e y ′(0) determinam a, b:
K
b = y(0) e a = y(0) · + y ′ (0).
2
O caso mais bonito a meu ver é quando
∆ < 0 ⇔ K 2 < 4L
1Essa idéia será generalizada no Método de Redução de Ordem, de D’alembert, na Seção 11.
3. NÃO-HOMOGÊNEAS, LINEARES DE SEGUNDA ORDEM 614

pois então √ √
−K + I 4L − K 2 −K − I 4L − K 2
r1 = e r1 =
2 2
são números complexos (conjugados).
Defina como na Seção 5 do Capı́tulo 31
√ √
−K+I 4L−K 2 −K 4L−K 2
·x ·x I· ·x
y = F1 (x) = e 2 =e 2·e 2 =
√ √
−K 4L − K 2 4L − K 2
= e 2 x · (cos( · x) + I sin( · x))
2 2
e
√ √ √
−K−I 4L−K 2 −K 4L − K 2 4L − K 2
·x
y = F2 (x) = e 2 = e 2 x · (cos( · x) − I sin( · x)).
2 2
Agora se usa a observação de que as combinações lineares de soluções de
f ′′ (x) + K · f ′ (x) + L · f (x) = 0
são também soluções dessa equação diferencial.
Então, somando ou subtraindo as soluções Complexas F1 e F2 acima obtenho
soluções Reais: √
F1 + F2 −K
x 4L − K 2
f1 (x) = = e 2 · cos( · x)
2 2
e √
F1 − F2 −K 4L − K 2
f2 (x) = = e 2 x · sin( · x).
2I 2
Agora as condiçoes y(0) e y ′(0) determinam a, b em
√ √
−K
x 4L − K 2 −K
x 4L − K 2
y = a · e 2 · cos( · x) + b · e 2 · sin( · x).
2 2
pois √
′ K 4L − K 2
y(0) = a e y (0) = − a + b · ,
2 2
ou seja:
2y ′(0) + Ky(0)
a = y(0) e b = √ .
4L − K 2


3. Não-Homogêneas, lineares de segunda ordem


Considero o problema da Seção 2 anterior, mas agora no caso não-homogêneo:
f ′′ (x) + K · f ′ (x) + f (x) = g(x),
em que tomei L = 1 apenas para simplificar a exposição.
Afirmo que basta encontrar alguma solução φ1 (x) desse problema, pois qualquer
outra φ2 (x) produz
(φ1 − φ2 )(x)
CAPÍTULO 40. EQUAÇÕES DIFERENCIAIS DE SEGUNDA ORDEM 615

uma solução do problema homogêneo:

f ′′ (x) + K · f ′ (x) + f (x) = 0,

que já conhecemos da Seção anterior y = a · f1 (x) + b · f2 (x). Logo:

φ2 (x) = a · f1 (x) + b · f2 (x) + φ1 (x).


H
Foi isso que aconteceu na Seção 8 do Capı́tulo 39, onde φ1 (x) = k2
é obviamnte
uma solução de
y ′′(x) + k 2 · y(x) = H.
Podemos enunciar como um princı́pio geral:

Afirmação 3.1. (Princı́pio de superposição)


Se φ1 (x) é uma solução particular do problema não-homogêneo

y ′′ (x) + P (x) · y(x) + Q(x) · y(x) = R(x)

e se
a · f1 (x) + b · f2 (x), a, b ∈ R
são soluções gerais do problema homogêneo

y ′′(x) + P (x) · y(x) + Q(x) · y(x) = 0

então:
a · f1 (x) + b · f2 (x) + φ1 (x)
é solução geral do não-homogêneo.

Demonstração.
Dada a φ1 (x), basta notar que se φ2 (x) é uma solução qualquer de

y ′′ (x) + P (x) · y(x) + Q(x) · y(x) = R(x),

então
φ2 (x) − φx
é solução de
y ′′ (x) + P (x) · y(x) + Q(x) · y(x) = 0.


Bom, mas e como encontrar uma solução particular φ1 (x) do caso não-homogêneo
? As próximas Seções 4 e 7 tratam disso.
4. NÃO HOMOGÊNAS: MÉTODO DE LAGRANGE DE VARIAÇÃO DE
PARÂMETROS 616

4. Não homogênas: Método de Lagrange de variação de parâmetros


Suponhamos conhecidas as soluções gerais a·f1 (x)+b·f2 (x), a, b ∈ R do problema
homogêneo
f ′′ (x) + K · f ′ (x) + L · f (x) = 0, K, L ∈ R.
É de Lagrange a idéia de buscar uma solução φ1 (x) da forma
φ1 (x) = a(x) · f1 (x) + b(x) · f2 (x)
para o problema não-homogêneo:
y ′′ (x) + K · y ′ (x) + L · y(x) = g(x).
É chamado de método de variação de parâmetros, já que o que é usualmente é con-
stante (a, b) vira função não-constante (a(x), b(x)). 2
Há liberdade na escolha de a(x), b(x) pois queremos apenas uma solução, não
todas; portanto sobre sua derivada
φ′1 (x) = a′ (x)f1 (x) + a(x)f1′ (x) + b′ (x)f2 (x) + b(x)f2′ (x)
vamos impôr uma condição extra simplificadora:
a′ (x)f1 (x) + b′ (x)f2 (x) = 0.
Assim
φ′1 (x) = a(x)f1′ (x) + b(x)f2′ (x).
Como queremos que
φ′′1 (x) + K · φ′1 (x) + L · φ(x) = g(x),
temos
(a(x)f1′ (x)+b(x)f2′ (x))′ +K ·(a(x)f1′ (x)+b(x)f2′ (x))+L·(a(x)·f1 (x)+b(x)·f2 ) = g(x);
ou seja, (tiro x por falta de espaço)
(a′ f1′ + af1′′ + b′ f2′ + bf2′′ ) + K(af1′ + bf2′ ) + L · (af1 + bf2 ) = g(x)
que produz, já que f1 , f2 são soluções do problema homogêneo:
a′ (x)f1′ (x) + b′ (x)f2′ (x) = g(x).
Criamos asiim um sistema de equações lineares nas incógnitas a′ (x), b′ (x):
a′ (x)f1 (x) + b′ (x)f2 (x) = 0 e a′ (x)f1′ (x) + b′ (x)f2′ (x) = g(x)
cuja solução (regra de Cramer) é:
−f2 · g f1 · g
a′ (x) = e b′ (x) = .
f1 · f2′ − f2 · f1′ f1 · f2′ − f2 · f1′
E finalmente obtemos, integrando:
2Repare, à medida que for lendo, que o método funciona inclusive se houvessem coeficientes
variáveis:
f ′′ (x) + K(x) · f ′ (x) + L(x) · f (x) = g(x).
A diferença é que não sabemos resolver ainda essa equação homogênea. Mas se soubermos, o método
se aplica do mesmo modo.
CAPÍTULO 40. EQUAÇÕES DIFERENCIAIS DE SEGUNDA ORDEM 617

Z
−f2 · g
a(x) = dx
f1 · f2′ − f2 · f1′
Z
f1 · g
b(x) = dx.
f1 · f2′ − f2 · f1′
Pode surgir uma dúvida: será que o determinante (chamado Wronskiano)
W (f1 , f2 ) := f1 · f2′ − f2 · f1′
não se anula em algum ponto ?
Se pode provar que não, se f1 e f2 são linearmente independentes.
Por exemplo, no caso em que L = 1, se voltamos na Seção 2 e calculamos esse
determinante, encontramos:
• para K = 0,
W(f1 , f2 ) = sin2 (x) + cos2 (x) ≡ 1
• para 0 < |K| < 2,
1 √
W(f1 , f2 ) = · e−Kx · 4 − K 2 6= 0
2
• para K = ±2,
W(f1 , f2 ) = −e±2x 6= 0
• para |K| > 2,
W(f1 , f2 ) = (r2 − r1 ) · e(r1 +r2 )·x 6= 0

5. Um problema da Putnam Competition, n.58, 1987


Problema: Se a função y = f (x) satisfaz a equação:
f ′′ (x) − 2 · f ′ (x) + f (x) = 2 · ex ,
considere as duas questões a seguir sobre ela:
a): f (x) > 0 ∀x ∈ R implica que f ′ (x) > 0 ∀x ∈ R ? Prove isso ou explique
como produzir contra-exemplos.

b): f ′ (x) > 0 ∀x ∈ R implica que f (x) > 0 ∀x ∈ R ? Prove isso ou explique
como produzir contra-exemplos.

Solução:
A Seção anterior 4 nos explicou como achar as soluções explı́citas dessas equação.
Como as soluções do caso homogêneo f ′′ (x) − 2 · f ′ (x) + f (x) = 0 são
f (x) = a · x · ex + b · ex , a, b ∈ R,
e o determinante Wronskiano é −e2x , então a solução especial φ obtida por variação
de parâmetros é:
φ = a(x) · xex + b(x) · ex =
= 2x · x ex + x2 · ex = x2 · ex .
5. UM PROBLEMA DA PUTNAM COMPETITION, N.58, 1987 618

Logo f (x) é da forma:


f (x) = a · x · ex + b · ex + x2 · ex , a, b ∈ R.
Para responder ao item a) vou mostrar que, mesmo se f é sempre positiva, f ′ (x)
pode se anular, desde que:
a2 a2
<b< + 1,
4 4
por exemplo se a = 1 e b = 21 .
Para isso noto que:
f (x) = ex · (x2 + a · x + b)
e que
f ′ (x) = ex · (x2 + (2 + a) · x + a + b).
Então:
f (x) > 0 ∀x ⇔ x2 + a · x + b > 0 ∀x ⇔
a2
⇔ a2 − 4b < 0 ⇔ < b.
4
Enquanto que:
f ′ (x) = 0 ⇔ x2 + (2 + a) · x + a + b = 0 ⇔
a2
⇔ (2 + a)2 − 4(a + b) ≥ 0 ⇔ b ≤ + 1.
4
Já o item b) tem uma resposta afirmativa.
De fato, se f ′ (x) > 0 ∀x então:
a2
+ 1 < b.
4
Inicialmente mostro que f (x) 6= 0 ∀x. Depois mostro que de fato f (x) > 0 ∀x.
Se supomos que f (x) = 0 para algum x então
a2
b≤ .
4
Mas assim chegamos num absurdo:
a2 a2
+1<b≤ .
4 4
Então pelo Teorema do Valor Intermediário, ou bem f (x) > 0 ∀x (como queremos
provar) ou bem f (x) < 0 ∀x. Neste último caso, como
f (x) = a · x · ex + b · ex + x2 · ex , a, b ∈ R,
f (0) < 0 implica que b < 0. Mas isso produz a contradição:
a2
+ 1 < b < 0.
4
CAPÍTULO 40. EQUAÇÕES DIFERENCIAIS DE SEGUNDA ORDEM 619

6. Equação diferencial de um circuito elétrico simples


No circuito elétrico simples ilustrado na Figura há uma resistência de R ohms,
um capacitor com Capacitância de C faradays, uma indutância de L henrys, ao qual
se aplica uma tensão de E(x) volts (x é o tempo).
R C

Quando o circuito é fechado, a a carga de Q(x) coulombs no capacitor satisfaz a


equação diferencial
1
L · Q′′ (x) + R · Q′ (x) + Q(x) = E(x),
C
como consequência da lei de Kirchhoff.
Note que Q′ (x) = I(x) é a corrente que circula no sistema.
Trata-se do tipo de equação diferencial que sabemos resolver, após as Seções 2 e
4.
Lá simplificamos o problema para valores L = 1 (que sempre pode se obter di-
vidindo pot L 6= 0).
Mantendo a suposição L = 1, o discriminante da equação caracterı́stica (da eq.
homogênea) é:
1
r2 + R · r + = 0
C
torna-se
4
∆ = R2 − .
C
Num Exercı́cio no livro de Boyce-Di Prima (Seção 3.9, ex. 16, p.117) encontra-se
os valores:
L = 1, R = 5 × 103 , C = 0.25 × 10−6 e E(x) ≡ 12.
6 6
Nesse caso, ∆ = 25 × 10 − 16 × 10 > 0, r1 = −1000, r2 = −4000 e as soluções
do sistema são portanto da forma:
y = Q(x) = a · e−1000x + b · e−4000x + φ1 (x)
onde, conforme a Seção 4, a solução particular φ1 (x) do caso não homogêneo pode
ser tomada
φ1 (x) = a(x) · e−1000x + b(x) · e−4000x
onde (escolhendo as constantes de integração iguais a zero)
Z
−12 · e−4000x
a(x) = dx = 4 · ×10−6 · e1000x
−3000 · e−5000x
7. NÃO-HOMOGÊNEAS: MÉTODO DE COEFICIENTES A DETERMINAR 620

e Z
12 · e−1000x
b(x) = dx = −10−6 · e4000x
−3000 · e−5000x
Ou seja:
y = Q(x) = a · e−1000x + b · e−4000x + 3 × 10−6 .
Impondo que Q(0) = 0 e Q′ (0) = 0 obtemos:
a = −4 × 10−6 e b = 10−6
e finalmente
y = −4 × 10−6 · e−1000x + 10−6 · e−4000x + 3 × 10−6
e portanto
lim Q(x) = 3 × 10−6 .
x→+∞
ln(2)
A seguir plotei esta solução. Note um ponto de inflexão em x = 1500
≈ 0.000462.

2,5E-6

2E-6

1,5E-6

1E-6

5E-7

0E0
0 0,0005 0,001 0,0015 0,002 0,0025 0,003
x

7. Não-homogêneas: Método de coeficientes a determinar


O método de variação de parâmetros exposto na Seção é geral, para equações de
segunda ordem lineares não-homogêneas com qualquer tipo de coeficientes, constantes
ou não.
Mas tem em si uma dificuldade que é a de que devemos conseguir fazer integrações.
E pode ser que às vezes fiquem complicadas.
Já o método que será exposto aqui nesta Seção, apesar de só se aplicar a equações
de segunda ordem lineares não-homogêneas a coeficientes constantes:
y ′′ (x) + p · y ′ (x) + q · y(x) = R(x), p, q ∈ R
e ainda com R(x) funções bem particulares, é puramente algébrico, não envolve por-
tanto integração.
CAPÍTULO 40. EQUAÇÕES DIFERENCIAIS DE SEGUNDA ORDEM 621

Começo com a situação bem simples em que


R(x) = A · eλ·x , A, λ ∈ R, A, λ 6= 0.
Como as derivadas das exponencias são exponenciais, é natural pensar que em
buscar uma solução particular da forma:
φ1 (x) = C · eλ·x , C 6= 0.
Ora:
[C · eλ·x ]′′ + p · [C · eλ·x ]′ + q · C · eλ·x =
= [λ2 + p · λ + q] · C · eλ·x .
Então é natural considerar dois Casos:
Caso 1): λ não é raı́z da equação caracterı́stica r 2 + p · λ + q = 0
Caso 2): λ é raı́z da equação caracterı́stica r 2 + p · λ + q.
No Caso 1 queremos que
[λ2 + p · λ + q] · C · eλ·x = A · eλ·x
e portanto:
A
C= .
[λ2 + p · λ + q]
No Caso 2 o que temos é que
eλ·x
é solução do problema homogêneo:
y ′′ (x) + p · y ′ (x) + q · y(x) = 0
e não é isso que queremos aqui. Vamor ter que adotar outra estratégia3.
Está mais do que na hora de introduzir uma notação, para o operador diferencial
linear :
L(f ) := f ′′ + p · f ′ (x) + q · f (x).
O chamo de operador e não de função porque seu domı́nio são as funções duas vezes
deriváveis (e não números ou pontos) e sua imagem também são funções, não números
ou pontos. De diferencial porque faz derivadas e de linear porque:
L(a · f1 + b · f2 ) = a · L(f1 ) + b · L(f2 ).
Com essa notação, pensando em λ como sendo qualquer:
L(C · eλ·x ) = (λ2 + p · λ + q) · C · eλ·x .
Então tomando λ como variável e derivando nessa variável :
∂L(C · eλ·x )
= (2λ + p) · C · eλ·x + (λ2 + p · λ + q) · x · C · eλ·x .
∂λ
Como o operador L faz derivadas em x, o Lemma de Schwartz4 dá que:
∂L(C · eλ·x ) ∂eλ·x
= L(C · )=
∂λ ∂λ
= L(C · x · eλ·x ).
3Praticamente a mesma estratégia aparecerá na Seção 2 do Capı́tulo 44
4que diz que não importa a ordem de derivações se as funções tem segundas derivadas contı́nuas
7. NÃO-HOMOGÊNEAS: MÉTODO DE COEFICIENTES A DETERMINAR 622

Portanto, igualando os dois lados:


L(C · x · eλ·x ) = (2λ + p) · C · eλ·x + (λ2 + p · λ + q) · x · C · eλ·x .
Como no Caso 2:
λ2 + p · λ + q = 0
então no Caso 2):
L(C · x · eλ·x ) = (2λ + p) · C · eλ·x ,
desde que
2λ + p 6= 0.
λ·x
Se quero que C · x · e seja solução do problema
L(f ) = A · eλx
e se [2λ + p 6= 0 então quero que valha:
L(C · x · eλ·x ) = (2λ + p) · C · eλ·x = A · eλ·x ,
ou seja,
A
C=
2λ + p
dá a buscada solução particular.
Agora resta tratar o Sub-Caso do Caso 2, em que:
λ2 + p · λ + q = 2λ + p = 0,
que é o caso em que λ é raı́z dupla da equação caracterı́stica.
Note que nesta situação
x · eλ·x
é solução do problema homogêneo5
L(f ) = f ′′ + p · f ′ + q · f = 0.
Novamente considero λ como uma variável e derivo a expressão de acima:
∂L(C · eλ·x )
= (2λ + p) · C · eλ·x + (λ2 + p · λ + q) · x · C · eλ·x ,
∂λ
obtendo do lado esquerdo:
∂ 2 L(C · eλ·x ) ∂L(C · x · eλ·x )
= =
∂λ2 ∂r
∂(C · x · eλ·x )
= L( ) = L(C · x2 · eλ·x )
∂λ
enquanto que do lado direito obtenho:
∂((2λ + p) · C · eλ·x + (λ2 + p · λ + q) · x · C · eλ·x )
=
∂λ
= 2 · C · eλ·x + (2λ + p) · C · eλ·x [λ + x] + (λ2 + p · λ + q) · x · C · λ · eλ·x .
Avaliando para o λ tal que
λ2 + p · λ + q = 2 · λ + p = 0
5Bem de acordo com o que obtivemos no item 2 da Afirmação 2.1
CAPÍTULO 40. EQUAÇÕES DIFERENCIAIS DE SEGUNDA ORDEM 623

obtemos
L(C · x2 · eλ·x ) = 2 · C · eλ·x ,
e como quero:
L(C · x2 · eλ·x ) = A · eλ·x
concluo
A
C=
2
é o valor buscado para termos solução especial do problema não-homogêneo.
A mesma discussão se aplica ao caso mais geral, em que o problema não homogêneo
é:
L(f (x)) = f ′′ + p · f ′ + qf = A(x) · eλx ,
onde A(x) é polinômio de grau k.
Ou seja:
Afirmação 7.1. Se λ ∈ R não é raı́z de λ2 + p · λ + q = 0 encontraremos solução
especial do tipo:
g(x) · eλx ,
onde g(x) é polinômio de grau n, para o problema:
L(f (x)) = f ′′ + p · f ′ + q = A(x) · eλx ,
onde A(x) é também polinômio de grau n.
Se λ ∈ R é raı́z simples de λ2 + p · λ + q = 0 encontraremos solução do tipo:
g(x) · x · eλx .
Se λ ∈ R é raı́z dupla de λ2 + p · λ + q = 0 encontraremos solução do tipo:
g(x) · x2 · eλx .
Observe que o caso λ = 0 também está compreendido.
Demonstração.
A mesma discussão em Casos, só que agora não se trata de determinar 1 coeficiente
mas todos os coeficientes do polinômio g(x), que aparecem resolvendo um sistema de
equações lineares.


O mesmo tipo de resultado se obtêm se o termo não homogêneo R(x) da equação


f ′′ + p · f ′ + q · f = R(x)
é da forma
R(x) = eax cos(bx) ou R(x) = eax sin(bx),
com a ou b podendo ter o valor 0.
Ou seja, se buscará solução para o problema não-homogêneo na classe
y = c1 · eax cos(bx) + c2 · eax sin(bx),
8. SISTEMAS DE EQUAÇÕES DIFERENCIAIS 624

a menos que λ = a + I · b seja raı́z da equação caracterı́stica de f ′′ + p · f ′ + qf = 0.


Neste caso se busca solução para o prroblema não-homogêneo na classe
y = c1 · x · eax cos(bx) + c2 · x · eax sin(bx).
Por exemplo, f ′′ +f ′ +f = 0 tem √por raı́zes da equação caracterı́stica λ2 +λ+1 = 0
os valores complexos: λ = − 21 ± I · 23 . Logo para o problema
x
f ′′ + f ′ + f = e− 2
busco soluções na classe
x
y = c · e− 2 ;
de fato,
x x x x
(c · e− 2 )′′ + (c · e− 2 )′ + c · e− 2 = e− 2
dá
x 1 1 x
e− 2 · ( − + 1) · c = e− 2
4 2
e portanto c = 43 .
Mas para o problema

′′ ′ − x2 3
f +f +f =e · cos( x)
2
preciso recorrer à classe:
√ √
− x2 3 − x2 3
y = c1 · x · e · cos( x) + c2 · x · e sin( x).
2 2
A Seção 8 a seguir dá exemplos.

8. Sistemas de equações diferenciais


Se pode transformar uma equação diferencial de ordem maior num sistema de
equações diferenciais de ordem mais baixa, ou, vice-versa, um sistema de equações
numa equação de ordem mais alta.
Vejamos exemplos (exercı́cios do livro de Bear, Differential equations, a concise
course, Dover, pag. 164):

Exemplo 1:
y ′(t) = y(t) + z(t) e z ′ (t) = y(t) + z(t).
Então
y ′ (t) = z ′ (t)
e portanto, se t pertence a um Intervalo, temos:
z(t) = y(t) + C, C ∈ R.
A primeira equação dá então:
y ′ (t) = y(t) + z(t) = 2 · y(t) + C
CAPÍTULO 40. EQUAÇÕES DIFERENCIAIS DE SEGUNDA ORDEM 625

e portanto, como aprendemos na Seção 4.1 do Capı́tulo 35:


C
y(t) = D · e2·t − .
2
Então
C
z(t) = D · e2·t + .
2
Exemplo 2:
A equação de segunda ordem
y ′′ (t) + y(t) = 2 · et
vira o sistema:
y ′ (t) = z(t) e z ′ (t) = 2 · et − y(t)
e vice-versa.
Uma solução particular do do problema não-homogêneo
y ′′(t) + y(t) = 2 · ex
salta aos olhos:
φ1 (x) = et ,
mas mesmo que não fosse tão evidente nela chegarı́amos seguindo a Seção 7, que
ensina: como 1 não é raı́z da equação caracterı́stica λ2 + 1 = 0, obtemos uma solução
particular
2
φ1 (x) = 2 · et
1 +1
do problema não-homogêneo. E portanto a solução geral desse problema é:
y(t) = a · cos(t) + b · sin(t) + et .

Exemplo 3:
Considere o sistema:
y ′ (t) = y(t) + z(t) + t e z ′ (t) = 4 · y(t) + z(t) + t + 4 · et .
Da primeira equação:
z(t) = y ′(t) − y(t) − t logo z ′ (t) = y ′′ (t) − y ′(t) − 1,
que posto na segunda dá:
y ′′ (t) − y ′ (t) − 1 = 4 · y(t) + [y ′(t) − y(t) − t] + t + 4 · et ,
ou seja,
y ′′(t) − 2 · y ′(t) − 3 · y(t) = 1 + 4 · et .
Aqui o melhor é separarmos em duas equações
y1′′ (t) − 2 · y1′ (t) − 3 · y1 (t) = 1
y2′′(t) − 2 · y2′ (t) − 3 · y2 (t) = 4 · et
e a solução buscada será da forma:
y(x) = y1 (x) + y2 (x).
9. UM PROBLEMA DA PUTNAM COMPETITION, N.2, 1939 626

Ora, a equação
y1′′ (t) − 2 · y1′ (t) − 3 · y1 (t) = 1
tem uma solução particular constante:
1
φ1 (x) ≡ − ,
3
enquanto que a equação
y2′′(t) − 2 · y2′ (t) − 3 · y2 (t) = 4 · et
tem uma solução particular:
4
φ2 (x) = · et = −et ,
12 − 2 · 1 − 3
(seguindo a Seção 7, já que 1 não é raı́z de λ2 − 2 · λ − 3 = 0, cujas raı́zes são −1, 3).
Então a solução geral é:
1
y(t) = a · e−t + b · e3·t − − et .
3

O leitor não terá dificuldade em resolver:

9. Um problema da Putnam Competition, n.2, 1939

Problema:
Resolver o sistema de equações:
x′ (t) = x(t) + y(t) − 3 e y ′ (t) = −2 · x(t) + 3 · y(t) + 1,
com as condições iniciais:
x(0) = y(0) = 0.

Solução:
A primeira equação dá:
y(t) = x′ (t) − x(t) + 3, logo y ′ (t) = x′′ (t) − x′ (t).
E a segunda dá
x′′ (t) − x′ (t) = −2 · x + 3 · [x′ (t) − x(t) + 3] + 1,
ou seja,
x′′ (t) − 4 · x′ (t) + 5 · x = 10.
Uma solução particular óbvia dessa equaão não-homogênea é a solução constante:
φ1 (x) ≡ 2.
E como a equação caracterı́stica λ2 − 4 · λ + 5 = 0 do problema homogêneo
x′′ (t) − 4 · x′ (t) + 5 · x = 0
tem raı́zes compexas conjugadas

λ = 2± −1,
CAPÍTULO 40. EQUAÇÕES DIFERENCIAIS DE SEGUNDA ORDEM 627

a solução geral do problema não-homogêneo é:


x(t) = a · e2·t · cos(t) + b · e2·t · sin(t) + 2.
Usando que x(0) = 0 obtenho a + 2 = 0, ou seja, a = −2.
Sabemos que y(t) = x′ (t) − x(t) + 3; portanto após derivar x(t) se escreve y(t) =

x (t) − x(t) + 3 em função de b e t. A condição y(0) = 0 dará que b = 1.
Logo a solução do sistema é:
x(t) = −2 · e2·t · cos(t) + e2·t · sin(t) + 2,
y(t) = −e2·t · cos(t) + 3 · e2·t · sin(t) + 1.

10. Homogêneas, não-singulares, coeficientes variáveis: redução a


constantes
Considero agora a equação homogênea de segunda ordem:
f ′′ (x) + P (x) · f ′ (x) + Q(x) · f (x) = 0,
onde agora pelo menos um dos coeficientes P (x) e Q(x) é uma função não constante.
Em Matemática sempre se tenta reduzir um problema a outro conhecido. Por
isso impõe-se a pergunta: em que condições este problema pode ser reduzido ao tratado
na Seção 2 ?

A resposta é que se consegue isso apenas na situação a seguir. Que é claramente


bastante restritiva, mas por incrı́vel que pareça é suficiente para resolvermos a impor-
tante Equação de Euler (também chamada de equação de Cauchy-Euler), na Seção 1
do Capı́tulo 44.
Afirmação 10.1. Um equação
f ′′ (x) + P (x) · f ′ (x) + Q(x) · f (x) = 0 com Q(x) > 0, ∀x
pode ser transformada através de uma mudança de variável
z = z(x) ou x = x(z)
numa equação
f ′′ (z) + αf ′ (z) + βf (z), α, β ∈ R e β>0
se e somente se
Q′ (x) + 2P (x) · Q(x)
3 ≡ C, C ∈ R
2 · Q(x) 2
e ademais isso é feito através da mudança:
Z p
z= Q(x) dx.

Demonstração.
Uso a notação y = f (x) a seguir ou y = y(x) no que segue.
Primeiro tomo por hipóteses:
Z p
Q′ (x) + 2P (x) · Q(x)
3 ≡C e z= Q(x) dx.
2 · Q(x) 2
10. HOMOGÊNEAS, NÃO-SINGULARES, COEFICIENTES VARIÁVEIS:
REDUÇÃO A CONSTANTES 628

Noto que
y = y(z),
dz
p
pois dx
= Q(x) > 0 garante que z(x) é uma função inversı́vel. Ou seja, x determina
z e também z determina x univocamente. Por isso posso dizer que y = y(z) = y(x(z))
e que y = y(x) = y(z(x)).
Posso também derivar a composta em x:
y = y(z(x)),
obtendo:
dy dy dz
(z(x)) = (z(x)) · =
dx dz dx
dy p
= · Q(x).
dz
E agora com a regra da composta e do produto:
d2 y d2 y dz dz dy d2 z
(z(x)) = ( (z(x)) · ) · + (z(x)) · =
d2 x d2 z dx dx dz d2 x
d2 y p p dy Q′ (x)
= 2 (z(x)) · Q(x) · Q(x) + (z(x)) · p
dz dz 2 Q(x)
d2 y dy Q′ (x)
= (z(x)) · Q + (z(x)) · p .
d2 z dz 2 Q(x)
Então se obtêm:
d2 y dy
0≡ 2
(z(x)) + P (x) · (z(x)) + Q(x) · y =
d x dx
2 ′
dy Q + 2P Q dy
= Q(x) · 2 + ( √ )· + Q · y(z)
dz 2 Q dz
e como Q(x) 6= 0 se chega em:
d2 y Q′ + 2P Q dy
0= 2 +( 3 )· + y(z)
dz 2Q 2 dz
que tem coeficiente constante pela hipótese.
Para provar a recı́proca, note que, se uma mudança z = z(x) levou
f ′′ (x) + P (x) · f ′ (x) + Q(x) · f (x) = 0
em
f ′′ (z) + αf ′(z) + βf (z), α, β ∈ R
então
d2 y dy
0= 2
(z(x)) + P (x) · (z(x)) + y =
dx dx
2 2
d y dz dy d z dy dz
= [ 2 · ( )2 + · 2 ] + P (x) · ( · ) + Q · y(z(x)) =
d z dx dz d x dz dx
2 2
dz dy d z dz dy
= ( )2 · 2 + [ 2 + P (x) ] · + Qy(z) =
dx dz d x dx dz
CAPÍTULO 40. EQUAÇÕES DIFERENCIAIS DE SEGUNDA ORDEM 629
dz 2
e dividindo por ( dx ) 6= 0 (pois é uma mudança de coordenadas) obtemos
d z 2dz
d2 y d2 x
+ P dx dy Q
0= 2 +( dz 2
)· + dz 2 y(z),
dz ( dx ) dz ( dx )
ou seja,
d2 z dz
d2 x
+ P dx Q
α= dz 2
e β= dz 2
> 0.
( dx ) ( dx )
De onde, s
dz Q d2 z Q′
= e = q ,
dx β d2 x 2β · Qβ
ou seja:
p Q′ + 2P Q
α· β= 3 .
2Q 2


11. Homogêneas, não-singulares, coeficientes variáveis: Método de


D’Alembert
Aqui considero a equação:
y ′′ (x) + P (x) · y ′(x) + Q(x) · y(x) = 0
do qual suponho ter uma solução conhecida:
y = y1 (x).
O método de redução de ordem (de D’Alembert) nos dirá como achar uma segunda
solução y2 (linearmente independente) desta equação através da resolução de uma
equação de ordem menor, ou seja, de ordem 1.
Para isso ele propõe:
y2 (x) := a(x) · f1 (x)
com a(x) função duas vezes derivável não constante.
Queremos que:
y2′′ (x) + P (x) · y2′ (x) + Q(x) · y2 (x) = 0,
ou seja, que:
[a′′ (x)y1 (x)+2·a′ (x)·y1′ (x)+a(x)y1′′ (x)]+P (x)·[a′ (x)y1 (x)+a(x)y1′ (x)]+Q(x)a(x)y1 (x) = 0,
ou ainda, reordenando os termos:
a′′ (x)·y1(x)+a′ (x)·[2·y1′ (x)+P (x)y1(x)]+a(x)·[y1′′ (x)+P (x)·y ′(x)+Q(x)·y1(x)] = 0,
que resulta em
a′′ (x) · y1 (x) + a′ (x) · [2 · y1′ (x) + P (x)y1(x)] = 0,
pois y1 (x) é solução da equação.
12. EXISTÊNCIA DE SOLUÇÕES DE EQUAÇÕES HOMOGÊNEAS E
NÃO-SINGULARES 630

Fazendo
A(x) = a′ (x)
obtemos a redução de ordem, pois temos agora de resolver a equação de primeira
ordem:
A′ (x) · y1 (x) + A(x) · [2 · y1′ (x) + P (x)y1 (x)] = 0,
ou seja, se y1 (x) 6= 0,
A′ (x) −[2 · y1′ (x) + P (x)y1 (x)] y ′ (x)
= = −2 1 − P (x)
A(x) y1 (x) y1 (x)
e portanto Z
−2
ln |A(x)| = ln(y1 (x) ) − P (x)dx
e R
−2 )
A(x) = ±eln(y1 (x) · e− P (x)dx
,
ou seja, R
e− P (x)dx
A(x) = .
y1 (x)2
onde, na prática, a constante de integração pode ser tomada C = 0, já que só queremos
uma solução. E obteremos a(x) através de mais uma integração:
Z
a(x) = A(x) dx

(novamente a constante de integração pode ser tomada C = 0, já que só queremos
uma solução).

12. Existência de soluções de equações homogêneas e não-singulares


O seguinte teorema tem como alcance as equações tratadas na Seção 10:
Afirmação 12.1.
i): Considere
y ′′(x) + P (x) · y ′ (x) + Q(x) · y(x) = 0,
onde P (x) e Q(x) são funções contı́nuas.
As soluções foram um sistema linear a · y1 + b · y2 . Por isso, dados y(x0 ) e y ′(x0 )
existe e é única a solução y = y(x) da equação satisfazendo essas condições iniciais
para x ∈ I, um intervalo em torno de x0 .
ii): Considere
y ′′(x) + P (x) · y ′ (x) + Q(x) · y(x) = 0,
onde P (x) e Q(x) admitem expansão em série de potências, com raio de convergência
R1 e R2 , em torno de x0 . Seja R := min{R1 , R2 }.
Dados y(x0 ) e y ′(x0 ) existe e é única a solução y = y(x) da equação satisfazendo
essas condições iniciais e y(x) é uma série de potências cujo raio de convergência em
torno de x0 é pelo menos R.
CAPÍTULO 40. EQUAÇÕES DIFERENCIAIS DE SEGUNDA ORDEM 631

Observo que se P (x) ou Q(x) não são contı́nuos não se pode garantir que as
soluções sejam todas funções limitadas. Uma equação importante que exemplifica
isso é a Equação de Legendre (explicitamente resolvida na Seção 3 do Capı́tulo 41),
que pode ser escrita como:
2x n(n + 1)
y ′′ + 2 · y′ − 2 = 0, n ∈ N
x −1 x −1
Se x ∈ (−1, 1) então há soluções do tipo a · y1 + b · y2 , com y1 e y2 independentes. Mas
se pode provar que as únicas soluções limitadas da equação definidas em [−1, 1] são
múltiplos de Pn , o chamado n-ésimo polinômio de Legendre.

Idéia da prova da Afirmação 12.1:


Posso dar uma idéia de como provar a existência e unicidade de soluções, do item
i).
A idéia é transformar essa equação de segunda ordem num sistema de equações
de primeira ordem, fazendo:
z(x) := y ′(x)
e criando o sistema:
y ′(x) = z(x) e y(x0 ) = a
z ′ (x) = −P (x) · z(x) − Q(x) · y(x) e z(x0 ) = b
Agora a idéia é usar o Método de Picard (Seção 3 do Capı́tulo 36) para cada uma
dessas equações, ou seja, definindo recursivamente:
Z x
y0 ≡ a, yn := a + zn−1 (t)dt
x0
e Z x
z0 ≡ b, zn := b + (−P (t) · zn−1 (t) − Q(x) · yn−1 (t))dt
x0
Um Exemplo: suponha a equação y ′′ + y = 0 e o sistema associado a ela:
y ′(x) = z(x) e y(0) = 1
z ′ (x) = −y(x) e z(0) = 0
Então: Z x Z x
y1 := 1 + 0 dt = 1, z1 := 0 + −1 dt = −x,
0 0
Z x Z x
x2
y2 := 1 + −x dt = 1 − , z2 := 0 + −1 dt = −x,
0 2 0
Z x Z x
x2 x2 x3
y3 := 1 + −x dt = 1 − , z3 := 0 + −(1 − ) dt = − x,
0 2 0 2 3!
Z x 3 Z x
x x2 x4 x2 x3
y4 := 1 + − x dt = 1 − + , z4 := 0 + −(1 − ) dt = − x,
0 3! 2! 4! 0 2 3!
Z x 3
x x2 x4
y5 := 1 + − x dt = 1 − + ,
0 3! 2! 4!
13. PROPRIEDADES DAS SOLUÇÕES DE EQUAÇÕES LINEARES DE
SEGUNDA ORDEM 632
Z x
x2 x4 x3 x5
z5 := 0 + −(1 − + ) dt = −x + − ,
0 2! 4! 3! 5!
Z x
x3 x5 x2 x4 x6
y6 := 1 + (−x + − ) dt = 1 − + −
0 3! 5! 2! 4! 6!
e já reconhecemos que estão aparecendo os termos iniciais yn da séries de potências
de:
y(x) = cos(x)
e os termos iniciais zn da série de potências de
z(x) = − sin(x).
Deixo para mais tarde a segunda afirmação ii), sobre a natureza de séries conver-
gentes das soluções.

13. Propriedades das soluções de equações lineares de segunda ordem


Daremos nas Seções 1, 2 e 3 do Capı́tulo 41 soluções explı́citas, como séries de
potências das equações:
• de Airy 6:
y ′′ (x) + x · y(x) = 0.
• de Hermite:
y ′′(x) − 2 · x · y ′ (x) + q · y(x) = 0, q ∈ R.
• de Legendre
(1 − x2 ) · y ′′(x) − 2x · y ′ (x) + p · (p + 1) · y(x) = 0
Mas apesar do caráter explı́cito das soluções não ficará claro que tipo de pro-
priedades têm essas funções, por exemplo se têm um número finito ou infinito de
zeros, se oscilam.
Aqui nesta Seçã0 veremos que essas propriedades podem ser obtidas da própria
equação, sem se saber explicitamente a solução.
Afirmação 13.1. Um solução y(x) não-identicamente nula de
y ′′ + x · y = 0
tem:

i): no máximo um7 zero em (−∞, 0) e

ii): infinitos8 zeros em (0, +∞).


6Aparece na literatura também a equação y ′′ (x) − x · y(x) = 0 como sendo a Equação de Airy.
Na Seção 1 do Capı́tulo 41 comparo as soluções.
7É possı́vel provar também que não tem nenhum.
8É possı́vel provar que em cada região limitada [x , x ] ⊂ (0, +∞) só há um número finito de
0 1
zeros de y(x).
CAPÍTULO 40. EQUAÇÕES DIFERENCIAIS DE SEGUNDA ORDEM 633

Demonstração.

De i):

Suponha que exista algum x0 < 0 onde y(x0 ) = 0.


Se acontecer y ′ (x0 ) = 0 então o item i) da Afirmação 12.1 implicaria que y ≡ 0, a
solução trivial.
Por exemplo, penso de agora em diante que
y ′ (x0 ) > 0
(o outro caso y ′ (x0 ) < 0 é análogo).
Num pequeno intervalo denotado I + à direita de x0 então y(x) > 0. Como x < 0
em I + , então −x · y(x) > 0 em I + e
y ′′(x) = −x · y(x) > 0 em I + .
Logo a primeira derivada y ′(x) cresce em I + . E esse crescimento de y ′ (x) continua
enquanto tivermos x < 0 e y(x) > 0. Em particular enquanto tivermos x < 0 e
y(x) > 0 teremos y ′(x) > 0. Suponha por absurdo que num x1 com x0 < x1 < 0
tenhamos y(x1 ) = 0. Então por Rolle terı́amos y ′ (x2 ) = 0 para algum x2 com
x0 < x2 < x1 . Contradizendo o fato que y ′ (x2 ) > 0, pois x2 < 0 e y(x2 ) > 0.
Ou seja, que y(x) não volta a se anular à direita de x0 , enquanto tivermos x < 0.
Por outro lado, num pequeno intervalo denotado I − à esquerda de x0 temos y(x) <
0, já que supusemos y ′(x0 ) > 0.
Como x < 0 em I − , então −x · y(x) < 0 em I − e
y ′′(x) = −x · y(x) < 0 em I − .
Logo a primeira derivada y ′ (x) vinha decrescendo em I − até chegar no valor y ′ (x0 ) >
0. Ou seja que é sempre y ′ (x) > 0 à esquerda de x0 .
Isso impede que haja outro zero de y(x) à esquerda de x0 (use o Teorema de
Rolle).
De ii):

Suponha por absurdo que haja um ponto x0 ≥ 0 com a propriedade de que


y(x) 6= 0, ∀x > x0 .
Vamos mostrar que tem que haver um ponto x1 com x0 < x1 onde y(x1 ) = 0,
produzindo um absurdo.
Suponho de agora em diante que y ′ (x0 ) > 0 e que y(x) > 0 ∀x > x0 (os outros
casos são análogos).
Então
y ′′ = −x · y(x) < 0, ∀x > x0 .
Ou seja a derivada y ′ (x) é uma função decrescente para ∀x > x0 .
Afirmo que y ′ (x) < 0 em algum ponto x com x > x0 . Para provar isso, faço a
mudança:
y ′ (x)
v(x) = − , para x > x0 ,
y(x)
13. PROPRIEDADES DAS SOLUÇÕES DE EQUAÇÕES LINEARES DE
SEGUNDA ORDEM 634

que está bem definida pois y(x) > 0. E noto que v(x) verifica9:
v ′ (x) = x + v(x)2 .
Então: Z x Z x
v(x) − v(x0 ) = t dt + v(t)2 dt ≥
x0 x0
Z x
≥ t dt.
x0
Como Z +∞
lim v(x) ≥ v(x0 ) + t dt = +∞,
x→+∞ x0
para algum x > x0 tem que valer:
v(x) > 0.
Então
y ′(x)
0 < v(x) = − e y(x) > 0
y(x)
implicam que y ′ (x) < 0 como querı́amos.
Estamos na situação em que, para x > x0 vale:
y(x) > 0, y ′ (x) < 0 e y ′′ (x) = −x · y(x) < 0 ∀x ∈ (x, +∞).
Então o Exercı́cio (resolvido) 10.18 do Capı́tulo 11 diz que y(x) voltará a se anular
em algum ponto à direita de x: contradição.


O que usamos na prova da Afirmação 13.1 se adapta para dar uma prova da
Afirmação mais geral:
Afirmação 13.2. Seja uma equação y ′′ + Q(x) · y = 0, ∀x ∈ R, onde Q(x) é uma
função contı́nua.
No que segue só considero soluções y(x) dessa equação que não são identicamente
nulas.
i) se Q(x) < 0 em I ⊂ R então y(x) tem no máximo um zero em I.
ii) se Q(x) > 0 em J ⊂ (0 + ∞) e se
Z +∞
Q(x) dx = +∞
0
então y(x) tem uma infinidade de zeros na semireta x > 0
iii) se Q(x) > 0 em J ⊂ (−∞, 0) e se
Z 0
Q(x) dx = +∞
−∞

então y(x) tem uma infinidade de zeros na semireta x < 0


9Uma equação de primeira ordem não-linear, chamada Equação de Riccati, que será discutida
em detalhe no Capı́tulo 45
CAPÍTULO 40. EQUAÇÕES DIFERENCIAIS DE SEGUNDA ORDEM 635

Demonstração.
Os itens i) e ii) são provados exatamente do mesmo jeito que provamos a Afirmação
13.1, já que as propriedades da função y = x que usamos naquela prova também são
propriedades da função y = Q(x).
Mas o item ii) exige uma pequena adaptação.
Tomamos um x0 < 0 que seja menor que o menor zero de y(x) (por absurdo).
Podemos supôr que sempre y(x) > 0 à esquerda de x0 (análogo se for sempre
negativa)
Precisamos mostrar que há algum ponto x < x0 onde y ′(x) > 0. Feito isso, como
y ′′(x) = −Q(x) · y(x) < 0
à esquerda de x0 , então o gráfico é côncavo para baixo no intervalo à esquerda de x0
e uma adaptação imediata do Exercı́cio 10.18 do Capı́tulo 11 dirá que y(x) volta a se
anular à esquerda de x0 (absurdo).
Mas fazendo:
y ′ (x)
v(x) = − , para x < x0 ,
y(x)
v(x) verifica
v ′ (x) = Q(x) + v(x)2 .
Portanto para x < x0 < 0:
Z x0 Z x0
v(x0 ) − v(x) = Q(t) dt + v(t)2 dt ≥
x x
Z x0
≥ Q(t) dt.
x
Como Z x0
lim −v(x) ≥ −v(x0 ) + Q(t) dt = +∞,
x→−∞ −∞
para algum x < x0 tem que valer:
v(x) < 0.
Então
y ′(x)
0 > v(x) = − e y(x) > 0
y(x)

implicam que y (x) > 0 como querı́amos.


14. Um problema da Putnam Competition, n. 15, 1955


Com a Afirmação 13.2 fica fácil fazer o seguinte:

Problema:
Considere a função y = f (x) solução de
f ′′ (x) = (x3 + a · x) · f (x), a ∈ R,
14. UM PROBLEMA DA PUTNAM COMPETITION, N. 15, 1955 636

com f (0) = 1 e f ′ (0) = 0.


Prove que f tem infinitos zeros à esquerda de algum K ∈ R e um número finito
à direita de algum L ∈ R.

Solução:
As condição f (0) = 1 já garante que y = f (x) não é identicamente nula.
Vou considerar três casos:

Caso 1): a = 0.
Neste caso
f ′′ (x) − x3 · f (x) = 0,
e Q(x) := −x3 < 0 em (0, +∞). Portanto a a Afirmação 13.2 garante que há no
máximo um zero à direita de K = 0. E também que há infinitos à esquerda de L = 0,
pois claramente
Z 0
−x3 dx = +∞
−∞

Caso 2): a > 0.


Neste caso
f ′′ (x) − (x3 + a · x) · f (x) = 0,
e
Q(x) := −x3 − a · x = −x · (x2 + a).
Ora, Q(x) < 0 se x > 0 e Q(x) > 0 se x < 0. Ademais,
Z 0
−x3 − a · x dx = +∞
−∞

Portanto as conclusões são as mesmas do Caso 1).

Caso 3): a < 0.


Neste caso também Q(x) := −x3 − a · x = −x · (x2 + a).
Agora Q(x) < 0 se x > 0 e√x2 > −a ou √ se x < 0 e x2 < −a.
Ou seja, Q(x) < 0 se x > −a ou se − −a < x < 0. √
Posso então
√ dizer que Q(x) < 0 se x está à direita de K := −a e portanto à
direita de −a há um número finito de√ zeros. √
Por outro lado, Q(x) > 0 se x < − −a ou se 0 < x < −a. √
Posso então dizer√que Q(x) > 0 se x está à esquerda de L := − −a e portanto
que à esquerda de − −a há um número infinito de zeros, já que:
Z 0
−x3 − a · x dx = +∞.
−∞

A Afirmação 13.2 mostra sua força quando combinada com a seguinte técnica para
eliminar o termo em y ′:
CAPÍTULO 40. EQUAÇÕES DIFERENCIAIS DE SEGUNDA ORDEM 637

Afirmação 14.1. Suponha que a função y(x) é solução de


y ′′ (x) + P (x) · y ′(x) + Q(x) · y(x) = 0
Suponha que uma mudança da forma:
y(x) = u(x) · v(x), onde u(x) 6= 0,
faça de v(x) a solução de uma equação da forma:
v ′′ (x) + S(x) · v(x) = 0.
Então R
1
u(x) = e− 2 P (t) dt

e de fato
′′ P 2 (x) P ′(x)
v (x) + (Q(x) − − ) · v(x) = 0.
4 2
−1
R
Em particular, como e 2 · P (t) dt > 0, o estudo dos zeros de y(x) se reduz ao estudo
dos zeros de v(x), que poder ser feito pela Afirmação 13.2
Demonstração.
Se faço
y(x) = u(x) · v(x)
então:
0 = y ′′ (x) + P (x) · y ′ (x) + Q(x) · y(x) =
= (u′′ + 2u′ · v ′ + u · v ′′ ) + P (x) · (u′ · v + u · v ′ ) + Q(x) · (u · v) =
= u · v ′′ + (2 · u′ + P (x) · u) · v ′ (x) + (u′′ + P (x) · u′ + Q(x) · u) · v(x).
Como quero eliminar o termo em v ′ , quero que:
2 · u′ (x) + P (x) · u(x) = 0
ou seja, para u(x) 6= 0:
u′ (x) 1
= − · P (x)
u(x) 2
e R
1
u(x) = e− 2 P (t) dt
.
Logo, substituindo acima esse u(x):
1
R 1 P ′(x)
0 = e− 2 P (t) dt
· [v ′′ (x) + (Q(x) − P 2(x) − ) · v(x)]
4 2
e portanto
1 P ′ (x)
v ′′ (x) + (Q(x) − P 2 (x) − ) · v(x) = 0.
4 2

15. O TEOREMA DE COMPARAÇÃO DE STURM 638

15. O Teorema de Comparação de Sturm


Afirmação 15.1. (Teorema de Comparação de Sturm)
Sejam z(x) uma solução de
z ′′ (x) + Q(x) · z(x) = 0
e y(x) uma solução não identicamente nula de
y ′′(x) + q(x) · y(x) = 0,
onde
Q(x) > q(x).
Então no intervalo aberto entre cada dois zeros sucessivos de y(x) há pelo menos
um zero de z(x).
Demonstração.
Sejam x0 , x1 dois zeros sucessivos da solução y(x). Por absurdo suponho que z(x)
não tem zeros em (x0 , x1 ) (pode aconetcer que z(x0 ) = 0 ou z(x1 ) = 0).
Posso supôr que as soluções z(x) e y(x) têm o mesmo sinal em (x0 , x1 ) (se não
multiplico uma por −1, já que isso não afeta os zeros).
Por exemplo, y, z > 0 em (x0 , x1 ). Também posso supor que
y ′(x0 ) > 0 enquanto que y ′(x1 ) < 0
(pois entre zeros sucessivos de y(x) há algum zero de y ′ (x) - Teorema de Rolle). Note
que se y ′(x0 ) = 0 ou y ′ (x1 ) = 0 então y ≡ 0 pelo Teorema de Existência e Unicidade.
Defino:
z(x)y ′ (x) − y(x)z ′ (x)
e noto que
[z(x)y ′ (x) − y(x)z ′ (x)]′ (x) = z(x)y ′′ (x) − y(x)z ′′ (x).
Então:
[z(x1 ) · y ′(x1 ) − z ′ (x1 ) · y(x1 )] − [z(x0 ) · y ′(x0 ) − z ′ (x0 ) · y(x0 )] =
Z x1
= (zy ′ − yz ′ ) ′ (t) dt =
Z x1 x0
= (z(t)y ′′ (t) − y(t)z ′′ (t)] dt =
Z x1x0
= y(t) · z(t) · (Q(t) − q(t)) dt > 0,
x0
ou seja,
z(x1 ) · y ′ (x1 ) − z ′ (x1 ) · y(x1 ) > z(x0 ) · y ′ (x0 ) − z ′ (x0 ) · y(x0 ).
Mas, quando calculo, obtenho:
z(x0 ) · y ′ (x0 ) − z ′ (x0 ) · y(x0 ) = z(x0 ) · y ′ (x0 ) ≥ 0,
z(x1 ) · y ′ (x1 ) − z ′ (x1 ) · y(x1 ) = z(x1 ) · y ′ (x1 ) ≤ 0,
uma contradição.

CAPÍTULO 40. EQUAÇÕES DIFERENCIAIS DE SEGUNDA ORDEM 639

16. Um problema da Putnam Competition, n. 22, 1961


Adaptando um pouco o que fizemos na prova da Afirmação 15.1 é possı́vel resolver:

Problema:
Seja y(x) uma solução de

y ′′(x) + (1 + x) · y(x) = 0, ∀x ≥ 0
com y(0) = 1 e y ′(0) = 0.
Prove que y(x) se anula exatamente uma vez em (0, π2 ). Determine também um
número K para que o zero x de y(x) verifique:
π
0<K<x< .
2
Solução:
Vou comparar √
y ′′ (x) + (1 + x) · y(x) = 0, x ≥ 0
com
w ′′ + w = 0,

pois para x > 0 temos 1 + x > 1.
Desta última equação tomo a solução w(x) = cos(x), para a qual sabemos que
w(0) = 1, w ′(0) = 0 e que seu primeiro zero é o ponto π2 , onde w ′( π2 ) = −1.
Considero:
y(x) · w ′ (x) − w(x) · y ′(x).
Então:
y(0) · w ′(0) − w(0) · y ′(0) = 0
π π π π π
y( ) · w ′ ( ) − w( ) · y ′( ) = −y( ).
2 2 2 2 2
π
Suponha por absurdo que y(x) não tem zero em (0, 2 ).
Então
π
−y( ) < 0.
2
Mas como fizemos na prova da Afirmação 15.1:
π π π π
0 > [y( ) · w ′ ( ) − w( ) · y ′ ( )] − [y(0) · w ′ (0) − w(0) · y ′ (0)] =
2 2 2 2
Z π Z π
2
′′ ′′
2 √
= (y(t)w (t) − w(t)y (t)] dt = y(t) · w(t) · t dt > 0,
0 0
uma contradição.
Seja então
π
0 < x0 <
2
um zero de y(x).
Para descobrir o número K < x0 , comparo a equação:
r
′′ π
v (x) + (1 + ) · v(x) = 0
2
16. UM PROBLEMA DA PUTNAM COMPETITION, N. 22, 1961 640

com √
y ′′ (x) + (1 + x) · y(x) = 0,
π
pois para 0 ≤ x < 2
temos:
r
π √
1+ > 1 + x.
2
′′

A solução de v (x) + (1 + 2 ) · v(x) = 0 da forma
s r
π
v(x) = cos( 1 + · x)
2
tem
v(0) = 1 e v ′ (0) = 0.
Suponha por absurdo que seu primeiro zero
π 1
x := · q p ,
2 1+ π 2

verifica:
x0 < x.
Como
v(x0 ) · y ′(x0 ) − y(x0 ) · v ′ (x0 ) = v(x0 ) · y ′ (x0 ) < 0
e
v(0) · y ′(0) − y(0) · v ′ (0) = 0
obtenho
0 > [v(x0 ) · y ′(x0 ) − y(x0 ) · v ′ (x0 )] − [v(0) · y ′(0) − y(0) · v ′ (0)] =
Z x0 Z x0 r
′′ ′′ π √
= (v(t)y (t) − y(t)v (t)] dt = v(t) · y(t) · ( − t) dt > 0,
0 0 2
uma contradição.
Logo
π 1 π
0 < K := · q < x0 < .
2 1+ π
p 2
2

Falta ainda ver que só há esse zero x0 de y(x) em (K, π2 ).
Suponha por absudo que existe x′0 outro zero de y(x) em (K, π2 ).
Então a Afirmação 15.1 diz que há algum zero da solução v(x) de
r
′′ π
v (x) + (1 + ) · v(x) = 0
2
no intervalo:
(x0 , x′0 ) se x0 < x′0
ou
(x′0 , x0 ) se x′0 < x0 .
De qualquer forma, seria uma solução v(x) com algum zero entre K e π2 .
CAPÍTULO 40. EQUAÇÕES DIFERENCIAIS DE SEGUNDA ORDEM 641

Mas, depois de K o próximo zero de v(x) está em


3π 1
·q p ,
2 1 + π2
que é um número maior que π2 . Uma contradição.
17. Exercı́cios
Exercı́cio 17.1. (resolvido)
O estudante Fábio Casula criou o seguinte exercı́cio, que é simples mas instrutivo.
Resolva por série de potências na origem a equação:
xy ′ − y = 0.
Explique por que não há unicidade das soluções com y(0) = 0.
Exercı́cio 17.2. (resolvido) P
Resolva por série de potências y = +∞ π n
n=0 an (x − 2 ) o problema
π π
y ′′ + y = 0, y( ) = 1 e y ′( ) = 1.
2 2
Mostre que a solução assim obtida coincide com y = sin(x).
Exercı́cio 17.3. (resolvido)
Para x > 0, considere a equação:
2 q
y ′′ (x) + y ′(x) + α y(x) = 0.
x x

i ) Mostre que a mudança de variável


v(x)
y(x) =
x
transforma-a numa equação do tipo:
v ′′ (x) + Q(x) v(x) = 0
(determine Q(x)).

ii) Considere
2 ′
y ′′ (x) + y (x) + q y(x) = 0, com q < 0
x
(ou seja, α = 0).
Dê a solução geral da equação correspondente
v ′′ (x) + Q(x) v(x) = 0
e daı́ obtenha a solução geral de
2 ′
y ′′ (x) + y (x) + q y(x) = 0.
x
CAPı́TULO 41

Equações com pontos não-singulares: Airy, Hermite e


Legendre

1. Solução explı́cita da Airy


.
De acordo com o item ii) da Afirmação 12.1 do Capı́tulo 40, as soluções da equação
de Airy:
y ′′ (x) + x · y(x) = 0.
devem ser séries convergentes ∀x ∈ R:
+∞
X
y= ai · xi .
i=0

Então, derivando termo a termo1:


+∞
X

y = i · ai · xi−1 ,
i=1
+∞
X
y ′′ = i · (i − 1) · ai · xi−2
i=2
e, supondo que resolve a equação, temos:
+∞
X +∞
X
i−2
i · (i − 1) · ai · x + ai · xi+1 = 0,
i=2 i=0
ou seja, introduzindo um ı́ndice novo no somatório:
+∞
X
2 · a2 + [(j + 2)(j + 1) · aj+2 − aj−1 ] · xj = 0.
j=1

Portanto sobre a0 e a1 não há qualquer restrição, mas:


a0 a1
a2 = 0, a3 = , a4 = , a5 = 0,
2·3 3·4
a3 a0 a4 a1
a6 = = , a7 = = ,
5·6 2·3·5·6 6·7 3·4·6·7
a6 a0
a8 = 0, a9 = = ,
8·9 2·3·5·6·8·9
a7 a1
a10 = =
9 · 10 3 · 4 · 6 · 7 · 9 · 10
1como se pode justificar
643
1. SOLUÇÃO EXPLÍCITA DA AIRY 644

etc, (supondo que se possa reagrupar à vontade as parcelas).


Uma análise mais detalhada mostra que:
a1
a3k = , k ∈ N.
(2 · 3)(5 · 6) . . . ((3k − 1)(3k))

a0
a3k+1 = , k ∈ N.
(3 · 4)(6 · 7) . . . ((3k)(3k + 1))

a3k+2 = 0, k = 0, 1, 2, . . .
Portanto se obtém:
+∞ +∞
X x3k X x3k+1
y = a0 ·(1+ )+a1 ·(1+ )
k=1
(2 · 3)(5 · 6) . . . ((3k − 1)(3k)) k=1
(3 · 4)(6 · 7) . . . ((3k)(3k + 1))

O teste da Razão dá para a primeira série:

|x3 |
lim = 0,
k→+∞ (3(k + 1) − 1)(3(k + 1)

ou seja que há convergência em módulo ∀x ∈ R.


Para terminar, um esclarecimento sobre a equação de Airy, que na literatura
aparece às vezes com sinais diferentes:

Afirmação 1.1. Se y = y(x) é solução de y ′′(x) + x · y(x) = 0, ∀x ∈ R então

f (x) := y(−x)

é solução de
f ′′ (x) − x · f (x) = 0, ∀x ∈ R,
Ou seja, a solução de uma equação é dada como reflexão no eixo dos y da solução
da outra.

Demonstração.
Se y ′′ (x) + x · y(x) = 0, ∀x ∈ R então em particular:

y ′′ (−x) + (−x) · y(−x) = 0, ∀x ∈ R.

Mas se f (x) := y(−x) então f ′ (x) = −y ′ (−x) e

f ′′ (x) = −(−y ′′ (−x)) = y ′′ (−x).

Logo f ′′ (x) − x · f (x) = 0, ∀x ∈ R.



CAPÍTULO 41. EQUAÇÕES COM PONTOS NÃO-SINGULARES: AIRY,
HERMITE E LEGENDRE 645

2. Solução explı́cita da Hermite


Considero a Equação de Hermite
y ′′(x) − 2 · x · y ′ (x) + q · y(x) = 0, q ∈ R,
para a qual busco soluções da forma:
+∞
X
y= ai · xi
i=0

e que devem ser convergentes ∀x, pelo item ii) da Afirmação 12.1 do Capı́tulo 40.
Então, derivando termo a termo2:
+∞
X

y = i · ai · xi−1 ,
i=1
+∞
X
y ′′ = i · (i − 1) · ai · xi−2
i=2
e, supondo que resolve a equação, temos:
+∞
X +∞
X +∞
X
0= i · (i − 1) · ai · xi−2 − 2 · x · i · ai · xi−1 + q · ai · xi =
i=2 i=1 i=0
X
=: bi · xi .
i=0
onde
b0 = 2 · a2 + 2 · q · a0 , b1 = 2 · 3 · a3 − 2 · a1 + 2 · q · a1
b2 = 3 · 4 · a4 − 4 · a2 + 2 · q · a2 , b3 = 4 · 5 · a5 − 2 · 3 · a3 + 2 · q · a3
b4 = 5 · 6 · a6 − 2 · 4 · a4 + 2 · q · a4
etc (supondo que se possa reagrupar à vontade as parcelas). 10
Mas se pode mostrar que uma série é identicamente nula se e só se cada coeficiente
é nulo, quer dizer,
∀i, bi = 0.
O que cria as relações:
1−q
a2 = −q · a0 , a3 = · a1
3
2−q 2 · q · (2 − q)
a4 = · a2 = · a0
6 12
2 · (3 − q) 2 · (1 − q) · (3 − q)
a5 = · a3 = · a1
4·5 3·4·5
etc.
Uma análise mais cuidadosa permite mostrar que de fato as relações são:
2i · q · (q − 2) · (q − 4) . . . · (q − 2i + 2)
a2i = , se i ≥ 1,
(2i)!
2como se pode justificar
2. SOLUÇÃO EXPLÍCITA DA HERMITE 646

2i · q · (q − 1) · (q − 3) . . . · (q − 2i + 1)
a2i+1 = , se i ≥ 1.
(2i + 1)!
De novo supondo que se pode reagrupar termos à vontade, escrevo então o que
obtivemos como:
X X X
y= ai · xi = a2i · x2i + a2i+1 · x2i+1 .
i=0 i=0 i=0

Podemos confirmar a convergência dessas séries para todo R.


Note que o Teste da Razão aplicado para
X
a2i · x2i
i=0

dá
|a2(i+1) x2(i+1) | |2 · q · (q − 1) · . . . · (q − 2i)x2 |
lim = lim = 0,
i→+∞ |a2i x2i | i→+∞ |(2i + 2) · (2i + 1) · q · (q − 1) · . . . · (q − 2i + 1)|

ou seja que converge emPmódulo ∀x ∈ R.


Analogamente para i=0 a2i+1 · x2i+1 .
Duas observações:
• Se
q = 0 ou q = n ∈ N
então ou X
a2i · x2i
i=0
é um polinômio (quando q = 0 ou q = n ∈ N é par) ou
X
a2i+1 · x2i+1
i=0

é um polinômio (quando q = n é ı́mpar).


Como se verifica, esses polinômios são:
a0 , se q = n = 0
a1 · x, se q = n = 1
a0 − 2 · a0 · x2 , se q = n = 2
2
a1 · x − · a1 · x3 , se q = n = 3
3
etc.
• Para q geral, pode-se escrever
X X
y= a2i · x2i + a2i+1 · x2i+1 =
i=0 i=0

2 · q · (q − 1) 3
= a0 · (1 − 2 · q · x2 + . . .) + a1 · (x − · x + . . .)
3
para pôr em evidência que há duas soluções independentes da equação cujas
combinações lineares dão a solução geral.
CAPÍTULO 41. EQUAÇÕES COM PONTOS NÃO-SINGULARES: AIRY,
HERMITE E LEGENDRE 647

3. Solução explı́cita da Legendre em torno de x = 0


A equação de Legendre é
2x p · (p + 1)
y ′′(x) − · y ′
(x) + · y(x) = 0, p∈R
1 − x2 1 − x2
é não-singular3 em x = 0.
Essa equação também pode ser escrita como:
(1 − x2 ) · y ′′(x) − 2x · y ′(x) + p · (p + 1) · y(x) =
e, às vezes, em aplicações, aparece numa forma camuflada:
((1 − x2 ) · y ′(x))′ + λ · y(x) = 0.
De acordo com o item ii) da Afirmação 12.1 do Capı́tulo 40, esta equação tem
soluções dadas por séries de potências convergentes em −1 < x < 1 (eventualmente
polinômios, dependendo de p especı́ficos), pois:
+∞
1 X
= x2n , se − 1 < x < 1.
1 − x2 n=0

Tomo um candidato a solução


+∞
X
y= cn · xn ,
n=0
calculo cada ingrediente da equação de Legendre posta na forma:
(1 − x2 ) · y ′′(x) − 2x · y ′ (x) + p · (p + 1) · y(x) = 0
e os reúno na equação; ou seja, faço:
+∞
X +∞
X
′ n−1
−2x · y = −2x · n · cn · x = [−2n · cn ] · xn ,
n=1 n=1
+∞
X
(1 − x2 ) · y ′′ = (1 − x2 ) · n(n − 1) · cn · xn−2 =
n=2
+∞
X +∞
X
= n(n − 1) · cn · xn−2 − n(n − 1) · cn · xn .
n=2 n=2
Pondo-os juntos na equação de Legendre e reagrupando os termos em ordem crescente
do expoente, obtemos:
[2 · 1 · c2 + p(p + 1)c0 ] · x0 + [3 · 2 · c3 − 2 · 1 · c1 + p(p + 1) · c1 ] · x1 +
+[4·3·c4 −2·1·c2 −2·2·c2 +p(p+1)·c2 ]·x2 +[5·4·c5 −3·2·c3 −2·3·c3 +p(p+1)c3 ]·x3 +. . . +
+[(n + 2) · (n + 1) · cn+2 − (n − 1) · n · cn − 2 · n · cn + p(p + 1) · cn ] · xn + . . . = 0,
de onde sai que:
(n + 2) · (n + 1) · cn+2 − (n − 1) · n · cn − 2 · n · cn + p(p + 1) · cn = 0, ∀n ≥ 0;
3Por outro lado, do ponto de vista do Capı́tulo 44 ela tem pontos singulares em x = 1 e x = −1
3. SOLUÇÃO EXPLÍCITA DA LEGENDRE EM TORNO DE X = 0 648

ou seja, surgem as recorrências:


(n − 1) · n + 2 · n − p(p + 1)
cn+2 = · cn =
(n + 2) · (n + 1)

n · (n + 1) − p(p + 1)
= · cn , ∀n ≥ 0,
(n + 2) · (n + 1)
que nos permitirão, dado c0 obter todos os ck com k pares4 e dado c1 obter todos os
cj com j ı́mpares (como descrito mais em detalhe abaixo).
E assim
+∞
X X X
y= cn · xn = c0 · ck xk + c1 · cj xj
n=0 k∈2N j∈2N+1

descreve o sistema linear de dimensão dois das soluções da equação diferencial.


Uma observação simples mas interessante é que as recorrências acima podem ser
re-escritas como:
n · (n + 1) − p(p + 1) (p + n + 1) · (p − n)
cn+2 = · cn = − · cn .
(n + 2) · (n + 1) (n + 2) · (n + 1)
Ou seja,
(p + 1) · p (p + 3)(p − 2) (p + 1) · p
c2 = − · c0 , c4 = · · c0 ,
2·1 4·3 2·1

(p + 5) · (p − 4) (p + 3)(p − 2) (p + 1) · p
c6 = − · · · c0 ,
6·5 4·3 2·1
e assim por diante. P
Isso nos indica que se p ∈ 2N é um Natural par então a série k∈2N ck xk fica
truncada no grau p, ou seja, vira um polinômio Pp , e:
X
y = c0 · P p + c1 · cj xj .
j∈2N+1
P j
Enquanto que no caso em que p ∈ 2N +1 é um Natural ı́mpar é a série j∈2N+1 cj x
que fica truncada no grau p, ou seja, vira um polinômio Pp de grau p e
X
y = c0 · ck + c1 P p .
k∈2N

Esse polinômios Pp que são soluções da equação de Legendre são chamados polinômios
de Legendre e são muito importantes na resolução de Equações Parciais, por exem-
plo. Veremos na Seção 4 do Capı́tulo 48 que os polinômios de Legendre devem ser
considerados harmônicos esféricos.

4 Denoto o conjunto dos pares por e 2N e dos ı́mpares por 2N + 1


CAPÍTULO 41. EQUAÇÕES COM PONTOS NÃO-SINGULARES: AIRY,
HERMITE E LEGENDRE 649

4. Polinômios de Legendre e expansão em série do potencial gravitacional


Os polinômios de Legendre são a base para as adaptações da teoria de atração
gravitacional de Newton - que a princı́pio é para um objeto pontual, zero dimensional
- para situações realı́sticas, em que os objetos que atraem tem diferentes formatos
tridimensionais.
Me contento aqui em indicar (sem dar uma prova completa por enquanto) como os
polinômios de Legendre aparecem em expansões em séries do potencial Newtoniano.
Seja um corpo pontual de massa M situado fora da origem, no ponto (a, b, c) do
espaço e seja

D = ||(a, b, c)|| = a2 + b2 + c2 .
Seja um outro corpo pontual de massa m << M situado em (x, y, z) e
p
d = ||(x, y, z)|| = x2 + y 2 + z 2 .
Seja
p
r= (x − a)2 + (y − b)2 + (z − c)2
a distância entre m e M.
Uma verificação imediata comprova que
∂( 1r ) ∂( 1r ) ∂( 1r ) −1
( , , ) = 3 · (x − a, x − b, x − c),
∂x ∂y ∂z r
o que significa que
GM
U=
r
é o potencial Newtoniano que produz a atração gravitacional:
GM (x − a, y − b, z − c)
− · ,
r2 r
Suponhamos agora que
d
0 < v :=
<1
D
ou seja que m está situado mais próximo da origem que M.
ˆ
No triângulo formado pela origem O e mais m e M, seja θ o ângulo mOM; a lei
dos cossenos (cf. Seção 3 do Capı́tulo 17) dá:
r 2 = D 2 + d2 − 2 · d · D cos(θ),
portanto
p p
r= D 2 + (vD)2 − 2 · vD · D cos(θ) = D · 1 + v 2 − 2v cos(θ)
e
1
U = GM · p .
D· 1+ v2 − 2v cos(θ)
Enquanto tivermos
|v 2 − 2v cos(θ)| < 1
5. ORTOGONALIDADE DOS POLINÔMIOS DE LEGENDRE 650
−1
podemos usar a série binomial com expoente 2
(cf. Seção 4 do Capı́tulo 31) e obter:
1 GM 1
U = GM · p = · (1 + v 2 − 2v cos(θ))− 2 =
D· 1+ v2 − 2v cos(θ) D
GM 1 1·3 2 1·3·5 2
= · [1 − (v 2 − 2v cos(θ)) + (z − 2v cos(θ))2 − (v − 2v cos(θ))3 + . . .]
D 2 2·4 2·4·6
Se re-escrevemos essa série como série de potências em v temos:
GM 1 3 3 5
U= · [1 + cos(θ) · v + (− + cos(θ)2 ) · v 2 + (− cos(θ) + cos(θ)3 ) · v 3 + . . .] =
D 2 2 2 2
+∞
GM X
= · Pn (cos(θ)) · v n .
D n=0
Temos:
1 3
1 = P0 (cos(θ)), cos(θ) = P1 (cos(θ)), − + cos(θ)2 = P2 (cos(θ)),
2 2
3 5
− cos(θ) + cos(θ)3 = P3 (cos(θ))
2 2
e o que se pode provar é que cada Pn é o polinômio de Legendre de grau n.
Noto que, para θ = 0:
−1 −1 −1
(1 + v 2 − 2v cos(0)) 2 = (1 + v 2 − 2v) 2 = (1 − v)2 2
= (1 − v)−1
e pela série geométrica (já que 0 < v < 1):
+∞
X
−1
(1 − v) = vn
n=0

o que é coerente com a escolha que se faz dos coeficientes dos Pn para que
Pn (1) = 1, ∀n ≥ 0.

5. Ortogonalidade dos polinômios de Legendre


Retomemos a equação de Legendre na forma:
((1 − x2 ) · y ′ (x))′ + λ · y(x) = 0
efaçamos:
λ = n · (n + 1), n∈N
para que tenha soluções polinomiais Pn (n-ésimo polinômio de Legendre).
A importância da lista de polinômios de Legendre decorre da seguinte propriedade:
Afirmação 5.1. (Ortogonalidade dos polinômios de Legendre)
Se n1 , n2 ∈ N são diferentes entre si então:
Z 1
Pn1 (t) · Pn2 (t) dt = 0.
−1
CAPÍTULO 41. EQUAÇÕES COM PONTOS NÃO-SINGULARES: AIRY,
HERMITE E LEGENDRE 651

Demonstração.
Sejam
λ1 := n1 · (n1 + 1), e λ2 := n2 · (n2 + 1)
e as equações de Legendre na forma:
((1 − x2 ) · Pn′ 1 (x))′ = −λ1 · Pn1
((1 − x2 ) · Pn′ 2 (x))′ = −λ2 · Pn2 .
De onde obtemos (por multiplicação e subtração dessa identidades)
Pn2 · ((1 − x2 ) · Pn′ 1 (x))′ − Pn1 · ((1 − x2 ) · Pn′ 2 (x))′ =
= (λ2 − λ1 ) · Pn1 · Pn2 .
Daı́, integrando o lado esquerdo (por partes):
Z
[Pn2 (x) · ((1 − x2 ) · Pn′ 1 (x))′ − Pn1 (x) · ((1 − x2 ) · Pn′ 2 (x))′ ] dx =
Z Z
= Pn2 (x) · ((1 − x ) · Pn1 (x)) dx − Pn1 (x) · ((1 − x2 ) · Pn′ 2 (x))′ dx =
2 ′ ′

Z
= Pn2 (x) · (1 − x ) · Pn1 (x) − Pn′ 2 (x) · (1 − x2 ) · Pn′ 1 −
2 ′

Z
−Pn1 (x) · (1 − x ) · Pn2 (x) + Pn′ 1 (x) · (1 − x2 ) · Pn′ 2 (x) dx =
2 ′

= (1 − x2 ) · [Pn2 (x) · Pn′ 1 (x) − Pn1 (x) · Pn′ 2 (x)]


e portanto a integral definida do lado direito é:
Z 1
(λ2 − λ1 ) Pn1 · Pn2 dx =
−1
Z 1
= [Pn2 (x) · ((1 − x2 ) · Pn′ 1 (x))′ − Pn1 (x) · ((1 − x2 ) · Pn′ 2 (x))′ ] dx =
−1
= 0,
2
pois o termo 1 − x se anula em 1, −1.
Como
λ1 6= λ2
então concluı́mos que Z 1
Pn1 · Pn2 dx = 0.
−1

CAPı́TULO 42

Equação com ponto singular: Hipergeométrica de Gauss

Na Seção 4 do Capı́tulo 31 vimos o desenvolvimento em série infinita de (1 + x)r ,


para qualquer r ∈ R, onde −1 < x < 1.
Agora introduzo uma série que generaliza a série binomial, bem como outras séries
já estudadas, como ln(1 + x) e arcsin(x).
Definição 0.1. Defino o sı́mbolo de Pochhammer
[r]n := r · (r + 1) · . . . · (r + n − 1).
Note que [1]n = n!.
Definição 0.2. Se c 6= 0 e c 6= −n, ∀n ∈ N, a série infinita:
+∞
X [a]n · [b]n n
F (a, b, c; x) := 1 + ·x
n=1
n! [c]n
é chamada de série hipergeométrica.
O nome que se dá a essa série se justifica pelos exemplos a seguir (como o leitor
pode verificar):
• (1 − x)−1 = F (1, b, b; x) (de acordo com a Seção 2 do Capı́tulo 29),
• arctan(x) = x · F ( 21 , 1, 32 ; −x2 ) (de acordo com a Seção 6 do Capı́tulo 30)
• ln(1 + x) = x · F (1, 1, 2; −x) (de acordo com a Seção 8 do Capı́tulo 30),
• (1 + x)r = F (−r, b, b; −x) (de acordo com a Seção 4 do Capı́tulo 31).
Afirmação 0.2.
i): A série F (a, b, c; x) converge em módulo para |x| < 1.

ii): A série y = F (a, b, c; x) é uma solução da equação diferencial:


Ea,b,c : x · (1 − x) · y ′′ + [c − (a + b + 1) · x] · y ′ − a · b · y = 0,
chamada equação hipergeométrica de Gauss com parâmetros a, b, c.

iii): se c 6∈ N então essa equação tem também como solução


y = x1−c · F (a − c + 1, b − c + 1, 2 − c; x).

Por ponto singular x de uma equação entendo aquele ponto x onde o coeficiente
P (x) ou o coeficiente Q(x) da equação
y ′′ (x) + P (x) · y ′(x) + Q(x) · y(x) = 0
não pode ser expresso como série de potências convergente num entorno de x.
653
654

Por isso a Equação hipergeométrica de Gauss tem ponto singular em x = 0 e em


x = 1.

Demonstração.
Para provar i), uso o Teste da Razão para demonstrar a convergência em módulo:
[a]n+1 ·[b]n+1
( (n+1)! [c]n+1
· xn+1 ) (a + n) · (b + n)
| |=| · x|
( [a]n!n[c]
·[b]n
n
· xn ) n · (c + n)
e
(a + n) · (b + n)
lim | · x| = |x|.
n→+∞ n · (c + n)
Para provar1 o item ii), começo procurando soluções da forma:
+∞
X
y(x) = xr · an · xn .
n=0
P+∞
Ou seja, supomos que, para algum r, y = xr · n=0 an · xn é solução da equação
hipergeométrica de Gauss. Note que:
+∞
X +∞
X
y ′(x) = r · xr−1 · an · xn + xr · n · an · xn−1 =
n=0 n=1
e
+∞
X +∞
X
′′ r−2 n r−1
y (x) = r · (r − 1)x · an · x + r · x · n · an · xn−1 +
n=0 n=1
+∞
X +∞
X
+r · xr−1 · n · an · xn−1 + xr · n(n − 1) · an · xn−2 .
n=1 n=2
Pondo isso na equação:
x · (1 − x) · y ′′(x) + [c − (a + b + 1) · x] · y ′(x) − a · b · y(x) ≡ 0,
obtemos à esquerda uma expressão em x cujo coeficiente do termo xr−1 é:
r · (r − 1) + c · r.
Como cada coeficiente tem que se anular, então:
r · (r − 1) + c · r = r · (r − (1 − c)) = 0.
Então r = 0 ou r = 1 − c.

Caso r = 0:
Colocando como solução da equação a série:
+∞
X +∞
X
0 n
x · an · x = an · xn
n=0 n=0

1As idéias por detrás da prova desta segunda afirmação são parte do Método de Fobenius, que
trataremos no Capı́tulo 44
CAPÍTULO 42. EQUAÇÃO COM PONTO SINGULAR: HIPERGEOMÉTRICA
DE GAUSS 655

obtemos
(a1 c − ab a0 ) · x0 + (2a2 + 2a2 c − (a + b + 1)a1 − ab a1 ) · x1 +
+(−2a2 + 6a3 − 2(a + b + 1)a2 + 3ca3 − ab a2 ) · x2 + . . . ≡ 0,
portanto cada coeficiente se anula, e daı́ obtemos:
ab [a]1 · [b]1
a1 = a0 · =: a0 ·
c 1! · [c]1
a + b + 1 + ab (a + b + 1 + ab) ab
a2 = · a1 = a0 · · =
2(c + 1) 2(c + 1) c
a(a + 1)b(b + 1) [a]2 · [b]2
= a0 · =: a0 · ,
2c(c + 1) 2! · [c]2
2a + 2b + 4 + ab (a + 2)(b + 2) a(a + 1)b(b + 1)
a3 = · a2 = a0 · · =:
3(c + 2) 3(c + 2) 2c(c + 1)
[a]3 · [b]3
=: a0 · .
3! · [c]3
E assim por diante se obtém, por indução:
[a]n · [b]n
an = a0 · ,
3! · [c]n
portanto a solução é:
+∞ +∞
X X [a]n · [b]n
a0 · an · xn = a0 · (1 + · xn ).
n=0 n=1
n! [c]n
Isto completa a prova de ii).

Caso r = 1 − c:

Por hipótese do item iii) c 6∈ N; em particular 1 − c 6= 0. Faço uma mudança de


variáveis:
y(x) = x1−c · z(x)
e uma conta mostra que, se y(x) é solução de:
x · (1 − x) · y ′′ + [c − (a + b + 1) · x] · y ′ − a · b · y = 0,
então z(x) é solução de Ea−c+1,b−c+1,2−c , ou seja,
x·(1−x)·z ′′ (x)+[(2−c)−((a−c+1)+(b−c+1)+1)·x]·z ′ (x)−(a−c+1)·(b−c+1)·z(x) = 0.
Pelo que já aprendemos do primeiro Caso, a série infinita y = F (a − c + 1, b − c +
1, 2 − c; x) aparece como solução, desde que
2 − c 6= −n, ∀n ∈ N,
pois na série y = F (a − c + 1, b − c + 1, 2 − c; x) os coeficientes são:
[a − c + 1]n [b − c + 1]n [a − c + 1]n [b − c + 1]n
=
n![2 − c]n n!(2 − c)(2 − c + 1) . . . · (2 − c + n)
1. INTEGRAL ELÍPTICA COMO SÉRIE HIPERGEOMÉTRICA 656

e 2 − c + n não pode se fazer igual a zero. Mas 2 − c = −n dá que c = n + 2 ∈ N,


contradizendo a hipótese adicional do item iii).


1. Integral elı́ptica como série hipergeométrica


Na Seção 4 do Capı́tulo 28 vimos que a integral
Z 2π r
a2
b· 1 − (1 − 2 ) sin2 (t)dt
0 b
2 2
dá o comprimento (perı́metro) da elipse xa2 + yb2 = 1. Pela simetria da elipse, esse
comprimento é:
Z πr
2 a2
4·b 1 − (1 − 2 ) · sin2 (t)dt.
0 b
Considero agora um par de funções do parâmetro x no integrando (cuja notação é
mais ou menos padrão na literatura):
Z πq
√ 2
E( x) := 1 − x · sin2 (t)dt.
0
Z π
√ 2 1
K( x) := p dt.
0 1 − x · sin2 (t)
Note que para z = sin(t) e 0 ≤ t ≤ π2 temos

1 − z 2 = cos(t),
logo, por mudança de variável, vale:
Z π Z 1
√ 2 1 1
K( x) := p dt = √ √ dz,
0 1 − x · sin2 (t) 0 1 − z2 · 1 − x · z2

que é outra maneira como K( x) aparece na literatura √ sobre funções e integrais
elı́pticas. Naquele contexto usualmente se denota x = k e
Z 1
√ 1
K( x) = K(k) = p dz.
0 (1 − z ) · (1 − k 2 · z 2 )
2

Afirmação 1.1.

dE( x) 1 √ √
i) : = · (E( x) − K( x)).
dx 2x

d2 E( x) 1 √ √ √ √
ii) : 2
= 2 · (2E( x) − E( x) · x − 2K( x) + 2K( x) · x).
dx 4x (x − 1)
CAPÍTULO 42. EQUAÇÃO COM PONTO SINGULAR: HIPERGEOMÉTRICA
DE GAUSS 657

iii): a função y = E( x) satisfaz a equação hipergeométrica E 1 ,− 1 ,1 , a saber:
2 2

1
x(1 − x) · y ′′ + (1 − x) · y ′ + · y = 0.
4
Demonstração.
De i):
Trata-se de derivar em relação ao parâmetro x. Pela Afirmação 9.1:
√ Z π p
dE( x) 2 ∂ 1 − x · sin2 (t)
= dt =
dx 0 ∂x
Z π
2 − sin2 (t)
= p 2
dt =
0 2 1 − x · sin (t)
Z π p
2 1 − x · sin2 (t) 1
= ( − p ) dt =
0 2x 2x · 1 − x · sin2 (t)
1
=: · (E(x) − K(x)).
2x
De ii):
Uma conta do mesmo tipo da anterior, mas mais longa, mostra que vale ii).

De iii):
Agora é só simplificar:
√ √ √
d2 E( x) dE( x) E( x)
x(1 − x) · + (1 − x) · + =
dx2 dx 4
1 1−x E
= − · (2E − E · x − 2K + 2K · x)) + (E − K) + ≡ 0.
4x 2x 4


De fato é sabido que:


r Z pi r
a2 2 a2
E( (1 − 2 )) := 1 − (1 − 2
)) · sin2 (t) dt =
b 0 b
π 1 1 a2
=· F ( , − , 1; x) (1 − 2 ).
2 2 2 b
x2 y2
Portanto a área da elipse a2 + b2 = 1 é:
π 1 1 a2
4·b·· F ( , − , 1; x) (1 − 2 ).
2 2 2 b
Não esqueça que preciso ter:
a2
|1 −
|<1
b2
para garantir a convergência da série hipergeométrica. Para a = 4 e b = 3 temos
|1 − 16
9
| = 7/9.
1. INTEGRAL ELÍPTICA COMO SÉRIE HIPERGEOMÉTRICA 658

Resolvi calcular as primeiras somas parciais da série


π 1 1 16
4 · 2 · · F ( , − , 1; x) (1 − ).
2 2 2 9
Obtive:

s1 = 6 · π, s2 ≈ 7.166666667 · π, s3 ≈ 6.996527778 · π,
s4 ≈ 7.051665381 · π, s5 ≈ 7.004760128 · π, s6 ≈ 7.027743702 · π
s7 ≈ 7.015453874 · π, s8 ≈ 7.022427864 · π, s9 ≈ 7.018296138 · π.
Uma aproximação proposta por S. Ramanujan, que mencionamos na Seção 4 do
Capı́tulo 28, é p
(3 · (a + b) − (a + 3b)(3a + b)) · π,
note que para a = 4 e b = 3 isso dá:

(21 − 195) · π ≈ 7.03575996 · π.
CAPı́TULO 43

Equação com ponto singular: a Equação de Bessel

1. A definição original de Bessel


A definição de Bessel para suas funções foi feita através de uma integral1, depen-
dendo de um parâmetro x:
Z π
Jν (x) := cos(ν · (t − x · sin(t))) dt, para ν ∈ N.
0

Afirmação 1.1.
A função y(x) = Jν (x) satisfaz a equação
1 1
y ′′ (x) + · y ′ (x) + ν 2 · (1 − 2 ) · y(x) = 0, ν ∈ N.
x x
A mudança z := ν · x leva essa equação na equação:
′′ 1 ′ (z 2 − ν 2 )
y (z) + · y (z) + · y(z) = 0.
z z2
Definição 1.1. Mais geralmente, se define a equação de Bessel como:
1 (x2 − ν 2 )
y ′′(x) + · y ′ (x) + · y(z) = 0, onde ν ≥ 0, ν ∈ R
x x2
Por ponto singular x de uma equação entendo aquele ponto x onde o coeficiente
P (x) ou o coeficiente Q(x) da equação
y ′′ (x) + P (x) · y ′(x) + Q(x) · y(x) = 0
não pode ser expresso como série de potências convergente num entorno de x.
Por isso a Equação de Bessel tem ponto singular em x = 0

Demonstração. (da Afirmação 1.1)

Vamos ter que derivar em relação ao parâmetro x da integral (veja Seção 9 do


Capı́tulo 36
1
y ′′ (x) + · y ′ (x) =
Z π 2 x Z
π
∂ cos(ν · (t − x · sin(t))) 1 ∂ cos(ν · (t − x · sin(t)))
= 2
dt + · dt =
0 ∂x x 0 ∂x
Z π Z π
2 2 ν
= −ν · cos(ν · (t − x · sin(t)) · sin(t) dt + · sin(ν · (t − x · sin(t)) · sin(t) dt.
0 x 0
1Também

se encontra na literatura a definição Jν (x) := 0 cos(ν · t − x · sin(t)) dt, o que não faz
muita diferença.
659
1. A DEFINIÇÃO ORIGINAL DE BESSEL 660

Agora integro por partes:


Z π
sin(ν · (t − x · sin(t)) · sin(t) dt =
0 | {z } | {z }
=f =g ′

= − cos(t) sin(ν · (t − x · sin(t))(π) + cos(t) sin(ν · (t − x · sin(t))(0)+


Z π
+ν · cos(ν · (t − x · sin(t)) · (1 − x · cos(t)) · cos(t) dt =
Z π 0 Z π
=ν· cos(ν · (t − x · sin(t)) − ν · x cos(ν · (t − x · sin(t)) · cos(t)2 dt,
0 0
onde usei que
sin(ν · (π − x · sin(π)) = sin(ν · π) = 0, se ν ∈ N.
Ou seja,
1
y ′′ (x) + · y ′ (x) =
Z π Z πx
ν2
= · cos(ν · (t − x · sin(t)) dt − ν 2 · cos(ν · (t − x · sin(t)) · (sin(t)2 + cos(t)2 ) dt =
x 0 0
Z π Z π
ν2 2
= · cos(ν · (t − x · sin(t)))) · cos(t) dt − ν · cos(ν · (t − x · sin(t))) dt.
x 0 0
Mas
Z π Z π
ν2 2
· cos(ν · (t − x · sin(t)))) · cos(t) dt − ν · cos(ν · (t − x · sin(t))) dt =
x 0 0
Z π Z π
ν2 2
= 2· cos(ν · (t − x · sin(t)))) · x · cos(t) dt − ν · cos(ν · (t − x · sin(t))) dt =
x 0 0
Z π
ν2
=− 2 · cos(ν · (t − x · sin(t)))) · (1 − x · cos(t) − 1) dt − ν 2 · y(x) =
x
Z π 0
ν ν2
=− 2 · cos(ν · (t − x · sin(t)))) · ν · (1 − x · cos(t)) dt − ν 2 · y(x) + 2 · y(x) =
x 0 x
ν ν2
= − 2 · [sin(ν · (t − x · sin(t)))(π) − sin(ν · (t − x · sin(t)))(0)]] − ν 2 · y(x) + 2 · y(x) =
x | {z } x
=0, ν∈N

ν2
2
= −(ν − 2 ) · y(x),
x
como querı́amos.
Para a segunda afirmação, basta notar que:
dy dy dz dy d2 y d2 y 2
= · = ·ν e = 2 ·ν .
dx dz dx dz dx2 dz
Portanto a equação obtida se escreve como:
d2 y 1 dy 1
ν2 · [ 2 + · + (1 − 2 ) · y(z)] = 0.
dz z dz z

CAPÍTULO 43. EQUAÇÃO COM PONTO SINGULAR: A EQUAÇÃO DE
BESSEL 661

Na Seção 5 do Capı́tulo 44 veremos como expressar algumas funções de Bessel


através de séries infinitas, que funcionarão inclusive para ν 6∈ N (introduzidas por
Lommel e Hankel).

A Afirmação a seguir será útil para detectarmos algumas equações de Bessel ca-
mufladas:
Afirmação 1.2. A equação de Bessel
x2 · y ′′ (x) + x · y ′(x) + (x2 − ν 2 ) · y(x) = 0,
com as mudanças
x = a · ub e y(x) = v(u) · uc , onde a, b, c ∈ R
se transforma na equação:
d2 v dv
u2 2
+ (2c + 1) · u · + [a2 · b2 · u2b + c2 − ν 2 · b2 ] · v(u) = 0.
du du
Assumirei essa Afirmação. Provarei por enquanto apenas um caso bem particular
desta Afirmação na Afirmação 3.1 deste Capı́tulo.

2. Zeros de funções de Bessel


Com o material que já desenvolvemos até aqui no Curso já poderemos dar algumas
informações qualitativas relevantes sobre os zeros das funções de Bessel:
Afirmação 2.1.
i): As soluções não triviais y(x) da equação de Bessel
1 ′ (x2 − ν 2 )
y ′′(x) + · y (x) + · y(z) = 0, onde ν ≥ 0, ν∈R
x x2
têm infinitos zeros.
Podemos dizer mais:
1
a): se 0 ≤ ν ≤ então as soluções y(x) tem infinidade de zeros em (0, +∞).
2
q
b): se ν > 12 então as soluções y(x) tem infinidade de zeros em ( ν 2 − 14 , +∞)
q
e, ademais, no máximo um zero no intervalo (0, ν 2 − 14 ).
1
ii): se ν = 2
então2 a equação tem como soluções3
1 1
y(x) = a · √ · sin(x) + b · √ · cos(x), a, b ∈ R
x x
2Um teorema de Liouville dirá que somente no caso ν = 12 + n, para n = 0 ou n ∈ N, é que as
soluções da equação de Bessel se reduzem
q a funções elementares q
3
A notação usual é y1 = J 12 (x) = π2 · √1x · sin(x) e y2 = J− 21 (x) = π2 · √1x · cos(x).
2. ZEROS DE FUNÇÕES DE BESSEL 662

iii): À medida que x cresce as soluções y(x) são aproximadas por funções do tipo:
1 1
a · √ · sin(x) + b · √ · cos(x), a, b ∈ R
x x

Demonstração.

De i):

Re-escrevo a equação como:


1 ′ (x2 − ν 2 )
y ′′(x) + · y (x) + · y(x) = 0.
x x2
Então a Afirmação 14.1 do Capı́tulo 40 reduz o estudo do número de zeros de y(x)
ao estudo do número de zeros de
(1 + 4 · (x2 − ν 2 ))
v ′′ (x) + · v(x) = 0,
4x2
onde foi feito R 1 √
1
v(x) := e 2 t dt · y(x) = x · y(x).
Agora a Afirmação 13.2 do Capı́tulo 40 diz que há uma infinidade de zeros da
solução v(x) de
(1 + 4 · (x2 − ν 2 ))
v ′′ (x) + · v(x) = 0,
4x2
na região onde x > 0 e onde vale:
(1 + 4 · (x2 − ν 2 ))
> 0.
4x2
Se 0 ≤ ν ≤ 21 , basta então que x > 0.
q
Mas se ν > 21 então preciso ter pelo menos x > ν 2 − 14 .
q
Como em (0, ν 2 − 41 ) temos 1 + 4 · (x2 − ν 2 ) < 0, então a a Afirmação 13.2 do
Capı́tulo 40 do diz que há no máximo um zero nesse intervalo.

De ii): Re-escreva
(1 + 4 · (x2 − ν 2 ))
v ′′ (x) + · v(x) = 0,
4x2
como
1 − 4ν 2
v ′′ (x) + (1 + ) · v(x) = 0.
4x2
1
Se ν = 2
então essa equação vira:
v ′′ (x) + v(x) = 0,
cujas soluções são a · sin(x) + b · cos(x). Como tı́nhamos no item i):
v(x)
y(x) = √
x
CAPÍTULO 43. EQUAÇÃO COM PONTO SINGULAR: A EQUAÇÃO DE
BESSEL 663

obtemos
a · sin(x) + b · cos(x)
y(x) = √ .
x
De iii):
Me contentarei por enquanto com uma explicação apenas heurı́stica: note que se
2
x >> 1 o termo 1−4ν
4x2
fica muito pequeno na equação
1 − 4ν 2
v ′′ (x) + (1 + ) · v(x) = 0;
4x2
essa equação se aproxima portanto da equação:
v ′′ (x) + v(x) = 0.
Se pode provar rigorosamente que para x >> 1:
a · sin(x) + b · cos(x)
y(x) ≈ √ .
x


Afirmação 2.2. Se ν < 12 , então em cada cada intervalo de tamanho π no semi-eixo


positivo há ao menos um zero da solução da equação de Bessel.
Se ν = 12 os zeros distam π um do outro, exatamente.
Se ν > 21 então dois zeros sucessivos da solução da equação de Bessel distam pelo
menos π um do outro.
Demonstração.
Na forma padrão a equação de Bessel é:
1 − 4ν 2
v ′′ (x) + (1 + ) · v(x) = 0;
4x2
Se ν < 21 , então:
1 − 4ν 2
1<1+ .
4x2
Como os zeros das soluções de y ′′(x) + y(x) = 0 estão em intervalos de tamanho π,
concluı́mos pelo Teorema de Comparação de Sturm (Afirmação 15.1 do Capı́tulo 40)
que em cada intervalo de tamanho π no semi-eixo positivo há ao menos um zero de
v(x).
Se ν = 21 já sabemos as soluções, explicitamente.
Se ν > 12 , então:
1 − 4ν 2
1>1+
4x2
e o Teorema de Comparação de Sturm dirá que dois zeros sucessivos da solução da
equação de Bessel distam pelo menos π um do outro (caso contrário, haveria mais de
um zero das soluções de y ′′(x) + y(x) = 0 num intervalo de tamanho menor que π).

3. ORTOGONALIDADE DAS FUNÇÕES DE BESSEL 664

3. Ortogonalidade das funções de Bessel


Ainda sem sabermos resolver explicitamente a equação de Bessel, mas sem pre-
cisarmos disso, vamos provar o seguinte fato notável:
Afirmação 3.1. Seja y(x) solução da Equação de Bessel
1 ′ (x2 − ν 2 )
y ′′(x) + · y (x) + · y(x) = 0.
x x2
E seja λ ∈ R \ {0} um zero dessa função.
Então:
i): z(x) := y(λ · x) é solução da equação
1 ′ (λ2 · x2 − ν 2 )
z ′′ (x) +
· z (x) + · z(x) = 0.
x x2
ii): λ1 ∈ R \ {0} e λ2 ∈ R \ {0} são distintos zeros de y(x) então
Z 1
x · y(λ1 · x) · y(λ2 · x) dx = 0
0

O segundo item desta Afirmação está na raı́z da utilidade das funções de Bessel,
principalmente porque pela Afirmação 2.1 há uma infinidade de zeros λn , n ∈ N, de
cada solução da equação com ν fixado.
Essa lista infinita de funções, aparecerá nos modos normais de vibração de um
tambor, na Seção 3 do Capı́tulo 49.

Demonstração. (da Afirmação 3.1)


Prova do item i):
Considero
u = λ · x, λ ∈ R \ {0}
como uma mudança de variável. Pela derivada da composta:
dy(λ · x) dy(λ · x)
·λ=
du dx
e
d2 y(λ · x) 2 d2 y(λ · x)
·λ = .
du2 dx2
Então obtemos:
1 d2 y(λ · x) 1 dy(λ · x) λ2 · x2 − ν 2
·[ + · + · y(λ · x)] =
λ2 dx2 x dx x2
=
d y(u) 1 dy(u) u2 − ν 2
2
= + · + · y(u).
du2 u du u2
Mas
d2 y(u) 1 dy(u) u2 − ν 2
+ · + · y(u) = 0
du2 u du u2
pois essa é a equação de Bessel de ı́ndice ν.
CAPÍTULO 43. EQUAÇÃO COM PONTO SINGULAR: A EQUAÇÃO DE
BESSEL 665

Logo
d2 y(λ · x) 1 dy(λ · x) λ2 · x2 − ν 2
+ · + · y(λ · x) = 0
dx2 x dx x2
Isto prova o item i).

Prova 4 do item ii):


Pelo item i) já provado, se λ1 6= λ2 são dois zeros de y(x) (solução da Bessel de
ı́ndice ν) e
z1 (x) := y(λ1 · x) e z2 (x) := y(λ2 · x),
então
d2 z1 (x) 1 dz1 (x) 2 ν2
+ · + (λ 1 − ) · z1 (x) = 0
dx2 x dx x2
e
d2 z2 (x) 1 dz2 (x) 2 ν2
+ · + (λ2 − 2 ) · z2 (x) = 0
dx2 x dx x
Multiplicando a primeira dessas duas equações por z2 (x) a segunda por z1 (x) e sub-
traindo, se consegue:
d2 z1 (x) d2 z2 (x) 1 dz1 (x) dz2 (x)
z2 · 2
− z1 · 2
+ · (z2 · − z1 · )=
dx dx x dx dx
= (λ22 − λ21 ) · z1 (x) · z2 (x).
O que é o mesmo que escrever:
dz1 (x) dz2 (x) ′ 1 dz1 (x) dz2 (x)
(z2 · − z1 · ) + · (z2 · − z1 · )=
dx dx x dx dx
= (λ22 − λ21 ) · z1 (x) · z2 (x)
e multiplicando esta identidade por x:
dz1 (x) dz2 (x) ′ dz1 (x) dz2 (x)
= x · (z2 · − z1 · ) + (z2 · − z1 · ) = (λ22 − λ21 ) · x · z1 (x) · z2 (x),
dx dx dx dx
o que consegue-se escrever como:
dz1 (x) dz2 (x) ′
[x · (z2 · − z1 · )] = (λ22 − λ21 ) · x · z1 (x) · z2 (x).
dx dx
Mas então, integrando:
dz1 (x) dz2 (x) dz1 (x) dz2 (x)
[x · (z2 · − z1 · )](1) − [x · (z2 · − z1 · )](0) =
dx dx dx dx
Z 1
2 2
= (λ2 − λ1 ) · x · z1 (x) · z2 (x) dx.
0
Mas
dz1 (x) dz2 (x)
[x · (z2 · − z1 · )](0) = 0
dx dx
e
dz1 (x) dz2 (x)
[x · (z2 · − z1 · )](1) = y(λ2) · y ′ (λ1 ) − y(λ1) · y ′(λ2 ) = 0
dx dx
4Repare como esta demonstração é muito parecida com a prova que demos da ortogonalidade
dos polinômios de Legendre
3. ORTOGONALIDADE DAS FUNÇÕES DE BESSEL 666

pelas escolhas de λ1 , λ2 .
Isso prova o item ii).

CAPı́TULO 44

Equações com pontos singulares do tipo regular

1. A Equação de Euler e sua redução a coeficientes constantes


Agora introduziremos uma equação muito importante, que tem coeficientes variáveis
e que tem ponto singular em x = 0, mas que felizmente é redutı́vel aos métodos da
Seção 2 do Capı́tulo 40, graças à Afirmação 10.1 daquele Capı́tulo.
Afirmação 1.1. (Equação de Euler) A equação
d2 y dy
x2 · 2
+p·x· + q · y = 0, p, q ∈ R e q > 0
dx dx
em intervalos que não contenham a origem x = 0 tem sua solução determinada pelas
raı́zes r1 , r2 da equação:
r · (r − 1) + p · r + q = 0
• se r1 , r2 ∈ R e r1 6= r2 então a solução geral é
y = a · |x|r1 + b · |x|r2 .
• se r1 = r2 = r ∈ R então a solução geral é:
y = a · |x|r + b · ln |x| · |x|r .
• se r1 = λ + I · µ e r2 = λ − I · µ são Complexos conjugados então a solução
geral é
y = a · |x|λ · cos(µ ln |x|) + b · |x|λ · sin(µ ln |x|).
Demonstração.
Note que, se divido por x 6= 0 a equação dada obtenho a equação:
d2 y p dy q
0= 2 + · + 2 ·y =
d x x dx x
2
dy dy
=: 2 + P (x) · + Q(x) · y
dx dx
para a qual se aplica a Afirmação 10.1 já que:
−2q 2pq
Q′ + 2P Q x3
+ x3 (pq − q) · |x|3
3 = 3 = 3
2Q 2 2( xq2 ) 2 q 2 x3
que é constante e igual a
p−1
√ , se x > 0
q
ou
1−p
√ , se x < 0.
q
667
1. A EQUAÇÃO DE EULER E SUA REDUÇÃO A COEFICIENTES
CONSTANTES 668

A Afirmação 10.1 ensina a transformar a equação de Euler em outra a coeficientes


constantes usando a mudança de variável:
Z p Z r
q
z= Q dx = dx
x2
ou seja,

z = q · ln(x), se x > 0
ou

z = − q · ln |x|, se x < 0.
No caso x > 0:

Seguindo as intruções da Afirmação 10.1 do Capı́tulo 40, obteremos a equação:


d2 y p − 1 dy
0= + √ · + y.
d2 z q dz
De fato, com

z := q · ln(x),
temos
dy dy √ 1
= · q·
dx dz x
e
d2 y d2 y 1 dy √ (−1)
2
= 2
·q· 2 + · q· 2 ,
dx dz x dz x
de onde:
d2 y dy
0 ≡ x2 ·
2
+p·x· +q·y =
dx dx
d2 y dy √ dy √
= 2 ·q− · q+ · p · q + q · y,
dz dz dz
e após dividir por q:
d2 y p − 1 dy
0= + √ · + y.
d2 z q dz
As soluções de
d2 y p − 1 dy
0= + √ · +y
d2 z q dz
são determinadas a partir das raı́zes r1 , r2 da equação caracterı́stica:
p−1
r 2 + √ · r + 1 = 0.
q
Como vimos na Afirmação 2.1:
• se há duas raı́zes reais:
p p
1 − p + (p − 1)2 − 4q 1−p+ (p − 1)2 − 4q
r1 = √ e r2 := √
2 q 2 q
então a solução geral é:
√ √
1−p+ (p−1)2 −4q 1−p− (p−1)2 −4q
√ ·z √ ·z
y(z) = a · e 2 q
+b·e 2 q
.
CAPÍTULO 44. EQUAÇÕES COM PONTOS SINGULARES DO TIPO
REGULAR 669

Quando fazemos

z= q · ln(x)
obtemos
√ √
1−p+ (p−1)2 −4q 1−p− (p−1)2 −4q
·ln(x) ·ln(x)
y(x) = a · e 2 +b·e 2 =:
√ √
1−p+ (p−1)2 −4q 1−p− (p−1)2 −4q
=: a · x 2 +b·x 2

e noto que:
p p
1−p+ (p − 1)2 − 4q 1−p− (p − 1)2 − 4q
e
2 2
são raı́zes de
r 2 + (p − 1) · r + q = r · (r − 1) + p · r + q = 0.
Como o caso x < 0 é completamente análogo, fazendo-se uma mudança
de variável x = −x, está provado o primeiro item da Afirmação.
• se
1−p
r1 = r2 = √ = −1
2 q
as soluções são:
y(z) = a · z · e−z + b · e−z
que dão:
√ √ √
y(x) = a · q ln(x) · e− q ln(x) + b · e− q ln(x) =:
√ √ √
=: a · q · ln(x) · x− q + b · x− q

e noto que − q = 1−p 2
é a única raı́z de
r 2 + (p − 1) · r + q = r · (r − 1) + p · r + q = 0.
• o caso em que r1 , r2 são Complexos é análogo.
O Caso x < 0 é completamente análogo.


Exemplo: (Exercı́cio do Bear, p. 164)


Resolver para t > 0 o sistema
y(t) t + z(t)
y ′ (t) = z(t) + e z ′ (t) = .
t t
A primeira dá:
y(t) y ′ (t) y(t)
z(t) = y ′(t) − logo z ′ (t) = y ′′ (t) − + 2 .
t t t
a segunda dá:
y(t)
′′ y ′ (t) y(t) y ′ (t) − t y ′ (t) y(t)
y (t) − + 2 =1+ =1+ − 2 ,
t t t t t
2. SOLUÇÃO DIRETA DA EQUAÇÃO DE EULER 670

ou seja,
2 ′ 2
y ′′(t) − · y (t) + 2 · y(t) = 1.
t t
Ora,
2 ′ 2
y ′′ (t) − · y (t) + 2 · y(t) = 0
t t
é a equação de Euler:
t2 · y ′′ (t) − 2 · t · y ′(t) + 2 · y(t) = 0,
cuja equação indicial
r · (r − 1) − 2 · r + 2 = 0
tem raı́zes 2, 1. Logo a solução geral dessa Euler é, para t > 0:
a · t2 + b · t.
Como os coeficientes da equação
2 ′ 2
y ′′ (t) − · y (t) + 2 · y(t) = 1
t t
não são constantes, para encontrar uma solução particular φ1 (t) dela uso o método de
variação de parâmetros (Seção 4 do Capı́tulo 40). De acordo com aquele resultado,
podemos tomar
φ1 (t) = a(t) · t2 + b(t) · t
onde: Z Z
1
a(t) = dt e b(t) = − 1 dt,
t
e portanto (tomando como 0 as constantes de integração):
a(t) = ln(t) e b(t) = −t
e finalmente
y(t) = a · t2 + b · t + φ(t) = a · t2 + b · t + ln(t) · t2 − t · t =
= t2 · (a′ + ln(t)) + b · t, a′ , b ∈ R.

2. Solução direta da equação de Euler


Aqui se dá uma nova abordagem, bem mais direta da equação.
Ela retoma uma idéia usada na Seção 7 do Capı́tulo 40 e antecipa uma idéia que
se usa quando se aprofunda o método de Frobenius, cujo inı́cio está no Capı́tulo 44.
Como já vimos as soluções todas da Equação de Euler na Seção anterior poderemos
aqui nos ater a alguns pontos especiais.
Considero o operador diferencial linear :
L(y(x)) := x2 · y ′′(x) + p · xy ′(x) + q · y(x)
e a equação de Euler:
L(y(x)) = 0.
Suponha que procuro uma solução da forma:
y = xr , r ∈ R, x > 0.
CAPÍTULO 44. EQUAÇÕES COM PONTOS SINGULARES DO TIPO
REGULAR 671

Então
L(xr ) = x2 · r · (r − 1) · xr−2 + p · x · r · xr−1 + q · xr =

= xr · [r · (r − 1) + p · r + q] = 0
e portanto r é raı́z da equação indicial:
r · (r − 1) + p · r + q = 0.
Há três casos a considerar, dos quais abordarei por enquanto apenas os dois primeiros.
Caso 1:) se r · (r − 1) + p · r + q = 0 tem duas raı́zes distintas:
r1 6= r2 ∈ R
então a solução geral é:
a · xr1 + b · xr2 , x > 0.
Caso 2:) se r · (r − 1) + p · r + q = 0 tem raı́z dupla.
Tomando essa raı́z r vemos que:
xr
é uma solução. Mas e como obter outra solução independente ?
Considero r como uma variável na expressão:
L(xr ) = xr · [r · (r − 1) + p · r + q]
e derivo-a em r (trocando depois a ordem de derivação em x e em r), obtendo à
esquerda :
∂L(xr ) ∂xr
= L( ) = L(xr · ln(x)),
∂r ∂r
já que
xr := er·ln(x) .
E à esquerda:
∂[xr · (r · (r − 1) + p · r + q)]
= r · xr−1 · (r · (r − 1) + p · r + q) + xr · (2 · r + p − 1).
∂r
Ou seja:
L(xr · ln(x)) = r · xr−1 · (r · (r − 1) + p · r + q) + xr · (2 · r + p − 1)
e quando avalio em r que é raı́z dupla da equação indicial, então anulo o lado direito:
L(xr · ln(x)) = 0
e concluo que
xr · ln(x)
é uma outra solução da equação de Euler, linearmente independente de xr .
Deixo a discussão do Caso de raı́zes complexas conjugadas para outra ocasião.
3. DEFINIÇÕES GERAIS E EXEMPLOS DE PONTOS SINGULARES
REGULARES 672

3. Definições gerais e exemplos de pontos singulares regulares


O que há em comum entre a Equação de Euler, a equação Hipergeométrica e a
equação de Bessel ?
Veremos que têm em comum a natureza de alguns de seus pontos singulares.
Para começar, a equação de Euler
x2 · y ′′ (x) + px · y ′(x) + q · y(x) = 0, p, q ∈ R e q > 0
pode ser reescrita como:
p q
y ′′(x) + y ′(x) + 2 · y(x) = 0,
x x
ou seja, tem x = 0 como ponto singular. Note que ao menos ela tem a a propriedade
de que:
p q
x · ( ) = p e x2 · ( 2 ) = q
x x
são constantes. Em particular são polinônios e em particular são séries convergentes
em torno de x = 0. Veremos que esta última condição já basta.
A equação Hipergeométrica, escrita como:
[c − (a + b + 1) · x] ′ a·b·y
y ′′ + ·y − = 0,
x · (1 − x) x · (1 − x)
tem a propriedade de que as funções:
[c − (a + b + 1) · x] c − (a + b + 1) · x a·b a · bx
x· = e x2 · =
x · (1 − x) 1−x x · (1 − x) 1−x
podem ser dadas por séries convergentes em torno de x = 0 (usando séries geométricas
de razão x com |x| < 1).
Também as funções:
[c − (a + b + 1) · x] c − (a + b + 1) · x a·b a · b(1 − x)
(1−x)· = e (1−x)2 · =
x · (1 − x) x x · (1 − x) x
podem ser dadas por séries convergentes em torno de x = 1.
Também a equação de Bessel, escrita como:
1 (x2 − ν 2 )
y ′′(x) + · y ′ (x) + · y(x) = 0,
x x2
tem a propriedade de que as funções:
1 (x2 − ν 2 )
x · = 1 e x2 · = x2 − ν 2
x x2
são polinômios e portanto são séries convergentes em x = 0.
Esses exemplos motivam um pouco a definição:
Definição 3.1. Seja uma equação y ′′(x) + P (x) · y ′ (x) + Q(x) · y(x) = 0 com ponto
singular em x.
Então x é dito um ponto singular regular se as funções
(x − x) · P (x) e (x − x)2 · Q(x)
podem ser dadas por séries convergentes em torno de x.
CAPÍTULO 44. EQUAÇÕES COM PONTOS SINGULARES DO TIPO
REGULAR 673

4. Inı́cio do Método de Frobenius


A solução da Equação de Euler vai nortear o estudo que faremos agora.
Lembre o que aprendemos no primeiro item da Afirmação 1.1: a equação de Euler
p q
y ′′(x) + · y ′ (x) + 2 · y(x) = 0, x > 0
x x
tem como soluções
y = a · xr1 + b · xr2
se a equação
r(r − 1) + p · r + q = 0
tem duas soluções distintas r1 , r2 ∈ R.

Isso motiva a seguinte definição (por simplicidade enunciada só para x = 0):

Definição 4.1. (Equação indicial607)


Seja y ′′ (x) + P (x) · y ′ (x) + Q(x) · y(x) = 0 com ponto singular regular em x = 0,
para a qual
x · P (x) = p0 + p1 · x + p2 · x2 + . . . e x2 · Q(x) = q0 + q1 · x + q2 · x2 + . . .
são séries convergentes.
Define-se sua equação indicial por:
r(r − 1) + p0 · r + q0 = 0
A seguinte Afirmação é parte de uma mais geral, que é o Método de Frobenius
geral.
Me contento, por enquanto, com este enunciado:
Afirmação 4.1. (Inı́cio do Método de Frobenius)
Suponha y ′′ (x) + P (x) · y ′(x) + Q(x) · y(x) = 0 com ponto singular regular em
x = 0, onde
x · P (x) = p0 + p1 · x + p2 · x2 + . . . e x2 · Q(x) = q0 + q1 · x + q2 · x2 + . . .
são séries convergentes.
• Se a equação indicial:
r(r − 1) + p0 · r + q0 = 0
tem uma raı́z dupla r ∈ R então existe uma solução da equação da forma:
X
y = xr · an xn ,
n=0+∞
P n
onde n=0+∞ an · x é uma série de potências convergente.
A série X
y= an xr+n
n=0+∞
é chamada série de Frobenius.
4. INÍCIO DO MÉTODO DE FROBENIUS 674

• Se a equação indicial:
r(r − 1) + p0 · r + q0 = 0
tem duas raı́zes distintas r1 , r2 ∈ R e se
r1 − r2 6∈ Z
então todas as soluções da equação são da forma:
X X
y = xr1 · an xn + xr2 · bn xn
n=0+∞ n=0+∞
P P
onde n=0+∞ an · xn e n=0+∞ bn · xn são séries de potências convergentes.
Demonstração. (Algumas idéias da Prova)
Nem vou discutir as questões de convergência das séries envolvidas, que suponho
convergem absolutamente.
Se começa buscando uma solução da forma
X
y = xr · cn xn , onde r ∈ R e x > 0,
n=0+∞

onde sempre podemos supor


c0 6= 0,
pois caso contrário troco r por r + 1.
Vamos montar cada ingrediente que aparece na equação diferencial, aplicá-los na
equação, e ver que condições se farão necessárias em r e nos coeficientes cn .
Primeiro, derivando termo a termo esse candidato e ordenando por potências,
obtém-se:
+∞
X +∞
X
y ′ = r · xr−1 · cn xn + xr · n · cn · xn−1 =
n=0 n=1
r−1
=x · [rc0 + c1 · (r + 1) · x + c2 · (r + 2) · x2 + . . .] =
+∞
X
= (r + n) · cn · xr+n−1 .
n=0
Como P+∞ P+∞
n=0 pn xn n=0 qn x
n
P (x) = e Q(x) =
x x2
então:
P+∞ n +∞
n=0 pn x
X

P (x) · y (x) = · (r + n) · cn · xr+n−1 =
x n=0
+∞
X +∞
X
= xr−2 · pn xn · (r + n) · cn · xn =
n=0 n=0
+∞
X n
X
= xr−2 · [ pn−k · (r + k) · ck ] · xn
n=0 k=0
CAPÍTULO 44. EQUAÇÕES COM PONTOS SINGULARES DO TIPO
REGULAR 675

onde obtive os coeficientes


n
X
pn−k · (r + k) · ck
k=0
n
de cada monômio x agrupando todos os que resultam, via distributividade do pro-
duto com a soma, como coeficientes dessa potência (chamado produto de Cauchy das
séries, que funciona se as séries convergem absolutamente).
Esta última expressão para P (x) · y ′(x) ainda pode ser escrita para uso futuro
como:
+∞ X
X n−1
′ r−2
P (x) · y (x) = x · [ pn−k · (r + k) · ck + p0 · (r + n) · cn ] · xn .
n=0 k=0

Do mesmo modo se obtém


P+∞ n
n=0 qn x
X
Q(x) · y = · xr · cn xn =
x2 n=0+∞

+∞ X
X n−1
r−2
=x · [ qn−k · ck + q0 · cn ] · xn .
n=0 k=0
P+∞
De y ′ = n=0 (r + n) · cn · xr+n−1 se obtém derivando termo a termo, para x > 0:
+∞
X
′′
y (x) = (r + n) · (r + n − 1) · cn · xr+n−2 =
n=0

+∞
X
r−2
=x · (r + n) · (r + n − 1) · cn · xn .
n=0
Colocando esses ingredientes todos juntos na equação:
y ′′ (x) + P (x) · y ′(x) + Q(x) · y(x) = 0
e fatorando xr−2 obtemos:
+∞
X Xn−1 n−1
X
{(r + n)(r + n − 1)cn + [ pn−k (r + k)ck + p0 (r + n)cn ] + [ qn−k ck + q0 cn ]} · xn =
n=0 k=0 k=0

+∞
X n−1
X
= {cn · [(r + n)(r + n − 1) + p0 (r + n) + q0 ] + ck · [pn−k (r + k) + qn−k ]} · xn = 0.
n=0 k=0
Isso significa o anulamento de todos os coeficientes dessa série de potências, cujos três
primeiros coeficientes são:
c0 · [r · (r − 1) + p0 · r + q0 ] = 0
c1 · [(r + 1) · r + p0 · (r + 1) + q0 ] + c0 · [p1 · r + q1 ] = 0,
c2 · [(r + 2)(r + 1) + p0 · (r + 2) + q0 ] + c1 · [p1 (r + 1) + q1 ] + c0 · [p2 r + q2 ] = 0
e assim por diante.
5. SOLUÇÕES EXPLÍCITAS DE ALGUMAS EQUAÇÕES BESSEL 676
P
Como c0 6= 0, o que concluimos é que se y = xr · n=0+∞ cn x
n
é uma solução
então r é uma raı́z da equação indicial:
r · (r − 1) + p0 · r + q0 = 0.
Escolhida uma raı́z r1 ∈ R da equação indicial e dado c0 vai-se obtendo por recorrência
os coeficientes cn , ∀n ≥ 1:
−c0 · [p1 · r1 + q1 ]
c1 = ,
[(r1 + 1) · r1 + p0 · (r1 + 1) + q0 ]
desde que
(r1 + 1) · r1 + p0 · (r1 + 1) + q0 6= 0,
ou seja , desde que r1 + 1 não seja raı́z d aequação indicial. E também, quando já for
conhecido c1 , teremos
−c1 · [p1 (r + 1) + q1 ] − c0 · [p2 r + q2 ]
c2 = ,
[(r + 2)(r + 1) + p0 · (r + 2) + q0 ]
desde que
(r + 2)(r + 1) + p0 · (r + 2) + q0 6= 0,
ou seja, desde r1 + 2 não seja raı́z da equação indicial.
E assim por diante.
Por isso as hipóteses de que há duas raı́zes distintas r1 , r2 da equação indicial e
de que
r1 − r2 6∈ Z
são suficientes para se obter duas soluções (independentes) da equação da forma:
X X
y = xr1 · an xn e y = xr2 · bn xn .
n=0+∞ n=0+∞

No caso da raı́z dupla só se obtém uma solução desse tipo.




5. Soluções explı́citas de algumas equações Bessel


Vamos usar a Afirmação 4.1 para descrever soluções de equações de Bessel. Em
geral não serão todas as soluções, pois se vê que a Afirmação 4.1 não abrange todas
as possibilidades para as raı́zes da equação indicial.
Os valores de ν na Equação de Bessel
1 (x2 − ν 2 )
y ′′ (x) + y ′(x) + · y(x) = 0
x x2
que mais nos interessam no momento são:
1 1
ν = 0, ν = 1, ν = e ν= .
3 4
Os dois primeiros são importantes em aplicações à Fı́sica enquanto que os dois últimos
serão usados para solucionar a equação de Airy e uma equação de Riccati no Capı́tulo
45.
CAPÍTULO 44. EQUAÇÕES COM PONTOS SINGULARES DO TIPO
REGULAR 677

Como nessa equação:


1
x · P (x) = x · = 1 = p0 e x2 · Q(x) = −ν 2 + x2 = q0 + q2 · x2 .
x
o ponto x = 0 é ponto singular regular e a equação indicial é:
r(r − 1) + r − ν 2 = 0,
ou seja, r 2 = ν 2 e as soluções são:
r1 = ν e r2 = −ν.
1
Nos casos ν = 3
ou ν = 41 , temos:
2 1
r1 − r2 =ou r1 − r2 =
3 2
e portanto se aplica o segundo item da Afirmação 4.1, criando pares de séries de
Frobenius.
Por exemplo, para ν = 31 , tomo a raı́z r1 = 13 e as primeiras recorrências dadas na
Afirmação 4.1 viram:
2
c1 · [ + 1] + c0 · [0] = 0,
3
1
c2 · [4 · ( + 1)] + c1 · [0] + c0 · [1] = 0
3
e assim por diante. Dado c0 6= 0 obtemos:
c0
c1 = 0 e c2 = − 1
4 · ( 3 + 1)
e com mais detalhe se pode comprovar que os coeficientes de ı́ndice ı́mpar se anulam:
c1 = c3 = c5 = c2n−1 = 0, ∀n ∈ N,
enquanto que os de ı́ndices pares são dados por
c0
c2n = (−1)n · 2n , ∀n ∈ N.
2 · n! · ( 3 + 1) · . . . · ( 31 + n)
1

1
A função de Bessel de primeira ordem de ı́ndice ν = 3
é a série de Frobenius:
+∞
1
X c0
y = x3 · (−1)n · · x2n
n=0
22n · n! · ( 31 + 1) · . . . · ( 31 + n)
para a qual se escolhe um valor especı́fico para c0 .
E a função de Bessel de segunda ordem e de ı́ndice ν = 31 é aquela associada à
raı́z r2 = − 13 , obtida analogamente via as recorrências.
Em seguida se vê que isso que fizemos para ν = 13 se generaliza, e sempre
c1 = c3 = c5 = c2n−1 = 0, ∀n ∈ N,
enquanto que os de ı́ndices pares são dados por
c0
c2n = (−1)n · 2n , ∀n ∈ N.
2 · n! · (ν + 1) · . . . · (ν + n)
5. SOLUÇÕES EXPLÍCITAS DE ALGUMAS EQUAÇÕES BESSEL 678

A função de Bessel de primeira ordem e de ı́ndice ν é a série de Frobenius:


+∞
ν
X c0
y=x · (−1)n · · x2n
n=0
22n · n! · (ν + 1) · . . . · (ν + n)

para a qual se escolhe um valor especı́fico para c0 .


A escolha padrão é:
1
c0 := ν ,
2 · ν!
onde, no caso de ν 6∈ N, se deve entender como:
ν! := Γ(ν + 1)
usando a função Gama da Seção 2 do Capı́tulo 27.
Com essa escolha de c0 a notação para as Bessel de primeira e segunda ordem,
quando r1 − r2 = 2 · ν 6∈ Z, é:
Jν (x) e J−ν (x).
No caso ν = 0 a Afirmação 4.1 não produz um par independente de soluções, mas
produz pelo menos (com c0 = 201·0! = 1) uma série de potências:
+∞
0
X 1
y=x · (−1)n · · x2n =
n=0
22n · n! · 1 · . . . · n

+∞
X 1 x 2n
= (−1)n · 2
· ( ) =: J0 (x)
n=0
(n!) 2
Esta é a função de Bessel de primeira ordem e ı́ndice ν = 0, denotada por J0 (x).
A mesma situação quando ν = 1, onde a Afirmação 4.1 dá pelo menos uma série
de potências (com c0 = 211·1! = 12 ) :
+∞
X 1 1
y = x1 · (−1)n · · 2n · x2n =
n=0
2 2 · n! · (1 + 1) · . . . · (1 + n)

+∞
X 1 x
= (−1)n · · ( )2n+1 =: J1 (x)
n=0
n! · (1 + n)! 2
Esta é a função de Bessel de primeira ordem e ı́ndice ν = 1, denotada por J1 (x).

A Afirmação a seguir é apenas o começo de uma lista de propriedades notáveis


das funções de Bessel (que iremos aumentando à medida que for preciso).
Mas já faz ressaltar a analogia entre o par J0 (x), J1 (x) e o par cos(x), sin(x).
Afirmação 5.1.
dJ0 (x)
= −J1 (x).
dx
CAPÍTULO 44. EQUAÇÕES COM PONTOS SINGULARES DO TIPO
REGULAR 679

Demonstração.
Aplicando o Teste da Razão se vê em seguida que ambas séries convergem em
módulo ∀x ∈ R.
Daı́ podemos derivar termo a termo:
+∞ n 1 x 2n
dJ0 (x) X d( (−1) · (n!)2 · ( 2 ) )
= =
dx n=0
dx

+∞
X 1 x 2n−1 1
= (−1)n · 2
· 2n · ( ) · =
n=1
(n!) 2 2

+∞
X 1 x 2n−1
= (−1)n · ·( ) =
n=1
(n − 1)! · n! 2
+∞
X 1 x 2n+1
=− (−1)n · ·( ) =: −J1 (x),
n=0
(n)! · (n + 1)! 2
onde na última linha apenas mudei o ı́ndice que uso no somatório.


1
6. A Equação de Bessel com ν = 3
e a solução da equação de Airy
Apliquemos a Afirmação 1.2 do Capı́tulo 43 ao caso em que queremos transformar
a Equação de Bessel na equação:
d2 v
u2 + u3 · v(u) = 0.
du2
Note que esta equação redunda na equação de Airy:
d2 v
+ u · v(u) = 0.
du2
Ou seja, queremos que a, b, c verifiquem:
2c + 1 = 0, 2b = 3, a2 · b2 = 1 e c2 − ν 2 · b2 = 0,
que dão (se tomamos a > 0:
1 3 2 1
c=− , b= , a= e ν= .
2 2 3 3
Então concluimos que a solução da equação de Airy se expressa como combinação de
funções de Bessel de ı́ndice ν = 31 :
1 2 3 2 3
v(u) = u−c · y(a · ub ) = u 2 · [c1 · J 1 ( u 2 ) + c2 · J− 1 ( u 2 )].
3 3 3 3
7. EQUAÇÃO HIPERGEOMÉTRICA COM C 6∈ Z 680

7. Equação hipergeométrica com c 6∈ Z


Retomemos o que vimos na Afirmação 0.2 do Capı́tulo 42, do ponto de vista da
teoria das singularidades regularees.
A equação hipergeométrica de Gauss com parâmetros a, b, c é:
Ea,b,c : x · (1 − x) · y ′′ + [c − (a + b + 1) · x] · y ′ − a · b · y = 0.
Vejamos que x = 0 é ponto singular regular e vejamos sua equação indicial (fica como
Exercı́cio verificar que x = 1 também é).
Ora, como:
c − (a + b + 1) · x −a · b
P (x) = e Q(x) = ,
x · (1 − x) x · (1 − x)
basta ver que:
c − (a + b + 1) · x −a · b · x
x · P (x) = e x2 · Q(x) =
1−x 1−x
podem ser dados por séries convergentes em torno de x = 0. E isso vem do fato que:
+∞
1 X
= xn , se − 1 < x < 1.
1 − x n=0
Como
x · P (x) = c + (c − a − b − 1) · x + . . . e x2 · Q(x) = −ab · x − −ab · x2 + . . .
a equação indicial é:
r · (r − 1) + c · r + 0 = 0,
cujas raı́zes são:
r1 = 0 e r2 = 1 − c.
se temos por hipótese que:
c 6∈ Z
então 0 6= 1 − c e ademais 1 − c 6∈ Z. O Segundo item da Afirmação 4.1 nos dá
então duas séries independentes como solução, uma delas uma série de potências
correspondendo à raı́z r1 = 0 e a outra uma série de Frobenius correspondendo à raı́z
r2 = 1 − c.
As recorrências dadas na Afirmação 4.1 farão reaparecer os coeficientes das séries
que demos por definição no Capı́tulo 42.
CAPı́TULO 45

Equações de Riccati

As equações diferenciais não-lineares são um universo.


Raramente se deixam tratar por métodos advindos do estudo das equações difer-
enciais lineares. Uma exceção foram as equações de Bernoulli (Seção 13 do Capı́tulo
38).
As Equações de Riccati são equações não-lineares de primeira ordem do tipo:
f ′ (x) = a0 (x) + a1 (x) · f (x) + a2 (x) · f 2 (x),
onde se supõe que a2 (x) 6≡ 0 e que a0 (x) 6≡ 0 para não recairmos em equações lineares
ou em equações de Bernoulli, já tratadas.
Pode parecer que seja uma classe pequena de equações mas de fato são muitas. As
soluções dessas equações abrangem várias das funções que já vimos no livro e muitas
outras.
Exemplos dessas equações e de suas diferentes soluções:
• Vimos na Primeira Parte do Curso que y = tan(x) satisfaz uma Equação de
Riccati:
tan′ (x) = sec2 (x) = 1 + tan2 (x).
• vimos na Seção 13 que a singela equação de Riccati:
f ′ (x) = x + f (x)2 ,
através da mudança:
−g ′ (x)
f (x) =
g(x)
produz
−g ′′ (x) g ′(x) 2
f ′ (x) = +( )
g(x) g(x)
e portanto
g ′′ (x) g ′ (x) 2 −g ′ (x) 2
− +( ) = x+( )
g(x) g(x) g(x)
o que dá:
g ′′ (x) + x · g(x) = 0
que é a equação de Airy.
Na Seção 6 do Capı́tulo 44 expressamos a solução da Equação de Airy
em termos de funções de Bessel.
1 f (x)2
• f ′ (x) = x(1−x 2 ) f (x) − 2
tem uma solução que é a função racional f (x) =
2x
x2 −1
, como se verifica diretamente.
681
1. SOLUÇÕES DE RICCATI SEGUNDO DANIEL BERNOULLI 682
1
• f ′ (x) = + y 2 se trasforma, com a mudança de variável
4x2
z
y= ,
x
na equação separável:
z′ 1
1 =
z2 + z + 4
x
que se integra facilmente:
Z Z
1 z′ 1
− 1 = 1 2 = = ln(x) + C,
z+2 (z + 2 ) x
de onde
1 1
y·x=z =− −
ln(x) + C 2
e
1 1
y=− − .
x · (ln(x) + C) 2x
1
• A primeira equação de Riccati na literatura foi
f ′ (x) = x2 + f (x)2 .
Com a mudança:
g ′ (x)
y(x) = −
g(x)
vira:
g ′′ (x) + x2 · g(x) = 0.
As soluções dessa equação de Riccati são combinações de funções de
Bessel, como veremos na Seção 4 do Capı́tulo 43.

1. Soluções de Riccati segundo Daniel Bernoulli


Afirmação 1.1. (Daniel Bernoulli)
Qualquer equação do tipo:
f ′ (x) = a + b · f (x)2 , a, b ∈ R, e a·b≥0
tem solução Liouvilliana.
Se
4·m 4·m
n = −2, n=− ou n = − , para m ∈ N,
2m + 1 2m − 1
então equação de Riccati:
f ′ (x) = xn + f (x)2
tem solução Liouvilliana.
1estudada por Johan Bernoulli, em 1694, de acordo com G. N. Watson A treatise on the theory
of Bessel functions , Cambrige, 1958. Aprendi a Afirmação 1.1 neste Tratado.
CAPÍTULO 45. EQUAÇÕES DE RICCATI 683

Bem mais difı́cil de justificar é o teorema de J. Liouville que diz que somente para
esses valores de n há soluções Liouvillianas.

Vamos precisar de uma observação:


Afirmação 1.2. Suponha n 6= 1:
I) A mudança de variáveis:
xn+1 1
u := e v := −
n+1 y
leva
y ′ = a · xn + b · y 2
em
−n −n
v ′ = b · (n + 1) n+1 · u n+1 + a · v 2 ,
onde
dv
v′ = .
du
II) A mudança de variáveis:
1 x
U := e V := −x2 · y −
x b
leva
y ′ = a · xn + b · y 2
em
V ′ = a · U −n−4 + b · V 2 ,
onde
dV
V′ = .
dU
Demonstração. (da Afirmação 1.2)

De I):
Basta aplicar a regra da derivada da composta:

1 dv dv dy dx
2
· = y2 · ( · · )=
v du dy dx du
1 −n
= y 2 · 2 · (a · xn + b · y 2) · ((n + 1) · u) n+1 =
y
1 −n
= (a · xn + b · y 2 ) · x−n = a + b · 2 · ((n + 1) · u) n+1
v
de onde obtenho:
dv −n −n
= b · (n + 1) n+1 · u n+1 + a · v 2 .
du
De II):
1. SOLUÇÕES DE RICCATI SEGUNDO DANIEL BERNOULLI 684

Agora não esqueço que, como y = y(x) e x = x(U) então


V = V (x(U), y(x(U)).
Portanto a regra da composta agora dá:
dV ∂V dx ∂V dy dx
= · + · · =
dU ∂x dU ∂y dx dU
1
= (−2xy − ) · (−x2 ) + (−x2 ) · (a · xn + b · y 2) · (−x2 )
b
e agora é imediato que
dV x
= a · xn+4 + b · (x2 · y + )2 =
dU b
−n−4 2
=a·U +b·V .


Demonstração. (da Afirmação 1.1)


Começo provando a primeira afirmação, que pode ser considerada o caso em que
o expoente de x é n0 = 0. Temos
f ′ (x) = a + b · f (x)2 .
Se a = 0 e b = 0 então f (x) ≡ C.
Se a = 0 mas b 6= 0 e f (x) 6≡ 02 faço
f ′ (x)
=b
f (x)2
e portanto
1
− =b·x+C
f (x)
ou seja,
1
f (x) = − .
bx + C
Se a 6= 0 e b = 0 então f (x) = a · x + C.
Se aq6= 0 e b 6= 0 então a condição a · b > 0 diz que têm o mesmo sinal. Logo posso
b
tomar a
∈ R. Então posso escrever a equação
f ′ (x) = a + b · f (x)2
como:
f ′ (x)
q =a
1 + ( ab f (x))2
ou ainda: r r
b f ′ (x) b √
· q =a· = ab.
a 1 + ( b f (x))2 a
a

2Usando o teorema de existência e unicidade


CAPÍTULO 45. EQUAÇÕES DE RICCATI 685

Portanto r
b √
arctan( f (x)) = ab · x + C,
a
de onde r
a √
f (x) = · tan( ab · x + C)
b
Uso no que segue a notação
y = f (x).
Agora o item II) da Afirmação 1.2 diz que, a partir do caso n0 = 0
y′ = a + b · y2,
passo para o caso:
V ′ = a · U −4 + b · V 2 ,
ou seja, onde
4
n1 = −4 = − .
2·1−1
Tomando a = b = 1 isso significa que
V ′ = U −4 + V 2
tem solução Liouvilliana, já que y ′ = 1 + y 2 tem solução Liouvilliana y = y(x) e
V = V (U) = −U −2 · y(U −1 ) − U −1
é composição/produto/soma de Liouvillianas, logo V = V (U) é Liouvilliana, como
querı́amos provar.
4
Se tı́vesemos tomado a = 1 e b = (−3) 3 > 0 então usando o item II) da Afirmação
1.2 terı́amos chegado no caso:
4
V ′ = U −4 + (−3) 3 · V 2
com solução Liouvilliana:
4
V = V (U) = −U −2 · y(U −1 ) − (U · (−3) 3 )−1 .
E o item I) da Afirmação 1.2 diz que, recomeçando neste caso n1 = −4:
4
V ′ = U −4 + (−3) 3 · V 2
chego em:
4 4 4
y ′ = (−3) 3 · (−3)− 3 · x− 3 + y 2 =
4
= x− 3 + y 2 .
ou seja, onde agora
4
n2 = − .
2·1+1
4
A solução Liouvilliana V = V (U) de V ′ = U −4 + (−3) 3 · V 2 produz, usando I), a
solução Liouvilliana:
1 1
y(x) = − =− −1 .
V (U(x)) V ((−3 · x) 3 )
1. SOLUÇÕES DE RICCATI SEGUNDO DANIEL BERNOULLI 686

Recomeçando neste caso, o item II) da Afirmação 1.2 diz que obtenho em uma
solução Liouvilliana de (a notação mantém as mesmas variáveis x, y):
4 8
y ′ = x−(− 3 )−4 + y 2 = x− 3 + y 2
ou seja, chegamos no caso
8 4·2
n3 = − = − .
3 2·2−1
8
Recomeçando neste caso, y ′ = x− 3 + y 2 , o item I) da Afirmação 1.2 conduz ao
caso em que:
8
8 4·2
n4 = 8 3 =− =− ,
−3 + 1 5 2·2+1
a equação obtida é (a notação mantém as mesmas variáveis x, y):
−5 8 − 8
y ′ = ( )− 5 · x 5 + y 2 .
3
Isso ainda não é o que queremos, pois queremos soluções Liouvillianas de:
−8
y′ = x5
+ y2.
Como sabemos como mudam os coeficientes das equações em cada modificação de
tipo I ou II, se vê em seguida que partindo da equação:
−5 8 4
y ′ = ( ) 5 + (−3) 3 · y 2
3
aı́ chegarı́amos em
−8
y′ = x 5 + y2.
4
Fica claro o formato dos números n = − 2·m±1 .
Já o caso n = −2:
f ′ (x) = x−2 + f (x)2
tem que ser tratado separadamente, pois
4·m
− 6= −2, ∀m ∈ N.
2m ± 1
Após a mudança
z
y= ,
x
f ′ (x) = x−2 + f (x)2 vira uma equação separável:
z′ 1
3 1 2 = .
4
+ (z + 2 ) x
1
Para resolvê-la faço u := z + 2
e daı́:
Z
2 u u′
√ · arctan( √ ) = 3 2
=
3 3
4
+ u
2
Z
1
= = ln(x) + C
x
CAPÍTULO 45. EQUAÇÕES DE RICCATI 687

de onde se obtém: √ √
−1 3 tan( 23 · (ln(x) + C))
y= + · .
2x 2 x


2. Assı́ntotas verticais de soluções de equações de Riccati


Apesar de que as equações
y ′(x) = xn + y(x)2 , ∀n ∈ N
não sejam tratáveis pela Afirmação 1.1, podemos contudo fazer uma afirmação qual-
itativa geral:
Afirmação 2.1. Cada solução y(x) de equações de Riccati:
y ′(x) = xn + y(x)2 , ∀n ∈ N
tem uma infinidade de assı́ntotas verticais .
Demonstração.
Considere a mudança de coordenadas:
R
g(x) := e− y dx
,
ou seja,
g ′ (x)
y(x) = − .
g(x)
Então
−g ′′ (x) · g(x) + g ′(x) · g ′(x) g ′′ (x) g ′ (x) 2
y ′(x) = = − + ( ) =
g 2(x) g(x) g(x)
g ′′ (x)
=− + y(x)2 .
g(x)
Ou seja,
g ′′ (x)
− = xn
g(x)
e portanto3:
g ′′ (x) + xn · g(x) = 0.
A Afirmação 13.2 do Capı́tulo 40 diz que g(x) tem uma infinidade de zeros (se n
é impar diz até que estão em (0, +∞)).
E nesses pontos onde g(x) = 0 não pode acontecer que também g ′(x) = 0 (se não
g é identicamente nula, pelo Teorema de Existência e Unicidade).
′ (x)
Logo y(x) = − gg(x) tem nesses pontos assı́ntotas verticais..


3Essa observação de como passar de Riccati para linear de segunda ordem será generalizada no
Exercı́cio 5.1
3. SOLUÇÕES DAS RICCATI SEGUNDO EULER 688

3. Soluções das Riccati segundo Euler


Se aprende a Afirmação a seguir no tratado de G. N. Watson, A treatise on the
theory of Bessel functions:
Afirmação 3.1. (Euler)
i) Suponha conhecida uma solução y1 (x) da equação de Riccati
y ′(x) = a0 (x) + a1 (x) · y + a2 · y 2.
Então outra solução é dada por:
1
y2 = y1 (x) +
v
onde Z
R R
a1 (t)+2a2 (t)y1 (t) dt
v(x) = e ·[ e− a1 (t)+2a2 (t)y1 (t) dt
· a2 (x) dx + C].

ii) Se y1 (x) e y2 (x) são soluções conhecidas da equação


y ′ (x) = a0 (x) + a1 (x) · y + a2 · y 2
então uma terceira solução y3 é dada por:
y2 (x) · w(x) − y1 (x)
y3 =
w(x) − 1
onde R
w(x) = C · e a2 (x)·(y1 (x)−y2 (x)) dx , C 6= 0.

iii): Se y1 , y2 , y3 são três soluções conhecidas de


y ′ (x) = a0 (x) + a1 (x) · y + a2 · y 2
então
y1 · (y3 − y2 ) − C · y2 · (y3 − y1 )
y4 := , onde C 6= 1
y3 − y2 − C · (y3 − y1 )
é uma quarta solução.
Demonstração.
De i):
A equação diferencial está nas hipóteses do Teorema de existência e unicidade,
pois
F (x, y) = a0 (x) + a1 (x) · y + a2 · y 2
é contı́nua nas duas variáveis e
∂F (x, y)
= a1 (x) + 2 · a2 (x) · y
∂y
também é contı́nua.
Portanto quaisquer duas soluções nunca se intersectam. Por isso se y1 (x) é con-
hecida e y2 (x) é ainda desconhecida, posso definir:
1
v(x) :=
y2 − y1 (x)
CAPÍTULO 45. EQUAÇÕES DE RICCATI 689
1
Ou seja, y2 (x) = y1 (x) + v(x)
.
Agora:
v ′ (x)
y2′ (x) = y1′ (x) −
v 2 (x)
e portanto
v ′ (x)
y1′ (x) − 2
= y2′ (x) = a0 (x) + a1 (x) · y2 + a2 (x) · y22 =
v
1 1 2
= a0 (x) + a1 (x) · (y1 (x) + ) + a2 (x) · (y1 (x) + ) =
v(x) v(x)
a1 a2 (x) · y1 1
= a0 (x) + a1 (x) · y1 (x) + + a2 (x) · y12(x) + 2 · + a2 · 2
v(x) v v
e portanto
v ′ (x) a1 a2 (x) · y1 1
2
= +2· + a2 · 2
v v(x) v v
ou seja:
v ′ (x) = (a1 (x) + 2 · a2 (x) · y1 ) · v(x) + a2 (x).
Essa equação diferencial em v é linear, logo o item ii) Afirmação 11.1 do Capı́tulo 35
dá que:
R
Z R
a1 (t)+2a2 (t)y1 (t) dt
v(x) = e · [ e− a1 (t)+2a2 (t)y1 (t) dt · a2 (x) dx + C].

De ii):
Suponha y1 , y2 soluções conhecidas e y3 ainda desconhecida. Pelo teorema de
existência e unicidade a função
y3 (x) − y1 (x)
w(x) :=
y3 (x) − y2 (x)
está bem definida (pois y3 6= y2 ), nunca se anula (pois y3 6= y1 ) e nunca vale 1 (pois
y1 6= y2 ).
Então
y2 (x) · w(x) − y1 (x) ′
y3′ (x) = ( ) (x) =
w(x) − 1
y2 (x) · w(x) − y1 (x) y2 (x) · w(x) − y1 (x) 2
= a0 (x) + a1 (x) · ( ) + a2 · ( ).
w(x) − 1 w(x) − 1
Usando que y1 (x) e y2 (x) são soluções aparecem simplificações que dão finalmente:
w ′(x)
= a2 (x) · (y1 (x) − y2 (x))
w(x)
ou seja R
a2 (x)·(y1 (x)−y2 (x)) dx
w(x) = C · e , C 6= 0.
De iii):
Usando o que aprendemos na prova do item ii) já sabemos que:
y3 (x) − y1 (x) R
= C1 · e a2 (x)·(y1 (x)−y2 (x)) dx , C1 6= 0
y3 (x) − y2 (x)
3. SOLUÇÕES DAS RICCATI SEGUNDO EULER 690

e, pelo mesmo motivo, que uma quarta solução teria que ser:

y4 (x) − y1 (x) R
= C2 · e a2 (x)·(y1 (x)−y2 (x)) dx , C2 6= 0, C2 6= C1 .
y4 (x) − y2 (x)

Portanto:
( yy44 (x)−y
(x)−y1 (x)
2 (x)
) C2
= =: C 6= 1.
( yy33 (x)−y
(x)−y1 (x)
2 (x)
) C1

Isolando y4 = y4 (C, y1, y2 , y3 ) nessa expressão se chega ao resultado. 

Um Exemplo:

Considere a equação de Riccati

y ′(x) = 1 − y(x)2 .

Ela tem duas soluções constantes:

y1 (x) ≡ −1 e y2 (x) ≡ 1.
1
Definindo v := y2 −y 1
≡ 21 como na prova do item ii) da Afirmação 3.1, vemos que
coerentemente com aquele item:
1
y2 = 1 = −1 + = −1 + 2.
v
Já o item iii) da Afirmação 3.1 nos diz que, definindo
R
2dt
w(x) := C · e = C · e2x+B

teremos uma terceira solução:

w(x) + 1 C · e2x+B + 1
y3 (x) = = .
w(x) − 1 C · e2x+B − 1

E o item iv) da Afirmação 3.1 nos diz que uma quarta solução é:

1 − y3 − D · (y3 + 1)
y4 (x) = , se D 6= 1, D 6= 0.
y3 − 1 − D · (y3 + 1)

Por exemplo, se tomo C = 1, B = 1, D = 2:

e2x+1 + 1 3 · y3 (x) + 1
y3 (x) = e y4 (x) = .
e2x+1 − 1 y3 (x) + 3
CAPÍTULO 45. EQUAÇÕES DE RICCATI 691
1
4. A Equação de Bessel com ν = 4
e a solução da Riccati y ′ = x2 + y 2
Sabemos resolver a Equação de Bessel com ν = 14 e que duas soluções indepen-
dentes são denotadas por J 1 (x) e J− 1 (x), as chamadas funções de Bessel de primeira
4 4
e segunda ordem.
Com isso estaremos em condição de dizer explicitamente o que são as soluções da
equação de Riccati:
y ′ = x2 + y 2 .
Como já vimos (na prova da Afirmação 2.1) a mudança
g ′ (x)
y(x) = −
g(x)
leva a equação em
g ′′ (x) + x2 · g(x) = 0.
Se usamos a Afirmação 1.2, vemos que esta equação, ou equivalentemente:
x2 g ′′ (x) + x4 · g(x) = 0
provém de uma equação de Bessel com ν = 41 , pois se comparamos os expoentes e
ı́ndices vemos que:
2c + 1 = 0, 2b = 4, a2 · b2 = 1 e c2 − ν 2 · b2 = 0
ou seja, c = − 12 , b = 2 e a = 21 , se a > 0, e ν = 14 . Então
1 1 1
g(x) = x 2 · [c1 · J 1 ( x2 ) + c2 · J− 1 ( x2 )].
4 2 4 2

′ 2 2
Agora vemos que as soluções de y = x + y são:
1
(x 2 · [c1 · J 1 ( 12 x2 ) + c2 · J− 1 ( 21 x2 )])′
y(x) = − 1
4 4
.
x 2 · [c1 · J 1 ( 21 x2 ) + c2 · J− 1 ( 12 x2 )]
4 4

5. Exercı́cios
Exercı́cio 5.1. A mudança:
g ′ (x)
y(x) = −
a2 (x) · g(x)
leva a solução da equação de Riccati geral:
y ′ (x) = a0 (x) + a1 (x) · y(x) + a2 (x) · y 2(x)
numa solução da equação linear de segunda ordem:
a′ (x) a0 (x)
g ′′ (x) − ( 2 + a1 (x)) · g ′ (x) + · g(x) = 0.
a2 (x) a2 (x)
Parte 3

Séries de Fourier e Equações diferenciais


parciais
CAPı́TULO 46

Séries de Fourier

As séries de Fourier, as funções de Bessel e os polinômios de Legendre serão cruciais


para a resolução das Equações Diferenciais Parciais mais fundamentais.
Este Capı́tulo deve muito ao livro muito motivador e muito bem escrito de H.
F. Davis, Fourier series and orthogonal functions, Allyn and Bacon, 1963. Nele se
encontrarão teoremas bem mais gerais que a Afirmação 3.1 que veremos a seguir.
Muito interessante e útil também o livro de Eli Maor, Trigonometric delights,
Princeton, 1998.
Sabemos que o perı́odo de sin(x) e de cos(x) é 2π, que o perı́odo de sin(n x) e
cos(n x) é 2π
n
e que o perı́odo de uma combinação linear do tipo
k
X
an · cos(nx) + bn · sin(nx)
n=1
é o maior deles, ou seja, 2π.
A questão é saber se é verdade que qualquer função f (x) periódica1 de perı́odo
2π pode ser escrita como
+∞
X
f (x) = a0 + an · cos(nx) + bn · sin(nx).
n=1
A questão assim colocada em toda generalidade é inabordável, por isso me re-
stringirei a tratar inicialmente2 o caso em que f é derivável e tem f ′ (x) contı́nua.
Do ponto de vista prático a questão tem muita utilidade:
• Imagine que se conhece a resposta de um sistema a cada entrada em forma
de onda sinusoidal; chamemos s1 o input sinusoidal e L(s1 ) o output (pos-
sivelmente com amplitude e fase diferente). Suponhamos que o sistema é
linear, ou seja, L(a · s1 + b · s2) = a · L(s1) + b · L(s2). Então se tivermos uma
escritura
X k
f (x) ≈ a0 + an · cos(nx) + bn · sin(nx),
n=1

1O importante é que haja uma periodicidade de f (x). Se o perı́odo p não for igual a 2π podemos
fazer uma mudança de variável:

z= x,
p
pois agora ∆x = p dá ∆z = 2π.
2Em algum outro momento redigirei as estensões aos casos em que há descontinuidades da f .
Essas surgem naturalmente quando se reproduz uma função que é definida apenas [a, b] para toda a
reta dos R, fazendo-a periódica.
695
1. SÉRIES DE FOURIER E SEUS COEFICIENTES 696

podemos saber a resposta a qualquer entrada f (x), pois pela linearidade:


k
X
L(f ) ≈ a0 + an · L(cos(nx)) + bn · L(sin(nx)).
n=1

• o som de um instrumento musical é esencialemte periódico, ao contrário de


ruı́dos e barulhos. Mas o som de um instrumento musical (aı́ incluı́da a
voz humana) é uma superposição de harmônicos (i.e. múltiplos inteiros da
frequência) de uma frequência fundamental. Há instrumentos cuja sonori-
dade tem uma mistura mais rica de harmônicos que outros. Nosso ouvido é
capaz de uma decomposição do som composto ao estilo da decomposição da
Série de Fourier, ao contrário do olho, que não faz uma decomposição da cor.

1. Séries de Fourier e seus coeficientes


As séries do tipo
+∞
X
a0 + an · cos(nx) + bn · sin(nx)
n=1
são séries trigonométricas.
Serão chamadas série de Fourier de uma função f se
Z 2π
1
a0 := f (t) dt,
2π 0
Z
1 2π
an := f (t) cos(nt) dt, n ∈ N
π 0
e Z
1 2π
bn := f (t) sin(nt) dt, n ∈ N
π 0
Observações:
• Em alguns textos se toma por definição
Z
1 2π
a0 := f (t) dt
π 0
e depois na série se põe
+∞
a0 X
+ an · sin(nx) + bn · cos(nx).
2 n=1

• Também a escolha do intervalo de integração poderá ser alterada, por exem-


plo, para [−π, π] se a função é 2π-periódica, ou em geral, para [−L, L] se a
função é 2L-periódica, onde se põe:
Z L
1
a0 := f (t) dt,
2L −L
Z
1 L nπ
an := f (t) · cos( · t) dt, n ∈ N
L −L L
CAPÍTULO 46. SÉRIES DE FOURIER 697

e Z
1 L nπ
bn := f (t) · sin( · t) dt, n ∈ N
L −L L
• Nem sempre se consegue calcular esses coeficientes, que são integrais, us-
ando funções elementares. Nesse caso se dão aproximações numéricas dos
coeficientes.

Exemplo 1:
Suponha uma função f dada por f (x) = −1 no intervalo [−π, 0] e por f (x) = 1
no intervalo [0, π] Note que por ser uma função ı́mpar,
a0 = 0 e an = 0, ∀n ≥ 1.
Já Z π
1
bn := · f (t) · sin(n · t) dt =
π −π
Z π
2
= · sin(n · t) dt =
π 0

2 cos(n · π) cos(n · 0)
· [− + ],
π n n
4
ou seja, bn = 0 se n ∈ N é par e bn = nπ se n ∈ N é ı́mpar.
Então, restringindo o domı́nio da f ao intervalo (0, π) (onde há continuidade e
derivabilidade) posso afirmar, pelo Teorema de Fourier 3.1 a seguir, que
4 1 1
f (x) ≡ 1 =· (sin(πx) + sin(3π · x) + sin(5π · x) + . . .).
π 3 5
A Figura a seguir dá f ≡ 1 e truncamentos para n ı́mpar, de n = 1 até n = 11:

1,2

0,8

0,6

0,4

0,2

0
0 0,2 0,4 0,6 0,8 1
x
1. SÉRIES DE FOURIER E SEUS COEFICIENTES 698

Tomando x = 21 obtenho a série de Leibniz (que vimos por outro método na Seção
7 do Capı́tulo 30):
π 1 1 1
= 1 − + − + ...
4 3 5 7
Exemplo 2:
Considero f (x) = x no intervalo [−π, π] e sua série de Fourier. Como
Z π
1
a0 := · t dt = 0,
2π −π
como
Z π
1
an := t · cos(nt)dt = 0
π −π

por ter um integrando que é função ı́mpar e como, pelo Exercı́cio 1.1 do Capı́tulo 24,
Z
1 π 2
bn := t · sin(nt) dt = (−1)n+1 · ,
π −π n

concluimos que a série de Fourier de f (x) em [π, π] se escreve como:


2 2 2 2
2 · sin(x) − · sin(2x) + · sin(3x) − · sin(4x) + · sin(5x) . . .
2 3 4 5
A Figura a seguir mostra y = x em vermelho ao lado de 2 · sin(x), 2 · sin(x) − 22 ·
sin(2x), etc.

1
x
-3 -2 -1 0 1 2 3
0

-1

-2

-3
CAPÍTULO 46. SÉRIES DE FOURIER 699

2. Séries de Fourier só de senos ou só de cossenos


Se ao invés de y = f (x) = x no Exemplo da Seção anterior tivéssemos tomado
qualquer função ı́mpar também terı́amos chegado à conclusão que:
Z π
1
a0 := · f (t) dt = 0
2π −π
e que Z
1 π
an := f (t) · cos(nt)dt = 0,
π −π
já que f (x) · cos(nx) é uma função ı́mpar em −π, π] também.
Então a série de Fourier de uma função ı́mpar é uma série só de senos.
Agora, se y = f (x) é uma função par, então
Z
1 π
bn := f (t) · sin(nt)dt = 0,
π −π
já que f (x) · sin(nx) é agora uma função ı́mpar em [−π, π].
Então a série de Fourier de uma função par é uma série só de cossenos.

3. Convergência pontual da Série de Fourier


Afirmação 3.1. (Convergência pontual)
Seja y = f (x) função periódica de perı́odo 2π, derivável, com derivada f ′ (x)
contı́nua.
Então para cada x ∈ [0, 2π] vale:
+∞
X
f (x) = a0 + an · sin(nx) + bn · cos(nx)
n=1

onde Z 2π
1
a0 := f (t) dt,
2π 0
Z
1 2π
an := f (t) cos(nt) dt, n ∈ N
π 0
e Z
1 2π
bn := f (t) sin(nt) dt, n ∈ N.
π 0
Demonstração.
Queremos controlar quanto vale
k
X
|f (x) − Sk (x)| := |f (x) − a0 − an · sin(nx) + bn · cos(nx)|,
n=1

à medida que k aumenta, pois queremos provar que, para cada x fixado,
lim |f (x) − Sk (x)| = 0.
k→+∞
3. CONVERGÊNCIA PONTUAL DA SÉRIE DE FOURIER 700

Para isso será útil reescrevermos


Z 2π k Z 2π Z 2π
1 X
Sk (x) := f (t) dt+ f (t) sin(n·t) dt ·sin(n·x)+ f (t) cos(n·t) dt ·cos(n·x).
2π 0 n=1 0 0

Primeiro, vejo que


Z 2π k Z 2π
1 X
Sk (x) = f (t) dt + f (t) cos(n · (x − t)) dt,
2π 0 n=1 0

onde usei a fórmula do cosseno da diferença para cos(n · x − n · t)


A seguir noto que para cada n:
Z 2π Z 2π
f (t) cos(n · (x − t)) dt = f (x − t) cos(n · t) dt
0 0
pela Afirmação 3.3 a seguir.
E portanto
Z 2π
sin((k + 21 ) · t)
Sk (x) = f (x − t) dt
0 2π sin( 2t )
pela Afirmação 3.4 a seguir.
Também a Afirmação 3.4 diz que:
Z 2π
sin((k + 12 ) · t)
dt = 1.
0 2π sin( 2t )
Como integro em t, posso escrever para cada x:
Z 2π Z 2π
sin((k + 21 ) · t) sin((k + 21 ) · t)
f (x) = f (x) · dt = f (x) · dt.
0 2π sin( 2t ) 0 2π sin( 2t )
Chegamos então, tomando a integral da diferença, em:
Z 2π
1 sin((k + 12 ) · t)
|f (x) − Sk (x)| = | · (f (x) − f (x − t)) · dt|
2π 0 sin( 2t )
A mudança de variável t = −t dá:
Z 2π
1 sin((k + 21 ) · t)
|f (x) − Sk (x)| = | · (f (x) − f (x + t)) · dt|
2π 0 sin( 2t )
Agora para x fixado vou introduzir uma função φx : [0, 2π] → R, y = φx (t), que
será contı́nua. A definição é:
f (x + t) − f (x) t
φx (t) := · , se t > 0
t sin( 2t )
e
f (x + t) − f (x) t
φx (0) := lim · =
tց0 t 2π sin( 2t )
t
= f ′ (x) · lim = f ′ (x) · 2.
tց0 sin( t )
2
CAPÍTULO 46. SÉRIES DE FOURIER 701

Ou seja que
Z 2π
1 1
φx (t) · sin((k + ) · t)|,
|f (x) − Sk (x)| = | ·
0 2π 2
R R
ou ainda que (usando o seno de uma soma e | | ≤ | |):

Z 2π Z 2π
1 t 1 t
|f (x) − Sk (x)| = | · φx (t) cos( ) · sin(kt) dt + · φx (t) sin( ) · cos(kt) dt|.
2π 0 2 2π 0 2
Para terminar a demonstração basta mostrar então que:
Z 2π
t
lim φx (t) cos( ) · sin(kt) dt = 0
k→+∞ 0 2
e que
Z 2π
t
lim φx (t) sin( ) · cos(kt) dt = 0.
k→+∞ 0 2
Vou provar algo mais forte na Afirmação 3.2 : que para cada x a série numérica
+∞ +∞ Z 2π
X
2
X t sin(kt)
ck := ( φx (t) cos( ) · √ dt)2
k=1 k=1 0 2 π

é convergente, pois isso implica3 que seu termo geral tende a zero:
Z 2π
2 t sin(kt)
0 = lim ck := lim ( φx (t) cos( ) · √ dt)2 ,
k→+∞ k→+∞ 0 2 π
o que claramente dá
Z 2π
t sin(kt)
0 = lim ck := lim φx (t) cos( ) · √ dt
k→+∞ k→+∞ 0 2 π
e portanto:
Z 2π
t
lim φx (t) cos( ) · sin(kt) dt
k→+∞ 0 2
(analogamente para a outra integral).


Afirmação 3.2. A série numérica


+∞ +∞ Z 2π
X
2
X t sin(kt)
ck := ( φx (t) cos( ) · √ dt)2
k=1 k=1 0 2 π

é convergente.

3Como já observamos na Seção 7 do Capı́tulo 22.


3. CONVERGÊNCIA PONTUAL DA SÉRIE DE FOURIER 702

Demonstração.
Como c2k ≥ 0, as somas
sk := c21 + c22 + . . . + c2k
formam uma sequência crescente. O Teorema fundamental de sequências diz que para
sn convergir basta existir uma cota superior:
sk ≤ K, ∀k ∈ N.
Vamos mostrar quedefortcoef essa cota é:
Z 2π
t
K= ( φx (t) cos( ) )2 dt,
0 2
que existe pois a função φx (t) · cos( 2t ) é contı́nua.
Para aliviar a notação denoto:
t
φ := φx (t) · cos( ).
2
Começo observando que:
Z 2π k Z 2π
X sin(nt) sin(nt)
0≤ [φ − φ √ dt · √ ]2 dt
0 n=1 0
π π
já que o integrando é ≥ 0. R

Mas, usando agora que 0 φ sin(nt)

π
dt são números, usando as propriedades lineares
da integral obtemos:
Z 2π k Z 2π
X sin(nt) sin(nt)
[φ − φ √ dt · √ ]2 dt =
0 n=1 0
π π
Z 2π k Z 2π k Z 2π
X sin(nt) sin(nt) X sin(nt) sin(nt)
= [φ − φ √ dt · √ ] · [φ − φ √ dt · √ ] dt =
0 n=1 0
π π n=1 0
π π
Z 2π k Z 2π
2 X sin(nt)
= φ dt − 2 · ( φ √ dt)2 +
0 n=1 0
π
Z
X 2π sin(nt) Z 2π Z 2π
sin(mt) sin(nt) sin(mt)
+ φ √ dt · φ √ dt · √ √ dt+
n6=m 0 π 0 π 0 π π
k Z 2π Z 2π
X sin(nt) sin(nt)2
+ ( φ √ dt)2 · .
n=1 0 π 0 π
Agora uso os itens iv) e vi) da Afirmação 3.5, que dizem que
Z 2π
sin(mt) · sin(nt) dt = 0 se m 6= n e m, n ∈ N,
0
e Z 2π
sin(nt)2
dt = 1 ∀n ∈ N.
0 π
CAPÍTULO 46. SÉRIES DE FOURIER 703

Portanto, do de acima:
Z 2π k Z 2π
2 X sin(nt)
0≤ φ dt − ( φ √ dt)2
0 n=1 0 π
e daı́
k Z 2π Z 2π
X sin(nt) 2
sk := ( φ √ dt)2 ≤ φ dt, ∀k ∈ N
n=1 0 π 0

como querı́amos.


Afirmação 3.3. Se y = f (x) tem perı́odo 2π então:


Z 2π Z 2π
f (t) cos(n · (x − t)) dt = f (x − t) cos(n · t) dt.
0 0

Demonstraç
R ão. 2π
Faça em 0
f (t) cos(n · (x − t)) dt a substituição:
t := x − t, dt = −dt,
que dá:
Z 2π Z x−2π
f (t) cos(n · (x − t)) dt = f (x − t) cos(n · t) (−dt) =
0 x
Z x
= f (x − t) cos(n · t) dt =
x−2π
Z 2π
= f (x − t) cos(n · t) dt,
0
pois tanto f quanto o cosseno são periódicas de perı́odo 2π.


Afirmação 3.4. Defina:


1 1
Dn (x) := + · [cos(x) + cos(2x) + . . . + cos(nx)].
2π π
Então
sin((n + 12 ) · x)
i) : Dn (x) = .
2π sin( x2 )
Z 2π
sin((n + 12 ) · t)
ii) : dt = 1.
0 2π sin( 2t )
Demonstração.

3. CONVERGÊNCIA PONTUAL DA SÉRIE DE FOURIER 704

Afirmação 3.5.
Z π
i): cos(m · M) · cos(n · M) dM = 0 se m 6= n e m, n ∈ N,
−π
Z 2π
ii): cos(m · M) · cos(n · M) dM = 0 se m 6= n e m, n ∈ N,
0
Z π
iii): sin(m · M) · sin(n · M) dM = 0 se m 6= n e m, n ∈ N,
−π
Z 2π
iv): sin(m · M) · sin(n · M) dM = 0 se m 6= n e m, n ∈ N,
0
Z π
π
v): sin(m · M)2 dM = ∀m ∈ N
0 2
Z 2π
vi): sin(m · M)2 dM = π ∀m ∈ N
0
Z π
π
vii): cos(m · M)2 dM = ∀m ∈ N
0 2
Z 2π
viii): cos(m · M)2 dM = π ∀m ∈ N
0
Z 2π
ix): sin(m · M) · cos(n · M) dM = 0, ∀m, n ∈ N,
0
Z π
x): sin(m · M) · cos(n · M) dM = 0, ∀m, n ∈ N,
−π

Demonstração.
Basta que eu prove um item e o leitor poderá facilmente adaptar a prova para os
outros.
Por ex. o item
Z 2π
ix): sin(m · M) · cos(n · M) dM = 0, ∀m, n ∈ N.
0
Noto que:
sin(mM + nM) = sin(mM) · cos(nM) + cos(mM) · sin(nM),
e que
sin(mM − nM) = sin(mM) · cos(nM) − cos(mM) · sin(nM),
de onde, somando as duas expressões, obtenho:
1
sin(mM) · cos(nM) = · (sin(mM + nM) + sin(mM − nM)).
2
Então
Z 2π Z 2π Z 2π
1
sin(mM) · cos(nM)dM = · ( sin((m + n)M) dM + sin((m − n)M)dM).
0 2 0 0
CAPÍTULO 46. SÉRIES DE FOURIER 705

Se m = n então
Z 2π Z 2π
1
sin(m · M) · cos(n · M) dM = · sin(mM + nM) dM =
0 2 0
−1 1
= cos(mM + nM)(2π) + cos(mM + nM)(0) = 0.
2(m + n) 2(m + n)
Se m 6= n então Z 2π
sin(m · M) · cos(n · M) dM =
0
−1 1
( cos(mM + nM) − cos(mM − nM)))(2π))+
2(m + n) 2(m − n)
1 1
( cos(mM + nM) + cos(mM − nM))(0) = 0.
2(m + n) 2(m − n)


Agora vou demonstrar os itens 4 i), ii), iii), iv) e ix) e x) da Afirmação anterior
de um modo unificado.
O interesse desta nova prova é que nela não usa nenhuma propriedade trigonométrica
das funções, usa somente a equação diferencial satisfeita pelas funções e que têm todas
em comum o perı́odo 2π, já que têm perı́odos 2π n
ou 2π
m
, n, m ∈ N.
Noto que para cada n ∈ N as funções yn := sin(n · x) ou yn (x) := cos(n · x) dos
itens i), ii), iii), iv) e ix) satisfazem a equação:
yn′′ (x) = −n2 · yn (x).
Então para n 6= m ∈ N:
ym (x) · yn′′ (x) − yn (x) · ym
′′
(x) = (m2 − n2 ) · ym · yn
e a integração por partes do lado esquerdo dá:
Z
ym (x) · yn′′ (x) − yn (x) · ym
′′
(x) dx =
Z Z
′ ′ ′
= ym (x) · yn (x) − ym (x) · yn (x) dx − yn (x) · ym (x) + yn′ (x) · ym
′ ′
(x) dx =
= ym (x) · yn′ (x) − yn (x) · ym

(x).

Como ym (x), ym (x), yn (x), yn′ (x) têm perı́odo 2π:
(ym (x) · yn′ (x) − yn (x) · ym

(x))(π) − (ym (x) · yn′ (x) − yn (x) · ym

(x))(−π) = 0
e
(ym (x) · yn′ (x) − yn (x) · ym

(x))(2π) − (ym (x) · yn′ (x) − yn (x) · ym

(x))(0) = 0.
Então concluo, calculando a integral definida do lado direito, que
Z π Z 2π
2 2
(m − n ) · ym · yn = 0 e (m2 − n2 ) · ym · yn = 0;
0 0
4Do mesmo jeito que fiz na prova da ortogonalidade dos polinômios de Legendre na Afirmação
5.1 do Capı́tulo 41
4. SÉRIES DE FOURIER DE COS(R · SIN(X)) E DE SIN(R · SIN(X)), R ∈ R706

como m 6= n saem os itens i), ii), iii), iv), ix) e x).

4. Séries de Fourier de cos(r · sin(x)) e de sin(r · sin(x)), r ∈ R


Há aplicações práticas relevantes dessas funções.
Suas expansões em série de Fourier são:
Afirmação 4.1. As expansões em séries de Fourier de
cos(r · sin(x)) e cos(r · sin(x))
são:
cos(r · sin(x)) = J0 (r) + 2 · (J2 (r) · cos(2x) + J4 (r) · cos(4x) + J6 (r) · cos(6x) + . . .),

sin(r · sin(x)) = 2 · (J1 (r) · sin(x) + J3 (r) · cos(3x) + J5 (r) · cos(5x) + . . .),
onde Jn (x) são as funções de Bessel.
Demonstração.
Pela definição dada Seção 1, Capı́tulo 43 e por ser o cosseno uma função par,
podemos escrever:
Z π
1
Jn (r) = · cos(r sin(t) − n · t) dt.
π 0
Agora
Z π Z
1 1
· cos(r sin(t)−n·t) dt = · [cos(r sin(t))·cos(n·t)+sin(r sin(t))·cos(n·t)] dt =
π 0 π
Z π Z
1 1
= · cos(r sin(t)) · cos(n · t) dt + · sin(r sin(t)) · cos(n · t) dt.
π 0 π
Usando a simetria de sin(x) em torno de π2 e usando que cos( π2 −x) = − cos( π2 + x)
se obtem5 que:
Z π
1
Jn (r) = · cos(r sin(t)) · cos(n · t) dt, se n = 0, 2, 4, 6 . . .
π 0
enquanto que:
Z π
1
Jn (r) = · sin(r sin(t)) · sin(n · t) dt, se n = 0, 2, 4, 6 . . .
π 0

Claramente cos(r · sin(x)) e de sin(r · sin(x)) são deriváveis (infinitas vezes). A


primeira é uma função par e a segunda uma função ı́mpar.
Portanto a Afirmação 3.1 e as observações da Seção 2 permitem concluir a demon-
stração. 

5verificar
CAPÍTULO 46. SÉRIES DE FOURIER 707

5. Convergência absoluta da Série de Fourier


A importância da Afirmação 3.1 diz que, sob hipótese na f , para cada x a série
de Fourier da f calculada em x converge para o número f (x).
Mas ainda não podemos assegurar que como um todo os gráficos dos truncamentos
da série de de Fourier tendam ao gráfico da f .
A Figura a seguir ilustra uma situação em que funções fn tendem pontualmente
para uma certa função f , quando n → +∞, mas onde sempre há um ponto retar-
datário, ou seja, algumas partes dos gráficos das fn se aproximam do gráfico limite f
mas sempre há uma região dos gráficos que ficou para trás. Nessas condições, se as fn
fossem truncamentos de séries, não estarı́amos autorizados a fazer várias operações
que precisamos, como integrar termos a termo, derivar termo a termo a série.

0,25

0,2

0,15

0,1

0,05

0
0 0,2 0,4 0,6 0,8 1
x

Fig.: Gráficos de y = fn (x) := xn − x2n , para n = 1, 2, 3, 4, x ∈ [0, 1]


convergindo pontualmente quando n → +∞ para f ≡ 0.
Afirmação 5.1. (Convergência uniforme e em módulo)
Seja y = f (x) função periódica de perı́odo 2π, duas vezes derivável (i.e. com f ′ (x)
′′
e f (x)).
Há convergência em módulo da série de Fourier:
+∞
X
|a0 | + | an · sin(nx) + bn · cos(nx) |
n=1

onde Z 2π
1
a0 := f (t) dt,
2π 0
Z
1 2π
an := f (t) cos(nt) dt, n ∈ N
π 0
e Z
1 2π
bn := f (t) sin(nt) dt, n ∈ N.
π 0
Ademais, para cada k, o tamanho:
k
X
| f (x) − (a0 + an · sin(nx) + bn · cos(nx)) |
n=1

só depende de k, valendo uniformemente ∀x.


5. CONVERGÊNCIA ABSOLUTA DA SÉRIE DE FOURIER 708

Demonstração.
Nesta prova usarei algumas vezes a Afirmação 5.2 a seguir.
O primeiro uso dela será, pondo para cada x:
u := (an , bn ) v = (sin(nx), cos(nx)),
1
| an · sin(nx) + bn · cos(nx) | ≤ (an 2 + bn 2 ) 2 .
A etapa crucial da prova é mostrar que a série numérica:
+∞
X 1
(an 2 + bn 2 ) 2
n=1

converge6, pois daı́ tiraremos tudo: de fato, com isso em mãos, pelo Teorema de
Comparação se séries numéricas, para cada x há convergência em módulo:
+∞
X +∞
X 1
|a0 | + |an · sin(nx) + bn · cos(nx) | ≤ |a0 | + (an 2 + bn 2 ) 2 < +∞.
n=1 n=1
Como já sabemos pela Afirmação 3.1 que para cada x:
+∞
X
f (x) = a0 + an · sin(nx) + bn · cos(nx),
n=1
então:
k
X +∞
X
| f (x) − (a0 + an · sin(nx) + bn · cos(nx)) | = | an · sin(nx) + bn · cos(nx)| ≤
n=1 n=k+1
+∞
X
≤ | an · sin(nx) + bn · cos(nx)| ≤
n=k+1
+∞
X 1
≤ (an 2 + bn 2 ) 2 < ǫ
n=k+1
P 1
se k é suficientemente grande, se soubermos que a série +∞ n=1 (an 2 + bn 2 ) 2 converge.
P 2 21
Como o termo geral da série +∞ 2
n=1 (an + bn ) é positivo, basta mostrar que ∀k:
k
X 1
(an 2 + bn 2 ) 2 ≤ K
n=1
para alguma constante K a ser determinada.
Para encontrar esse K começo considerando a derivada f ′ (x).
Considero a série de Fourier de y = f ′ (x) que denoto
X
a′0 + n = 1+∞ a′n cos(nx) + b′n sin(nx).
Por hipótese essa função ainda é derivável mais uma vez, portanto há convergência
pontual para cada x:
X
f ′ (x) = a′0 + n = 1+∞ a′n cos(nx) + b′n sin(nx).
6Cuidado P+∞ 1
P+∞ 1
que n=1 n2 converge mas n=1 n não.
CAPÍTULO 46. SÉRIES DE FOURIER 709

E ademais, modificando um pouco a prova da Afirmação 3.2 se pode provar que para
qualquer k:
k Z 2π
a′0 2 X ′ 2 ′ 2 1
+ (an + bn ) ≤ · (f ′ (x))2 dx,
2 n=1
π 0
o que dá a convergência de
+∞
a′0 2 X ′ 2 2
+ (an + b′n ).
2 n=1
Agora noto que, integrando por partes:
Z
′ 1 2π ′
an := f (t) cos(nt) dt =
π 0
Z 2π
1
= · [f (2π) cos(n2π) − f (2π) cos(n2π) + f (t) sin(nt) · n dt] =
π 0
Z 2π
1
= · f (t) sin(nt) · n dt =: n · bn ,
π 0
já que f tem perı́do 2π.
E também que: Z 2π
′ 1
bn := · f ′ (t) sin(nt) · n dt =
π 0
Z 2π
1
= · [f (2π) cos(n2π) − f (2π) cos(n2π) − f (t) cos(nt) · n dt] =
π 0
=: −n · an .
Em suma,
(b′ )2 (a′ )2
∀n, (an )2 = n2 e (bn )2 = n2 ,
n n
Ou seja,
k k
X
2 2 21
X 1 1
((an ) + (bn ) ) = · ((a′n )2 + (b′n )2 ) 2
n=1 n=1
n
A Afirmação 5.2 a seguir, pondo em Rk os seguintes vetores
1 1 1
u := (1, . . . , ) v = ( ((a′1 )2 + (b′1 )2 ) 2 , . . . , ((a′k )2 + (b′k )2 ) 2 ),
k
dá a desigualdade
k k k
X 1 ′ 2 ′ 2 21
X 1 1 X ′ 2 1
· ((an ) + (bn ) ) ≤ ( 2
) 2 · ( (an ) + (b′n )2 ) 2 .
n=1
n n=1
n n=1
Ora, as séries
+∞
X 1
n=1
n2
e
+∞
a′0 2 X ′ 2 2
+ (an + b′n )
2 n=1
6. A SOLUÇÃO DA EQUAÇÃO DE KEPLER VIA SÉRIE DE FOURIER E
FUNÇÕES DE BESSEL 710

convergem, portanto ∀k:


k k
X 1
X 1 1
((an )2 + (bn )2 ) 2 = · ((a′n )2 + (b′n )2 ) 2 ≤ K
n=1 n=1
n
para algum K, como querı́amos. 

Afirmação 5.2. (Caso particular da desigualdade de Cauchy-Schwartz)


Sejam dois vetores em Rn : u = (v1 , . . . , vn ) e v = (v1 , . . . , vn ). Então
n
X n
X
1 1
2
| u1 · v1 + . . . + u2 · v2 | ≤ ( ui ) · ( 2 vi 2 ) 2 .
i=1 i=1

6. A solução da equação de Kepler via série de Fourier e funções de


Bessel
Minha referência para esta Seção é o livro de A. Gray e B. G. Mathews, A treatise
on Bessel functions and their applications to physics, McMillan, 1895.
Vimos na Seção 11 do Capı́tulo 39, a dedução da Equação de Kepler :
M = φ − e · sin(φ)
onde
• φ é a anomalia excêntrica (definida na Seção 11 do Capı́tulo 39 e ilustrada
na Figura a seguir),
• M = 2·π·T
T0
é a anomalia média,
• T tempo transcorrido do ponto P (T ) na trajetória, desde o perihélio em A e
T0 o perı́odo da órbita.

Q
Y

ϕ θ

p A X
O F

O que se quer é resolver essa equação, determinando φ em função de M:


φ = φ(M),
pois isso daria φ = φ(T ), que é o que preciso para ter a posição do planeta em cada
tempo T (já que a a trajetória elı́ptica é suposta conhecida).
CAPÍTULO 46. SÉRIES DE FOURIER 711

Note que, mesmo que ainda não saibamos explicitamente o que é φ(M), podemos
afirmar que:
• a expressão φ(M) − M se anula em M = k · π, onde k = 0, 1, 2, 3 . . .;
• φ(M) − M é periódica em M de perı́odo 2 · π,
• φ(M) − M é uma função ı́mpar.
Isso motiva, de acordo com a Seção 2, a busca de uma expansão em série de
Fourier-senos dessa função:
Afirmação 6.1. Se φ = φ(M) é solução de M = φ − e · sin(φ), com 0 < e < 1 e se
+∞
X
φ(M) − M = bν · sin(ν · M).
ν=1
então os coeficientes verificam
1 2
bν = bν (e) = · · Jν (e), ∀ν ∈ N,
ν π
onde Z π
Jν (x) = cos(ν · (t − x · sin(t))) dt.
0

Demonstração.
Se tivéssemos essa expressão
+∞
X
φ(M) − M = bν · sin(ν · M)
ν=1
e se pudéssemos derivá-la em M termo a termo, obterı́amos:
+∞
dφ X
−1= ν · bν (e) · cos(ν · M).
dM ν=1
Agora, para cada ν0 fixado, multiplico termo a termo:
+∞
dφ X
cos(ν0 · M) · ( − 1) = ν · bν (e) · cos(ν · M) · cos(ν0 · M)
dM ν=1
e depois integro, termo a termo:
Z π +∞ Z π
dφ X
cos(ν0 · M) · ( − 1) dM = ν · bν (e) · cos(ν · M) · cos(ν0 · M) dM.
0 dM ν=1 0

De acordo com a Afirmação 3.5 da Seção 1:


Z π
cos(ν · M) · cos(ν0 · M) dM = 0 se ν 6= ν0 e ν, ν0 ∈ N,
0
Z π
π
cos(ν0 · M)2 dM = , ∀ν0 ∈ N.
0 2
De onde concluiremos que, para cada ν ∈ N:
Z π
dφ π
cos(ν · M) · ( − 1) dM = · ν · bν (e),
0 dM 2
6. A SOLUÇÃO DA EQUAÇÃO DE KEPLER VIA SÉRIE DE FOURIER E
FUNÇÕES DE BESSEL 712

ou seja, para cada ν ∈ N:


Z π
2 dφ
bν (e) = · cos(ν · M) · ( − 1) dM =
νπ 0 dM

Z π
2 dφ
= · cos(ν · M) · dM,
νπ 0 dM
onde a última igualdade sai de que:
Z π
sin(ν · M) sin(ν · M)
cos(ν · M) dM = (π) − (0) = 0.
0 ν ν

Mas como:
φ(0) = 0 e φ(π) = π

e como temos
M = φ − e · sin(φ),

posso fazer uma substituição na integral:


Z π Z π
2 dφ 2
· cos(ν · M) · dM = · cos(ν · (φ − e · sin(φ))) · dφ
νπ 0 dM νπ 0

e portanto
Z π
2
bν (e) = · cos(ν · (φ − e · sin(φ))) · dφ.
νπ 0

Quer dizer, relembrando a Definição do começo da Seção 1 do Capı́tulo 43 (usando φ


no papel de t):
1 2
bν (e) = · · Jν (e), ν ∈ N.
ν π


Na figura a seguir plotei para e = 0.9 o gráfico da aproximação


10
X
φ10 (M) := M + bν (0.9) · sin(ν · M)
ν=1

em vermelho junto com a diagonal y = M em verde. Se vê bem como um planeta


descrevendo uma trajetória elı́ptica vai bem rápido em seu perihélio (M = 0) e como
vai lentamente em seu afélio (M = π).
CAPÍTULO 46. SÉRIES DE FOURIER 713

0
0 1 2 3 4 5 6
M

Fig: y = φ10 (M) em vermelho, y = M em verde, M ∈ [0, 2π]


7. Exercı́cios
Exercı́cio 7.1. Considere f : [−π, π] → R, f (x) = x2 .
Redefina os coeficientes de Fourier para [−π, π]. Usando que f é par, prove que
sua série de Fourier é:
π2 cos(2x) cos(3x) cos(4x)
f (x) = − 4 · (cos(x) − + − + . . .)
3 22 32 42
Avaliando f em x = π conclua o seguinte resultado de Euler:
π2 1 1 1
= 1+ 2 + 2 + 2 + ...
6 2 3 4
CAPı́TULO 47

Equações Diferenciais Parciais

1. Observações gerais, tipos, separação de variáveis, soluções clássicas


• Uma equação diferencial parcial é uma equação que envolve uma função
y = f (x1 , x2 , . . . , xn ) de mais de uma variável e suas derivadas parciais:
∂y ∂2y
F (x1 , . . . , xn , y, , . . . , 2 , . . .) = 0.
∂x1 ∂x1
• A ordem da equação é a maior ordem de derivação que aparece na equação,
por exemplo:
∂3y ∂2y ∂y
+ 2+ + x1 · x2 = 0
∂x3 ∂x2 ∂x1 ∂x1 ∂x3
é uma equação parcial de terceira ordem.
• A equação será homogênea se não há termo independente de y = f (x) ou de
suas derivadas; em outras palavras, se y = f (x) ou suas derivadas aparecem
em cada termo. Por exemplo, a equação anterior não é homogênea, mas
∂3y ∂2y ∂y
+ 2+ =0
∂x3 ∂x2 ∂x1 ∂x1 ∂x3
é homogênea.
• A equação é linear se y e suas derivadas figuram apenas na potência 1
e estão multiplicados apenas por funções das variáveis independentes (in-
cluindo constantes). Podem aparecer expressões não-lineares nas variáveis
independentes.
Por exemplo, a equação
∂3y ∂2y ∂y
+ 2+ =0
∂x3 ∂x2 ∂x1 ∂x1 ∂x3
é linear, bem como:
∂3y ∂2y ∂y
+ 2+ + ex1 ·x2 · x23 = 0,
∂x3 ∂x2 ∂x1 ∂x1 ∂x3
apesar do termo independente ex1 ·x2 · x23 .
Porém
∂3y ∂2y ∂y
+ ( 2 )2 + sin( )=0
∂x3 ∂x2 ∂x1 ∂x1 ∂x3
não é linear.
715
1. OBSERVAÇÕES GERAIS, TIPOS, SEPARAÇÃO DE VARIÁVEIS,
SOLUÇÕES CLÁSSICAS 716

Também
∂y ∂y
(x21 + x32 ) · + =0
∂x2 ∂x1
é linear, embora
∂y ∂y
y· + =0
∂x2 ∂x1
não seja linear.
• Uma equação é apenas semi-linear se é linear nas derivadas de ordem máxima.
O exemplo anterior, apesar de não-linear, é semilinear. A semi-linearidade
já é uma informação importante, havendo técnicas para lidar com essas
equações.
• A linearidade da operação de tomar derivada faz com que uma equação linear
e homogênea defina um operador linear LF :
y 7→ LF (y).
Por exemplo, se F (x1 , x2 , y, ∂y
x1
∂y
, . . .) = 5 · ∂x 1
∂y
+ 3 · ∂x 2
= 0 e se a, b ∈ R, temos:
a · y1 + b · y2 7→ LF (a · y1 + b · y2 ) :=
∂(a · y1 + b · y2 ) ∂(a · y1 + b · y2 )
:= 5 · +3· =
∂x1 ∂x2
∂y1 ∂y ∂y2 ∂y2
= a · [5 · +3· ] + b · [5 · +3· ]=
∂x1 ∂x2 ∂x1 ∂x2
= a · LF (y1 ) + b · LF (y2 ).
Note que LF não seria linear se a equação F = 0 não fosse homogênea.
• O importante desta observação é que, quando a equação parcial F = 0 é
linear e homogênea, ou seja, LF é operador linear, então as soluções y1 , y2
de F = 0 podem ser superpostas como a· y1 + b· y2, produzindo outra solução.
• Na linguagem da álgebra linear, a superposição de soluções diz que LF = 0
define um subespaço linear (núcleo) do espaço de funções onde se pode aplicar
LF .
Ao contrário do que acontecia com as equações diferenciais ordinárias, o
espaço LF = 0 pode ser um espaço vetorial de dimensão infinita. A vasta
possibilidade de escolha de soluções está na base de três conceitos: P
• i) a idéia de buscar soluções que são somas infinitas de soluções +∞ n=1 an yn
(caso convirjam).
• ii) o processo de separação de variáveis, em que se restringe a busca de
soluções y(x1 , x2 , . . . , xn ) às da forma:
y(x1 , x2 , . . . , xn ) = y1 (x1 ) · y2 (x2 ) · . . . yn (xn ).
• iii) a necessidade de se impor condições iniciais ou de fronteira à solução
y(x1 , . . . , xn ) para poder ter unicidade de soluções. Por exemplo, se uma das
variáveis é temporal, t := xn , e se impõe condições iniciais
y(x1 , . . . , xn−1 , 0) = g(x1 , . . . , xn )
estamos num problema de Cauchy.
CAPÍTULO 47. EQUAÇÕES DIFERENCIAIS PARCIAIS 717

Se impomos, na fronteira ∂U do domı́nio U ⊂ Rn onde está definida a


equação, uma condição
y| ∂U = g
estamos num problema de Dirichlet. Se impomos
∂y
= g,
∂η |∂U
∂y
onde ∂η é a derivada direcional na direção normal à fronteira ∂U, temos um
problema de Neumann. Os problemas de Dirichlet e Neumann podem ser
combinados.
Dada uma equação F (x1 , . . . , y, ∂y
x1
, . . . . . .) = g(x1 , . . . , xn ) não-homogênea,
ainda podemos usar a parte homogênea dela para definir um operador linear.
• Apesar de que em geral pode acontecer que
∂ 2 f (x1 , x2 ) ∂ 2 f (x1 , x2 )
6=
∂x1 ∂x2 ∂x2 ∂x1
lidaremos sempre com funções paras as quais não importa a ordem em que
se deriva. De acordo com o Lema de Schwartz, para isso é suficiente que f e
suas derivadas parciais de primeira e segunda ordem sejam contı́nuas. Serão
chamadas soluções clássicas da equação.

2. Equações parciais de primeira ordem e o método das caracterı́sticas


3. A Equação da difusão do Calor
Nesta Seção tentei modelar a difusão1 de Calor sem usar os elementos ∆x, ∆t dos
livros de Fı́sica e Equações diferenciais, mas ao contrário usando alguns Teoremas de
Valor Médio.
A heurı́stica dos ∆x, ∆t é forte, mas se usamos ao contrário alguns Teoremas da
Parte I do Curso aumentamos a unidade do texto.
Experimentalmente se verifica que a trasmissão de Calor entre dois discos de área
A, com temperaturas T1 e T2 , postos a uma distância d é
|T2 − T1 |
k·A· ,
d
onde a constante k > 0 depende do material dos discos. Essa lei experimental é
associada a Fourier.
Vamos pensar num problema essencialmente unidimensional, ou seja, em algo
como um arame cuja seção transversal tem área constante A e pequena em relação ao
comprimento. Ele será posto na direção do eixo dos x, com inı́cio em x = 0 e término
em x = 2π.
Pensaremos que a temperatura nos pontos do arame é da forma2
T (x, t),
1ou de substâncias quı́micas
2as funções envolvidas, temperatura, densidade, etc, serão supostas com tantas derivadas quanto
necessário
3. A EQUAÇÃO DA DIFUSÃO DO CALOR 718

ou seja, que é constante em cada seção transversal.


Também pensaremos que o arame só troca calor com o ambiente pelas seções
transversais inicial s0 e final s2π , estando no resto isolado termicamente.
A taxa com que o Calor C passa pela seção transversal Sx0 do arame é:
∂T
C ′ (x0 ) = −k · A · (x0 , t),
∂x
o que pode ser justificado fazendo d → 0 na lei experimental. O sinal negativo nos
permite interpretar essa fórmula como dizendo que o fluxo de calor vai da esquerda
para direita, se ∂T (x∂x
0 ,t)
< 0, enquanto que o fluxo de calor vai da direita para a
esquerda, se ∂T
∂x
> 0.
Penso agora num pedaço do arame, que vai da seção transversal Sx0 até a seão
transversal Sx1 , e que simbolizo por A × [x0 , x1 ].
A taxa total com que o calor entra no pedaço A × [x0 , x1 ] através da sua fronteira
Sx0 ∪ Sx1 é então:
∂T ∂T
−k · A · (x0 , t) + k · A · (x1 , t) =
∂x ∂x
∂T ∂T
= kA · ( (x1 , t) − (x0 , t)).
∂x ∂x
A quantidade total de calor que entra em A × [x0 , x1 ] no tempo de t0 a t1 é:
Z t1
∂T ∂T
kA · ( (x1 , z) − (x0 , z)) dz.
t0 ∂x ∂x
Nesse intervalo de tempo de t0 a t1 cada ponto3 z ∈ A × [x0 , x1 ] teve uma mudança
de temperatura:
T (z, t1 ) − T (z, t0 ).
A variação média da temperatura de A × [x0 , x1 ] nesse intervalo de tempo de t0 a t1
é dada por: Z x1
1
· T (z, t1 ) − T (z, t0 ) dz.
x1 − x0 x0
O quanto mudou a temperatura em A × [x0 , x1 ] depende da quantidade de Calor
que entrou, que calculamos acima, mas também das propriedades fı́sicas do material
codificadas numa contante 1s e da massa de A × [x0 , x1 ], que é dada por:
Z x1
ρ(x) · A dx,
x0

onde ρ = ρ(x) é a densidade (que é suposta só depender de x e não da temperatura).


Isso se escreve então como:
Z x1 R t1 ∂T ∂T
1 1 t0 kA · ( ∂x (x1 , z) − ∂x (x0 , z)) dz
· T (z, t1 ) − T (z, t0 ) dz = · R x1 =
x1 − x0 x0 s x0
ρ(x) · A dx
R t1 ∂T ∂T
k t0 ∂x (x1 , z) − ∂x (x0 , z) dz
= · R x1 .
s x0
ρ(x) dx
3Assumimos que a temperatura de cada ponto da seção Sz é a mesma
CAPÍTULO 47. EQUAÇÕES DIFERENCIAIS PARCIAIS 719

Mas pelo Teorema do Valor Médio de Integrais:


R x1
x0
T (z, t1 ) − T (z, t0 ) dz
= T (ξ, t1 ) − T (ξ, t0 ) para algum ξ ∈ (x0 , x1 ),
x1 − x0
logo
Z x1 Z t1
k ∂T ∂T
T (ξ, t1) − T (ξ, t0) · ρ(x) dx = · (x1 , z) − (x0 , z) dz.
x0 s t0 ∂x ∂x
Agora dividimos tudo por (t1 − t0 ) · (x1 − x0 ):
∂T
R x1 R t1 (x1 ,z)− ∂T (x0 ,z)
T (ξ, t1) − T (ξ, t0) x0 ρ(x) dx k t0 ∂x ∂x
x1 −x0
dz
· = ·
t1 − t0 x1 − x0 s t1 − t0
1
(note que pude pôr para dentro da integral á direita).
x1 −x0
Agora o Teorema do Valor Médio de Integrais dá:
R x1
x0
ρ(x) dx
= ρ(τ ), para algum τ ∈ (x0 , x1 )
x1 − x0
e o Teorema do Valor Médio de Lagrange dá:
∂T
∂x
(x1 , z)− ∂T
∂x
(x0 , z) ∂2T
= (ω, z), para algum ω ∈ (x0 , x1 )
x1 − x0 ∂x2
(que depende de z, ω = ω(z) ∈ (x0 , x1 )).
Portanto: R t1 ∂ 2 T
T (ξ, t1 ) − T (ξ, t0 ) k t0 ∂x2 (ω, z) dz
· ρ(τ ) = · =
t1 − t0 s t1 − t0
∂2T
= (ω, η), para algum η ∈ (t0 , t1 ),
∂x2
onde na última iguladade usei mais uma vez o Teorema do Valor médio de Integrais.
Note agora que t1 → t0 implica que η → t0 . Também note que x1 → x0 implica
que:
ξ → x0 , τ → x0 e ω → x0 .
Portanto, fazendo t1 → t0 e x1 → x0 em
T (ξ, t1 ) − T (ξ, t0 ) k ∂2T
= · (ω, η),
t1 − t0 s ρ(τ ) ∂x2
obtemos em x = x0 e t = t0
∂T (x, t) k ∂ 2 T (x, t)
(x, t) = · (x, t).
∂t s ρ(x) ∂x2
Na literatura se costuma chamar:
k
α2 := > 0.

Isso que fizemos em dimensão 1 se generaliza a mais dimensões espaciais.
4. PROBLEMAS DE ESFRIAMENTO UNIDIMENSIONAIS 720

Por isso, a equação diferencial (parcial, linear, de segunda ordem) que rege a
mudança da temperatura4 T = T (x, y, t) é a chamada Equação da Difusão do Calor :
∂2T ∂2T ∂T
α2 · ( 2
+ 2
)=
∂x ∂y ∂t
ou se T = T (x, y, z, t) é:
∂2T ∂2T ∂2T ∂T
α2 · ( 2
+ 2
+ 2
)= .
∂x ∂y ∂z ∂t
Esse coeficiente α2 é muito pequeno para a água e alto para o cobre, por exemplo.
Um exemplo. Para as funções f1 = −x2 − y 2 , f2 = x2 + y 2 e f3 = x2 − y 2 a origem
(0, 0) é ponto de máximo, mı́nimo e de séla, respectivamente. E os Laplacianos são
respectivamente :
∂ 2 f1 ∂ 2 f1 ∂ 2 f2 ∂ 2 f2 ∂ 2 f3 ∂ 2 f3
+ = −4, + = 4 + = 0.
∂x2 ∂y 2 ∂x2 ∂y 2 ∂x2 ∂y 2
Intuitivamente, a equação da difusão do calor diz que se o Laplaciano num ponto P é
negativo, então num entorno de P há menos calor que em P e portanto a temperatura
de P diminui; já se o Laplaciano num ponto P é positivo, então num entorno de P
há mais calor que em P e portanto a temperatura de P aumenta.
Quando se estabiliza a temperatura temos:
∂2T ∂2T
+ = 0.
∂x2 ∂y 2
ou
∂2T ∂2T ∂2T
+ + 2 =0
∂x2 ∂y 2 ∂z
e essas equações serão estudadas no Capı́tulo 48.

4. Problemas de esfriamento unidimensionais

Problema 1 - homogêneo:

Considere um arame isolado do ambiente, exceto pelos extremos, com uma dis-
tribuição de temperatura f (x), x ∈ [0, L] no tempo t = 0. Imagine que começa a
sofrer resfriamento porque seus extremos são postos a 0 grau e assim mantidos ∀t > 0.
Por exemplo suponha que f (x) ≡ C 6= 0 no instante t = 0. Queremos determinar
T (x, t), a função temperatura no tempo t, onde
T (x, 0) = f (x) ≡ C > 0
e
T (0, t) ≡ 0 e T (L, t) ≡ 0, ∀t > 0.
É natural prever que ao longo do tempo cada ponto do arame tenderá a ter temper-
atura zero. Mas queremos determinar de modo quantitativamente exato como isso
acontece.
4bem como outros processos de difusão de gase, etc, em meios homogêneos
CAPÍTULO 47. EQUAÇÕES DIFERENCIAIS PARCIAIS 721

Pela equação do Calor:


∂ 2 T (x, t) ∂T (x, t)
α2 · = .
∂x2 ∂t
Façamos a hipótese simplificadora de separação de variáveis:
T (x, t) = T1 (x) · T2 (t).
A equação do calor vira:
d2 T1 (x) dT2 (t)
α2 · · T2 (t) = T1 (x) · ,
dx2 dt
ou seja, para x ∈ (0, L) e t > 0:
1 d2 T1 (x) 1 1 dT2 (t)
· 2
= 2· · .
T1 (x) dx α T2 (t) dt
Como o lado esquerdo só depende de x e o direito só de t, para que haja essa igualdade
ambos são constantes iguais ao mesmo λ ∈ R. Obtemos assim duas equações:
d2 T1 (x)
− λ · T1 (x) = 0, com T1 (0) = T1 (L) = 0, T1 6≡ 0,
dx2
e
dT2 (t)
− α2 λ · T2 (t) = 0, T2 (t) 6≡ 0.
dt
Destas duas equações ordinárias, iniciaremos analisando a equação em x, pois ela
está equipada de informação extra T1 (0) = T1 (L) = 0. As soluções de
d2 T1 (x)
− λ · T1 (x) = 0, com T1 (0) = T1 (L) = 0, T1 6≡ 0,
dx2
pela Afirmação 2.1 do Capı́tulo 40, dependem de λ:
√ √
• i): se λ < 0, são da forma T1 (x) = a · cos( −λ · x) + b · sin( −λ · x). As
analisaremos a seguir.
• ii): se λ = 0, são da forma T1 (x) ≡ D · t + E, com D, E ∈ R. Mas como
T1 (0) = 0 então E = 0. Como T1 (L) = 0 então√
T1 (x) ≡

0 e será descartada.
λ·x − λ·x
• iii): se λ > 0, são da forma T1√(x) = a ·√e +b·e . Como T√ 1 (0) = 0
λ·L − λ·L
então a + b = 0. Como a · (e −e ) = 0 então a = 0 ou λ = 0.
Qualquer uma dessas condições dá T1 (x) ≡ 0. Descartado.
Na situação que restou, ou seja, o item i):
√ √
T1 (x) = a · cos( −λ · x) + b · sin( −λ · x),
para que tenhamos T1 (0) = T1 (L) = 0 precisamos que a = 0, pois 0 = T1 (0) = a. E
de √
0 = T1 (L) = b · sin( −λ · L)
obtemos que √
−λ · L = π · n, n ∈ N,
ou seja que
π 2 n2
−λ = 2 .
L
4. PROBLEMAS DE ESFRIAMENTO UNIDIMENSIONAIS 722

Em resumo, as soluções de
d2 T1 (x) π 2 n2
+ · T1 (x) = 0, com T1 (0) = T1 (2π) = 0, T1 6≡ 0
dx2 L
são da forma:
π·n
Bn · sin( · x), n ∈ N, Bn ∈ R
L
Voltando à segunda equação, ficamos com:
dT2 (t) π 2 n2
+ α2 2 · T2 (t) = 0, T2 (t) 6≡ 0,
dt L
cujas soluções são
2 n2 π 2 ·t
An · e−α L2 , An ∈ R.
Afirmo que as somas finitas
N
X 2 n2 π 2 ·t π·n
Cn · e−α L2 · sin( · x),
n=1
L

(onde Cn = An · Bn ) são soluções.


Isso se deve à linearidade da equação diferencial parcial e também pela homo-
geneidade da equação diferencial e da condição de contorno:
T (0, t) = T (L, t) = 0.
Mais ainda, se pode provar que a série infinita
+∞
X 2 2
2 n π ·t π·n
T (x, t) = Cn · e−α L2 · sin( · x)
n=1
L

é solução da equação.
Como:
+∞
X π·n
C ≡ f (x) = T (x, 0) = Cn · sin( · x),
n=1
L
reconhecemos os Cn como os coeficientes de uma série de Fourier de senos da função
constante f ≡ C, do Exemplo 1 da Seção 2 do Capı́tulo 46: Cn = 0 se n ∈ N é par e
Cn = 4C

se n ∈ N é ı́mpar.
Suponho para a figura a seguir o caso bem particular:
C ≡ 1, L=π e α = 1.
Na figura a seguir dou o truncamento até n = 11 de
+∞
4 X 1 2
T (x, t) = · · e−(2n−1) ·t · sin((2n − 1) · x)
π n=1 2n − 1
1 1 1 1 1
com t = , , , , ,1
40 30 10 6 2
CAPÍTULO 47. EQUAÇÕES DIFERENCIAIS PARCIAIS 723

0.8

0.6

0.4

0.2

0 0.5 1 1.5 2 2.5 3

Problema 2 - não-homogêneo:

Uma situação mais geral: um arame isolado do ambiente, exceto pelos extremos,
com uma distribuição de temperatura f (x) ≡ C, x ∈ [0, L] no tempo t = 0, que
começa a sofrer resfriamento segundo:
∂ 2 T (x, t) ∂T (x, t)
α2 · 2
= .
∂x ∂t
Só que agora
T (0, t) ≡ c < C e T (L, t) ≡ 0, ∀t > 0.
Ou seja, a condição de fronteira não é mais homogênea.
O que fazer ? Pois agora a soma de soluções ∀n que fizemos no Problema 1 já
não é mais possı́vel. A idéia é reduzir este Problema 2 a um problema do tipo do
Problema 1, e usar aquela técnica.
Para isso considere
c
f (x) = − · x + c,
L
qu claramente satisfaz
d2 f (x)
f (0) = c, f (L) = 0, ≡0
dx2
e obviamente
df
,
dt
pois f (x) não depende de t.
Considere
ˆ t) := T (x, t) − f (x).
T (x,
4. PROBLEMAS DE ESFRIAMENTO UNIDIMENSIONAIS 724

Note que esta função recai no problema anterior, pois:


ˆ t)
∂ 2 T (x, ˆ t)
∂ T (x,
α2 · =
∂x2 ∂t
e
ˆ t) = T (0, t) − f (0) = c − c = 0 e T (L,
T (0, ˆ t) = T (L, t) − f (L) = 0,
apenas a distribuição inicial de calor mudou, pois:
ˆ 0) = T (x, 0) − f (x) = (C − c) + c · x.
T (x,
L
Ou seja, no final da resolução do novo problema, segundo as técnicas que de-
screvemos no Problema 1, teremos que calcular coeficientes de Fourier de uma função
linear: (C − c) + Lc · x. E depois obtemos:
ˆ t) + f (x).
T (x, t) = T (x,
Note que os termos exponenciais de T (x, ˆ t) vão para zero quando t cresce e portanto
os gráficos de T (x, t) - para cada t - tendem ao d3 f (x).
Para L = π, α = 1, os coeficientes de Fourier agora são
Z π
2 c
Cn := · ((C − c) + · x) · sin(nx) dx
π 0 L
e
+∞
c X 2
T (x, t) = − · x + c + Cn · e−n ·t · sin(n · x).
L n=1
Na figura a seguir usei C = 1 e c = 21 , truncamento em n = 11, com t =
1 1 1 1 1 1
, , , , , 1 e pus também o gráfico da reta − 2π
40 30 10 6 2
· x + 12 .
1

0.8

0.6

0.4

0.2

0 0.5 1 1.5 2 2.5 3

x
CAPı́TULO 48

O operador de Laplace e as equações do calor e da onda

1. Laplaciano em coordenadas polares e esféricas


Precisaremos nas Seções seguintes expressar o Laplaciano, inicialmente dado em
coordenadas cartesianas (x, y) ou (x, y, z) em coordenadas polares (r, θ) ou em esféricas
(ρ, θ, φ).
Este último sistema põe
0 ≤ ρ, 0 ≤ θ2π e 0 ≤ φ < π.
A figura a seguir mostra bem que:
x = (ρ sin(φ)) · cos(θ), y = (ρ sin(φ)) · sin(θ) e z = ρ cos(φ).

θ
y

Afirmação 1.1.
i): Seja y = f (x, y) com derivadas de segunda ordem contı́nuas1.
2 2
O Laplaciano ∂∂xf2 + ∂∂yf2 se escreve em cordenadas polares (r, θ) como:

1 ∂2f 1 ∂( r · ∂f
∂r
)
2 2
+ · .
r ∂θ r ∂r
ii): Seja y = f (x, y, z) com derivadas de segunda ordem contı́nuas.
1Para ∂2f ∂2f
que possamos usar ∂x∂y = ∂y∂x

725
1. LAPLACIANO EM COORDENADAS POLARES E ESFÉRICAS 726

∂2f ∂2f ∂2f


O Laplaciano ∂x2
+ ∂y 2
+ ∂z 2
se escreve em cordenadas esféricas (r, θ, φ), com
0 < φ < π, como:
∂2f 2 ∂f 1 ∂2f cot(φ) ∂f 1 ∂2f
+ · + · + + · .
∂ρ2 ρ ∂ρ ρ2 ∂φ2 ρ2 ∂φ ρ2 sin2 (φ) ∂θ2
Demonstração.
De i):
Temos
x = x(r, θ) = r cos(θ) e y = y(r, θ) = r sin(θ),
logo
f (x, y) = f (x(r, θ), y(r, θ))
e pela regra da composta em duas variáveis:
∂f ∂f ∂x ∂f ∂y
= · + · =
∂θ ∂x ∂θ ∂y ∂θ
∂f ∂f
=− · sin(θ) r + · cos(θ) r.
∂x ∂y
Para que o que segue fique mais claro, lembre que:
∂f ∂f
(x, y) = (x(r, θ), y(r, θ))
∂x ∂x
∂f ∂f
(x, y) = (x(r, θ), y(r, θ)).
∂y ∂y
Também:
∂2f ∂2f ∂f ∂2f ∂f
2
=− · sin(θ) r − · cos(θ) r + · cos(θ) r − · sin(θ) r =
∂θ ∂x∂θ ∂x ∂y∂θ ∂y
∂2f ∂2f ∂f
= −[ · (− sin(θ) r) + cos(θ) r] · sin(θ) r − · cos(θ) r+
∂x2 ∂x∂y ∂x
∂2f ∂2f ∂f
+[ · (− sin(θ) r) + 2 cos(θ) r] · cos(θ) r − · sin(θ) r =
∂y∂x ∂y ∂y
∂2f 2 2 ∂2f 2 2 ∂2f
= 2
sin (θ) r + 2
cos (θ) r − 2 · sin(θ) cos(θ)r 2 −
∂x ∂y ∂x∂y
∂f ∂f
− · cos(θ) r − · sin(θ) r.
∂x ∂y
Por outro lado,
∂f ∂f ∂f
r· =r·( · cos(θ) + · sin(θ))
∂r ∂x ∂y
e daı́:
∂( r · ∂f
∂r
) ∂f ∂f ∂2f ∂2f
= · cos(θ) + · sin(θ) + r cos(θ) + r sin(θ) =
∂r ∂x ∂y ∂x∂r ∂y∂r
∂f ∂f ∂2f ∂2f ∂2f
= cos(θ) + · sin(θ) + 2 · r cos2 (θ) + 2 · r sin2 (θ) + 2 · sin(θ) cos(θ) r.
∂x ∂y ∂x ∂y ∂x∂y
CAPÍTULO 48. O OPERADOR DE LAPLACE E AS EQUAÇÕES DO CALOR
E DA ONDA 727

Agora é só fazer a soma e obter:


∂f
1 ∂2f 1 ∂( r · ∂r ) ∂2f ∂2f
+ · = + .
r 2 ∂θ2 r ∂r ∂x2 ∂y 2

De ii):
Contas mais longas, mas do mesmo estilo, agora usando que:
x = ρ sin(φ) cos(θ), y = ρ sin(φ) sin(θ) e z = ρ cos(φ).


2. Estado estacionário do calor num disco e expansão em séries de


Fourier
Esta Seção 2 e a próxima Seção 4 têm um bocado de heurı́stica, e várias afirmações
sem prova. Mas mostra como a teoria de equações diferenciais parciais está ligada a
problemas fı́sicos concretos, bem como conecta a teoria com coisas já aprendidas no
Curso. 11
Minhas referências são o livro do Simmons, Differential equations, de H. F. Davis,
Fourier series and orthogonal functions e de Boyce-diPrima.
Imagine uma disco maciço de raio 1 feito de material homogêneo, cujos pontos
serão parametrizados em coordenadas polares 0 ≤ r ≤ 1, 0 ≤ θ ≤ 2π.
Imagine agora que o cı́rculo de raio 1 que é a fronteira é mantido aquecido, de tal
modo que sua temperatura é dada por uma função:
f = f (θ), 0 ≤ θ ≤ 2π.
E suponha que isso é feito até que a temperatura no interior do disco não mude mais.
Nesse momento a temperatura T (r, θ) do disco anula o Laplaciano em coordenadas
polares:
1 ∂2T 1 ∂( r · ∂T
∂r
)
+ · =0
r 2 ∂θ2 r ∂r
Queremos resolver esta equação, com a condição (chamada condição de fronteira)
T (1, θ) = f (θ),
e para isso fazemos ainda mais uma suposição, de separação de variáveis, ou seja, de
que2:
T (r, θ) = T1 (r) · T2 (θ).
Então a equação que queremos resolver vira:
1 d2 T2 (θ) 1 dT1 (r) d2 T1 (r)
0=· T1 (r) · + · T2 (θ) · + T2 (θ) · ,
r2 dθ2 r dθ dr 2
de onde se obtem, após multiplicar por r 2 :
1 d2 T1 (r) dT1 (r) −1 d2 T2 (θ)
· (r 2 · + r · ) = · .
T1 (r) dr 2 dr T2 (θ) dθ2
2são as aplicações fı́sicas que justificam essas suposições
2. ESTADO ESTACIONÁRIO DO CALOR NUM DISCO E EXPANSÃO EM
SÉRIES DE FOURIER 728

A observação agora é que o lado direito é função apenas de θ enquanto o esquerdo é


função apenas de r. A conclusão é que ambos são constantes = λ ∈ R. O que produz
duas equações diferenciais ordinárias:
d2 T1 (r) dT1 (r)
r2 · 2
+r· − λ · T1 (r) = 0,
dr dr
e
d2 T2 (θ)
+ λ · T2 (θ) = 0.
dθ2
As soluções desta última equação, de acordo com a Afirmação 2.1 do Capı́tulo 40 são
da forma: √ √
• i): T2 (θ) = a · e −λ·x + b · e− −λ·x se λ < 0. Mas queremos que T2 (θ) tenha
perı́odo 2π. Logo excluı́mos essa possibilidade.
• ii): T2 (θ) = a · x + b, se λ = 0. Só será periódica, e de fato constante, se
a = 0. √ √
• iii): T2 (θ) = a · cos( λ · θ) + b · sin( λ · θ), se λ > 0, que são periódicas.
Só que se tomamos, no Caso ii), λ = 0 então a equação (de Euler)
d2 T1 (r) dT1 (r)
r2 · 2
+r· − λ · T1 (r) = 0
dr dr
vira:
d2 T1 (r) dT1 (r)
r2 · 2
+r· = 0,
dr dr
cuja solução, pela Afirmação 1.1 do Capı́tulo 40, é:
T1 (r) = c + d · ln(r);
se d 6= 0 essas soluções não ficam limitadas quando r → 0, o que é inaceitável do
ponto de vista da situação fı́sica tratada. Mas se d = 0 então a conclusão geral é que:
T (r, θ) = T1 (r) · T2 (θ) ≡ c · a
é uma função constante. √
No Caso iii), para termos T2 (θ) com perı́odo 2π, o λ > 0 tem de ser

λ = n ∈ N,
11 ou seja,
λ = n2 .
A equação de Euler
d2 T1 (r) dT1 (r)
r2 · 2
+r· − λ · T1 (r) = 0,
dr dr
cuja equação asssociada é r 2 = n2 , de acordo com a Afirmação 1.1 do Capı́tulo 40,
tem soluções:
T1 (r) = a · r n + b · r −n ,
só que a parte r −n fica ilimitada quando r → 0 e é abandonada.
Portanto, a conclusão é que funções do tipo:
Tn = a · r n · cos(n · θ) + b · r n · cos(n · θ), n∈N
são soluções das equações que nos interessam.
CAPÍTULO 48. O OPERADOR DE LAPLACE E AS EQUAÇÕES DO CALOR
E DA ONDA 729
P
A idéia é buscar para a solução desejada combinações lineares n an Tn dessas
soluções e, de fato, séries infinitas do tipo:
+∞
X
T (r, θ) = a0 + r n · (an cos(nθ) + bn sin(nθ)).
n=1
Como
+∞
X
f (θ) = T (1, θ) = a0 + an cos(nθ) + bn sin(nθ),
n=1
reconhecemos aı́ uma Série de Fourier, para a qual sabemos que3:
Z 2π
1
a0 := · f (φ) dφ,
2π 0
e Z 2π Z 2π
1 1
an := · f (φ) cos(nφ) dφ e bn := · f (φ) sin(nφ) dφ.
π 0 π 0

3. A fórmula integral de Poisson


Concluı́mos na Seção anterior que a temperatura no disco unitário em estado
estacionário é dada em coordenadas polares por:
+∞
X
T (r, θ) = a0 + r n · (an cos(nθ) + bn sin(nθ)) =
n=1
Z 2π +∞ Z
1 X
n1 2π
= f (φ) dφ + r ·( f (φ) cos(nφ) dφ · cos(nθ)+
2π 0 n=1
π 0
Z
1 2π
+ f (φ) sin(nφ) dφ · sin(nθ))),
π 0
onde f = f (θ) é a temperatura no cı́rculo unitário.
Tomando r ≤ r < 1 podemos garantir a convergência em módulo e uniforme da
série e trocar a ordem entre a integração e a soma infinita. Assim obtemos
Z +∞
1 2π 1 X n
T (r, θ) = f (φ) · [ + r · (cos(nφ) cos(nθ) + sin(nφ) sin(nθ))]dφ =
π 0 2 n=1
Z +∞
1 2π 1 X n
= f (φ) · [ + r · cos(n(φ − θ))] dφ.
π 0 2 n=1
Para continuarmos faremos uma incursão sobre os números Complexos e séries infini-
tas Complexas.
Suponha que para um número complexo com |z| < 1 faça sentido e convirja a
série geométrica complexa:
+∞
X 1
zn = .
n=0
1−z
3uso φ ao invés da variável t pois φ lembra a variável θ enquanto que t evocaria o tempo
3. A FÓRMULA INTEGRAL DE POISSON 730

Ou seja, que valha:


+∞
X 1 z
zn = −1 = .
n=1
1−z 1−z
Agora escreva z com |z| < 1 na forma polar:
z = r · eIψ := r · (cos(ψ) + I · sin(ψ)), 0 ≤ r < 1, 0 ≤ ψ < 2π.
Portanto:
+∞
1 X n 1 z
+ z = + =
2 n=1 2 1−z
1 1−z
+z·
= =
2 |1 − z|2
1 1 − r cos(ψ) + Ir sin(ψ)
= + (r cos(ψ) + Ir sin(ψ)) · =
2 |1 − r · cos(ψ) − Ir sin(ψ)|2
1 r cos(ψ) − r 2 + Ir sin(ψ)
= + =
2 1 + r 2 − 2r cos(ψ)
1 − r 2 + I · 2r sin(ψ)
= .
2 · (1 + r 2 − 2r cos(ψ))
Mas vale:
z n = r n · (cos(nψ) + I · sin(nψ))
portanto:
+∞ +∞ +∞
1 X n 1 X n X
+ z = + r · cos(nψ) + I · r n · sin(nψ) =
2 n=1 2 n=1 n=1
1 − r2 2r sin(ψ)
= 2
+I · .
2 · (1 + r − 2r cos(ψ)) 2 · (1 + r 2 − 2r cos(ψ))
Comparando as partes Real e Imaginária obtemos:
+∞
1 X n 1 − r2
+ r · cos(nψ) = .
2 n=1 2 · (1 + r 2 − 2r cos(ψ))
Assim termina a incursão sobre os complexos.
Fazendo
ψ =φ−θ
então a integral que tı́nhamos obtido:
Z +∞
1 2π 1 X n
T (r, θ) = f (φ) · [ + r · cos(n(φ − θ))] dφ
π 0 2 n=1
pode ser reescrita agora como:
Z 2π
1
T (r, θ) = f (φ) · K(r, θ, φ) dφ,
2π 0
onde fizemos
1 − r2
K(r, θ, φ) := ;
1 + r 2 − 2r cos(φ − θ)
CAPÍTULO 48. O OPERADOR DE LAPLACE E AS EQUAÇÕES DO CALOR
E DA ONDA 731

este é o núcleo de Poisson no disco unitário e que facilmente se generaliza para discos
de raio R como
R2 − r 2
K(r, θ, φ, R) := 2 .
R + r 2 − 2rR cos(φ − θ)
Ou seja que, para expressarmos a solução do problema de distribuição estacionária
de calor no disco T (r, θ) basta fazermos a integral do produto da temperatura no bordo
com o núcleo de Poisson. Essa idéia se generaliza para outros domı́nios que não são
discos.

4. Estado estacionário do calor na esfera e série de polinômios de


Legendre
A equação diferencial parcial (linear, de segunda ordem) que rege a mudança da
temperatura 4 T = T (x, y, z, t) é:
∂2T ∂2T ∂2T ∂T
k2 · (
2
+ 2
+ 2
)= .
∂x ∂y ∂z ∂t
Ou seja, se o Laplaciano num ponto P é negativo, então num entorno de P há
menos calor que em P e portanto a temperatura de P diminui; já se o Laplaciano
num ponto P é positivo, então num entorno de P há mais calor que em P e portanto
a temperatura de P aumenta.
Quando se estabiliza a temperatura temos:
∂2T ∂2f ∂2f
+ + = 0.
∂x2 ∂y 2 ∂z 2
Imagine uma bola maciça de raio 1 feita de material homogêneo, cujos pontos serão
parametrizados em coordenadas esféricas por 0 ≤ ρ ≤ 1, 0 ≤ θ ≤ 2π e 0 ≤ φ ≤ π.
Imagine agora que a superfı́cie da bola é mantida aquecida, de tal modo que a
temperatura na superfı́cie é dada por uma função f (1, θ, φ), que para simplificar,
vamos supôr é constante ao logo de cada meridiano, ou seja,
f (1, θ, φ) = f (φ), 0 ≤ φ ≤ π.
E suponha que isso é feito até que a temperatura no interior da esfera não mude
mais. Nesse momento a temperatura T (ρ, θ, φ) da esfera, que suponho da forma
T (ρ, φ), anula o Laplaciano em coordenadas esféricas:
∂2T 2 ∂T 1 ∂2T cot(φ) ∂T
+ · + · + = 0.
∂ρ2 ρ ∂ρ ρ2 ∂φ2 ρ2 ∂φ
(expressão mais simples que na Afirmação 1.1 pois T (ρ, φ) independende de θ).
Isso pode ser escrito, multiplicando por ρ2 , se 0 < φ < π, como:
∂2T ∂T ∂2T cos(φ) ∂T
ρ2 · 2
+ 2ρ · + 2
+ · =
∂ρ ∂ρ ∂φ sin(φ) ∂φ
∂T ∂T
∂(ρ2 · ∂ρ
) 1 ∂(sin(φ) · ∂φ
)
= + · = 0.
∂ρ sin(φ) ∂φ
4bem como alguns processos de difusão em meios homogêneos
4. ESTADO ESTACIONÁRIO DO CALOR NA ESFERA E SÉRIE DE
POLINÔMIOS DE LEGENDRE 732

Agora queremos resolver esta equação, com a condição (chamada condição de


fronteira)
T (1, φ) = f (φ),
e para isso fazemos ainda mais uma suposição, como na Seção anterior, de separação
de variáveis, ou seja, de que5:
T (ρ, φ) = T1 (ρ) · T2 (φ).
Então a equação que queremos resolver vira:
dT1 (ρ) d2 T1 (ρ) d2 T2 (φ) cos(θ) dT2 (φ)
0 = 2ρ · T2 (φ) · + ρ2 · T2 (φ) · + T1 (ρ) · + · T1 (ρ) · ,
dρ dρ2 dφ2 sin(θ) dφ
o que pode ser re-escrito como:
1 dT1 (ρ) d2 T1 (ρ) −1 cos(θ) dT2 (φ) d2 T2 (φ)
· [2ρ · + ρ2 · ] = · [ · + ].
T1 (ρ) dρ dρ2 T2 (φ) sin(θ) dφ dφ2
Como na Seção anterior, a observação agora é que o lado direito é função apenas de
φ enquanto o esquerdo é função apenas de ρ.
A conclusão é que ambos são constantes = λ ∈ R. O que produz duas equações
diferenciais ordinárias:
d2 T1 (ρ) dT1 (ρ)
ρ2 · 2
+ 2ρ · − λ · T1 (ρ) = 0
dρ dρ
e
d2 T2 (φ) cos(θ) dT2 (φ)
+ · + λ · T2 (φ) = 0.
dφ2 sin(θ) dφ
A equação
d2 T1 (ρ) dT1 (ρ)
ρ2 · 2
+ 2ρ · − λ · T1 (ρ) = 0
dρ dρ
é uma equação de Euler, que tratamos na Afirmação 1.1 do Capı́tulo 40.
A equação indicial associada é:
r(r − 1) + 2 · r − λ = 0
ou seja, cujas raı́zes r1 , r2 são:

−1 ± 1 + 4λ
.
2
Se fosse 1 + 4λ = 0 então a Afirmação 1.1 do Capı́tulo 40 diria que as soluções
são da forma:
1 1
T1 (ρ) = a · ρ− 2 + b · ln(ρ) · ρ− 2 .
Mas este tipo de solução não é limitada quando ρ → 0 e não tem significado fı́sico
relevante.
Agora se 1 + 4λ < 0, então
p
−1 −(1 + 4λ) √
r1 = +I · e r2 = r1 , onde I = −1
2 2
5são as aplicações fı́sicas que justificam essas suposições
CAPÍTULO 48. O OPERADOR DE LAPLACE E AS EQUAÇÕES DO CALOR
E DA ONDA 733

e novamente a Afirmação 1.1 do Capı́tulo 40 diria que as soluções são da forma:


p p
−1 −(1 + 4λ) −1 −(1 + 4λ)
T1 (ρ) = a · ρ 2 · cos( ln(ρ)) + b · ρ 2 · sin( ln(ρ)).
2 2
Novamente soluções sem sentido fı́sico, pois não são limitadas quando ρ → 0.
Resta então que:
1 + 4λ > 0
e que, pela mesma Afirmação, as soluções são da forma:
√ √
−1+ 1+4λ −1− 1+4λ
T1 (ρ) = a · ρ 2 +b·ρ 2 .
Para que haja limitação na solução quando ρ → 0, imponho que:

−1 + 1 + 4λ
>0
2
e faço b = 0, ficando então comanda

−1+ 1+4λ
T1 (ρ) = a · ρ 2 .
Agora se faz a suposição de que o número:

−1 + 1 + 4λ
>0
2
seja da forma √
−1 + 1 + 4λ
= n ∈ {0} ∪ N
2
ou seja, de que:
λ = n · (n + 1)
e
T1 (ρ) = a · ρn , n ∈ N.
Retornando á segunda equação:
d2 T2 (φ) cos(θ) dT2 (φ)
+ · + λ · T2 (φ) = 0,
dφ2 sin(θ) dφ
esta agora se escreve:
d2 T2 (φ) cos(θ) dT2 (φ)
+ · + n(n + 1) · T2 (φ) = 0.
dφ2 sin(θ) dφ
Agora façamos:
τ = cos(φ) e φ = arccos(τ ), onde φ ∈ (0, π),
e portanto a última equação pode ser re-escrita:
d2 T2 (φ) τ dT2 (φ)
2
+√ · + n(n + 1) · T2 (φ) = 0.
dφ 1−τ 2 dφ
Por outro lado, como T2 = T2 (φ(τ )):
dT2 dT2 dφ dT2 −1
= · = · (√ )
dτ dφ dτ dφ 1 − τ2
4. ESTADO ESTACIONÁRIO DO CALOR NA ESFERA E SÉRIE DE
POLINÔMIOS DE LEGENDRE 734

e
d2 T2 1 d2 T2 τ dT2
2
= 2 2
− 3 .
dτ 1 − τ dφ (1 − τ 2 ) 2 dφ
De onde se obtêm:
d2 T2 dT2
(1 − τ 2 ) · 2
− 2τ + n(n + 1)T2 =
dτ dτ
d2 T2 (φ) τ dT2 (φ)
= 2
+√ · + n(n + 1) · T2 (φ) = 0,
dφ 1−τ 2 dφ
nossa equação. Agora reconhecemos em
d2 T2 dT2
(1 − τ 2 ) · − 2τ + n(n + 1)T2 = 0
dτ 2 dτ
a equação de Legendre do Capı́tulo 41.
Como mais uma vez queremos que T2 (τ ) fique limitada para
−1 ≤ τ ≤ 1 ou seja 0 ≤ φ ≤ π,
então temos que tomar as soluções limitadas em [−1, 1] da Equação de Legendre
d2 T2 dT2
(1 − τ 2 ) · − 2τ + n(n + 1)T2 = 0,
dτ 2 dτ
ou seja, como se pode provar, :
T2 (τ ) = a · Pn (τ ) = a · Pn (cos(φ)),
onde Pn é o n-ésimo polinômio de Legendre. Isso para cada n = 0, 1, 2, 3, . . ., portanto
pelo que vimos encontramos soluções particulares da forma:
Tn = an · ρn · Pn (cos(φ)), an ∈ R.
Pela linearidade do Laplaciano, o que faz é somar essas soluções particulares Tn ,
mais propriamnte, se considera uma série infinita como candidata a solução:
+∞
X
T (ρ, φ) := an · ρn · Pn (cos(φ));
n=0

e como foi dada


f (φ) = T (1, φ)
então terı́amos como consequência
+∞
X
f (φ) = an · Pn (cos(φ)),
n=0

ou seja,
+∞
X
f (arccos(τ )) = an · Pn (τ ).
n=0
CAPÍTULO 48. O OPERADOR DE LAPLACE E AS EQUAÇÕES DO CALOR
E DA ONDA 735

Baseados na ortogonalidade dos polinômios de Legendre Pn (τ ) (Seção 5 do Capı́tulo


40) e imitando o que fizemos para determinar os coeficientes das séries de Fourier, se
pode provar que6 que:
Z 1
1
an = (n + ) · f (arccos(τ )) · Pn (τ ) dτ.
2 −1

Por esta razão os polinômios de Legendre são chamados de harmônicos esféricos.

Exemplo:
Considerei uma fatia da bola de raio 1, aquela quando θ = π2 , pois nesse caso:
π π
x = ρ sin(φ) cos( ) = 0, y = ρ sin(φ) sin( ) = ρ sin(φ) e z = ρ cos(φ),
2 2
a fatia obtida cortando com o plano x = 0 no espaço.
Variando agora φ de 0 a π estamos indo do pólo Norte ao Sul, pois z = ρ cos(φ).
Então pensei numa função f (φ) que dá a temperatura na superfı́cie que imite o
que acontece na temperatura do globo terrestre, em que há temperaturas negativas
no Norte e no Sul e com máximas em geral no equador, φ = π2 :
π2
f (φ) = 1 − (φ − ,
)
que tem:
π2 π
f (0) = f (π) = 1 − ≈ −1.4 e f ( ) = 1.
4 2
Fiz no Maple approximações numéricas dos coeficientes a0 , . . . , a6 e obtive
6
X
T (ρ, φ) ≈ an · ρn · Pn (cos(φ)) ≈
n=0

1 3
≈ 0.5325988995 − 0.8305268694 10−14 · ρ · cos(φ) − 1.111111111 · ρ2 · (− + cos(φ)2 )−
2 2
5 3 3 35 15
−0.1223884111 10−14·ρ3 ·( cos(φ)3 − cos(φ))−0.3200000000·ρ4·( + cos(φ)4 − cos(φ)2 )−
2 2 8 8 4
63 35 15
−0.3914846856 10−15 · ρ5 · ( cos(φ)5 − cos(φ)3 + cos(φ))−
8 4 8
5 231 315 105
−0.1509297052 · ρ6 · (− + cos(φ)6 − cos(φ)4 + cos(φ)2 ).
16 16 16 16
Também esta aproximação T (ρ, φ) dá que:
lim T (ρ, φ) ≈ 0.5325988995.
ρ→0

6se f ((arccos(τ )) for tratável


5. EXERCÍCIOS 736

5. Exercı́cios
1
Exercı́cio 5.1. i) Seja U(x, y) = − √ um potencial gravitacional no plano (x, y)
x2 +y 2
de uma partı́cula com massa situada na origem . Mostre que no plano fora da origem:
1
∇U = 3 .
(x2 + y 2) 2
1
ii) Seja V (x, y, z) = − √ um potencial gravitacional no espaço (x, y, x) de
x2 +y 2 +z 2
uma partı́cula com massa situada na origem . Mostre que no espaço fora da origem
∇V ≡ 0.
CAPı́TULO 49

Equação da onda e as vibrações de cordas e membranas

1. Vibração de uma corda com extremos fixos, sem atrito


Considero uma corda de comprimento L presa nos extremos (a corda está posta
no eixo dos x com extremos em 0 e L), com densidade constante ρ e submentida a
uma tensão T . Vamos supor que seus pontos se deslocam apenas na direção vertical
e que a amplitude desse deslocamento é pequena.
Sem de deter na obtenção da equação diferencial, postulo que o deslocamento
vertical y(x, t) satisfaz:
∂ 2 y(x, t) 1 ∂ 2 y(x, t) 1 ρ
= · , onde = .
∂x2 k2 ∂t2 k2 T
As condições iniciais do problema são:
∂y(x, 0)
y(x, 0) = g(x) e = h(x),
∂t
que dão um formato e uma velocidade inicial à corda.
As condições que que expressam o fato dos extremos estarem fixos são:
y(0, t) = y(L, t) = 0, ∀t ≥ 0
e
∂y(0, t) ∂y(L, t)
= = 0, ∀t ≥ 0.
∂x ∂x
O problema é descrever o que acontece para t > 0, onde a idealização do problema
(que abstrai atrito e amortecimentos) conduzirá a uma solução em que a corda vibra
para sempre.
A separação de variáveis:
y(x, t) = y1 (x) · y2 (t)
produz:
∂ 2 (y1 (x) · y2 (t)) 1 ∂ 2 (y1 (x) · y2 (t))
− · =
∂x2 k2 ∂t2
∂ 2 y1 (x) 1 ∂ 2 y2 (t)
= · y 2 (t) − · y 1 (x) · = 0,
∂x2 k2 ∂t2
de onde:
1 ∂ 2 y1 (x) 1 1 ∂ 2 y2 (t)
· = · · .
y1 (x) ∂x2 k 2 y2 (t) ∂t2
737
1. VIBRAÇÃO DE UMA CORDA COM EXTREMOS FIXOS, SEM ATRITO 738

O lado esquerdo só depende de x e o direito só de t, portanto devem ser constantes e
iguais a λ ∈ R. Então
∂ 2 y1 (x)
− λ · y1 (x) = 0
∂x2
e
∂ 2 y2 (t)
− λ · k 2 · y2 (t) = 0.
∂t2
Para que a solução desta última equação seja periódica a única possibilidade é que
λ < 0. Então
√ √
y2 (t) = a · cos( −λk · t) + b · sin( −λk · t), a, b ∈ R.
Com λ < 0 as soluções de
∂ 2 y1 (x)
− λ · y1 (x) = 0
∂x2
são √ √
y1 (x) = c · cos( −λ · x) + d · sin( −λ · x), c, d ∈ R.
Mas quero que y(x, t) = y1 (x) · y2 (t) verifique y(0, t) ≡ 0 e para isso preciso que se
anule um coeficiente:
c = 0.

E para que y(L, t) = d · sin( −λ · L) ≡ 0 preciso que:

−λ · L = n · π, n ∈ N
ou seja,
√ n·π
−λ = , n∈N
L
e portanto:
n·π n·π n·π
d · sin( · x) · [a · cos( · k · t) + b · sin( · k · t)]
L L L
é uma solução que depende de n ∈ N fixado (chamdo um modo normal de vibração
da corda e quando n = 1 o modo fundamental ). Pela linearidade da equação o que se
faz é buscar somas dessas soluções, mas ∀n ∈ N:
+∞
X n·π n·π n·π
y(x, t) := sin( · x) · [an · cos( · k · t) + bn · sin( · k · t)]
n=1
L L L
onde as constantes dn foram absorvidas nas outras.
A determinação dos coeficientes an , bn depende de se fazer uso das condições ini-
ciais:
+∞
X n·π
y(x, 0) = an · sin( · x) = g(x)
n=1
L
e (por derivação termo a termo e posterior avaliação em t = 0):
+∞
∂y(x, 0) X n·π n·π
= bn · · k · sin( · x) = h(x).
∂t n=1
L L
Se vê então que os an e os
n·π
bn · ·k
L
CAPÍTULO 49. EQUAÇÃO DA ONDA E AS VIBRAÇÕES DE CORDAS E
MEMBRANAS 739

são os coeficientes de Fourier de g(x) e h(x) respectivamente. E esses nós já sabemos
como determinar.

2. Vibração de uma corda infinita: Fórmula de D’Alembert


Considero uma corda de densidade constante ρ submetida a uma tensão T mas
que agora é pensada como tendo comprimento infinito, disposta ao longo do eixo dos
x.
Vamos supor que seus pontos se deslocam apenas na direção vertical e que a
amplitude desse deslocamento é pequena.
Como antes já fizemos, postulo que o deslocamento vertical y(x, t) satisfaz:
∂ 2 y(x, t) 1 ∂ 2 y(x, t) 1 ρ
= · , onde = .
∂x2 k2 ∂t2 k 2 T
As condições iniciais do problema são:
∂y(x, 0)
y(x, 0) = g(x) e = h(x), x∈R
∂t
que dão um formato e uma velocidade inicial à corda.
Considero a seguinte mudança de variáveis:
u := x + k · t e v := x − k · t.
Afirmo que nessas novas variáveis a função y(x, t) = y(x(u, v), t(u, v)) satisfaz1 a
equação diferencial:
∂2y
= 0.
∂u ∂v
Essa forma da equação que rege a vibração de uma corda ou uma onda é chamada
de forma canônica.
De fato, pela regra da derivada da composta:
∂y ∂y ∂x ∂y ∂t ∂y 1 ∂y −1
= · + · = · + · ( ),
∂v ∂x ∂v ∂t ∂v ∂x 2 ∂t 2k
pois
u+v
x=
2
e
u−v
t= .
2k
Mas não podemos esquecer que:
∂y ∂y
e
∂x ∂t
são funções de x = x(u, v) e de y = y(u, v). Portanto:
∂y ∂y
∂2y ∂( 1 · 1
− 2k · )
= 2 ∂x ∂t
=
∂u∂v ∂u
1Supondo que essa função tem derivadas parciais de segunda ordem em x, t que são elas mesmas
funções contı́nuas
2. VIBRAÇÃO DE UMA CORDA INFINITA: FÓRMULA DE D’ALEMBERT740

1 ∂ 2 y ∂x 1 ∂ 2 y ∂t 1 ∂ 2 y ∂x 1 ∂ 2 y ∂t
= · 2· + · · − · · − · · =
2 ∂x ∂u 2 ∂t∂x ∂u 2k ∂x∂t ∂u 2k ∂t2 ∂u
1 ∂2y 1 ∂2y 1 ∂2y 1 ∂2y
= + − − = 0,
4 ∂x2 4k ∂t∂x 4k ∂x∂t 4k 2 ∂t2
onde na última igualdade usei que
∂2y ∂2y
=
∂t∂x ∂x∂t
se y(x, t) tiver derivadas de segunda ordem contı́nuas (Lema de Schwarz) e
∂ 2 y(x, t) 1 ∂ 2 y(x, t)
− · = 0.
∂x2 k2 ∂t2
Mas
∂y
∂2y ∂ ∂v
= =0
∂u∂v ∂u
∂y
quer dizer que ∂v
só depende de v:
∂y
= z(v).
∂v
E agora integrando em v obtenho:
Z
y(u, v) = z(v)dv + q(u) =: p(v) + q(u);

ou seja:
y(x(u, v), t(u, v)) = p(v) + q(u) = p(x − k · t) + q(x + k · t).
As condições iniciais para t = 0 dão:
y(x, 0) = p(x − k · 0) + q(x + k · 0) = p(x) + q(x) = g(x)
e
∂y(x, 0)
= p′ (x) · (−k) + q ′ (x) · (k) = k · (−p′ (x) + q ′ (x)) = h(x),
∂t
de onde
1
−p′ (x) + q ′ (x) = · h(x)
k
e daı́ integrando: Z x
1
−p(x) + q(x) = · h(ξ)dξ + C.
k 0
Junto com:
p(x) + q(x) = g(x)
obtemos um sistema de duas equações lineares, de onde:
Z x
1 1 C
q(x) = · g(x) + · h(ξ)dξ +
2 2k 0 2
e Z x
1 1 C
p(x) = · g(x) − · h(ξ)dξ − =
2 2k 0 2
Z 0
1 1 C
= · g(x) + · h(ξ)dξ − .
2 2k x 2
CAPÍTULO 49. EQUAÇÃO DA ONDA E AS VIBRAÇÕES DE CORDAS E
MEMBRANAS 741

Já que essas são as expressões de p(x) e q(x) ∀x então posso usá-las para p(x − k · t)
e q(x + k · t), de onde sai a fórmula clásssica (Fórmula de D’Alembert):
Z x+k·t
g(x − k · t) + g(x + k · t) 1
y(x, t) = p(x − k · t) + q(x + k · t) = + h(ξ) dξ.
2 2k x−k·t

Algumas observações: a expressão


y(x, t) = p(x − k · t) + q(x + k · t)
já indica que a solução é uma superposição de uma onda que se move para frente com
velocidade k e de outra que se move para trás com velocidade k. Pois para cada t0
fixado os gráficos de p(x − k · t0 ) são trasladados horizontais para a frente do gráfico
de y = p(x) enquanto que os gráficos de q(x + k · t0 ) são trasladados horizontais para
trás do gráfico de y = q(x).
Suponha agora, por um momento, que h(x) ≡ 0; portanto, pela Fórmula de
D’Alembert:
g(x − k · t) + g(x + k · t)
y(x, t) = p(x − k · t) + q(x + k · t) = .
2
Se a função y(x, 0) = g(x) é identicamente nula fora de um certo intervalo [a, b] então:
g(x − k · t) + g(x + k · t)
y(x, t) =
2
diz que para t > 0 o mesmo formato do formato do gráfico de y = g(x) se propaga
para frente e para trás, com velocidade k, mas com metade da amplitude.
Agora, ao contrário suponha y(x, 0) = g(x) ≡ 0 e que h(x) ≥ 0 é uma função
contı́nua não nula apenas em um certo intervalo [a, b]. Este caso corresponde a uma
corda sendo percutida numa pequena região [a, b] (por exemplo uma corda de piano
percutida pelo martelo do piano). Então a fórmula:
Z x+k·t
1
y(x, t) = h(ξ) dξ
2k x−k·t
descreve a propagação ao longo da corda da percussão e diz que enquanto [x − k ·
t, x + k · t] não intersectar [a, b] a corda continua sem deslocamento vertical. E que
mesmo se o intervalo [x − k · t, x + k · t] contendo [a, b] for bem maior que [a, b] o
deslocamento vertical continua da ordem de:
Z x+k·t
1
h(ξ) dξ.
2k x−k·t

3. Modos normais de vibração de um tambor circular e as funções de


Bessel
Considero um tambor circular, de raio a, e quero determinar os modos de vibração
da membrana do tambor. Suponho que o deslocamento de cada ponto da membrana
é apenas vertical, dado pela função
z = w(x, y, t)
3. MODOS NORMAIS DE VIBRAÇÃO DE UM TAMBOR CIRCULAR E AS
FUNÇÕES DE BESSEL 742

e que o bordo não se move, ou seja,


w(x, y, t) = 0 se x2 + y 2 = 1.
Sem me deter, por enquanto, em como se obtém a equação diferencial que rege
esse fenômeno, postulo que verifica:
∂2w ∂2w 1 ∂2w
+ = · ,
∂x2 ∂y 2 k 2 ∂t2
onde se pode dar a interpretação fı́sica:
1 ρ
2
= ,
k T
onde ρ é a densidade (suposta constante) da membrana e T é a tensão aplicada à
membrana.
A primeira separação de variáveis que vamos impôr é pensar que:
w(x, y, t) = u(x, y) · q(t).
Então
∂ 2 (u(x, y) · q(t)) ∂ 2 (u(x, y) · q(t)) 1 ∂ 2 (u(x, y) · q(t))
+ = 2·
∂x2 ∂y 2 k ∂t2
dá:
∂ 2 u(x, y) ∂ 2 u(x, y) u(x, y) ∂ 2 q(t)
( + ) · q(t) = ·
∂x2 ∂y 2 k2 ∂t2
e portanto (supondo u 6= 0 se x2 + y 2 < 1):
1 ∂ 2 u(x, y) ∂ 2 u(x, y) 1 1 ∂ 2 q(t)
·( + ) = · · .
u(x, y) ∂x2 ∂y 2 k 2 q(t) ∂t2
Já que o lado esquerdo é função só de x, y e o direito só de t concluimos que:
1 ∂ 2 u(x, y) ∂ 2 u(x, y)
·( + )=λ∈R
u(x, y) ∂x2 ∂y 2
e que
1 1 ∂ 2 q(t)
· · = λ ∈ R.
k 2 q(t) ∂t2
Na situação idealizada que consideramos, após ser posta em movimento a membrana
oscila para sempre, portanto queremos que a função q(t) seja periódica. Como ela
verifica:
∂ 2 q(t)
= λ · k 2 · q(t)
∂t2
só será periódica se λ < 0, de acordo com a Afirmação 2.1 do Capı́tulo 40. E nesse
caso: √ √
q(t) = a · cos( −λk 2 · x) + b · sin( −λk 2 · x).
A outra equação ficou então:
∂ 2 u(x, y) ∂ 2 u(x, y)
+ = λ · u(x, y), com λ < 0.
∂x2 ∂y 2
CAPÍTULO 49. EQUAÇÃO DA ONDA E AS VIBRAÇÕES DE CORDAS E
MEMBRANAS 743

Como o domı́nio é o disco x2 + y 2 ≤ a é natural pensarmos em usar coordenadas


polares r, θ onde u(x, y) = u(r, θ) e onde o laplaciano é:
1 ∂ 2 u(r, θ) 1 ∂(r · ∂u∂r
)
· + · .
r2 ∂θ2 r ∂r
Fazendo uma nova separação de variáveis
u(r, θ) = R(r) · Θ(θ)
nossa equação
∂R(r)·Θ(θ)
1 ∂ 2 R(r) · Θ(θ) 1 ∂(r · ∂r
)
2
· 2
+ · = λ · R(r) · Θ(θ)
r ∂θ r ∂r
produz (após fazer as derivações exigidas e reagrupar):
1 ∂2 Θ 2 r ∂R r 2 ∂ 2 R
· = λr − − .
Θ ∂θ2 R ∂r R ∂r 2
Como o lado esquerdo só depende de θ e o direito só de r concluimos que:
1 ∂2 Θ
· =µ∈R
Θ ∂θ2
e que
r ∂R r 2 ∂ 2 R
λr 2 −
− = µ ∈ R.
R ∂r R ∂r 2
Como vimos há pouco, para que Θ(θ) seja periódica temos necessariamente que ter:
µ < 0.
Então:
√ √
Θ(θ) = a · cos( −µ · θ) + b · sin( −µ · θ).
Se pode justificar que:

−µ = n ∈ N
e mesmo estender ao caso
µ = 0,
que corresponde a uma solução independente de θ (simetria circular).
A outra equação, lembrando que µ = −n2 e após multiplicar por R(r), fica da
forma:
∂2R ∂R
r2 · 2 + r · + R · (−λ · r 2 − n2 ) = 0.
∂r ∂r
Já que
−λ > 0,
esta equação se parece muito com a equação de Bessel2:
∂ 2 (α · Jn (x)) ∂(α · Jn (x))
x2 · + x · + (α · Jn (x)) · (x2 − ν 2 ) = 0, ν ≥ 0, α∈R
∂x2 ∂x
2Na notação já indico que se trata de um múltiplo α da função de Bessel de primeira ordem
Jν (x), pois as funções de Bessel de segunda ordem Yν (x) produzem soluções ilimitadas em x = 0, o
que não faz sentido no nosso caso
3. MODOS NORMAIS DE VIBRAÇÃO DE UM TAMBOR CIRCULAR E AS
FUNÇÕES DE BESSEL 744

De fato, como vimos no primeiro item da Afirmação 3.1 do Capı́tulo 43 a mudança


de variável:

x = −λ · r
leva a equação de Bessel na nossa equação

2 ∂2R ∂R
r · 2 +r· + R · (−λ · r 2 − n2 ) = 0.
∂r ∂r
Em suma, concluo que:

R(r) = α · Jn ( −λr).
Agora intervém a exigência de que:

R(a) = 0

pois queremos que a borda circular do tambor fique fixa. Ou seja, já que α 6= 0:

Jn ( −λa) = 0

Pra simplificar a exposição suponhamos que

a=1

e portanto

−λ
é um zero da n-ésima função de Bessel de primeira ordem.
Já vimos na Seção 2 do Capı́tulo 43 que há uma infinidade de zeros para cada
n ∈ N fixado. E desses zeros se conhecem aproximações numéricas. E na Afirmação
3.1 vimos as relações de ortogonalidade entre funções de Bessel Jν (λx), para disitintos
λ.
Ou seja, para cada n fixado (n ∈ N ∪ {0}), há uma infinidade de pontos:

−λ =: λn,m , m ∈ N

ordenados em ordem crescente, que são zeros de Jn .


Variando n, m obtemos os modos normais de vibração da membrana do tambor:

w(r, θ, t) = Jn (λn,m r)·[a1 ·cos(n·θ)+a2 ·sin(n·θ)]·[a3 ·cos(λn,m ·k·x)+a4 ·sin(λn,m ·k·x)].

O caso n = 0 dá soluções com simetria circular:

w(r, t) = J0 (λ0,m r) · a1 · [a3 · cos(λ0,m · k · x) + a4 · sin(λ0,m · k · x)].

Para n = 0 mas aumentando o m ∈ N aparecem m anéis concêntricos em fase


oposta, como ilustra a figura:
CAPÍTULO 49. EQUAÇÃO DA ONDA E AS VIBRAÇÕES DE CORDAS E
MEMBRANAS 745

Mas para n = 1 há a solução do tipo


w(r, θ, t) = J1 (λ1,m r) · sin(θ) · [a3 · cos(λ1,m · k · x) + a4 · sin(λ1,m · k · x)].
que se anula para θ = 0, π, ou seja ao longo do diâmetro horizontal do cı́rculo. O
semidisco superior se move em fase oposta ao semidisco inferior, como ilustra a Figura:

Quando n = 1 e m = 2 além desses semidiscos superior e inferior em fase oposta


se juntam dois anéis concêntricos em fase oposta, veja Figura:

E assim por diante.


Parte 4

Cálculo diferencial e integral sobre os


números Complexos
CAPı́TULO 50

Um portal para o Cálculo Complexo

Neste Capı́tulo faço aparecer as propriedades do Cálculo sobre os Complexos, de


modo ainda concreto e matematicamente informal, a partir do estudo de fluxos em
estado estacionário.
Devo muito ao livro de Stephen Fisher, Complex variables, Segunda edição, Dover,
1986.
Os números complexos z = a + I · b podem ser somados, subtraı́dos, multiplicados:
(a + I · b) + (c + I · d) := (a + b) + I · (b + d),
(a + I · b) − (c + I · d) := (a − c) + I · (b − d),
(a + I · b) · (c + I · d) = a · c + a · I · d + I · b · c + b · d · I 2 =
= (ac − bd) + I · (ad + bc),
2
onde usei que I = −1.
E essas operações são comutativas e distributivas, como o leitor pode conferir.
O que é crucial é que se z 6= 0 então z tem inverso multiplicativo.
De fato, se z = a + I · b isso significa que a 6= 0 ou que b 6= 0. Então a2 + b2 > 0 e
faz sentido o número Complexo:
a b
w := 2 2
−I · 2
a +b a + b2
e para ele
a b a b
z·w = w·z =( 2 2
·a+ 2 2
· b) + I · ( 2 2
·b− 2 · a) =
a +b a +b a +b a + b2
= 1 + I · 0 = 1,
−1
ou seja, w = z .
A noção de conjugação para z = a + I · b é dada por:
z := a − I · b
e permite expressar w = z −1 de modo mais elegante:
z
w = 2 , onde |z|2 := a2 + b2 .
|z|
É óbvio que z = z e que z1 + z2 = z1 + z2 . O leitor pode comprovar que
z1 · z2 = z1 · z2 .
No que segue retomo a definição que dei na Seção 5 do Capı́tulo 31:
ez = ex+I·y := ex · (cos(y) + I sin(y)) =
= ex cos(y) + I · ex sin(y).
749
750

O leitor pode verificar que:

ez = ez .

Vamos usar as noções de soma, produto, inverso multiplicativo e de conjugação


para definir no que segue algumas aplicações:

f : C → C.

As Figuras a seguir mostram f (z) = z, f (z) = z 2 e f (z) = ez como campos de


vetores:

0,5

y 0
-1 -0,5 0 0,5 1
x

-0,5

-1

Fig.: O campo vetorial produzido por f (z) = ez

y 0
-2 -1 0 1 2
x

-1

-2

Fig.: O campo vetorial produzido por f (z) = z


CAPÍTULO 50. UM PORTAL PARA O CÁLCULO COMPLEXO 751

y 0
-2 -1 0 1 2
x

-1

-2

Fig.: O campo vetorial produzido por f (z) = z 2

Podemos imaginar que se tratam de fluxos de partı́culas em estado estacionário, ou


seja, na situação em que há um campo de velocidades que só depende da posição (x, y)
e não do tempo. As partı́culas se movimentam segundo esse campo de velocidades,
ocupando o lugar deixado por outras.
As Figuras a seguir mostram algumas curvas integrais desses três campos. Na
Seção 3 veremos qual o método geral para encontrá-las. Representama trajetória
seguida pelas partı́culas submetidas a esses campos de velocidades.

y 0
-1 -0,5 0 0,5 1 1,5 2
x

-1

-2

Fig.: Algumas curvas integrais ex · sin(y) = C do campo f (z) = ez


752

y 0
-2 -1 0 1 2
x

-1

-2

Fig.: Algumas curvas integrais x · y = C (hipérboles) do campo f (z) = z

y 0
-2 -1 0 1 2
x

-1

-2

Fig.: Algumas curvas integrais y 3 − 3x2 y = C (cúbicas) do campo f (z) = z 2


Como as curvas integrais do campo f (z) = z 2 são cúbicas, e como as cúbicas
são estrelas neste Curso, resolvi plotar uma delas separadamente (formada de três
ramos).

y 0
-2 -1 0 1 2
x

-1

-2
CAPÍTULO 50. UM PORTAL PARA O CÁLCULO COMPLEXO 753

Fig.: Uma curva integral y 3 − 3x2 y = C (cúbica) do campo


√ f (z) = z 2 ,
onde se vê as três assı́ntotas y = 0 e y = ± 3x.

Tome agora qualquer cı́rculo Cz0 ,r centrado em z0 ∈ C, de raio r. Se z0 = a+I ·b ≡


(a, b) então posso parametrizar Cz0 ,r por:
γ(t) = ( a + r · cos(t), b + r · sin(t) ), t ∈ [0, 2π].
O vetor tangente de γ é:
τγ := (−r · sin(t), r · cos(t) ).
Considero1
Z Z 2π
f (z) · τz := f (a + r · cos(t), b + r · sin(t)) · τz dt.
Cz0 ,r 0

Agora considere o vetor normal 2ao cı́rculo Cz0 ,r :


nγ := (r · cos(t), r · sin(t))
e defina a integral
Z Z 2π
f (z) · nz := f (a + r · cos(t), b + r · sin(t)) · nz dt.
Cz0 ,r 0

Afirmação 0.1.
Tome qualquer cı́rculo Cz0 ,r centrado em z0 ∈ C, de raio r.
i): Então Z Z
z · τz = 0 e z · nz = 0.
Cz0 ,r Cz0 ,r
ii): Então Z Z
z 2 · τz = 0 e z 2 · nz = 0.
Cz0 ,r Cz0 ,r
iii): Então: Z Z
ez · τz = 0 e ez · nz = 0.
Cz0 ,r Cz0 ,r

Demonstração.
De i):
Neste caso:
Z Z 2π
z · τz = −ar sin(t) − r 2 sin(t) cos(t) − br cos(t) − r 2 sin(t) cos(t) dt =
Cz0 ,r 0
Z 2π Z 2π Z 2π
2
= −ar sin(t) dt − br cos(t) dt − 2r sin(t) cos(t) dt = 0.
0 0 0
1onde o · no integrando é o produto escalar do vetor do plano representado por f (z) ∈ C com o
vetor tangente
2há a possibilidade de se tomar o sinal oposto nessa definição de vetor normal, mas escolhemos
este.
754

E Z Z 2π
z · nz = ar cos(t) + r 2 cos2 (t) − br sin(t) − r 2 sin2 (t) dt =
Cz0 ,r 0
Z 2π Z 2π Z 2π
2
= ar cos(t)dt − br sin(t)dt + r cos2 (t) − sin2 (t)dt =
0 0 0
Z 2π Z 2π Z 2π
2
= ar cos(t)dt − br sin(t)dt + r cos(2 t)dt = 0.
0 0 0

De ii):
Só para diminuir o tamanho da conta suponho que z0 = (0, 0).
Como:
z 2 = x2 − y 2 + I · 2xy = x2 − y 2 − I · 2xy,
então facilmente se obtem:
Z Z 2π
3
z 2 · τz = −r 3 cos2 (t) sin(t) − sin3 (t) dt = 0,
Cz0 ,r 0

pois a primitiva em questão é:


sin2 (x) cos(x) 2 cos(x)
− cos3 (x) + + + C.
3 3
Já Z Z 2π
3
z2 · nz = r cos3 (t) − 2 · sin2 (t) cos(t) dt = 0,
Cz0 ,r 0
pois agora a primitiva é:
−2 sin3 (x) cos2 (x) sin(x) 2 sin(x)
= + + + C.
3 3 3
De iii):
Temos: Z
ez · τz =
Cz0 ,r
Z 2π
= (ea+r cos(t) cos(b + r sin(t)), −ea+r cos(t) sin(b + r sin(t)) · (−r sin(t), r cos(t)) dt =
0
Z 2π
= −rea+r cos(t) · ( cos(b + r sin(t)) sin(t) + sin(b + r sin(t)) cos(t) ) dt = 0,
0
pois a primitiva em questão é:
b + r sin(t) 2
ea+r cos(t) · (−1 + 2 cos( ) ) + C.
2
Já Z
ez · nz =
Cz0 ,r
Z 2π
= (ea+r cos(t) cos(b + r sin(t)), −ea+r cos(t) sin(b + r sin(t)) · (r cos(t), r sin(t)) dt =
0
CAPÍTULO 50. UM PORTAL PARA O CÁLCULO COMPLEXO 755
Z 2π
= rea+r cos(t) · (cos(b + r sin(t)) cos(t) − sin(b + r sin(t)) sin(t)) dt = 0,
0
pois a primitiva em questão é:
b + r sin(t) b + r sin(t)
2ea+r cos(t) sin( ) cos( ) + C.
2 2


Se γ : [c, d] → C, γ(t) = (x(t), y(t) é uma curva parametrizada, fechada, sem


auto-intersecções3 Definimos para h(z) = u(z) + I · v(z):
Z Z Z d
h(z) · τγ := udx + vdy := u(x(t), y(t)) · x′ (t) + v(x(t), y(t)) · y ′(t) dt
γ γ c
e
Z Z Z d
h(z) · nγ := udy − vdx := u(x(t), y(t)) · y ′ (t) − v(x(t), y(t)) · x′ (t) dt.
γ γ c
R
Definição 0.1. Se um campo v tem γ z · τz = 0 ao longo de toda curva fechada sem
auto-intersecções, então vRé chamado de conservativo.
Se um campo v tem γ z · nz = 0 ao longo de toda curva fechada sem auto-
intersecções, então se diz que que v não tem fontes nem sumidouros.
O que a Afirmação 0.1 indica, apesar de só tratar de cı́rculos, é que os três exemplos
acima são conservativos e não tem fontes nem sumidouros.

Agora considero a seguinte aplicação do plano no plano:


1
f : C \ {0} → C, f (z) := .
z
Note que:
1 1 z z
= ( ) = ( 2) = 2.
z z |z| |z|
Se vemos z 6= 0 como um vetor no plano C = R2 , o fato que
z
f (z) = 2
|z|
nos diz que f associa a cada vetor reprsentado por z um outro vetor que tem a mesma
direção e sentido que z mas:
• |f (z)| > |z| se |z| < 1
• |f (z)| < |z| se |z| > 1
• f (z) = z se |z| = 1.

3Dizemos que é fechada se γ(c) = γ(d) e dizemos que é sem autosintersecções se γ(t1 ) = γ(t2 )
somente se t1 = t2 ou t1 = c e t2 = d.
756

A Figura o ilustra:

0,5

y 0
-1 -0,5 0 0,5 1
x

-0,5

-1

Essa f : C \ {0} → C, f (z) := z1 é chamada em Geometria de inversão no Cı́rculo


unitário centrado na origem;
O Exercı́cio 6.2 dá o modo de construir f (z) geometricamente a partir de z.
Note que ela é uma involução: f (f (z)) = z, isto é, f ≡ f −1 .
Tome qualquer cı́rculo Cz0 ,r centrado em z0 = a + I · b ≡ (a, b), de raio r,
parametrizado por:
γ(t) = ( a + r · cos(t), b + r · sin(t) ), t ∈ [0, 2π].
Se (0, 0) 6∈ Cz0 ,r , posso considerar
Z Z
z
f (z) · τz := · τz .
Cz0 ,r Cz0 ,r |z|2
e Z Z
z
f (z) · nz := · nz .
Cz0 ,r Cz0 ,r |z|2
Afirmação 0.2.
Denote no que segue Dz0 ,r o disco fechado cujo bordo é Cz0 ,r .

i): Tome qualquer cı́rculo Cz0 ,r centrado em z0 ∈ C, de raio r, tal que (0, 0) 6∈
Cz0 ,r . Então Z
1
· τz = 0.
Cz0 ,r z
ii): Se (0, 0) 6∈ Dz0 ,r , então
Z
1
· nz = 0.
Cz0 ,r z
CAPÍTULO 50. UM PORTAL PARA O CÁLCULO COMPLEXO 757

iii): Se z0 = (0, 0) então


Z
1
· nz = 2π.
Cz0 ,r z

Demonstração.

Do item i):
1 z
Temos f (z) = z
= |z|2
e
Z
z
· τz =
Cz0 ,r |z|2

Z 2π
−ar sin(t) − r 2 sin(t) cos(t) + br cos(t) + r 2 sin(t) cos(t)
= dt =
0 a2 + b2 + r 2 + 2ar cos(t) + 2br sin(t)

Z 2π
−ar sin(t) + br cos(t)
= dt,
0 a2 + b2 + r 2 + 2ar cos(t) + 2br sin(t)
onde reconhecemos derivadas logarı́tmicas e portanto primitivas:

1
· ln |a2 + b2 + r 2 + 2ar cos(t) + 2br sin(t)| + C.
2

Do item ii):
z
Temos f (z) = |z|2
e
Z
f (z) · nz =
Cz0 ,r

Z 2π
ar cos(t) + r 2 cos2 (t) + br sin(t) + r 2 sin2 (t)
= dt =
0 a2 + b2 + r 2 + 2ar cos(t) + 2br sin(t)

Z 2π
r 2 + ar cos(t) + br sin(t)
= dt
0 a2 + b2 + r 2 + 2ar cos(t) + 2br sin(t)
Faz sentido considerar uma função ângulo

θ(z) = θ(x + I · y),

que dá o ângulo que z (como vetor com base na origem) forma com o eixo positivo dos
x, pois (0, 0) 6∈ Dz0 ,r . Ela é derivável e ademais |θ(z1 ) − θ(z2 )| < 2π para quaisquer
dois z1 , z2 ∈ Dz0 ,r
Veja a Figura:
758

z0

θ
θ

Como vimos na prova do item ii) da Afirmação 7.1 do Capı́tulo 36:


∂θ x
= 2
∂y x + y2
e
∂θ −y
= 2 ,
∂x x + y2
o que, para pontos (a + r cos(t), b + r sin(t)) de Cz0 ,r , significa:
∂θ x a + r cos(t)
= 2 =
∂y x + y2 a2 + b2 + r 2 + 2ar cos(t) + 2br sin(t)
e
∂θ −y −b − r sin(t)
= 2 2
= 2 2 2
.
∂x x +y a + b + r + 2ar cos(t) + 2br sin(t)
Portanto, como
dx dy
( , ) = (−r sin(t), r cos(t))
dt dt
vemos que
Z Z 2π
∂θ dy ∂θ dx
f (z) · nz = · + · =
Cz0 ,r 0 ∂y dt ∂x dt
Z 2π
= θ′ (t) dt =
0
Z (a+r,b)
= dθ = 0.
(a+r,b)

Do item iii):
Se z0 = (0, 0) então: Z
f (z) · nz =
C(0,0),r
Z 2π
r 2 · cos2 (t) + r 2 sin2 (t)
= dt = 2π,
0 r2
que indica que o ângulo determinado por (r, 0) está mal definido, pois a ele se soma
2π quando fazemos um giro completo no cı́rculo e voltamos em (r, 0).
CAPÍTULO 50. UM PORTAL PARA O CÁLCULO COMPLEXO 759

O que a Afirmação 0.2 indica, apesar de só tratar de cı́rculos, é que f (z) = 1z
é conservativo e que num pequeno entorno de cada ponto z0 ∈ C, z0 6= 0, não tem
fontes nem sumidouros.
Mas para a fonte z0 = 0 se define a potência do campo z1 como
Z
1
· nz = 2π
Cz0 ,r z
−1 −z
Note que se tomo agora o campo z
= |z|
, ilustrado a seguir:

0,5

y 0
-1 -0,5 0 0,5 1
x

-0,5

-1

então ele tem um sumidouro em z0 = 0 e se define a potência desse sumidouro


por Z
−1
− · nz = 2π.
Cz0 ,r z

1. O Teorema de Green e as Relações de Cauchy-Riemann


O que significa para as funções coordenadas u(z), v(z) de um campo h(z) :=
u(z) + I · v(z) (com u e v deriváveis, com derivadas parciais contı́nuas) o fato de ser
conservativo e não ter fontes nem sumidouros ?
Ou seja, o fato de ter
Z Z
h(z) · τγ = 0 e h(z) · nγ = 0,
γ γ

para qualquer curva fechada sem autointersecção γ.


Seja γ : [c, d] → C, γ(t) = (x(t), y(t) e seu interior U. Por exemplo, se γ é um
cı́rculo, U é o disco que ele limita.
1. O TEOREMA DE GREEN E AS RELAÇÕES DE CAUCHY-RIEMANN 760

Se U não tem buracos (é simplesmente conexo), pelo Teorema de Green 4 temos:
Z Z
0 = h(z) · τγ := udx + vdy =
γ γ
Z
∂v ∂u
= − (
) dxdy
U ∂x ∂y
e Z Z
0 = h(z) · nγ := udy − vdx =
γ γ
Z
∂u ∂v
= ( + ) dxdy.
U ∂x ∂y
Ora, se acontecesse que
∂v ∂u
− 6= 0
∂x ∂y
ou se acontecesse que
∂u ∂v
+ 6= 0
∂x ∂y
então, pelo Princı́pio de Inércia das funções contı́nuas, essas funções seriam não-nulas
numa pequena região U. E para uma pequena curva γ cercando essa região terı́amos
por Green Z Z
h(z) · τγ 6= 0 ou h(z) · nγ 6= 0.
γ γ
Como isso não ocorre, pela nossa suposição, temos que concluir que valem:
∂v ∂u ∂u ∂v
− ≡0 e + ≡ 0,
∂x ∂y ∂x ∂y
ou seja,
∂v ∂u ∂u ∂v
≡ e =− .
∂x ∂y ∂x ∂y
Como já vimos, a Afirmação 0.1 sugere que os campos z, z 2 e ez são conservativos e
não têm fontes nem sumidouros. Portanto se denotamos por
u(z) + Iv(z)
as coordenadas de cada um desses três campos z, z 2 ou ez , temos que:
∂v ∂u ∂u ∂v
≡ e ≡− .
∂x ∂y ∂x ∂y
Portanto para as coordenadas
u(z) − I · v(z) = u(z) + I · (−v(z))
de cada um dos campos conjugados z, z 2 ou ez podemos escrever:
∂(−v) ∂u ∂u ∂(−v)
≡− e ≡ .
∂x ∂y ∂x ∂y
4Por enquanto o assumo, sem prová-lo
CAPÍTULO 50. UM PORTAL PARA O CÁLCULO COMPLEXO 761

Obtivemos assim para as coordenadas u(z) + I(−v(z)) dos campos z, z 2 ou ez o que


se chama de relações de Cauchy-Riemann.

2. A integral complexa e a idéia da primitiva Complexa


Definição 2.1. (Integral Complexa)
Seja h : C → C uma função com domı́nio e valores complexos.
Denoto h(z) = u(z) + I · v(z), ou seja, h((x, y)) = u(x, y) + I · v(x, y) .
E seja γ uma curva parametrizada no plano, derivável, γ : [c, d] → C, γ(t) =
(x(t), y(t)). Façamos duas definições:
Z Z d
h(z) dz := (u(t) + I · v(t)) · (x′ (t) + I · y ′ (t)) dt :=
γ c
Z d Z d
′ ′
:= u(t) · x (t) − v(t) · y (t) dt + I · v(t) · x′ (t) + u(t) · y ′(t) dt.
c c

Afirmação 2.1.
Z Z Z
f (z) dz = f (z) · τz + I · f (z) · nz .
Cz0 ,r Cz0 ,r Cz0 ,r

Demonstração.
Imediata após a Definição 2.1.


Afirmação 2.2.
i): Para qualquer cı́rculo Cz0 ,r :
Z Z
z dz = 0 e z 2 dz = 0,
Cz0 ,r Cz0 ,r

bem como: Z
ez dz = 0.
Cz0 ,r
ii): Se (0, 0) 6∈ Dz0 ,r , então
Z
1
dz = 0.
Cz0 ,r z
Mas se z0 = (0, 0) então Z
1
dz = 2π · I.
Cz0 ,r z
Demonstração.
Com a Afirmação 2.1 vemos que isso é exatamente o que dizem as Afirmações 0.1
e 0.2.

2. A INTEGRAL COMPLEXA E A IDÉIA DA PRIMITIVA COMPLEXA 762

O item i) da Afirmação 2.2 faz parecer que estamos criando funções inúteis, pois
suas integrais ao longo de cı́rculos são zero. Mas é o contrário, esta anulação é que
nos permitirá criar novas funções no plano para as quais valerá um tipo de teorema
fundamental do Cálculo.
De fato, suponha que não só em cı́rculos temos
Z
f (z) dz = 0
Cz0 ,r

mas façamos a suposição surpreendente de que em qualquer curva fechada sem auto-
intersecção γ tenhamos Z
f (z) dz = 0.
γ
Afirmo que, fixado um ponto z0 arbitrário no domı́nio da f , poderı́amos então
definir: Z z Z
G(z) := f (z)dz := f (z)dz
z0 Cz0 ,z
usando qualquer curva parametrizada (derivável) que sai de z0 e chega em z.
Em termos gerais, a idéia é que se tomo qualquer outra Cz′ 0 ,z que sai de z0 e chega
em z sem intersectar Cz0 ,z terı́amos:
Z Z
f (z)dz = f (z)dz,
Cz0 ,z Cz′ 0 ,z

pois Z Z
f (z)dz − f (z)dz =
Cz0 ,z Cz′ 0 ,z
Z Z
= f (z)dz + f (z)dz =
Cz0 ,z −Cz′ 0 ,z
Z Z
= f (z)dz = f (z)dz = 0,
Cz0 ,z −Cz′ 0 ,z γ

onde γ = Cz0 ,z − Cz′ 0 ,z é a curva fechada sem auto-intersecção que se forma ao irmos
de z0 a z por Cz0 ,z e retornarmos a z0 pela Cz′ 0 ,z .
Afirmação 2.3. i): Se para toda curva fechada sem auto-intersecção γ temos
Z
f (z) dz = 0
γ

então a função Z z
G(z) := f (z)dz
z0

está bem definida e G′ (z) = f (z). Ou seja, G(z) é uma primitiva Complexa de f (z).

ii): Escrevendo G(z) = U(z) + I · V (z) temos


∂U ∂V
G′ (z) = +I · =
∂x ∂x
CAPÍTULO 50. UM PORTAL PARA O CÁLCULO COMPLEXO 763

∂V ∂U
= −I · ,
∂y ∂y
de onde
∂U ∂V ∂V ∂U
≡ e ≡− ,
∂x ∂y ∂x ∂y
que são as relações de Cauchy-Riemann.
Demonstração.
Por enquanto justifico apenas o item ii). Deixo i) para a Seção 1 do Capı́tulo 51.

f (z) − f (z)
G′ (z) = lim
z→z z−z
e esse limite pleno nos permite tomar qualquer direção de aproximação de z para z;
o que é exigido apenas é que:
||z − z|| → 0.
Então posso tomar por exemplo uma direção horizontal para aproxima z e obter:
para G(z) = U(z) + I · V (z) e z = a + Ib:
U(a + h + Ib) + I · V (a + h + Ib)
G′ (z) = lim =
h→0 h + I0
U(a + h, b) V (a + h, b)
= lim +I · =
h→0 h h
∂U ∂V
=: ( +I · )(z).
∂x ∂x
1
Ou posso tomar uma direção vertical de aproximação para z e obter, já que I
= −I:
U(a + I(b + h)) + I · V (a + I(b + h))
G′ (z) = lim =
h→0 Ih
−IU(a + I(b + h)) V (a + I(b + h))
= lim + =
h→0 h h
∂U ∂V
= (−I · + )(z).
∂y ∂y
Comparando as duas expressões:
∂V ∂U ∂U ∂V
G′ (z) = −I · = +I ·
∂y ∂y ∂x ∂x
obtemos:
∂U ∂V ∂V ∂U
≡ e ≡− .
∂x ∂y ∂x ∂y

3. CURVAS INTEGRAIS COMO PARTE IMAGINÁRIA DAS PRIMITIVAS
COMPLEXAS 764

3. Curvas integrais como parte imaginária das primitivas Complexas


Afirmação 3.1. Ainda sob as hipóteses das Afirmação 2.3. Se
Z z
G(z) := f (z)dz = U(z) + I · V (z),
z0
então:

i): as curvas dadas implicitamente por V (z) = C são curvas integrais do campo
vetorial definido por f (z).

ii) A função U(z) é o potencial do campo f (z), ou seja,


∂U ∂U
( , ) = f (z).
∂x ∂y

iii) As curvas V (z) = C e U(z) = C são ortogonais.


Demonstração.
De i):
Pelo Teorema da Função implı́cita (Teorema 2.1 do Capı́tulo 15), onde a curva
V (z) = C é um gráfico y = y(x), temos
dy − ∂V
= ∂V∂x ,
dx ∂y

portanto o vetor tangente a V (z) = C é:


∂V ∂V
( ,− ).
∂y ∂x
Por outro lado, pela Afirmação 2.3 e pelo Teorema Fundamental do Cálculo sobre
os Complexos, temos que
∂U ∂V
G′ (z) = +I · = f (z).
∂x ∂x
Ora, as relações de Cauchy-Riemann dão, em particular, que:
∂U ∂V
≡ .
∂x ∂y
e portanto
∂V ∂V ∂U ∂V
( ,− )=( ,− ) = f (z).
∂y ∂x ∂x ∂x
De ii):
Como
∂U ∂V
−I · = f (z),
∂x ∂x
basta usar a relação de Cauchy-Riemann:
∂V ∂U
− = .
∂x ∂y
CAPÍTULO 50. UM PORTAL PARA O CÁLCULO COMPLEXO 765

De iii):
Queremos ver se há anulação do produto escalar:
∂U ∂U ∂V ∂V
( , )·( , ) ≡ 0.
∂x ∂y ∂x ∂y
Ora, pela duas relações de Cauchy-Riemann:
∂U ∂V ∂U ∂V ∂U ∂U ∂U ∂U
· + · = · (− )+ · ≡0
∂x ∂x ∂y ∂y ∂x ∂y ∂y ∂x


Foi assim que numa Seção 50 obtivemos as curvas integrais dos três campos f (z) =
ez . f (z) = z e f (z) = z 2 . Pois
Z Z Z
z z z2 z3
e dz = e + C, z dz = + C, e z 2 dz = +C
2 3
e suas partes imaginárias V (z) são respectivamente:
y 3 − 3x2 y
ex · sin(y), x·y e .
3
Já suas partes Reais U(z) são respectivamente:
x2 y 2 x3
ex · cos(y), − e − xy 2
2 2 3
Nas figuras a seguir coloco juntas as curvas ortogonais U(z) = C e V (z) = C
desses três exemplos:

y 0
-1 -0,5 0 0,5 1 1,5 2
x

-1

-2

Fig.: Curvas ortogonais ex · sin(y) = C e ex · cos(y) = C.


4. A EXPONENCIAL COMPLEXA E OS RAMOS DO LOGARITMO
COMPLEXO 766

y 0
-2 -1 0 1 2
x

-1

-2

x2 y2
Fig.: Curvas ortogonais x · y = C e 2
− 2
= C.

y 0
-2 -1 0 1 2
x

-1

-2

x3
Fig.: Curvas ortogonais 3
− xy 2 = C e y 3 − 3x2 y = C.

4. A exponencial Complexa e os ramos do logaritmo Complexo


A definição que demos:
ea+I·b := ea · (cos(b) + I · sin(b))
faz que a exponencial complexa não seja injetiva.
De fato, note que ela é periódica, no sentido de que
ez+2πI = ez .
CAPÍTULO 50. UM PORTAL PARA O CÁLCULO COMPLEXO 767

Vista mais em detalhe, note que ez manda as retas horizontais y = C em


ea · (cos(C) + I sin(C))
que são semi-retas saindo da origem na direção do vetor unitário (cos(C) + I sin(C).
E que ez manda segmentos verticais dados por x = C e 0 ≤ y ≤ π em semicı́rculos
de raio eC centrados na origem:
eC · (cos(y) + I sin(y)), 0 ≤ y ≤ π.
Se vê então que ez manda a faixa horizontal H0,π : 0 ≤ y ≤ π no semiplano
H0 : y ≥ 0.
Afirmo que essa aplicação ez : H0,π → H0 é bijetora: de fato, dado w := x + I · y
com y > 0, determino primeiro qual ângulo b, com 0 ≤ b ≤ π, que o vetor (x, y)
forma com o eixo dos x > 0. Então:
w = x + I · y = r · (cos(b) + I sin(b)),
para 0 < r = |x + Iy| = |w|.
E agora tomo a := ln(|w|).
Portanto esse a + I · b é tal que ea+I·b = x + I · y = w.
Essas operações que fizemos para descobrir o a + Ib enviado em w = x + Iy pela
z
e podem ser resumidas como:
z = x + I · y = |w| · ((cos(b) + I sin(b)) 7→ z = ln(|w|) + I · θ
onde θ é o ângulo entre 0 e π formado pelo vetor (x, y) com o eixo dos x > 0.
A Figura a seguir ilustra essas observações:
y y

πI

ez

Fig.: ez manda a faixa horizontal 0 ≤ y ≤ π no semiplano y ≥ 0.

E do mesmo modo se pode ver que ez manda a faixa horizontal 0 < y < 2π no
plano menos o semi-eixo dos x ≥ 0, bijetoramente.
Ou seja, para qualquer w = x + Iy no plano menos o semi-eixo dos x ≥ 0 faz
sentido a operação
w = x + I · y = |z| · ((cos(b) + I sin(b)) 7→ z = ln(|w|) + I · θ
onde θ é o ângulo entre 0 e 2π formado pelo vetor (x, y) com o eixo dos x > 0.
Essa operação
w = x + I · y = |w| · ((cos(b) + I sin(b)) 7→ z = ln(|w|) + I · θ
5. O TEOREMA FUNDAMENTAL DO CÁLCULO SOBRE OS COMPLEXOS768

onde θ é o ângulo entre 0 e 2π formado pelo vetor (x, y) com o eixo dos x > 0 será
chamada de o ramo do logaritmo natural Complexo com argumento θ entre 0 e 2π.
Também poderı́amos estabelecer que o argumento ficasse entre −π e π por exemplo
e terı́amos outro ramo do logaritmo natural Complexo.
Afirmação 4.1. Considere ln(w) o ramo logaritmo natural Complexo com argumento
θ entre 0 e 2π.
Suponha que existe a derivada complexa:
ln(w) − ln(w)
ln′ (w) := lim .
w→w w−w
Então
1
ln′ (w) = .
w
Demonstração.
Para w = x + I · y temos:
p
ln(w) := ln( x2 + y 2 ) + I · θ(x, y), onde 0 < θ < 2π.
Pelo que aprendemos na prova do item ii) da Afirmação 2.3,
p
∂ ln( x2 + y 2) ∂θ(x, y)
ln′ (w) = +I · =
∂x ∂x
1 2x −y
= · 2 + I · =
2 x + y2 x2 + y 2
x y
= 2 − I · ,
x + y2 x2 + y 2
(pelo que vimos na prova do item ii) da Afirmação 7.1 do Capı́tulo 36 e que já usamos
há pouco neste Capı́tulo).
Mas:
x y w 1
2 2
−I · 2 2
= 2
= ,
x +y x +y |w| w
como querı́amos.
En passant, aproveito para checar as relações de Cauchy-Riemann para as com-
ponentes do ramo do ln(w):
p
∂ ln( x2 + y 2) x ∂θ
= 2 2
= ,
∂x x +y ∂y
(pelo que vimos na prova do item ii) da Afirmação 7.1 do Capı́tulo 36) e
p
∂θ(x, y) −y ∂ ln( x2 + y 2 )
= 2 =− .
∂x x + y2 ∂y


5. O Teorema fundamental do Cálculo sobre os Complexos


(Em elaboração)
CAPÍTULO 50. UM PORTAL PARA O CÁLCULO COMPLEXO 769

6. Exercı́cios
Exercı́cio 6.1. Verifique que:
z1 · z2 = z1 · z2 , ∀z1 , z2 ∈ C
e que:
ez = ez .
Exercı́cio 6.2.
Considere a construção geométrica a seguir, ilustrada na Figura;
Tome z com 0 < |z| < 1. Considere a reta por (0, 0) e por z, denotada rz . Levante
uma perpendicular pz a rz passando por z. Por um dos pontos one pz intersecta o
cı́rculo trace a tangente tz ao cı́rculo.
pz

tz
rz

Considere o ponto tz ∩ rz .
i) Mostre que z1 = tz ∩ rz . Dica: semelhança de triângulos.
ii) para z com |z| > 1 inverta a construção, começando por traçar uma tangente
ao cı́rculo, etc. conclua que obterá também z1 .
CAPı́TULO 51

Os Teoremas Fundamentais

1. A primitiva Complexa

771
CAPı́TULO 52

Soluções detalhadas de alguns Exercı́cios

0.1. Capı́tulo √ 2: Exercı́cio 9.6:


−1
i) f (x) = √x 3

ii) f −1 (x) = 3√x − 1


iii) f −1 (x) = q3
x+1
iv) f −1 (x) = 3 − 51 (−10 + x)
v) O enunciado não diz, mas de fato y > 0, pois x ∈ (0, 1) dá 1−x2 > 0 e portanto
x
y = 1−x 2 > 0.

Agora
x
y= ⇔ y · x2 + x − y = 0,
1 − x2
e precisamos resolver essa equação quadrática em x, para termos x = x(y).
Ora, por Báskara as soluções são:
p p
−1 + 1 − 4y (−y) −1 + 1 + 4y 2
x1 = = ,
2y 2y
p
−1 − 1 + 4y 2
x2 = .
2y
Precisamos ficar com a solução que seja positiva, pois por hipótese x ∈ (0, 1).
x
Como y = 1−x 2 > 0 e a solução positiva é:
p
−1 + 1 + 4y 2
x := x1 = .
2y
Ou seja, a candidata a função inversa é:
p
−1 + 1 + 4y 2
x= ,
2y
x
que faz sentido ∀y > 0 (mostraremos mais adiante que a imagem de y = 1−x 2 é de

fato todo R>0 ).


Preciso conferir que x( y(x) ) ≡ x, o que não está nada óbvio neste exemplo.
Vejamos: q
x 2
−1 + 1 + 4( 1−x 2)
x( y(x) ) = x =
2 ( 1−x 2)
q
(1−x2 )2 +4x2
−1 + (1−x2 )2
= x =
2 ( 1−x2 )
773
774
q
(1+x2 )2
−1 + (1−x2 )2
= x =
2 ( 1−x 2)

1+x 2
−1 + 1−x 2
x = x.
2 ( 1−x2 )

0.2. Capı́tulo 3:
Exercı́cio 6.2:
ii) Primeiro noto que:
x2 − x > 0 ⇔ x · (x − 1) > 0 ⇔
x > 0 e x − 1 > 0 ou x < 0 e x − 1 < 0.
Ou seja, se x > 1 (mais forte que x > 0) ou se x < 0 (mais forte que x < 1).
Em suma, se x ∈ (−∞, 0) ∪ (1, +∞).
iii) As raı́zes de 3x2 − 2x − 1 = 0 são: x1 = − 31 e x2 = 1. Logo
1
3x2 − 2x − 1 = (x + ) · (x − 1).
3
Portanto preciso determinar onde o produto (x + 31 ) · (x − 1) é positivo.
Ou ambos fatores nesse produto são positivos ou ambos são negativos, ou seja:
1 1
x > − e x > 1 ou x < − e x < 1.
3 3
Tomando apenas as informações mais fortes:
1
x > 1 ou x < − ,
3
1
ou seja, x ∈ (−∞, − 3 ) ∪ (1, +∞).

Exercı́cio 6.3
Solução n. 1:
O que se quer provar é que:
 + △ ≤ |  | + |△|, caso 0 ≤  + △,
ou que
−( + △) ≤ | | + |△|, caso  + △ < 0.
Caso 0 ≤  + △: obviamente que valem
 ≤ | | e △ ≤ |△|,
e somando essas duas desigualdades obtemos o desejado:
 + △ ≤ | | + |△|.
Caso +△ < 0: então pelo menos um deles é negativo, por exemplo, suponhamos
que  < 0. Por absurdo, suponha que
|| + |△| < −( + △).
CAPÍTULO 52. SOLUÇÕES DETALHADAS DE ALGUNS EXERCÍCIOS 775

Como || = −, cancelamos esses termos na desigualdade anterior e obtemos então
que:
|△| < −△.
Se 0 < △ então chegamos no absurdo:
0 < △ =: |△| < −△ < 0.
Se △ ≤ 0 então −△ =: |△| < −△ é outro absurdo.
Logo
−( + △) ≤ || + |△|, caso ( + △) < 0.

Solução n. 2: (do estudante Walter Ferreira Diniz Júnior)


A propriedade xiii) da Afirmação 3.1 do Capı́tulo 3, dá, como caso particular, que:
0 ≤ x1 ≤ x2 ⇔ 0 ≤ x21 ≤ x22 .
Ou seja que
| + △| ≤ || + |△| ⇔ ( + △)2 ≤ (|| + |△|)2 .
Mas então queremos saber se:
2 + 2 ·  · △ + △2 ≤ 2 + 2 · || · |△| + △2 ,
ou seja, se
 · △ ≤ || · |△|.
Se  e △ têm o mesmo sinal então há igualdade nessa expressão. Se  e △ têm
sinais opostos há desigualdade estrita.
0.3. Capı́tulo 4:

Exercı́cio 4.5:
Não temos informação nenhuma sobre a sequência, exceto que seus termos são
negativos. Por isso o melhor é raciocinar por absurdo.
Suponha por absurdo que limn→+∞ xn = L > 0. Considere
ǫ := L = |L − 0|,
ou seja, a distância entre L e 0. Pela definição de limn→+∞ xn , dado esse ǫ tem que
haver um nǫ ∈ N tal que:
n > nǫ ⇒ |xn − L| < ǫ.
Mas coma escolha de ǫ := L isto quer dizer:
n > nǫ ⇒ |xn − L| < L,
ou seja, ou bem
xn − L < L, se 0 ≤ xn − L,
ou bem
−(xn − L) = L − xn < L, se xn − L < 0.
No primeiro caso, 0 < L ≤ xn e no segundo caso 0 = L − L < xn .
em ambos chegamos numa contradição com a hipótese xn < 0 ∀n.
Logo L ≤ 0.
776

Por exemplo, a sequência − n1 < 0 tem L = 0.


0.4. Capı́tulo 5:
0.5. Capı́tulo 6:
Exercı́cio 9.4:
Se x 6= 0 a função é resultado da composição de duas funções contı́nuas, x1 e sin(x),
e do produto com x: logo é contı́nua em x 6= 0.
Precisamos mostrar que em x = 0 temos:
1
lim x sin( ) = 0,
x→0 x
pois esse foi o valor associado a f (0) = 0.
Ou seja, precisamos ver que se xn é qualquer sequência com limn→+∞ xn = 0
então:
1
lim xn sin( ) = 0.
n→+∞ xn
1
Mas como | sin( xn ) | ≤ 1, dado ǫ tomamos nǫ tal que:
| xn | < ǫ
e teremos:
1 1
| xn sin( ) | = | xn | · | sin( ) | <
xn xn
< ǫ · 1 = ǫ,
o que siginifica
1
lim xn sin( ) = 0.
n→+∞ xn
O Maple plota assim o gráfico de y = x sin( x1 ) perto da origem:

0,04

x
-0,1 -0,05 0 0,05 0,1
0

-0,04

-0,08

Exercı́cio 9.9
CAPÍTULO 52. SOLUÇÕES DETALHADAS DE ALGUNS EXERCÍCIOS 777

i):
q
√ x2 · (5 + x1 )
5· +xx2
lim = lim =
x→+∞ x+2 x→+∞ x · (1 + x2 )
q q
|x| · 5 + x1 5 + x1
= lim = lim =
x→+∞ x · (1 + 2 ) x→+∞ 1 + 2
x x
q
5 + limx→+∞ x1 √
= = 5,
1 + limx→+∞ x2
onde se usou a continuidade da raı́z quadrada e que x > 0.
ii):
q

2
5·x +2 x2 · (5 + x22 )
lim = lim =
x→−∞ x+2 x→−∞ x · (1 + x2 )
q q
|x| · 5 + x22 5 + x22
= lim = lim − =
x→−∞ x · (1 + 2 ) x→−∞ 1 + x2
x
q
5 + limx→−∞ x22 √
=− = − 5,
1 + limx→−∞ x2
onde se usou que x < 0.

Exercı́cio 9.10:
Fazemos aparecer quocientes:

√ √ x2 + x + x
lim ( x2 + x − x ) = lim ( x2 + x − x ) · [ √ ]=
x→+∞ x→+∞ x2 + x + x
x2 + x − x2 x
= lim √ = lim √ =
x→+∞ x2 + x + x x→+∞ x2 + x + x
x
x 1 1
= lim √ = lim q = .
x→+∞ 2
x +x+x x→+∞ x2
+ x +1 2
x x2 x2

Exercı́cio 9.12:
No Curso se mostrou que todo polinômio Real de grau ı́mpar tem alguma raı́z
Real.
Mas para esses polinômios o Teorema do Valor Intermediário mostra que há raı́z
no intervalo [−1, 0), já que
f (−1) := −1 − (ǫ1 + . . . + ǫn ) + 1 < 0,
f (0) = 1.
O problema aqui é mostrar que só há uma Raı́z Real para cada um desses
polinômios.
778

Suponhamos por absurdo que a equação


x2n+1 + ǫ1 · x2n−1 + ǫ2 · x2n−3 + . . . + ǫn−1 · x3 + ǫn · x + 1 = 0
tenha duas raı́zes x1 , x2 , com x1 < x2 . Então pelo Teorema de Rolle a derivada da
função
f (x) := x2n+1 + ǫ1 · x2n−1 + ǫ2 · x2n−3 + . . . + ǫn−1 · x3 + ǫn · x + 1
tem que se anular num ponto x ∈ (x1 , x2 ). Mas
f ′ (x) := (2n+1)·x2n +ǫ1 ·(2n−1)·x2n−2 +ǫ2 ·(2n−3)·x2n−4 +. . .+ǫn−1 ·3·x2 +ǫn = 0
não tem Raı́z Real, pois cada um de seus monômios tem grau par, os ǫi ≥ 0, para
i = 1, . . . , n − 1 e ǫn > 0.
Logo só há uma raı́z Real.
Agora dado um x ∈ [−1, 0) fixado, resolvo a seguinte equação linear em ǫ:
x3 + ǫ · x + 1 = 0
obtendo:
−1 − x3
ǫ=
x
e facilmente se vê que ǫ ≥ 0 e é zero quando x = −1.
7
A seguir ploto três gráficos, de y = x3 + 1, de y = x3 + 4
· x + 1 cuja raı́z é − 21 e
63
de y = x3 + 16 · x + 1 cuja raı́z é − 14 .

15

10

0
-2 -1 0 1 2
x
-5

-10

-15

0.6. Capı́tulo 7:
Exercı́cio 8.3:
Resolver o sistema
y − 5x − 2 = 0 e 2y − 10x − 1 = 0,
significa, geometricamente, intersectar as retas:
10x + 1 1
y = 5x + 2 e y = = 5x + .
2 2
Porém essas retas tem o mesmo coeficiente angular 5, logo são paralelas e distintas
(pois seus coeficientes lineares são distintos).
CAPÍTULO 52. SOLUÇÕES DETALHADAS DE ALGUNS EXERCÍCIOS 779

Por isso não consigo resolver o sistema.

Exercı́cio 8.6
i) Quero que o coeficiente angular a′ da reta contendo o segmento P Q seja
1
a′ = −
a
paera que haja ortogonalidade com a reta y = ax + b.
Ora então quero:
(ax + b) − B 1
a′ := =− .
x−A a
Isso produz uma equação:
(a2 + 1) x + a(b − B) − A = 0.
A solução é
A − a(b − B)
x= .
a2 + 1
Portanto
A − a(b − B) A − a(b − B)
Q=( 2
, a·( ) + b ).
a +1 a2 + 1
ii) Se temos x = A então :
A − a(b − B)
A=
a2 + 1
isso dá
a2 A + a(b − B) = 0.
Supondo por um momento a 6= 0, divido por ele e obtenho:
a A + (b − B) = 0,
ou seja, aA + b = B. Mas isso significa que P = (A, B) ∈ r.
A conclusão é que, se x = A, então
ou P = Q = (A, B) ou a = 0.
No caso a = 0 temos uma reta r horizontal e Q é a projeção vertical de P sobre essa
reta.

Exercı́cio 8.8:
y2
As coordenadas x dos pontos de intersecção da elipse x2 + b2
= 1 com a reta
y = −x + 5 são as soluções da equação quadrática em x:
(−x + 5)2
x2 + − 1 = 0,
b2
ou seja, soluções de:
(b2 + 1) · x2 − 10 · x − b2 + 25 = 0.
O discriminante dessa equação é:
∆ := 100 − 4 · (b2 + 1) · (25 − b2 ).
780

Esse discriminante se anula quando há uma raı́z dupla, ou seja há tangência. Portanto
quero:
100 − 4 · (b2 + 1) · (25 − b2 ) = 0 ⇔
⇔ 24 · b2 − b2 · b2 = 0 ⇔ b2 · (b2 − 24) = 0,
ou seja b2 = 24, já que b 6= 0

Exercı́cio 8.9:
De y = x1 obtenho x = y1 . Ou seja, quando postas no mesmo sistema de coorde-
nadas:
1
f (x) = f −1 (x) = .
x
−1
Uma função com a propriedade f = f é chamada de involução.
O gráfico da função inversa é sempre obtido da função original por reflexão na
diagonal. Como essas funções coincidem no item vi), então concluimos que a operação
de refletir o gráfico de y = x1 o faz recair emcima dele mesmo. Isso é a simetria em
relação à diagonal.

0.7. Capı́tulo 8:
Exercı́cio 5.4:
Note primeiro que a função h(x) dada por
sin(k · x)
se x 6= 0 e h(0) := 1,
k·x
é a composição h := f (g(x)) da função contı́nua
sin(x)
f (x) := , se x 6= 0 e f (0) := 1,
x
com a função contı́nua g(x) := k · x.
Logo h é contı́nua e portanto
sin(k · x)
lim = 1.
x→0 k·x
Mas então:
sin(k · x)
lim · k = k,
x→0 k·x
ou seja,
sin(k · x)
lim = k.
x→0 x
Para calcular
tan(j · x)
lim
x→0 sin(k · x)
escrevo, para x 6= 0:
tan(j · x) sin(j · x) j sin(j · x) k·x 1
:= = · · · .
sin(k · x) cos(j · x) · sin(k · x) k j·x sin(k · x) cos(j · x)
CAPÍTULO 52. SOLUÇÕES DETALHADAS DE ALGUNS EXERCÍCIOS 781

Usando o que vimos acima (bem como limite de produto e inverso e a continuidade
do cosseno) o limite
tan(j · x)
lim
x→0 sin(k · x)

vira
j sin(j · x) k·x 1 j
· lim · lim · lim = .
k x→0 j · x x→0 sin(k · x) x→0 cos(j · x) k
0.8. Capı́tulo 9:
Exercı́cio 6.6:
Fixe x 6= 0. No que segue, se x < 0 tome x < 0 e se x > 0 tome x > 0.
Traço retas secantes ao gráfico de y = x1 ligando (x, x1 ) a cada (x, x1 ), cujo coeficente
angular é:
1 x−x
x
− x1 xx
ax := = =
x−x x−x
x−x 1 −1
= · = < 0,
(x − x) x x xx
(pois x e x têm o mesmo sinal).
As secantes são portanto retas de coeficiente angular ax <. Passando ao limite
quando x → x o que dá para prever é que a reta tangente terá coefciente angular
a ≤ 0.
Vejamos que de fato a < 0.
Pela definição de coeficiente angular da reta tangente, fixado x 6= 0:
f (x + h) − f (x)
a := f ′ (x) = lim =
h→0 h
1 1 x−(x+h)
x+h
− x (x+h) x
= lim = lim =
h→0 h h→0 h
−h −1
= lim = lim =
h→0 (x + h) x h h→0 (x + h) x

−1
= 2 <0
x
−1
(na última etapa uso que a função de h dada por (x+h) x
é contı́nua ! Logo seu limite
quando h → 0 é simplesmente seu valor em h = 0).

Exercı́cio 6.8:
Noto que
f (x + h) − f (x) f (x + (−h)) − f (x)
f ′ (x) := lim = lim ,
h→0 h h→0 (−h)
por ser um limite bi-lateral.
Então:
f (x + h) − f (x) f (x + (−h)) − f (x)
2 · f ′ (x) = lim + lim =
h→0 h h→0 (−h)
782

f (x + h) − f (x) + f (x) − f (x + (−h)) f (x + h) − f (x + (−h))


= lim = lim ,
h→0 h h→0 h
de onde:
f (x + h) − f (x − h))
f ′ (x) = lim .
h→0 2h
A função descontı́nua em x = 0 dada por g(0) = 0 e g(x) = 1, se x 6= 0 tem
g(0 + h) − g(0 − h)
= 0,
2h
logo
g(0 + h) − g(0 − h)
lim = 0.
h→0 2h
0.9. Capı́tulo 10:
Exercı́cio 6.4:
Primeiro testo se (−1, −1) e (2, 3) estão em todos os gráficos de:
y = fb (x) := (4/3 − b) · x2 + b · x + (2b − 7/3), b ∈ R.
De fato:
−3
(4/3 − b) · (−1)2 + b · (−1) + (2b − 7/3) = = −1,
3
e
9
(4/3 − b) · 22 + b · 2 + (2b − 7/3) =
= 3.
3
O coeficiente angular da secante a todos os gráficos y = fb (x) ligando (−1, −1) a
(2, 3) é:
3+1 4
a= = .
2+1 3
Pelo Teorema de Lagrange devem haver pontos xb (dependendo de b, a princı́pio
...) tais que
4
xb ∈ (−1, 2) e fb′ (xb ) = .
3
Vejamos quem são os xb . Temos
fb′ (x) = 2 · (4/3 − b) · x + b,
4
e igualando a 3
criamos uma equção em x:
4
2 · (4/3 − b) · x + b = ,
3
de onde
1 43 − b 1
x= ·(4 )= ,
2 3 −b 2
ou seja ∀b: xb = 21 . Por isso quando fazemos um zoom numa faixa vertical em torno
de
1 1
( , fb ( ) )
2 2
vemos todos os gráficos parecidos com retas paralelas, de mesma inclinação 34 .
CAPÍTULO 52. SOLUÇÕES DETALHADAS DE ALGUNS EXERCÍCIOS 783

0.10. Capı́tulo 11:


Exercı́cio 10.5:

Nas Figuras a seguir não usei a mesma escala nos eixos x e y, por isso as figuras
são apenas qualitativamente corretas.

2
x
-1 -0,5 0 0,5 1
0

-2

-4

-6

-8

Figura: y = f1 (x) = x3 − x2 (verm.), f1′ (x) (verde), f1′′ (x) (amar.)

0
-1 -0,5 0 0,5 1 1,5

-2 x

-4

-6
784

Figura: y = f2 (x) = x2 − x3 (verm.), f2′ (x) (verde), f2′′ (x) (amar.)

15

10

0
-1 0 1 2 3
x

-5

-10

Figura: y = f3 (x) = −2x2 + x3 (verm.), f3′ (x) (verde), f3′′ (x) (amar.)

20

15

10

0
-1 -0,5 0 0,5 1
x
-5

Figura: y = f4 (x) = x4 − 2x2 (verm.), f4′ (x) (verde), f4′′ (x) (amar.)
CAPÍTULO 52. SOLUÇÕES DETALHADAS DE ALGUNS EXERCÍCIOS 785

80

60

40

20

0
-1 -0,5 0 0,5 1 1,5 2
x
-20

Figura: y = f5 (x) = 3x4 − 4x3 (verm.), f5′ (x) (verde), f5′′ (x) (amar.)
Esta última Figura merece um zoom perto da origem:

20

15

10

0
-0,4 -0,2 0 0,2 0,4 0,6
x
-5

Exercı́cio 10.6:
Note que
x3 + C · x2 = −( (−x)3 − C(−x)2 ).
Ou seja que o gráfico de y = x3 +C ·x2 pode ser obtido refletindo o de y = x3 −C ·x2
primeiramente no eixo x (passar de x a −x) e, depois, refletindo no eixo y (passar de
y para −y).
786

A Figura a seguir mostra em vermelho y = x3 − C · x2 , em verde o de y =


(−x)3 − C(−x)2 e em amarelo o de y = x3 + C · x2 . para C = 3.

100

50

0
-3 -2 -1 0 1 2 3
x

-50

-100

Exercı́cio 10.8
Um reta rλ por (A, B) tem equação:
y = λx − λA + B.
Note que λ 6= a pois λ = a daria paralelismo entre a reta rλ e y = ax. Pode acontecer
que λ ≤ 0. Mas se λ > 0 então λ < a, já que rλ precisa formar um triângulo no
primeiro quadrante. Ou seja,
B >a·A>λ·A
e portanto a intersecção de rλ e y = ax é o ponto do primeiro quadrante:
B − λA B − λA
( , a· )
a−λ a−λ
A intersecção de rλ com o eixo dos y > 0 é:
(B − λA, 0).
1
A área do triângulo formado pela origem e esses dois pontos é 2
· ||D|| onde

0 0 1

D= 0 B − λA 1
B−λA a · B−λA 1
a−λ a−λ

Esse determinante é imediato (desenvolvendo pela coluna de 1 s):


(B − λA)2
D=−
a−λ
CAPÍTULO 52. SOLUÇÕES DETALHADAS DE ALGUNS EXERCÍCIOS 787

ou seja a área do triângulo é


1 (B − λA)2
A(λ) = · .
2 a−λ
Então:
−1 (B − λA) · (2Aa − λ · A − B)
A′ (λ) = ·
2 (a − t)2
e pontos crı́ticos de A(λ) estão em:
B 2Aa − B
λ= e λ= .
A A
Mas a reta com λ = B A
que passa por (A, B) é y = B
A
x e não forma um triângulo com
as outras duas.
Portanto a solução deve ser λ = 2Aa−B
A
. Podemos conferir que:
(Aa − B)2
A′′ (λ) = 2 ·
(a − t)3
cujo sinal é sempre positivo.
Portanto λ = 2Aa−BA
é o ponto de mı́nimo buscado.
Nele a área do triângulo (de menor área portanto) vale:
2A · (B − Aa).

Exercı́cio 10.17:
Primeiro vou usar a intuição sugerida pela figura. A figura parece indicar que
a reta tangente a y = x3 em (1, 1) consegue passar entre os dois gráficos, apenas
tocando o gráfico verde. Como só consideramos x < 1 ela é uma boa candidata.
Ou seja, conjecturo que a reta
y = 3x − 2
tangencia o gráfico de y = x3 − 3x2 + 3x − 2 e passa entre os dois gráficos sem
intersectar o gráfico de y = x3 , desde que restrinjamos
x ∈ (−2, 1).
Como é a intersecção de y = 3x − 2 com y = x3 − 3x2 + 3x − 2 ?
Faço 3x − 2 = x3 − 3x2 + 3x − 2 e obtenho x3 − 3x2 = 0, ou seja
x2 · (x − 3) = 0.
Então a reta y = 3x−2 tangencia y = x3 −3x2 +3x−2 no ponto (0, −2) (e intersecta-a
também no ponto (3, 7), mas esse ponto não nos interessa).
E onde y = 3x − 2 intercecta y = x3 , além do ponto (1, 1) ? Faço:
x3 = 3x − 2,
ou seja, quero resolver x3 − 3x + 2 = 0. Se não vejo imediatamene as soluções, posso
pensar assim: como x = 1 é ponto de tangência, então:
x3 − 3x + 2 = (x − 1)2 · (ax + b)
−b
e o outro ponto será x = a
.
788

Ora, por divisão obtenho


x3 − 3x + 2 = (x − 1)2 · (x + 2),
portanto x = −2. Mas este ponto não pertence ao intervalo (−2, 1). Ou seja, que
y = 3x − 2 passa entre os gráficos, tocando o gráfico verde em (0, −2).

Exercı́cio 10.18:
Como o gráfico é côncavo para baixo em [0, +∞), ele fica por baixo da reta
tangente de qualquer de seus pontos.
Considero a reta tangente em (x, f (x)):
y = f ′ (x) · x + f (x) − f ′ (x) · x.
Essa reta intersecta o eixo dos x em
f ′ (x) · x − f (x) f (x)
x= ′
= x− ′ =: K,
f (x) f (x)
onde x < K pois 0 < − ff′(x)
(x)
.
Então f (x) tem que ficar negativa para x < K. Pelo T.V.I. tem que ter zero entre
x e K.
0.11. Capı́tulo 12:
0.12. Capı́tulo 13:
Exercı́cio 6.1:
Se n = 1 então claramente:
1! = 1 ≥ 20 = 1.
Supondo válida a desigualdade até n − 1 (n ≥ 2):
n! = n · (n − 1)! ≥ n · 2n−2 .
Ora,
2n−1
n · 2n−2 = n · =
2
n
= 2n−1 · ≥ 2n−1 ,
2
onde usei na última desigualdade que n ≥ 2.
0.13. Capı́tulo 14:
Suponha que sabemos:
sin(x + y) = sin(x) · cos(y) + cos(x) · sin(y),
Faço o seguinte: fixo y e olho a identidade acima apenas em x.
Derivo o lado esquerdo, pela regra da derivada da composta:
(sin(x + y))′ = cos(x + y) · 1,
e o lado direito:
(sin(x) · cos(y) + cos(x) · sin(y))′ = cos(x) · cos(y) + (− sin(x) · sin(y)) =
= cos(x) · cos(y) − sin(x) · sin(y).
CAPÍTULO 52. SOLUÇÕES DETALHADAS DE ALGUNS EXERCÍCIOS 789

Igualando o lado esquerdo e o direito:


cos(x + y) = cos(x) · cos(y) − sin(x) · sin(y).
0.14. Capı́tulo 15:
Exercı́cio 6.1:
Note que:
∂F (x, y) ∂F (x, y)
= −3 x2 e = 2 y,
∂x ∂y
logo calculados em (1, 1):
∂F (x, y) ∂F (x, y)
= −3 e = 2.
∂x ∂y
Então num pequeno entorno de (1, 1) a curva é dada pelo gráfico de y = y(x).
Mas a curva não é globalmente um gráfico y = y(x), pois para cada valor x > 0
temos dois valores de y.
Note que se um ponto da curva y 2 − x3 = 0 tem x = 0, então y 2 = 0 e portanto
y = 0, ou seja é a origem.
E note que nenhum ponto da curva y 2 − x3 = 0 tem coordenada x < 0.
0.15. Capı́tulo 16:
Exercı́cio 6.1:
iii): Usando a derivada a composta:
sin3 (x3 )′ = 3 sin2 (x3 ) · cos(x3 ) · (3x2 )
iv): Usando a regra da derivada do produto:
(sin(x) cos(x))′ = cos(x) cos(x) + cos(x)(− sin(x)) = cos2 (x) − sin2 (x).
v): Usando a regra da derivada do quociente:
x4 + x2 + 1 ′ (4x3 + 2x)(3x4 + 4x2 + 1) − (x4 + x2 + 1)(12x3 + 8x)
( ) = .
3x4 + 4x2 + 1 (3x4 + 4x2 + 1)2
vi): Usando a regra da composta:
√ 1 1 −1 x
( 1 − x2 )′ = ((1 − x2 ) 2 )′ = (1 − x2 ) 2 (−2x) = − √
2 1 − x2
xv): pela composta:
((3x + 4)100 )′ = 100 · (3x + 4)99 · 3 = 300 · (3x + 4)99 .
0.16. Capı́tulo 19. Exercı́cio 3.1:
Defina a função: √
x2 + 25 8 − x
f (x) := + ,
v2 v1
que dá o tempo gasto pelo salva-vidas para chegar no ponto B.
Ou melhor, considere:
√ v2
g(x) := v2 · f (x) = x2 + 25 + · (8 − x) =
v1

=: x2 + 25 + k · (8 − x),
790

cujo domı́nio é [0, 8].


Trata-se de minimizar f ou, equivalentemente, minimizar g.
Para isso calcule separadamente

g(0) = 5 + 8k e g(8) = 89.
Mas: √
89 − 5
g(8) > g(0) ⇔ > k,

8
89−5
e como 0.55 ≈ 8
e supusemos k ≤ 0.5 então:
g(8) > g(0).
Agora basta buscar no intervalo aberto (0, 8) pelo ponto onde
g ′ (x) = 0.
Ora,
x √
g ′ (x) = √ −k =0 ⇔x=k· x2 + 25.
x2 + 25
Daı́ obtemos, elevando ao quadrado:
x2 = k 2 · (x2 + 25),
ou seja,
x2 (1 − k 2 ) = 25 · k 2
e r
25 · k 2 5k
x(k) = 2
=√ ,
1−k 1 − k2
pois a solução negativa não nos interessa. Claramente:
5k 0
lim x(k) = lim √ = = 0.
k→0 k→0 1 − k2 1
E nesse ponto x(k) temos o valor:
r
1
g(x(k)) = 8k + 5(1 − k 2 ) · .
1 − k2
Agora r
1
g(0) − g(x(k)) = 5 + 5(k 2 − 1) ·
1 − k2
e não está tão claro se g(0) − g(x(k)) ≥ 0, para todos os k no intervalo 0 ≤ k ≤ 0.5.
Ora, r
1
5 + 5(k 2 − 1) · ≥0 ⇔
1 − k2
r
2 1
⇔ 5 ≥ 5(1 − k ) ·
1 − k2
e elevando ao quadrado quero ter:
25 · (1 − k 2 )2
25 ≥
1 − k2
CAPÍTULO 52. SOLUÇÕES DETALHADAS DE ALGUNS EXERCÍCIOS 791

que equivale a :
1 − k 2 ≥ 1 − 2k 2 + k 4 ,
ou seja,
0 ≥ k 2 · (k 2 − 1).

0.17. Capı́tulo 20:


Exercı́cio 8.2: Como (x0 , y0 ) está na elipse:
x20 y02
+ 2 = 1,
a2 b
obtenho:
x20 · b2 + y02 · a2 = a2 b2 .
Como
2 · x(t) · x′ (t) 2 · y(t) · y ′(t)
+ = 0,
a2 b2
a informação das taxas de variação −1 e 1 dá:
2 · x0 · (−1) 2 · y0 · 1
+ = 0,
a2 b2
de onde
−2 · x0 · b2 + 2 · y0 · a2
= 0,
a2 · b2
ou seja
−2 · x0 · b2 + 2 · y0 · a2 = 0.
Ao lado de
x20 · b2 + y02 · a2 = a2 b2
forma-se um sistema de duas equações lineares nas incógnitas a2 e b2 .
Multiplicando a última por 2, a primeira por x0 6= 0 e depois somando-as, obtemos:
2 · y0 · (x0 + y0 ) · a2 = 2 · a2 · b2 ,
e como a 6= 0:
b2 = y0 · (x0 + y0 ).
Depois obtenho
a2 = x0 · (x0 + y0 ),
usando de novo
−2 · x0 · b2 + 2 · y0 · a2 = 0.
Os outros itens têm respostas imediatas, pois sabemos as coordenadas dos focos
e as dos vértices em função de a e b.
792

0.18. Capı́tulo 21:


Exercı́cio 8.1:
Se escrevemos
π π π
x1 = sin( ) + sin(π),
2 2 2
π π π 2π π
x2 = sin( ) + sin( ) + sin(π),
3 3 3 3 3
π π π 2π π 3π π
x3 = sin( ) + sin( ) + sin( ) + sin(π),
4 4 4 4 4 4 4
π π π 2π π
x4 = sin( ) + sin( ) + . . . + sin(π),
5 5 5 5 5
fica mais fácil reconhecer que cada xi é uma soma de Riemann da função sin : [0, π] →
π
R, onde a partição tem norma i+1 .
Em geral:
π π π 2π π (i + 1)π
xi = sin( )+ sin( ) + ...+ sin( ).
i+1 i+1 i+1 i+1 i+1 i+1
Quando i → ∞ a norma da partição tende a zero.
Como sin(x) é uma função contı́nua, os itens i) e ii) garantem que
Z π
lim xi = sin(x) dx.
i→∞ 0
Mais adiante, pelo Segundo Teorema fundamental, veremos que:
Z π
sin(x) dx = 2.
0

Exercı́cio 8.3:
Se x < 0 então Z Z
x x
F (x) := | t | dt = −t dt =
−1 −1
−t2 −t2 −x2 1
=( )(x) − ( )(−1) = + .
2 2 2 2
Se x ≥ 0 podemos fazer:
Z x Z 0 Z x
F (x) = | t | dt = | t | dt + | t | dt =
−1 −1 0
Z x
1
= + t dt =
2 0
1 x2
= + .
2 2
Ou seja que a função F (x) obtida integrando o módulo tem uma descrição difer-
ente, dependendo se x < 0 ou x ≥ 0.
Note que pelo Primeiro Teorema Fundamental, F ′ (x) = | x |, logo não existe
′′
F (0).
Ou seja, que F (x) é menos suave em em x = 0 que f (x) = x3 + 21 .
A figura a seguir apresenta F (x) (vermelho) e f (x) = x3 + 12 (verde):
CAPÍTULO 52. SOLUÇÕES DETALHADAS DE ALGUNS EXERCÍCIOS 793

1,5

0,5

0
-1 -0,5 0 0,5 1
x

-0,5

0.19. Capı́tulo 22:


Exercı́cio 16.3:
Primeiro busco o ponto de y = f (x) = ln(x) x
onde f ′ (x) = 0. Pela derivada do
quociente:
1
′ x
x − ln(x) 1 1 − ln(x)
f (x) = 2
= ,
x x2
e f ′ (x) = 0 exatamente onde 1 − ln(x) = 0, ou seja, onde ln(x) = 1.
Sabemos então que a solução é x = exp(1).
Podemos calcular a segunda derivada f ′′ (x), para confirmarmos que f ′′ (exp(1)) <
0. Caso isso valha, a Afirmação 2.1 do Capı́tulo 10 diz que x = exp(1) é ponto de
máximo local. E portanto concluiremos que x = exp(1) é ponto de máximo global
(já que não há outro candidato).
Ora,
(1 − ln(x))′ x2 − (1 − ln(x)) 2x
f ′′ (x) = =
x4
− 1 x2 − (1 − ln(x)) 2x −3x + 2x ln(x)
= x = ,
x4 x4
e portanto f ′′ (exp(1)) = − exp(1)
e4
< 0.

Exercı́cio 8.6:
Como arcsin′ (x) = √ 1
1−x2
então:
x√ 1
F ′ (x) = [1 − x2 ]′ + ( arcsin(x))′ =
2 2
1 √ x 1 1 1 1
=[ 1 − x2 + · √ · (−2x)] + √ =
2 2 2 1−x 2 2 1 − x2
794

1√ 1 1 1 1
= 1 − x2 − x2 √ + √ =
2 2 1 − x2 2 1 − x2
1√ 1 1 − x2
1 − x2 + √ =
2 2 1 − x2

= 1 − x2 .

Exercı́cio 16.2:
ln(1+x)
O programa Maple plota y = x
completando em x = 0 o valor
ln(1 + x)
lim =1
x→0 x
De fato posso escrever:
ln(1 + x) − 0 ln(1 + x) − ln(1)
lim = lim
x→0 x x→0 x
e esse último limite é nada mais nada menos que uma derivada:
ln(1 + x) − ln(1)
ln′ (1) := lim .
x→0 x
Ora ln′ (1) = 11 = 1.

Exercı́cio 16.13:
2
A função y = f (x) = e−x tem, pela regra da composta e pelo fato que (ex )′ = ex ,
derivada
2
f ′ (x) = e−x · (−2x).
lno f ′ (x) se anula apenas em x = 0 (pois exp não se anula nunca). Já a segunda
derivada é (pela regra do produto e da composta):
2
f ′′ (x) = (e−x · (−2x))′ =
2 2
= (e−x · (−2x))(−2x) + e−x (−2) =
2
= 2e−x (2x2 − 1).
q q
logo f ′′ (x) se anula em x = + 12 e x = − 12 .
Esses dois pontos são pontos de máximo/mı́nimo da f ′ (x) e pontos de inflexão da
f.

Exercı́cio 16.14:
Os pontos (x, y) da reta tangente ao gráfico de y = ln(x) no ponto (e, 1) são os
pontos que verificam:
y−1
= ln′ (e),
x−e
pois o valor da derivada ln′ (e) é por definição o coeficiente angular da reta tangente.
Mas ln′ (e) = 1e , lno
y−1 1
=
x−e e
CAPÍTULO 52. SOLUÇÕES DETALHADAS DE ALGUNS EXERCÍCIOS 795

de onde
x
y−1 = −1
e
x
e portanto y = e , que é uma reta pela origem.
Por reflexão na diagonal se obtem o gráfico da função inversa exp(x).
E a reflexão na diagonal da reta y = xe é x = ye , ou seja, a reta y = ex. Essa é a
tangente ao gráfico de y = exp(x) em (1, e), como também se pode verificar a partir
de:
y−e
= exp′ (1) = exp(1) =: e.
x−1
Exercı́cio 16.15:
As primitivas de produto/quociente Não são o produto/quociente de primitivas.
Quando aparecem produtos é natural imaginar qu surgiram de se derivar composições
de funções.
vi): Por isso as primitivas de f (x) = 2x cos(x2 ) são
F (x) = sin(x2 ) + C.
x
vii): As primitivas de 2
cos(x2 ) são:
sin(x2 )
F (x) = + C.
4
2
viii): As primitivas de xex são
2
ex
2
e as de ex cos(ex ) são
sin(ex ) + C.
As primitivas de soma/subtração são a soma/subtração de primitivas.
x): Portanto as primitivas de f (x) = a0 xn + a1 xn−1 + . . . + an são
xn+1 xn
a0 + a1 + . . . + an x + C.
n+1 n
0.20. Capı́tulo q23: Exercı́cio q 7.1:
Temos P1 = (− C , b), P2 = ( Cb , b). A área de ∆P1 OP2 é
b

r 3
1 b b2
· (2 · )·b= 1.
2 C C2
Por outro lado a área da região abaixo da reta y = b e acima da parábola é a diferença:
r Z √b
b C
2· · b − √ C · x2 dx =
C − C b

q q
r b 3
b ( C) ( Cb )3
=2· ·b−C ·[ + ]=
C 3 3
3 3
b2 2 b2
=2· 1 − · 1 =
C2 3 C2
796
3
4 b2
= · 1.
3 C2
Exercı́cio 7.4: Os gráficos de y = 8x + 2 e de de y = x4 + 2. se intersectam em
pontos cujas coordenadas x verificam:
8x + 2 = x4 + 2 ⇔ 8x = x4 ⇔ x · (x3 − 8) = 0 ⇔ x = 0, 2.
Ou seja, nos pontos (0, 0) e (2, 18).
Para x ∈ [0, 2] vale que 8x + 2 ≥ x4 + 2, pois:
8x + 2 ≥ x4 + 2 ⇔ 8x ≥ x4 ⇔ 0 ≥ x · (x3 − 8)
e como x ≥ 0, basta ter 0 ≥ x3 − 8. Isso é verdade, já que 8 ≥ x3 sai de 2 ≥ x
elevando-se ao cubo.
A Figura a seguir dá uma idéia da pétala.

20

15

10

0 0,5 1 1,5 2
x

A área da pétala é a diferença entre a área do trapézio sob y = 8x + 2 e a área


sob o gráfico de y = x4 + 2.
É dada por: Z 2 Z 2
8x + 2 dx − x4 + 2 dx
0 0
e vale portanto pelo Segundo Teorema do Cálculo:
25 48
[4 · (2)2 + 2 · (2)] − [ − 2 · 2] =
5 5
pois Z
8x + 2 dx = 4x2 + 2x + C
e Z
x5
x4 + 2 dx = + 2x + C.
5
Exercı́cio 7.5: Note que
• o integrando é a diferença entre as funções x − x2 e a função x3 .
• x − x2 > 0 para 0 < x < 1.
• Ademais
x − x2 > x3 ,
para x pequenos, pois
x − (x2 + x3 ) > 0
CAPÍTULO 52. SOLUÇÕES DETALHADAS DE ALGUNS EXERCÍCIOS 797

para x pequenos.
• Porém certamente a partir de um certo x deve acontecer que
x − x2 < x3 ,
devido ao expoente 3.
Para qual x ≥ 0 temos x − x2 = x3 ? Ou seja, onde x3 + x2 − x = 0 ? Nas soluções
de:
x (x2 + x − 1) = 0,
ou seja, em x = 0 ou na solução positiva de (x2 + x − 1), que é

−1 + 5
a := ∼ 0.6.
2
A partir desse a ∼ 0.6 vale x − x2 < x3 .
Então escrevo:
Z b Z a Z b
2 3 2 3
x − x − x dx = x − x − x dx + x − x2 − x3 dx
0 0 a
e portanto:
Z b
x − x2 − x3 dx = 0 ⇔
0
Z a Z b
2 3
⇔ x − x − x dx = − x − x2 − x3 dx.
0 a
Mas Z Z
b b
2 3
− x − x − x dx = −(x − x2 − x3 ) dx =
a a
Z b
= x3 − (x − x2 ) dx.
a
Em suma,
Z a Z b
2 3
x − x − x dx = x3 − (x − x2 ) dx.
0 a
Ora, Z a
(x − x2 ) − x3 dx
0

é uma Área, pois (x − x2 ) − x3 ≥ 0 na região x ∈ [0, a]. E também


Z b
x3 − (x − x2 ) dx
a

é uma Área, pois agora x3 − (x − x2 ) ≥ 0 se x ≥ a.


Na Figura a seguir os gráficos de y = x − x2 > 0 (vermelho) e de y = x3 (verde)
formam um peixe (x ∈ 0, b].
Ra Rb
O peixe tem a área do corpo ( 0 (x − x2 ) − x3 dx) igual a área do rabo a x3 − (x −
x2 ) dx (b ∼ 0.9).
798

0,7

0,6

0,5

0,4

0,3

0,2

0,1

0
0 0,2 0,4 0,6 0,8
x

Exercı́cio 7.8:
Para saber de onde até onde considerar a Área precisamos saber as abscissas dos
pontos onde os gráficos de y = x4 e de y = a se intersectam.
1 1
Ou seja, resolver x4 = a, o que dá x = −a 4 e x = a 4 .
1 1 5
Vamos subtrair da área do retângulo de base 2a 4 e altura a (que é 2a 4 a = 2a 4 )
a área sob o gráfico de x4 .
Esta última é dada pelo importante Teorema Fundamental do Cálculo. Na notação
do Curso:1 5
1 x5 1 x5 1 a4
Ax4 , −a 4 ( a ) = (a ) − (−a ) = 2
1 4 4 4
5 5 5
lno a área que buscamos é
5
5 a4 4 5
2a 4 − 2 = 2( a 4 ).
5 5
Como exigimos que seja
5 4 5
= 2( a 4 )
2 5
concluimos que
5 25
a4 =
16
4
25 5
e portanto a = ( 16 ) .
0.21. Capı́tulo 24:
Exercı́cio 1.4:
Faço integração por partes na terceira linha:
Z π Z π
2n−1
sin (θ) dθ = sin2n+1 (θ) · sin−2 (θ) dθ =
0 0
1
1Na
R a4 x5 x5
notação usual de integrais −a 4
1 x4 dx = 1
5 |a 4 − 1
5 |−a 4
CAPÍTULO 52. SOLUÇÕES DETALHADAS DE ALGUNS EXERCÍCIOS 799
Z π
= sin2n+1 (θ) · csc2 (x) =
0 Z π
2n+1 2n+1
= − sin (π) cot(π) + sin (0) cot(0) − (2n + 1) sin2n (θ) cos(θ)(− cot(θ)) dθ =
Z π 0 Z π
2n−1 2
= (2n + 1) sin (θ) · cos (θ) dθ = (2n + 1) sin2n−1 (θ) · (1 − sin2 (θ)) dθ =
0 Z π 0 Z
π
2n−1
= (2n + 1) sin (θ) dθ − (2n + 1) sin2n+1 (θ) dθ,
0 0
de onde sai a afirmação.
0.22. Capı́tulo 25: Exercı́cio 12.4:
Basta usar a substituição x = cos(θ).
0.23. Capı́tulo 26:
0.24. Capı́tulo 27:
0.25. Capı́tulo 28:
0.26. Capı́tulo 30:
0.27. Capı́tulo 31:
0.28. Capı́tulo 32:
0.29. Capı́tulo 35:
Exercı́cio 14.1: O aspecto qualitativo do gráfico:

35

30

25

20

15

10
0 1 2 3 4
x

que faz com que não seja desintegração de nenhuma substância radioativa é a ex-
istência de um ponto de inflexão próximo de x = 3.
Como a desintegração segue a lei
f (x) = f (0) · e−kx ,
onde k > 0 depende de cada substância, então:
f ′ (x) = −k · f (0) · e−kx < 0, ∀x
e
f ′′ (x) = k 2 · f (0) · e−kx > 0, ∀x,
isso impede a existência de inflexões, já que f ′′ (x) > 0 não muda de sinal.

Exercı́cio 14.4:
800

A solução da equação f ′ (x) = −kf (x) é


f (x) = f (0) · e−kx , ∀x.
f (0)
Portanto f (τ ) := 2
é também:
f (τ ) = f (0)e−kτ .
Logo dividindo por f (0):
1
= e−kτ .
2
Aplicando ln em ambos lados:
1
ln( ) = ln(e−kτ ) = −kτ,
2
e portanto:
ln( 12 ) − ln(2) ln(2)
τ= = = .
−k −k k
Por definição de τ̂ temos: f (τ̂ ) := f (0)
4
é também:
f (τ̂ ) = f (0) · e−kτ̂ .
lno dividindo por f (0):
1
= e−kτ̂ .
4
Aplicando ln em ambos lados:
1
ln( ) = ln(e−kτ̂ ) = −kτ̂ ,
4
e portanto:
ln( 212 ) − ln(22 ) 2 ln(2)
τ̂ = = = .
−k −k k
Ou seja, τ̂ = 2τ .
f (0)
Para a τ̌ temos por definição f (τ̌ ) := √
2
é também
f (τ̌ ) = f (0)e−kτ̌ .
lno dividindo por f (0):
1
√ = e−kτ̌ .
2
Aplicando ln em ambos lados:
1
ln( √ ) = ln(e−kτ̌ ) = −kτ̌ ,
2
e portanto
1
ln( 1 ) − ln(2 2 )
1
1 ln(2)
22
τ̌ = = = .
−k −k 2 k
Ou seja, τ̌ = 21 τ .

Exercı́cio 14.6:
Sabemos que a solução da equação, com f (0) = 1 é f (x) = e−kx .
CAPÍTULO 52. SOLUÇÕES DETALHADAS DE ALGUNS EXERCÍCIOS 801

Queremos x tal que f ′ (x) = −1, onde


f ′ (x) = −k e−kx .
Logo queremos encontrar x tal que:
−1 = −k e−kx ,
1
ou seja, k
= e−kx , ou seja, ln( k1 ) = −kx, de onde
ln(k)
x= .
k
Resolvi fazer um exemplo, com k = 2 e portanto x = ln(2)2
.
−2x
Pedi para o Maple plotar os gráficos de y = f (x) = e e de y = −x para
ln(2) ln(2)
x∈[ − 0.1, + 0.1]
2 2
e o resultado aparece a seguir:

0,6

0,4

0,2

0
0,28
0,32
0,36
0,4
0,44
x
-0,2

-0,4

Exercı́cio 14.10:
Como é uma equação linear, a solução geral é:
R 1
Z R −1
dx dx
y(x) = e 1+x · [C + (−x) · e 1+x dx].

Como 1 + x ≥ 1:
Z Z
x 1+x−1
y(x) = (1 + x) · [C − dx] = (1 + x) · [C − dx] =
1+x 1+x
Z
1
= (1 + x) · [C − (1 − ) dx] = (1 + x) · [C − x + ln(1 + x)].
1+x
E y(0) = 1 · [C − 0 + 0] = C.
Para ver que limx→+∞ y(x) = −∞, basta ver que
lim (−x + ln(1 + x)) = −∞.
x→+∞

Para isso basta ver que


lim e−x+ln(1+x) = 0
x→+∞
1+x
o que vale pois e−x+ln(1+x) = ex
.
802

0.30. Capı́tulo 36.


Exercı́cio 16.1:
Quero um fator integrante µ(x) para a equação:
((n + 1)xn−1 y n + n2 xn y n−1) · y ′(x) + nxn−2 y n+1 + n(n + 1)xn−1 y n = 0.
Ou seja, quero que valha
µ′ (x) · [(n + 1)xn−1 y n + n2 xn y n−1] + µ(x) · [(n + 1)(n − 1)xn−2 y n + n3 xn−1 y n−1 ] =
= µ(x) · [n(n + 1)xn−2 y n + n2 (n + 1)xn−1 y n−1],
ou seja:
µ′ (x) (n + 1)xn−2 y n + n2 xn−1 y n−1 1
= n−1 n 2 n n−1
=
µ(x) (n + 1)x y + n x y x
e portanto µ(x) = x serve.
A equação obtida multiplicando por x:
((n + 1)xn y n + n2 xn+1 y n−1 ) · y ′ (x) + nxn−1 y n+1 + n(n + 1)xn y n = 0
agora é exata e a solução geral é:
Z x
U(x, y) := [ntn−1 cn+1 + n(n + 1)tn cn ] dt+
a
Z y
+ [(n + 1)xn tn + n2 xn+1 tn−1 ] dt =
c
= xn cn+1 + nx n+1 n
c − C1 + xn y n+1 + nxn+1 y n − xn cn+1 + nxn+1 cn =
= xn y n+1 + nxn+1 y n − C1 ,
ou seja
xn y n+1 + nxn+1 y n = C1
são as curvas solução.
0.31. Capı́tulo 37:
Exercı́cio 4.1:
3
A equação da reta tangente de y = a · x 4 − x por
3
(x, y) = (x, a · x 4 − x)
é:
3a − 1 3 3a 1
y=( · x 4 − 1) · x + a · x 4 − x − ( · x− 4 − 1) · x.
4 4
Um conta imediata mostra que essa reta passa por (− x3 , x3 ).
3
A função y = f (x) = a · x 4 − x corta o eixo dos x em x = 0 e em x = a4 . A partir
deste ponto f (x) < 0.
1
Enquanto que f ′ (x) = 3a 4
· x− 4 − 1, que só está definida para x > 0, se anula
em x = ( 43 )4 ; ademais f ′ (x) > 0 no intervalo (0, ( 43 )4 ) e f ′′ (x) > 0 no intervalo
(( 34 )4 ), +∞).
Ou seja, que em (0, ( 43 )4 ) a função cresce, tem em x = ( 34 )4 um máximo absoluto,
e depois sempre decresce.
CAPÍTULO 52. SOLUÇÕES DETALHADAS DE ALGUNS EXERCÍCIOS 803

Temos
3 a
lim a · x 4 − x = lim x · ( 1 − 1) = +∞ · (−1) = −∞,
x→+∞ x→+∞ x4
enquanto que
3a − 1
lim f ′ (x) = lim · x 4 − 1 = −1,
x→+∞ x→+∞ 4
ou seja que há uma assı́ntota oblı́qua de inclinação −1 para y = f (x).
5
Também f ′′ (x) = − 3a 16
x− 4 < 0 ∀x, ou seja que a função sempre é côncava para
baixo.
A área da região é:
Z a4
3 4a 4 x2 a8
a · x 4 − x = ( x 7 − )(a4 ) = .
0 7 2 14
A figura aseguir dá três exemplos, em vermelho, verde e amarelo, com a =
1, 1.3, 1.5 e onde
x x 1 1
(− , ) = (− , ).
3 3 3 3

0,6
0,4
0,2
0
-1 -0,20 1 2 3
-0,4 x
-0,6

0.32. Capı́tulo 38:


0.33. Capı́tulo 39:
0.34. Capı́tulo 40. Exercı́cio 17.1:
Note que
+∞
X +∞
X
n ′′
x·( an x ) − ( an xn ) = 0
n=0 n=0
pode ser re-escrito como
+∞
X +∞
X
n
n · an · x − an · xn = 0
n=0 n=0
804

ou seja,
(n − 1) · an = 0, ∀n ≥ 0.
Se n 6= 1, então an = 0. Se n = 1, então sobre a1 não há nenhuma condição.
Logo as soluções são y = a1 · x, que são retas pela origem.
A não-unicidade da solução segue do fato que se colocamos a equação em forma
padrão:
y
y ′ = =: P (x, y)
x
vemos que P (x, y) é descontı́nuo em x = 0.

Exercı́cio
P 17.2:
Se y = +∞ π n
n=0 an (x − 2 ) então

y ′′ + y = 0
dá
+∞ +∞
X π n−2 X π
n(n − 1)an (x − ) + an (x − )n = 0
n=2
2 n=0
2
e após pôr o ı́ndice k = n − 2 na primeira série e mantendo k = n na segunda:
+∞ +∞
X π X π
(k + 2)(k + 1)ak+2(x − )k + ak (x − )k = 0,
k=0
2 k=0
2
ou seja,
(k + 2)(k + 1)ak+2 + ak = 0, ∀k ≥ 0
e daı́ a recorrência:
ak
ak+2 = − .
(k + 2)(k + 1)
As condições iniciais y( π2 ) = 1 e y ′ ( π2 ) = 0 dão a0 = 1 e a1 = 0.
A recorrência em seguida dá:
a0 (−1)k
a2k = (−1)k ·
= , ∀k ≥ 0.
(2k)! (2k)!
Logo, chamando k de n novamente, temos como solução do problema:
+∞
X (−1)n π 2n
y= (x − ) .
n=0
(2n)! 2
Mas reconhecemos aı́ a série do cosseno aplicado em x − π2 .
Logo y = cos(x − π2 ) = sin(x).

Exercı́cio 17.3:
De i):
Basta calcular
v′x − v v′ v
y ′ (x) =2
= − 2,
x x x
′′ ′ ′ 2
v x−v v x − 2xv v ′′ v′ 2v
y ′′ (x) = 2
− 4
= − 2 2
+ 3
x x x x x
CAPÍTULO 52. SOLUÇÕES DETALHADAS DE ALGUNS EXERCÍCIOS 805

e portanto:
2 ′ q v ′′ v′ 2v 2 v ′ v q v
0 = y ′′(x) + y (x) + α y(x) = − 2 2 + 3 + · ( − 2,) + α =
x x x x x x x x x x
′′
v q v
= + α ,
x x x
mas então
q
v ′′ + α v = 0.
x
De ii):
Como agora
v ′′ + qv = 0, q<0
então √ √
−qx
v = c1 e + c2 e− −qx

portanto √ √
−qx
e e− −qx
y = c1 + c2 .
x x

Você também pode gostar